1200 by Subjects-1
1200 by Subjects-1
1200 by Subjects-1
BY SUBJECTS
pg. 1
pg. 2
Cardiovascular
Emergency
➢ Chapter includes:
Ischemic heart disease
Aortic dissection
Abdominal aortic dissection
CHF
Hypertension and medication
DVT (PE can be found in pulmonary emergency)
Valvular heart disease
Cardio-infectious
Multiple cardio-vascular related MCQ’s
pg. 3
1. A 57-year-old female with a history of hypertension presents with headache, mild confusion,
and vomiting. She describes an acutely worsening global headache over the last several hours
with nausea and vomiting. A family member states she is confused. She admits to
noncompliance with her blood pressure medications for the last week. Vital signs are: T 98.4, HR
92, BP 220/130, RR 20, SpO2 97%. Examination reveals a patient in moderate discomfort,
papilledema, hypertensive retinopathy, and a non-focal neurologic examination. Laboratory
studies, EKG, and noncontract CT brain are all normal. Which of the following is the most
appropriate next step in management?
A. Neurosurgical consultation
C. Lumbar puncture
D. Corticosteroids
Answer B. The patient has hypertensive encephalopathy. The history of acute headache with
vomiting and confusion in the setting of severely elevated blood pressure is characteristic.
Neuroimaging with CT scan is often normal. The treatment is immediate reduction of blood
pressure by as much as 30%. By definition, hypertensive encephalopathy is reversible when
blood pressure is reduced. Neurosurgical consultation may be indicated later in the course, but
blood pressure management should be instituted early. Lumbar puncture would be
contraindicated in this circumstance due to the papilledema indicating increased intracranial
pressure. Corticosteroids are indicated in cases of temporal arteritis, which is on the differential
diagnosis but is far less common than hypertensive encephalopathy and requires the presence
of an elevated erythrocyte sedimentation rate. MRI of the brain can add important structural
information but is not indicated emergently.
2. A 61-year-old male presents to the ED with a chief complaint of chest pain. His EKG is shown
below. Which of the following is the most likely explanation?
C. Acute ischemia
pg. 4
Answer B. The patient’s EKG appears to demonstrate an ectopic atrial rhythm with inverted P
waves in lead II. However, the QRS axis is also flipped 180 degrees from normal. These findings
indicate limb lead reversal. Specifically, the left leg and the right arm have been reversed. As a
result, lead II becomes inverted, leads I and III become inverted and switch places, while leads
aVR and aVF simply switch places. The only limb lead that is unaffected is aVL, which remains
unchanged.
3. Which of the following parenteral agents is the initial preferred agent for blood pressure
management in patients with acute aortic dissection?
A. Hydralazine
B. Enalapril
C. Labetalol
D. Diltiazem
E. Nicardipine
Answer C. Initial treatment should focus on reducing the patient’s blood pressure to a systolic
blood pressure of 100 to 120 mm Hg or lower if the patient can tolerate it using parenteral
beta-blockers (labetalol, esmolol, propranolol). Beta-blockers are the preferred agent because
they also reduce heart rate and aortic wall stress by reducing the rate of systolic blood pressure
rise. The nonselective calcium channel blockers (CCBs) (verapamil and diltiazem) are reasonable
alternatives in patients who are unable to tolerate beta-blockers (e.g., due to bronchospasm). If
pg. 5
the patient’s blood pressure remains elevated above this range despite maximal beta-blocker
therapy, or if the patient’s blood pressure is at goal, with normal mentation, urine output, and
renal function, then the blood pressure can be further reduced using nitroprusside.
Nitroprusside is not considered a first-line agent because its use triggers reactive tachycardia
and increased contractility. Thus, it is important to use it only in patients who have first achieved
effective beta-blockade. As a direct arterial vasodilator, hydralazine should be avoided because
its use results in reflex tachycardia and it has an unpredictable blood pressure response curve.
Otherwise, each of the remaining agents may be used as a second-line agent after
beta-blockade is initiated.
A. The two pads for cardioversion should both be placed on the front of the chest
pg. 6
Answer C. This patient presents with atrial fibrillation with a rapid ventricular response (AFib
with RVR). Traditionally, electrical cardioversion has been a treatment option to restore normal
sinus rhythm as long as the patient has been symptomatic for <48 hours. However, there are
increasing data suggesting that the risk of a cardiac thrombus rises as early as 12 hours after
onset. A transesophageal echocardiogram (TEE) should be performed to exclude a thrombus.
Furthermore, evidence suggests that patients older than 65 have better outcomes when
treatment focuses on rate control rather than rhythm control. Diltiazem or beta-blockers such
as metoprolol are the treatment agents of choice. While digoxin may be useful, it has a
narrower therapeutic window and more side effects of therapy. If electrical cardioversion is
performed, it is unclear whether anterior–posterior (front and back) or anterior–lateral (right
upper front and left lower front) pad placement is best. Anticoagulation is often necessary for
patients who convert to normal sinus rhythm. The CHA2DS2– VASc is the current scoring system
used to determine annual stroke risk and the need for anticoagulation. The scoring system is
heavily tied to age, and females also earn an extra point for their gender. Given this patient’s
history of hypertension and diabetes, she will easily be considered high-risk and anticoagulation
will be recommended.
5. A 29-year-old male is sent by his primary care doctor’s office for evaluation of an abnormal
EKG. He is asymptomatic and his physical examination is normal. The EKG is shown in below.
Which of the following is the most appropriate next step in management?
A. No acute therapy
B. Atropine 1 mg IV
C. Amiodarone 150 mg IV
D. Transcutaneous pacing
E. Synchronized cardioversion at 50 J
pg. 7
Answer A. The patient has asymptomatic first-degree atrioventricular (AV) block with a
prolonged PR interval but no dropped beats. This is commonly seen in healthy individuals, has
no prognostic significance, and requires no further evaluation or management.
6. When compared with patients with deep venous thrombosis (DVT) of the lower extremities,
patients with an upper extremity DVT
A. Tend to be older
Answer B. Patients with an upper extremity DVT tend to be younger and leaner than their lower
extremity DVT counterparts. They less commonly have an inherited or acquired hypercoagulable
state but more commonly have an underlying diagnosis of cancer. In fact, most upper extremity
DVTs are associated with an indwelling catheter (often used for chemotherapy or long-term
antibiotics). Though patients with an upper extremity DVT experience complications less often
than patients with lower extremity DVTs, approximately 6% of patients still develop a PE (versus
as many as one-third of patients with a lower extremity DVT), so patients require long-term
anticoagulation, typically for 3 to 6 months after diagnosis.
7. Which of the following is true regarding troponin I in acute myocardial infarction (MI)?
Answer E. Troponin I is highly specific for cardiac muscle. It starts rising between 3 and 6 hours
after infarction, peaks at 12 to 24 hours, and returns to normal within 7 days. The specificity of
troponin I is excellent at 6 hours post infarction, but the sensitivity does not approach 100%
until 12 hours.
pg. 8
8. A 67-year-old female with a history of hypertension and hyperlipidemia is brought in to the
ED after a syncopal episode. The patient reports eating lunch with a friend when she began to
feel dizzy. She can’t recall anything else. No seizure activity or chest pain was noted. Her EKG is
shown below . Which of the following is true?
A. Since the patient’s rhythm is normal, there is no increased risk of a serious cardiac outcome
B. The patient has a right bundle branch block, which is not associated with an increased risk of
a serious cardiac outcome
D. T inversions in leads I and aVL combined with <1 mm ST depression in V6 are concerning for
ischemia
E. The patient is likely symptomatic from her low heart rate as a result of beta blocker therapy
Answer C. The patient’s EKG reveals a left bundle branch block (LBBB) which is associated with a
greater than three times risk of a serious cardiac cause. Any left bundle conduction abnormality,
including an anterior or posterior hemiblock is associated with a serious cardiac outcome. Also
predictive of serious cardiac outcomes though not quite as strongly is any nonsinus rhythm
either on EKG or on cardiac monitoring. Right bundle blocks, Q waves, and nonspecific ST
segment changes are not predictive of more serious cardiac outcomes.
9. Which of the following is a possible complication of left ventricular assist device (LVAD)
placement and use?
pg. 9
A. Gastrointestinal bleeding
C. Pump thrombosis
D. Driveline infection
Answer E. Left ventricular assist devices (LVADs) are associated with a host of complications. All
patients require anticoagulation to reduce the risk of pump thrombosis. While the use of
anticoagulants increases bleeding risk, the risk is further increased because patients with LVADs
develop an acquired von Willebrand syndrome because the pump machinery breaks the
normally long vWF chains into shorter, less effective segments. The driveline connects the
extracorporeal electronic controller to the motor which is attached to the heart. As with any
foreign body, drivelines are susceptible to infection and are covered with a sterile dressing.
Right heart failure is a frequent complication either secondary to the increased right heart
workload or to a malignant rhythm such as ventricular tachycardia or fibrillation (which has less
effect on the left heart because it is device dependent).
10. A 7-year-old female presents with syncope without a prodrome. EKG shows QT
prolongation. Family history is most likely to reveal which of the following?
A. Cystic fibrosis
B. Hirschsprung disease
D. Deafness
E. Short stature
11. Which of the following is true regarding EKGs in posterior wall MI?
pg. 10
A. Abnormalities are most commonly seen in V3.
Answer E. Posterior wall MIs occur with infarction of the right coronary artery or left circumflex
artery, depending on the anatomic dominance of the patient. Standard EKG electrodes are
anteriorly placed and may not exhibit infarction patterns of ST elevation—posterior electrodes
are more accurate at diagnosing posterior wall MIs. Infarction patterns in posterior wall MIs are
opposite of those in anterior wall MIs—posterior MIs will show ST depression instead of ST
elevation, R waves instead of Q waves, and upright T waves instead of T-wave inversions. Leads
V1 and V2 are the most specific anterior leads for posterior wall infarction. Tall R waves, like Q
waves, are the latest findings in posterior MIs.
Answer B. Applying a magnet to the pulse generator causes the pacemaker to default to
asynchronous pacing function by turning off (or bypassing) the sense amplifier. In other words, a
magnet “turns the sensing function off.” The pacemaker will deliver impulses (pacer spikes) at
its predefined default rate (sometimes called the “magnet rate”) regardless of the heart’s
intrinsic electrical activity. The result is an irregular heart rate, since the pacemaker ignores the
patient’s intrinsic rhythm and is therefore not synchronized to the patient’s intrinsic rhythm.
13. In patients receiving a normal 12-lead EKG, which of the following is true regarding
reciprocal ST depressions seen in acute MI?
B. They are more commonly seen in inferior MIs than anterior MIs.
pg. 11
C. They have poor specificity for acute MI.
D. They are more commonly seen in acute pericarditis than acute MI.
E. They are usually upsloping when seen in the setting of acute MI.
Answer B. Reciprocal ST depressions associated with ST elevations have high specificity for acute
MI (reported to be >90%) and are not usually present in nonischemic conditions. Their presence
confers a higher risk of poor outcomes. Reciprocal ST depressions are usually downsloping. Most
inferior wall MIs exhibit reciprocal ST depressions, whereas most anterior wall MIs do not.
14. A 55-year-old female presents with a progressively worsening, painful, swollen, and
discolored left leg shown below. Which of the following is the most likely pathophysiologic
mechanism?
A. Thrombosis
B. Embolus
C. Infection
D. Autoimmune
E. Allergy
Answer A. The patient has phlegmasia cerulea dolens, which is diagnosed by the clinical triad of
cyanosis, pain, and edema due to massive lower extremity venous thrombosis. The cyanosis is
due to a combination of increased venous backflow and compromised arterial flow. Phlegmasia
alba dolens is similar but lacks cyanosis and instead appears as a blanched extremity. Treatment
pg. 12
involves admission, intravenous anticoagulation, and possibly surgical thrombectomy and
fasciotomy.
15. In patients older than 85 years, which of the following is the most common symptom during
MI?
A. Chest pain
B. Dyspnea
C. Syncope
E. Fever
Answer B. Elderly, diabetic, and female patients frequently have atypical anginal symptoms. The
extreme elderly (>85 years) experience shortness of breath as the most common symptom
during an MI. Nausea, typical chest pain, syncope, and fatigue may also be present. The
emergency physician (EP) can never clinically rule out the diagnosis of acute coronary syndrome
in elderly patients just because they lack frank chest pain. Choices D and E occur more
frequently in dehydration and infection than in MI.
16. A 62-year-old female with a history of hypertension, diabetes mellitus, and emphysema
presents with a 3-week history of dyspnea on exertion, three-pillow orthopnea, and bilateral
lower extremity edema. She denies fevers, cough, or chest pain. A chest x-ray is shown in Figure
below. The patient’s electrocardiogram (EKG) does not shows any ischemic changes. Her vitals
are: Respiratory rate 22 per minute, pulse 108, BP 154/88, pulse oximetry 88% on room air.
What is the next best step in management?
E. Order a B-type natriuretic (BNP) level in order to determine the best management.
pg. 13
Answer C. This patient has an exacerbation of congestive heart failure (CHF). Although she has
no prior diagnosis of CHF, her clinical history is consistent with the diagnosis. Pleural effusions
are common in CHF and thoracentesis generally has no role in their management because they
typically resolve with diuretics. This patient also lacks infectious symptoms, which would suggest
the presence of pneumonia or a parapneumonic effusion. In patients with severe respiratory
compromise as a result of large pleural effusions in the setting of CHF, a secure airway through
rapid sequence intubation would be the first step. Although this patient is hypoxic, she has not
yet been given supplemental oxygen, so it is premature to intubate this patient at this point.
Furthermore, noninvasive ventilation might be attempted before establishing a secure airway in
pg. 14
patients with only mild to moderate respiratory distress. BNP levels may be helpful in
determining the etiology of respiratory distress in patients who have mixed cardiopulmonary
disease without a clear clinical picture. This patient’s clinical picture clearly points toward a
diagnosis of CHF, so a BNP level would not provide any additional information.
17. Which of the following is usually the earliest symptom to occur in patients with aortic
stenosis (AS)?
B. Angina
C. Syncope
D. Orthopnea
18. Among patients presenting with an NSTEMI, which of the following describes the
appropriate use of antiplatelet therapy?
A. Aspirin alone, given in doses of 162 to 325 mg, is the standard of care.
Answer C. Dual antiplatelet therapy is the recommended approach in patients with NSTEMI.
Upon arrival to the ED, all patients should be given between 162 and 325 mg of uncoated aspirin
to be chewed immediately. Uncoated aspirin is preferred over enteric coated aspirin because of
its more rapid absorption. At these doses, aspirin nearly immediately, completely, and
permanently (until a new platelet is made) inhibits thromboxane A2 production which is critical
pg. 15
for platelet aggregation. In addition to aspirin, patients should receive either clopidogrel (a
thienopyridine, along with ticlopidine and prasugrel) or a GPIIbIIIa inhibitor, such as abciximab,
eptifibatide, or tirofiban. Most guidelines recommend clopidogrel as the first-line agent in
patients being managed without PCI while clopidogrel or a GPIIbIIIa inhibitor can be used in
patients subjected to an invasive strategy. While some studies have suggested that higher
loading doses should be used, the recommended loading dose of clopidogrel in the setting of
NSTEMI remains 300 mg unless immediate catheterization is going to be performed (in which
case the recommended dose is 600 mg).
19. A 56-year-old male with a history of a prosthetic aortic valve presents for evaluation of fever
and chills for 1 week. Echocardiogram reveals vegetations on the prosthetic valve. Which of the
following is the most likely etiologic organism?
A. Coagulase-negative Staphylococcus
B. Streptococcus viridans
C. Haemophilus influenzae
D. Pseudomonas aeruginosa
E. Klebsiella pneumoniae
Answer A. The patient has endocarditis of a prosthetic valve, which is most commonly due to
coagulase-negative staphylococci. S. viridans is the most common cause of native valve
endocarditis. Choices C, D, and E are uncommon causes of native valve endocarditis.
20. A 66-year-old female who recently completed chemotherapy for colon cancer through a
right chest Port-a-Cath presents with a chief complaint of right arm pain, swelling, and
weakness. The right hand is warm, and well perfused. The radial pulse is equal to the
contralateral side. There is no evidence of cellulitis. An upper extremity deep venous thrombosis
(DVT) is suspected. Which of the following is the next best test?
C. D-dimer
E. CT angiography
pg. 16
Answer D. Doppler ultrasonography is the standard test for evaluating the presence of both
lower and upper extremity DVT. While MRA has excellent specificity, it is only 80% sensitive for
the diagnosis of upper extremity DVT. CT angiography has not been evaluated extensively, and
its test characteristics are not yet known. Direct venography is a highly accurate method of
diagnosing upper extremity DVT, but it is invasive and time consuming, so it should only be used
in cases in which ultrasound is nondiagnostic though suspicion remains high. Finally, while
negative D-dimers can be used in low-risk patients to exclude the presence of a lower extremity
DVT, no validated criteria exist for proper risk stratification among patients with a possible
upper extremity DVT. Therefore, the D-dimer cannot be used to exclude upper extremity DVT.
21. A significantly elevated lactate dehydrogenase level (LDH) in a patient with a left ventricular
assistance device (LVAD) likely indicates which of the following?
A. Fluid overload
D. Opportunistic infection
Answer E. Pump thrombosis is one of the most common, serious complications of left
ventricular assist devices (LVADs). All patients with LVADs are anticoagulated to reduce this risk.
Thrombosis within the pump leads to hemolysis which releases lactate dehydrogenase (LDH), a
common intracellular enzyme. LDH levels >1,000 are strongly suggestive of pump thrombosis. In
addition, patients with pump thrombosis may have a seemingly paradoxically increased palpable
pulse on examination as blood is no longer routed through the pump, as well as symptoms and
findings of worsening heart failure. Right ventricular heart failure is a possible complication of
LVAD most often because of the increased workload placed on the right ventricle as the LVAD
increases overall cardiac output. However, LVAD patients are also susceptible to malignant
dysrhythmias like ventricular tachycardia and ventricular fibrillation as well as pulmonary
embolism which can also present with right ventricular failure. Since circulation in patients with
LVADs is dependent on a mechanical device, these patients may not be aware when they
experience a malignant rhythm such as ventricular fibrillation or ventricular tachycardia.
22. Which of the following is the most specific EKG finding in acute pericarditis?
A. Concave ST elevations
B. Convex ST elevations
C. PR depressions
pg. 17
D. Hyperacute T waves
Answer C. The EKG changes over time in patients with pericarditis. Acutely, concave ST elevation
and PR depression with tall T waves are seen. Depression of the PR segment is the most specific
EKG finding for acute pericarditis. Concave ST elevation is also common, but can be seen in a
variety of other conditions, including benign early repolarization and left ventricular
hypertrophy. An STsegment to T-wave ratio of >0.25 argues in favor of acute pericarditis.
Convex ST elevations are more likely to be due to myocardial infarction than pericarditis.
Hyperacute T waves are seen more often in hyperkalemia and infarction than in pericarditis. AV
blocks are rarely seen in acute pericarditis. Chronic EKG changes associated with pericarditis
include return of ST segments to baseline, T-wave flattening, T-wave inversion, and then
complete normalization after a few weeks to months.
23. A patient presents with findings concerning for endocarditis. Which of the following findings
is most sensitive in endocarditis?
C. Heart murmur
D. Splenomegaly
E. Petechiae
Answer C. Fever is the most common symptom in infective endocarditis but is nonspecific.
Audible heart murmur or fever is present in almost 90% of patients with endocarditis. Murmurs
are less common and fever is more common in IV drug users. Choices A, B, D, and E occur in less
than half the number of patients with endocarditis. Some type of vasculitic skin lesion will occur
in most patients, but a specific type such as Osler nodes, Janeway lesions, petechiae, or splinter
hemorrhages each occur in less than one-fourth of patients. Splenomegaly is present in roughly
one-third.
24. A 59-year-old female with a history of sick sinus syndrome status post pacemaker placement
presents to the ED with a chief complaint of dizziness and shortness of breath. Her vital signs are
T 98.6, P 42, BP 96/48, and SaO2 97% on RA. Her initial EKG demonstrates sinus bradycardia,
with occasional ventricular escape beats. No pacer spikes are observed. After a magnet is
applied to the patient’s pacemaker, a repeat EKG demonstrates a paced rhythm at a rate of 70
pg. 18
beats per minute. Which of the following is the most likely cause of the patient’s pacemaker
dysfunction?
A. Oversensing
C. Battery depletion
D. Failure to sense
E. Failure to capture
Answer A. Applying a magnet to the pulse generator causes the pacemaker to default to
asynchronous pacing by turning off (or bypassing) the sense amplifier. In other words, a magnet
“turns the sensing function off.” The pacemaker will deliver impulses (pacer spikes) at its
predefined default rate (sometimes called the “magnet rate”) regardless of the heart’s intrinsic
electrical activity. Since the pacemaker began pacing the patient’s heart appropriately after
magnet application, it is clear that it is generating impulses appropriately, and that those
impulses are appropriately captured by the heart (i.e., the pacer-spike–evoked ventricular
depolarization in the patient’s heart as intended). Thus, the problem lies in sensation. The
patient’s pacemaker was misinterpreting signal “noise” as actual native activity when none was
present. Since it detected what it thought was a native beat, it inhibited its output generation.
This is called oversensing and is more common in single-lead devices.
25. A 15-year-old previously healthy female is brought to the ED by her parents with a chief
complaint of shortness of breath (SOB), palpitations, lightheadedness, and severe fatigue. The
patient had been diagnosed with “a viral infection” by her pediatrician 5 days before presenting
to the ED. While she initially seemed to improve, she complains of feeling SOB with a sense that
her heart is “beating out of her chest” for the past day. Her parents noted she was breathing
fast, and she appeared to be “winded” even when sitting down. Upon arrival, she is pale and
ill-appearing, with vitals of T 98.6, P 143, RR 28, BP 105/70, and SaO2 of 92% on RA. A chest
x-ray demonstrates fluffy bilateral infiltrates. Her lung examination reveals tachypnea with
increased work of breathing, and rales without wheezes. Which of the following is true of the
patient’s likely underlying problem?
D. Two sets of blood cultures should be drawn and IV antibiotics, and aggressive IV fluids should
be administered to treat a likely superimposed bacterial infection.
pg. 19
Answer C. This patient is presenting with acute CHF, likely due to virally mediated myocarditis.
While patients with acute myocarditis often present in a more subtle manner than this patient,
patients may present with a broad range of symptoms, from vague generalized malaise to
fulminant heart failure. Therefore, the ED physician must maintain a high degree of suspicion in
patients presenting to the ED with new symptoms in the convalescent phase of a viral illness.
The most common arrhythmia in patients with myocarditis is sinus tachycardia. Patients rarely
present with supraventricular tachycardia, but when present, the rate is typically 180 to 260
beats per minute. Though the CK-MB and troponin are frequently measured in adults with chest
pain, CK-MB has not been shown to be either sensitive or specific for pediatric patients with
myocarditis. Elevated troponin levels also demonstrate poor sensitivity (34%) but relatively high
specificity (82%) for myocarditis. Therapy is not based on the levels of any biomarker. Since this
patient is presenting with fulminant heart failure, loop diuretics (e.g., furosemide) and inotropes
(e.g., dobutamine, milrinone) may be essential in her management. Vasodilators (e.g.,
nitroglycerin) are also a mainstay of therapy, but this patient’s relative hypotension may limit
their use. Early consultation with a pediatric cardiologist is also essential as the patient should
be transferred to a center with the capability of providing extracorporeal membrane
oxygenation, should that become necessary. The patient’s lack of fever or cough points against
pneumonia as a likely diagnosis. Echocardiography is an essential diagnostic tool, but patients
only rarely have pre-existing, undiagnosed congenital heart disease. Most often,
echocardiography reveals global left ventricular dysfunction, with variable degrees of left
ventricular cavity enlargement.
D. In patients with mechanical valves, auscultation of a metallic closure sound indicates serious
valvular dysfunction.
Answer C. A small degree of chronic hemolysis occurs in all patients, which in most cases is
clinically insignificant if appropriate iron supplementation is instituted. Mechanical, not porcine,
valves require anticoagulation with Coumadin, and the INR is generally kept between 2.5 and
3.5. Mechanical valves always have a metallic closure sound—absence of this sound indicates
valve dysfunction. The risk of endocarditis depends on the length of time the valve is functioning
but occurs in less than half the number of patients.
pg. 20
27. The amount of time after which a limb exposed to ischemia at room temperature (“warm
ischemia”) begins to develop irreversible damage is:
A. 1 hour
B. 3 hours
C. 6 hours
D. 12 hours
E. 24 hours
Answer C. After 6 hours of warm ischemia, 10% of patients will begin to develop irreversible
damage to muscles and nerves. After 12 hours, 90% of patients will have irreversible damage.
28. A 62-year-old male is being transported to the hospital by EMS with a chief complaint of
chest pain and shortness of breath. After placing the patient on oxygen and giving aspirin and
sublingual nitroglycerin, the EMS providers transmit the initial 12-lead EKG, which is shown in
Figure below. Which of the following is the most appropriate next step upon EMS arrival?
A. Prepare the catheterization laboratory’s ST-elevation myocardial infarction (STEMI) team for
immediate cardiac catheterization.
B. Start a nitroglycerin drip to be titrated until the patient is pain-free, and give a loading dose of
low–molecular-weight heparin.
C. Start a nitroglycerin drip, and give a loading dose of clopidogrel, and a loading dose of low–
molecular-weight heparin.
D. Start a nitroglycerin drip, and give 25 mg of oral metoprolol, and a loading dose of low–
molecular-weight heparin.
E. Place nitroglycerin paste on the patient’s chest, and give a loading dose of clopidogrel, oral
metoprolol, and a loading dose of low–molecular-weight heparin.
pg. 21
Answer A. The patient’s EKG demonstrates ST elevation in lead aVR with profound, diffuse ST
depressions in the remaining leads (see Fig. below). While lead aVR is routinely ignored in
clinical practice, it provides useful information. In patients with chest pain, ST elevation in lead
aVR, in concert with diffuse ST depressions, is a highly specific indicator of left main disease.
While all of the interventions noted in the other answers may be appropriate, none of them
favors immediate percutaneous coronary intervention (PCI) by cardiac catheterization, which is
the standard of care among patients with STEMI.
29. Which of the following is the most common complication of mitral stenosis (MS)?
A. Atrial fibrillation
B. Endocarditis
E. Pneumonia
Answer A. The most common complication of MS is atrial fibrillation, which puts the patient at
high risk for thrombus formation and embolism. Atrial fibrillation occurs due to the severe atrial
hypertrophy that results from the stenosed mitral valve preventing flow into the left ventricle.
30. Which of the following is the most appropriate medication to induce rapid, measured
reduction of blood pressure in hypertensive crises?
A. Nifedipine
B. Isoproterenol
pg. 22
C. Phenoxybenzamine
D. Nitroprusside
E. Hydrochlorothiazide
Answer D. Only intravenous medications are appropriate for rapid, measured control of blood
pressure. Nitroprusside is very easily titrated and extremely effective, making it the drug of
choice for hypertensive crises. Nifedipine has been associated with severe side effects due to its
unpredictable response. Isoproterenol is a β-agonist and will not decrease blood pressure.
Phenoxybenzamine is an α-blocking agent used mostly in the prevention of catecholamine surge
during therapy for pheochromocytoma. Hydrochlorothiazide is an oral medication appropriate
for outpatient therapy for chronic hypertension.
31. A 43-year-old otherwise healthy male presents with signs and symptoms of pneumonia.
Antibiotics are ordered and the patient is admitted. An admission electrocardiogram (EKG) is
ordered and is shown in Figure below. Which of the following is most likely?
pg. 23
Answer D. This patient’s EKG reveals classic or “type 1” Brugada syndrome. Brugada syndrome is
characterized by a pseudo-RBBB and persistent ST elevation in V1–V3. There are three unique
patterns of ST elevation consistent with Brugada syndrome. In type 1, the elevated ST segment
is convex facing upward, and gradually descends to an inverted T wave. This is referred to as a
“coved type” Brugada pattern, and is most common. Types 2 and 3 have identical patterns, in
which the elevated ST segment (≥1 mm in type 2, <1 mm in type 3) first descends and then rises
again after nearing the baseline, creating a “saddle back” appearance. It is associated with an
upright or biphasic T wave. Brugada syndrome is not normally associated with any structural
abnormalities. Most “standard” cardiac tests, including echocardiography, stress testing, and
cardiac MRI are unrevealing. However, patients are at a much increased risk for sudden cardiac
arrest due to ventricular arrhythmias. Unlike patients with hypertrophic obstructive
cardiomyopathy, sudden cardiac arrest is not typically due to exercise and more commonly
occurs in sleep. Most patients with Brugada syndrome undergo electrophysiology testing to
determine their risk for cardiac arrest as well as the need for an automated implantable
cardioverter-defibrillator
32. Which of the following is the most common risk factor associated with aortic dissection?
A. Smoking
B. Atherosclerosis
C. Marfan syndrome
D. Hypertension
Answer D. Although all the choices are risk factors for aortic dissection, the most common is
hypertension. Though approximately half the number of patients with Marfan syndrome
develop aortic dissections, <10% of all dissections are in Marfan patients.
33. Which of the following has the greatest immediate effect on preload in the management of
acute congestive heart failure (CHF)?
A. Morphine
B. Enalaprilat
C. Digoxin
D. Furosemide
E. Nitroglycerin
pg. 24
Answer E. Nitroglycerin directly dilates the great veins and reduces left ventricular preload.
Afterload reduction by nitrates is much less prominent. Morphine and loop diuretics such as
furosemide are also used for preload reduction but have a smaller effect than nitrates. While
morphine has been used for years as part of the standard treatment of CHF for preload
reduction, there have been few rigorously conducted randomized trials confirming its benefit.
Morphine most likely exerts its preload reducing effect by reducing anxiety and subsequent
catecholamine production and release. Loop diuretics cause preload reduction by inhibiting
renal sodium reabsorption thereby increasing urine volume and decreasing plasma volume.
However, the elevated afterload in patients with CHF reduces renal perfusion and thereby limits
the effectiveness of loop diuretics. Afterload reduction is most commonly achieved through the
use of angiotensin converteing enzyme (ACE) inhibitors such as enalaprilat, although
nitroprusside and hydralazine may also be used to achieve this effect. Digoxin causes an
increase in cardiac contractility without major changes on preload or afterload, and has no role
in the management of acute CHF. In fact, prior digoxin use has been shown to correlate to
increased in-hospital mortality among patients admitted with CHF.
34. A 67-year-old female with coronary artery disease (CAD) presents with dyspnea, which is her
last anginal equivalent. She is known to have a pre-existing left bundle branch block. Which of
the following EKG findings, when present in two contiguous leads, indicates the highest risk for
acute MI?
A. 4 mm of discordant ST elevation
B. 4 mm of concordant ST elevation
C. 2 mm of ST depression
D. Tall T waves
E. Inverted T waves
Answer B. The Sgarbossa criteria were devised to assess the likelihood of infarction in patients
with a left bundle branch block. Three different electrocardiographic criteria are given specific
scores: A total score of 3 or greater indicates that the patient likely has acute MI. Concordant (in
the same direction as the QRS complex) ST elevation >1 mm is given 5 points, ST depression >1
mm in V1– V3 is given 3 points, and discordant ST elevation >5 mm (in the opposite direction as
the QRS complex) is given 2 points.
35. Which of the following is true regarding the diagnosis of aortic dissection?
pg. 25
C. Transthoracic echocardiogram is useful to confirm the diagnosis.
Answer A. MRI is the most specific test for the diagnosis of aortic dissection. Logistic difficulties
prevent routine use of MRI in this setting—for this reason, CT aortogram is the most commonly
used test and has excellent sensitivity and specificity. Chest x-ray is abnormal in most cases, but
the sensitivity is not high enough to rule out the diagnosis in high-risk patients.
Transesophageal, not transthoracic, echocardiography may provide useful structural
information about the descending aorta, heart, and pericardium, but CT aortogram and MRI are
far more specific. Aortography is used only in confirmatory settings. Electrocardiography is
useful only in ruling out other causes of the patient’s symptoms and has no utility in confirming
the diagnosis of aortic dissection.
36. A 35-year-old female presents with severe weakness, lightheadedness, and chest pain. Her
blood pressure is 70/40 and her pulses are weak. The EKG is shown in Figure below. Which of
the following is the most important next step in management?
A. Adenosine 6 mg IV
B. Diltiazem 20 mg IV
C. Amiodarone 300 mg IV
D. Synchronized cardioversion at 50 J
E. Defibrillation at 200 J
pg. 26
Answer D. The EKG shows a regular, narrow complex tachycardia at a rate of 175, most likely
paroxysmal atrioventricular (AV) nodal re-entrant tachycardia (AVNRT). The patient is
hemodynamically unstable and synchronized cardioversion at 50 J is indicated after appropriate
sedation. In contrast to patients with AVNRT and atrial flutter, patients with rapid atrial
fibrillation, which can appear regular at high rates, do not typically respond to 50 J for
cardioversion. Thus, if the patient does not respond at 50 J, the energy level should be
immediately stepped up to 100 J and then 200 J, or to the equivalent energy dose if using
biphasic equipment. Adenosine is the next best option, but hypotension and severe symptoms
warrant more emergent conversion to sinus rhythm. Diltiazem and amiodarone can be used to
convert the rhythm, but will cause further hypotension. Defibrillation is only indicated in
patients who lack pulses (ventricular fibrillation, pulseless ventricular tachycardia).
37. A 55-year-old male presents with severe chest pain radiating to the back. A CT scan of the
chest reveals both ascending and descending aortic dissections. Which of the following is true
regarding this patient?
C. Aspirin should be given in case the dissection has extended to the coronary arteries.
Answer B. The patient has a Stanford class A aortic dissection—a tear involving the ascending
aorta. Management involves emergent surgical repair along with early, aggressive BP control.
Patients with aortic dissection often complain of severe chest pain radiating to the back or both
arms. Aspirin should not be given to any patients suspected of having an aortic dissection, as
this may increase the degree of bleeding into the false lumen. HR should be kept well below 100
to minimize the shear stress on the wall of the aorta that is related to the number of beats per
minute. Transesophageal, not transthoracic, echocardiography may provide useful structural
information about the descending aorta, heart, and pericardium, but CT aortogram and MRI are
far more specific.
38. A 4-week-old infant is brought in by his parents breathing fast and appearing sweaty. His
respiratory rate is 54, BP 88/60, P 200. His skin is pink and capillary refill is about 2 seconds. His
EKG is shown. Which the following is true (Fig. below)?
A. A bag of ice should be held over the infant’s nose and mouth for 30 seconds
pg. 27
D. Electrical cardioversion should be performed immediately
Answer A. This patient has supraventricular tachycardia (SVT). Since the diving reflex is very
effective in neonates, the first step is to hold a bag of ice or a frozen towel over the patient’s
nose and mouth for 30 seconds. If the infant fails to convert, the next step is to use adenosine.
Verapamil is avoided in infants because it may cause significant hypotension. Digoxin, on the
other hand, may be effective, but has a slow onset. Electrical cardioversion is effective, but is
reserved for patients with refractory symptoms or who are in shock. Given that this patient has
good perfusion and is maintaining his blood pressure, electrical cardioversion is not needed.
Finally, while WPW can lead to SVT, it is not possible to determine if WPW underlies the
patient’s SVT until after cardioversion.
39. A 63-year-old male with a history of nephrolithiasis presents to the ED with acute onset right
flank pain. He receives an uncontrasted CT scan which is unrevealing, and he has no hematuria.
A repeat study with contrast is performed, a slice of which is shown in Figure below. The patient
feels better and has minimal pain. His vital signs are T 98.4, P 84, RR 16, BP 156/82, SaO2 98% on
RA. Which of the following is true (Fig. below)?
pg. 28
E. An upper extremity pulse deficit is the most specific physical examination finding.
Answer E. Acute aortic dissection is a hypertensive emergency (see Fig. below). Initial treatment
should focus on reducing the patient’s blood pressure to a systolic blood pressure of 100 to 120
mm Hg using parenteral beta-blockers (labetalol, esmolol, propranolol). Beta-blockers are the
preferred agent because they also reduce heart rate and aortic wall stress by reducing the rate
of systolic blood pressure rise. While hypertension is the most common risk factor,
approximately 25% of patients have no history of hypertension and approximately one-third are
normotensive at presentation. EKGs are not usually helpful to differentiate acute aortic
dissection from myocardial infarction as most EKGs in patients with either condition are
nonspecific. There are ongoing investigations into the use of low D-dimer levels as a means of
excluding aortic dissection. However, there are no validated prediction rules to determine the
exact pretest probability in patients with suspected aortic dissection, and no outcome studies
have demonstrated the safety of using “negative” D-dimer levels as a means to rule out aortic
dissection. As with pulmonary embolism, elevated D-dimer levels do not significantly increase
the likelihood of acute aortic dissection. An upper extremity blood pressure disparity is a
classically described, though somewhat unreliable finding in patients with aortic dissection. A
pulse deficit between the upper extremities is the most reliable physical examination finding,
and pulses should always be assessed to determine whether there are discrepancies.
Unfortunately, the sensitivity of this finding is low, as only 15% of patients in the International
Registry of Acute Aortic Dissection (IRAD) study had such a deficit.
pg. 29
40. Which of the following is true regarding heart transplant recipients?
Answer C. Due to denervation in the transplanted heart and the consequent lack of vagal tone,
the resting heart rate averages between 100 and 110 beats per minute. However, the heart rate
can increase up to 70% of the maximum for age due to circulating endogenous catecholamines
and upregulation of β-adrenergic receptors. Although rare, tamponade can occur in the
transplanted heart because of scar tissue formation and its ability to contain pericardial fluid or
blood under pressure. Before the advent of cyclosporine, acute rejection presented as
acute-onset CHF or atrial dysrhythmias with a new S3 and diffusely decreased QRS voltage on
EKG. These features are now only present in cases of severe failure, and typical acute rejection,
which occurs in 75% to 85% of patients, is diagnosed by endomyocardial biopsy. There is an
increased risk of endocarditis with invasive procedures, so antibiotic prophylaxis should be used
in any procedure expected to produce bacteremia.
pg. 30
41. A 57-year-old male smoker with hypertension, hypercholesterolemia, and peripheral
vascular disease presents unable to walk due to pain. He has had progressive worsening of pain
in both his legs on walking for several weeks, and the left calf has been extremely painful for 2
days. Vital signs are T 98.2, HR 90 regular, BP 175/90, RR 20, SpO2 98% RA. Examination reveals
a regular heart rate, 1+ dorsalis pedis (DP)/posterior tibialis (PT) pulses on the right, absent
DP/PT pulses on left, ABI 0.55 on the right and 0.40 on the left, no bruits, and no signs of
infection. Which of the following is the most likely pathophysiologic mechanism?
A. In situ thrombosis
B. Arterial embolism
C. Inflammation
D. Vasospasm
Answer A. The patient has chronic peripheral vascular disease leading to acute in situ arterial
thrombosis in the left leg. Ankle–brachial index (ABI) is >0.9 in patients without peripheral
vascular disease, and bilateral reduction in the ABI indicates chronic in situ disease. Arterial
embolism is also a very common cause of acute arterial occlusion, but the history of
progressively worsening claudication before the acute event is more often seen with in situ
thrombosis. Isolated inflammation, vasospasm, and fistulas may also cause arterial occlusion but
are far less common than thrombosis and embolism.
42. A 54-year-old male presents with a 4-hour history of chest pain, which has resolved at
presentation. He also complains of diaphoresis, shortness of breath, and orthopnea. Vital signs
are 98.6, 60, 22, 97/60, 92% on RA. Physical examination demonstrates an S3 and bilateral
crackles to the midlung fields. The EKG is shown in Figure below. Which of the following is the
most appropriate definitive management?
D. Admit for immediate heparin, G2b3a inhibitor, and cardiac care unit admission
pg. 31
Answer A. The patient has acute anterior ST-segment elevation myocardial infarction (STEMI)
with evidence of congestive heart failure (CHF) due to cardiogenic shock. In these patients,
immediate angioplasty has been shown to reduce mortality more than fibrinolytic therapy and is
the preferred definitive therapy. Choices C and E are not appropriate for patients with acute
STEMI. Choice D represents treatment of a non-ST-segment elevation myocardial infarction
(NSTEMI).
43. A 32-year-old female with a history of systemic lupus erythematosus (SLE) presents with
chest pain. EKG shows sinus tachycardia. Which of the following is the most likely cause?
C. Pericarditis
D. Myocarditis
E. Endocarditis
Answer C. Pericarditis is the most common cardiac complication of SLE. While not usually
serious by itself, associated pericardial effusions can potentially cause hemodynamic
compromise. Chest pain is the most common symptom of pericarditis in SLE patients.
Myocarditis also commonly occurs in patients with SLE, but it is often clinically silent and overt
symptoms are more likely to be related to signs of CHF such as shortness of breath or peripheral
edema. Coronary artery disease can complicate SLE as well but is less common than either
pericarditis or myocarditis. Noninfective endocarditis (also known as Libman–Sacks endocarditis)
pg. 32
is present in a minority of patients with SLE. Lesions can get superinfected and cause valvular
insufficiency, often without frank chest pain. The most common overall cause of chest pain in
patients with SLE is likely musculoskeletal disease.
44. A 56-year-old diabetic male presents to the ED after a brief episode of chest pain. His initial
EKG reveals no ischemia and his troponin is negative. Which of the following is true about
coronary CT angiography (CCTA) in this patient?
A. If the HEART score stratifies the patient as low risk, the CCTA provides no additional useful
information
D. CCTA may provide useful prognostic information to help with risk stratification
45. Which of the following is indicated for treatment of a stable, wide-complex regular
tachycardia at a rate of 200 in a patient with Wolff–Parkinson–White (WPW) syndrome?
A. Adenosine
B. Amiodarone
C. Esmolol
D. Digoxin
E. Procainamide
pg. 33
Answer E. WPW syndrome is the most frequently occurring accessory pathway syndrome.
Patients have an accessory conductive pathway (bundle of Kent) from the atria to the ventricles
which preexists the ventricular myocytes before the AV node conducts the normal sinoatrial
depolarization wave. As a result, patients with WPW have a shortened PR interval and a delayed
QRS upstroke, called the δ wave. Patients with WPW syndrome can have re-entrant
dysrhythmias, in which the accessory pathway can either conduct in a retrograde fashion
producing an “orthodromic” pattern (in which the AV node conducts in the normal direction,
which results in a narrow QRS complex) or an anterograde fashion forming an “antidromic”
pattern (in which forward [or anterograde] conduction occurs through the accessory pathway
first before cycling around to pass through the AV node backward, which produces a wide QRS
complex since the initial conduction does not utilize the Purkinje system). A patient with WPW
syndrome, tachycardia, and wide QRS complexes suggests the presence of an antidromic
conduction pattern (where the accessory pathway conducts anterograde and the AV node
conducts retrograde). Agents with relatively “pure” or isolated AV nodal activity such as
adenosine or verapamil (which are the preferred agents in orthodromic re-entrant tachycardias
in the setting of WPW) are contraindicated in this circumstance, as inhibition of the AV node will
promote faster conduction through the anterograde accessory pathway, which may cause
degeneration into an unstable rhythm such as ventricular tachycardia. The treatment of choice
in stable antidromic or irregular tachycardias in WPW patients is procainamide, while
amiodarone is considered a second-line agent. Unstable patients require cardioversion.
46. A 65-year-old male presents with a syncopal event without prodromal symptoms. His vital
signs are 98.6, 60, 18, 142/75, 99% RA. The EKG is shown in Figure below. Which of the following
is the most appropriate next step in management?
B. Cardioversion at 50 J
C. Defibrillation at 200 J
D. Amiodarone 150 mg IV
pg. 34
Answer E. The patient has Mobitz type II second-degree AV block, which is an indication for
admission to a telemetry unit and possible pacemaker placement. The PR interval is prolonged
and does not show any observable pattern of increasing prolongation as in Mobitz type I
second-degree AV block (Wenckebach phenomenon). Dropped QRS complexes do not occur in
any predictable pattern like they do in Mobitz type I. Cardioversion and amiodarone are not
indicated in stable brady dysrhythmias, and defibrillation of an awake patient is never indicated.
47. A 17-year-old basketball player collapses and dies while playing a game. Which of the
following is the most likely cause?
B. Pulmonary embolism
C. Hypertrophic cardiomyopathy
E. Spontaneous pneumothorax
Answer C. Hypertrophic cardiomyopathy is the most common cause of atraumatic sudden death
in young athletes. Choices A, B, and D are far less common in this age-group. Choice E can occur
in tall, thin athletes, and although death may result from tension physiology, it is almost never
sudden or without preceding symptoms.
pg. 35
48. Which of the following is the most important factor in determining the risk of rupture of an
abdominal aortic aneurysm (AAA)?
B. Hypertension
E. Male gender
Answer C. Aneurysms >5 cm in size are at greatest risk of rupture, although smaller aneurysms
may also rupture. Debate exists as to the optimal time of elective repair of asymptomatic
aneurysms, but patients presenting to the ED with symptomatic aneurysms should always be
aggressively evaluated.
49. Which of the following represents the approximate proportion of acute myocardial
infarctions (MIs) that occur without a history of chest pain?
A. 1%
B. 5%
C. 10%
D. 25%
E. 50%
Answer D. Elderly patients, diabetic patients, and women with acute MIs may have atypical
symptoms without the classic history of left-sided chest pain. Dyspnea, nausea, and diaphoresis
may be anginal equivalents in many patients. Additionally, patients may deny “pain”—rather,
they may describe their symptoms as “pressure” or “discomfort.”
50. A 65-year-old female presents in out-of-hospital cardiac arrest with ventricular fibrillation.
Which of the following is the most effective management for this condition?
A. Defibrillation
B. Pericardiocentesis
C. Epinephrine IV
pg. 36
D. Amiodarone IV
51. Which of the following is most effective in reducing mortality from acute MI?
A. Metoprolol
B. Aspirin
C. Nitroglycerin
D. Abciximab
E. Morphine
Answer B. Antiplatelet therapy with aspirin is still the most effective and cheapest medical
therapy for treatment of acute MI. No other single pharmacologic agent can boast of as large a
mortality reduction, including metoprolol and abciximab. Nitroglycerin and morphine do not
reduce mortality in acute MI, but they are effective in managing symptoms of angina.
52. A 43-year-old female presents with a painful area in her calf for several days. She read on
the internet about deep venous thromboses and is worried she has one. She does not take oral
contraceptive therapy and is a nonsmoker. She has a definite, focal area of tenderness in her
calf without obvious cellulitis. A duplex ultrasound reveals a 1.5-cm superficial vein thrombosis
below the knee. Which of the following is the next best step in management?
D. Warfarin
pg. 37
E. Intravenous tissue plasminogen activator
Answer C. The patient has superficial thrombophlebitis, which can be managed supportively
with NSAIDs, rest, and heat. Without symptoms or signs of or risk factors for a pulmonary
embolism, evaluation for pulmonary embolism with a CT is not warranted. Repeat ultrasound is
usually undertaken several days to a week after the initial evaluation of DVT. Warfarin therapy is
not usually initiated alone due to its early prothrombotic effect. Furthermore, anticoagulation is
only considered for large superficial thromboses (>5 cm) and thromboses above the knee, as
these are more likely to progress to deep venous thromboses. Intravenous tissue plasminogen
activator is only indicated for large pulmonary emboli with hemodynamic effect.
53. A 22-year-old male presents with palpitations. He reports no chest pain, shortness of breath,
or lower extremity edema. He states that he “got really drunk” the night before. He denies any
past medical history, family history, or illicit drug use. His examination is unremarkable except
for irregulartachycardia (Fig. below). Which of the following is the most likely etiology?
B. Pulmonary embolism
C. Alcohol use
D. Hypertension
E. Diabetes
pg. 38
Answer C. The EKG demonstrates atrial fibrillation with rapid ventricular response. The most
likely cause in this young adult with no medical history except for alcohol abuse is “holiday heart
syndrome,” which can occur within 2 days of an alcohol binge. It often resolves spontaneously
but may require rate control therapy and possibly anticoagulation. In the absence of cocaine
use, MI would be extremely unlikely in an otherwise healthy 22-year-old. Pulmonary embolism
is an important cause of atrial fibrillation but is not likely in the absence of chest pain, dyspnea,
or risk factors. The physical examination is unremarkable, so hypertension is unlikely. Diabetes
does not confer an increased risk of atrial fibrillation.
54. A 55-year-old patient presents with palpitations for 1 week (Fig. below). The patient is sent
to the emergency room (ER) for further evaluation. Which of the following is the most
appropriate next step in management?
A. Diltiazem 20 mg IV
B. Esmolol 50 μg/kg/minute IV
C. Enoxaparin 1 mg/kg SC
D. Amiodarone 150 mg IV
E. Adenosine 6 mg IV
Answer C. The EKG demonstrates atrial flutter at a ventricular rate of 50. The risk of atrial
thrombus increases with the amount of time the patient is in atrial fibrillation or atrial flutter.
Emergent management of atrial fibrillation or flutter involves reduction of rate to below 100 and
pg. 39
anticoagulation if the duration of the dysrhythmia is longer than 48 hours, unless
echocardiogram indicates no cardiac thrombus. This patient is not tachycardic and requires no
rate controlling agents such as diltiazem or esmolol. Amiodarone is not indicated as this may
actually terminate the atrial flutter and put the patient back into sinus rhythm and at risk for
thrombo embolus. Adenosine is indicated for paroxysmal supraventricular tachycardia—it may
be used in unclear cases of narrow complex tachycardia but has no role in obvious atrial flutter.
55. A 65-year-old female presents with lightheadedness. She denies chest pain or shortness of
breath. Vital signs are 99.0°F, 160, 20, 144/75, 96% RA (Fig. below). Which of the following is the
most appropriate next step in management?
A. Adenosine
B. Diltiazem
C. Amiodarone
D. Cardioversion at 50 J
E. Cardioversion at 200 J
Answer C. The EKG shows a wide-complex, regular tachycardia, which is almost always
ventricular tachycardia (VT). Amiodarone, procainamide, or lidocaine may be used to treat
stable VT. Although cardioversion may also be performed, it is painful for awake patients and
may not be necessary for patients without hemodynamic instability. Adenosine and diltiazem
are both used in patients with narrow-complex tachycardias and have no role in ventricular
pg. 40
dysrhythmias. Supraventricular tachycardia (SVT) with aberrant conduction can also cause a
regular, wide-complex tachycardia. VT is far more common, and may be distinguished from SVT
with aberrancy by the presence of fusion beats, atrioventricular dissociation, wider QRS
complexes (>0.14 second), and concordance of precordial leads. When in doubt, the emergency
physician should always treat regular wide-complex tachycardia as VT.
56. A 34-year-old male presents with chest pain for 4 hours, resolved at presentation. He has
been smoking marijuana, which he obtained from an unknown source. His vital signs are 98.6°F,
115, 20, 167/95, 98% RA. His physical examination reveals pupillary dilation, tachycardia, and
diaphoresis. His electrocardiogram (EKG) demonstrates ST depressions in V3–V5. Initial troponin
level is 0.20 ng/mL. Which of the following is the most appropriate therapy at this time?
C. Abciximab
D. Metoprolol
E. Aspirin
Answer E. The patient is exhibiting a sympathomimetic picture after smoking marijuana, which
commonly occurs because of crack cocaine additives. His myocardial ischemia is likely because
of a combination of vasospasm and hyper aggregatory platelets. Aspirin is indicated for
treatment. His chest pain is resolved and he is not having an ST-segment elevation myocardial
infarction, so PTCA, tPA, and GpIIbIIIa inhibitors are not warranted. Metoprolol is relatively
contraindicated in cocaine induced chest pain as there is a theoretical risk of reducing cardiac
output by inhibiting β-receptors in the setting of increased peripheral vascular resistance (due to
α-receptor stimulation).
57. A 62-year-old female with a history of hypertension presents with sudden onset of severe
chest pain with radiation to the back. Her vital signs are: 98.7, 100, 18, 195/90, 99% RA. You
suspect the diagnosis of aortic dissection. Her EKG and chest x-ray are normal. Which of the
following is the next best step in evaluation?
A. D-dimer
B. Coronary angiogram
C. CT aortogram
D. V-Q scan
E. Transthoracic echocardiogram
pg. 41
Answer C. One out of every six patients with aortic dissection has a normal chest
x-ray—therefore, normal chest x-ray cannot be used to rule out an aortic dissection. CT
aortogram is the optimal test to evaluate for aortic dissection. An alternative would be
transesophageal echocardiogram, though this may be difficult to obtain in an emergent fashion.
The role of D-dimer in the evaluation of aortic dissection is limited to patients who lack high-risk
features such as abrupt or severe pain, aortic disease, pulse deficit, shock, or focal neurologic
deficit. These lower risk patients may be ruled out with a negative D-dimer test. Emergent
coronary angiogram should be reserved only for patients with ST-elevation myocardial infarction
or unremitting unstable angina. V-Q scan is important in the evaluation of pulmonary embolism,
not aortic dissection. Transthoracic echocardiogram lacks the sensitivity necessary to rule out
aortic dissection.
D. Characteristic EKG changes usually distinguish Prinzmetal angina from acute myocardial
infarction (MI).
Answer B. Prinzmetal (or variant) angina is characterized by chest pain caused by coronary
artery vasospasm, which can result in ST-elevation myocardial infarction, arrhythmia, and
sudden death. Prinzmetal angina can occur in concert with atherosclerotic heart disease or may
be completely unrelated. A relative reduction in nitric oxide is hypothesized to be the cause. It is
often clinically and electrocardiographically indistinguishable from atherosclerotic coronary
artery disease (CAD). Patients with Prinzmetal angina can have a decrease, increase, or no
change in their pain with exercise—for this reason, history of exertional angina or exercise stress
testing is of limited value in diagnosing Prinzmetal angina. Variant angina may be relieved by
nitroglycerin. β-blockers, as in atherosclerotic CAD, form part of the cornerstone of
management.
59. A 45-year-old female with a history of untreated hyperthyroidism presents with acute onset
of left foot pain. Physical examination reveals normal vital signs, an irregular heart rhythm, clear
lung sounds, and loss of pulses in the left foot with decreased capillary refill, and cyanotic,
paralyzed toes. Which of the following is the most appropriate next step in management?
pg. 42
C. Anticoagulation alone
D. Lumbar sympathectomy
Answer A. The patient has acute arterial occlusion from arterial embolism, likely due to atrial
fibrillation caused by hyperthyroidism. Treatment involves anticoagulation and emergent
embolectomy due to the limb-threatening nature of the occlusion. Bypass surgery is usually
used in patients who have in situ thrombosis. Anticoagulation alone is used as adjunctive ED
therapy for patients with acute arterial occlusion, but it is usually not adequate to treat
limb-threatening ischemia due to an embolus. Lumbar sympathectomy and hyperbaric oxygen
therapy provide no benefit in these circumstances.
60. A prostaglandin E1 infusion may help with which of the following lesions?
Answer E. Prostaglandin E1 can maintain the patency of the ductus arteriosus which allows
mixing between the left (aorta) and right (pulmonary artery) sides of the circulation. Thus, a
patient with any ductal dependent lesion will benefit from a prostaglandin E1 infusion. These
lesions include both cyanotic and noncyanotic lesions. Left-sided lesions include critical aortic
stenosis, critical coarctation, hypoplastic left heart syndrome, and interrupted aortic arch.
Right-sided lesions include tetralogy of Fallot with pulmonary atresia, tricuspid atresia, and
critical pulmonic stenosis. Transposition of the great arteries is also duct-dependent and as it
consists of parallel circulations, it exists in a category by itself.
61. A 71-year-old male with a prosthetic heart valve presents to the ED. Which of the following
is an indication for antibiotic prophylaxis?
A. IV placement
D. Urethral catheterization
pg. 43
E. Nasogastric tube insertion
Answer C. In the ED, antibiotic prophylaxis is needed for surgical procedures of infected skin and
soft tissue or urethral catheterization in patients with a known urinary infection. Routine
catheterization does not require prophylaxis. Dental procedures are among the highest risk
procedures but are not typically performed in the ED. If an I and D of a periapical dental abscess
is planned, then prophylaxis is appropriate.
62. A 67-year-old male with hypertension presents with acute onset of abdominal pain. The pain
is periumbilical and radiates to the left lower quadrant. On physical examination, BP is 140/90
and an abdominal mass is noted near the umbilicus. A bedside ultrasonograph is shown in
Figure 8-2. After the ultrasonography, the BP drops to 70/40 and the patient becomes
lightheaded and dizzy. Which of the following is the most appropriate next step in
management?
D. Emergent surgery
E. IV crystalloid to normalize BP
Answer D. The patient has an AAA seen on ultrasonograph. In association with hypotension and
abdominal pain this must be interpreted as an acutely rupturing AAA, and the only management
that will save him is surgery. Further confirmatory imaging in this scenario will only prolong
definitive management with little yield and high risk for a poor outcome. Administration of
pg. 44
intravenous fluids, though important in resuscitation efforts pending operating room and
surgeon availability, will not be able to normalize BP or prevent certain death in patients who
are hypotensive with a rupturing AAA.
64. In an acute NSTEMI, which of the following advantages does bivalirudin possess over
unfractionated and low–molecular-weight heparins?
A. Improved mortality
pg. 45
patients with NSTEMI undergoing an “early-invasive” percutaneous intervention (PCI) and for
patients at increased risk for bleeding who are undergoing immediate PCI. Fondaparinux,
enoxaparin, and unfractionated heparin are the preferred agents among patients with NSTEMI
not undergoing PCI (“conservative” management).
65. Among patients who have contraindications to beta-blocker therapy, which of the following
calcium channel blockers (CCBs) can be given as a substitute for beta-blockers to patients with a
non-ST elevation MI (NSTEMI)?
A. Nimodipine
B. Diltiazem
C. Amlodipine
D. Nifedipine
Answer B. The majority of CCBs used in human pharmacology are dihydropyridines, which are
relatively vascular selective with minimal effects on cardiac contractility and conduction. In
contrast, verapamil and diltiazem are non-dihydropyridines and are less potent vasodilators
with more pronounced depressive effects on cardiac contractility and conduction. The use of
beta-blockers in patients with NSTEMI is a class I recommendation due to their demonstrated
benefit in reducing mortality among patients with acute myocardial infarction. This is primarily
based on their ability to limit myocardial oxygen demand by reducing the heart rate as well as
the force with which the heart contracts. In patients who are unable to tolerate beta-blockers,
non-dihydropyridine CCBs, such as verapamil or diltiazem, can be used instead. Each of the
other listed CCBs is a dihydropyridine with limited cardiac effects. Nimodipine has a specific
affinity for the cerebral vasculature and is used clinically only in patients with subarachnoid
hemorrhage to limit vasospasm.
66. A 69-year-old male with a history of hypertension presents with sudden onset of severe
chest pain with radiation to the back. His vital signs are: 99.0, 100, 18, 195/90, 99% RA. You
diagnose a Stanford Type B aortic dissection on CT aortogram. Which of the following is the next
best step in management?
A. Nitroglycerin IV
B. Labetalol IV
C. Nicardipine IV
D. Nifedipine PO
pg. 46
E. Clonidine PO
Answer B. Patients with aortic dissection require aggressive management of both heart rate and
blood pressure. The shear forces that contribute to the development and propagation of a false
lumen in aortic dissection are worsened by elevated blood pressure as well as the number of
times the heart pumps. Labetalol accomplishes reduction of both heart rate and blood pressure
and is the optimal first-line agent for management of aortic dissection. Nicardipine is another
reasonable choice, but has less effect on the heart rate and, therefore, should be started after
labetalol. Nitroglycerin can cause reflex tachycardia and should not be the first-line agent for
aortic dissection. Oral medications should not be given in aortic dissection due to unreliable
absorption and therapeutic effect.
67. Which of the following is true regarding the physical examination for patients with
abdominal aortic aneurysm (AAA)?
E. Abdominal obesity does not appreciably affect the ability to palpate aortic aneurysm.
Answer C. Aneurysms >5 cm in size are usually palpable on physical examination and
approximately half the number of aneurysms between 4 and 5 cm are palpable. Audible
abdominal bruits due to AAA are rare. Rupture of an aneurysm due to even vigorous palpation
almost never occurs. Femoral pulses are usually intact in patients with AAA. Truncal obesity
makes detection of AAA on physical examination much more difficult.
68. A 46-year-old morbidly obese (BMI >40) male patient is referred to the ED to exclude a deep
venous thrombosis (DVT) after he presented to his primary care physician’s office with calf
swelling and pain for 1 week. His subsequent Doppler study demonstrates a thrombosis limited
to the peroneal vein. Which of the following describes the next best steps in management?
A. Initiate anticoagulation with warfarin or a direct factor Xa or thrombin inhibitor, and refer for
primary care follow-up
B. Treat for superficial thrombophlebitis with pain medicine and anti-inflammatory medicine
C. Refer the patient for a repeat Doppler study in 1 week but do not initiate treatment
pg. 47
D. Initiate anticoagulation with warfarin or dabigatran and low–molecular-weight heparin, and
refer him to interventional radiology for placement of an inferior vena cava filter
E. Prescribe furosemide for fluid retention, and counsel the patient to wear support hose and to
elevate the legs
Answer A. Due to its higher complication rate, proximal (in the thigh) DVT is a more important
clinical entity than distal (in the calf ) DVT. Proximal DVT is also more commonly associated with
persistent rather than transient risk factors. More than 90% of pulmonary emboli originate from
a proximal DVT. In low-risk patients diagnosed with a distal DVT who also have transient risk
factors (e.g., recent travel, recent prolonged immobilization), a conservative approach involving
serial Doppler examinations is reasonable since it limits the significant risks associated with
anticoagulation. Many of these patients will experience complete recanalization over time
without any extension of the thrombosis to the proximal veins. However, obese patients with a
BMI >40 are at significantly increased risk for development of a DVT. Since the patient’s obesity
is a chronic problem that is not going to be resolved in a short time frame, he is best treated
with anticoagulation as for patients with a proximal DVT.
D. Dissection into coronary arteries occurs most often in the left coronary artery.
Answer B. Syncope with aortic dissection mandates evaluation of the pericardial space to check
for hemopericardium and tamponade. Dissections extending back to the pericardium are
ascending and should be taken emergently to surgery. The most common symptom of aortic
dissection is chest pain, although dyspnea is often seen as well. Aortic regurgitation is not
usually seen in aortic dissection but indicates a valvular disruption when present. Dissection into
the coronary arteries occurs rarely, but when it does, the right coronary artery is most
frequently affected. Interarm differences in blood pressure due to unilateral extension of the
dissection into the subclavian artery occur in the minority of patients with aortic dissection.
However, in patients with undifferentiated chest pain, interarm differences of >20 mm Hg
indicate a higher likelihood of aortic dissection.
70. A 65-year-old female with a history of diabetes and hypertension presents with a 5-hour
history of chest pain. Physical examination demonstrates blood pressure of 100/55, jugular
pg. 48
venous distension, and hepatomegaly. No crackles are noted. EKG demonstrates inferior ST
depressions. You suspect the possibility of right ventricular infarction. You instruct the ED
technician to place the rV4 lead at
Answer B. Right ventricular infarction can be better assessed with a right-sided EKG, which is the
mirror image of a standard 12-lead EKG. The right-sided V4 lead (rV4) is placed at the right fifth
intercostal space, midclavicular line. Elevation of the ST segment in this lead is most specific for
right ventricular infarction. Nitrates should be avoided in patients with right ventricular
infarction, as these patients are very dependent upon passive filling of the right heart by the
great veins (preload) due to the decreased active contraction of the right ventricle. Reducing the
preload with nitroglycerin will reduce the passive filling and cause hypotension.
71. A 65-year-old female presents with 2 hours of severe, diffuse, progressively worsening acute
abdominal pain. Blood pressure is 150/90, and abdominal examination demonstrates a palpable
pulsatile mass. Abdominal CT scan is shown in Figure 9-3. Which of the following is true
regarding management of this patient?
B. The patient should be crossmatched for 10 units of packed red blood cells.
pg. 49
Answer B. The patient has a ruptured abdominal aortic aneurysm (AAA), as indicated by the
adjacent retroperitoneal hematoma. Patients with ruptured AAAs must undergo immediate
operative repair unless there is an absolute contraindication to surgery, as loss of complete
blood volume can occur within minutes. There is no evidence that aggressive reduction of blood
pressure is helpful in AAA, and it increases the unnecessary risk of hypotension and worsened
hypoperfusion. Once a ruptured AAA is definitively diagnosed by CT scan, there is little role for
further imaging in the ED with angiography or ultrasound. Observation of patients with ruptured
AAAs is contraindicated.
A. Procainamide
B. Electrical DC cardioversion
C. Adenosine
D. Amiodarone
E. Verapamil
pg. 50
Answer B. This patient is presenting with an unstable, wide-complex tachycardia. Given that the
patient is unstable (hypotensive, pale, diaphoretic), the exact details of her underlying rhythm
won’t change her need for immediate DC cardioversion. The vignette points to the fact that this
patient likely has atrial fibrillation with concomitant WPW (irregular rhythm), but all
wide-complex tachycardias should first be considered to be ventricular in origin until proven
otherwise. If it’s clear that the patient is stable and has atrial fibrillation with WPW, the
treatment of choice is procainamide followed by amiodarone or ibutilide. Agents which
preferentially block AV node conduction such as adenosine, verapamil, or diltiazem should be
avoided because their use may lead to unopposed “shunting” of electrical activity through the
unblocked accessory pathway, worsening the dysrrhtyhmia.
73. A 41-year-old female is referred from her dentist because her blood pressure was 212/105
when she presented for her root canal. She complains of dental pain in the area of the affected
tooth, but she denies headache, blurry vision, nausea, or extremity weakness or numbness. The
remainder of her review of systems is negative. Her physical examination is unrevealing except
for carious teeth. She has never been diagnosed with hypertension, but admits that she had not
seen a physician “in years.” She has no drug allergies or known contraindications to any specific
drug therapy. Which of the following is the next best step?
A. Administer clonidine 0.1 mg PO every hour until her blood pressure is less than 185/110 and
then discharge.
B. Obtain a chest x-ray, EKG, head CT, CBC, basic metabolic panel, and cardiac enzymes to help
determine her disposition.
C. Start the patient on hydrochlorothiazide 25 mg PO daily, and discharge her to follow-up with
her primary care physician.
D. Administer one dose of IV hydralazine until the patient’s BP is lower than 185/110, then
switch to oral therapy, and discharge if asymptomatic.
E. Initiate an IV labetalol infusion and admit the patient with a diagnosis of hypertensive
urgency.
pg. 51
and has no role in the ED management of hypertension unless it is part of the patient’s existing
regimen. There is no role for admission, extensive testing, or IV therapy in asymptomatic
patients. Symptomatic patients should undergo a workup based on their symptoms, and should
only receive IV therapy if there is also an emergent condition necessitating an emergent
reduction in blood pressure. It is reasonable to check renal function if starting antihypertensive
therapy, particularly with ACE inhibitors, but it is not mandatory if the patient has close
follow-up.
74. A 57-year-old female with a history of tobacco abuse, hypertension, and end-stage renal
disease on hemodialysis through a left upper extremity arteriovenous fistula presents from her
dialysis clinic with a chief complaint of chest pain. The pain began 3 hours after the start of her
session, and enough fluid was removed to bring her close to her dry weight. She has no
associated shortness of breath. Her vitals are stable. The initial EKG demonstrates nonspecific
changes. Her chest x-ray is normal. Her subsequent blood tests reveal an elevated cTnI of 0.83
g/μL (reference normal <0.49). She was given four chewable 81-mg aspirin upon arrival to the
ED. Which of the following is true?
A. A cardiac troponin T should be ordered because it is more specific for myocardial infarction in
patients on hemodialysis.
B. The patient has had an non-ST-elevation myocardial infarction (NSTEMI) and appropriate
treatment should be initiated.
D. The MB fraction of creatine kinase should be ordered because it is the most specific test for
myocardial infarction in patients on hemodialysis.
E. Specific treatment for an NSTEMI should be withheld since only serial, repeated enzyme
elevations are diagnostic for a myocardial infarction.
Answer B. Falsely positive and nondiagnostic elevations of serum troponins and creatine kinase
are common among dialysis patients. However, cTnI is much more specific for myocardial injury
than either cardiac troponin T (cTnT) or the CK-MB. In a large trial of asymptomatic dialysis
patients, only 0.4% had cTnI values greater than 0.6 μg/L. In smaller studies of patients with
suspected myocardial infarction, there has been no trend in false elevations of cTnI. While it is
true that the most specific finding for a myocardial infarction is serial elevation of cTnI, an initial
significant elevation in the setting of a history, and findings suggestive of NSTEMI should prompt
initiation of appropriate treatment. Delays in treatment and poorer outcomes are common
among patients with end-stage renal disease, in part due to the mistaken belief that most
increased cTnI values are falsely elevated. Even among patients without chest pain, there is
increasing evidence that elevated troponin is correlated with increased short- and long-term
mortality due to significant cardiac events. The etiology for this finding is not entirely clear.
pg. 52
75. A 63-year-old male with a history of coronary artery disease, diabetes, chronic obstructive
pulmonary disease (COPD), and congestive heart failure (CHF) presents with increased cough
and shortness of breath. His vital signs are: 99.5, 102, 24, 185/100, 92% on RA. He has bibasilar
crackles and wheezes as well as 2+ lower extremity pitting edema. You order a CBC, basic
metabolic panel, chest x-ray, EKG, and troponin. Which of the following additional tests will be
helpful in determining the most likely diagnosis?
A. Lactate
B. Blood culture
E. Prothrombin time
Answer D. Brain natriuretic peptide (BNP) aids in the diagnosis of CHF especially in cases where
the signs and symptoms of COPD can mimic those of an acute heart failure exacerbation. If
elevated, the BNP level points to CHF as the likely etiology. There are important false positives
to an elevated BNP level, including sepsis and pulmonary embolism. Importantly, obesity can
falsely depress BNP levels. Lactate is an excellent indicator of prognosis and mortality, but is not
specific enough to point to a certain diagnosis. The yield on blood cultures in general is
extremely low, especially in cases where infection is not the leading diagnosis. An arterial blood
gas is not generally useful in the acute setting except to evaluate for hypercarbia, as hypoxia can
be estimated using pulse oximetry and acidosis can be estimated with a venous blood gas if
needed. The prothrombin time is not useful in the absence of suspected coagulopathy or known
anticoagulation use.
76. Which of the following is the strongest risk factor for developing an abdominal aortic
aneurysm (AAA)?
A. Hypertension
B. Obesity
C. Male gender
D. Hypertriglyceridemia
Answer E. Family history in older patients with abdominal or flank pain is an extremely strong
risk factor—an affected first-degree relative puts an individual at 20 times higher the risk than
pg. 53
the general population. Other risk factors include age older than 50, peripheral vascular disease,
hypertension, and patients with other large artery aneurysms.
77. A 65-year-old male with a left ventricular assist device presents with fatigue, dizziness, and
lightheadedness. His device has alarmed several times. You have a page out to the patient’s
cardiologist. Which of the following tests can you anticipate the cardiologist will recommend?
B. CT coronary angiography
C. Echocardiogram
D. Cardiac MRI
E. Ventilation–perfusion scan
Answer C. In patients with left ventricular assist devices (LVADs), echocardiogram is extremely
helpful for routine outpatient monitoring of ventricular function as well as evaluating for
symptoms suggesting malfunction, including fatigue, lightheadedness, and dizziness. Although
not all LVAD patients who present to the ED should receive an echocardiogram, the symptoms
in the case do warrant this. CT of the chest or coronary vessels is not indicated routinely in this
case. Cardiac MRI is contraindicated in patients with LVADs. A ventilation–perfusion scan is used
to evaluate pulmonary embolism, and is not routinely ordered in patients without explicit chest
pain, shortness of breath, or other symptoms of venous thromboembolic disease.
78. Among patients presenting to the ED with chest pain, which of the following physical
examination findings is the biggest risk factor for aortic dissection?
Answer C. Hypertension is the most common risk factor for aortic dissection though it is present
in <75% of patients with diagnosed aortic dissection. Young patients with aortic dissection are
much less likely to have hypertension (only approximately one-third of patients under 40 have
hypertension). Marfan syndrome is a connective tissue disorder characterized by several
skeletal, cardiovascular, and ocular manifestations. Arachnodactyly (long, slender fingers) is one
pg. 54
of the major diagnostic criteria and one of the most easily recognized manifestations of the
disease. Other major manifestations include a short trunk with long legs, pectus carinatum or
excavatum, reduced elbow extension, and significant scoliosis. While an upper extremity blood
pressure disparity is a classically described finding in patients with aortic dissection, it is
unreliable and should be at least 20 mm Hg. A pulse deficit between the upper extremities is a
more reliable sign of underlying dissection but is present in only 15% of patients. The classic
heart murmur in aortic dissection is the murmur of aortic regurgitation, which is present in up to
one-third of patients and more than 40% of patients with type A dissections. Obesity is not a risk
factor for aortic dissection. Power lifting, or very intense weightlifting, can cause transient
elevations of intra-aortic pressure to more than 300 mm Hg and is thought to be a risk factor for
dissection. Other risk factors include cocaine use, third-trimester pregnancy, and other
connective tissue disorders such as that found in Turner syndrome or Ehlers– Danlos syndrome.
79. A 69-year-old female with a history of end-stage renal disease on hemodialysis 3 days per
week presents to the ED with severe dyspnea. The patient missed her last dialysis session due to
a transportation problem and subsequently became short of breath. On the day of presentation,
she felt progressively worse, such that she was breathing “hard and heavy.” In the ED, her vitals
are T 97.1°F, P 110, BP 172/102, RR 28, SaO2 84% on RA. Her lung examination reveals diffuse
rales, with diminished breath sounds. Though she is on dialysis, she continues to make small
volumes of urine. Which of the following is the best initial treatment?
E. Intravenous enalaprilat
Answer B. This patient is presenting with symptoms of acute decompensated heart failure
(ADHF) due to severe volume overload as a result of missing dialysis. The approach to such
patients mirrors the approach to patients with ADHF without concomitant renal failure. Since
this patient presents with clear respiratory distress, stabilization of her oxygenation and
ventilatory status is the chief concern. While furosemide, nitroglycerin, and CPAP may also be
helpful in this setting, noninvasive ventilation with bilevel positive airway pressure will most
rapidly and completely address her respiratory distress. CPAP is functionally similar to PEEP, as it
is not a ventilatory mode, and patients must initiate all breaths. While CPAP has demonstrated
efficacy in patients with ADHF, bilevel positive airway pressure allows clinicians to specify a
pressure gradient between inspiration (inspiratory positive airway pressure) and expiration
(expiratory positive airway pressure) which allows for true ventilation. Intravenous enalaprilat is
not recommended for the treatment of ADHF, since its effects on blood pressure are
unpredictable and its use may be associated with worsened outcomes in patients with ADHF in
the setting of acute myocardial infarction.
pg. 55
80. Patients stricken with viral-induced myocarditis will most likely present with symptoms and
findings consistent with:
D. Pericarditis
E. Cardiogenic syncope
Answer C. Viral and postviral myocarditis are the most common causes of myocarditis. The most
common manifestation of acute and chronic myocarditis is dilated cardiomyopathy with a
decreased ejection fraction leading to symptoms of CHF. Patients with acute lymphocytic
myocarditis can occasionally present with findings that are confused with an acute coronary
syndrome. However, the coronary arteries are normal upon subsequent catheterization. While
CHF is the most common presentation, patients can present with variable degrees of heart
block, pericarditis, syncope, malignant ventricular arrhythmias, or sudden cardiac death.
81. Which of the following is the most common valve affected in rheumatic heart disease?
A. Tricuspid
B. Pulmonic
C. Mitral
D. Aortic
Answer C. The mitral valve is affected about two-third of the time in rheumatic heart disease,
followed by the aortic valve in a quarter of cases and then the tricuspid and pulmonic valves.
Rheumatic heart disease is the most common cause of mitral regurgitation in non-Western
countries (in Western countries, mitral valve prolapse is the most common cause of mitral
regurgitation). The murmur of mitral regurgitation is best heard at the apex of the heart and
lasts throughout the systolic phase.
pg. 56
82. A 24-year-old male presents with non-exertional, atypical chest pain. His EKG is shown (Fig.
below). Which of the following is true?
Answer C. Benign early repolarization (BER) is a common pattern of EKG changes that are not
thought to reflect any underlying cardiac disease but which can be confused with cardiac
ischemia (see Fig. ). Almost 90% of healthy young men have ST-segment elevation of 1 to 3 mm
in at least one precordial lead. BER is more common among young patients, as well as among
men, and African Americans. The pattern of changes seen in BER include ST elevation which is
most prominent in the anterior to early lateral precordial leads, i.e., V2–V5, with concave
upward morphology of the ST segment (sometimes referred to as the “smiley-face”
morphology) and with the amplitude of the ST elevation ≤2 mm in the precordial leads and ≤0.5
mm in the limb leads, symmetric, concordant T waves, notching or slurring of the J point or
terminal end of the QRS, and relative stability over time. Identification of the J point is best
achieved by looking for the first change in direction as the QRS complex shifts into the ST
segment. Isolated ST elevation in the limb leads is rarely benign and should be considered
ischemic until proven otherwise.
pg. 57
83. A 22-year-old male presents with an episode of left-sided chest pain that lasted 4 hours, but
resolved an hour before presenting to the ED. The pain was dull and radiated to his left
shoulder. The patient states that he used cocaine at a party 12 hours ago. Physical examination
is unremarkable and vital signs are normal. The electrocardiogram (EKG) is shown in Figure
below. Which of the following is most appropriate at this time?
A. Nitroglycerin
B. Metoprolol
C. Morphine
D. Aspirin
Answer D. The EKG demonstrates no specific findings consistent with acute myocardial ischemia
or infarction. Cocaine-induced myocardial ischemia is due to both vasospasm and
hyperaggregatory platelets, which may cause acute thrombosis. Aspirin is indicated in patients
with cocaine-induced chest pain until it is known for certain that MI or ischemia is not present.
In a patient without current symptoms of chest pain, nitroglycerin and morphine are not
indicated. Metoprolol is relatively contraindicated in cocaine-induced chest pain as there is a
theoretic risk of reducing cardiac output in the face of increased peripheral vascular resistance.
tPA is not emergently indicated in a patient with chest pain without ST-elevation myocardial
infarction (STEMI).
pg. 58
84. Which of the following can be used to treat hypertrophic cardiomyopathy?
A. Digitalis
B. Isoproterenol
C. Furosemide
D. Phenylephrine
E. Metoprolol
85. A 25-year-old male presents with chest pain. The pain lasts for seconds at a time, is not
pleuritic, and is non-exertional. He has no past medical history, takes no medications, is a
nonsmoker, and does not drink alcohol or use drugs. His vital signs are normal and his physical
examination is unremarkable. His EKG reveals a normal sinus rhythm without ST-segment
changes. His chest x-ray is normal. Which of the following is the next best step in management?
Answer A. Given his lack of risk factors or EKG abnormalities, the patient is at extremely low risk
for acute coronary syndrome. He is far more likely to have a gastrointestinal, pulmonary, or
idiopathic etiology for his chest pain. It is reasonable to discharge this patient home and have
him follow up with his primary care physician at this time. Giving this patient full-dose aspirin
therapy for presumed acute coronary syndrome would be more likely to result in aspirin-related
side effects than any potential benefit from thrombus prevention or treatment. Enoxaparin
would be reasonable in patient where pulmonary embolism was a likely diagnosis—in this case,
there is nothing from the history or physical examination to suggest pulmonary embolism.
pg. 59
Admission to the hospital for this extremely low risk patient is more likely to result in
nosocomial complications than any benefit to the patient’s health. IV tPA poses a far greater risk
of hemorrhage than any benefit to this patient who lacks any diagnostic criteria for myocardial
infarction or pulmonary embolism.
86. A 55-year-old male without any past medical history presents with chest pain. Physical
examination is normal. The EKG is shown in Figure below. Which of the following represents the
likely site of pathology?
A. Pericardium
B. Pulmonary artery
Answer D. The EKG demonstrates deeply inverted T waves in the anterior leads, which is specific
for the LAD distribution. When associated with unstable angina, these T waves indicate stenosis
of the proximal LAD, called the Wellens syndrome. Electrocardiographic findings of pericarditis
include concave ST elevation, sinus tachycardia, and PR depression. Sinus tachycardia and right
axis deviation are common EKG findings in patients with pulmonary artery embolism. Lateral (I,
L, V5, V6) and posterior (V1, V2) abnormalities are often seen in left circumflex lesions.
Right-sided coronary artery disease (CAD) may cause an inferior MI (II, III, aVF) or posterior MI
(V1, V2).
pg. 60
87. A 56-year-old female is sent immediately to the ED by her primary care provider for a blood
pressure of 200/100. She is asymptomatic and has not taken her hydrochlorothiazide and
lisinopril for 6 months. Which of the following is the next best step in management?
A. Nitroglycerin 0.4 mg SL
B. Labetalol 20 mg IV
C. Hydralazine 20 mg IV
Answer E. The patient has asymptomatic hypertension, which should not immediately be
treated with intravenous antihypertensives. Instead, a period of observation followed by a blood
pressure recheck should be pursued. If blood pressure is still elevated at that time, it is
reasonable to restart her on her oral medications while avoiding complete normalization of her
blood pressure in the ED. Given her chronic hypertension, her cerebral autoregulation zone for
optimal cerebral blood flow is likely higher than in a normotensive individual, and rapidly
reducing her blood pressure to 120/70 could cause severe hypoperfusion of the brain.
Sublingual nitroglycerin is not an appropriate therapy for asymptomatic hypertension and
should be reserved for reduction in preload in acute heart failure exacerbation and pain control
in acute myocardial infarction.
88. An 81-year-old female with no reported past medical history except hypertension presents
with acute onset of shortness of breath. Her vital signs are: 98.6, 105, 24, 195/92, and 94% on 4
L NC. She has mild respiratory distress and significant wheezing on bilateral lung examination
along with peripheral edema. Which of the following is the next best step in management?
B. Methylprednisolone IV
C. Azithromycin 500 mg IV
D. Labetalol 20 mg IV
Answer E. Without a prior history of obstructive lung disease, this patient with wheezing likely
has acute heart failure, due possibly to acute coronary syndrome. A diagnostic evaluation should
proceed and direct further management, such as with venodilators and diuretics. There is often
a temptation to treat all wheezing with albuterol nebulizer therapy, but in patients with
cardiogenic causes of wheezing, this can worsen the outcome by increasing heart rate and thus,
pg. 61
cardiac workload. Albuterol and methylprednisolone would be reasonable with a past medical
history of COPD or asthma. Azithromycin would be reasonable if COPD exacerbation were
suspected—as monotherapy, it would not be appropriate for community-acquired pneumonia
in this age-group. Labetalol IV should be avoided in the setting of acute heart failure as it may
further reduce cardiac output.
89. A 74-year-old male with hypertension and chronic kidney disease presented to the ED for
evaluation after a fall. He reports some recent difficulty walking due to “dizziness.” Vitals are P
48, RR 20, BP 105/80, SaO2 97% on room air. His EKG is shown. Blood tests reveal a hemoglobin
of 9.2 g/dL, BUN 30 mg/dL, creatinine 1.6 mg/dL, sodium 141 mEq/L, and potassium of 5.1
mEq/L. Which of the following is the best next step (Fig. below)?
A. Atropine 1 mg IV
E. Kayexalate 15 g PO
Answer B. The patient’s EKG reveals type I second-degree atrioventricular heart block (Mobitz
type I or Wenckebach block). Patients with this type of block experience progressive
prolongation of the PR interval before a nonconducted P wave (a P wave that is not followed by
a QRS complex). This is usually an asymptomatic, very stable rhythm in young, athletic patients
with high vagal tone. However, it may be symptomatic in elderly patients with underlying heart
disease who have trouble tolerating the bradycardia that frequently accompanies the rhythm. In
such patients, a pacemaker is usually required. This patient suffered a fall, likely as a result of
bradycardia and the concomitant relative cerebral hypoperfusion due to type I second-degree
atrioventricular heart block. There doesn’t appear to be an obvious, reversible cause of his heart
block. Given that he has stable vital signs, there is no indication for emergent pacing, or
atropine. Amiodarone is not helpful. Kayexalate is not needed to treat the patient’s minimal
hyperkalemia.
pg. 62
90. Which of the following is more characteristic of endocarditis in IV drug users than in nondrug
users?
C. Splinter hemorrhages
E. Roth spots
Answer D. Endocarditis in intravenous drug users (IVDUs) is different from that in nondrug users
in several ways: Patients are generally younger, almost universally have fever, and have more
pulmonary symptoms due to right-heart vegetations causing septic emboli to the lungs.
Although S. aureus is a far more common cause of endocarditis in IVDUs, it has less mortality
among this patient population. Heart murmurs are less commonly heard in endocarditis in
IVDUs because the tricuspid valve, the most common valve affected, is difficult to hear on
physical examination. Splinter hemorrhages and Roth spots are, if anything, less common in
right-heart endocarditis as the lungs will filter most septic emboli.
91. A 33-year-old male presents by EMS with a chief complaint of palpitations and mild chest
pain. Upon EMS arrival, they found the patient awake and normotensive, appearing stable and
in a wide-complex tachycardia with a rate of 200. They administered adenosine and the patient
converted to a normal sinus rhythm. Which of the following is true?
Answer D. Approximately 10% of patients with ventricular tachycardia (VT) are responsive to
adenosine. Fortunately, adenosine-sensitive VT tends to be more stable than other forms of VT
and less commonly leads to sudden cardiac death. Still, it is important to recognize that an
undifferentiated wide-complex tachycardia should be treated as VT rather than supraventricular
tachycardia (SVT) with aberrancy. The patient’s stability does not increase the likelihood of SVT
or decrease the likelihood of VT.
pg. 63
92. A patient presents with the electrocardiogram (EKG) shown in Figure below. Which of the
following is the most likely pathophysiologic mechanism?
A. Reentry
B. Increased automaticity
D. Pre-excitation
E. Infarction
pg. 64
contraindicated in this circumstance, as inhibition of the AV node will cause faster conduction
through the anterograde accessory pathway, which is already at high risk for degeneration into
an unstable rhythm. The treatment of choice in stable antidromic or irregular tachycardias in
patients with WPW is amiodarone or procainamide. Unstable patients require cardioversion.
93. Which of the following arteries supplies the atrioventricular (AV) node in most people?
Answer B. The right coronary artery supplies the AV node in >80% of patients, and the left
circumflex artery supplies the AV node in the rest.
94. A 58-year-old male with a history of hypertension and diabetes presents by EMS with chest
pain that developed while he was cutting grass. The patient was normotensive upon EMS arrival.
EMS administered two sublingual nitroglycerin tablets en route. Upon arrival, his initial vital
signs are P 75, RR 18, BP 92/64, SaO2 98% on room air. His EKG is shown (Fig. below). Which of
the following is true?
pg. 65
Answer A. This patient has an inferior ST elevation MI with reciprocal ST depressions anteriorly.
While the patient likely has concomitant inferior involvement of the left ventricular wall, the
patient’s hypotension after nitroglycerin strongly suggests right ventricular involvement. A
right-sided EKG demonstrating ST elevation in V4R is considered diagnostic (>90% sensitivity and
specificity). Agents that reduce preload such as nitrates, opioids, or beta-blockers should be
avoided or used with caution in patients with right ventricular involvement or signs of right
ventricular failure (elevated jugular venous pressure, peripheral edema, hepatomegaly, clear
lung examination). Instead, such patients should receive small boluses of IV fluid to optimize
preload. In patients who remain hypotensive despite preload, dopamine may be a helpful
adjunct.
Answer E. Labetalol antagonizes α-1, β-1, and β-2 adrenergic receptors. The β-blockade is much
greater than the α-blockade, although both function to provide rapid, predictable BP control
without causing reflex tachycardia. This blunting of the tachycardic response along with
pg. 66
concomitant arteriolar vasodilation makes labetalol ideal as the initial single agent for
management of aortic dissection, where reduction of heart rate is as important as reduction of
BP. Orthostatic hypotension with oral labetalol is rare, but IV formulations can cause major
changes. Nitroprusside and labetalol are roughly equal in their ability to control BP rapidly and
predictably.
96. Which of the following is the most common initial dysrhythmia in symptomatic patients with
Wolff– Parkinson–White (WPW) syndrome?
E. Torsade de pointes
97. Which of the following is the number one cause of death in patients with congestive heart
failure (CHF)?
C. Pneumonia
D. Stroke
E. Pulmonary embolism
pg. 67
Answer A. Patients with CHF most commonly die due to hemodynamic decline, followed by
dysrhythmia. Mortality from fluid overload has been reduced by β-blockers and angiotensin
converting enzyme inhibitors, and short-term mortality from dysrhythmias has been reduced by
automated internal cardiac defibrillators (AICDs). Choices B, C, D, and E are all important, but
are less common causes of death in patients with CHF.
98. Which of the following is the earliest electrocardiogram (EKG) finding in acute myocardial
infarction (MI)?
A. Hyperacute T waves
B. ST elevation
C. ST depression
D. T-wave inversion
E. Q waves
99. A 20-year-old male presents to the ED with several days of progressive chest pain, fatigue,
myalgias, and exertional dyspnea. He states that he had the “flu” 1 week before. He denies illicit
drug use or family history of heart disease. Physical examination reveals a temperature of
100.5°F, heart rate of 125, no murmurs on cardiac examination, and scattered bilateral crackles
on lung examination. EKG demonstrates sinus tachycardia, chest x-ray reveals cardiomegaly and
mild pulmonary edema, and laboratory reports are normal except for troponin I, which is
elevated at 10 ng/mL. Which of the following is the most likely etiology?
A. MI
B. Aortic dissection
C. Viral infection
D. Stroke
pg. 68
Answer C. The patient most likely has myocarditis, which in the United States is usually because
of viruses, most commonly Coxsackie B. A viral prodrome usually precedes overt signs of cardiac
involvement, such as chest pain or signs of heart failure. No common ED laboratory or imaging
study is helpful in making the diagnosis of myocarditis—antimyosin scintigraphy or more
invasive endomyocardial biopsy is indicated. Coronary artery disease is uncommon in patients of
this age in the absence of risk factors. Aortic dissection is uncommon in the absence of trauma
or history of hypertension or Marfan disease. Stroke is not suggested by the symptoms, and
diabetic ketoacidosis (DKA) is unlikely with normal glucose and bicarbonate.
100. A 63-year-old female with a history of congestive heart failure (CHF) presents for
evaluation of shortness of breath. Her vital signs are P 115, BP 190/100, RR 28, SaO2 82% RA.
Physical examination reveals diffuse rales. Which of the following is true?
A. Patients in CHF presenting with severe hypertension have a higher mortality rate than
normotensive or hypotensive patients
C. The patient should undergo rapid sequence intubation and mechanical ventilation
Answer D. Patients with acute decompensated CHF (ADHF) and severe hypertension benefit
from afterload and preload reduction and intravenous nitroglycerin (NTG) achieves this goal.
Sublingual nitroglycerin is rapidly absorbed and provides a dose equivalent to more than 50
mcg/minute for a brief period. Thus, it should be used while an IV infusion is being prepared. In
addition, oxygen combined with noninvasive positive pressure ventilation with BiPAP or
continuous positive airway pressure (CPAP) helps recruit alveoli (from positive end expiratory
pressure) and is a critical component of initial therapy. While many patients ultimately need
diuresis, intravenous loop diuretics such as furosemide offer limited benefit in the hyperacute
setting of ADHF. In particular, furosemide induces some degree of arteriolar constriction
resulting in increased systemic vascular resistance and mean arterial pressure which undermines
the goals of treatment. Finally, as shown by Gheorghiade M et al., there is negative correlation
between hypertension and mortality in the setting of ADHF. This may be because patients with
severe long-standing hypertension typically have diastolic heart failure which is easy to address
in an acute exacerbation while normotensive or hypotensive patients have systolic heart failure.
101. A 35-year-old marathon runner who is asymptomatic presents from an outside hospital
with an “abnormal EKG” (Fig. below). Your advice to the patient is:
pg. 69
B. Atropine 1 mg IV
C. Amiodarone 300 mg IV
D. Adenosine 6 mg IV
Answer E. The EKG demonstrates Wenckebach phenomenon or Mobitz type I AV block. The PR
interval progressively increases in length and predictably drops a QRS beat after the second
prolongation. In the absence of symptoms of other serious cardiac disease, management
involves outpatient follow-up. Pacemaker placement would be required for patients with Mobitz
type II second-degree or third-degree AV block. Atropine is used to treat hemodynamically
unstable brady dysrhythmias. Amiodarone may be used to treat a variety of tachydysrhythmias
but is not indicated for patients with AV blocks. Adenosine is used to abort supraventricular
tachycardias.
102. Which of the following is the most common symptom in patients diagnosed with an
abdominal aortic aneurysm (AAA)?
A. Nausea
B. Abdominal distention
C. Constipation
pg. 70
D. Urinary retention
E. Asymptomatic
Answer E. Aneurysms are most often asymptomatic and diagnosed incidentally on routine
examination or imaging studies. Most symptomatic patients complain of vague abdominal pain
or abdominal distension, although urinary retention and constipation may occur. Nausea occurs
only in the setting of other symptoms pointing to the diagnosis. Acute abdominal pain or
distension in patients at risk for AAA indicates rapid expansion or rupture.
A. Chest pain
B. Fever
D. S4 heart sound
E. Leukocytosis
pg. 71
pg. 72
Pulmonary and thorax
emergency
➢ Chapter includes:
COPD
Asthma
Superior vena cava syndrome
Thoracic outlet syndrome
Pulmonary embolism
Spontaneous pneumothorax and tube insertion
Plural effusion (non -cardiac origin)
Respiratory distress syndrome
Multiple MCQ’s pulmonary and thorax
pg. 73
1. Which of the following is the most common cause of acute respiratory distress syndrome
(ARDS)?
A. Sepsis
B. Near drowning
E. Pancreatitis
Answer A. Sepsis is the most common risk factor or condition, leading to the development of
ARDS. ARDS is defined as the development of acute respiratory failure, with noncardiogenic
pulmonary edema (established by the presence of bilateral infiltrates consistent with pulmonary
edema on chest radiography and a pulmonary capillary wedge pressure ≤18 mm Hg indicating
that the edema is noncardiogenic), and severe hypoxia such that the PaO2:FiO2 ratio ≤200.
Identical findings in patients with a PaO2:FiO2 ratio ≤300 are diagnosed with acute lung injury
(ALI), instead of ARDS, but the etiology of ALI is the same. Although sepsis is the most common
cause of ARDS, severe trauma complicated by shock, multiple blood transfusions, and aspiration
are all independent risk factors. Pancreatitis and near drowning may result in ARDS but are less
common causes. Such causes also illustrate the idea that the ARDS may be due to direct lung
injury (e.g., severe pneumonia with sepsis) or indirect lung injury (e.g., severe pancreatitis). The
latter is presumably due to the widespread release of cytokines and other proinflammatory
mediators.
2. A 57-year-old female with a history of hypertension presents for evaluation of a 1-day history
of pleuritic right-sided chest pain. She has no leg pain or swelling. She is afebrile with a P 82, RR
14, SaO2 97% on room air. Which of the following is true?
A. The pulmonary embolism (PE) rule-out criteria ("PERC rule") can be used to exclude
pulmonary embolism
E. Negative Doppler venous imaging of the patient’s legs excludes a pulmonary embolism
Answer B. D-dimer values increase naturally with age, and a recent review revealed that using
an age-adjusted d-dimer cutoff value determined by multiplying the patient’s age in years × 10
pg. 74
mcg/L reduces false positives without substantially increasing false negatives. As with the
normal cutoff value, the patient should meet low-risk criteria prior to d-dimer testing. The PE
rule-out criteria (“PERC” rule) applies only to patients <50 years old. A CT pulmonary angiogram
would not likely be needed in patients with negative d-dimer tests. Patients who are low-risk by
Wells criteria have <2% risk of venous thromboembolism. Thus, it may be reasonable to avoid
further testing if an alternative diagnosis is made. However, given the morbidity and mortality
associated with undiagnosed and untreated pulmonary embolism, further testing should be
pursued in the absence of an alternative explanation for the patient’s symptoms. Negative lower
extremity Doppler imaging does not exclude a PE.
3. Which of the following is true regarding the role of ipratropium in asthma management?
C. Ipratropium is most useful as an adjunct for patients with severe asthma exacerbations.
D. Ipratropium has never been proven to be of benefit in patients with acute asthma
exacerbations.
4. A 26-year-old female presents with dyspnea and pleuritic chest pain and is subsequently
diagnosed with a PE. She is not pregnant, takes no oral contraceptive therapy, and is a
nonsmoker, but she notes that her mother has had two PEs. Which of the following is the most
likely cause of this patient’s PE?
pg. 75
A. Plasminogen deficiency
B. Nephrotic syndrome
C. Cervical cancer
D. Factor V Leiden
E. Protein S deficiency
Answer D. There are numerous risk factors for PE, which can be divided into inherited and
acquired disorders. Inherited hypercoagulable disorders should be suspected in patients with a
documented PE who are younger than 40 years old, or in similar patients who have a positive
family history or who have recurrent PEs. Of the inherited disorders, factor V Leiden is the most
common, and may be present in as many as 20% of patients with venous thromboembolic
disease. In normal individuals, factor V is normally inactivated (along with factor VIII) by protein
C, thereby disrupting the normal clotting cascade. Patients with factor V Leiden have a factor V
protein that is resistant to inactivation by protein C, which allows the coagulation cascade to
continue in an unregulated manner, resulting in a thrombophilic state. Plasminogen deficiency
and protein S deficiency are also inherited disorders that increase the risk of thromboembolic
disease, but they are far less common. Nephrotic syndrome is also associated with an increased
risk of PE due to a relative deficiency of coagulation cascade regulatory proteins, which are lost
in the urine. Cancer in any form is a well-described risk factor for PE.
5. Which of the following is the most frequently affected structure in thoracic outlet syndrome?
A. Subclavian artery
B. Subclavian vein
C. Ulnar nerve
D. Radial nerve
E. Median nerve
Answer C. The thoracic outlet syndrome comprises a group of pathologic conditions associated
with compression of the structures at the junction of the upper extremity and trunk. The
findings are neurologic (95%), venous (4%), and arterial (1%). The most commonly affected
structure is the ulnar nerve.
6. Thirty minutes after a 35-year-old female presents to the ED with a severe asthma
exacerbation, you intubate her because she is showing signs of fatigue and ventilatory failure.
You use ketamine and succinylcholine, and pass the endotracheal through the vocal cords
pg. 76
without difficulty. A colleague who is assisting you aggressively “bags” the patient until the
respiratory therapist connects the mechanical ventilator. Thirty seconds after intubation, the
nurse reports that the patient’s blood pressure (BP) has dropped to 93/46. Her BP before
intubation was 138/80. The patient has an 18- gauge peripheral intravenous (IV) line in her left
antecubital fossa and her trachea appears midline. What is the best course of action?
A. Ask the nurses to place a second large-bore peripheral IV and immediately bolus the patient
with 2 L of normal saline.
C. Disconnect the ventilator but keep the endotracheal tube (ETT) in place and allow the patient
to exhale.
7. The most widely used critical care ventilatory strategy in acute asthmatic patients aims to
accomplish which of the following objectives?
A. Patients are purposefully hypoventilated, maintaining elevated PaCO2 values, to keep their
airway pressures at safe levels to avoid barotrauma.
B. Patients are purposefully hyperventilated to bring their PaCO2 levels back to normal because
ventilatory failure is the primary reason for intubating patients in status asthmaticus.
pg. 77
C. The initial ventilator settings are no different than for a patient intubated for airway
protection due to altered mental status.
D. Inspiratory flow rates are set very low to avoid causing very high peak airway pressures due
to bronchoconstriction.
E. The inspiratory flow curve should be a ramp-style wave instead of a square-style wave to
maximize expiratory time.
Answer A. Due to their significant airflow obstruction, mechanically ventilated asthmatics are at
risk for lung hyperinflation and concomitantly elevated airway pressures. To avoid these
problems, the most commonly used ventilatory strategy is “permissive hypercapnia.” In this
strategy, the patient is ventilated at settings that ensure adequate time for exhalation, which
limits air trapping and subsequent auto-PEEP and lung hyperinflation (with elevated plateau
pressures). Expiratory time can be maximized by limiting the time spent during inspiration (by
setting high inspiratory flow rates and by using a square-wave form), as well as by decreasing
minute ventilation (by decreasing either VT or RR). The byproduct of these changes is
hypercapnia. Previous approaches aimed to normalize alveolar CO2 by using higher respiratory
rates and tidal volumes but resulted in increased morbidity and mortality from airway
barotrauma. Peak pressure reflects the pressure applied to the large- and medium-sized airways
as air is pushed into the lungs by the ventilator. It strongly reflects airway resistance and tends
to be very high in asthmatics due to their significant airway obstruction. Plateau pressure
reflects the pressure applied to the small airways and alveoli after the air settles in the lungs. It
is extremely important to monitor plateau pressure and ensure that it remains <35 cm H2O to
avoid alveolar overdistension and barotrauma. When plateau pressure is normal and intrinsic
PEEP is <15 cm H2O, peak pressure elevations are immaterial. Therefore, while high inspiratory
flow rates elevate the peak pressure, it is unnecessary to reduce flow rates to decrease peak
pressure. In fact, it is most often necessary to increase the inspiratory flow rate in order to
decrease inspiratory time, which also results in increased peak pressure. Peak pressure is
determined by the rate of airflow, not the absolute volume of air nor the respiratory rate.
B. Patients with anaerobic lung abscesses typically present with acute-onset chest pain, cough,
and fever.
C. Lung abscess occurs most commonly in patients with poor oral hygiene.
pg. 78
Answer C. The development of a lung abscess is most commonly a consequence of aspiration in
patients with poor oral hygiene. Less commonly, it may occur as a result of necrotizing
pneumonia. Classically it has been thought that anaerobic bacteria are responsible for the great
majority of lung abscesses. However, recent studies have revealed that the microbiology may
differ between immunocompetent and immunocompromised patients. Although anaerobes are
predominant in immunocompetent patients, patients with depressed immune systems are more
frequently infected with aerobic bacteria such as S. aureus, Pseudomonas aeruginosa, Klebsiella
pneumoniae, and H. influenzae. Frequently, however, infections are polymicrobial. Nonbacterial
organisms such as fungi and parasites may also cause lung abscesses. S. pneumococcus is not a
common cause. Patients with anaerobic or fungal lung abscesses typically experience an
indolent course of fever, productive cough, night sweats, anorexia, and weight loss. The sputum
is classically malodorous, which should be a clue to the diagnosis. Immunocompromised
patients presenting with a lung abscess as a result of aerobic necrotizing pneumonia present
with acute symptoms of pneumonia and may be quite ill. Chest radiography typically reveals a
cavitary lesion with an air–fluid level. Antibiotics remain the cornerstone of therapy although
surgical management may ultimately be necessary. Typical surgical indications are medical
treatment failure, suspected cancer, or congenital lung malformation. Lung abscesses very
rarely develop in the setting of pediatric community acquired pneumonia and only occur
typically in patients who are immunocompromised because of acquired immune deficiencies,
malignancy or chemotherapy, and in patients who are predisposed to aspiration (children with
neurologic disorders, decreased mental status, impaired cough, or swallowing dysfunction).
A. Most cases of primary pulmonary infection with Histoplasma capsulatum are undetected and
resolve without treatment.
Answer A. H. capsulatum is the most common pulmonary fungal infection worldwide. It may
infect immunocompetent as well as immunocompromised individuals, but primary infection is
almost always asymptomatic and very few cases are ever brought to the attention of a
physician. In endemic regions, at least 80% to 90% of the population has positive skin testing by
the age of 20. Histoplasma and Blastomyces are endemic to the Mississippi and Ohio River
valleys in the United States, whereas Coccidioides is found in the arid southwest. In contrast,
Cryptococcus is ubiquitous throughout the world, without a specific distribution. It generally
causes disease in immunocompromised hosts, and is the most common cause of life-threatening
pg. 79
fungal infection in patients with human immunodeficiency virus (HIV). Except in a few rare
cases, patients with fungal pulmonary infection are not capable of transmitting disease to
others. Of the fungi that cause systemic mycoses, Blastomyces most commonly causes
disseminated disease.
10. A 44-year-old alcoholic man presents with shortness of breath, fever, and productive cough.
Chest xray demonstrates a left lower lobe infiltrate. The diagnosis of pneumonia is made. Which
of the following is the most likely cause?
A. Staphylococcus aureus
B. Streptococcus pneumoniae
C. Klebsiella pneumoniae
D. Mycobacterium tuberculosis
E. Mycoplasma pneumoniae
Answer B. Although the incidence of K. pneumonia is higher in alcoholics than nonalcoholics, the
overall most common cause of CAP in the alcoholic patient is still pneumococcus. Alcoholic
patients tend to have a higher incidence of aspiration pneumonia and tuberculous disease
compared with the general population. Alcohol itself is immunosuppressive and predisposes
patients to a higher incidence of bacterial infections. Treatment of CAP in the alcoholic patient is
similar to that of the general population—third-generation cephalosporin plus a macrolide or a
fluoroquinolone. Anaerobic coverage may be added for patients who are at particular risk for
aspiration pneumonia.
11. A 23-year-old female with a history of sickle cell disease presents with fever, chills, cough,
and dyspnea. A chest x-ray demonstrates a focal infiltrate in the right lower lobe. Which of the
following is the most appropriate management at this time?
Answer E. The patient has acute chest syndrome, as indicated by the presence of fever, cough,
dyspnea, and new infiltrate on chest x-ray in the setting of sickle cell disease. The diagnosis is
pg. 80
based on the presence of a new infiltrate plus at least one of the following: chest pain, a
temperature >38.5ºC, or respiratory distress manifest by increased work of breathing or cough,
wheezing, tachypnea, or hypoxemia (or a combination). It is a very common cause of death in
patients with sickle cell disease. The most common cause of acute chest syndrome is infection
(pneumonia), while pulmonary infarction and fat embolism are the next most common causes.
Overall, however, the etiology cannot be determined in more than one-third of cases.
Treatment involves fluids, analgesics, oxygen, and antibiotics. Heparin is used to manage
pulmonary thromboembolism but is not indicated in patients with undifferentiated acute chest
syndrome. It may be indicated in patients with acute chest syndrome due to pulmonary
thromboembolism, but this is evaluated on a case-by-case basis in close consultation with a
hematologist. Tissue plasminogen activator may be indicated for acute stroke, MI, or severe
pulmonary thromboembolism. Angioplasty is indicated for acute STEMI but has no role in the
management of acute chest syndrome. Albuterol and prednisone are used to treat
bronchospastic lung disease, which is not part of the pathophysiologic process of acute chest
syndrome though reactive airway disease may also be present since it is such a common entity.
12. Which of the following is the most common cardiac rhythm in patients with pulmonary
embolism (PE)?
B. Atrial fibrillation
C. Atrial flutter
D. Ventricular tachycardia
E. Sinus tachycardia
Answer A. Although sinus tachycardia is the most common arrhythmia in patients with PE,
normal sinus rhythm remains the most common cardiac rhythm. A recent study demonstrated
that roughly two-third of patients with PE have normal sinus rhythm (there was no significant
difference from control patients). That doesn’t suggest, however, that most patients with PE
have normal EKGs, as rhythm is only one aspect of EKG analysis. In fact, most patients with PEs
have been found to have abnormal EKGs, but no single abnormality has been shown to have
sufficient sensitivity or specificity to aid in the diagnosis. Therefore, the role of EKG in the
evaluation of PE is to rule out the presence of alternative diagnoses, such as cardiac ischemia or
pericarditis.
13. You are consulting the thoracic surgery service after having recently placed a chest tube in a
22-yearold male for a 25% spontaneous pneumothorax. The consultant asks you whether there
is still an “air leak” present. Assuming the patient’s lung re-expanded appropriately, what is the
most likely significance of an air leak?
pg. 81
A. All patients will have an air leak after chest tube insertion.
B. The chest tube was inserted into a branch of the patient’s tracheobronchial tree.
C. The suction holes on the chest tube are not completely inserted into the chest cavity.
D. The water seal chamber does not have enough water in it.
Answer E. Tube thoracostomy is the most common procedure performed in the setting of
thoracic trauma. It is also the primary means by which a primary or secondary spontaneous
pneumothorax is treated (a small pneumothorax may be observed without intervention). The
presence of a pneumothorax must have been caused by a defect in the patient’s bronchial tree
resulting in communication between the bronchioles and the pleural space. The chest tube is
placed within the pleural space such that when suction is applied to the chest tube the
accumulated pleural air will be removed allowing the lung to expand. However, because the
original lesion does not heal instantaneously, the application of suction to the pleural space may
also remove air directly from the bronchial system through the original defect that caused the
pneumothorax. The expectation is that the rate of air accumulation in the pleural space through
the defect is slower than the rate of air removal, thereby allowing the lung to expand. The
vacuum is maintained until there is no longer any evidence of a leak (therefore signifying that
the defect has healed). The easiest way to test for the presence of a leak is to ask the patient to
cough and then examine the water seal chamber for air bubbling up through the column of
water. Coughing increases intrathoracic pressure which forces air through the defect, if one is
still present. In the case of a very small defect, there may not be a detectable leak after correct
tube placement. Leaks can also be due to inadequate insertion of the chest tube such that some
of the suction holes lie outside the pleural cavity. Alternatively, there may simply be a leak in the
vacuum tubing or connectors. In these cases, however, the lungs will frequently fail to expand as
the suction capacity is “wasted” by continuously removing air from the limitless ambient
environment instead of the pleural space. This will be evident by virtue of a constant bubbling in
the water seal chamber. The amount of water in the water seal chamber has no effect on the
amount of air escaping from the system.
pg. 82
Answer E. Although it may be counterintuitive, most patients develop primary spontaneous
pneumothorax while at rest. Traditionally, expiratory chest x-rays were thought to aid in the
diagnosis of pneumothorax. Because the relative size of the chest cavity is thought to decrease
during expiration, and since the size of the pneumothorax is theoretically constant, the
pneumothorax should occupy a greater fraction of the chest cavity and, therefore, be easier to
detect upon expiration. Clinically, however, expiratory films have not demonstrated much
utility. Pneumomediastinum is a less common but generally benign finding, and is frequently
self-limited. It is usually due to persistent elevations in intrathoracic pressure such as that
caused by repetitive severe coughing, asthma exacerbations, or seizures. In contrast, secondary
pneumomediastinum is a morbid diagnosis that results from significant underlying disease such
as Boerhaave syndrome. Hamman sign or crunch is a physical examination finding in the setting
of pneumomediastinum. It describes the crunch-like sound heard upon cardiac auscultation as
the heart expands against the mediastinal air. The most frequent physical examination finding in
pneumomediastinum is SQ emphysema, frequently in the neck. Male smokers have more than a
20-fold increased risk of developing a spontaneous pneumothorax, whereas woman smokers
have a more than 10-fold increased risk. Other risk factors include tall height and cold weather.
There is an increased incidence in the fall and winter.
15. Which of the following is true regarding patients with a primary spontaneous
pneumothorax?
A. Pleuritic chest pain and dyspnea are the most common symptoms.
E. Atrial fibrillation is the most common cardiac rhythm in the acute setting of a spontaneous
pneumothorax.
Answer A. Ipsilateral pleuritic chest pain and dyspnea are the most common symptoms of a
primary spontaneous pneumothorax. Hemoptysis is uncommon in spontaneous pneumothorax
and would signify a specific etiology for the pneumothorax such as tumor. Patients may
occasionally be asymptomatic or have nonspecific complaints. Many patients delay treatment
for up to 1 week, and symptoms tend to resolve without treatment in 24 to 48 hours. Primary
spontaneous pneumothorax is three times more common in men than in women and typically
occurs in tall, healthy young men. Other factors associated with spontaneous pneumothorax are
smoking, changes in atmospheric pressure, mitral valve prolapse, and Marfan syndrome. The
most common arrhythmia is a mild sinus tachycardia.
16. Which of the following is the most common cause of death among nursing home residents?
pg. 83
A. Congestive heart failure
B. Pneumonia
C. Urosepsis
D. Massive stroke
E. Myocardial infarction
Answer B. Pneumonia is the most common cause of death among residents of long-term care
facilities. It is also the most common reason for transfer to such a facility. As with many other
diseases, the clinical presentation of pneumonia in the elderly may be very vague or atypical.
Frequently, elderly patients may lack fever, cough, chest pain, headache, and myalgias. They
may also not be strong enough or oriented enough to vocalize complaints about dyspnea.
Studies have revealed that in general, elderly persons manifest fewer overall symptoms than do
their younger cohort in the setting of pneumonia. S. pneumoniae remains the most common
pathogen in both community-acquired pneumonia (CAP) and in pneumonia acquired in a
nursing home setting. Although the etiology of nursing home-acquired pneumonia is often
undetermined, the microbiology more closely resembles CAP than nosocomial pneumonia.
Interestingly, the risk of invasive pneumococcal disease is fourfold higher in the nursing home
population than in elderly persons living in the community. Although its efficacy has not been
100% validated, most authors agree that all patients of long-term care facilities should be
vaccinated against both influenza and Streptococcus pneumoniae.
A. A 72-year-old female nursing home resident being treated for a chronic obstructive
pulmonary disease exacerbation.
E. A 28-year-old asymptomatic medical student who recently had a positive pressure differential
(PPD) test.
Answer D. Respiratory syncytial virus (RSV) is the most common cause of bronchiolitis, followed
by parainfluenza, influenza, adenovirus, and rhinovirus. To avoid infecting other uninfected
patients and health care workers (who then transmit infections to yet more patients), patients
with RSV bronchiolitis should be placed in respiratory isolation on droplet precautions. If
multiple patients with RSV bronchiolitis are admitted, then the group could be isolated as a
cohort until discharge. Bacillus anthracis is not transmitted from person to person. Patients with
pg. 84
COPD exacerbations and CAP also do not require isolation. Finally, the medical student with a
PPD but no symptoms of active TB has latent TB infection and is not contagious to other people.
18. Which of the following is a reasonable indication to perform tube thoracostomy in a patient
with a parapneumonic pleural effusion?
19. Patients with thoracic outlet syndrome most commonly present with:
A. Neurologic symptoms
B. Ischemic symptoms
C. Infectious symptoms
D. Pain
pg. 85
Answer A. The thoracic outlet syndrome comprises a group of pathologic conditions associated
with compression of the structures at the junction of the upper extremity and trunk. The
findings are neurologic (95%), venous (4%), and arterial (1%). The elevated arm stress test is the
best physical examination tool to determine the presence of thoracic outlet syndrome. The test
involves abducting the shoulders to 90 degrees with the elbows flexed at 90 while opening and
closing the fists for 3 minutes. A positive result is indicated by arm fatigue and pain, and the
inability to keep it abducted.
20. A 44-year-old male without past medical history presents with fever and cough. His vital
signs are: 100.2, 89, 18, 122/72, 99% RA. His chest x-ray is shown (Fig. below). Which of the
following is the most appropriate therapy for this patient?
A. Supportive care
B. Oseltamivir
C. Doxycycline
D. Piperacillin–tazobactam
pg. 86
care is not sufficient for patients with presumed bacterial pneumonia. Oseltamivir is not
effective for patients without influenza. Piperacillin–tazobactam or vancomycin plus cefepime
would be regimens used for severely ill patients with high risk for Pseudomonas infection (those
with comorbidities, recent exposure to antibiotics, or recent hospital stays).
21. Which of the following are common findings in patients diagnosed with Pneumocystis carinii
pneumonia (PCP)?
C. Pleural effusions
Answer E. Because PCP cannot be cultured, and the gold standard in making the diagnosis
remains invasive (bronchoscopy and subsequent staining), there has been much interest in
trying to find surrogate serum markers to indicate the presence of P. jirovecii. Most of this
interest has focused on LDH. Unfortunately, although it is true that LDH levels are elevated in
the setting of PCP, this finding is not specific for PCP. However, the level of LDH appears to
correlate with the degree of radiographic severity. This supports the idea that elevated LDH
levels are more a reflection of generalized lung inflammation than a marker of any particular
organism. Lactate levels and liver transaminases have no role in the diagnosis of PCP. Most
patients with PCP and hypoxia have a respiratory alkalosis, as their respiratory rate (and depth)
increases with the severity of their hypoxia. Finally, patients with CD4 counts <200 are most
susceptible to PCP, and it is this population who should be on chemical prophylaxis. Pleural
effusions are not common in PCP.
22. A 26-year-old previously healthy man is brought to the ED after being found unconscious
outside a bar after having vomited on himself. He is drunk and afebrile, and his pulse oximetry
reveals a saturation of 96%. His chest x-ray reveals a minimal right-sided streaky infiltrate (Fig.
below). Which of the following is true?
A. The patient likely has a developing bacterial pneumonia and requires broad-spectrum
antibiotics.
B. The patient likely has a chemical pneumonitis caused by the aspiration of acidic gastric
contents, which may later develop into pneumonia.
pg. 87
D. Corticosteroids have been proven to be beneficial in patients such as these.
E. This patient is probably suffering from a combination of chemical pneumonitis and bacterial
infection.
Answer B. This patient has aspiration pneumonitis, which is a chemical pneumonitis caused by
the aspiration of acidic gastric contents. The severity of lung injury increases as the pH drops,
and most studies agree that the pH must be <2.5 to cause significant injury. Chemical aspiration
most commonly occurs in patients who have depressed levels of consciousness, as in this
patient, and in young persons. Aspiration is otherwise a common problem among elderly
persons in nursing homes after strokes. Though they are widely used, antibiotics and
corticosteroids are not routinely recommended in patients with aspiration pneumonitis. Gastric
acid inhibits bacterial growth, so gastric contents are sterile under normal conditions. Unless the
patient has a coexisting condition promoting gastric bacterial growth, antibiotics should not be
used. Such conditions include patients on antacid therapy, patients receiving enteral feeding,
and patients with known gastroparesis or small bowel obstruction. Corticosteroids are also
widely used in patients with chemical aspiration, although they have not been proven to be
beneficial.
23. A 50-year-old male without past medical history presents with acute onset of cough,
shortness of breath, and fever for 2 days. He has crackles in his left middle lung field and chest
x-ray is shown in Figure below. Which of the following is the most likely etiologic cause?
A. Streptococcus pneumoniae
B. Haemophilus influenza
C. Staphylococcus aureus
D. Mycoplasma pneumoniae
pg. 88
E. Klebsiella pneumoniae
Answer A. The patient has community acquired pneumonia (CAP). The location is likely in the
lingula, as the left heart border is obscured. S. pneumoniae is still the most common cause in
patients over 40 years of age. H. influenzae is a very common cause in patients with any chronic
medical disease such as chronic obstructive pulmonary disease (COPD) or diabetes. S. aureus is
primarily a nosocomial pathogen but is common in the setting of concomitant influenza
infection. Mycoplasma is one of the most common causes of CAP in young adults (<40 years)
and usually causes an interstitial pattern on chest x-ray. Klebsiella is seen far more often in the
setting of COPD or alcoholism.
24. Which of the following x-ray findings is diagnostic for pulmonary embolism (PE)?
A. Atelectasis
C. Elevated hemidiaphragm
Answer E. Although >75% of patients with PE will have an abnormal chest x-ray, no single finding
is diagnostic for PE. Westermark sign (decreased peripheral vascular markings in the lung field
affected by the embolus) and Hampton hump (a wedge-shaped homogeneous density with its
base along the pleural surface and its apex pointed toward the hilum representing pulmonary
infarction) are the classic findings, but both findings have very low sensitivity and specificity. In
addition to the other findings listed, parenchymal infiltrates, hilar or mediastinal enlargement,
pg. 89
cardiomegaly, pleural effusions, pulmonary edema, and a prominent central pulmonary artery
(Fleischner sign) may also be seen. None are sensitive or specific findings.
25. A 34-year-old male with AIDS presents with cough, malaise, and exertional dyspnea for 8
days. He stopped his highly active antiretroviral therapy several weeks prior due to side effects.
His vital signs are 100.4°F, 95, 22, 165/85, 95% RA. Chest radiography shows an interstitial
pneumonia pattern. Which of the following is the most appropriate treatment?
A. Clindamycin
B. Trimethoprim–sulfamethoxazole
C. Doxycycline
D. Piperacillin–tazobactam
E. Primaquine
Answer B. The patient likely has pneumocystis pneumonia (PCP). Pneumocystis is a fungal
opportunistic infection commonly seen in AIDS patients when the CD4 count drops below 250
cells per mcL. Exertional dyspnea (rather than dyspnea at rest), nonproductive cough, low-grade
fevers, and malaise in an acute or subacute pattern are characteristic. Chest radiographs usually
show an interstitial pneumonia pattern but can be completely normal in up to one-third of
cases. First-line treatment of Pneumocystis pneumonia is with trimethoprim–sulfamethoxazole.
Supplemental corticosteroid therapy is added to improve outcomes if the PaO2 is less than 70
mm Hg. Clindamycin plus primaquine is third-line therapy, after pentamidine. Doxycycline
represents excellent monotherapy for CAP in the young adult but does not adequately treat
Pneumocystis pneumonia. Piperacillin–tazobactam possesses broad-spectrum coverage for
most intra-abdominal and nosocomial infections but is not indicated for treatment of
Pneumocystis pneumonia.
26. The combination of ceftriaxone and azithromycin is a common dual-antibiotic regimen used
in the empiric treatment of community-acquired pneumonia. For which of the following
organisms is azithromycin included?
A. Haemophilus influenzae
B. Staphylococcus aureus
C. Legionella pneumophila
D. Streptococcus pneumoniae
E. Moraxella catarrhalis
pg. 90
Answer C. Legionella spp., along with Chlamydia spp. and Mycoplasma spp. all respond to
macrolide therapy. The remaining organisms are usually treated with cephalosporins or
fluoroquinolones.
27. A 65-year-old smoker presents with sudden onset of shortness of breath for 2 hours. He
denies chest pain, fever, productive cough, or lower extremity edema. Vitals signs are 99°F, 95,
24, 200/95, 92% on RA. He has bilateral wheezes. Which of the following best differentiates
between obstructive lung disease and congestive heart failure (CHF)?
B. D-dimer
D. Troponin I
E. Angiotensin-converting enzyme
Answer C. The patient presents with symptoms of either obstructive lung disease or flash
pulmonary edema with reactive bronchospasm, also called cardiac asthma. The two entities can
be clinically indistinguishable. BNP is released in response to ventricular stretch during CHF.
Normal BNP levels significantly reduce the likelihood of CHF exacerbation. Both obstructive lung
disease and cardiac asthma may respond to albuterol therapy. D-dimer has a high negative
predictive value for pulmonary embolism (PE) but has no role in the diagnosis of CHF or
obstructive lung disease. Troponin I has high specificity for acute myocardial infarction and
would not be elevated in CHF exacerbations without infarction. Angiotensin-converting enzyme
levels are used to evaluate sarcoidosis.
28. A 45-year-old female with a history of asthma presents with her typical acute asthma
exacerbation. She has had a runny nose and a sore throat but denies fever or productive
sputum. Examination reveals bilateral wheezes. Her vital signs are 98.6°F, 110, 24, 156/95, 99%
RA. She feels slightly better after an albuterol nebulizer treatment. You send off labs, and the
white blood cell (WBC) count returns elevated at 13.5K. Which of the following is the next best
course of action?
pg. 91
Answer C. The patient has a typical acute asthma exacerbation. The mainstays of treatment are
bronchodilators for bronchospasm and corticosteroids for airway inflammation. Despite her
upper respiratory symptoms and the presence of leukocytosis, she has no signs of superimposed
pneumonia, so empiric treatment is not indicated. A chest x-ray could be obtained to further
evaluate the possibility of pneumonia. Arterial blood gas sampling is only helpful in patients who
are deteriorating despite aggressive bronchodilator therapy and should not routinely be
obtained in the evaluation of most acute asthma exacerbations. Patients with asthma
exacerbations should not be discharged without providing either systemic or inhaled
corticosteroid therapy.
29. A 61-year-old male smoker recently performed some repairs on several air-conditioning
units during the late summer. He is now brought in by his family confused, with high fevers,
chills, a dry cough, and diarrhea. The test that will best determine the likely specific cause of his
illness is which of the following?
B. Blood cultures
C. Sputum cultures
D. Chest x-ray
E. Serology testing
30. Which of the following is the most common cause of superior vena cava syndrome (SVCS)?
A. Lung cancer
pg. 92
B. Deep venous thrombosis (DVT) of the superior vena cava
C. Tuberculosis
E. Aortic aneurysm
Answer B. While historically, syphilitic aortic aneurysms and tuberculosis accounted for the
majority of cases of SVCS, the majority of cases today are due to DVT of the SVC in patients with
an indwelling catheter such as a PICC line or Port-a-Cath. Many of these patients have such
devices because of cancer treatment, but their cancers are not the primary cause of SVC
obstruction. Such patients are managed with anticoagulation and may need to have the device
removed. Among patients without indwelling catheters, lung cancer (primarily nonsmall cell) is
the most common cause while lymphoma is the next most common. The symptoms include
head and facial edema, as well as edema of the neck and arms. The edema is often associated
with plethora, cyanosis, and visibly distended subcutaneous vessels. The course is typically
indolent, and patients rarely present with acute airway compromise. Patients with SVCS due to a
malignancy are typically managed with palliative chemotherapy and radiation since the
long-term prognosis is poor.
31. A patient presenting to the ED with which of the following is least likely to have audible
wheezes on physical examination?
D. Asthma
E. Sarcoidosis
Answer E. Wheezes are continuous, high-pitched, musical sounds that can be heard on
inspiration or expiration. They are caused by high-velocity airflow through a narrowed airway (in
much the same way that a murmur is caused by high-velocity blood flow through a narrowed
vessel or valve). Any pulmonary disease characterized by obstruction can result in audible
wheezes on physical examination. A good, short mnemonic for obstructive airway disease is
LACE:
Asthma
Chronic bronchitis
pg. 93
Emphysema
32. A 72-year-old male develops acute-onset shaking chills and shortness of breath a few days
after an URI. His past medical history is significant for diabetes and stable coronary artery
disease. Vital signs are 101°F, 115, 24, 144/95, 90% RA. Physical examination reveals a patient
with rigors and right lower lobe crackles. Which of the following is the most appropriate next
step in management?
Answer C. The patient has evidence of moderately severe community-acquired pneumonia, with
tachycardia, tachypnea, and hypoxia. Accordingly, the patient should be admitted to the
hospital for intravenous antibiotics and fluids and oxygen. Community-acquired pneumonia in
this age-group is most commonly due to pneumococcus, atypical organisms, and gram-negative
bacilli. Among the choices in the preceding text, C is the most appropriate. Other acceptable
regimens could include a fluoroquinolone or ampicillin–sulbactam. Oral medications and
discharge would not be appropriate because of the vital sign abnormalities and comorbidities.
Clindamycin possesses little gram-negative coverage and would not be adequate therapy.
Piperacillin–tazobactam is a potent, broad-spectrum, antipseudomonal antibiotic used only for
nosocomial pathogens causing severe illness.
33. A 24-year-old male came to the ED with dyspnea and pleuritic chest pain for 10 days,
complaining that his breathing is getting worse. You are surprised to find a 40% right-sided
pneumothorax and you immediately place a chest tube, connecting it to wall suction. Although
the patient initially improved, 1 hour later he began coughing vigorously, and he appears
pg. 94
tachypneic and dyspneic. Assuming he has not experienced a recurrence or worsening of his
pneumothorax, what is the most likely cause of his problem?
B. Development of empyema
34. Which of the following is the most common arrhythmia in patients with pulmonary
embolism (PE)?
B. Sinus tachycardia
C. Atrial fibrillation
E. Ventricular fibrillation
pg. 95
Answer B. The chief use of EKG in the evaluation of patients with a suspected PE is to rule out
the presence of other causes of chest pain such as myocardial ischemia or acute MI and
pericarditis. Numerous EKG findings have been reported in the setting of acute PE, although
“S1Q3T3” is perhaps the most famous. Several papers have reliably refuted the utility of this
finding. S1Q3T3 was first described in a 1935 study of seven patients who all likely had massive
pulmonary embolus. Similar studies have all been plagued by a selection bias for patients with
large or massive PE. Several EKG findings are more specific in patients with massive PE, but
other clinical signs and symptoms (e.g., hypoxia, dyspnea, tachypnea, and chest pain) are
typically more useful than EKG.
35. A 24-year-old male with no past medical history presents with intermittent fevers, shortness
of breath, cough, and malaise for 1 week. Vital signs are 100.3°F, 85, 20, 145/92, 98% RA. Chest
x-ray demonstrates patchy interstitial infiltrates. Which of the following is the most appropriate
treatment regimen?
A. Cefdinir
C. Doxycycline
D. Amoxicillin–clavulanic acid
E. Clindamycin
Answer C. The patient has community-acquired pneumonia (CAP), and while the nature of the
infiltrate on chest x-ray can’t be used to predict the bacterial etiology of pneumonia, interstitial
patterns are most commonly associated with “atypical” organisms, such as Chlamydia and
Mycoplasma, especially for the purposes of board testing. Of all the regimens listed, only
doxycycline covers both typical and atypical CAP organisms. Cefdinir and amoxicillin–clavulanic
acid lack atypical organism coverage and should be supplemented with a macrolide for optimal
CAP treatment. Vancomycin plus gentamicin offers excellent nosocomial coverage but is not
necessary in this young, healthy patient with no comorbidities or toxic features. Clindamycin
lacks appropriate gram-negative coverage for organisms such as H. influenza or Klebsiella.
B. Levofloxacin
pg. 96
D. Pentamidine
E. Doxycycline
Answer A. Both primaquine/clindamycin and pentamidine are alternative regimens for PCP
treatment. In addition, trimethoprim–dapsone has equal efficacy to TMP-SMX and can be used
in patients who are allergic to sulfa. Pentamidine can only be administered intravenously or by
inhalation. In addition, pentamidine use is complicated by significant side effects. Therefore,
primaquine and clindamycin comprise the best alternative outpatient regimen. Intravenous
pentamidine is the treatment of choice for patients unable to tolerate intravenous TMP-SMX. In
addition to allergic responses, TMP-SMX has many side effects including a high incidence of skin
rash and bone marrow suppression. In the outpatient regimen of primaquine and clindamycin,
the clindamycin is included because primaquine lacks activity against community-acquired
pathogens. Because only patients with mild to moderate disease will be discharged from the
hospital, it is imperative to include coverage for routine community-acquired pneumonia (CAP)
in addition to giving antibiotics which target PCP. In mild cases, or when the patient’s CD4+
T-cell count hovers close to 200, it may be difficult to distinguish between PCP and CAP.
37. A 29-year-old female presents after going running on the treadmill with cough, shortness of
breath, and chest tightness. She has no throat swelling or rash. Her examination is normal
except for bilateral wheezes. She has been taking an over-the-counter medication for her
intermittent migraine headaches. What medication is the most likely cause?
A. Acetaminophen
B. Phenylephrine
C. Naproxen
D. Capsaicin
E. Paracetamol
Answer C. The patient likely has aspirin-exacerbated respiratory disease, which is caused by
aspirin and other NSAIDs. NSAIDs inhibit cyclo-oxygenase, reducing the formation of
prostaglandin E2 (PGE2) from arachidonic acid. PGE2 normally serves to inhibit formation of
inflammatory leukotrienes. With the formation of PGE2 reduced, leukotrienes are increased,
triggering airway inflammation and bronchospasm. Exercise can trigger these bronchospastic
attacks. Of the answer choices, only naproxen is an NSAID. Phenylephrine is an alpha-1
adrenergic agent with little effect on airway physiology. Capsaicin is a topical agent that reduces
accumulation of substance P in peripheral sensory neurons. Paracetamol is the formulation of
acetaminophen that is widely used in the United Kingdom.
pg. 97
38. Which of the following test results is most concerning in a patient with an acute asthma
exacerbation?
A. Leukocytosis of 14,000
B. K+ 3.4
C. Hyperlactatemia
D. Eosinophilia
Answer E. The most common acid–base disturbance in the setting of acute asthma
exacerbations is a respiratory alkalosis. However, concomitant metabolic acidosis due to lactic
acidosis occurs in up to 28% of patients. The etiology of the lactatemia is not known but it has
been hypothesized to occur because of fatiguing respiratory muscles. However, recent case
reports have demonstrated that lactatemia can occur even in intubated patients who are
paralyzed and, therefore, have no respiratory muscle action. Regardless, the clinical relevance of
an elevated lactate level accompanying an acute asthma exacerbation is not known, and lactate
levels do not predict respiratory failure in critically ill patients. The presence, however, of a
respiratory acidosis as indicated by the ABG above is an ominous sign. A normal or elevated
PCO2 in a tachypneic asthmatic typically suggests severe obstruction and impending ventilatory
failure. Exceptions to this rule occur most commonly in patients with underlying chronic
obstructive pulmonary disease who may retain CO2 at baseline. However, such patients should
have a normal pH and an elevated HCO3− due to chronic renal compensation. Leukocytosis, mild
hypokalemia, and eosinophilia are not useful in the management of acute asthma.
39. Which of the following vasculitis syndromes is most likely to present as pulmonary
hemorrhage requiring emergent airway management?
A. Churg–Strauss syndrome
B. Polyarteritis nodosa
C. Wegener granulomatosis
D. Goodpasture syndrome
E. Mixed cryoglobulinemia
pg. 98
succumbing to DAH. A recent study estimated that mortality has improved somewhat, with a
2-year survival rate of 50% in all treated patients. DAH is the most common cause of death.
40. A 65-year-old male presents with dyspnea on exertion for several weeks. He does not
regularly see physicians and reports no past medical history (Fig. below). Which of the following
is the most likely etiology of his symptoms?
A. Bacterial pneumonia
B. Viral pneumonia
D. PE
E. Pancreatitis
pg. 99
Answer C. Chest x-ray demonstrates cardiomegaly and bilateral pleural effusions. The most
common cause of pleural effusion in Western countries is CHF. Malignancy, pneumonia, and PE
are less common causes. Outside of Western countries, tuberculosis is the most common cause.
Pancreatitis can cause a solitary left-sided pleural effusion.
41. A 63-year-old female is brought to the ED by her children because she is lethargic and has
labored breathing. They last saw her 4 days ago when she seemed well. Her vital signs are T
101.8°F, HR 120 per minute, RR 32 per minute, and an SaO2 of 89% on 100% oxygen by face
mask. She is intubated and placed on assist-control ventilation. A subsequent chest x-ray reveals
diffuse bilateral infiltrates, and normal heart size. You suspect she has severe pneumonia and
acute respiratory distress syndrome (ARDS). Which of the following summarizes the best
ventilation strategy?
A. Due to low compliance, patients with ARDS need higher tidal volumes and higher positive
endexpiratory pressure (PEEP) to ensure adequate ventilation.
B. Due to significant airway obstruction, such patients require very low or no PEEP similar to
asthma patients to avoid air trapping.
C. Due to high compliance, such patients require lower tidal volumes and lower PEEP to improve
oxygenation.
D. Due to low compliance, such patients require lower tidal volumes and higher PEEP to avoid
barotrauma.
Answer D. In ARDS, the alveoli are flooded with protein-rich fluid due to leaky pulmonary
capillaries. The result is poorly ventilated and poorly compliant alveoli (i.e., fluid-filled alveoli are
difficult to distend). Owing to the presence of poorly compliant alveoli, both peak and plateau
airway pressures in ARDS are higher than in healthy subjects. Ventilating such patients with
“normal” or high tidal volumes further elevates the airway pressure and may result in
barotrauma and further injury to the lung. The ARDS Network group of investigators published a
study in 2000 describing a “lung protective” strategy in which such patients were ventilated with
tidal volumes that were much lower than normal. The idea of this strategy is to reduce
ventilator-associated lung injury as a result of alveolar overdistension from high tidal volumes
and airway pressures. Owing to the low tidal volumes, however, such patients require higher
PEEP levels to recruit alveoli in order to ensure adequate oxygenation. The goal is to use the
lowest PEEP required to achieve an FiO2 ≤0.50. An inadvertent result of this strategy is
hypoventilation and a resultant rise in PaCO2 levels (hypercapnia). Some studies suggest that
the resulting acidosis (respiratory acidosis from high PaCO2 levels) may actually be protective,
although the ARDS Network investigators treated the acidosis with bicarbonate infusions. An
ideal strategy regarding this has not yet been developed.
pg. 100
42. A 22-year-old male presents with cough, fever, and shortness of breath for 3 days. He has no
past medical history. His vital signs are 100.5°F, 92, 22, 122/72, 98% RA. Examination reveals left
lower lung field crackles that do not clear on coughing. Which of the following is the most
appropriate therapy?
A. Doxycycline PO
B. Linezolid PO
C. Cephalexin PO
D. Clindamycin PO
E. Piperacillin–tazobactam IV
43. A 23-year-old male with a history of human immunodeficiency virus (HIV) presents with
shortness of breath, fever, and malaise. A chest x-ray is shown (Fig. 9-8). Arterial blood gas
shows a PaO2 of 60 mm Hg. Which of the following, in addition to antibiotics, is the most
appropriate therapy?
A. Albuterol
B. Prednisone
C. Aspirin
D. Vasopressin
E. Hyperbaric oxygen
pg. 101
Answer B. This HIV patient has bilateral fluffy infiltrates consistent with PCP.1 Over
three-fourths of all patients with acquired immunodeficiency syndrome (AIDS) will develop PCP
at some point in their lifetimes. It is also the most common identifiable cause of death in
patients with AIDS. Pneumocystis is classified as a protozoan, but it has many characteristics of a
fungus. Symptoms of PCP, like all pneumonias, include fever, cough, and shortness of breath,
but a subacute or mild course is characteristic. Chest radiography classically demonstrates
diffuse, bilateral interstitial infiltrates but can be completely normal up to 20% of the time.
First-line therapy is with TMP-SMX. Adjunctive corticosteroid therapy is indicated in patients
who have significant hypoxia (PaCO2 <70 mm Hg). Albuterol may be used in patients with
pneumonia or bronchitis who have a large bronchospastic component to their symptoms.
Aspirin is not indicated in most infectious processes. Vasopressin may be used in patients with
septic shock who are adequately volume resuscitated. Hyperbaric oxygen does not currently
have a role in the management of PCP.
44. A 28-year-old male presents with a chief complaint of abrupt-onset right-sided chest pain
and shortness of breath. His vitals are P 105, RR 22, BP 142/90, SaO2 97% on room air (Fig. 9-9).
Which of the following is true?
E. Re-expansion pulmonary edema usually occurs more than 24 hours after re-expansion
pg. 102
Answer A. The patient has a spontaneous large right-sided pneumothorax. Historically, the
decision about whether to treat a spontaneous pneumothorax invasively hinges on the size of
the pneumothorax. Unfortunately, there is no standard definition about what constitutes a
“large” pneumothorax. Some studies define “large” as a rim of air ≥2 cm at the hilum (which
roughly correlates to half the lung volume) while others use 3 cm as a threshold. However, in
stable patients even a large pneumothorax can be successfully treated with oxygen
administration alone. The main benefit of invasive treatment is more rapid resolution and
avoidance of hospital admission. Aspiration is a safe, less invasive technique than traditional
tube thoracostomy. This is usually performed with a commercial kit using a one-way valve and
manual aspiration. If, after 2.5 L of air has been aspirated, there remains a significant
pneumothorax, tube thoracostomy should be performed. In patients with a simple
pneumothorax, small caliber tubes (≤22 Fr) should be used. Re-expansion pulmonary edema is a
frequent complication of pneumothorax treatment, though often asymptomatic. It tends to
develop fairly rapidly after re-expansion.
45. Possible complications of acute asthma exacerbations include all of the following except:
A. Pneumothorax
pg. 103
B. Subconjunctival hemorrhage
C. Subcutaneous emphysema
D. Pulmonary embolus
E. MI
46. A 38-year-old female with a history of asthma presents to the emergency department (ED)
with a chief complaint of wheezing and chest tightness typical of her asthma. She had recently
run out of her medicines and complains of upper respiratory symptoms for the last week.
Accompanying her is her 6-year-old son and 72-year-old mother, both of whom have asthma.
Which of the following summarizes the treatment differences of acute asthma exacerbations
between these groups?
A. Corticosteroids are avoided in pediatric populations because of concerns about their effects
on growth.
B. The treatment of children with acute asthma exacerbations is similar to the treatment of
adults and includes β-agonists, anticholinergics, and corticosteroids.
C. Cromolyn sodium has a prominent role in the treatment of acute exacerbations of pediatric
but not adult asthma.
D. The first-line agent in treating elderly patients with acute asthma is ipratropium due to the
high likelihood of underlying coronary artery disease and subsequent risk with
β-agonist–induced tachycardia.
E. Leukotriene modifiers have recently been shown to be useful in acute asthma exacerbations
in elderly but not in pediatric or young adult patients.
pg. 104
inflammatory mediators from mast cells through chlorine channel blockade (also known as a
mast cell stabilizing agent) and is used to prevent inflammation in long-term management.
There are limited data that suggest that leukotriene receptor antagonists (e.g., montelukast,
zafirlukast) are beneficial in the treatment of patients with acute asthma exacerbations who fail
initial β-agonist therapy. Until further studies are done, however, leukotriene modifiers are not
currently recommended for acute asthma management.
47. A 71-year-old female without any past medical history presents with acute onset of
shortness of breath and cough. She denies fever or productivity to her cough. Her vital signs are:
99.5, 110, 22, 122/72, 91% on RA. Her physical examination is unremarkable except for the
tachycardia. Her complete blood count (CBC), chemistry, troponin, EKG, and chest x-ray are all
normal. Which of the following is the most appropriate next step in management?
C. Administer piperacillin–tazobactam
D. Give dexamethasone
Answer A. With shortness of breath, tachycardia, hypoxia, a normal physical examination, and a
normal chest x-ray, pulmonary embolism is a very likely diagnosis. Diagnostic evaluation with
either a D-dimer or a CT angiogram should be pursued. An argument can be made to do either
test, but among the answer choices, D-dimer is the only one present. BNP levels are only helpful
in the setting of signs of CHF complicated by possibility of other diagnoses such as COPD. There
is no indication for giving piperacillin at this time, given that the patient has no risk factors for
nosocomial pathogens. Similarly, without a history of COPD or other obstructive lung disease,
neither steroids nor albuterol is indicated.
48. A 24-year-old male with no past medical history presents with sudden onset of shortness of
breath and chest pain. He appears uncomfortable. Vital signs are 98.6°F, 100, 24, 150/92, 96%
RA. He has significantly decreased breath sounds on his right side. Chest x-ray is shown in Figure
below. Which of the following is the most appropriate next step in management?
C. Pleurodesis
pg. 105
E. Albuterol nebulizer
Answer A. The patient has a 100% spontaneous right-sided pneumothorax. Treatment involves
supplemental oxygen and tube thoracostomy. Pneumothoraces generally resorb at a rate of 2%
per day, and this rate is quadrupled by 100% oxygen. The presence of 100% oxygen causes
intra-alveolar nitrogen washout and creates a nitrogen gradient that draws nitrogen-rich room
air from the pleural space into the alveolar space. However, patients with a large pneumothorax
require thoracostomy with tubes ranging from 7- to 14-Fr pigtail catheters to 28-Fr tubes. Size
36 Fr tubes are usually reserved for fluid-containing pneumothoraces, such as
hemopneumothorax in trauma. Pleurodesis is usually performed in patients who have had
multiple pneumothoraces due to a secondary cause. Positive pressure ventilation of any form
can only exacerbate the pneumothorax as the positive intraalveolar pressure will serve to
worsen intrapleural air accumulation. Albuterol nebulizer treatment is helpful in bronchospasm
but has little benefit in pneumothorax management.
49. A 47-year-old male smoker with a history of hypertension and hyperlipidemia presents for
evaluation of left-sided mid back pain and shortness of breath. He reports he awoke with the
pain which has worsened throughout the day. His vitals are: T 98.6°F, RR 22, P 100, BP 124/71,
SaO2 94% on 4 L NC. A CT scan is performed and reveals the image shown in Figure below. An
hour after presenting for care, his vital signs are essentially unchanged. Which of the following is
true?
pg. 106
B. An echocardiogram should be performed to guide therapy
Answer D. The patient’s CT scan reveals a large “saddle” embolus in both branches of the
pulmonary artery. Fortunately, the patient’s vital signs indicate relative hemodynamic stability.
tPA and surgical or catheter-based embolectomy are reserved for patients with hemodynamic
instability who may not respond well to conservative therapy with intravenous anticoagulants.
Echocardiograms are often performed to determine the extent of right heart strain as a proxy
for the extent of clot. However, this patient’s clot burden is easily visible and known to be
significant. The prognostic value of right heart strain in this patient is not clear. Despite his
significant clot burden, the patient’s vital signs have been stable in his brief stay in the
emergency department and there is no evidence of hemodynamic compromise. As
catheter-based embolectomy techniques improve and more data are gathered, the balance may
shift to a more interventional approach.
50. A 62-year-old female with a history of O2-dependent chronic obstructive pulmonary disease
(COPD) presents to the ED with a chief complaint of dyspnea and increased cough productive of
yellow phlegm. The patient uses 2 L of O2 at home. The patient is an ill-appearing, dyspneic
woman speaking in sentence fragments. Her SaO2 reads 85% on 4 L of O2 by nasal cannula. As
pg. 107
you start to increase her O2, you wonder if you are going to eliminate her respiratory drive. The
next best step is to:
B. Increase the O2 to 6 L, because minute ventilation changes little in COPD patients exposed to
higher levels of oxygen
C. Decrease the O2 to 2 L (her baseline) in order to increase the patient’s respiratory drive
D. Perform an arterial blood gas (ABG) to assess the patient’s exact ventilatory and oxygenation
status because you do not know what effect changing the oxygen will have
E. Leave the O2 at 4 L because the patient is likely hypoxic at baseline and continue treatment
hoping the patient will improve
Answer B. Hypoxemia is the most immediate life threat to patients with COPD exacerbations.
This patient’s pulse oximetry of 85% indicates severe hypoxemia. Although an arterial blood gas
(ABG) could be performed to verify this patient’s hypoxemia and to elucidate the degree of CO2
retention, it is clear that what this patient needs is oxygen. A reasonable goal of oxygen therapy
should be to titrate it to a saturation of 90%. Early observational studies demonstrated that
some degree of worsening hypercapnia usually occurs following increased oxygen delivery to
patients with COPD. This should not be confused with a decrease in the patient’s respiratory
drive. Instead, increasing hypercapnia results primarily from worsened ventilation/perfusion
(V˙/Q˙) mismatching and because of the Haldane effect in which oxygenated erythrocytes have a
decreased affinity for CO2, causing CO2 offloading and an increase in blood CO2 concentrations.
Although this patient may eventually require intubation, it is prudent to try noninvasive
measures, including noninvasive ventilation before endotracheal intubation.
A. Elderly patients
B. Pregnancy
D. Postoperative patients
Answer E. Nearly all patients with venous thromboembolism have elevated D-dimer levels.
Therefore, assays measuring D-dimer levels have proliferated as they are a highly sensitive but
poorly specific screening test for pulmonary embolism (PE). Due to its elevation in numerous
other scenarios, an elevated D-dimer level can never be used as proof of the presence of a
pg. 108
pulmonary embolus. However, because of the high sensitivity of most D-dimer assays
(sensitivity varies depending on the assay used), normal D-dimer levels can safely rule out the
presence of a pulmonary embolus in very low-risk patients.
52. You diagnose a 68-year-old female with community-acquired pneumonia and decide to
admit her to the hospital for inpatient management. Apart from early and appropriate
intravenous antibiotic therapy, which of the following would be most likely to improve
outcome?
A. N-acetylcysteine
B. Oseltamivir
C. Albumin
D. Corticosteroids
E. Lactulose
Answer D. A 2015 meta-analysis by Siemieniuk et al. found that time to clinical stability, hospital
length of stay, and development of ARDS were all reduced when adjunctive corticosteroid
therapy was given with antibiotics for hospitalized patients with community-acquired
pneumonia. The study did not find a statistically significant reduction in mortality, although the
trend was in that direction (relative risk 0.67, 95% CI 0.45 to 1.01). Predictably, corticosteroid
use was associated with hyperglycemic episodes. Further study will be needed before
corticosteroids are prescribed in protocolized fashion for patients with community-acquired
pneumonia. Oseltamivir is used to treat influenza, but is not indicated in patients without
relevant signs. N-acetylcysteine is used to manage acetaminophen overdose and may have
some promise as a general free radical treatment, but this has yet to be proven. Albumin is
useful in patients with spontaneous bacterial peritonitis, but not in pneumonia. Lactulose is
used to present hyperammonemia in patients with cirrhosis.
53. Which of the following patients is at highest risk of developing a lung abscess?
pg. 109
Answer C. Lung abscesses are usually a consequence of aspiration of contaminated
oropharyngeal flora. Therefore, the same population of patients who are at risk for aspiration
are also at risk for the development of a lung abscess. Risk factors include any syndrome that
results in depressed levels of consciousness and consequently increases the risk of aspiration
such as alcoholism, massive stroke, head trauma, seizures, and anesthesia. Patients with poor
oral hygiene are particularly susceptible because of the increased numbers of organisms among
their oral flora. Still, lung abscesses develop in a scant minority of patients who aspirate
contaminated oral flora. Although immunocompromised patients are also at risk, the HIV+
patient with a CD4+ T-cell count >200 is unlikely to develop a lung abscess. Although drug abuse
may lead to depressed levels of consciousness, IV drug use puts patients at greater risk for
infectious complications related to poor skin sterilization techniques, such as endocarditis,
phlebitis, or other local skin infections. It also increases the risk of pneumonia due to S. Aureus,
which rarely causes necrotizing pneumonia. Patients with Parkinson disease may have
significant dementia, but it is primarily a motor disease that should not initially put patients at
high risk for aspiration.
54. Which of the following is a criterion for acute respiratory distress syndrome (ARDS)?
B. Cardiomegaly
E. Acute onset
55. A 55-year-old female with chronic obstructive pulmonary disease (COPD) presents with
acute dyspnea, purulent cough, and fever. Vital signs are 100.4°F, 110, 22, 175/90, 98% on 4 L
NC. She has mild neck muscle retractions and bilateral rhonchi. Chest x-ray shows no focal
infiltrate. Albuterol nebulizer, methylprednisolone 125 mg IV, and levofloxacin 500 mg IV have
only minimally improved respiratory distress. Which of the following is the next best step in
management?
pg. 110
A. Azithromycin 500 mg IV
B. Vancomycin 1 g IV
C. Aminophylline 5 mg/kg IV
Answer D. The patient is not responding to initial therapy for COPD exacerbation. She is in
moderate respiratory distress but has a normal mental status. In patients with COPD,
noninvasive positive pressure ventilation is an excellent alternative to endotracheal intubation
and mechanical ventilation. It reduces the risk of endotracheal intubation, ICU length of stay,
and overall hospital length of stay. Azithromycin adds little to fluoroquinolone coverage for
common pathogens in COPD exacerbations. Vancomycin is used to treat methicillin-resistant S.
aureus, but this would not be suspected in a patient without frank pneumonia or nosocomial
exposure. Aminophylline is a phosphodiesterase inhibitor that is used in asthma exacerbations
but is not as effective in COPD exacerbations.
56. A 35-year-old male presents with acute onset of pleuritic, right-sided chest pain for 2 days
associated with mild dyspnea. He has no fever, cough, or chills. He reports no past medical
history except smoking. Vital signs are 98.9°F, 105, 20, 120/70, 97% RA. Examination is
unremarkable except for tachycardia. Electrocardiogram (EKG) shows sinus tachycardia without
any other abnormalities, and chest x-ray reveals a small right-sided pleural effusion with clear
lung fields. Which of the following is the next best step in management?
B. Order D-dimer
Answer B. The patient has pleuritic chest pain, shortness of breath, and sinus tachycardia,
indicating that he is at some risk for PE. The presence of pleuritic chest pain in the setting of a
pleural effusion in a young patient without signs of CHF, pneumonia, or malignancy is most
commonly due to PE. D-dimer can be used as a risk stratification test for PE. Blood cultures are
rarely helpful in previously healthy patients with dyspnea even with established diagnosis of
pneumonia. Gastrointestinal cocktail is only helpful for therapeutic management of
gastroesophageal reflux disease or peptic ulcer disease and should never be used as a diagnostic
maneuver. The differential diagnosis in this patient includes atypical pneumonia, such as from
pg. 111
Mycoplasma species. Amoxicillin–clavulanic acid does not adequately cover atypical organisms
and should not be used in this case, due the absence of crackles, fever, or focal infiltrate on
chest x-ray. Right-sided EKGs are helpful in evaluation of a right-sided MI to assess for right
ventricular involvement.
57. A 65-year-old asymptomatic man presents with the EKG as shown in Figure below. Which of
the following is the most likely associated condition?
A. Hypertriglyceridemia
B. Rheumatic fever
D. Pulmonary embolism
E. Hyperkalemia
Answer C. The EKG demonstrates multifocal atrial tachycardia (MAT), which is associated most
commonly with COPD, followed by congestive heart failure, hypokalemia, and
hypomagnesemia. There are P waves of at least three different morphologies associated with an
irregular tachycardia. Management involves treatment of the underlying condition. Patients
with MAT rarely have hemodynamic compromise as a result of their dysrhythmia.
58. Cancer of which of the following organs is the most common cause of superior vena cava
(SVC) syndrome?
pg. 112
A. Breast
B. Lung
C. Testicle
D. Colon
E. Thyroid
Answer B. Historically, the majority of cases of SVC syndrome (obstruction of the SVC) were due
to syphilitic aortic aneurysms, tuberculosis, and cancer. Today they are primarily due to deep
venous thrombosis of the SVC in patients with an indwelling catheter such as a PICC line or
Port-a-Cath. Many of these patients have such devices because of cancer treatment, but their
cancers are not the primary cause of SVC obstruction. Among neoplastic causes, lung cancer is
the most common. Breast and testicular cancer are less common causes of SVC syndrome. Colon
and thyroid neoplasms are uncommon causes. Symptoms include facial and upper extremity
edema, shortness of breath, cough, and chest pain. Diagnosis is made by physical examination,
chest x-ray, and advanced imaging such as echocardiography, CT, and/or MRI. Management has
traditionally involved radiation therapy, but chemotherapy and surgical evaluation have
potential roles in the acute treatment as well.
59. Which of the following is the best adjunct to the physical examination in assessing the
severity of an asthma attack?
C. Chest x-ray
Answer B. The PEFR (in L/second) and the FEV1 (in L) are both valuable adjuncts in assessing the
severity of airflow obstruction. Because of its portability and ease of application, however, PEFR
is much more easily measured and is more useful for EPs. Although the absolute value of the
PEFR may be useful, it is most useful in comparison to a patient’s typical best (as a percentage).
A patient who generates a PEFR <50% of his or her typical best has severe airflow obstruction.
The utilization of ABG determination widely varies in clinical practice. As a general rule, patients
in whom oxygenation can’t be reliably assessed due to an erratic or absent pulse oximetry
waveform should receive an ABG. Repeat ABGs are generally not needed to determine whether
a patient is improving or deteriorating. Chest x-rays are not generally useful in patients with
asthma exacerbations. They do not reflect the severity of the exacerbation. They are generally
pg. 113
useful only for those patients in whom pneumonia, or a complication (e.g., pneumothorax) is
suspected, or in those patients who remain refractory despite optimal therapy. Neither
continuous cardiac monitoring, nor routine EKGs are useful in routine asthma exacerbations.
Both of these may be useful, however, in older patients with coexisting cardiac disease. Most
patients have pulse rates between 90 and 120, and only 15% exceed this value.
60. A 22-year-old male basketball player presents to the ED after developing sudden shortness
of breath and a painful sensation on the left side of his chest, which worsens with breathing.
Initial chest x-ray reveals a small left apical pneumothorax. The patient’s vitals are 128/72, P 95,
RR 22 and shallow, and SaO2 of 97% on RA. The best course of action is:
A. Administer supplemental O2 and admit the patient to the hospital for 24-hour observation
with repeat chest films q6h.
B. Administer supplemental O2, observe the patient in the ED, and repeat a chest film in 3 to 6
hours.
C. Discharge the patient home with routine follow-up the next day with his Pneumocystis carinii
pneumonia (PCP).
pg. 114
61. A 50-year-old female with no past medical history presents with acute onset of fever, chills,
shortness of breath, and cough. She has crackles in her right middle lung field and chest x-ray
shows right middle lobe consolidation. Vital signs in the emergency department (ED) are normal.
Which of the following is the most appropriate next step in management?
A. Blood cultures
E. Antibiotics
Answer E. The patient has both clinical and radiographic evidence of CAP. Treatment with
antibiotics should be initiated as soon as possible. Routine blood cultures and sputum Gram
stain in immunocompetent patients with CAP are not helpful and rarely change management.
CT scan of chest is not indicated unless PE is suspected or the chest x-ray does not show
consolidation despite the presence of strong clinical signs of pneumonia. Arterial blood gas is
not helpful in the routine evaluation of pneumonia in the absence of suspicion of hypercarbia.
62. In most patients diagnosed with community-acquired pneumonia, which of the following
contributes most to a patient’s Pneumonia Severity Index (PSI) score?
B. Cirrhosis
C. Age
D. Sex
E. Pulse >125
Answer C. The PSI is a scoring system which was first developed in a retrospective, observational
study by Fine et al. in 1997. The aim of the study was to develop a rule to identify patients with
low 30-day mortality from community-acquired pneumonia. Calculating the PSI is a two-step
process. In step one, the physician determines if a patient has any “high-risk” criteria, the
presence of which is independently associated with mortality. These criteria include patients
older than 50, any of five comorbid diseases (neoplastic disease, congestive heart failure,
cerebrovascular disease, renal disease, and liver disease), and five physical examination findings
(altered mental status, pulse ≥125 per minute, respiratory rate ≥30 per minute, systolic blood
pressure <90 mm Hg, and temperature, <35°C or ≥40°C). Patients without any of these criteria
are classified as risk class I. All other patients are assigned to risk classes II, III, IV, or V based on
pg. 115
the number of points calculated in step 2 of the PSI. In step 2, patients are assigned points for
each risk factor that they have. Of all factors, age is the most heavily weighted in calculating the
PSI.
63. A 78-year-old male nursing home resident is brought to the ED with fever and hypoxia. He
has a history of dysphagia and dysarthria secondary to stroke and receives tube feedings and
small spoon feeds. Approximately 1 week before admission, he had an episode of vomiting and
appeared to choke on some of the regurgitated contents. His chest x-ray now shows bilateral
lower lobe infiltrates. He has a fever of 101.8°F, a WBC of 16,000, and a pulse oximetry of 92%
on room air. What is the most likely diagnosis and appropriate initial treatment?
Answer E. This patient is at risk for aspiration pneumonia by virtue of his decreased airway
protection and tube feeds. Although there is no definitive way to diagnose aspiration
pneumonia, this patient’s history of witnessed vomiting with choking in the setting of a patient
at risk for aspiration is virtually diagnostic for aspiration. Aspiration pneumonitis is caused by an
inflammatory response in the lungs after the aspiration of low pH gastric contents. Aspiration
pneumonitis develops within hours of aspiration, and its severity is directly related to the
volume and pH of the aspirated material. Treatment is supportive, and antibiotics and steroids
are not useful. However, episodes of aspiration pneumonitis may become secondarily infected,
resulting in aspiration pneumonia. More commonly, aspiration pneumonia develops without
significant preceding pneumonitis. Such infections are due to the aspiration of oropharyngeal
flora and are most commonly polymicrobial. While anaerobes have classically been implicated in
cases of aspiration, anaerobic antibiotic coverage may actually only be needed in patients with
very poor oral hygiene, putrid sputum, or evidence of necrotizing pneumonia or lung abscess
chest x-ray. Metronidazole is a specific therapy for anaerobic infections, but its use as
monotherapy in aspiration pneumonia is associated with a high failure rate. The
fluoroquinolones are broad-spectrum agents and they achieve good tissue levels. However, they
do not need to be combined with corticosteroids, for which there is no role in the setting of
aspiration, despite their wide use. Aztreonam does not have any gram-positive coverage, so it
should not be used as monotherapy. First-line antibiotic therapy in nursing home residents with
aspiration pneumonia may include piperacillin–tazobactam, levofloxacin, or ceftazidime.
64. Which of the following is the most common symptom seen in pulmonary tuberculosis (TB)?
pg. 116
A. Weight loss
B. Night sweats
C. Shortness of breath
D. Chest pain
E. Hemoptysis
65. Pulmonary infections with which of the following may be transmitted from person to
person?
A. Coxiella burnetii
B. Y. pestis
C. Histoplasma capsulatum
D. Francisella tularensis
E. Bacillus anthracis
pg. 117
aggressive disease and many patients progress rapidly to septic shock and death without early
treatment. Initially, patients may complain of typical symptoms of pneumonia, and their chest
x-rays frequently show alveolar infiltrates. Chest x-rays may also reveal a picture consistent with
acute respiratory distress syndrome (ARDS), with diffuse bilateral patchy infiltrates and
cavitation. None of the other agents demonstrate person-to person transmission.
pg. 118
CPR and anesthesia
Ventilation technics
Chapter includes
CPR and post CPR
RSI and planning intubation
C-pap , Bi-pap
Anesthesiology
1. Delayed sequence intubation (DSI) is best used for which of the following patients?
A. An elderly chronic obstructive pulmonary disease (COPD) patient with hypoxia, CO2
retention, and excessive somnolence
B. A 23-year-old trauma patient with hypoxia and a large hemothorax after a rollover motor
vehicle collision (MVC)
C. An agitated asthmatic patient, who persistently pulls off his oxygen mask exclaiming he can’t
breathe
E. A middle-aged congestive heart failure (CHF) patient with hypoxia and severe hypertension.
Answer C. Delayed sequence intubation (DSI) is a strategy to more safely secure a definitive
airway in agitated, delirious patients whose agitation makes preoxygenation suboptimal or
impossible. Historically, rapid sequence intubation (RSI) was emergently applied to all patients in
need of a definitive airway. However, in hypoxic patients who resist preoxygenation efforts due
to delirium or agitation, DSI offers an opportunity to safely preoxygenate patients prior to
administering paralytics in preparation for intubation. To achieve this, serial doses of ketamine
are used, typically starting at the low end of the scale, e.g., 1 mg/kg IV bolus followed by serial
pg. 119
0.5 mg/kg boluses until dissociation is achieved. Ketamine is an ideal agent because it doesn’t
depress airway reflexes or respiratory drive. Once the patient is dissociated and more
compliant, preoxygenation can occur using a nonrebreather or a nasal cannula at 15 L/minute
plus a noninvasive mask attached to a ventilator to provide CPAP at 5 to 15 mm Hg. As
summarized by its creator, Scott Weingart, MD, another perspective is to think of DSI as
procedural sedation, in which the procedure is effective preoxygenation. Once the patient is
adequately preoxygenated, a paralytic agent can be given and intubation can proceed. Once
dissociated, some patients experience such a profound improvement in their respiratory
mechanics that DSI will obviate the need for intubation altogether.
2. A 52-year-old previously healthy female collapses while coaching her daughter’s soccer team
and bystanders initiate chest compressions. When EMS arrives, they find the patient in
ventricular fibrillation. EMS immediately defibrillates the patient and initiates ACLS. They also
placed a temporary supraglottic airway. After following ACLS algorithms for 14 minutes
including three defibrillation attempts, the patient experiences a return of spontaneous
circulation (ROSC). Shortly after arrival in the emergency department (ED), a definitive airway is
obtained and placement is confirmed by x-ray. The patient is noted to be comatose, with a GCS
of 3, and has the following vital signs: T 95.8°F, P 115, BP 79/40 (mean arterial pressure [MAP] =
53), SaO2 97% on the ventilator. Which of the following is true?
A. The patient is not a candidate for therapeutic hypothermia because her GCS score is too low.
B. The patient should undergo immediate cooling with a specialized intravenous cooling
catheter.
C. The patient is not likely to benefit from cooling because her core temperature is already low.
D. Vasopressors are needed to raise her blood pressure prior to the initiation of cooling.
E. Shivering is an expected, benign side effect of therapy, and does not require treatment.
pg. 120
vasopressors prior to the initiation of cooling. Ongoing hypotension is a contraindication for
therapeutic hypothermia. Other contraindications to therapeutic hypothermia include patients
with a core temperature <30°C (86°F) upon arrival, patients with poor baseline mental status,
terminally ill patients, pregnancy (relative), traumatic arrest (relative), and patients with
inherited clotting disorders. Shivering is an expected and common side effect of therapy but is
not benign. Shivering leads to increased oxygen consumption, an elevated heart rate, increased
work of breathing, and a generalized increase in the stress response. The heat generated from
shivering can also impede cooling. Shivering should be managed with analgesia and sedation,
with paralytics used if all other methodologies fail.
3. A 56-year-old female presents with 1 hour of generalized hives, throat fullness, and cough.
She is allergic to certain nuts and just ate a brownie that she thinks had nuts in it. She
immediately used the Epi-pen that she carries with her and took 50 mg diphenhydramine orally,
but she still feels throat fullness. Vital signs are 99.9°F, 110, 22, 132/75, 94% RA. She appears
anxious and has dysphonia. On oropharyngeal examination, you cannot see the inferior border
of her moderately edematous uvula and she is spitting her saliva out. She has wheezes
bilaterally. Which of the following is the most appropriate next step in management?
A. Chest x-ray
Answer E. The patient has a severe allergic reaction with signs of airway compromise. While the
patient’s presentation does not include a rash, hypotension, or GI symptoms, management is
identical to patients with anaphylaxis and should include prompt treatment with epinephrine.
Epinephrine can be repeated every 5 minutes if needed, though few patients need more than
two epinephrine doses. When more than two doses are needed, an epinephrine infusion should
be considered. In addition, antihistamines, beta-agonist nebulizers, and corticosteroids may
help. In this case, the patient is at significant risk for airway collapse and requires repeat
epinephrine and preparation for immediate airway control. Chest x-ray and arterial blood gas
measurements are time consuming and produce unacceptable delays with minimal diagnostic
yield. FFP is reported as a possible treatment for angiotensin-converting enzyme (ACE) inhibitor
angioedema. FFP does not play a role in allergic reactions. Noninvasive positive pressure
ventilation can provide ventilatory support but is contraindicated in the face of impending
airway obstruction due to edema.
pg. 121
4. A 27-year-old female with pneumonia requires tracheal intubation. Which of the following is
the best position to maximize the duration of safe apnea during intubation?
A. Trendelenburg
B. Reverse Trendelenburg
D. Supine
Answer E. Multiple studies after demonstrated a longer time to desaturation in patients placed
in a head upright (head of bed elevated ≥20 to 30 degrees) and chin forward position (external
auditory meatus aligned with the sternal notch). In immobilized patients who can’t bend the
spine can be placed into reverse Trendelenburg, in which the head of the bed is elevated at least
30 degrees higher than the foot of the bed.
5. A 68-year-old male with a history of chronic obstructive pulmonary disease (COPD) presents
to the emergency department (ED) with worsening dyspnea, cough, and subjective intermittent
fevers. He tells you that he spent 1 week in the intensive care unit (ICU) 6 months ago after
being intubated for a similar episode and states “I don’t ever want to be intubated again.” You
discuss the use of noninvasive positive pressure ventilation (NIPPV) with him (bilevel positive
airway pressure [BiPAP]) and he is agreeable. With which of the following comorbidities is BiPAP
safe to use?
A. Excessive secretions
B. Decreased sensorium
C. Severe hypertension
D. Midfacial trauma
E. Uncooperative patient
Answer C. NIPPV has revolutionized the treatment of COPD, cardiogenic pulmonary edema, as
well as neuromuscular disease (e.g., myasthenia gravis). Contraindications for NIPPV include
noncompliance (which is the most common reason for treatment failure), midfacial trauma
(preventing an appropriate fit for the mask), excessive secretions or retention of secretions,
decreased sensorium with absent cough and pharyngeal reflexes, recent gastric surgery
(because of possible gastric distension), and vasopressor-dependent hypotension. Hypertension
does not affect, nor is it affected by NIPPV.
pg. 122
6. The most common cause of death in recipients of a solid-organ transplant is:
B. Infection
C. Drug toxicity
D. Organ rejection
Answer B. Due to the need for powerful immunosuppressant agents, all solid-organ transplant
recipients are at increased risk for infection. Infections are divided into three time
periods—those occurring within the first month of transplantation, those occurring between 1
and 6 months after transplantation, and those occurring >6 months after transplantation.
Nosocomial agents are prominent in the first month, although cytomegalovirus (CMV) is the
most prevalent infection between 1 and 6 months (particularly CMV pneumonitis).
7. A 24-year-old male presented to the ED with an acute asthma exacerbation that was
refractory to optimal treatment. A few minutes after intubating him, the resuscitation nurse
tells you that the patient is waking up and “fighting the vent.” Your best initial course of action is
to
A. Paralyze the patient with cisatracurium to ensure that he “synchronizes” with the vent.
B. Give the patient a bolus of IV propofol and place the patient on a propofol drip.
D. Paralyze the patient with succinylcholine to ensure that he “synchronizes” with the vent.
E. Check a stat metabolic panel to ensure that the patient is not hyperkalemic.
Answer B. Inadequate sedation of recently intubated patients is a common problem in the ED.
Although the common initial response to the waking patient is to infuse paralytics, such action is
typically not necessary. Usually, all that is required is adequate sedation with propofol or with
benzodiazepines (such as midazolam) with or without narcotic agents such as morphine or
fentanyl. It is critical to provide adequate analgesia, as many patients who require mechanical
ventilation have suffered significant, painful traumatic injuries, which may be the cause of their
distress. Paralytics should be reserved for the patient with persistently high plateau pressures
despite optimal ventilatory management and sedation. In such patients, paralysis ensures that
the patient’s intrinsic respiratory effort will not interfere with the ventilator’s attempts at
delivering breaths. However, because paralysis increases the risk of aspiration and subsequent
pg. 123
pneumonia, paralytics should not be routinely used. Ketamine would be effective but is short
acting and would need to be followed by another agent. Hyperkalemia is not likely to cause the
patient’s agitated mental status.
8. You decide to intubate a 54-year-old male with severely altered mental status. Your favorite
induction agent for rapid-sequence intubation is etomidate. The respiratory therapist asks you
whether etomidate is safe for this patient because of the potential for septic shock. You tell him:
A. “Etomidate can cause increased intracranial pressure and laryngospasm, but this is
uncommon.”
B. “Etomidate transiently reduces adrenal function but doesn’t cause increased mortality.”
C. “Good point, I’ll try thiopental instead—I’ve never tried it and this is a good time to test it
out.”
D. “Etomidate has no side effects and should be used as an induction agent for all patients.”
E. “You should go to pharmacy school, medical school, PA school, or nursing school before
providing advice on pharmacotherapy.”
Answer B. Etomidate does transiently reduce adrenal function (<24 hours in duration) but this
decrease has not been shown to increase mortality. Several studies raised the possibility that
the reduced adrenal function could cause increased mortality in septic shock patients but
subsequent analyses showed that this was not true. Ketamine, not etomidate, can cause
increased intracranial pressure and laryngospasm. Thiopental is a reasonable induction agent for
rapid sequence intubation, but attempting to use a new drug for the first time is not a sound
strategy for a critically ill patient when other alternatives are available. Etomidate does have
side effects such as myoclonus, vomiting, and reduced adrenal function, but none of these is
associated with increased mortality. Choice E lies well outside the normal bounds of
professionalism and violates basic tenets of patient safety organizations.
B. Epinephrine
C. Vasopressin
D. Defibrillation
E. None of these
pg. 124
Answer E. Mechanical device CPR has not been shown to increase survival to hospital discharge
over manual CPR. It may increase the ability of first responders to safely carry out other tasks,
but no well-designed study has demonstrated an increase in patient survival. Epinephrine does
not increase survival to hospital discharge in patients with sudden cardiac arrest—it may
increase return of spontaneous circulation. Vasopressin has been removed from the 2015 AHA
guidelines for out-of hospital cardiac arrest management as it is no more effective than
epinephrine and it unnecessarily increases complexity of management. Defibrillation is not
effective in patients with pulseless electrical activity.
10. A 44-year-old male is brought to the ED in cardiac arrest due to ventricular fibrillation. He
responds to a defibrillation attempt with return of spontaneous circulation, but he is still
comatose. You control his airway with endotracheal intubation. You decide to initiate
therapeutic hypothermia. Which of the following is thought to be a beneficial mechanism in
therapeutic hypothermia?
C. Reduction in pH
A. Analgesia
B. Anesthesia
C. Antipsychosis
pg. 125
D. A and B
E. A, B, and C
A. Procainamide
B. Amiodarone
C. Metoprolol
D. Lidocaine
E. Verapamil
Answer D. Phenytoin and lidocaine are class IB antidysrhythmic agents. Class I agents have their
primary effects on fast sodium channels and slow down action potential depolarization and
conduction. These effects are greatest in class IC agents (flecainide, propafenone), moderate in
class IA agents (procainamide, quinidine), and least in class IB agents. Unlike classes IA and IC,
class IB agents shorten repolarization time and action potential duration. Phenytoin is not used
in the ED as an antidysrhythmic, because it may cause important cardiac conduction
abnormalities in patients who take the drug for its antiepileptic effects. Loading phenytoin
intravenously can occur at a rate no greater than 50 mg/minute, as its diluent, propylene glycol,
can cause dysrhythmias or hypotension when given too quickly. Fosphenytoin lacks this diluent
and may be given more quickly.
13. You are working in the ED when a 74-year-old female with chronic obstructive pulmonary
disease (COPD) presents with an acute COPD exacerbation. You start her on bilevel positive
airway pressure (BiPAP) at a rate of 10, an inspiratory positive airway pressure (IPAP) of 10 mm
Hg, and an expiratory positive airway pressure (EPAP) of 4 mm Hg. Twenty minutes later, the
pg. 126
patient’s oxygenation has not improved. Which of the following changes would most likely
increase this patient’s oxygenation?
E. Decrease the patient’s EPAP from 4 to 2, and decrease the IPAP from 10 to 5
Answer D. BiPAP ventilation emulates pressure support mechanical ventilation with positive end
expiratory pressure (PEEP) in which the machine cycles between two different pressure levels
during inspiration and expiration. This is in contrast to continuous positive airway pressure in
which the pressure is the same throughout inspiration and expiration (i.e., IPAP = EPAP). When a
patient’s oxygenation status is not improving, the two main adjustments that can be made are
to increase the inspired FIO2 and to increase the PEEP. With BiPAP, there is a limit to the
amount of oxygen that can be applied because the oxygen is not being delivered to the lower
airways, as is the case with mechanical ventilation. However, the EPAP (i.e., PEEP) can be
increased in order to recruit additional alveoli (by stenting them open with increased pressure at
the end of expiration), which increases the surface area for gas exchange and hopefully results
in improved oxygenation. In order to maintain the PPD during inspiration, the IPAP may be
concomitantly increased (to ensure ongoing adequate ventilation). If these measures fail, the
patient requires intubation and mechanical ventilation.
14. A 67-year-old female with a history of severe heart failure s/p left ventricular assist device
placement presents by EMS to the emergency department (ED) after a witnessed collapse in the
hospital cafeteria. The patient was brought to the ED emergently after a “Code Blue” was called.
Which of the following is true?
B. The patient may not have a palpable pulse even when awake and alert
C. Ventricular arrhythmias do not impact heart function because patients are dependent on a
machine for circulation
Answer B. Left ventricular assist devices (LVADs) are reserved for patients with end-stage heart
failure refractory to conventional treatments. They are typically used as a bridge to transplant
pg. 127
but have recently been increasingly used as definitive treatment, so-called “destination
therapy.” LVADs are continuous flow rather than pulsatile devices so patients most often do not
have palpable pulses. As a result, blood pressure measurements with either an automated or
manual cuff are unreliable. Instead of conventional blood pressure measures, LVAD patients are
managed based on mean arterial pressure (MAP). The goal MAP is typically 60 to 90 and is most
easily approximated by listening with a Doppler over the brachial artery as a blood pressure cuff
is slowly deflated. The pressure at which the first audible sounds occur is the MAP and is used to
guide management. When a patient with an LVAD presents with a depressed level of
consciousness, the best approach remains to focus on the ABCs. In the case of circulation,
providers should auscultate over the chest wall to listen for the “hum” of the LVAD motor,
indicating that it is working. If it is, the MAP can be assessed as above. CPR should be avoided
unless there are no signs of life (apneic, MAP of 0) because CPR may dislodge the LVAD cannula
resulting in uncontrolled hemorrhage. Ventricular fibrillation and tachycardia are frequent in
patients with LVADs and defibrillation is needed to restore normal rhythm, especially since the
right ventricle is unassisted and depends on normal conduction to pump normally
15. A 75-year-old male is brought to the ED in out-of-hospital cardiac arrest. Which of the
following treatments will be most effective at improving the patient’s likelihood to be
discharged from the hospital in good neurologic condition?
A. Vasopressin
B. Low-dose epinephrine
C. High-dose epinephrine
Answer E. The only treatments that improve survival with good neurologic outcome in out-of
hospital cardiac arrest are defibrillation, high-quality chest compressions, and post-resuscitation
targeted temperature management. Importantly, no medication is able to accomplish this
outcome when given on a protocolized basis. Patients with known hyperkalemic cardiac arrests
may benefit from emergent calcium and bicarbonate therapy. Epinephrine does improve return
of spontaneous circulation in out-of-hospital cardiac arrest, but it has not been shown to
improve survival with good neurologic outcome.
16. A 57-year-old male presents in out-of-hospital cardiac arrest with pulseless electrical activity
(PEA). Prehospital advanced cardiac life support, including CPR and administration of several
doses of epinephrine IV, has been performed for 20 minutes. His monitor still reveals PEA.
Which of the following is the next best step in the management of this patient?
pg. 128
B. Defibrillation at 150 J
C. Vasopressin IV
Answer E. The patient has an extremely poor prognosis, given the persistence of PEA despite
heroic efforts. An end-tidal CO2 level of <10 mm Hg at 20 minutes of resuscitation predicted
death with 100% accuracy. Cardioversion, either synchronized or unsynchronized (aka
defibrillation), has no role in the management of PEA. Vasopressin does not offer any benefit
over epinephrine IV and should no longer be used in the management of cardiac arrest. CT brain
is not indicated in patients with cardiac arrest—however, if the patient were to have return of
spontaneous circulation, then CT brain may be considered to evaluate for the possibility of
intracranial hemorrhage.
pg. 129
Gastroenterology and
surgery
Emergency
➢ Chapter includes:
GI and surgical MCQ’s
Exception GI and surgical infectious disease including
diarrheal disease are in infectious disease 1
pg. 130
1. A 77-year-old male with Parkinson disease is brought to the hospital with obstipation. His
abdomen is distended and mildly tender with decreased bowel sounds. His abdominal x-ray is
shown below . Which of the following is the most likely diagnosis?
B. Viscus perforation
C. Sigmoid volvulus
D. Swallowed air
Answer C. The image indicates sigmoid volvulus. Abdominal radiography lacks sensitivity for
small bowel ischemia and signs such as thumbprinting and pneumatosis are rare. Viscus
perforation would be suspected if there were free air seen under the diaphragm, but abdominal
radiography is incompletely sensitive for evaluation of perforation. Swallowed air would be
unlikely to cause abdominal distention and obstipation. Gastric outlet obstruction might not
demonstrate any findings on x-ray. Of all the answer choices, sigmoid volvulus is the only
diagnosis that can be made solely relying on abdominal radiography.
pg. 131
2. A 68-year-old female presents with a 2-day history of left lower quadrant pain. CT reveals
diverticulitis. Which of the following is true?
B. Avoiding nuts, seeds, and corn has not been shown to decrease recurrence.
C. Patients younger than 40 years with diverticulitis should have resection of the diseased
segment of colon.
Answer B. The recurrence rate after a single episode of uncomplicated diverticulitis is 20% to
30%. Furthermore, a high-fiber diet may help prevent further episodes. There is no evidence
that avoiding nuts, corn kernels, popcorn, or seeds is associated with an increased rate of
recurrence, despite the fact that patients are commonly advised against ingesting these
substances. Young patients with diverticulitis are a special population because they tend to have
more aggressive disease and the recurrence rate of diverticulitis is higher. Although resection of
the disease segment remains elective, some authors recommend resection in all young patients
due to their higher risk of recurrence. CT is the study of choice in the ED because it reliably
visualizes the site of inflammation and is very useful for detecting various complications (e.g.,
abscesses, perforation, fistulas). Colonoscopy is useful to exclude colon cancer but is deferred
until the acute illness is treated. The mortality rate of hospitalized patients with diverticulitis is
1% to 6% for those requiring only medical management and 12% to 18% for those requiring
surgery.
3. A 62-year-old female presents with left lower quadrant abdominal pain. You suspect acute
sigmoid
A. Vomiting
B. Hematochezia
C. Dysuria
E. Anorexia
Answer D. Patients with acute sigmoid diverticulitis usually have pain in the left lower quadrant
as well as a change in bowel habits, either constipation or diarrhea. Vomiting, hematochezia,
pg. 132
dysuria, and anorexia are seen in lower frequencies in acute diverticulitis. Anorexia is
particularly sensitive for acute appendicitis if associated with right lower quadrant pain.
4. A 48-year-old male presents to the emergency department with abdominal pain 4 months
after an uncomplicated Roux-en-Y gastric bypass. The patient’s initial postoperative course was
uncomplicated and he has been losing weight as expected. However, over the past few weeks,
he has noted intermittent, crampy, and diffuse abdominal pain that appears unrelated to eating.
In the ED, his vital signs, blood tests, and CT are normal. What’s the likely explanation for the
patient’s symptoms?
A. Drug seeking
B. Internal hernia
C. Gastrogastric fistula
E. Cholelithiasis
Answer B. Roux-en-Y bypass creates potential areas of herniation in the mesentery which are
closed at the time of surgery. Despite closure, some patients experience intestinal herniation
through mesenteric defects, resulting in abdominal pain. Herniation is often intermittent and
difficult to diagnose by CT. While the classic “mesenteric swirl sign” may be present, the
majority of patients may have a normal CT. Thus, patients with otherwise unexplained
abdominal pain status post bypass surgery may need repeat exploration to exclude an internal
hernia. Gastrogastric fistulas most often occur in patients with a stapled but undivided stomach
(the fistula forms between the remaining stomach “pouch” and the undivided remnant), a
procedure that was performed in early gastric bypass procedures. In contrast to common
beliefs, gastroesophageal reflux is not a common complication of gastric bypass surgery.
Cholelithiasis is a common complication of gastric bypass but the patient’s history is not
suggestive of biliary colic as his symptoms are not tied to eating.
5. A 24-year-old male presents with abdominal fullness. He is very nervous, but in no acute
distress. Vital signs and physical examination are normal. An obstructive radiography series is
ordered. Which of the following is the most appropriate next step in management (x-ray shown
below)?
A. MRI abdomen
B. Surgery
C. Polyethylene glycol
D. NG aspiration
pg. 133
E. Endotracheal intubation
C. Even in the third trimester, most pregnant women still have pain in the right lower quadrant.
D. Due to anatomic changes, appendicitis in pregnant women occurs most often in the third
trimester.
pg. 134
E. Fetal abortion occurs in 50% of pregnant women with perforated appendicitis.
Answer C. Pregnant women develop appendicitis at the same rate as not pregnant women. Due
to nearby gynecologic structures, women with appendicitis are more frequently misdiagnosed.
As many as 33% of women with appendicitis are initially misdiagnosed, and as many as 45% of
women with symptoms of appendicitis are found to have a normal appendix during surgery.
Although CMT is more common in gynecologic diseases, as many as one-fourth of women with
appendicitis have CMT upon physical examination. Even in the third trimester, most pregnant
women still have pain in the right lower quadrant. In pregnant women, appendicitis occurs
slightly more often in the second trimester, although the reasons for this are not known. Fetal
abortion complicates perforated maternal appendicitis in 20% of cases.
7. A 67-year-old female with diabetes, hypertension, and a history of an aortoiliac bypass graft
presents with abdominal pain and dark, “funny-smelling” stools. She states the symptoms
started 2 days ago and seem to have gone away as she had a “nearnormal” bowel movement
this morning and no longer has pain. On examination, she has guaiac positive, dark brown, but
not melenic stool. Which of the following must be excluded as a cause of hemorrhage?
C. Aortoenteric fistula
D. Duodenal ulcer
E. Ischemic colitis
Answer C. This patient has an aortoenteric fistula until proven otherwise. The classic
presentation is abdominal pain and GI bleeding that resolves spontaneously (the so-called
herald bleed) in a patient with a prior history of aortic graft. The initial bleeding is thought to
stop when splanchnic pressure drops and allows adequate clot to form. However, the initial
bleed is often followed by a massive and often fatal hemorrhage days or possibly weeks later.
The fistula is caused by S. aureus or Escherichia coli infection of the prior graft and typically
forms between the aorta and the distal duodenum. Therefore, all patients with a history of
aortic graft and GI bleeding should undergo esophagogastroduodenoscopy to search for a fistula
in the distal duodenum. Most of these patients, however, have an alternative, more common
cause of GI bleeding.
8. A 33-year-old female presents with a chief complaint of dysphagia. She feels a sensation of
foods, particularly solids, getting “stuck” in her chest and she sometimes needs to raise her arms
above her head or straighten her back after eating to help things pass. She also complains of
pg. 135
intermittent substernal burning chest pain. Her doctor has been treating her for
gastroesophageal reflux disease (GERD) for the last 9 months but she seems to be getting worse.
What is the likely cause of her symptoms?
A. Nutcracker esophagus
C. Schatzki ring
D. Achalasia
E. Zenker diverticulum
Answer D. Achalasia is an esophageal motility disorder due to failure of the lower esophageal
sphincter to relax and a complete absence of esophageal peristalsis. It affects men and women
equally and presents between the third and fifth decade. Dysphagia is the most common
symptom. All patients have difficulty with solid foods but two-thirds of patients describe
dysphagia with liquids as well. Patients may stand after eating, raise their arms above their
head, or straighten their back to increase esophageal pressure and help esophageal emptying.
Patients with achalasia have symptoms for an average of 2 years before diagnosis and they are
often treated for GERD due to the presence of burning chest pain. Diffuse esophageal spasm
and nutcracker esophagus are hypermotility disorders, resulting in exceedingly strong
esophageal peristaltic contractions. The most common complaint is chest pain, although the
presence of dysphagia is less common and more variable. Schatzki ring is a fibrous band-like
stricture in the distal esophagus that is present in up to 15% of the normal population. Patients
who develop symptoms typically present with acute esophageal obstruction.
9. Which of the following oral medications is most likely to cause serious damage to the
esophageal mucosa when swallowed?
A. Amoxicillin
B. Potassium chloride
C. Metoprolol
D. Lovastatin
E. Hydrochlorothiazide
Answer B. Certain oral medications cause severe esophageal irritation when swallowed—
doxycycline, tetracycline, aspirin, and potassium chloride. Of these, potassium chloride is the
most caustic, sometimes leading to esophageal perforation and penetration into the mediastinal
great vessels.
pg. 136
10. A 45-year-old female presents with intermittent epigastric pain and nausea for several
weeks. She denies fever, diarrhea, cough, and chest pain. She does not take medications, drink
alcohol, or smoke. She has had an extensive workup including CT scan of the abdomen and
pelvis, gall bladder ultrasound, and cardiac stress testing, all of which were normal. You suspect
gastritis. Which of the following is the most likely cause?
B. NSAID use
C. Alcohol use
E. Radon exposure
Answer A. Helicobacter pylori infection is the most common cause of gastritis and peptic ulcer
disease. H. pylori is a gram-negative bacillus that causes gastric and duodenal mucosal
inflammation. It is implicated in over half of all cases of gastritis and peptic ulcer disease.
Treatment is with combinations of proton pump inhibitors and antibiotics (e.g., lansoprazole +
amoxicillin +clarithromycin). The next most common cause of gastritis and peptic ulcer disease is
NSAID use. Although the patient denies history of medication use, NSAIDs should be specifically
queried about as patients frequently forget over-the-counter medications that they are taking.
Alcohol can cause gastritis but usually in moderate to large quantities and regular use.
Peppermint puts patients at risk for GERD exacerbations but does not usually cause frank
gastritis. Radon gas exposure is likely the number one source of background ionizing radiation
and puts patients at risk for lung cancer but does not usually directly cause gastritis.
11. A 44-year-old alcoholic man presents with hematemesis. Which of the following is the most
likely cause?
B. Gastric varices
C. Esophageal varices
D. Boerhaave syndrome
E. Arteriovenous malformation
Answer A. The most common cause of upper GI bleeding in both the general population and
alcoholics is peptic ulcer disease. When combined with NSAIDs and other ulcerogenic drugs,
alcohol can contribute to the loss of protective gastric mucosal lining. Treatment in alcoholics is
the same as for nonalcoholics, with special attention to electrolyte management, prevention of
pg. 137
alcohol withdrawal, and attention to seizure precautions. Gastric and esophageal varices occur
at a higher rate in those alcoholics who have a history of portal hypertension from cirrhosis.
Boerhaave syndrome refers to rupture of the esophagus from forceful vomiting. It usually occurs
in alcoholics and presents more often as sepsis and hypovolemia due to mediastinitis rather
than GI bleed. Arteriovenous malformation is a more common cause of lower GI bleed than
upper GI bleed.
12. A 43-year-old female presents with acute onset of upper abdominal pain. She has right
upper quadrant tenderness, but lacks rebound tenderness or a Murphy sign. She has no fever
and normal labs and her pain has resolved. Her abdominal ultrasound is shown below. Which of
the following is the next best step in management?
A. Intravenous antibiotics
Answer E. The patient’s gall bladder ultrasound shows multiple gallstones with shadowing.
Given the absence of fever, leukocytosis, or Murphy sign, there is no strong clinical evidence to
suggest acute cholecystitis. There is also no pericholecystic fluid and no mention is made of a
pg. 138
sonographic Murphy sign, both of which would be signs of acute cholecystitis. In a patient with
symptomatic gallstones, surgical evaluation can be undertaken to assess for need for
cholecystectomy—this does not need to be emergent (though urgent outpatient referral is
preferred to reduce repeated visits for biliary colic). Intravenous antibiotics are used for acute
cholecystitis. ERCP is preferred in cases of gallstone pancreatitis, when the lipase is elevated in
the presence of gallstones. Percutaneous biliary drains are not used often in the emergency
setting. HIDA scan is only used when ultrasound is equivocal for biliary pathology in cases with
strong clinical suspicion.
A. Baking soda
B. Salicylate
C. Corticosteroids
D. Insoluble fiber
Answer E. Sitz is a word that comes from the German word, sitzen, meaning “to sit.” A sitz bath
refers to any device that allows a patient to immerse only the perineum and buttocks in water
while draping the rest of their body outside the tub. While patients may add various
medications to the water, it is not recommended and there is no evidence that such additives
help. Sitz baths are thought to help a variety of perianal complaints, such as hemorrhoids,
because anal canal pressure decreases significantly in warm water (40°C) and blood flow
improves.
14. A 48-year-old female presents with chronic intermittent abdominal pain and diarrhea for 1
year. She reports having episodes of moderate abdominal cramping four to five times per week.
She has normal vital signs and an unremarkable abdominal examination. The patient has had
multiple negative diagnostic studies, including gall bladder ultrasound, CT abdomen/pelvis, MRI
abdomen/pelvis, stool studies, and colonoscopy. You suspect irritable bowel syndrome. Which
of the following is the best next step in management until she sees her primary care physician?
A. Ondansetron 4 mg po qd
C. Prednisone 20 mg po qd
pg. 139
Answer A. The diagnosis of irritable bowel syndrome (IBS) is made when patients have at least 3
days of abdominal pain per month over 3 months along with changes in stool frequency and
appearance. The patient likely has diarrhea-type IBS (IBS-D), as opposed to constipation-type
(IBS-C) or mixed-type (IBS-M). Ondansetron, a serotonin-3 receptor blocker that reduces
intestinal fluid secretion, is a reasonable starting treatment for IBS-D. Polyethylene glycol and
psyllium fiber therapies are reasonable treatments for IBS-C, but may worsen IBS-D, as they
promote stool production. Most patients with IBS do not have a significant inflammatory or
acute infectious component, so prednisone and ciprofloxacin are not indicated.
E. Gastric infarction and death occur in 80% of patients if not rapidly diagnosed and treated.
Answer D. Gastric volvulus is a rare disorder that chiefly occurs in older people and results from
twisting of the stomach about its long axis (organoaxial volvulus). Twenty percent of cases occur
in infants younger than 1 year due to congenital diaphragmatic defects. In older people, it is
frequently associated with large paraesophageal hiatal hernias. The classic triad is known as
Borchardt triad, and consists of severe epigastric pain and abdominal distension, vomiting, and
the inability to pass a nasogastric tube. If the diagnosis is suspected, the ED physicians should
attempt to pass a nasogastric tube because this occasionally reduces the volvulus. Due to its
redundant blood supply, gastric infarction is uncommon, even in delayed cases, occurring in as
many as 25% of cases.
16. A 45-year-old obese woman presents with right upper quadrant abdominal pain for several
weeks. Her pain is worse after she eats. Which of the following clinical features supports the
diagnosis of biliary colic?
D. Fever >101.5°F
pg. 140
Answer C. Biliary colic, also known as symptomatic cholelithiasis, refers to upper abdominal pain
caused by gallstones. The name “colic” is actually a misnomer—biliary colic usually causes
constant, steady pain occurring from 2 to 6 hours, often starting 1 to 2 hours after eating.
“Colicky,” spasmodic pain is rare. Radiation of pain to the right scapula or shoulder increases the
likelihood of biliary disease but is not at all sensitive. Biliary colic is no worse on eating protein,
but may be worse after eating fatty foods. Fever is more characteristic of acute cholecystitis or
other intra-abdominal inflammation rather than simple biliary colic. Antecedent use of NSAIDs
supports a diagnosis of gastritis, peptic ulcer disease, or viscous perforation rather than biliary
colic. Gallstone formation is, however, increased by fibrates, octreotide, oral contraceptive
agents, and total parenteral nutrition.
17. A 22-year-old Latino male who recently emigrated from Mexico presents to the ED with
fever, chills, abdominal pain, and intermittent nausea and vomiting. A CT scan of his abdomen is
shown in Figure below. Which of the following is true?
Answer B. Amebiasis occurs in 10% of the world’s population and amebic liver abscess is the
most common extraintestinal manifestation of the disease. Patients with amebic liver abscesses
more commonly have an acute presentation than patients with pyogenic liver abscesses.
pg. 141
Transmission occurs through the fecal–oral route and is usually due to contaminated water or
food products. Infections are always caused by Entamoeba histolytica and bacterial
superinfections are uncommon. Alkaline phosphatase is elevated in 75% of patients and
aminotransferases are increased in 50%. Elevated bilirubin levels are uncommon and reflect
biliary obstruction. In contrast to pyogenic liver abscesses, the treatment is nonsurgical, and
involves metronidazole 750 mg t.i.d. for 7 days. Following metronidazole, some authorities
recommend an additional course of a luminal amebicide such as iodoquinol, diloxanide furoate,
or paromomycin.
18. A 47-year-old male with a history of alcoholism presents with a chief complaint of jaundice,
right upper quadrant pain, and a low-grade fever. He had his last drink earlier in the day. Which
of the following is true?
A. His AST and ALT levels will be more than 10 times normal.
C. The patient’s fever suggests an infectious cause of his symptoms is most likely.
Answer E. This patient has acute alcoholic hepatitis. Although most cases of alcoholic hepatitis
are subclinical or asymptomatic, it may be life threatening. Patients presenting for treatment
are characterized by fever, right upper quadrant pain, jaundice, anorexia, and occasionally
nausea and vomiting. Physical examination most commonly reveals hepatomegaly (as opposed
to the shriveled, firm liver in patients with cirrhosis), jaundice, ascites, splenomegaly, and signs
of alcohol withdrawal. AST and ALT levels are only modestly elevated, typically remaining below
300 U/L, and the ratio of AST:ALT is usually greater than 2:1. Bilirubin levels are elevated but are
variable depending on the severity of the disease. Leukocytosis is common, with a mean WBC
count of 12,400 per mm3, whereas counts up to 20,000 are not uncommon. Fever is a common
finding but concomitant infection is uncommon. Bilirubin levels and the prothrombin time (PT)
have been classically used to stratify patients into low- or high-risk categories by calculating the
Maddrey discriminant function.
Levels over 32 correspond with severe disease and a 1-month mortality >50%.
19. A 44-year-old male presents with acute onset of severe abdominal pain. His abdomen is
tender throughout and he exhibits rebound tenderness. His x-ray is shown in Figure below .
Which of the following is the next best step in management?
A. Heparin
pg. 142
B. Azithromycin
C. Octreotide
D. Paracentesis
E. Surgical consultation
Answer E. The patient’s x-ray shows evidence of free air, indicating a likely viscus perforation.
Broad-spectrum antibiotics, NPO status, and surgical consultation are mandatory.
Anticoagulation would be contraindicated in someone who is likely going to the operating room
for bowel surgery. Azithromycin would not be adequate treatment for bowel surgery as it would
miss important gram negative and anaerobic pathogens. Octreotide is used for cirrhotic patients
with acute variceal bleeding and does not have a role in the management of viscus perforation.
Paracentesis should be used as a diagnostic maneuver to evaluate spontaneous bacterial
peritonitis in patients with cirrhosis.
20. A 25-year-old male presents with bright red blood after having a bowel movement. He
reports some itching and pain in his rectum. Examination reveals a nonthrombosed external
hemorrhoid. Which of the following is the most appropriate, long-term management option?
pg. 143
A. Hydrocodone–acetaminophen
B. Topical corticosteroids
D. Topical nitroglycerin
E. Topical nifedipine
Answer C. External hemorrhoids are best treated with nonpharmacologic therapies. The WASH
regimen (warm water, analgesia, stool softeners, and high-fiber diet) is probably the most
effective regimen and also the least likely to cause unacceptable side effects. Opioid pain relief
without accompanying stool softeners is not recommended as they will further exacerbate
constipation and straining. Topical corticosteroids can be used acutely (though little data exist
for true benefit), but long-term use can cause skin atrophy. Topical nitroglycerin is used in
patients with anal fissures but is not recommended in patients with non-thrombosed external
hemorrhoids. Topical nifedipine may be used in anal fissures and thrombosed external
hemorrhoids but is not recommended for non-thrombosed external hemorrhoids.
21. A 42-year-old male with a history of alcohol abuse presents with acute epigastric abdominal
pain. His workup reveals acute pancreatitis and he denies any prior history of pancreatitis. A CT
scan that was ordered as part of his workup is shown in Figure below. Which of the following is
true?
pg. 144
Answer E. The CT scan reveals a hypodense fluid collection surrounding an inflamed pancreas.
Acute fluid collection is associated with acute pancreatitis in 30% to 50% of patients. These fluid
collections resolve spontaneously in most patients. A pseudocyst is a fluid collection that
persists for 4 to 6 weeks and becomes encapsulated by a wall of fibrous or granulation tissue.
Unless pancreatitis is due to an infectious agent, acute infection of fluid collections or of
necrotic pancreatic tissue is uncommon.
Infection usually occurs within the first 2 weeks although abscess formation may not occur until
1 month after the acute infection. If an abscess does occur, urgent drainage is required.
22. A 34-year-old male presents with chronic intermittent abdominal pain and constipation. His
primary care physician and gastroenterologist gave him the diagnosis of irritable bowel
syndrome. Which of the following is true regarding irritable bowel syndrome?
Answer D. The diagnosis of irritable bowel syndrome (IBS) is made when patients have at least 3
days of abdominal pain per month over 3 months along with changes in stool frequency and
appearance. There are three main types of IBS, diarrhea-type IBS (IBS-D), constipation-type
(IBS-C), and mixed-type (IBS-M). IBS is a syndrome of lower pain thresholds in the ileum, colon,
and rectum, resulting in greater pain perception with even normal bowel stimuli. Affirmation of
the diagnosis and reassurance have been proven effective in improving outcomes in patients
with IBS. IBS is not associated with cancer, is not deforming or deteriorating, and doesn’t reduce
life expectancy. Surgery has no role in treatment of IBS, though patients with IBS often have a
higher incidence of abdominal surgery due to chronic symptomatology and seeking health care
services. IBS is not a type of thought disorder and antipsychotics have no current role in
management. Weight loss, significant rectal bleeding, fever, onset after age 50, and unexplained
anemia are not consistent with IBS and should be pursued further.
pg. 145
D. SMA embolism.
Answer D. Emboli in the SMA account for 50% of the cases of acute mesenteric ischemia. Emboli
usually originate in the left atrium or ventricle. Most emboli lodge just distal to a major branch
point, and greater than 50% of SMA emboli are located just distal to the origin of the middle
colic artery. Thrombosis of the SMA and mesenteric vein thrombosis each account for
approximately 15% of cases.
24. The most common complication of peptic ulcer disease (PUD) is:
A. Perforation
B. Gastric adenocarcinoma
C. Gastrointestinal bleeding
D. Obstruction
Answer C. PUD is the most common cause of upper gastrointestinal bleeding (UGIB), accounting
for half of all cases. Bleeding is the most common complication of PUD, and perforation is the
next most common. Gastric outlet obstruction occurs in only 2% of patients with known PUD.
pg. 146
from 10% to 50%. However, recent studies suggest that many patients with acalculous
cholecystitis develop symptoms and are diagnosed as outpatients, and they have a natural
history that is similar to calculous disease. Some of these patients may be miscategorized, since
microscopic calculi are evident in pathologic specimens, but there remain a large number of
patients with true acalculous disease who are otherwise healthy.
26. A 15-year-old male presents with progressively worsening groin and scrotal pain and
swelling over the last 8 hours. He noticed a bulge in his scrotum the day before when he lifted a
heavy object. Physical examination demonstrates an afebrile patient with moderate abdominal
tenderness and fullness, with bowel sounds present in his right hemiscrotum. The testes are not
tender or enlarged. Which of the following is the most appropriate next step in management?
D. Oral hydration
E. Urinalysis
Answer A. The patient has evidence of an incarcerated hernia. The presence of fullness and
bowel sounds in the scrotum indicates a hernia, and tenderness with inability to easily reduce
the hernia indicates incarceration. Ice packs should be applied to the area to reduce the
attendant bowel edema and an attempt should be made to reduce the hernia, under procedural
sedation if necessary. Operative reduction is required if the hernia can’t be reduced. Operative
reduction and IV antibiotics would also likely be necessary if the patient presented with signs of
strangulation such as fever, acidosis, and/or severe tenderness. Outpatient referral for an
incarcerated hernia is not appropriate. Oral hydration may maintain euvolemia, but the
possibility of procedural sedation or surgery should preclude any oral intake. Incarcerated
hernia is a clinical diagnosis and urinalysis will not aid diagnostic accuracy.
27. A 52-year-old female presents with right upper quadrant pain and fever for 2 days. The pain
is unaffected by eating, but she states that it hurts to breathe and cough. Vital signs are 102.5°F,
HR 110,RR 20, 165/92, 100% RA. She appears fatigued and in some pain. She does not have
jaundice. Her lungs are clear. She has definite right upper quadrant tenderness without a
positive Murphy sign. Chest radiography is normal. You order a right upper quadrant ultrasound,
which shows a normal gall bladder, but a large, cavitary, fluid-filled lesion in her right hepatic
lobe. Which of the following is the most likely etiology?
A. Sterile inflammation
B. Entamoeba histolytica
pg. 147
C. Streptococci
D. Candida
E. Polymicrobial
Answer E. The patient has evidence of liver abscess, with fever, right upper quadrant pain, and
the ultrasound findings. Pleuritic pain and cough are common and can confuse the clinical
picture by seemingly supporting a pulmonary etiology. Liver abscesses are usually pyogenic
(80%) but can be amebic (10% to 15%) or fungal (<10%). Pyogenic liver abscesses are usually due
to biliary obstruction or cholangitis but can be caused by other intra-abdominal infections such
as diverticulitis or appendicitis. Pyogenic liver abscesses are usually polymicrobial, with
concurrent gram-negative, gram-positive, and anaerobic offenders. Antibiotic treatment should
therefore be broad-spectrum. Sterile liver abscesses are extremely uncommon, occurring rarely
in cases of hepatic vasculitis or rheumatologic diseases. E. histolytica is the cause of amebic liver
abscesses, which are suspected in cases of foreign travel (often with a preceding intestinal
illness) or oral–anal sexual contact causing fecal–oral contamination. Streptococci alone are not
usually a cause of liver abscesses. Candidal infection represents less than 10% of all liver
abscesses.
28. A 57-year-old male with a history of alcoholism and esophageal varices presents after two
episodes of bloody emesis. Which of the following is true?
A. The patient can be discharged if he has a negative nasogastric (NG) tube lavage, stable vital
signs, and a normal initial hemoglobin level
D. Patients requiring a transfusion should receive blood products to achieve a hemoglobin ≥10
g/dL
Answer E. There are few differences in the ED management of patients with presumed variceal
upper GI hemorrhage compared to other causes of upper GI bleeding. The initial focus is on
stabilizing unstable patients with intravenous fluids and blood products as needed to maintain a
target hemoglobin >7 g/dL unless patients are at high risk for, or have evidence of, cardiac
ischemia or other end-organ dysfunction attributable to anemia. NG tube lavage has not been
clearly shown to be useful. While a frankly positive lavage predicts high risk lesions, a negative
lavage does not conclusively rule them out. IV fluids may dilute the hemoglobin concentration
but they don’t diminish the actual hemoglobin content (and thus, the oxygen carrying capacity
of blood). While intravenous proton pump inhibitors and somatostatin analogues may not offer
pg. 148
a significant mortality benefit to patients, they are well tolerated with few significant side
effects. Their use may be helpful, possibly in consultation with a gastroenterologist who will be
performing esophagoduodenoscopy.
A. Nearly all patients with appendicitis younger than 3 years have evidence of perforation at the
time of appendectomy.
Answer E. The incidence of perforation at the time of appendectomy has an inverse correlation
with the age of the patient. More than 90% of patients younger than 3 years have evidence of
perforation in the operating room. In contrast, only 15% of adolescents have perforation at the
time of appendectomy. This difference relates to the difficulty and subsequent delay in making a
diagnosis in infants and toddlers. Most patients younger than 2 years have diffuse tenderness
rather than focal tenderness over the right lower quadrant. Appendicoliths are considered
pathognomonic for appendicitis but are only present in roughly 15% of cases. On the basis of
the few studies performed to date, most authors recommend CT as superior to ultrasonography
for the diagnosis of appendicitis. However, more data need to be collected before CT is routinely
recommended as the standard of care as ultrasound may perform nearly as well, particular in
pediatric centers with appropriately selected patients.
30. A 52-year-old female with no past medical history presents with fever, right lower quadrant
pain, and occasional diarrhea for the past several days. She takes no medications and has not
traveled internationally recently. Her CT scan reveals terminal ileitis. Which of the following is
the most likely cause?
A. Ulcerative colitis
B. C. difficile
C. Yersinia
E. Giardiasis
pg. 149
Answer C. Yersinia enterolitica can cause a terminal ileitis which is very similar to Crohn disease.
Symptoms are virtually identical to Crohn disease and can also resemble appendicitis. In mild
cases, Yersinia colitis can be treated supportively, but in more severe cases, fluoroquinolone or
doxycycline therapy may be used (trimethoprim–sulfamethoxazole in children). Ulcerative colitis
almost never involves the terminal ileum and is limited to the colon. C. difficile usually causes a
colitis due to antibiotic use and doesn’t usually affect the terminal ileum. Irritable bowel
syndrome does not cause objective abnormalities on imaging. Giardiasis generally causes a
watery diarrhea on exposure to infected water without evidence of terminal ileitis.
31. Which of the following is true regarding irritable bowel syndrome (IBS)?
32. Which of the following is the most common cause of duodenal perforations?
A. Foreign body
B. Neoplasia
C. Crohn disease
D. Tuberculosis
Answer E. Peptic ulcer disease is the most common cause of both duodenal and gastric
perforations. The other answer choices are less common causes. Crohn disease tends to cause
pg. 150
distal small bowel perforations more often than duodenal. Diverticulitis and neoplasia are the
most common causes of large bowel perforations.
A. Toxic megacolon is more common in patients with Crohn disease than ulcerative colitis.
E. Anal fissures in patients with Crohn disease tend to be located in the posterior midline.
Answer B. Although rectal involvement is uncommon in patients with Crohn disease and nearly
universal in patients with ulcerative colitis, perianal complications are much more common in
patients with Crohn disease. The anal canal is the most terminal segment of the large intestine.
In approximately 25% of patients with Crohn disease, perianal complications may occur before
the onset of overt disease. Toxic megacolon is more common in patients with ulcerative colitis.
Erythema nodosum occurs most often in women patients with Crohn disease. Anal fissures
associated with Crohn disease are typically eccentrically located. In patients without Crohn
disease, more than 90% of fissures are located in the posterior midline.
A. Pain that wakes patients in the middle of night is typical of duodenal ulcers.
B. The incidence of bleeding from gastric ulcers is approximately two times that of duodenal
ulcers.
C. Helicobacter pylori is the major risk factor linked to the development of duodenal ulcers but
has
D. Barium contrast is the diagnostic study of choice to diagnose peptic ulcer disease.
E. Only 50% of those people infected with H. pylori will develop a peptic ulcer in their lifetime.
Answer A. The pain of duodenal ulcers is usually described as a burning or gnawing epigastric
sensation that is decreased with food or antacids. The pain typically occurs 2 to 3 hours after a
meal. Classically, two-third of patients with duodenal ulcers describe pain that wakes them from
sleep in the middle of the night, although few patients have pain on waking in the morning. The
pain of gastric ulcers tends to occur more quickly after meals and may even be precipitated by
pg. 151
food in some patients. Therefore, anorexia and weight loss occur in approximately 50% of
patients with gastric ulcers but rarely occur in patients with duodenal ulcers. Duodenal ulcers
are twice as likely to be complicated by bleeding as gastric ulcers. H. pylori, although more
commonly found in the setting of duodenal ulcers, is the major risk factor for the development
of either type of ulcer. Flexible endoscopy is the diagnostic study of choice for peptic ulcer
disease. Finally, only 15% to 20% of patients colonized with H. pylori will develop a peptic ulcer
in their lifetime.
35. Which of the following is the major pathophysiologic mechanism in gastroesophageal reflux
disease (GERD)?
Answer B. While all mechanisms contribute to GERD, lower esophageal dysfunction is the major
one. This underscores the importance of behavioral changes in the management of GERD. While
proton pump inhibitors and antihistamines are important to reduce acid production, reduction
in smoking, alcohol intake, caffeine use, fatty foods, and large meals, exercise and lying down
immediately after eating is also necessary for successful treatment.
A. Plain films are more often diagnostic in cases of cecal volvulus than sigmoid volvulus.
C. Cecal volvulus is diagnosed by visualizing a massively dilated cecum in the left upper quadrant
on x-ray.
Answer C. Sigmoid volvulus accounts for 75% of volvulus cases, whereas cecal volvulus accounts
for the remaining 25%. Although patients with cecal volvulus are younger than patients with
sigmoid volvulus, they are not young, as affected patients are typically in their late fifties. Plain
films are diagnostic in 80% of cases of sigmoid volvulus but <50% of cases of cecal volvulus. The
pg. 152
classic findings of cecal volvulus include a massively dilated cecum typically in the left upper
quadrant or epigastrium (i.e., not within the right abdomen). Although selected patients with
sigmoid volvulus may be candidates for endoscopic reduction, patients with cecal volvulus
almost always require surgical repair due to the difficulty of accessing this area endoscopically
and the need for cecopexy to prevent recurrence.
37. A 74-year-old female presents with acute onset of severe, diffuse abdominal pain and
nausea. Her vitals are: 100.5, 112, 18, 100/67, 99% on RA. Her heart reveals an irregular rhythm,
but her abdominal examination exhibits only mild tenderness. You order comprehensive labs, an
upright chest x-ray, and a CT of the abdomen with IV contrast. Which of the following is the next
best step in management?
A. Broad-spectrum antibiotics
C. Barium enema
D. HIDA scan
E. Norepinephrine
Answer A. Given her age, pain out of proportion to physical examination findings, and presumed
atrial fibrillation, the patient likely has acute mesenteric ischemia, which is usually caused by an
arterial embolism. Acute mesenteric ischemia carries with it the possibility of infarcted or
necrotic bowel, and all patients strongly suspected of this condition should receive early
broad-spectrum antibiotics, intravenous hydration, and anticoagulation with heparin. Tissue
plasminogen activator is not indicated, as surgery is a very possible treatment strategy. Barium
enema is used to diagnose and treat intussusception and would not be appropriate here. HIDA
scan is a confirmatory test for biliary pathology when ultrasound is nondiagnostic.
Norepinephrine and other vasopressors should not be used in the setting of acute mesenteric
ischemia as this may further constrict an already narrowed blood vessel.
38. What is the sensitivity of plain abdominal radiographs for the diagnosis of small bowel
obstruction (SBO)?
A. 10%
B. 25%
C. 50%
D. 75%
E. 90%
pg. 153
Answer D. Abdominal radiography is generally considered the first-line imaging modality to
evaluate patients with suspected SBO. The sensitivity of plain radiography is about 75%. If
radiographs are normal, but the diagnosis of SBO is still suspected clinically, then CT
abdomen/pelvis, with both sensitivity and specificity of over 85%, can be used to confirm the
diagnosis. CT also identifies many causes of SBO (such as volvulus or intussusception), other
diagnoses in the differential of SBO, and complications such as closed-loop obstruction. To avoid
unnecessary ionizing radiation, use of abdominal radiography is most efficient in patients with
known prior SBOs who present with typical signs and symptoms without systemic toxicity or
severe focal tenderness (which could indicate a closed-loop process).
39. A 46-year-old female presents with a large amount of bright red bloody stool. In the ED she
experiences another episode of significant rectal bleeding. The most likely underlying cause is:
A. Angiodysplasia
B. Diverticulosis
C. Ischemic colitis
D. Colon cancer
Answer B. Diverticulosis is the most common cause of lower gastrointestinal bleeding (LGIB),
which may be massive in 5% of cases. However, most cases of LGIB due to diverticulosis are mild
and resolve without intervention. Interestingly, although most diverticula are located in the left
(descending) or sigmoid colon, most bleeding diverticula are located in the right colon. This is
fortuitous, because most cases of angiodysplasia, a common cause of severe LGIB in the elderly,
are also located in the right colon. Angiographic studies performed to localize bleeding,
therefore, canalize the superior mesenteric artery (SMA) first.
40. A 56-year-old male presents with postoperative bloating. The consulting surgeon suspects an
ileus. Which of the following lab findings is most likely contributing to the patient’s symptoms?
A. Lactic acidosis
B. Hypokalemia
C. Hypomagnesemia
D. Hypercalcemia
E. Uremia
pg. 154
Answer B. Postoperative ileus is a common, expected outcome after laparotomy. However,
multiple electrolyte abnormalities may also cause or exacerbate the condition. Hypokalemia is
the most common electrolyte abnormality responsible for ileus, though hypomagnesemia,
hyponatremia, hypocalcemia, and uremia may also contribute.
41. A 39-year-old male with a surgical history of appendectomy presents with abdominal pain
and vomiting. You strongly suspect small bowel obstruction. Which of the following abdominal
examination findings is most likely to be present?
A. Distention
B. Focal tenderness
D. Rebound tenderness
E. Percussion tenderness
Answer A. About two-thirds of patients with small bowel obstruction exhibit abdominal
distention, making it the most common physical examination finding seen. Focal tenderness is
uncommon, as are rebound or percussion tenderness—these are more indicative of peritonitis.
High-pitched bowel sounds are certainly consistent with small bowel obstruction, but are not as
commonly seen as distention.
A. Perforation
B. Bleeding
C. Obstruction
D. Diverticulitis
Answer D. Diverticulitis occurs in 10% to 30% of patients with diverticulosis. Severe bleeding
occurs in only 3% to 5% of patients with diverticula. Most bleeding is minor and resolves
spontaneously without intervention.
pg. 155
43. Which of the following is true about upper gastrointestinal bleeding (UGIB) due to peptic
ulcer disease (PUD)?
Answer B. Hemorrhage occurs in approximately 15% of patients with PUD, although it is twice as
likely in patients with duodenal ulcers as in patients with gastric ulcers. Roughly half the number
of patients with UGIB due to PUD present with melena and hematemesis. Isolated melena
occurs in only 20% of patients and isolated hematemesis occurs in only 30% of patients. The
mortality rate has remained relatively unchanged over the last 30 years, and is between 6% and
10%. Perforation is accompanied by hemorrhage in roughly 10% of cases. Therefore, these
entities are not usually confused upon evaluation.
44. A 50-year-old male presents with inability to swallow his secretions and feeling like
something is stuck in his throat after swallowing a piece of steak. You suspect an esophageal
food impaction. Which of the following is true regarding this patient?
Answer A. Esophageal food impactions usually occur in patients who have pre-existing anatomic
abnormality of the esophagus, such as a Schatzki ring, stricture, or cancer. Nitroglycerin IV and
glucagon IV are usually attempted but resolve the obstruction less than a quarter of the time.
Definitive management usually involves endoscopic removal of the food bolus. Despite the fact
that alcohol relaxes the lower esophageal sphincter, alcohol actually puts patients at higher risk
for esophageal food impaction, probably because of behavioral eating changes. Papain should
never be used to facilitate passage of food boluses because it partially digests the
already-inflamed esophagus as well as the intended food target.
pg. 156
45. A 47-year-old male with a history of alcohol abuse presents with a 1-day history of constant,
dull, non-radiating epigastric pain. He recently drank more alcohol than usual. His vital signs are
98.5°F, 100, 18, 145/85, 99% RA. He is tender in the epigastrium without rebound or guarding.
Serum lipase is significantly elevated. Which of the following is the next best step in evaluation?
A. Triphasic CT pancreas
B. HIDA scan
C. MR cholangiopancreatography (MRCP)
Answer E. The patient has classic alcoholic pancreatitis, the most common cause in men and the
second most common cause overall (after gallstones). Autodigestion from pancreatic enzymes
due to obstructing gallstones or ethanol damage is the primary pathogenesis. Lipase is the most
specific diagnostic test for pancreatitis and is almost as sensitive as amylase. In most cases of
acute pancreatitis, emergent abdominal imaging is not necessary. CT abdomen/pelvis may be
pursued if a complication such as abscess, necrosis, or pseudocyst is suspected by the presence
of fever or systemic toxicity. Ultrasound of the gallbladder should be performed in any patient
without a history of alcohol abuse. HIDA scan is used as a more sensitive study (compared to
ultrasound) to evaluate gallbladder obstruction. Both MRCP and ERCP are used to evaluate
cases of gallstone pancreatitis.
46. A 54-year-old male with a history of chronic alcohol abuse is brought in by his wife because
of a “big stomach” and confusion. She tells you his stomach has been big for 1 year but over the
last month she adds that he seems to be forgetting things, has had difficulty sleeping, and has
not been tending to his own appearance. Which of the following is true?
pg. 157
levels do not exclude the diagnosis. Roughly 25% of patients will have non-nitrogenous causes of
encephalopathy. However, azotemia is the most common precipitant and GI bleeding is a very
common cause of azotemia and may frequently trigger or exacerbate HE. The treatment of
choice is ammonia-lowering therapy with lactulose, or with neomycin in the case of
azotemia-induced causes. Otherwise, treatment should be directed at the underlying
precipitant.
47. A 55-year-old male presents from his doctor’s office with nausea, vomiting, right upper
quadrant pain, and abnormal outpatient labs. Which of the following is consistent with alcoholic
hepatitis?
C. Hypermagnesemia
D. Leukocytosis
Answer D. Alcoholic hepatitis is a type of acute ethanol-induced liver disease. Patients usually
experience nonspecific symptoms such as fever, myalgias, nausea, and vomiting, in addition to
the more specific sign of right upper quadrant tenderness. Laboratory work often shows mild
elevations of WBC count, transaminases, and PT. Significant elevations of transaminases are far
more likely to be seen in viral or drug-induced hepatitis. Most patients who have acute alcoholic
hepatitis have a chronic history of alcohol abuse and are usually magnesium-deficient.
Treatment of alcoholic hepatitis involves symptom control, intravenous fluids, correction of
electrolyte abnormalities, and supplementation with thiamine and glucose.
48. A 22-year-old male presents for substernal chest burning for several weeks which is worse
after eating. He denies fever, cough, or shortness of breath. He does not smoke, drink alcohol,
drink caffeine, or take over-the-counter nonsteroidal anti-inflammatory drugs (NSAIDs). His only
medication is a pill for acne. His physical examination, chest x-ray, and EKG are normal. Which of
the following is the most likely cause?
A. Atypical pneumonia
B. Asthma
C. Pericarditis
D. Pill esophagitis
E. Psychogenic
pg. 158
Answer D. Given his history of acne, he is very likely taking doxycycline, which is a common
cause of pill esophagitis. Other medications that cause pill esophagitis include aspirin and
NSAIDs, iron supplements, and bisphosphonates. Symptomatology is the same as that for
gastroesophageal reflux disease. Pill esophagitis can be treated supportively or with proton
pump inhibitors. Prevention by drinking a full glass of water with the offending medication is
useful. Pneumonia is less likely given the lack of dyspnea, fever, cough, crackles, or chest x-ray
abnormality. Asthma would be unlikely without cough or shortness of breath. Pericarditis is
possible but usually causes some positional changes or EKG abnormalities, sometimes with an
antecedent viral syndrome. Psychogenic somatic symptoms should be a diagnosis of exclusion.
49. A 58-year-old female with no prior history of abdominal surgery presents with progressive
abdominal bloating and distension over the past several days as well as a 1-day history of
nausea and vomiting (Fig below). Which of the following is most likely responsible?
A. Incarcerated hernia
B. Intussusception
C. Cancer
D. Volvulus
E. Gallstone ileus
Answer C. The image demonstrates a small bowel obstruction (SBO). Postoperative adhesions
account for more than 60% of cases, malignant neoplasms account for approximately 20%, and
pg. 159
incarcerated hernias account for roughly 10% of cases. The classic mnemonic for the top three
causes of small bowel obstruction is “ABC,” representing adhesions, bulges (hernias), and
cancer. However, neoplasms are actually the second most common cause. Since this patient has
never had abdominal surgery, adhesive disease is not likely. Most tumors are metastatic lesions
that cause extrinsic compression of the intestine secondary to peritoneal implants that have
spread from an intraabdominal primary tumor such as from the colon, ovary, or pancreas. The
most common hernias to result in obstruction are ventral and inguinal hernias. Flat (or supine)
films classically demonstrate multiple dilated loops of small intestine in a “stepladder” pattern
without evidence of distal small bowel or colonic distention. Upright films demonstrate multiple
air–fluid levels. As a general rule, the larger the number of dilated bowel loops, the more distal
the obstruction. The overall sensitivity of plain films for SBO is roughly 60%.
50. An 8-year-old male is brought in by his parents after apparently ingesting a pin. He looks well
and has a normal physical examination. A flat plate of the abdomen is shown in Figure 7-8.
Which of the following is true?
D. The patient can be safely discharged home with follow-up with his pediatrician.
E. The patient’s parents should give consent for an emergent laparotomy for surgical removal.
pg. 160
Answer A. Eighty percent to 90% of objects that have made it into the stomach will pass through
the remainder of the GI tract without difficulty. However, as many as 15% to 35% of sharp or
pointed objects may cause perforation if untreated. The ileocecal valve is the most common site
of perforation. Objects that are in the stomach are amenable to endoscopic removal, but
objects distal to this point generally cannot be retrieved. Because this child is asymptomatic,
there is no indication for surgery at this point. Surgery is only indicated in cases of perforation,
hemorrhage, fistula formation, or obstruction. Because this child has a relatively high risk of
perforation, he cannot be discharged home. Appropriate management includes daily abdominal
radiographs to follow the passage of the object. Surgical management should be individualized
but may be considered if the object fails to pass for a number of days.
A. Encephalopathy
D. Ascites
E. Hepatocellular carcinoma
Answer D. Ascites is the most common complication of cirrhosis, occurring in roughly 60% of
patients with compensated cirrhosis for 10 years. Spontaneous bacterial peritonitis is a
complication of ascites and occurs in 8% to 25% of patients with cirrhosis and ascites.
Esophageal varices are also common and occur in 25% to 40% of patients with cirrhosis. Of
those patients, 30% develop bleeding within 2 years. Hepatic encephalopathy is also a common
complication, but its incidence depends on the criteria used to diagnose encephalopathy. It
most commonly presents as a sleep disturbance, although patients may have trouble with
mood, disorientation, or speech.
52. Which of the following is the gold standard for diagnosing choledocholithiasis?
pg. 161
Answer D. MRCP is the current preferred test for the diagnosis of stones in the common bile
duct (CBD) among patients without a contraindication (e.g., pacemaker). While ERCP is effective,
MRCP is noninvasive, and easy to perform. Although transabdominal US is the gold standard for
the diagnosis of cholelithiasis, only 50% of stones in the CBD can be visualized by US. This is due,
in part, to the proximity of the duodenum to the CBD and the interference that occurs due to
luminal bowel gas. However, US may detect a dilated CBD in excess of 6 mm (the upper limit of
normal) in up to 75% of cases. Therefore, US may suggest but does not confirm the diagnosis.
Endoscopic ultrasound (EUS), however, is far superior to transabdominal US and equal to MRCP
in sensitivity and specificity for the diagnosis of CBD stones. EUS also provides better resolution
than MRCP, and is a dynamic rather than static test making it useful for concomitant stone
removal. The primary disadvantage to EUS is that it is an invasive procedure. CT scanning is
extremely useful in diagnosing complications of gallstones, such as perforation of the
gallbladder, abscess formation, pericholecystic fluid, pancreatitis, and gas in the gallbladder
wall. The sensitivity and specificity of CT has been improving with the use of helical scanners,
however, and its sensitivity and specificity is now roughly 85%. Likewise, cholescintigraphy has
inadequate sensitivity and specificity to be useful for the detection of CBD stones.
53. A 62-year-old female with a history of chronic atrial fibrillation presents with 2 hours of
acute-onset, continuous, excruciating abdominal pain. A few minutes after the onset of pain,
she had an urge to defecate and had a large, forceful bowel movement. She denies bloody or
melenic stools or a history of postprandial pain. On examination, her abdomen is soft, flat, and
only mildly tender. Labs reveal a leukocytosis of 16,000 per mm3 with a normal chemistry. An
initial flat and upright abdominal film is nonspecific and a CT scan is read by the radiologist as
having evidence of small bowel thickening. What is the next most important step in
management?
Answer A. This patient has mesenteric ischemia, which is a disease that still has a mortality rate
of 70%. Therefore, any patient older than 50 years who has risk factors for acute mesenteric
ischemia and who presents with acute-onset abdominal pain should be presumed to have
mesenteric ischemia until proven otherwise. The key to diagnosis is recognizing patients at risk,
which includes any patient older than 50 years who presents with acute abdominal pain and
who has known vascular disease, cardiac arrhythmias, recent MI, hypovolemia, hypotension, or
sepsis. The most commonly cited clinical finding is patient pain that is out of proportion to
tenderness elicited on physical examination. This is a nonspecific finding that needs to be
pg. 162
considered carefully in light of the clinical scenario. Unfortunately, there are no laboratory
markers or radiologic studies apart from angiography that have sufficient sensitivity and
specificity to exclude acute MI early in its course. Lactate levels are elevated in approximately
100% of patients with bowel infarction, but this is a late finding and mortality rates are high by
the time infarction has occurred. Plain films are most commonly nonspecific, although even
minimal findings such as ileus correspond to more severe disease and a higher mortality rate.
The sensitivity of CT has been cited to be as high as 82%, but the most common early finding is
bowel wall thickening, present in 26% to 96% of cases. Unfortunately, this is also the least
specific finding and is often not present in mesenteric ischemia due to arterial embolism or
thrombosis, which are the most common causes of acute MI. Pneumatosis intestinalis or gas in
the portal venous system is a specific finding but is only present after bowel infarction has
occurred. Angiography is the key to diagnosis and allows for therapeutic intra-arterial infusion of
papaverine, a potent vasodilator, or thrombolytic drugs.
54. A 62-year-old female with a history of known peptic ulcer disease presents with a chief
complaint of hematemesis and black tarry stools. Her vital signs include a pulse of 105 and an
SBP of 115 mm Hg. Her initial hemoglobin is 9.6 g/dL. Which of the following is most likely to be
useful in this patient?
Answer E. Patients with severe upper gastrointestinal bleeding (UGIB) require emergent blood
transfusion which may be life-saving. In patients with more moderate bleeding, continuous
infusions of proton pump inhibitors (PPIs) have been shown to improve the outcome by
reducing the need for blood products and reducing the need for reintervention. However,
intermittent bolus administration is much less effective than continuous infusion. This is
because continuous PPI infusions maintain gastric pH >4 (the threshold for pepsin inactivation).
Bolus administration allows gastric pH to fluctuate and episodes of increased gastric acidity may
disrupt clot formation. H2 inhibitors do not alter the natural history of UGIB.
Sengstaken–Blakemore tubes are rarely used adjuncts to stop hemorrhage from esophageal
varices. Though they are effective, they have a high complication rate and should only be used
when endoscopy is not immediately available. Ewald tubes are large-bore nasogastric tubes
used for gastric lavage. They may be useful to irrigate the stomach before endoscopy or in cases
of acute toxic overdose as a means of decontamination. Octreotide is primarily used for acute
variceal hemorrhage although it may be useful as an adjunct in cases of nonvariceal UGIB. This
question is still being studied.
pg. 163
55. A 56-year-old male with a history of alcoholic cirrhosis presents with altered mental status.
According to his wife, he reportedly stopped taking his lactulose several days prior to his
emergency department (ED) evaluation. He is disoriented and confused and demonstrates
asterixis. He has normal vital signs. Laboratory studies are unremarkable except for a chronically
elevated prothrombin time (PT). Non-contrast CT of the brain is normal. The ammonia level is
only mildly elevated. Which of the following is the next best step in management?
A. Consult psychiatry
B. Give lactulose
E. Consult neurology
Answer B. The patient has clinical evidence of hepatic encephalopathy (HE), likely grade II.
Grade I is marked by mild personality changes and cognitive dysfunction; grade II by
disorientation, confusion, lethargy, and asterixis; grade III by somnolence and inability to follow
commands; and grade IV by coma. Ammonia levels are usually, but not always, elevated in HE
and do not correlate accurately with the degree of encephalopathy, which should be clinically
assessed. Lactulose should be used to reduce hyperammonemia even when ammonia levels are
only mildly elevated. Colonic bacteria convert lactulose into lactic acid, which then acidifies
ammonia into ammonium in the gut, inhibiting its resorption and promoting its stool passage.
Lactulose may be given by nasogastric tube in patients with altered mental status. Patients with
cirrhosis and altered mental status in the ED should never be presumed to have a psychogenic
cause of their symptoms. Endotracheal intubation is usually used for patients in grades III or IV
HE, where loss of airway protection is more likely. Lumbar puncture is not routinely indicated in
afebrile cirrhotic patients with altered mental status; additionally, fresh frozen plasma would be
indicated beforehand to avoid epidural bleeding in these coagulopathic patients. Emergent
neurology consultation is rarely necessary in these patients.
pg. 164
Answer A. Only 5% of intussusception cases occur in adults and almost all of them occur in the
small intestine. In contrast to pediatric patients, adults with intussusception almost always have
an identifiable lead point, which is a malignancy 75% of the time (other lesions include
inflammatory lesions and Meckel diverticulum). In adults, the diagnosis is best made with a CT
scan. Although barium or water contrast enemas may diagnose and reduce intussusception, it is
not as useful in adults because most lesions are in the small intestine and it is thought that the
contrast material may help spread malignant cells. Most patients present with signs of
incomplete obstruction (only 20% of patients have complete obstruction) with a chief complaint
of abdominal pain. Large bowel obstruction in adults is most commonly due to malignancy, with
volvulus and diverticulitis being the next most common causes.
A. Polyethylene glycol
B. Activated charcoal
C. Hemodialysis
D. Endoscopic removal
E. Operative removal
Answer A. People ingest packets of illicit drugs to evade law enforcement officials by two main
methods—packing and stuffing. Body packers ingest large amounts of drug in well-sealed
packets in a methodical manner; body stuffers, on the other hand, are under time pressure to
ingest a small number of packets quickly to avoid being captured with the drugs in their
possession. Body packers are less likely than body stuffers to have packets rupture, but more
likely to die from ruptured packets, as they usually contain a large amount of drug. Management
of asymptomatic patients involves diagnosis with oral contrast radiographs, followed by bowel
irrigation with polyethylene glycol and admission to a monitored bed. Charcoal may be of
benefit in symptomatic patients shortly after packet ingestion. Hemodialysis is used only if
packets have ruptured and drug has been absorbed, but cannot be set up quickly enough in
these patients, as death will be sudden. Endoscopic removal is rarely indicated as rupture may
occur during packet withdrawal. Operative removal is emergently indicated in patients with any
symptoms suggestive of cocaine packet rupture, as uncontrolled sympathomimetic crisis will
cause death despite even the most aggressive nonsurgical measures.
pg. 165
58. A 42-year-old female presents with acute onset of abdominal pain. She reports no alcohol
use. Her only past medical history is cholecystectomy. Her abdomen is tender in the epigastrium
and her lipase returns at 2,500 U/L. Her CBC and her basic metabolic panel return from the lab
as unreadable, despite multiple different blood draws. Which of the following is the most likely
etiology?
A. Alcohol
B. Gallstones
C. Hypertriglyceridemia
D. Ibuprofen
E. Rhinovirus
59. A 27-year-old female presents with chronic, intermittent, diffuse, crampy abdominal pain
and intermittent non-bloody diarrhea for 10 months. She has seen her primary care physician
multiple times for these symptoms and has had a right upper quadrant ultrasound, CT
abdomen/pelvis, magnetic resonance imaging (MRI) abdomen/ pelvis, HIDA scan, urinalysis, and
a comprehensive chemistry lab panel, all of which have been negative. Her vital signs are 98.6°F,
85, 16, 110/65, 99% rheumatoid arthritis (RA). Her examination is unremarkable and her urine
pregnancy test is negative. Which of the following is the most appropriate therapy for this
patient?
A. Fiber supplementation
B. Lithium
C. Desipramine
Answer C. The patient likely has diarrhea-predominant irritable bowel syndrome (IBS). A
hypersensitivity to bowel wall peristalsis resulting in altered gut motility and pain perception is
pg. 166
the proposed pathophysiology. While few treatments have undergone rigorous
randomized-controlled trials, low-dose tricyclic antidepressant therapy (such as desipramine or
amitriptyline) has been found to be effective in many cases. The emergency physician should
never prescribe tricyclics withoutconsulting the primary care provider and ensuring adequate
follow-up. Fiber supplementation is used primarily for constipation-type (IBS). Lithium is used to
treat bipolar disorder but has no current role in the management of IBS. Empiric use of
antibiotics in IBS or any other cause of chronic intermittent diarrhea is not recommended. Due
to radiation exposure and high cost, repeat CT scanning in a patient without focal abdominal
findings (especially when one has already been performed for the symptoms) is not
recommended.
60. Which of the following signs has over 90% sensitivity for acute appendicitis?
A. Obturator sign
B. Rovsing sign
C. Psoas sign
D. Kehr sign
Answer E. Physical examination findings generally produce only moderate (50% to 80%)
sensitivity in the evaluation of acute appendicitis. Of the signs listed, only Rovsing sign
approaches this sensitivity. The obturator sign’s sensitivity is so low that it is no longer a
clinically useful study. The psoas sign is used to detect a retrocecal appendix but sensitivity is
reported at under 50%. The Kehr sign (referred pain at the shoulder due to diaphragmatic
irritation) is seen more often in the setting of splenic rupture, ectopic pregnancy, and
diaphragmatic lesions.
61. Which of the following is most useful in differentiating a patient with acute cholangitis from
a patient with acute cholecystitis?
A. Jaundice
B. Fever
C. Abdominal tenderness
D. Leukocytosis
E. Murphy sign
pg. 167
Answer A. There is considerable overlap in the clinical presentation of patients with acute
cholecystitis and acute cholangitis. However, patients with acute cholecystitis rarely exhibit
jaundice and tend to be less toxic-appearing. Although the cystic duct is usually blocked in acute
cholecystitis, the hepatic and common bile ducts are patent and free of infection and
inflammation. Charcot triad (fever, right upper quadrant pain, jaundice) is the hallmark of acute
cholangitis. Fever is nearly universal, present in 95% of patients, right upper quadrant
tenderness in 90% and jaundice in 80%. Hypotension and altered mental status are present in
15% of patients and suggests gram-negative sepsis. When present in concert with Charcot triad,
these findings are known as Reynold pentad. Although mildly elevated bilirubin levels may be
present in patients with acute cholecystitis, these levels rarely rise above 4 mg/dL.
62. A 44-year-old male 2 years s/p laparoscopic roux-en-y gastric bypass presents for evaluation
of generalized abdominal pain and food intolerance. The patient reports mild bloating and a
fullness sensation that he hasn’t had before. CT reveals a mesenteric swirl sign. Which of the
following is the most likely diagnosis?
A. Marginal ulcer
C. Biliary colic
D. Internal hernia
E. Intussusception
Answer D. This patient is presenting with symptoms of a small bowel obstruction (SBO) due to
an internal hernia. The mesenteric swirl sign is pathognomonic for an internal hernia although it
is often missed or absent. Furthermore, internal hernias often present with vague symptoms
without clear obstruction and can be missed by radiology. Therefore, it is imperative to discuss
these patients with their surgeons and to maintain a high degree of suspicion in patients with
symptoms suggestive of a possible bowel obstruction in the setting of abdominal pain and a past
history of laparoscopic roux-eny gastric bypass. Marginal ulcers and dilation of the gastric
remnant are other possible postoperative complications, but gastric dilatation will be clear on
CT scan, and neither condition produces the noted swirl sign.
63. A 76-year-old female with a history of atrial fibrillation presents with acute, diffuse
abdominal pain for 6 hours. The pain has been consistently severe since onset, and she
developed chills for the last 2 hours. She reports that she ran out of her coumadin 2 weeks prior.
Vital signs are 100.2°F, 115, 18, 168/88, 99% RA. Her abdomen is mildly tender diffusely without
focal rebound tenderness. Which of the following is true regarding this patient’s most likely
diagnosis?
pg. 168
A. A normal WBC count essentially excludes serious sequelae
Answer B. With the combination of atrial fibrillation, inadequate anticoagulant therapy, and
acute, diffuse abdominal pain, the patient likely has acute mesenteric ischemia. Diffuse pain out
of proportion is characteristic. CT angiography is the diagnostic test of choice. As with most
serious causes of abdominal pain in the elderly, WBC count is a poorly sensitive marker for
significant disease. An elevated lactate is a sign of advanced ischemia or infarction, but
sensitivity for early ischemia is very low. Mortality of acute mesenteric ischemia is around 50%
but is considerably higher when infarction has occurred. The most common etiology is
thromboembolism (usually from a cardiac etiology), followed by direct arterial thrombosis.
Venous thrombosis and low cardiac output each causes around 5% of cases.
A. 12 hours
B. 24 hours
C. 48 hours
D. 96 hours
E. 1 week
Answer C. Acutely thrombosed hemorrhoids should be excised within the first 48 hours.
Although excision provides rapid relief from pain, the natural history of thrombosed
hemorrhoids is spontaneous resolution after several days. Therefore, patients with only mild
pain from thrombosis and patients who have already dealt with symptoms for several days
should be managed conservatively. Excision in these patients will not provide any added relief.
65. An 85-year-old male presents with right upper quadrant pain, chills, and nausea. His pain is
worse after eating. He is febrile, appears toxic, and has significant right upper quadrant
tenderness to palpation. You suspect acute cholecystitis. A right upper quadrant ultrasound
reveals pericholecystic fluid and gallbladder wall thickening but no gallstones. Which of the
following is the next best step in management?
pg. 169
A. CT abdomen/pelvis with oral contrast
D. IV ampicillin–sulbactam
Answer D. The patient has clinical evidence of acute cholecystitis. While the large majority of
cholecystitis is due to gallstones, acalculous cholecystitis can occur in about 10% of cases. The
elderly and immune-compromised are at risk. Patients with acalculous cholecystitis are at higher
risk of complications such as gangrene and perforation and require aggressive antibiotic
management and early surgical consultation. Computed tomography is not indicated in the
evaluation of acute cholecystitis, except in cases of cholangitis, which would be suspected with
hyperbilirubinemia. Patients with acute acalculous cholecystitis should never be discharged
home.
66. Which of the following is the most common location of uncomplicated anal fissures?
A. Anterior midline
B. Posterior midline
C. Right lateral
D. Left lateral
E. Circumferential
Answer B. Ninety percent of anal fissures are located in the posterior midline. The remaining
10% are located in the anterior midline. Fissures located elsewhere should prompt
consideration of an underlying disease, such as Crohn disease, leukemia, HIV infection,
tuberculosis, or syphilis.
B. Anal fissure
C. Hemorrhoids
D. Lichen sclerosus
pg. 170
E. Diabetes mellitus
Answer A. All of the listed items are potential causes of pruritus ani, but the presence of fecal
matter on the perianal skin is the most common.
68. A complaint that may help differentiate patients with Crohn disease from patients with
irritable bowel syndrome is:
A. Nocturnal diarrhea
B. Bloating
C. Weight loss
E. Bilious vomiting
Answer A. Nocturnal symptoms point against a diagnosis of irritable bowel syndrome but are
common in patients with Crohn disease. Weight loss also points against irritable bowel
syndrome but is a less specific complaint. Bilious vomiting is not common in either disease.
69. A 65-year-old African-American woman presents with left lower quadrant abdominal pain
and slightly loose stools for 3 days. She is afebrile but is tender in the left lower quadrant. You
suspect diverticulitis. Which of the following is true regarding diverticulitis?
Answer B. Diverticulitis refers to inflammation and infection of colonic diverticula, which are
very common in the Western population (prevalence over 5% in people over 45 and over 75% in
people over 85). Uncomplicated diverticulitis refers to disease without frank septic shock,
obstruction, abscess, perforation, or necrosis. Most patients with uncomplicated diverticulitis
never have a recurrence, though the chance of recurrence increases with younger age at onset
of disease. In the Western world, diverticulitis occurs far more commonly in the left
(descending) colon; in Japan, right-sided disease is more common. Outpatient antibiotic
pg. 171
regimens include ciprofloxacin or trimethoprim–sulfamethoxazole plus metronidazole or
amoxicillin–clavulanic acid. High-fiber diets have been shown to reduce both symptoms and
chances of recurrence. Barium enema is a reasonable test to diagnose diverticulosis but should
never be used in the acute setting of diverticulitis.
A. Acetaminophen toxicity
B. Idiopathic
E. Reye syndrome
Answer A. Hepatitis B infection is the most common viral cause of acute liver failure. However,
worldwide, acetaminophen toxicity is the most common overall cause. Idiopathic causes are
probably the third most frequent cause of liver failure.
71. A 67-year-old female presents with acute, severe periumbilical and epigastric abdominal
pain. A flat plate and upright is shown (Fig. below). Which of the following is true?
pg. 172
Answer E. Most patients suffering from peptic ulcer perforation are elderly, and NSAID use is
involved in roughly 50% of cases. Smoking is the leading risk factor for perforation among young
patients. Cocaine use may lead to perforation of ulcers of the juxtapyloric region, possibly
because of vasoconstriction or vascular thrombosis. The pain of peptic ulcer perforation is
classically described in three stages. Initially, patients present with severe, diffuse abdominal
pain and may present with signs of shock. After minutes to hours, this phase tends to resolve
and patients begin to look and feel better and may have normal vital signs. However, they will
nearly always have signs of peritonitis upon physical examination. Most notably, patients will
have a rigid, tender abdomen, and signs of pelvic peritoneal inflammation (assessed by a rectal
examination). The final stage is characterized by worsening abdominal pain, abdominal
distension, and signs of sepsis. Only ∼2/3 of patients will have evidence of intraperitoneal free
air on upright chest or abdominal radiography. Insufflating 50 mL of air through an NG tube
before x-ray improves sensitivity. Treatment includes IV antibiotics, IV fluids, IV PPIs, and an NG
tube. Endoscopy is contraindicated because the insufflation of air may open a spontaneously
closed perforation.
72. Which of the following is true regarding acute mesenteric ischemia (AMI)?
A. Most patients have a lactic acidosis early in the course of their illness.
Answer B. Owing to its deceptively innocuous early course, the diagnosis of AMI remains
problematic. Therefore, the mortality rate has remained essentially unchanged at roughly 70%.
The key to diagnosis is recognizing patients at risk, such as any patient older than 50 years of age
who presents with acute abdominal pain and who has known vascular disease, cardiac
arrhythmias, recent myocardial infarction, hypovolemia, hypotension, or sepsis. The most
commonly cited clinical finding is pain that is out of proportion to tenderness elicited on physical
examination. This is a nonspecific finding that needs to be considered carefully in light of the
clinical scenario. Unfortunately, there are no laboratory markers or radiologic studies apart from
angiography that have sufficient sensitivity and specificity to exclude AMI early in its course.
Lactate levels are elevated in approximately 100% of patients with bowel infarction, but this is a
late finding and mortality rates are high by the time infarction has occurred. Plain films are most
commonly nonspecific, although findings such as ileus correspond to more severe disease and a
higher mortality rate. The sensitivity of CT has been cited to be as high as 82%, but the most
common early finding is bowel wall thickening, present in 26% to 96% of cases. Unfortunately,
this is also the least specific finding and is often not present in mesenteric ischemia due to
pg. 173
arterial embolism or thrombosis, which is the most common cause of AMI. Pneumatosis
intestinalis or gas in the portal venous system is a specific finding but is only present after bowel
infarction has occurred. In the absence of angiography, the treatment is emergent laparotomy.
73. Which of the following is true regarding lipase and amylase in the setting of acute
pancreatitis?
C. Amylase peaks earlier and remains elevated for a longer period than lipase.
D. The degree of elevation of either amylase or lipase correlates with disease severity.
E. The amylase to lipase ratio may be useful in determining the etiology of pancreatitis.
Answer A. Lipase and amylase have roughly the same sensitivity for diagnosing acute
pancreatitis, although their sensitivity depends on the threshold value above normal used to
establish the diagnosis (most authors suggest a cutoff of three times the upper limit of normal).
Lipase is almost certainly more specific than amylase, because almost all lipase originates from
the pancreas. However, there is a small amount of gastric lipase, and lipase levels may be
elevated in the setting of a gastric or duodenal ulcer, severe renal insufficiency, or in some cases
of bowel obstruction. Although both enzymes tend to rise at approximately the same rate,
lipase remains elevated for a longer period of time (lipase remains elevated for 8 to 14 days,
although amylase returns to normal after 5 to 7 days). The degree of elevation of amylase or
lipase does not correlate with disease severity. The ratio of amylase to lipase has not proven to
be useful.
74. A 42-year-old female presented to the emergency department (ED) with acute-onset
epigastric abdominal pain and nausea without vomiting. Her workup revealed acute pancreatitis
and she was admitted. After receiving appropriate analgesics and antiemetics, she is now
hungry and wants something to eat. Which of the following is true?
A. She should undergo a period of bowel rest for 48 hours regardless of her laboratory results.
B. She requires ongoing nasogastric (NG) suctioning until pancreatic enzyme abnormalities
resolve.
D. She should be allowed to eat if abdominal computed tomography (CT) reveals resolution of
the signs of pancreatitis.
pg. 174
Answer E. The decision about when to resume feeding and what patients should be allowed to
eat is a matter of ongoing controversy. In the past, all patients underwent continuous NG
suctioning or were kept NPO. Currently, the only indication for NG suctioning is intractable
vomiting or ileus. Some studies suggest that early enteral nutrition may actually improve
outcomes. Laboratory and radiographic evidence of pancreatitis is likely to persist until patient
discharge, so these are not useful guides for resuming feeding. Many authors now recommend
that enteral feeds should be started as soon as a patient is able to tolerate them. Although there
is a dearth of evidence regarding the composition of feeds, it is known that pancreatic
secretions decrease as carbohydrate composition exceeds 50% of the caloric content of the diet.
Therefore, it makes sense to start with a low calorie, carbohydrate-rich diet, and steadily
increase both the caloric and the fat content of the diet over a period of days.
75. Which of the following is the most common cause of large bowel obstruction?
A. Malignancy
B. Sigmoid volvulus
C. Adhesions
D. Diverticular disease
E. Fecal impaction
Answer A. Malignant neoplasms account for more than half of all cases of large bowel
obstruction .Volvulus and diverticulitis (either through stricture, abscess, or phlegmon
formation) are the second and third most common causes.
Answer E. Diverticuli are most common in the left colon, particularly the sigmoid colon, among
patients from the Western world. Japanese patients most commonly have diverticula in the
right colon. Even among Japanese who have immigrated to the United States and who consume
pg. 175
a low fiber, high-fat Western diet, diverticula are typically limited to the right colon. However,
such patients have a significantly higher incidence of disease than their counterparts in Japan.
The most common treatment for diverticular disease (in the absence of diverticulitis) is a
high-fiber diet. Fiber bulks the stools theoretically preventing stools from getting caught in
existing diverticuli while also helping to prevent the development of further diverticuli.
Diverticulosis is the most common cause of massive lower GI bleeding. It usually occurs due to
diverticuli in the right colon.
77. A 46-year-old male presents with chest pain 8 hours after having upper endoscopy for
dysphagia (Fig. below). Esophageal perforation is suspected and a screening chest x-ray is
ordered. Which of the following is likely to be found on physical examination?
C. Shock
Answer D. “Hamman crunch” refers to the crunching sound heard during cardiac auscultation in
the setting of pneumomediastinum. Radiographic abnormalities are present in up to 90% of
pg. 176
patients with esophageal perforation, and include pneumomediastinum, subcutaneous
emphysema, pleural effusion, and pulmonary infiltrate. Pleural effusions may occur and are
usually right sided if the perforation occurs in the midesophagus, but left sided if the perforation
occurs in the distal esophagus. Pneumoperitoneum may also occur if the patient has a rupture
of the intra-abdominal esophagus.
78. An institutionalized patient with psychiatric disease presents with abdominal pain,
distension, and nausea without vomiting. The image shown in Figure 12-14 suggests which of
the following diagnoses?
A. Sigmoid volvulus
C. Intussusception
D. Diabetic gastroparesis
E. Hirschsprung disease
pg. 177
Answer A. The image shows sigmoid volvulus. Sigmoid volvulus is primarily a disease of the
elderly as well as patients in long-term care facilities and patients with neurologic or psychiatric
disease. It is thought that medications used to treat psychiatric and neurologic disease may have
detrimental effects on colonic motility predisposing to volvulus. It is also frequently associated
with chronic constipation. Patients with small bowel obstruction usually have a history of
vomiting, and plain films would reveal circumferential plica circularis instead of
noncircumferential haustrae of the large bowel. Intussusception is rare in adults and x-ray
findings are neither sensitive nor specific. Diabetic gastroparesis would demonstrate a large
gastric bubble or dilation rather than colonic dilation. Almost all patients with Hirshsprung
disease are diagnosed before the age of 2.
79. A 52-year-old male with a long-standing history of alcohol abuse presents with a chief
complaint of vomiting blood. He was last seen by a doctor 8 months ago because of abdominal
swelling and he was told he had “a liver problem.” Which of the following is most likely to be
useful in this patient?
A. Octreotide
B. Famotidine
C. Pantoprazole
D. Vasopressin
E. Propranolol
Answer A. Octreotide is a longer lasting, synthetic analog of somatostatin. It inhibits the release
of several vasodilatory hormones such as glucagon, and has direct effects on vascular smooth
muscle. The end result of its effects is selective vasoconstriction of the splanchnic vasculature,
thereby decreasing pressure and bleeding in existing esophageal varices. Propranolol is useful as
a prophylactic adjunct to prevent rebleeding. Vasopressin has been used in the past to control
variceal hemorrhage but it has prohibitive side effects, including myocardial ischemia.
Therefore, it is currently not recommended for routine use.
80. A 23-year-old female presents with right lower quadrant pain. Which of the following
historical features increases the likelihood of acute appendicitis?
pg. 178
D. Unchanged pain with movement
E. Extreme hunger
Answer A. Migratory pain from the periumbilical region to the right lower quadrant has
moderately high specificity for appendicitis. Radiation of pain to the scapula is more suggestive
of biliary pathology. Pain from appendicitis is usually less than 24 hours in duration, but
sometimes patients present later if there is temporary relief of the pain (immediately after
perforation). Movement and coughing usually exacerbate the pain of appendicitis and other
causes of peritoneal irritation. Anorexia is very sensitive for the diagnosis of appendicitis.
pg. 179
Infectious disease and
dermatology 1
➢ Chapter includes:
Isolated Infectious disease
Infectious related diarrhea
Sexually transmitted disease
Dermatology related infectious condition
Dermatology
HIV, HIP , and related condition
✓ Exception (meningitis, peritonitis, encephalitis,
systemic infection and sepsis) are in infections
chapter 2
✓ Exception cardio related infectious are in cardio
chapter
pg. 180
1. A 35-year-old female without any past medical history presents with a red, painful region on
her right arm where she had a bug bite 3 days before. She denies fever. She is allergic to
penicillin. Vital signs are normal. Physical examination is remarkable for a 10 × 6 cm2 area on
her right arm that is red, warm, tender, and sharply demarcated. There is no lymphangitic
streaking or axillary lymphadenopathy. You diagnose her with cellulitis. Which of the following is
the most appropriate choice of antibiotic?
A. Clindamycin
B. Doxycycline
C. Dicloxacillin
D. Linezolid
E. Metronidazole
Answer A. Cellulitis in the healthy patient is most often caused by Streptococci and
Staphylococci. In most communities, methicillin-resistant Staphylococcus aureus (MRSA)
accounts for a significant number of staphylococcal infections. Thus, initial antibiotic therapy is
usually with a penicillinaseresistant penicillin or first-generation cephalosporin to treat
streptococci plus TMP-SMX to treat MRSA. Doxycycline covers MRSA but is inadequate against
streptococci, so it should be combined with amoxicillin (or cephalexin). Clindamycin can be used
as monotherapy, but there is significant resistance to clindamycin among MRSA isolates in many
communities. This patient has an allergy to penicillin, and dicloxacillin is therefore
contraindicated. In addition, dicloxacillin is ineffective against MRSA. Linezolid is an effective,
relatively new treatment that is generally reserved for infections that are not amenable to
treatment with the other options listed above. Metronidazole covers anaerobes only and would
not be appropriate as monotherapy for cellulitis. Note that in its most recent guidelines (2011),
the Infectious Disease Society of America differentiates between purulent and nonpurulent
cellulitis. The rationale behind this division is to illustrate that MRSA is typically responsible for
abscess formation and the development of purulent drainage. Thus, in cases in which both
cellulitis and an abscess are present (or forming), empiric therapy directed only at MRSA is
appropriate, and dual therapy is unnecessary. Clindamycin, doxycycline, TMP-SMX, and linezolid
are all useful for monotherapy in this setting.
A. Generalized lymphadenopathy
B. Tonsillar exudates
C. Koplik spots
D. Febrile seizures
pg. 181
E. Pastia lines
3. A 24-year-old male presents with bloody diarrhea for 2 days. Which of the following
antibiotics is considered first-line therapy to treat all of the following organisms: Salmonella,
Shigella, Yersinia, Vibrio, and enterotoxigenic E. coli (ETEC)?
A. Trimethoprim–sulfamethoxazole
B. Doxycycline
C. Metronidazole
D. Ciprofloxacin
E. Erythromycin
Answer D. Ciprofloxacin is the drug of choice for most causes of invasive bacterial diarrhea. The
one exception to this is Campylobacter diarrhea, which has a high resistance to ciprofloxacin,
especially in strains from Southeast Asia. Treatment with macrolides is preferred with
Campylobacter infections. Trimethoprim–sulfamethoxazole can be used to treat Yersinia, Vibrio,
ETEC, and certain parasites. Doxycycline is used as alternate therapy for Vibrio. Metronidazole is
the drug of choice to treat C. difficile and Giardia.
4. A 28-year-old female presents with fever and painful oral lesions for 2 days as shown below .
Which of the following is the most likely cause?
pg. 182
A. HSV-1
B. HSV-2
C. Epstein–Barr virus
D. Coxsackievirus
E. Group A streptococcus
Answer D. The patient has vesicular lesions on the soft palate consistent with herpangina,
caused by coxsackievirus. In children, this often presents as hand–foot–mouth disease.
Management is supportive, with special emphasis on pain control, as lesions are extremely
painful and can limit oral intake. HSV-1 can cause herpetic gingivostomatitis, which causes
lesions on the hard palate and gums and can be treated with valacyclovir. HSV-2 usually causes
genital herpes rather than oral lesions. Epstein–Barr virus causes infectious mononucleosis,
resulting in pharyngitis usually without discrete lesions. Group A streptococcus causes strep
throat, a pharyngitis with exudate on the tonsils but without discrete oral ulcers.
A. It is not contagious.
pg. 183
Answer A. Botulism occurs as a result of the toxin released by the anaerobic bacillus Clostridium
botulinum. Botulinum toxin is considered the most potent poison known to humans, but it is not
transmitted from person to person. Clinical botulism manifests as a descending paralysis, usually
involving the cranial nerves. The mechanism is irreversible inhibition of presynaptic
acetylcholine release. Antibiotics have little effect on treatment of botulism. Treatment involves
supportive care, mechanical ventilation, and antitoxin therapy.
C. Thrush in otherwise healthy newborns is self-limited and does not require treatment.
E. Maceration and lichenification with thick scale is the hallmark of cutaneous candidiasis.
Answer B. Oral candidiasis is the most common form of candidal infection and Candida species
colonize the oropharynx of 80% of healthy infants by 3 to 4 weeks of age. Oral candidiasis is also
an AIDS-defining illness. Because Candida spp. are part of the normal flora of so many people,
cultures are rarely useful. Instead, diagnosis is based on clinical examination and the finding of
white, curdlike exudates on the buccal and gingival mucosa and less frequently on the tongue
and soft palate. The exudates can be scraped away to reveal an erythematous, mildly eroded,
and painful mucosa underneath. Although the infection is frequently self-limited, infants with
thrush should be treated with oral nystatin suspension to hasten healing and primarily to
prevent problems with feeding (due to pain). Tinea cruris is most commonly caused by
Trichophyton spp., as with other dermatophyte infections. The hallmark of cutaneous
candidiasis is the presence of satellite papules and pustules beyond the margins of a patch of
macerated, sometimes weeping skin with scalloped borders. It typically occurs in intertriginous
areas such as the groin, axilla, or underneath pendulous skin folds.
7. A 26-year-old previously healthy male presents to the ED in January with a chief complaint of
a 2-day history of fever, cough, diffuse body aches, and general malaise. He reports no history of
influenza vaccination. His vital signs are P 110, BP 130/75, RR 18, SaO2 97% RA. Which of the
following is true?
B. Immunizing the patient with the influenza vaccine in the ED will hasten recovery
pg. 184
E. All of the above
Answer C. The patient in this vignette has influenza, which tends to peak in winter. When it is
started within the first 48 hours of illness, oseltamivir (Tamiflu) has been shown to reduce the
duration of symptoms due to influenza by about half a day. However, patients taking it have
more nausea and vomiting. In effect, using oseltamivir trades one category of symptoms for
another. It has not been shown to reduce complications or mortality from influenza, even
among the elderly or chronically ill. Influenza vaccination does not hasten recovery in patients
with active influenza.
8. A 34-year-old male with a history of HIV presents with headache. Contrast CT scan of the
brain is shown below. Which of the following is the most appropriate therapy?
A. Surgical excision
B. Mebendazole
D. Methylprednisolone
pg. 185
Answer C. The patient has a ring-enhancing lesion seen in the left parietal area on contrast CT
scan of the brain. In a patient with HIV, toxoplasmosis, due to the parasite Toxoplasma gondii, is
the most likely cause. Treatment is with sulfadiazine, pyrimethamine, and adjunctive leucovorin.
Corticosteroids may be used as adjunctive therapy in severe cases. Surgical excision is not
indicated in toxoplasmosis. Mebendazole is an agent used to treat other parasitic infections.
Clindamycin may be used to treat brain abscesses along with surgical drainage.
A. Shigella spp.
B. Giardia lamblia
C. Salmonella spp.
D. Rotavirus
Answer E. ETEC is responsible for 45% to 50% of traveler’s diarrhea. Shigella is the second most
common cause of traveler’s diarrhea, whereas viral causes and protozoa make up the
remainder.
10. The most common sexually transmitted organism in the United States is:
A. T. pallidum
B. C. trachomatis
C. Neisseria gonorrhoeae
D. HSV
E. H. ducreyi
Answer B. C. trachomatis causes ocular trachoma in many third-world countries (considered the
most common cause of preventable blindness worldwide) and is an important sexually
transmitted disease in the United States. In the United States, C. trachomatis is a major cause of
pelvic inflammatory disease (or cervicitis) as well as urethritis in men. A different serotype of the
organism is also responsible for lymphogranuloma venereum (LGV), a more invasive sexually
transmitted disease.
pg. 186
11. A 22-year-old female presents with intermittent fever and chills for 2 weeks. She has no past
medical history, but reports using intravenous heroin several times a week. Physical
examination reveals a febrile, ill-appearing woman with a heart murmur. Blood cultures are
most likely to reveal which of the following organisms?
A. S. aureus
B. Streptococcus pneumoniae
C. Streptococcus viridans
D. Pseudomonas aeruginosa
E. Candida albicans
Answer A. IV drug uses (IVDUs) have bacteria that are transmitted through the skin to superficial
veins, into the greater venous circulation, and eventually to the heart. These pathogens (S.
aureus in more than three-fourths of cases) lodge most commonly in the tricuspid valve to cause
right-heart endocarditis and may lead to septic pulmonary emboli.
12. Which of the following is true with respect to carditis caused by Lyme disease?
A. Patients most commonly experience initial symptoms several years after the initial tick bite.
B. Patients most commonly present with variable degrees of atrioventricular (AV) block.
D. The prognosis of patients with Lyme carditis is poor, as almost one-third of patients
ultimately require a heart transplant.
E. Most patients are sero-negative for anti-Borrelia antibodies at the time of presentation.
Answer B. Carditis complicates approximately 5% of untreated patients with Lyme disease, and
more commonly affects men. Cardiac manifestations occur during the early phase of the illness,
typically within a few weeks to a few months of infection. The most common manifestation is
AV block of varying degrees. Patients with a PR interval >300 milliseconds are at highest risk for
progressing to complete heart block. Occasionally, patients require permanent or temporary
pacemakers. However, most patients recover quickly without sequelae within 1 to a few weeks.
Occasionally, patients have persistent first-degree heart block. Patients less commonly suffer
from myopericarditis and symptoms of CHF. When present, myopericarditis is typically
asymptomatic and not clinically relevant. Overall, the prognosis for patients with Lyme carditis is
very good. The diagnosis is made when patients with appropriate findings give a history of tick
exposure, or have positive antibody tests. While patients with early localized Lyme disease can
have false-negative antibody, most patients with Lyme carditis are sero-positive.
pg. 187
13. Which of the following is true regarding viral hepatitis?
A. Adult patients infected with acute hepatitis B are more likely to become chronic carriers than
patients infected with acute hepatitis C.
B. Hepatitis C is most commonly acquired through sexual intercourse with an infected person.
D. Both direct and indirect bilirubin are typically elevated in roughly equal amounts.
Answer D. Adult patients infected with hepatitis B develop chronic hepatitis B <5% of the time.
In contrast, patients with acute hepatitis C infection become chronic carriers 80% to 90% of the
time. Hepatitis C is most commonly acquired by intravenous drug use (IVDU) or blood
transfusions. In the United States, both blood donors and the donated blood are screened for
hepatitis C, so the risk of infection through this route is quite low and IVDU predominates.
However, worldwide, <40% of the blood supply is tested for hepatitis C. Hepatitis C is the least
common viral cause of fulminant hepatic failure. Coinfection with hepatitis B and D is the most
common. The white blood cell count has almost no utility in the setting of hepatitis infection.
14. Which of the following is the most common cause of acute food poisoning in the United
States?
A. Clostridium perfringens
B. S. aureus
C. Escherichia coli
D. Bacillus cereus
E. Vibrio parahaemolyticus
Answer A. Although C. perfringens is probably the most common cause of acute food poisoning
in the United States, all of the above bacteria may cause foodborne illness. Their classic
associations are given below.
pg. 188
• C. perfringens: Toxin-mediated, predominantly diarrheal illness that occurs 6 to 12 hours after
ingestion and is common in meats, poultry, and gravies. Requires ingestion of live organisms
because the toxin is produced in vivo. The illness resolves in 24 hours with supportive care.
• S. aureus: Toxin-mediated emetogenic illness that occurs 1 to 6 hours after ingestion. Common
in high-protein foods such as ham, eggs, poultry, custard-based pastries as well as potato or egg
salads. Symptoms typically resolve within 8 hours without specific treatment.
• B. cereus: Causes two distinct illnesses. First is toxin-mediated illness that results in acute food
poisoning, predominantly characterized by vomiting. Very similar to the syndrome caused by S.
aureus except it almost always occurs after ingestion of fried rice. Second illness is a diarrheal
illness almost indistinguishable from the illness caused by C. perfringens. As with C. perfringens,
it is a toxin mediated event but requires ingestion of live organisms as the toxin is produced in
vivo.
A. Pneumonia
B. Subconjunctival hemorrhage
C. Pneumothorax
D. Otitis media
pg. 189
complications include seizures (0.3% to 0.6%), encephalopathy (0.1%), and even carotid artery
dissection.
16. Which of the following is true about extrapulmonary tuberculosis (TB) infection?
Answer A. Mycobacterium tuberculosis can affect nearly any organ system in the body. The
most common extrapulmonary manifestation is painless lymphadenopathy, usually in the
cervical region. Although classically described as a disease of childhood, the peak incidence is
between 20 and 40 years of age. Although the nodes are initially discrete, they may form a firm,
matted, nontender mass over time. Pericarditis may result from direct extension of TB from
mediastinal lymph nodes, or the spine, lungs, or sternum. The onset may be insidious or acute
and may result in a restrictive pericarditis. The CNS is not spared in extrapulmonary TB, and
tuberculous meningitis is the most common presentation of neurologic TB disease. In such
cases, the CSF usually reveals very high protein levels, ranging from 100 to 500 mg/dL, although
levels as high as 2 to 6 g/dL have been reported. In contrast, CSF glucose concentration usually is
<45 mg/dL. Skeletal TB most commonly involves the spine (Pott disease), with most lesions in
the thoracic pine. Vertebral destruction usually begins at the anteroinferior portion of the
vertebral body, eventually resulting in an anterior wedge defect and a palpable bony
prominence posteriorly (termed Gibbus). M. tuberculosis typically spreads to the bilateral
adrenal glands, causing bilateral adrenal enlargement and subsequent destruction which
ultimately causes adrenal insufficiency.
17. What is the approximate sensitivity and specificity of the monospot test for infectious
mononucleosis due to Epstein Barr virus (EBV)?
A. 50%, 50%
B. 80%, 80%
C. 95%, 80%
D. 80%, 95%
E. 95%, 95%
pg. 190
Answer D. The heterophile antibody test, or monospot, is a rapid assay for the diagnosis of
infectious mononucleosis due to EBV. It has lower sensitivity early in the illness (<70% in the first
week) but steadily improves in the next few weeks. Specificity is excellent and reported to be
near 100%. It is important to tell patients suspected of having infectious mononucleosis with a
negative monospot test that either the illness is early in its course or a non-EBV cause is
possible. Infectious mononucleosis is subacute to chronic multisystem syndrome of pharyngitis,
lymphadenopathy, splenomegaly, hepatitis, and fatigue. Splenomegaly is perhaps the most
important clinical feature for the emergency physician as demonstration of this finding requires
strict avoidance of contact sports and aggressive physical activity to prevent splenic rupture.
Treatment is supportive. Almost one-third of all patients with infectious mononucleosis carries
group A Streptococcus (GAS), which confounds the diagnosis of a patient with febrile pharyngitis
and leads to inappropriate antibiotic use.
A. Lyme disease
B. Babesiosis
D. Ehrlichiosis
E. Tularemia
Answer C. Rocky Mountain spotted fever (RMSF), caused by Rickettsia rickettsii transmitted by
the Derma centor tick, causes a centripetal rash, spreading from the wrists and ankles toward
the trunk. Despite its name, cases are seen most commonly in the southeastern states, but
nearly all states have reported cases. The characteristic symptoms are fever, constitutional
symptoms, abdominal pain, along with the centripetal rash. Antibiotic therapy (specifically with
doxycycline) has steadily improved the mortality to about 1%. Doxycycline can even be given to
children for optimal treatment of RMSF, as no other antibiotic improves outcomes as well. The
other tick-borne diseases mentioned are rarely associated with a centripetal rash. Syphilis,
dengue, Kaposi sarcoma, and coxsackievirus can also cause centripetal rashes.
19. A 5-year-old male is brought to the ED by his parents for diarrhea. He has had multiple, loose
bowel movements mixed with blood for 2 days, associated with shaking chills. The patient has
not traveled or been exposed to antibiotics. He appears dehydrated and ill. Which of the
following is the most likely cause?
A. Clostridium difficile
B. Campylobacter
pg. 191
C. Adenovirus
E. Giardia
Answer B. The patient has evidence of gastroenteritis with bloody, loose bowel movements, also
known as dysentery. Dysentery is more likely to be due to a bacterial source than viral or
parasitic. Although viruses cause the majority of cases of gastroenteritis, Campylobacter is one
of the most common causes of bacterial gastroenteritis and dysentery. C. difficile should be
suspected in any patient with a history of exposure to antibiotics. Adenovirus is a common
cause of pediatric viral gastroenteritis but is usually associated with watery rather than bloody
diarrhea. ETEC is the most common cause of traveler’s diarrhea, but rarely causes dysentery
especially when there is no history of foreign travel. Giardia causes a subacute, watery diarrhea,
often with a history of camping.
A. Permethrin 5% cream
B. Lindane 1% lotion
D. Fluconazole 150 mg PO
Answer A. Permethrin has become the treatment of choice for scabies because it is equally
efficacious to lindane, yet it is not appreciably absorbed through the skin making systemic side
effects less likely. Malathion shampoo can be used for pediculosis capitis (head lice), although
permethrin is still preferred because of its more pleasant odor and more rapid administration
(malathion requires 8 to 10 hours of administration in cases of head lice while permethrin
requires only 10 minutes).
21. A 64-year-old male with a history of hypertension presents to the ED with a painful rash on
the right side of his back spreading to his trunk (see Fig. below). Which of the following
underlying diseases should be suspected?
pg. 192
C. Asplenia
D. Rheumatoid arthritis
Answer E. This patient has herpes zoster (shingles). Although shingles may be more likely to
occur in patients with leukemia, Hodgkin lymphoma as well as other malignancies, most cases
occur in otherwise healthy patients.
22. A 62-year-old healthy male presents with a 1-day history of painful, partially crusted
vesicular rash over his left flank radiating to his left hemiabdomen that appears most consistent
with shingles. Which of the following is true?
B. Antiviral therapy decreases the rate of postherpetic neuralgia with or without steroids
D. The shingles vaccine may help hasten recovery from the rash and acute neuritis
pg. 193
E. All of the above
Answer C. Antiviral therapy has been shown to hasten recovery from the shingles rash as well as
the acute neuritis that accompanies it. However, neither steroids, nor antivirals, nor a
combination of the two has been shown to decrease postherpetic neuralgia. Steroids are not
recommended for patients with shingles. Antiviral therapy is only recommended for patients
presenting within 72 hours of onset. There is no role for the shingles vaccine in patients with
active shingles.
23. A 5-year-old male is brought for evaluation of penile erythema. He is uncircumcised and has
erythema, edema, and a semisolid discharge around the glans without accompanying phimosis
or paraphimosis. He has had several similar episodes in the past with identification of yeast.
Which of the following is the most appropriate diagnostic test?
B. Postvoid residual
C. Intravenous pyelogram
D. Serum glucose
E. Retrograde urethrogram
Answer D. The patient has evidence of fungal balanoposthitis, or infection of the foreskin and
glans. Recurrent such infections may be due to an immune-compromised state, most commonly
diabetes. Balanoposthitis most often occurs in patients who are uncircumcised and is usually
due to infection from typical skin flora. Urinalysis may be indicated to evaluate for sexually
transmitted infection. Treatment is with penile hygiene and antibiotics directed at the most
likely organisms. Liver function tests would not be helpful in evaluation of penile infection.
Postvoid residual volume and intravenous pyelogram evaluate for urinary obstruction.
Retrograde urethrogram is used in patients with pelvic trauma and suspected urethral injury.
24. A 52-year-old previously healthy American-born male presents 2 months after having a
purified protein derivative (PPD) placed on his left forearm. He had been in Central America
approximately 6 months ago with the Peace Corps. He states that the injection site became
quite large, red, and firm, but he never followed up with a doctor. He now wants to know what
to do. He denies cough, weight loss, drenching night sweats, or fever. His chest x-ray is
unrevealing. Which of the following is most likely true?
pg. 194
C. He has latent TB infection and he may infect other persons.
D. He has latent TB infection and requires prolonged treatment with INH and B6.
E. He must be admitted for acid-fast bacilli sputum cultures to determine if he has been infected
with TB.
Answer D. The Mantoux test is the most common test used to screen for TB exposure and
infection. It consists of an intradermal injection of 10 units (0.1 mL) of standardized PPD from M.
tuberculosis. Positivity is determined by the amount of induration, not erythema, in response to
the injection and is typically measured between 48 and 72 hours. Induration <5 mm in diameter
is negative and induration >15 mm is positive. Induration between 5 and 15 mm may be
positive, depending on other factors, such as prior immunization with Bacille Calmette–Guérin
(BCG) or the presence of immunosuppression as in patients with HIV. As the BCG vaccine is not
used in this country and because this patient is American-born, BCG is an unlikely cause of his
positive test result. In this country, patients with HIV are considered to have a positive PPD if the
amount of induration exceeds 5 mm, as are patients who have had close contact with active TB
and patients who have a fibrotic chest x-ray. Patients in any other high-risk group are considered
positive when the amount of induration exceeds 10 mm in diameter. Regardless of the group to
which a patient belongs, a positive PPD means that a patient has been exposed to and infected
with M. tuberculosis and that the organism remains in their body. Because this patient does not
have symptoms of active disease (e.g., fever, cough, hemoptysis, night sweats, anorexia, and
weight loss) and does not have any findings on his chest x-ray, he has latent, not active, disease.
Patients with latent TB infection are not contagious, but they require treatment with INH to
dramatically reduce their lifetime risk of developing active TB.
25. Which is the most appropriate management for a patient with suspected smallpox lesions?
A. Acyclovir
B. Vaccination
C. Ganciclovir
D. Rifampin
E. Isolation
Answer E. Smallpox is due to Variola major, a member of the poxvirus family. It was eradicated
in the 1970s and now exists only in laboratories. Mortality of smallpox is almost 30% and
contagiousness is extremely high. The only effective strategies for management are prevention,
vaccination, and isolation. Vaccination is only effective if given before or within a few days of
exposure. No antivirals or antibiotic are effective in management.
pg. 195
26. Which of the following is true regarding malaria?
27. A 23-year-old female presents with fever, myalgias, and headache for 3 days. She then
developed a rash, which started on her wrists and ankles and has now spread all over her body.
The non-blanching rash is shown in Figure below. Which of the following is the most likely
etiology?
A. Rickettsia rickettsii
B. Borrelia burgdorferi
C. Coxsackievirus
D. N. gonorrhoeae
E. N. meningitidis
pg. 196
Answer A. In the setting of a headache and nonspecific symptoms, the onset of a petechial rash
at the wrists and ankles with subsequent central spread is consistent with a diagnosis of Rocky
Mountain spotted fever (RMSF). This tick-borne illness is caused by R. rickettsii. Treatment of
RMSF is with doxycycline and admission to the hospital. B. burgdorferi is the causative
bacterium of Lyme disease. Coxsackievirus causes herpangina and myocarditis. Gonococcus may
cause a vesicular rash in association with septic arthritis, cervicitis, or pelvic inflammatory
disease (PID). Meningococcus may cause a petechial rash with signs and symptoms of
meningitis, but the onset is much more acute and severe than RMSF.
28. A 38-year-old female presents to the ED complaining of throat irritation and a 3-week
history of an episodic spastic cough. The cough is worse at night and is occasionally so forceful
that she vomits after she coughs. She is accompanied by her teenage son who has had a similar
illness for 6 weeks without abating. She is a nonsmoker. What is the most likely cause of her
illness?
pg. 197
A. Corynebacterium diphtheriae
B. Legionella pneumophila
C. Bordetella pertussis
D. M. pneumoniae
E. S. pneumoniae
29. A 65-year-old female presents with the rash shown (Fig. below). Which of the following is
the best treatment plan?
A. Supportive care
C. Valacyclovir
pg. 198
Answer C. The patient has shingles, due to varicella zoster virus. Management involves antiviral
therapy, which is especially effective for patients over 50 years of age. Valacyclovir is preferred
to acyclovir due to the reduced frequency of dosing (3x/day vs. 5x/day). Supportive care is only
considered in younger patients with mild symptoms. Antibiotics are not used for management
of shingles and should only be used if a secondary bacterial cellulitis is suspected. A
meta-analysis of five trials showed that routine use of adjunctive steroid therapy is not
beneficial for any clinical endpoints in shingles. The risk–benefit profile of tricyclic
antidepressants does not support their routine use, especially in older adults.
30. A 44-year-old female presents with the rash shown (see Fig. below). Which of the following
is true regarding this diagnosis?
A. Therapy instituted later than 72 hours after symptoms onset is of uncertain benefit
pg. 199
Answer A. The rash is that of shingles, due to varicella zoster virus. Management involves
antiviral therapy with valacyclovir, which is especially effective if given within 72 hours of
symptom onset; after this time, benefit is uncertain. Gabapentin has been shown to be
ineffective as adjunctive therapy. Postherpetic neuralgia is seen in about 15% of patients.
Patients with shingles rarely develop abscesses and their presence should trigger an evaluation
for alternate diagnosis. Viral load testing is not indicated in what is a purely clinical diagnosis.
31. A 20-year-old male presents with a painful, ulcerated lesion on his penis. He noticed it 3 days
before and the pain became progressively worse. Examination shows a tender, 1-cm ulcerated
lesion at the base of his penis with a single, large, tender inguinal lymph node. Gram stain of the
ulcer shows gram-negative bacilli. Which of the following is the most likely cause?
B. Chlamydia trachomatis
C. Staphylococcus epidermidis
D. Haemophilus ducreyi
E. Treponema pallidum
Answer D. The patient has evidence of chancroid, caused by H. ducreyi, a gram-negative bacillus.
A painful chancre-like lesion combined with a solitary tender unilateral lymph node which may
also ulcerate is classic. Chancroid, unlike syphilis (caused by T. pallidum), is painful and tender.
Treatment of chancroid is with azithromycin or ceftriaxone. HSV can cause ulcerated or vesicular
lesions, but these are usually grouped and Gram stain of the lesions will be negative. C.
trachomatis, a spirochete, may cause lymphogranuloma venereum, which is manifested by a
painless ulcer combined with significant lymphadenopathy with a negative Gram stain. S.
pg. 200
epidermidis may cause skin lesions in the genital region but Gram stain would show
gram-positive cocci.
32. Which of the following is true about acute diarrheal illnesses due to Shigella spp.?
B. Compared to infection with Salmonella spp., infection with Shigella spp. requires an unusually
large inoculum.
C. Colicky abdominal pain with high fever and diarrhea is the most common presentation.
Answer C. Abdominal pain and diarrhea occur in practically all cases of shigellosis and is often
accompanied by fever. However, only 35% to 40% of patients have evidence of blood in their
stools. Resistance to trimethoprim–sulfamethoxazole is widespread, making fluoroquinolones
the drug of choice. Antimotility drugs may be safely given if antibiotics are also administered but
are contraindicated when used alone as they may actually worsen the clinical course. Most
infections are caused by Shigella sonnei, with only a minority of infections caused by Shigella
dysenteriae. Although Salmonella spp. requires a very large inoculum to cause disease, infection
with Shigella spp. requires a very small inoculum, making Shigella the most efficient enteric
human pathogen known. Fecal leukocytes are almost universally detected in patients with
shigellosis, as Shigella invades the mucosa resulting in local destruction and inflammation.
Seizures may occur in children infected with Shigella.
33. A 19-year-old Mexican female with no past medical history is brought in to the emergency
room after a witnessed generalized seizure. Head CT reveals a round brain mass. The most likely
culprit is:
A. Trypanosoma cruzi
B. Taenia solium
C. Taenia saginata
D. Trichuris trichiura
E. Leishmania braziliensis
pg. 201
produces cysts in the brain, which appear as ring-enhancing lesions on contrast CT scan. Seizures
are the most common serious manifestation of disease. Praziquantel and corticosteroids are the
medical treatment. Neurosurgical consultation is required in all cases to evaluate and manage
increased intracranial pressure. T. cruzi causes Chagas disease, a cardiomyopathy due to
parasitic infection occurring primarily in Latin America. T. saginata, the beef tapeworm, causes a
self-limited gastroenteritis. T. trichiura, the whipworm, also causes gastroenteritis, which may
lead to iron-deficiency anemia due to malabsorption. L. braziliensis causes chronic cutaneous
ulcerations.
34. A 34-year-old male presents with skin lesions shown (see Fig. below). Which of the following
is the most likely cause?
A. Mycoplasma pneumoniae
B. HSV
C. HIV
D. HPV
E. HHV-3
Answer B. The patient has target lesions, which are a common sign of erythema multiforme. The
most common cause overall is HSV. Mycoplasma pneumoniae is the most common bacterial
cause. HIV and HPV do not usually cause erythema multiforme. HHV-3 (human herpesvirus-3) is
also known as varicella zoster virus, causing shingles and chickenpox rather than target lesions
or erythema multiforme.
pg. 202
35. Which of the following statements about Pseudomonas aeruginosa is correct?
A. Most patients with cystic fibrosis (CF) are ultimately colonized with P. aeruginosa and are
susceptible to infection.
C. P. aeruginosa is less common among patients admitted to the intensive care unit (ICU) with
severe pneumonia.
pg. 203
36. A 47-year-old previously healthy male presents to the ED with a chief complaint of fever,
severe body aches, headache, and fatigue after recently returning from a family vacation in the
Caribbean. Which of the following is the most likely cause of his symptoms?
A. Amebiasis
B. Dengue
C. Chikungunya
D. Zika
E. Malaria
Answer B. Dengue is a viral infection endemic to the Caribbean that is also the most prevalent
mosquito-borne disease worldwide. Few patients are symptomatic, but patients with “classic”
dengue fever complain of fever, headaches, fatigue, and severe myalgias and arthralgias, a
syndrome which is sometimes called “break bone fever.” There are no effective antiviral
medications for dengue so treatment is supportive. Chikungunya and Zika are also
mosquito-borne viral infections which are usually asymptomatic and for which treatment is
supportive. Chikungunya is more widely distributed in Africa while a Zika virus outbreak recently
took place in the Americas and Caribbean. Malaria is a more virulent mosquito-borne parasitic
illness that is treated with specific antimalarial drugs. Finally, amebiasis is most commonly an
asymptomatic infection caused by a protozoan, Entamoeba histolytica, that spreads through
contaminated food or water. Symptomatic patients present with dysentery from colitis or
sometimes amebic liver abscesses.
37. A 47-year-old male presents with erythema of his left foot for several days with intermittent
chills. Examination reveals a warm, erythematous, and tender dorsal foot consistent with
cellulitis. He indicates that he has had methicillin-resistant Staphylococcus aureus skin infection
in the past. Which of the following is the best treatment for this patient?
A. Doxycycline
B. Azithromycin
C. Dicloxacillin
D. Cephalexin
E. No antibiotics—watchful waiting
pg. 204
among the options presented. Azithromycin and cephalexin do not cover MRSA and are
responsible for treatment failure. Dicloxacillin is the drug of choice for methicillin-sensitive
Staphylococcus aureus but not for MRSA. Avoiding antibiotics is not a proper treatment strategy
for a clinically significant cellulitis with early signs of systemic involvement (chills).
38. A 44-year-old male with AIDS presents with chronic diarrhea, flatulence, and generalized
malaise for 1 month. Which of the following is the most likely cause of his symptoms?
A. Cryptosporidium spp.
B. Campylobacter spp.
C. Giardia lamblia
D. Escherichia coli
E. Enteromonas hominis
Answer A. Cryptosporidium is the most common cause of chronic diarrhea in patients with AIDS.
However, it is much less common in the era of successful antiretroviral therapy.
Cryptosporidiosis is usually self-limited in immunocompetent patients as well as in patients with
AIDS when the CD4 count is >180 per μL. In contrast, patients with CD4 counts <100 may
develop a chronic course of diarrhea and weight loss. Patients with CD4 counts <50 may
experience fulminant diarrhea.
39. A 42-year-old healthy male experiences crampy abdominal pain and persistent, violent
recurrent retching, and vomiting 3 hours after eating a hamburger, corn, and potato salad at a
backyard picnic. Which of the following is true about his illness?
E. This syndrome is the most common cause of acute food poisoning in the United States
Answer D. This patient experienced acute food poisoning due to Staphylococcus. The illness is
not caused by infection of the bacteria but by a heat-stable enterotoxin produced by the
bacteria before ingestion. Staphylococcus proliferates with ease in foods with a high protein
content, such as ham, eggs, poultry, custard-based pastries as well as potato or egg salads. The
pg. 205
illness occurs 1 to 6 hours after ingestion and is typically acute in onset. Abdominal pain and
vomiting are the most prominent symptoms although occasionally a mild diarrhea may also be
present. Symptoms are self-limited, typically resolving within 8 hours and only rarely lasting for
a full day. Because the disease is caused by a heat-stable toxin, cooking will not remove the
toxin once it is formed, and antibiotics have no role in treatment. Although B. cereus causes a
similar illness by virtue of a heat-stable toxin it produces, it almost always occurs after ingestion
of fried rice. Of note, B. cereus may produce a second, different syndrome characterized
primarily by diarrhea and abdominal pain. This latter syndrome results from the production of a
heat-labile toxin that is released in vivo after ingestion of live organisms. It is clinically similar to
food poisoning caused by Clostridium perfringens and results from ingestion of meats or
vegetables colonized with the bacterium.
40. Which of the following animals confers the highest risk of transmitting rabies?
A. Squirrel
B. Raccoon
C. Hamster
D. Bat
E. Rabbit
Answer B. Rabies is a virus that carries virtually 100% mortality. Humans contract the virus from
being bitten by infected animals. Raccoons are the number one vector though bats, skunks,
coyotes, and foxes all may carry rabies. Dogs and cats may also carry rabies, but the vast
majority of domesticated animal bites do not cause rabies, due to vaccination programs. The
virus is transmitted from the animal’s saliva through open skin into the victim’s bloodstream. It
then invades peripheral nerves and travels up the spinal cord to the brain. The incubation period
in humans lasts 1 to 3 months. A nonspecific viral prodrome affects most patients. Specific
symptoms of rabies include altered mental status and hydrophobia (inability to swallow water
or saliva due to hyperactive airway reflexes). Coma and death are inevitable, and no effective
treatment exists once rabies is clinically evident. Postexposure prophylaxis, consisting of local
wound care, human rabies immune globulin (HRIG), and rabies vaccine, is essential to prevent
rabies infection.
41. A 29-year-old male emergency medicine resident physician presents immediately after a
needlestick from performing a procedure on a known chronic hepatitis B (HB) virus carrier. He
has not ever received vaccination for HB. Which of the following is the next best step in
management?
A. Ribavirin
pg. 206
B. Test for hepatitis B e antibody (HBeAb)
C. HB vaccine
Answer E. The patient has never received vaccination for HB virus. Therefore, the best step in
management is to provide active immunization (with vaccine) as well as passive immunization
(with immunoglobulin). Ribavirin is used in the treatment of hepatitis C infection in combination
with interferon alpha. HBeAb demonstrates immunologic response to HBe antigen, a marker
which connotes a higher infectivity. Testing for HBeAb has no role in the evaluation of acute
hepatitis exposure. Vaccine or immunoglobulin alone is insufficient protection against exposure
in an unvaccinated patient. If the patient had ever been vaccinated, then testing his blood for
HB surface antibody (if positive indicates immunity) before treatment with vaccine and
immunoglobulin is reasonable.
42. A 26-year-old female presents with a rash on her legs. She states that she has been feeling
somewhat tired lately and notes generalized body and joint aches, as well as a sore throat. Over
the last day or so, she has noted a rash developing over her anterior shins. The rash is very
tender to the touch and is nonpruritic (Fig. below). Which of the following is true?
pg. 207
Answer B. The patient has erythema nodosum, which is thought to be a hypersensitivity
reaction to a number of different antigens. It most commonly occurs in women (female:male
ratio of 5:1) during the third decade but it frequently occurs in children as well. Patients often
experience a vague prodrome of fever, malaise, and arthralgias, followed by the development of
painful oval erythematous nodules typically over the shins. Individual lesions are not pruritic and
are usually self-limited, lasting approximately 2 weeks, although new lesions may continue to
appear such that the entire illness lasts up to 6 weeks. The most common cause is streptococcal
infection in children and streptococcal infection and sarcoidosis in adults. Other causes include
tuberculosis (TB), coccidioidomycosis, Yersinia or Chlamydia infection, inflammatory bowel
disease, Hodgkin lymphoma, pregnancy, and drugs including oral contraceptives and
sulfonamides. The lesions usually respond to high-dose aspirin (650 mg every 4 hours) or NSAIDs
(e.g., naproxen or indomethacin) and bed rest. Occasionally, patients are treated with
supersaturated potassium iodide (mechanism is uncertain). Corticosteroids are effective but are
rarely used and may worsen the underlying infection if one is present.
A. Erythema marginatum
B. Endocarditis
pg. 208
C. Migratory arthritis
D. Glomerulonephritis
E. A, B, and C
Answer E. Although acute pharyngitis is a common reason for presentation to the emergency
room, GAS is the cause of a minority of infections, ranging from 5% to 30% (estimated to be 5%
to 10% in adults, as it is a less common cause of adult pharyngitis than pediatric pharyngitis).
GAS is, however, the most common bacterial cause of the disease and, regardless of the
etiology, it is the only common cause that requires treatment. As the pharyngitis is self-limited
with or without treatment, the reason for treating GAS pharyngitis is to prevent the sequelae of
the disease, primarily acute rheumatic fever. Acute rheumatic fever is a nonsuppurative
complication of GAS pharyngitis. The other rare nonsuppurative complications include
poststreptococcal glomerulonephritis (PSGN) and streptococcal toxic shock syndrome.
Suppurative complications include peritonsillar cellulitis or abscess, otitis media, sinusitis,
necrotizing fasciitis, and meningitis. The ability of antibiotic therapy to prevent the development
of suppurative complications is not well defined, but the effect is thought to be small. However,
antibiotics are thought to be more effective in reducing the rate of acute rheumatic fever, and
they have been shown to decrease the length and severity of acute GAS pharyngitis. There is no
definitive evidence that antibiotic therapy reduces the rate of PSGN. Acute rheumatic fever
typically occurs 2 to 4 weeks after acute pharyngitis develops, and most commonly occurs in
children 4 to 9 years old. The diagnosis of rheumatic fever is made in the presence of one or
more of the five major Jones criteria in association with an antecedent GAS pharyngitis. The
major Jones criteria include: pancarditis, migratory arthritis, CNS involvement (classically
Sydenham chorea), erythema marginatum (diffuse rash sparing the face), and subcutaneous
nodules.
44. A 26-year-old previously healthy male presents with a chief complaint of a 2-day history of
abdominal cramps and multiple episodes of watery brown non-bloody diarrhea. Which of the
following is most likely?
A. Campylobacter spp.
B. Salmonella spp.
C. Shigella spp.
D. Escherichia spp.
E. Yersinia spp.
pg. 209
Answer A. Campylobacter spp. is found in the stools of 5% to 14% of patients presenting with a
chief complaint of diarrhea. The exact incidence in the United States is not known due to
under-reporting and sporadic testing.
45. A 44-year-old male presents to the ED with a 3-day history of a painful rash on his right
buttock and leg (Fig. below). He has a history of inflammatory bowel disease and is on
immunosuppressive therapy with 6-mercaptpurine and low-dose methotrexate. He first
presented to his primary care physician who referred him to the emergency room due to
concern for disseminated varicella zoster virus (VZV) infection. Which of the following is true?
C. Given the time since onset, the patient should receive corticosteroids without antiviral
treatment.
D. Mortality is most often caused by sepsis from bacterial superinfection of skin lesions.
Answer E. Patients with disseminated VZV most often present with widely distributed vesicles
outside the initially involved dermatome. Visceral involvement can occur, and includes
pneumonitis, hepatitis, and meningoencephalitis. Though bacterial superinfection of skin lesions
occurs, most deaths caused by disseminated VZV are due to VZV pneumonitis. In VZV
pneumonitis, patients develop respiratory failure which is often refractory to treatment, even
when it is started early. There is no firm evidence that corticosteroids are beneficial in the
treatment of VZV (either dermatomal or disseminated), or in the prevention of postherpetic
neuralgia. However, some experts still recommend their use, especially in patients with pain
syndromes refractory to other treatments. While the benefit of acyclovir has not been
established among immunocompetent patients presenting after 72 hours of symptoms, it
pg. 210
should be used in any immunocompromised patient, as well as in patients in whom new lesions
are actively appearing. There is no evidence of significant resistance. Ultimately, disseminated
VZV is rarely fatal, even among patients with disseminated disease.
A. Ten percent of adult patients infected with acute hepatitis A become chronic carriers.
B. Pregnant women infected with acute hepatitis E are more susceptible to fulminant hepatitis.
C. Children infected with hepatitis A virus develop symptoms more often than adults.
Answer B. Patients with hepatitis A never develop chronic disease. Approximately 100% of
patients recover from acute illness within 6 months, although fulminant hepatitis leading to
death rarely occurs. The mortality rate of hepatitis E infection during pregnancy depends on the
trimester during which a woman is infected. The maternal mortality rate is only 1.5% for
infections in the first trimester, 8.5% for those in the second trimester, and 21% for those in the
third trimester. Roughly 5% of children infected with hepatitis A are symptomatic at
presentation versus 70% to 80% of adults. In endemic areas, such as Southeast Asia, most of the
population is infected as children, and most of the community is immune by age 10. Hepatitis A
is the most common cause of viral hepatitis worldwide. Hepatitis A is transmitted through the
fecal–oral route. Although percutaneous transmission may occur (e.g., through a needlestick
injury), it is very rare because the concentration of the virus in the blood is quite low and the
duration of viremia is brief.
47. Upon examining a patient with a rash, applying light manual pressure to the skin adjacent to
the skin lesions of the rash causes the top layers of skin to peel away and rub off. Which of the
following is the likely underlying diagnosis?
A. Roseola infantum
B. Bullous impetigo
C. Pemphigus vulgaris
D. Tinea corporis
E. Erysipelas
pg. 211
Answer C. A positive Nikolsky sign occurs when pressure applied to the margin of a blistered or
ulcerated lesion expands the lesion into the adjacent apparently normal skin. This is also known
as marginal modification. In addition, direct pressure applied to normal-appearing skin that is
distant from any blistered lesions may also result in erosion or ulceration. This is known as direct
modification. These findings occur because of intraepidermal acantholysis (separation of
keratinocytes from their neighbors within the epidermis). Nikolsky sign is most commonly
associated with pemphigus vulgaris but may also be found in staphylococcus scalded skin
syndrome and toxic epidermal necrolysis. There are also multiple reports of other disease
associations, but the finding (particularly of direct modification) is very specific for pemphigus
vulgaris.
48. A 23-year-old male is bitten on his forearm by a raccoon. Which of the following is the most
appropriate anatomical region to administer human rabies immune globulin (HRIG)?
A. Deltoid
B. Gluteus maximus
D. Contralateral forearm
E. Corpora cavernosum
Answer C. Raccoons are the most common wild animals with rabies. HRIG should be
administered as close to the bite site as possible. When this is not suitable (such as in the digits),
give as much HRIG as possible at the bite site and the remainder at any skeletal muscle distant
from the site. Choices A, B, and D are all reasonable choices. The distal extremities, especially
the penis, should not be used as HRIG sites. Rabies vaccine should be administered at a site
distant from both the bite and HRIG. Along with passive and active immunization, local wound
care is extremely important, as soap and povidone–iodine cleansing solutions are 100% virucidal
when used early enough.
A. Prophylaxis with erythromycin is recommended for adults who come into contact with
pertussis infected individuals.
B. Almost all cases of pertussis in adolescents and adults occur in previously unvaccinated
patients.
pg. 212
D. Older children infected with B. pertussis have the most severe disease.
E. The clinical course in symptomatic adults is characterized by a mild cough that resolves within
3 to 7 days.
Answer A. The incidence of B. pertussis infection is rising, with the number of reported cases in
the United States increasing sixfold since 1980. This has occurred despite immunization rates of
80% among young children. Most of the increase is due to an increased number of adolescents
and adults diagnosed with the disease. Furthermore, it is widely thought that the true scope of
the problem is grossly underestimated because of the failure of physicians to recognize the
illness, as well as their failure to report the illness when it is diagnosed. Almost all cases of
pertussis in adolescents and adults occur in patients who have been previously vaccinated or in
patients who have been previously infected with B. pertussis. Contrary to popular belief, neither
B. pertussis infection nor vaccination with either the cellular or acellular vaccine confers lifelong
immunity. In fact, natural infection with B. pertussis results in approximately 15 years of
immunity to reinfection. This is much greater than the 4 to 8 years’ worth of immunity offered
by the vaccines (the acellular vaccine, which is currently used in the United States, offers a
shorter duration of immunity than the cellular vaccine, roughly from 4 to 6 years). The
recommended vaccination schedule in the United States advises that infants be vaccinated at 2,
4, and 6 months of age, with boosters at 18 months and then again between 4 and 6 years.
Therefore, most children born in the United States should carry immunity through the ages of
10 to 12 years. Since the most severe illness occurs in children less than 1 year, adolescents and
adults have not been offered booster shots beyond childhood. However, because of the rising
incidence of recognized disease in adolescents and adults, as well as the likely enormous
asymptomatic disease burden in this population, the CDC and the US Advisory Committee on
Immunization Practices (ACIP) now recommends that all adults under 65 receive a single
acellular booster along with tetanus and diphtheria in Tdap (Boostrix), which is often given in
the ED in the setting of trauma (in place of isolated tetanus toxoid). Patients >65 years old
receive Adacel, a different Tdap formulation. B. pertussis infection in adults ranges from
subclinical infection to a prolonged illness mostly characterized by a nagging, paroxysmal cough,
which may interfere with sleep. The mean duration of illness in adults is 36 to 48 days.
Erythromycin is the drug of choice for the treatment of B. pertussis infection although
azithromycin and clarithromycin have been shown to be equally efficacious with fewer side
effects. TMP-SMX is an additional alternative. Erythromycin is also recommended for
prophylaxis of individuals who have come into contact with patients who are infected. It is
thought to be effective in preventing disease as long as it is given before the onset of symptoms.
Owing to the decreased infectivity of B. pertussis as the disease progresses, prophylaxis is
generally unnecessary in individuals who come into contact with a patient who has been
symptomatic for >3 weeks.
50. A 36-year-old female presents to the ED with a chief complaint of finger pain and swelling.
She had previously been treated for an acute paronychia a few months ago, but states it “won’t
go away.” Examination reveals a dystrophic nail, loss of the cuticle, as well as an area of tender,
pg. 213
erythematous swelling along the proximal nail fold, without abscess formation. This chronic
paronychia is best treated with:
A. Topical corticosteroids
B. Oral antibiotics
C. Oral antifungals
E. Oral antivirals
Answer A. Due to the appearance of the nail, the cause of chronic paronychia has traditionally
been attributed to fungal superinfection. However, studies demonstrate a much better response
to topical glucocorticoids than to oral antifungals. Thus, it is most likely due to an underlying
dermatitis and is most likely a manifestation of eczema. As a result, patients are instructed to
keep their hands dry and use gloves for any type of wet work.
51. A 23-year-old female without past medical history presents with acute onset of watery
diarrhea and abdominal cramping 3 hours after eating dinner at a local restaurant. She has no
nausea, vomiting, or fever. Her vital signs are 98.6°F, 85, 18, 115/70, 99% RA. She is nontoxic,
and her abdominal examination shows minimal diffuse tenderness. Which of the following is the
most appropriate next step in management?
A. Oral ciprofloxacin
B. Oral metronidazole
Answer E. The patient’s acute gastroenteritis is likely from food poisoning with a preformed
toxin. The most common causes are Clostridium perfringens and S. aureus. Management for
both toxin-induced and viral gastroenteritis involves supportive care only. Antibiotics are of no
value as the toxin is already preformed. Stool leukocytes are rarely helpful in the management
of gastroenteritis as they seldom change management due to poor specificity. Ova and parasite
examination is helpful in cases of subacute, watery diarrhea lasting longer than 1 week and is
not routinely used in acute diarrheal episodes.
pg. 214
52. You are following up on a positive chlamydia Polymerase chain reaction (PCR) result for a
24-yearold female who presented to the ED 2 days prior. Her gonorrhea result is negative. Chart
review indicates that she had no past medical history and lives and works on a Caribbean island
as a lifeguard, but is still in town for 1 more day. Which of the following is the best treatment
regimen for this patient?
Answer A. One-time azithromycin dosing is the standard of care for chlamydial infection.
Doxycycline therapy for 7 days is also reasonable, but it carries with it the risk of significant
reaction due to photosensitivity. A lifeguard is likely to be exposed to significant sunlight and
doxycycline would not be the best choice for this patient. Albendazole is an antiparasitic used in
neurocysticercosis infection and does not have a role in chlamydial infection. Ceftriaxone would
be the drug of choice for patients with gonococcal infection. Vancomycin is not considered an
appropriate treatment for uncomplicated chlamydial infection.
53. A 29-year-old male presents to the ED with a 3-week history of diarrhea, crampy
intermittent abdominal pain, and a 10-lb weight loss. He denies any bloody stools or vomiting
even though his stool guaiac test is positive. He returned from a 1-month trip to India
approximately 6 weeks ago and had no problems while he was there. Which of the following is
the most likely cause of his symptoms?
A. Shigella spp.
B. Enterotoxigenic E. coli
C. Enterobius vermicularis
D. Entamoeba histolytica
E. Campylobacter spp.
Answer D. Bloody diarrhea in a traveler or immigrant from an endemic area should always raise
the possibility of amebic colitis. While patients with bacterial dysentery can also have bloody
stools and crampy abdominal pain, their symptoms do not typically last for longer than 1 week.
Enterotoxigenic E. coli is the most common cause of traveler’s diarrhea but is noninvasive,
causes a non-bloody, water diarrhea, and typically resolves within a few days to a week.
pg. 215
Entamoeba is found worldwide, but it is particularly endemic to the Indian subcontinent, central
and South America, and Africa. Extraintestinal manifestations, the most common of which is a
liver abscess, only rarely occur.
54. A 42-year-old male presents in January to your emergency department (ED) with fever,
myalgias, cough, sore throat, and malaise for 4 days. He has no past medical history. His vitals
are: 100.0, 86, 16, 118/72, 100% RA. His physical examination is unremarkable. Which of the
following is the next best step in management?
A. Prescribe oseltamivir
B. Prescribe amantadine
C. Prescribe azithromycin
Answer E. The patient has signs and symptoms of influenza virus at the peak time for influenza
season. He exhibits no signs of pneumonia or dangerous vital signs warranting further
evaluation. Oseltamivir may be useful to reduce duration of illness in the early setting (before 48
hours of symptoms), but is not effective after this in a previously healthy patient. Amantadine
and other M2 viral proton channel blockers are no longer indicated for treatment of influenza
due to nearly 100% resistance. Azithromycin, while commonly overused in this exact clinical
setting, is not indicated in this patient without evidence of pneumonia. Ampicillin–sulbactam, a
powerful beta-lactam antibiotic, is similarly not warranted in this patient without evidence of
bacterial infection.
55. Which of the following are the two most common causes of diarrhea in AIDS patients?
Answer A. CMV each causes one-fifth to one-third of all cases of diarrhea in AIDS patients. The
other choices do cause diarrheal illness in AIDS patients but far less commonly than choice A.
pg. 216
56. A 27-year-old male presents to the ED with pruritus ani, tenesmus, and yellowish mucoid
discharge from his rectum. Upon further questioning, he acknowledges recent unprotected anal
intercourse. The recommended regimen for treating this patient is:
E. Lopinavir
Answer B. Any patient with a history of recent unprotected anal intercourse who presents with
symptoms of proctitis should be treated empirically for N. gonorrhoeae proctitis. Because
concurrent infection with Chlamydia trachomatis is common in patients infected with
gonorrhea, empiric therapy should cover this organism as well. Podophyllin is a treatment for
human papillomavirus (condyloma acuminatum). Valacyclovir is a treatment for herpes proctitis,
penicillin is a treatment for syphilis, and lopinavir is a protease inhibitor used to treat human
immunodeficiency virus (HIV). Owing to this patient’s high-risk lifestyle, he should undergo
testing for HIV and for syphilis but empiric treatment is not necessary.
57. A 52-year-old diabetic man presents to the ED with fever, crampy abdominal pain, and
watery brown diarrhea. He recently completed a 14-day course of clindamycin, which was
prescribed by his primary care doctor after she performed an incision and drainage of a small
cutaneous abscess on his flank. The patient’s symptoms started toward the end of his antibiotic
therapy and he has been taking diphenoxylate “around the clock” since then without much
benefit. Which of the following is true?
B. The patient’s diarrhea is an expected side effect of his recent antibiotic therapy.
pg. 217
associated with the development of C. difficile colitis. However, the use of other antibiotics such
as cephalosporins and fluoroquinolones may also result in colitis. The gold standard for
diagnosis is considered to be a cell cytotoxicity assay but due to the cost and labor involved,
most laboratories use a toxin detection assay. Positive stool cultures are not diagnostic because
the bacteria are often present in healthy subjects and are increasingly present in patients who
have been on antibiotics. Diphenoxylate (Lomotil) or other antimotility agents may worsen the
disease by allowing further overgrowth and increased time for toxin action. Colonoscopy is not
required for treatment but may help to rule out other causes or confirm the diagnosis. In mild
cases, withdrawal of the offending antibiotic may be all that is necessary. If symptoms do not
rapidly resolve, metronidazole is the treatment of choice with vancomycin reserved for
refractory cases. Children with C. difficile colitis typically have a more severe course, especially
in children undergoing chemotherapy.
58. A 30-year-old male presents with right upper quadrant pain and fever. He has just returned
from a trip to Brazil. His vital signs are 101.0°F, 105, 22, 155/90, 100% RA. He is exquisitely
tender to palpation in the right upper quadrant. You order an ultrasound, which shows a
cavitary, fluid-filled lesion in the right hepatic lobe and a normal gallbladder. Which of the
following is the most appropriate initial treatment?
A. Vancomycin
B. Amikacin
C. Metronidazole
D. Methylprednisolone
E. Surgical drainage
Answer C. The patient has evidence of a liver abscess. Overall, the most common type is
pyogenic, which is usually polymicrobial in etiology. However, with the patient’s history of
foreign travel to an endemic country, amebic liver abscess, caused by the parasite E. histolytica,
is most likely. Metronidazole therapy is the most appropriate initial choice of treatment. If there
is no clinical response to metronidazole, then surgical drainage or drainage by interventional
radiolog (IR) is considered. Amebiasis affects about 10% of the world’s population, focused
specifically in Central and South America and Asia. It usually causes intestinal disease, but
hepatic involvement is the most common extraintestinal manifestation. Pulmonary symptoms
can occur due to inflammation and frank pulmonary involvement with either right-sided pleural
effusions or abscess rupture into the pulmonary airspaces. Vancomycin or gentamicin alone is
not adequate therapy for either pyogenic or amebic liver abscess. There is no role for steroid
monotherapy in this infectious process.
pg. 218
59. A 42-year-old male presents with pain, warmth, and swelling over his posterior elbow (Fig.
below). The patient reports frequently having to lean on his elbow while performing electrical
work as part of his job. The patient has full range of motion but flexion and extension of the
elbow results in increased pain. Fluid is subsequently aspirated from the affected area. Which of
the following is true?
C. Septic bursitis may be present if the bursal white blood cell count is >10,000 per mm3
Answer C. This patient has olecranon bursitis. Olecranon bursitis most commonly occurs
because of repetitive microtrauma caused by leaning or rubbing of the elbow. Hematogenous
spread affects deeper bursae. Although most cases of olecranon bursitis are sterile
inflammatory reactions, septic bursitis may account for as many as 33% of olecranon bursitis
cases. Furthermore, because of the superficial location, septic bursitis is most common in the
olecranon and prepatellar bursae, and rarely occurs elsewhere. Predisposing factors to septic
bursitis include patients who are immunocompromised because of diabetes, renal insufficiency,
and cancer as well as patients with anatomic abnormalities of the joint spaces and surrounding
structures such as patients with gouty or rheumatoid arthritis (both of which may involve the
bursa). Aspiration of the bursa is the only reliable means available to help differentiate between
pg. 219
septic and sterile bursitis. In contrast to the higher number of leukocytes in septic arthritis,
septic bursitis may be present when the WBC count is only 10,000 per mm3 or lower (one study
used 2,000 as a threshold). However, the average cell count in septic bursitis exceeds 60,000.
Gram stain is positive in only 50% of cases, and Staphylococcus aureus is the most common
responsible organism.
60. Which of the following statements is true regarding scarlet fever and acute rheumatic fever?
B. Acute rheumatic fever is a complication of acute GAS pharyngitis that occurs more commonly
in adults.
C. Scarlet fever occurs acutely with GAS pharyngitis, but acute rheumatic fever does not typically
occur until 2 to 4 weeks after GAS pharyngitis.
D. Scarlet fever is now a rare occurrence due to the H. influenzae type B (HiB) vaccine.
E. Both scarlet fever and acute rheumatic fever can be prevented by the use of antibiotics for
GAS pharyngitis.
61. Which of the following is the most effective method of eliminating symptoms in acquired
immune deficiency syndrome (AIDS) patients with Cryptosporidium diarrhea?
pg. 220
A. Loperamide
C. Azithromycin
D. Metronidazole
E. Octreotide
Answer B. Cryptosporidium parvum is a parasite, which often causes subacute and chronic
diarrhea in patients with AIDS. HAART is the best treatment for Cryptosporidium diarrhea.
Symptoms are virtually eliminated if CD4 counts are maintained >100 cells per μL. Antidiarrheal
agents and antibiotics work with only varying degrees of success and the symptoms are often
recurrent after these drugs are stopped. Octreotide has no role in the management of
HIV-associated infectious diarrhea.
62. A 22-year-old diabetic man presents 1 day after being punctured in his left arm during a
hiking accident. Since the accident, he has noted increasingly intense pain in his arm along with
mild swelling and redness. On examination, his arm is noted to be mildly swollen and
erythematous with an innocuous appearing puncture wound on the volar aspect of his right
forearm. His arm is extremely tender, although there is no crepitus. A plain film is obtained
which reveals subcutaneous emphysema. Which of the following is the next best step in
management?
B. Surgical consult
Answer B. This patient has necrotizing fasciitis caused by Clostridium spp., which gained entry
when his arm was punctured while hiking. Clostridium perfringens is the most common species
and is prevalent in soil. Pain is the most common early finding and is generally intense and
unremitting. Swelling, pallor, and tenderness rapidly develop. Hemorrhagic bullae and brownish,
serosanguineous discharge may develop as the wound progresses. Crepitus may also be present,
but it is neither a sensitive nor specific finding. Treatment should be prompt, including
aggressive surgical debridement and intravenous antibiotics. Despite this, amputation is
frequently necessary. Use of advanced imaging to delineate spread along fascial planes delays
definitive therapy and may result in a worse outcome.
pg. 221
63. A 27-year-old male presents with a rash (Fig. 9-2). The patient states that his wife first
noticed a single spot on his back approximately 10 days ago. He denies fever or pain and
complains only of mild pruritus. Over the last few days, many new “spots” have cropped up on
his back and trunk. On examination, you note that the lesions have a fine scale around the
border and seem to be arranged along the skin lines of the back. Which of the following is the
most likely diagnosis?
A. Secondary syphilis
B. Pityriasis rosea
C. Molluscum contagiosum
D. Tinea corporis
E. Atopic dermatitis
Answer B. This patient has pityriasis rosea, which is a common, self-limited rash that is thought
to be of viral etiology. It most commonly occurs between the ages of 10 and 35 with a mean age
of 23. Approximately 20% to 50% of patients present with a “herald patch,” a single, 2- to 10-cm
salmonpink plaque with a fine silvery scale that rings the border of the lesion. Many patients
believe they have ringworm, and the lesion may commonly be mistaken for ringworm before
the development of additional lesions (KOH testing can rule this out). Approximately 1 to 2
pg. 222
weeks after the herald patch, multiple additional 1- to 2-cm round and oval lesions develop on
the trunk. The longitudinal axis of the oval lesions runs along the skin lines of the back and the
overall pattern may resemble the branches of a pine tree. For this reason, pityriasis is often said
to have a “Christmas tree” distribution. No treatment is necessary and the lesions resolve within
2 to 3 months, although ultraviolet B (UVB) phototherapy will hasten resolution and decrease
pruritus.
64. Security is called to help restrain an agitated patient in the ED. In helping to restrain the
patient, one of the officers is inadvertently stuck by a contaminated needle that the nurse was
using to obtain an IV. The patient is known to have chronic active hepatitis B (HepB) and the
officer says he was immunized once against HepB but is a “nonresponder.” Which of the
following is true?
A. Passive immunization with hepatitis B immune globulin (HBIG) but not active immunization
with the HepB vaccine should be given.
B. The HepB vaccine is incompatible with typical prophylactic drug therapy for human
immunodeficiency virus (HIV).
C. The HepB vaccine should still be given since many initial nonresponders will respond to a
second HepB vaccine series.
D. The patient’s wound should be washed with a dilute bleach solution to denature the protein
coat of the virus.
E. HepB is transmitted much less effectively than hepatitis C through needlestick injuries.
65. A 34-year-old female presents with an infection. You decide to prescribe penicillin. She
states that she is allergic to penicillin but cannot recall the reaction and has never been skin
pg. 223
tested. You consider switching to a cephalosporin. Which of the following is the best estimate
for cross-reactivity in this patient?
A. <1%
B. 10%
C. 25%
D. 33%
E. 50%
Answer A. Patients with reported penicillin allergies rarely have true immune-mediated allergic
reactions. Most simply have one of a variety of adverse medication effects. The historically
quoted cross-reactivity rate between cephalosporins and penicillins of 10% is likely flawed. Early
studies demonstrating this 10% rate occurred at a time when penicillin impurities were present
during cephalosporin production in laboratories. Furthermore, most second-, third-, and
fourth-generation cephalosporins have a penicillin cross-reactivity rate no greater than other
antibiotics. Current estimates of cross-reactivity in patients with reported penicillin allergies are
<1%, and some experts argue that it is far less than 1%. Patients with confirmed true allergic
reactions to penicillin still have a <5% cross-reactivity rate. Patients who report a
non-anaphylactic allergy to penicillin can safely receive a cephalosporin if watched briefly in the
ED for adverse events.
66. A 32-year-old female presents with cough for 1 week following several days of upper
respiratory symptoms. Her vital signs are: 99.2, 75, 16, 119/68, 100% on RA. Her physical
examination is normal. You strongly suspect acute bronchitis. Which of the following about this
diagnosis is true?
pg. 224
Sputum color has no proven association with bacterial illness in bronchitis. NSAIDs are perhaps
the most important element of supportive care recommended for the vast majority of cases of
acute bronchitis (except in geriatric patients). Admission to the hospital for a patient with this
chief complaint and with no comorbidities, normal vital signs, and a normal physical
examination is not warranted.
A. Food-borne
B. Infant
C. Wound
D. Respiratory
E. Cardiac
68. A 25-year-old G1P0 presents at 18 weeks of gestation with a chief complaint of painless
vaginal discharge (Fig. below). Her pregnancy has progressed normally to date. Speculum
examination reveals an adherent whitish discharge with a pH of 6.0. The wet prep is shown.
Which of the following is true?
C. No treatment is required until the third trimester, after organogenesis has occurred.
D. Treatment is elective, as the infection poses no health risks to the mother or fetus.
pg. 225
Answer A. This patient has bacterial vaginosis (BV), which is the most common lower genital
tract infection among women of reproductive age. Clue cells, which are squamous vaginal
epithelial cells coated with bacteria, are evident on the wet mount. Classically, the disease has
been diagnosed when three of the four Amsel criteria are present:
• Vaginal pH >4.5
Though these criteria remain widely used, they have been criticized for their subjectivity. New
assays are in development. Though Gardnerella vaginalis is present in approximately 95% of
cases, BV is a polymicrobial infection with poorly understood origins. Furthermore, although it
was originally considered a relatively benign illness, recent research has shown a clear
correlation with preterm labor and delivery, preterm premature rupture of membranes,
spontaneous abortion, chorioamnionitis, and postpartum infections such as endometritis.
Therefore, all symptomatic pregnant and nonpregnant women with BV should be treated. As in
nonpregnant women, metronidazole is the treatment of choice (250 mg orally three times daily
for 7 days) although clindamycin is an acceptable alternative (300 mg orally twice daily for 7
days).
69. Which of the following is true regarding genital herpes simplex infection?
pg. 226
D. Vesicles should be unroofed to allow drainage of fluid.
Answer A. Herpes simplex virus (HSV) type 2 causes most genital herpes infections. Primary HSV
infection is almost always more severe than recurrences. Outbreaks can occur with any systemic
or local stress. The lesions are usually extremely painful and irritating and should never be
drained or unroofed as this will inoculate other sites. The sensitivity and specificity of the Tzanck
smear are both <80% and viral culture is the gold standard for diagnosis. Antibiotics should not
be used in the absence of clear signs of bacterial superinfection. Treatment of HSV outbreaks is
with acyclovir, famciclovir, or valacyclovir. Continuous treatment may be necessary to prevent
outbreaks in susceptible individuals.
Answer E. Tinea versicolor is a fungal infection of the skin caused by Pityrosporum ovale (oval
form) and Pityrosporum obiculare (round form). These organisms were previously called
Malassezia furfur. Tinea versicolor is a benign and common fungal infection of the skin that
most commonly occurs in areas of the skin with increased sebaceous activity. It is most common
in the upper trunk but it commonly spreads to the arms, neck, and abdomen. Lesions may be a
variety of colors, but are classically hypopigmented white or tan macules and patches.
Griseofulvin is not effective against these fungi, but multiple other agents are effective,
including topical ketoconazole, selenium sulfide, and terbinafine as well as oral itraconazole,
fluconazole, and ketoconazole. Although such therapy is highly effective, recurrence is common
after it is discontinued.
71. A 25-year-old female presents with white vaginal discharge. Physical examination
demonstrates grayish-white discharge with no cervical lesions. A wet mount is positive for the
pg. 227
presence of clue cells. The patient states she developed a rash after several days of treatment
for an identical problem 1 year ago. Which of the following is the best therapy for this patient?
B. Metronidazole 2 g PO once
E. Azithromycin 1 g PO once
Answer C. The patient has evidence of bacterial vaginosis, a bacterial overgrowth process due to
polymicrobial infection with Gardnerella, Mycoplasma, and anaerobes. Clue cells refer to vaginal
squamous epithelial cells lined with bacteria. The sniff test (fishy odor of the discharge with
addition of potassium hydroxide) may also be positive in bacterial vaginosis. Treatment with an
antibiotic to cover anaerobic bacteria must be instituted for at least 1 week, unlike in cases of
Trichomonas vaginitis, in which one-time therapy with a 2-g dose of metronidazole is adequate.
First-line therapy for bacterial vaginosis is with metronidazole 500 mg b.i.d. for 7 days, but
clindamycin is an acceptable alternative. The single, 2-g dose of metronidazole is not efficacious
and not recommended. Furthermore, the patient’s report of a rash after being treated for a
similar problem raises the possibility of an allergy to metronidazole, so it should be avoided,
even if the prior rash only appeared after several doses. Fluconazole is used to treat candidal
vaginitis. Ceftriaxone is used to treat gonorrheal infections. Azithromycin is used to treat
chlamydial infections and a number of other sexually transmitted diseases.
72. A 24-year-old previously healthy woman presents with a cutaneous abscess on her left thigh.
She is afebrile and her thigh examination reveals a tender, fluctuant area 4 cm × 6 cm in size.
There is no surrounding erythema. In addition to incision and drainage, which of the following
has been shown to provide the better outcomes in abscess management?
E. None of these
pg. 228
reasonable pain control for these patients, but many may require opiate pain medications.
Antibiotics are indicated in patients with signs of cellulitis. In many communities,
methicillin-resistant Staphylococcus aureus must be covered by the antibiotics
selected—cephalexin alone is not adequate for these patients.
73. A 36-year-old female with a history of asthma presents with the following rash (Fig. below).
It started out as exquisitely pruritic normal-appearing skin and then progressed to this after she
scratched it frequently for several days. Which of the following is the most likely diagnosis?
B. Eczema
E. Erysipelas
Answer B. The history of normal-appearing skin with intense pruritus gives eczema the moniker,
“the itch that rashes.” Patients often complain of significant pruritus with minimal findings.
Once they scratch the area, the characteristic maculopapular lesion with scales will appear.
Treatment includes topical steroids, which must be of sufficient potency to reduce the
inflammation—often over-the counter 1% hydrocortisone cream is not enough to manage these
outbreaks. Prevention with adequate skin moisturization is essential. Group A streptococcal skin
infection will cause a cellulitis-like picture, with definite erythema, warmth, tenderness, and
generally lacking in scales. An example of Group A streptococcal infection is erysipelas, which is
a sharply demarcated, bright red area of skin. Toxic epidermal necrolysis involves significant
areas of skin exfoliation along with evidence of dehydration and possibly multiorgan
pg. 229
dysfunction. Toxic shock syndrome occurs with the classic “sunburn” rash in a patient with
hypotension and multiorgan failure.
74. A 24-year-old female presents with a diffuse rash (Fig. 10-12). She had “sores” of her mouth
and eyes as well as numerous “spots” on her trunk, but now the rash has spread throughout her
body. She is uncomfortable and her skin is warm and tender. Pressure applied to skin adjacent
to the lesions seems to extend the lesion into the normal-appearing skin. In exploring her recent
history, which of the following is most likely?
Answer B. The patient has toxic epidermal necrolysis (TEN), which is a vesiculobullous disease
that is characterized by diffuse epidermal detachment. It is part of a continuum with
Stevens–Johnson syndrome (SJS) and is diagnosed when the degree of epidermal detachment
exceeds 30% of the body surface area (whereas SJS is diagnosed when epidermal detachment is
<10%). Drug exposure, particularly to sulfonamides, and to a lesser degree, anticonvulsants, is
the most common cause of SJS and TEN. A recent urinary tract infection implies that this patient
had been taking trimethoprim/sulfamethoxazole or possibly nitrofurantoin when she developed
TEN. When normal appearing skin sloughs or develops ulceration in response to gentle pressure,
patients are described as having a positive Nikolsky sign.
75. Which of the following tick-borne illnesses is responsible for the most deaths?
pg. 230
A. Rocky Mountain spotted fever
B. Ehrlichiosis
C. Lyme disease
E. Babesiosis
Answer A. Rocky Mountain spotted fever (RMSF), caused by Rickettsia rickettsii transmitted by
the Dermacentor tick, is responsible for the most tick-related deaths in the United States.
Despite its name, cases are seen most commonly in the southeastern states, but nearly all states
have reported cases. The characteristic symptoms are fever, constitutional symptoms,
abdominal pain, and a centripetal rash (spreading from extremities to trunk). Antibiotic therapy
(specifically with doxycycline) has steadily improved the mortality to about 1%. Doxycycline can
even be given to children for optimal treatment of RMSF, as no other antibiotic improves
outcomes as well. The other tick-borne diseases mentioned are rarely associated with high
mortality and are not as common as RMSF.
Answer E. Babesiosis is a tick-borne protozoan illness that results in hemolytic anemia (after
infection, it “lives” inside red blood cells). Clinical infection ranges from mild to severe. Most
patients will have a subacute illness consisting of vague constitutional symptoms such as fever,
general malaise, and cough, as well as weight loss and night sweats. Immunocompromised
patients are at highest risk of severe infection, particularly asplenic patients. Diagnosis is made
by pathologist examination of a thin blood smear (like malaria). A combination of atovaquone
and azithromycin is the recommended treatment.
pg. 231
C. The sensitivity of radiographs is higher early in the illness.
Answer E. Osteomyelitis is a bacterial infection of the bone that generally follows a subacute
course. The most common cause is S. aureus, but streptococci and gram-negative bacilli are also
implicated. Patients usually complain of pain in the affected bone, but do not appear toxic and
often lack vital sign abnormalities. The sensitivity of ESR is approximately 90%, and the
sensitivity of C-reactive protein may be even higher. Radiographs, although commonly used to
evaluate osteomyelitis, have notoriously poor sensitivity in the first week after the onset of
symptoms. Bone scintigraphy and MRI are the tests of choice in diagnosing osteomyelitis. MRI is
superior to CT scan in characterizing the infection.
78. A 28-year-old female presents with a low-grade fever, arthralgias, fatigue, and the rash
shown (Fig.below). Which of the following is the best therapy?
A. Acyclovir
B. Azithromycin
C. Doxycycline
D. Amoxicillin–clavulanic acid
E. Supportive care
pg. 232
Answer C. The rash depicted is erythema chronicum migrans, a sign of early Lyme disease. A
nonspecific viral syndrome accompanies this about a week after a tick bite carrying the
spirochete B. burgdorferi. First-line treatment for Lyme disease is doxycycline; amoxicillin is the
second-line treatment. Acyclovir would not be appropriate here given the nonviral etiologic
agent. Azithromycin can be used to treat Lyme disease, but is the third-line agent, because it is
not as effective as either doxycycline or amoxicillin. Amoxicillin–clavulanic acid would be
unnecessarily broad therapy for Lyme disease. Supportive care is not appropriate in the early
stage of the illness.
B. Fecal excretion
C. Viremia
D. Transaminitis
E. Jaundice
Answer C. Hepatitis A virus infection is extremely common worldwide, but many cases are
asymptomatic. Viremia starts first (2 to 4 weeks postexposure), followed by fecal excretion (4 to
6 weeks), transaminitis and nonspecific symptoms such as nausea and vomiting (5 to 10 weeks),
and jaundice (6 to 10 weeks). Jaundice is usually the first symptom that causes patients to seek
medical attention. By that time, contagiousness and viremia are minimal.
6. A 20-year-old male presents with fever, sore throat, and fatigue for 1 week. He took a friend’s
antibiotic after 2 days of illness and developed a red rash all over his body, so he stopped the
antibiotic. His oropharynx is mildly red without exudates, and he has cervical lymphadenopathy
and splenomegaly. Rapid strep testing and monospot are both negative. Which of the following
is the most likely diagnosis?
B. Infectious mononucleosis
C. Influenza
D. Herpangina
E. Diphtheria pharyngitis
pg. 233
80. A 15-year-old male presents to the pediatric emergency room with a chief complaint of
burning pain to the “tip” of his penis. He is uncircumcised and denies any past history of
genitourinary disease. He admits to being sexually active and uses condoms intermittently but
denies penile discharge. Physical examination reveals an erythematous, mildly edematous glans
without penile discharge. The scrotal examination is unrevealing and there is no inguinal
lymphadenopathy. His blood sugar on a finger stick is 96. Which of the following is the most
likely cause of his problem?
A. Chlamydia trachomatis
B. Escherichia coli
C. Gardnerella vaginalis
E. Candida albicans
Answer A. This patient is presenting with symptoms and findings of balanitis, inflammation of
the glans of the penis. When it is accompanied with inflammation of the foreskin, it is called
balanoposthitis. In adults, balanitis is frequently a sexually transmitted disease (due primarily to
Chlamydia and Neisseria) or a fungal complication of diabetes (Candida). In children, the cause is
more often multifactorial but is most commonly associated with poor hygiene among smaller
children. In a sexually active adolescent using intermittent protection, the most likely cause is a
sexually transmitted disease, and Chlamydia is the most common of these. Penile swabs can be
obtained for a saline preparation, KOH preparation, group A Streptococcal culture, as well as for
PCR testing for Chlamydia and Neisseria to help determine the cause in cases in which it is
unclear.
81. A 34-year-old female with acquired immune deficiency syndrome (AIDS) presents after a
first-time seizure. Which of the following is the most likely cause of her seizure?
A. Mass lesion
B. Meningitis
C. Stroke
D. Electrolyte abnormality
E. Syphilis
Answer A. The single most common identifiable cause of seizures in human immunodeficiency
virus (HIV)/AIDS patients is toxoplasmosis, which causes seizures through mass effect. Other
mass lesions such as malignancy and abscess are additional causes. In cases that are not due to
pg. 234
any identifiable cause, HIV-encephalopathy is postulated to be the etiology. Meningitis, usually
cryptococcal, is the second most common identifiable cause. Electrolyte abnormalities, stroke,
and neurosyphilis are less commonly implicated.
82. Clinically, condyloma acuminatum may be differentiated from condyloma lata by:
D. Foul odor
83. A 20-year-old previously healthy female college student presents to the ED with diarrhea.
She went on a camping trip 2 weeks ago but denies any other travel. She notes seven to eight
watery, foul smelling stools per day and generalized abdominal cramping. A test for fecal
leukocytes, ordered in triage after the patient had a large diarrheal stool, is negative. Which of
the following is the best management?
Answer B. This patient is infected with Giardia lamblia, the most common enteric parasite
infection worldwide. It is most commonly acquired through drinking contaminated water.
However, foodborne and person-to-person transmissions also occur. Backpackers frequently
pg. 235
contract the illness after drinking inadequately treated or untreated stream water that is
contaminated by animal or human fecal matter. The diarrheal illness is therefore often known as
backpacker’s diarrhea. Giardiasis is a noninflammatory, noninvasive diarrhea, so leukocytes are
not seen in stool samples. Instead, diagnosis relies on the detection of trophozoites or cysts
(“ova and parasites”) in stool specimens. Detection of ova and parasites varies from 60% to 80%
after a single stool to greater than 90% after three stools. However, when the clinical history is
consistent with giardiasis, a diagnosis can be made presumptively. To prevent chronic infection
as well as person-to-person transmission, both symptomatic and asymptomatic patients should
be treated. The treatment of choice is metronidazole 250 mg t.i.d. for 7 days and is generally
well tolerated.
84. Among adults, which of the following is the most common cause of acute diarrheal illness in
the United States?
A. Viruses
B. Escherichia coli
C. Campylobacter spp.
D. Giardia lamblia
E. Staphylococcus aureus
Answer A. Noroviruses, which include Norwalk virus, are responsible for 50% to 80% of all cases
of acute infectious diarrhea. However, most patients with acute infectious diarrhea do not seek
medical treatment. Patients who seek medical attention are more likely to have a bacterial
cause, most commonly Campylobacter spp.
85. A 24-year-old female presents with persistent cough for 4 weeks. She had upper respiratory
infection (URI)-like symptoms 2 weeks before and then developed a persistent cough for the
next month. She states she has had coughing fits many times during the day and “can’t stop
coughing” for almost a minute once she starts. Which of the following is true regarding this
patient?
pg. 236
Answer A. The patient has evidence of whooping cough caused by B. pertussis, a gram-negative
coccobacillus. According to the CDC’s definition, whooping cough should be considered in the
absence of a more likely diagnosis when a patient has a “cough illness” lasting >2 weeks with
one of the following symptoms: Paroxysms of coughing, or inspiratory “whoop,” or post-tussive
vomiting, or apnea in infants <1 year. The disease occurs in three phases—the catarrhal phase, a
nonspecific URI-like syndrome lasting 1 to 2 weeks; the paroxysmal phase lasting up to 1 month,
with paroxysms of coughing fits; and the convalescent phase lasting up to several months, with
a chronic, intermittent cough. Antibiotic therapy with macrolides is usually only effective in the
catarrhal phase, but should be given to patients to reduce the high degree of contagiousness.
Cultures are useful only in the catarrhal phase, and have low sensitivity during the paroxysmal
phase. Mortality is low and usually due to superinfection, most commonly from pneumonia.
86. A 72-year-old male presents with a rash on his right abdomen (Fig. below). He complains of a
1-week history of burning discomfort extending from his low back to his low right abdomen, and
a 1-day history of a rash over his abdomen only. The area “tingles” and burns and is “sensitive”
to the touch. Which of the following is the most likely etiologic agent?
E. Parvovirus
Answer B. The patient has a dermatomal rash consistent with shingles, caused by VZV. Shingles
is due to reactivation of dormant VZV—primary VZV is also known as chickenpox. Many patients
with shingles will have a prodrome of pain before the rash appears, and vesicles are not always
present. However, the description of this patient’s pain is typical of shingles, and pain is, in
pg. 237
general, the most common complaint for which patients seek evaluation and treatment.
Immunocompromised patients may have disseminated zoster infection, which spreads beyond
the initial dermatome and ultimately results in systemic involvement. Older patients (>50 years
of age) with shingles recover faster with antiviral therapy if treatment is started within 72 hours
of the onset of symptoms. Otherwise, treatment is supportive with effective analgesia. While
glucocorticoids are often used, their use does not reduce postherpetic neuralgia so they are not
recommended. Steroids appear to hasten recovery, but their benefit is modest, and their use is
associated with significant undesirable side effects, particularly in older patients.
87. A 19-year-old male presents with fevers, chills, malaise, and rash for 5 days. He recently
returned from a camping trip. The rash is macular and located on the wrists, ankles, palms, and
soles. Routine laboratory work including lumbar puncture is normal. Which of the following is
the most appropriate next step?
A. Watchful waiting
B. Doxycycline
C. Acyclovir
D. Clindamycin
E. TMP-SMX
Answer B. With the history of a recent camping trip, tick-borne illness should be suspected. In
this case, the patient likely has Rocky Mountain spotted fever, caused by the intracellular
bacterium, Rickettsia rickettsii. The organism causes endothelial cell damage, resulting in diffuse
vasculitis and multiple organ microinfarctions and failure. Acute respiratory distress syndrome,
DIC, and shock all may occur in serious cases. Rash on the wrists and ankles following a
nonspecific viral-like syndrome is characteristic. Laboratory work is usually normal, though
thrombocytopenia may occur. Treatment is with doxycycline or chloramphenicol. Choices A, C,
D, and E will not adequately treat the rickettsial infection.
88. A 27-year-old female with HIV presents for evaluation of a 2-day history of fevers, chills,
rigors, cough, and purulent sputum. Which of the following is the most likely cause of infection?
B. Streptococcus pneumoniae
C. Crytptococcus
D. Histoplasmosis
E. Tuberculosis
pg. 238
Answer B. Though pneumocystis pneumonia (PCP) is an important consideration in patients with
HIV, typical organisms associated with community-acquired pneumonia predominate. This is
increasingly true as the majority of patients are taking extremely effective highly active
antiretroviral therapy (HAART). Unless the CD4 count is <200, opportunistic infections are
uncommon. Furthermore, the abrupt onset of this patient’s symptoms along with purulent
sputum is typical of patients with community-acquired bacterial pneumonia. In contrast,
patients with PCP have a nonproductive cough, dyspnea, and a more gradual onset. Tuberculosis
should be considered in all HIV patients presenting with pulmonary symptoms. However, the
symptoms are typically gradual and subacute to chronic. Patients with early HIV behave like
patients who are HIV negative, with symptoms of fever, cough, night sweats, and weight loss.
Chest x-ray may reveal cavitation and patients may present with blood-streaked sputum.
Cavitation and hemoptysis are often absent in patients with more advanced HIV, who may
present with dissemination (e.g., fever without a source) or extrapulmonary TB (cervical
lymphadenopathy).
A. Fetal macrocephaly
E. Mortality of 50%
Answer B. Zika virus is a flavivirus, the same genus as dengue, West Nile, and yellow fever. It is
associated with fetal microcephaly, rather than macrocephaly, in babies of infected mothers, as
well as Guillain–Barre syndrome in the patients themselves. Characteristic symptoms include
fever, myalgias, and rash, although many patients are completely asymptomatic. The 2015
outbreak of Zika started in Brazil and spread throughout the Americas. Mortality is extremely
low and as of mid-2016, no vaccine or pharmacotherapy has been established.
90. A 22-year-old male presents with a painful, red area on his right leg for the last 3 days. It is
spreading up his leg and he noted fevers, chills, and lightheadedness today. He also describes a
pg. 239
sunburn-like rash on his chest, back, and arms but denies recent sun exposure. Vital signs are
102.5°F, 112, 22, 82/45, 94% RA. Which of the following is true regarding this patient?
D. Mortality is <1%.
Answer B. The patient likely has toxic shock syndrome (TSS), given the preceding skin infection,
sunburn-like diffuse rash, hypotension, and fever. TSS is a toxin-mediated disease, either due to
staphylococcal TSS toxin-1 or S. pyogenes exotoxins A and B. Staphylococcal TSS used to occur
secondary to superabsorbent tampons in menstruating women, but now, like streptococcal TSS,
it is largely because of systemic and postsurgical infections. Multiorgan failure is characteristic of
both processes. Blood cultures are usually negative in staphylococcal TSS and positive in
approximately half of the cases of streptococcal TSS. Mortality for staphylococcal and
streptococcal TSS is 5% and 30%, respectively. Management involves removal of any foreign
bodies, antibiotics directed at streptococci and staphylococci, vasopressors, intravenous fluids,
and intensive care monitoring.
D. Oral blisters that spontaneously rupture to form painful erosions are the earliest
manifestation.
Answer D. Pemphigus vulgaris is the most common form of pemphigus, although it is a rare
disease. It most commonly affects older individuals in the sixth decade. Nikolsky sign, which
occurs when pressure applied to the margin of a blistered or ulcerated lesion expands the lesion
into the adjacent apparently normal skin, is the clinical hallmark of the disease. Although
antibiotics are important in cases of secondary infection, corticosteroids are the mainstay of
therapy. Most patients have oral lesions that start as blisters, which then spontaneously rupture
to leave painful oral ulcers that are slow to heal. Oral ulcers may precede cutaneous disease by
pg. 240
several months. The cause of the disease is unknown, although there is a genetic predisposition
and an autoimmune mechanism involving circulating IgG antibodies to keratinocytes.
92. The most common cause of sexually transmitted disease (STD) in the United States is:
A. Trichomonas vaginalis
B. Chlamydia trachomatis
C. Treponema pallidum
D. Neisseria gonorrhoeae
E. Candida spp.
Answer B. C. trachomatis is the most common bacterial cause of STD in the United States. Most
women infected with C. trachomatis are asymptomatic (as many as 85%), although up to
one-third of women will have signs of infection upon physical examination. C. trachomatis takes
up residence in the endocervix and causes an intense inflammatory reaction resulting in
mucopurulent cervicitis. Treatment consists of a single 1-g dose of azithromycin or 7 days of 100
mg of doxycycline given twice daily. In the setting of more severe infection (pelvic inflammatory
disease), doxycycline should be given for 14 days and combined with empiric treatment for N.
gonorrhoeae and probably Trichomonas as well.
93. A 24-year-old female presents to the ED with the rash shown in Figure below. She notes that
she had a recent “cold sore” and brief febrile illness that resolved on its own. She now
complains of general malaise and a rash. The rash is symmetric and is spread over her
extremities as well as her palms and soles. The remainder of her examination is unremarkable.
Which of the following is true?
pg. 241
Answer D. This patient has erythema multiforme (EM) minor (no mucosal involvement), which is
a relatively benign disease that results from a host immune response to antigenic exposure. EM
minor is at the benign end of a clinical spectrum that includes EM major, Stevens Johnson
syndrome (SJS), and toxic epidermal necrolysis (TEN). The classic rash is typically symmetric and
involves the extremities, palms, and soles. The archetypal lesion is a “target” lesion measuring
<3 cm and consisting of two concentric rings around a dusky, central disk. Mucosal involvement
never occurs in EM minor, whereas it is very common in EM major and is almost exclusively
limited to the oral cavity. Nikolsky sign describes the tendency for the top layer of skin to rub off
when it is touched. It is classically associated with pemphigus vulgaris. Corticosteroids are
sometimes used for treatment but their benefit has not been proved. Immunoglobulin plays no
role in treatment. The lesions resolve without intervention within 1 to 3 weeks. Although almost
50% of cases of EM minor are idiopathic, HSV is the most common infectious agent triggering
the disease. EM is typically caused by infection, whereas SJS and TEN are typically due to drug
exposure.
94. Which of the following is true regarding West Nile virus (WNV) infections?
C. The most common presentation is “West Nile fever,” in which patients present with fever,
headache, and malaise, associated with diffuse myalgias.
pg. 242
severe encephalitis. WNV is almost wholly transmitted by mosquito bites, though wild birds
serve as a reservoir for viral amplification. The majority of patients are symptomatic. Among
patients who have symptoms, West Nile fever is the most common presentation, consisting of
constitutional symptoms that are indistinguishable from other viral illness, including common
infections such as influenza. Due to the lack of specific symptoms in such patients, they will not
likely be diagnosed with WNV. Fortunately, the disease is most often self-limited, so making a
firm diagnosis is not imperative. Among patients with neuro-invasive disease, encephalitis,
meningitis, or flaccid paralysis are the most common presenting symptoms. Such patients can
be diagnosed by the detection of IgM and IgG antibodies due to the rapid seroconversion that
occurs among infected patients. While treatment adjuncts are being studied for patients with
severe disease, treatment remains supportive.
95. Which of the following is the most common opportunistic infectious agent in AIDS patients?
A. Tuberculosis
C. Cryptococcus neoformans
D. Toxoplasma gondii
E. Cytomegalovirus (CMV)
Answer B. More than three-fourths of all patients with AIDS will develop PCP at some point in
their lifetimes. It is also the most common identifiable cause of death in AIDS patients.
Pneumocystis is classified as a protozoan, but has many characteristics of a fungus. Symptoms of
PCP, like all pneumonias, include fever, cough, and shortness of breath, but a subacute or mild
course is characteristic. Chest radiography classically demonstrates diffuse, bilateral interstitial
infiltrates, but can be completely normal up to 20% of the time. First-line therapy is with
TMP-SMX. Corticosteroids are indicated in patients who have significant hypoxia. Pentamidine,
dapsone, or clindamycin plus primaquine may be used as alternatives. The other answer choices
are all common opportunistic infections in AIDS patients, but occur less often than
Pneumocystis.
96. In patients with measles, where are Koplik spots most likely to be seen?
A. Hard palate
B. Soft palate
C. Tonsils
pg. 243
D. Tongue
E. Buccal mucosa
Answer E. Koplik spots are irregularly shaped bright red macules that contain a central,
punctate, bluish-white spot. Their presence is pathognomonic for measles. They are most
commonly found on the buccal mucosa and are classically located opposite the second molars.
pg. 244
Infectious disease and
systemic infection
➢ Chapter includes:
Meningitis, meningococcemia and encephalitis
Systemic inflammatory response syndrome
Sepsis, septicemia and septic shock
Necrotizing fasciitis
Peritonitis
Dermatology and systemic infectious relation
pg. 245
1. A 47-year-old diabetic male is being treated for septic shock. After receiving 4 L of fluid, a
passive leg raise test is performed and results in no change in his blood pressure. Which of the
following is the next step?
B. Broad-spectrum antibiotics
C. Vasopressors
D. Jugular vein central line placement to assess central venous pressure (CVP)
E. Lactate measurement
Answer C. A passive leg raise is performed by first having the patient sit semi-upright with their
head elevated at about 45 degrees, with their legs flat on the bed. After a few minutes, the head
is placed flat, and the legs are elevated to 45 degrees. This results in a transient central fluid
bolus of as much as 300 mL, caused by moving fluid that had pooled in the legs to the
intrathoracic cavity. Patients who are fluid responsive should experience a rise in blood
pressure. In this case, an increase in blood pressure is a surrogate for improved cardiac output.
One study showed that patients needed at least a 17% increase in systolic blood pressure to be
considered “fluid responsive.” An alternative to using blood pressure is to use a noninvasive
means of assessing cardiac output such as a NICOM device or by following EtCO2 in
mechanically ventilated patients. Since this patient had no improvement after a passive leg
raise, he is not fluid responsive, so the next step is start vasopressors.
2. A 72-year-old female presents with fever, stiff neck, and headache. You strongly suspect
bacterial meningitis. Which of the following is the most appropriate treatment strategy?
A. Dexamethasone
C. Ampicillin
D. A and B
E. A, B, and C
pg. 246
ceftriaxone are effective against the major pathogens seen in bacterial meningitis
(Streptococcus pneumoniae, Neisseria meningitidis, and Haemophilus influenza). Importantly, in
patients at extremes of age, Listeria monocytogenes becomes more prevalent and requires
specific therapy with ampicillin. In this patient, dexamethasone, vancomycin, ceftriaxone, and
ampicillin are all warranted.
3. A 45-year-old female with alcoholic cirrhosis presents with abdominal distention and pain
with low grade fever. You suspect spontaneous bacterial peritonitis (SBP) and perform a
paracentesis. The total cell count is 300 cells per mm3 with 60% neutrophils. Which of the
following is the most appropriate next step in management?
A. Discharge home
C. IV cefotaxime
D. IV vancomycin
E. IV metronidazole
Answer B. The patient does not have definitive evidence of SBP. Diagnosis of SBP is suspected
with an ascitic fluid neutrophil count >250 cells per mm3. In this case, the neutrophil count is
180 cells per mm3 (i.e., 60% × 300). Leukocyte esterase testing with urine dipstick on ascitic fluid
has a high negative predictive value for SBP and may be used as an earlier marker to exclude the
diagnosis. SBP is caused by combination of reduced hepatic phagocytotic activity with a
protein-poor (therefore, complement-poor) ascitic fluid environment. The incidence of SBP
occurs at about 25% of all cirrhotic patients per year. Gram-negative organisms predominate,
most commonly E. coli and Klebsiella, though streptococci are also well represented. It is crucial
to note that up to half of all patients with SBP lack fever and only a minority have significant
abdominal tenderness or true rebound tenderness. Treatment with cefotaxime would be the
appropriate regimen for SBP. Vancomycin and metronidazole do not adequately cover
gram-negative organisms and should not be used alone in the treatment of SBP.
4. An 85-year-old female presents from a nursing home with fever and hypotension. According
to the paramedic, she began to have lethargy and refused to get out of bed 1 day ago. There is
no reported history of cough, shortness of breath, diarrhea, headache, or rash. Her vital signs
are: 102.4°F, 122, 22, 72/44, 95% RA. The patient’s physical examination is nonfocal, and routine
laboratory work is sent. Which of the following is the most appropriate empiric antibiotic
therapy at this time?
A. Ampicillin
pg. 247
C. Vancomycin and piperacillin–tazobactam
5. What is the most common cause of death in patients with toxic epidermal necrolysis (TEN)?
A. Electrolyte abnormalities
B. Respiratory failure
C. Sepsis
D. Dehydration
E. Ventricular dysrhythmias
Answer C. Fluid loss in patients with TEN is sizable but not as severe as in burn patients.
However, the widely denuded skin is an easy access point for a variety of bacteria and sepsis is
the most common cause of death.
Answer C. The patient is presenting in septic shock from an unclear source. She presents from a
nursing home, where rates of methicillin-resistant S. aureus (MRSA) and P. aeruginosa are
extremely high. Treatment involves broad-spectrum antibiotic coverage to cover both these
pathogens. Metronidazole and clindamycin afford excellent anaerobic coverage, but
piperacillin–tazobactam also kills anaerobes and double coverage of anaerobes is not routinely
indicated. Choice B would be a reasonable regimen in a patient with septic shock who is not at
risk for nosocomial pathogens.
6. Which of the following is the most common organism isolated in spontaneous bacterial
peritonitis (SBP)?
A. Escherichia coli
B. Staphylococcus aureus
C. Streptococcus pneumoniae
D. Klebsiella pneumoniae
E. Anaerobic species
pg. 248
Answer A. E. coli is isolated in 47% to 55% of the cases of SBP and gram-negative organisms are
the most common etiologic agents as a group. K. pneumoniae is the second most commonly
isolated organism. This is followed by S. pneumoniae, and other Streptococcus and
Staphylococcus species. Although there have been isolated reports of anaerobic and
polymicrobial infections in SBP, they are generally not considered to be causes of SBP. Fever or
abdominal pain in a patient with ascites should raise the suspicion of infection and prompt a
paracentesis. The presentation of SBP may be subtle, however, and include only mental status
changes without abdominal pain or tenderness upon examination. All patients with an ascitic
fluid neutrophil count ≥250 per mm3 and a clinical picture consistent with infection should be
treated with antibiotic therapy.
Glucose: Normal
Protein: Normal
A. Ceftriaxone
B. Vancomycin
C. Acyclovir
D. Itraconazole
E. Amphotericin B + flucytosine
Answer E. With a positive India ink stain of the CSF, the patient has fungal meningitis, most likely
due to Cryptococcus neoformans, an opportunistic infection common in patients with AIDS.
Patients usually present with symptoms typical of aseptic meningitis. The CSF WBC and protein
are usually only slightly elevated. India ink staining has approximately 80% sensitivity, so CSF
should also be sent for cryptococcal antigen, which has close to 100% sensitivity and specificity.
Treatment for cryptococcal meningitis is with amphotericin B plus flucytosine, which is superior
to itraconazole monotherapy. Fluconazole monotherapy may be used in very mild cases.
Ceftriaxone and vancomycin are used for therapy of bacterial meningitis. Acyclovir is used for
pg. 249
herpes simplex encephalitis (HSE), which is suggested by altered mental status and elevated CSF
RBC count.
8. Due to its significant morbidity and mortality, early recognition of necrotizing fasciitis is
critical. Which of the following is most helpful in making an early diagnosis?
A. Crepitus on examination
Answer D. There is no single, wholly reliable finding in the setting of early necrotizing fasciitis.
However, extreme pain with minimal cutaneous findings is often the first manifestation of a
serious underlying problem and may be the only indication of an imminent infectious tsunami.
Patients often have relatively unexplained pain (i.e., no history of trauma or prior surgical
intervention) that is rapidly progressive in the relative absence of associated skin findings (since
it is a fascial infection first, which later extends to the skin and underlying muscle). Once the
initial erythema yields to ecchymoses with vesicles and bullae, extensive damage has already
taken place in the subcutaneous tissues. Among cases of type II necrotizing fasciitis (primarily
caused by Group A streptococci in previously healthy patients), crepitus is only present in 10% of
patients. There is a laboratory-based scoring system to help risk-stratify patients in whom the
diagnosis is being considered, called LRINEC (Laboratory Risk Indicator for Necrotizing Fasciitis).
While an elevated CRP ≥150 mg/L accounts for the largest number of points within this scoring
system, it is only useful in patients in whom the diagnosis is already seriously being considered.
In such cases, the clinician’s suspicion of the disease has likely already triggered surgical
consultation.
9. A 32-year-old male presents to the ED with headache and fever for 2 days. He also reports a
stiff neck and photophobia. Past medical history is unremarkable. Physical examination reveals a
febrile patient with nuchal rigidity, no papilledema, and no focal neurologic deficits. Which of
the following is the most appropriate next step in management?
A. Antibiotic therapy
pg. 250
E. Lumbar puncture
Answer B. The patient has clear clinical evidence of meningitis. Given the time course, it is
unclear whether the etiology is viral or bacterial, so a cautious approach should be taken.
Ideally, the patient should have a lumbar puncture as soon as possible, but antibiotics should
not be delayed in such cases as they do not appear to significantly affect culture results in the
first 4 hours of therapy. Corticosteroids are now part of the standard of care for treatment of
suspected bacterial meningitis as they improve functional outcomes. The indications for CT scan
before lumbar puncture are altered mental status, focal neurologic deficit, suspected brain mass
lesion, and signs of increased intracranial pressure. MRI of the brain has little role in the
emergent evaluation of meningitis.
10. A previously healthy 35-year-old male is brought to the ED with a chief complaint of fever
and altered mental status. The patient had been well until 1 day prior to admission, when he
developed a fever and general malaise. He was too ill to work on the day of evaluation, and
when his wife returned from work, she found him hot, sweaty, and “out of it.” The patient
presents with a temperature of 102.1 F, P 103, RR 18, BP 125/75, SaO2 97% RA. He is awake, but
somnolent and confused. A lumbar puncture is performed which reveals gram-positive cocci in
pairs. His wife asks about receiving prophylactic therapy. Which of the following is most
appropriate?
E. No prophylaxis is indicated.
Answer E. The gram stain reveals gram-positive cocci most consistent with Streptococcus
pneumonia. Chemoprophylaxis is only necessary for meningitis caused by N. meningitidis or
Haemophilus influenzae (principally caused by type B, which is now quite rare in the United
States due to the Hib vaccine), which are both gram-negative organisms (Neisseria are diplococci
and Haemophilus are bacilli). The classic regimen in exposed adults is Ciprofloxacin 500 mg
orally × 1 dose. Children should be treated with either Rocephin IM × 1 (125 mg IM × 1 in
children <15 years of age) or oral Rifampin (agebased dosing twice daily for 2 days). There is
some evidence of Ciprofloxacin resistance in certain parts of Minnesota and North Dakota,
where Ciprofloxacin becomes a second-line agent. Therefore, Rocephin and Rifampin are also
reasonable alternatives in adult patients who have had close contact with an affected individual.
Azithromycin is another alternative if local isolates demonstrate resistance to Ciprofloxacin.
pg. 251
11. A 24-year-old male presents with headache, stiff neck, fever, and rash shown (Fig. below).
Which of the following is the next best step in management?
C. Lumbar puncture
D. Ceftriaxone
E. Vancomycin
Answer D. The purpura and clinical history are both consistent with meningococcemia and
meningitis due to N. meningiditis. This causes a fulminant, overwhelming septic picture which
can lead quickly to death if not treated immediately. As a gram-negative organism,
meningococcus should be treated with ceftriaxone. Although formal diagnosis of bacterial
meningitis involves CT brain in most cases before a lumbar puncture to obtain cerebrospinal
fluid (CSF), in cases where critical illness is evident, antibiotics and steroids should be given
before confirmatory diagnostic maneuvers. Vancomycin is certainly an important antibiotic to
give in most cases of meningitis (as it kills gram-positive organisms such as pneumococcus), but
it should not precede ceftriaxone in cases where meningococcus is likely.
pg. 252
A. Herpes virus
B. Mumps virus
C. Enteroviruses
D. Neurocysticercosis
E. Adenovirus
Answer C. Enteroviruses, such as coxsackievirus, account for more than half of all the cases of
encephalitis. Herpes virus is the most common cause of severe encephalitis in the United States.
Typically, in frank infections of the brain parenchyma (encephalitis), there is some degree of
concomitant infection and inflammation of the meninges. Therefore, meningoencephalitis is a
more accurate term describing most of these infections. When the infection is limited to the
meninges, it is termed meningitis, reflecting meningitis in the absence of a pyogenic organism.
Mumps is another important cause of aseptic meningitis, especially in the setting of outbreaks
of the virus such as in Iowa in early 2006. Meningoencephalitis is the most common
complication of childhood mumps and frequently occurs at the same time as parotitis.
Neurocysticercosis is a rare cause typically found in immigrants from areas where undercooked
pork harboring the parasite Taenia solium may be eaten. Adenovirus is an uncommon cause.
13. A 54-year-old diabetic woman presents with perineal pain, fever, and lethargy. Her perineal
examination is shown in Figure below. Which of the following is the most appropriate next step
in management?
A. Corticosteroids
B. Oral antibiotics
C. Intravenous antibiotics
E. MRI of pelvis
pg. 253
Answer D. The patient has evidence of necrotizing fasciitis of the perineal area, commonly
referred to as Fournier gangrene. Fournier gangrene is a systemic, life-threatening,
polymicrobial infection, which requires intravenous antibiotics and surgical debridement.
Intravenous immunoglobulin and hyperbaric oxygen therapy may be helpful in certain cases.
Corticosteroids are not indicated except in cases of concomitant adrenal insufficiency.
Antibiotics without surgical debridement are not sufficient for management. The diagnosis is
usually made clinically—in unclear cases, CT or MRI may aid the diagnosis but neither is 100%
sensitive or specific.
14. A rash that starts on the wrists and ankles is typical in cases of:
A. Meningococcemia
C. Rubeola (measles)
E. Pityriasis rosea
Answer A. Roughly 70% of patients with meningococcemia have a rash which usually begins as
petechiae of the wrists and ankles. However, the rash may also be composed of light pink
macules that become petechial later in the disease course. Petechial lesions may have a
“smudged” appearance although purpura may have a “gun-metal gray” center. Rubella and
rubeola are maculopapular eruptions that tend to begin on the face and spread inferiorly. HSP is
a vasculitis that is characterized by palpable purpura on the buttocks and lower extremities.
Pityriasis rosea is a benign, self-limited rash characterized by patches and plaques that typically
occur on the trunk.
pg. 254
15. Among close contacts of a patient diagnosed with meningitis caused by Neisseria
meningitidis, what is the approximate likelihood of transmission without prophylactic antibiotic
therapy?
A. <1%
B. 5%
C. 25%
D. 50%
E. 90%
Answer A. Even among close contacts of patients with meningitis caused by N. meningitidis, the
approximate rate of transmission is only 1 in 250 (0.4%). While this rate is very small, it is 500 to
800 times larger than the likelihood of infection in the general population
16. A 28-year-old male presents with an acute, progressively worsening headache and fever for
1 day. Physical examination reveals a toxic-appearing, slightly confused patient with fever and
meningismus. Which of the following is the most appropriate next step in management?
C. Lumbar puncture
Answer E. The patient has clear evidence of meningitis. The acute onset, progressive nature, and
severe symptoms all indicate a likely bacterial cause. With evidence of alteration in mental
status, CT scan before lumbar puncture may be advisable. However, in cases where bacterial
meningitis is clinically suspected, antibiotics should be administered before the patient is sent to
CT scan or lumbar puncture is performed. Additionally, adult patients with suspected bacterial
meningitis should receive dexamethasone with or before the first dose of antibiotics.
17. A 54-year-old male with a history of diabetes presents with 3 days of a slight red but
extremely painful left lower extremity. He appears fatigued and ill. His vital signs are: 100.0, 122,
20, 100/70, 99% RA. The affected leg is swollen, warm, and only mildly erythematous. He has
exquisite tenderness in the area. Strongly suspect necrotizing fasciitis. Which the following tests
would be most helpful in evaluating that diagnosis?
pg. 255
A. Platelets
B. Potassium level
C. C-reactive protein
D. Procalcitonin
Answer C. C-reactive protein is part of the Laboratory Risk Indicator for Necrotizing Fasciitis
(LRINEC) score for screening of necrotizing fasciitis. An LRINEC score below a specific cutoff
indicates that necrotizing fasciitis is extremely unlikely in patients with signs and symptoms of
severe cellulitis. The LRINEC score contains the following variables: C-reactive protein, WBC,
sodium level, creatinine level, hemoglobin, and glucose. Platelets are part of the qSOFA score
for general sepsis. Procalcitonin is used to differentiate between bacterial and nonbacterial
etiology in lower respiratory infections. Serum mycoplasma antigen is used to evaluate for
Mycoplasma pneumoniae in patients withpneumonia, but mycoplasma is not a typical pathogen
seen in necrotizing fasciitis.
18. A 68-year-old male presents with headache and “acting like a jerk,” according to the nursing
staff. He has no reported past medical history and his family who are with him state that his
personality is completely different from his usual pleasant one. Physical examination
demonstrates an irritable man complaining loudly of a headache with altered attention, not
oriented to place or time, and lacking in meningeal signs. His vital signs are: 100.2, 82, 20,
143/92, and 99% on RA. His urinalysis and chest x-ray are normal. Which of the following
represents the next best step in management?
Answer D. The patient is exhibiting evidence of acute herpes simplex encephalitis. Fever,
headache, and personality changes are the most common symptoms. Empiric antiviral
treatment for herpes simplex encephalitis is indicated, as early therapy improves mortality and
has few side effects and definitive diagnostic testing can take significant time. Discharging a
patient with fever and delirium home is decidedly suboptimal therapy. Dexamethasone therapy
may play a role in herpes simplex encephalitis, but it is not yet proven to be of definite benefit in
these patients and is not as important as antiviral therapy at this time. Chemical restraint with
pg. 256
haloperidol should not be instituted given the side effect profile in the geriatric population as
well as the simple fact that the patient has not demonstrated a harm to himself or others. MRI
brain with contrast would be a very reasonable study in the evaluation of herpes simplex
encephalitis, but the duration of the test in most EDs would likely worsen outcomes if empiric
treatment is not given first.
19. A 62-year-old male with a history of controlled hypertension presents to the ED with fever,
headache, and vomiting. He is mildly somnolent on examination and has evidence of mild neck
stiffness. Suspecting meningitis as the cause of his symptoms, which of the following empiric
regimens should you start?
A. Ceftriaxone, vancomycin
B. Ceftriaxone, ampicillin
Answer D. Patients older than 50 years are at risk for S. pneumococcus and N. meningitidis, as
well as L. monocytogenes. Ceftriaxone covers pneumococcus, whereas vancomycin is necessary
for resistant pneumococcus. However, in older patients at risk for Listeria infection, ampicillin is
also necessary. Patients younger than 50 who are immunocompromised or who are alcoholic
are also at risk for Listeria infection and should be covered with ampicillin. All patients with
suspected meningitis should be given 10 mg of dexamethasone IV just before or with the first
dose of antibiotics. Adjunctive dexamethasone reduces mortality and neurologic sequelae. The
reduction is most marked in patients with intermediate disease severity, defined as those
patients with a Glasgow Coma Scale rating of 8 to 11, as well as in patients with pneumococcal
meningitis. Amphotericin is used for confirmed fungal infections.
20. Which of the following elements is present in the new quick Sepsis-related Organ Function
Assessment (qSOFA) score that may replace systemic inflammatory response syndrome (SIRS)
criteria?
C. Temperature >99.0°F
pg. 257
Answer A. The qSOFA score is a modified version of the full SOFA score, which was advocated by
the Society of Critical Care Medicine in 2016. The SIRS criteria were felt to be not sufficiently
sensitive or specific for the definition of sepsis. The qSOFA score includes: systolic blood
pressure <100 mm Hg, Glasgow Coma Scale (GCS) <14, and respiratory rate >21 per minute. The
full SOFA score is a more complex scoring system and includes: PaO2/FiO2 ratio, platelets,
bilirubin, blood pressure, GCS, and creatinine. It is unclear at the present time whether the
qSOFA will replace SIRS, but it is unlikely that the full SOFA will be in widespread use in the
prehospital or ED setting due to its complexity. The wrong answer choices can be guessed by
their overly insensitive cutoffs (e.g., a glucose >100 is very common in all patients, not just
septic ones). Heart rate >90 is an element of the SIRS criteria rather than the qSOFA.
21. A 28-year-old previously healthy male presents to the ED with severe medial left upper leg
pain × 1 day. The pain was mild at first but has “become unbearable” and is making it difficult
for him to walk. He denies any recent trauma, or excessive exercise involving the leg. He initially
noted minimal redness over the site which has started to become a “little purplish in spots.” He
also complains of fever, fatigue, myalgias, and nausea. Examination of his leg reveals a large
area of moderate erythema with scattered ecchymotic areas and a few clear bullae. There is no
crepitus. X-ray of the leg is unrevealing. His vital signs are T 102.4°F, P 122, BP 98/64, RR 20,
SaO2 96% on RA. Which of the following is the most appropriate next step?
Answer A. This patient is presenting with findings most consistent with type II necrotizing
fasciitis caused by group A Streptococcus (GAS). Type I necrotizing fasciitis is a polymicrobial
infection caused by a mixture of aerobes and anaerobes that most commonly occurs in diabetic
and postsurgical patients. In contrast, type II necrotizing fasciitis is a monomicrobial infection,
primarily caused by GAS that occurs in previously healthy patients. Regardless of the cause,
necrotizing fasciitis is a rapidly progressive, severe infection associated with a significant
mortality rate, even with optimal treatment. While there is no single, wholly reliable finding in
the setting of early necrotizing fasciitis, extreme pain with minimal cutaneous findings is often
the first manifestation. Once the initial mild erythema develops into ecchymoses with vesicles
and bullae, extensive damage has already taken place in the subcutaneous tissues. The primary
treatment is immediate surgical debridement. While antibiotics are also essential, the mortality
rate is close to 100% without surgery. Though a monomicrobial infection is likely, broad
antibiotic coverage including anaerobes is recommended until an agent is identified.
pg. 258
Clindamycin is the recommended therapy for GAS (as opposed to penicillin) and anaerobic
coverage, while ampicillin–sulbactam broadens the coverage to include more anaerobes and
gram-negative coverage. Recently hospitalized patients may need broader gramnegative
coverage with other agents. Neither corticosteroids nor IVIG has any role in treatment, though
some small studies of IVIG show early promise. As with other imaging studies, MRI is most
useful when gas is present in the tissue, though this is an uncommon finding in GAS fasciitis. MRI
also tends to overestimate the degree of deep tissue involvement. Ultimately the diagnosis and
decision to operate is primarily made based on clinical grounds.
22. An 82-year-old female with a history of hypertension presents with generalized weakness
and alteration of mental status for 2 days. Her vital signs are: 101.4, 110, 22, 85/45, 94% RA.
Urinalysis shows >100 WBC and her chest x-ray is unremarkable. Appropriate antibiotics are
already ordered. Which of the following is the most appropriate next step in her management?
B. Norepinephrine 5 mcg/kg/minute
D. Dobutamine 5 mcg/kg/minute
E. Dopamine 5 mcg/kg/minute
Answer A. The patient has clear evidence of septic shock. She meets 2/4 systemic inflammatory
response syndrome (SIRS) criteria, has definite evidence of an infection, and exhibits end-organ
dysfunction with her acute encephalopathy and hypotension. Intravenous antibiotics and
adequate fluid resuscitation are crucial. Without a history of severe congestive heart failure, this
patient should receive a 30 mL/kg bolus of 0.9 normal saline as the preferred initial
resuscitation. The remainder of the choices are all vasoactive agents which will increase blood
pressure, but are only indicated after fluid resuscitation has been optimized.
23. A 69-year-old male with diabetes, hypertension, hyperlipidemia, and COPD presents with
signs and symptoms of sepsis. A chest x-ray reveals pneumonia. A lactate level is measured. At
what lactate level does patient mortality rise significantly?
pg. 259
Answer D. Though definitions of severe sepsis have been changing, it’s clear that an initially
elevated lactate level and a persistently elevated lactate level (inadequate lactate clearance) are
predictors of increased mortality. An initial lactate level >4 mmol/L predicts a 27% mortality
rate, versus only 7% mortality in septic patients with lactate levels of 2.5 to 4.0 mmol/L and <5%
mortality for patients with a lactate level <2.5 mmol/L.
24. Which of the following findings is seen in most patients with meningococcemia?
B. Skin lesions
C. Hypothermia
D. Seizure
E. Arthritis
25. Which of the following is a criterion for the systemic inflammatory response syndrome
(SIRS)?
A. Systolic BP <90
B. Diastolic BP <60
C. HR <90
D. Temperature <36°C
E. RR <20
pg. 260
Answer D. The SIRS was devised to be able to formally describe basic vital sign and laboratory
abnormalities associated with severe inflammation, such as infection, trauma, burns, and severe
medical illness. SIRS is said to be present when two out of the following four criteria are present:
• HR >90.
• Sepsis is the presence of SIRS with a suspected or proven infectious cause. Septic shock is
defined as sepsis plus inadequate tissue perfusion, which may be manifested as hypotension or
end-organ dysfunction. Note that hypotension is not a criterion for SIRS or sepsis, but is present
in most cases of septic shock
26. A 51-year-old male with a history of end-stage renal disease who recently started continuous
ambulatory peritoneal dialysis (CAPD) presents to the ED with a chief complaint of abdominal
pain and fever for 1 day. He has noted a cloudy effluent which is a change from the baseline
clear character of the dialysate. A presumptive diagnosis of peritonitis is made. Which of the
following empiric strategies is best?
Answer A. Patients with peritoneal dialysis (PD) associated peritonitis rarely develop bacteremia
which forms the basis for intraperitoneal (as opposed to intravenous) antibiotic therapy, which
can frequently be performed as an outpatient presuming the patient is otherwise stable and has
adequate pain control. The infection is almost always limited to the peritoneal cavity and a few
cellular layers of the peritoneum. Gram-positive infections are most common, and due to the
increasing prevalence of methicillin resistance among Staphylococcus isolates, vancomycin is
typically recommended. However, 15% of infections are due to gram-negative organisms, so a
third- or fourth-generation cephalosporin, an aminoglycoside or a carbapenem, is also needed.
While the third- and fourth generation cephalosporins also have significant activity against
gram-positive organisms, they are inadequate for methicillin-resistant organisms, so they are
not recommended for monotherapy. Catheter removal or catheter exchange is not typically
needed unless the peritonitis proves to be refractory. Due to the ease of drug delivery and,
more importantly, the higher intraperitoneal drug concentrations achieved with intraperitoneal
pg. 261
dosing, it is preferred over intravenous dosing. However, intraperitoneal drug delivery results in
significant systemic absorption such that blood levels of vancomycin and gentamicin require
monitoring when their use is prolonged. Fortunately, fungal peritonitis is rare, occurring in less
than 2% of patients. When present, it is difficult to treat and often requires catheter removal.
27. A 68-year-old female presents with a chief complaint of severe weakness, general malaise,
and fever. The patient is hypotensive, and is found to have pneumonia on chest x-ray. The
patient received broad-spectrum antibiotics but remains hypotensive despite 5 L of fluids.
Which of the following is the best next step?
A. Norepinephrine
B. Vasopressin
C. Dopamine
D. Dobutamine
E. Phenylephrine
28. A 59-year-old male with a history of alcoholic cirrhosis presents with worsening abdominal
pain and swelling. He normally receives therapeutic paracenteses every few weeks but has not
had one in a month. He has no fever, vomiting, or diarrhea. Vital signs are 98.9°F, 85, 18,
105/65, 99% RA. He has mild cognitive deficits per his baseline with hepatic encephalopathy
(HE). He is in no respiratory distress. Abdominal examination demonstrates moderate distention
with fluid wave but only mild tenderness and no rebound or guarding. Which of the following is
the most appropriate next step in management?
A. Diagnostic paracentesis
B. Therapeutic paracentesis
C. CT of abdomen/pelvis
pg. 262
Answer A. Given his history of cirrhosis and resulting ascites, the patient is at risk for SBP. SBP is
caused by combination of reduced hepatic phagocytotic activity with a protein-poor (therefore,
complement-poor) ascitic fluid environment. The incidence of SBP occurs at about 25% of all
cirrhotic patients per year. Gram-negative organisms predominate, most commonly E. coli and
Klebsiella, though streptococci are also well represented. It is crucial to note that up to half of all
patients with SBP lack fever and only a minority have significant abdominal tenderness or true
rebound tenderness. Diagnostic paracentesis can evaluate for SBP with an ascitic fluid
neutrophil count <250 cells per mm3. Leukocyte esterase testing with urine dipstick on ascitic
fluid has a high negative predictive value for SBP and may be used as an earlier marker to
exclude the diagnosis. Treatment involves a third-generation cephalosporin, a fluoroquinolone,
or ampicillin–sulbactam. Therapeutic (i.e., large volume) paracentesis in the emergent setting
should only be performed in cases of significant respiratory distress where the source is
suspected to be inadequate tidal volumes due to tense ascites. CT of the abdomen and pelvis
has no role in the evaluation of SBP and should instead be pursued in cases of focal abdominal
tenderness or suspicion of abscess or other intra-abdominal pathology. Ultrasound of the right
upper quadrant may evaluate for further liver disease of biliary pathology, but this is not
suspected in the absence of focal abdominal tenderness or jaundice. Patients suspected of
having SBP should not be discharged home until SBP and other abdominal emergencies are
ruled out.
30. A 37-year-old previously healthy woman is brought to the ED by her family with a complaint
of fever, headache, and delirium with a depressed level consciousness. Empiric antibiotics for
pg. 263
community-acquired meningitis were started and a CT scan was performed, which was negative.
Her lumbar puncture revealed the following: WBC 412 per mm3 with a differential of 98%
lymphocytes, elevated protein, and normal glucose. Which of the following should be added to
her regimen?
A. Acyclovir
B. Ampicillin
C. Dexamethasone
D. Amphotericin B
Answer A. This patient’s pleocytosis with a lymphocytic predominance in concert with a normal
glucose and elevated protein level supports the diagnosis of a viral etiology. Herpes simplex
encephalitis (HSE) is the most common cause of severe encephalitis in the United States despite
the fact that arboviruses such as West Nile virus continue to get significant media coverage. The
most common symptoms of HSE are fever, altered mental status, decreased level of
consciousness, and focal neurologic findings. Focal seizures may also occur (temporal lobe
seizures). Because acyclovir is extremely well tolerated and is very effective for treating HSE, it
should be started as soon as the diagnosis of HSE is suspected. Dexamethasone is used as an
adjunctive treatment in bacterial meningitis and should be given just before or with the first
dose of antibiotics. Amphotericin B is an antifungal agent.
31. A 67-year-old female presents in septic shock due to pyelonephritis. Her vital signs are:
101.2, 110, 20, 94/59, 99% on RA. Apart from intravenous antibiotic therapy, which of the
following is most likely to help reduce her mortality?
A. Corticosteroids
D. Phenylephrine therapy
Answer E. Septic shock should be treated aggressively with intravenous antibiotics and fluids.
Corticosteroids have not been shown in most cases to reduce mortality in patients with
pyelonephritis (although they likely help in patients with community-acquired pneumonia and
acute respiratory distress syndrome (ARDS)). Merely checking a lactate level does not prevent
mortality; a sepsis protocol must contain an effector arm to have a clinician act on the basis of
that lactate level. Similarly, placing a central venous catheter by itself does not improve
pg. 264
mortality. Norepinephrine would not yet be indicated in a patient with moderate hypotension
before aggressive (i.e., 30 mL/kg) hydration is pursued.
32. Which of the following is the definition of the shock index (SI)?
C. HR/SBP
E. HR/pulse pressure
Answer C. The shock index (SI) is a simple tool that can be used to quickly identify patients in
shock. A shock index of 0.5 to 0.7 is considered normal while a shock index ≥0.9 is predictive of
increased mortality and morbidity. The SI also has value in trauma patients, as an SI ≥0.9
doubles the risk of a massive transfusion and dramatically increases the risk of mortality.
33. A 47-year-old female with cirrhosis presents to the ED with mild abdominal pain. Her
temperature is 99.1°F and she has a protuberant abdomen with significant ascites. You suspect
spontaneous bacterial peritonitis. Aside from antibacterial therapy with IV ceftriaxone, which of
the following is most likely to reduce her mortality?
A. Amphotericin
B. Albumin
C. Vancomycin
D. Oseltamivir
E. Midodrine
Answer B. Patients with spontaneous bacterial peritonitis (SBP) may benefit from early therapy
with IV albumin. This benefit may also extend to prevention of renal impairment. Antifungal
therapy may be helpful if patients exhibit signs of infection in the face of aggressive antibacterial
therapy. Vancomycin is often used for catheter-associated peritonitis, but is less useful to treat
the gramnegative infections usually seen in SBP. Oseltamivir could be used in this patient to
treat influenza, but the patient has no signs or symptoms suggestive of this. Midodrine, an
alpha-agonist, is often used to maintain blood pressure in hypotensive patients with end-stage
cirrhosis, but there was no history suggestive of this in the scenario.
pg. 265
34. A 75-year-old female presents to the ED for evaluation of meningitis. She has had 12 hours
of acute onset of headache, stiff neck, and fever. Physical examination demonstrates a febrile
patient in moderate distress with nuchal rigidity and severe photophobia. A lumbar puncture is
performed with
On the basis of these results, the presumptive diagnosis of bacterial meningitis is made. Which
of the following is the most appropriate therapy?
B. Ceftriaxone alone
Answer E. The treatment of bacterial meningitis varies by age of the patient. All patients should
be treated with a third-generation cephalosporin to cover meningococcus, pneumococcus, and
gram-negative bacilli. At the extremes of age, Listeria monocytogenes is a more common
pathogen and should be specifically treated with either ampicillin or
trimethoprim–sulfamethoxazole. Vancomycin may be used in addition to these therapies to
cover methicillin-resistant S. aureus in endemic areas. Dexamethasone should be given before
or simultaneously with the first dose of antibiotics.
35. A 45-year-old male presents with signs and symptoms of bacterial meningitis. Which of the
following is the single most likely cause?
A. Group B streptococcus
B. Listeria monocytogenes
C. Streptococcus pneumoniae
D. Neisseria meningitides
E. Hemophilus influenzae
pg. 266
Answer C. The most common cause of bacterial meningitis in middle-aged and older adults is
pneumococcus. Group B streptococcus is common among neonates, Listeria in extremes of age,
and meningococcus in young adults. H. influenzae used to be a very common cause of bacterial
meningitis among children, but the introduction of the HiB vaccine has drastically reduced the
frequency of this pathogen.
36. A 44-year-old female with a history of end-stage renal disease on peritoneal dialysis presents
with abdominal pain. You suspect peritonitis as a cause of her pain. Which of the following is the
most likely etiologic agent?
A. Staphylococcus species
B. Candida species
C. Escherichia coli
D. Klebsiella
E. Pseudomonas
Answer A. Unlike cases of spontaneous bacterial peritonitis (SBP), where gram-negative enteric
organisms predominate, patients with peritoneal dialysis catheters tend to develop peritonitis
from gram-positive organisms, most commonly Staphylococcus species. Clinical signs and
symptoms of infection can be extremely mild and asymptomatic infection is common. Diagnosis
is through analysis of the peritoneal fluid—100 or more WBC per mm3 with a predominance of
neutrophils or a positive Gram stain makes the diagnosis. Most cases can be treated on an
outpatient basis with intraperitoneally administered antibiotics. The antibiotics of choice are
vancomycin plus any antibiotic with good gram-negative coverage, such as a fluoroquinolone,
third-generation cephalosporin, aminoglycoside, or aztreonam.
pg. 267
Endocrine and electrolytes
Emergency
➢ Chapter includes:
Electrolytes emergency
Acid-base emergency
Thyroid, parathyroid and adrenal emergency
Rhabdomyolysis
multiple endocrine MCQ’s
pg. 268
1. Clinically significant hypermagnesemia almost always occurs in the setting of:
A. Renal insufficiency
B. Pancreatitis
C. Trauma
D. Laxative abuse
E. Alcoholism
Answer A. The kidneys are almost wholly responsible for magnesium excretion and are able to
enhance excretion in the setting of a magnesium load. Therefore, in the absence of renal
insufficiency, hypermagnesemia rarely occurs. Abuse of magnesium-containing laxatives may
cause a transient increase in magnesium levels but will not persist in the setting of normal renal
function. Trauma could feasibly lead to hypermagnesemia if associated with rhabdomyolysis.
2. A 72-year-old male arrives at the hospital with acute right-sided facial droop and right arm
and leg weakness. He is immediately sent to the CT scanner where he has a generalized seizure
lasting just over a minute. While the seizure terminates without intervention, the patient is
brought back to the emergency department immediately because of severe bradycardia and
ventricular escape beats. In addition to atropine, what other measures will most likely be
helpful?
A. Dopamine
B. Continuous albuterol
C. Epinephrine
D. Bicarbonate
E. Calcium
Answer D. Generalized seizures can result in a significant lactic acidosis due to anaerobic
metabolism in the affected musculature as the muscles undergo repeated contractions. In
generalized seizures, the amount of lactate produced can be significant resulting in a profound
systemic acidosis. While the acidosis will eventually resolve without intervention provided the
patient has normal renal function and stops seizing, intravenous bicarbonate should be given to
normalize the acid–base status as soon as possible to address the patient’s bradycardia. While
dopamine and epinephrine have chronotropic effects due to their β1-agonist activity, neither
medication will help resolve the patient’s acidosis. Albuterol is an effective, though transient,
treatment for severe hyperkalemia as it induces a potassium shift into cells.
pg. 269
3. A 55-year-old female presents with 1 to 2 days of palpitations, anxiety, agitation, tachycardia,
and hypertension. Her thyroid-stimulating hormone level is very low and her free T4 and free T3
levels are elevated. Which of the following is the most appropriate sequence in which to
administer ideal therapies?
Answer D. Thyroid storm treatment requires sequential inhibition of the following: hormone
synthesis, hormone release, and peripheral conversion from T4 to T3. Thionamides, including
propylthiouracil and methimazole, prevent the synthesis of new thyroid hormone. Potassium
iodide serves as a source of inorganic iodide, which prevents release of preformed hormone
from the thyroid gland. In the acute setting potassium iodide can actually promote new
hormone synthesis and should, therefore, always be given after thionamide therapy in thyroid
storm. Propranolol is used next to treat cardiovascular effects of thyrotoxicosis as well as to
block peripheral conversion of T4 to the more metabolically active T3. Glucocorticoids are used
to treat any underlying adrenal insufficiency and may have some effect in preventing peripheral
conversion.
A. Alcoholism
B. Diuretic therapy
D. Chronic diarrhea
Answer A. Between 30% and 80% of alcoholics have magnesium deficiency. Patients with
hypomagnesemia are frequently asymptomatic or manifest only nonspecific symptoms. The
most prominent symptoms in the ED are neuromuscular and cardiovascular, and magnesium
deficiency tends to mimic calcium deficiency. The mechanism of hypomagnesemia in alcoholism
is thought to be a combination of malnutrition, increased renal excretion, and GI losses from
vomiting and diarrhea. Diuretic therapy is also a very prevalent cause of hypomagnesemia,
although the subsequent volume loss increases magnesium reabsorption in the proximal tubule.
pg. 270
Therefore, magnesium depletion in the setting of diuretic therapy tends to be modest.
Hypomagnesemia is the most common electrolyte abnormality in ambulatory diabetic patients
and is also common in DKA. In these patients, magnesium is lost through the urine due to
glycosuria.
5. A 52-year-old male with a history of alcoholism presents with nausea and vomiting. He has
been drinking more excessively than usual over the last few days after his wife left him. He has
mild, epigastric abdominal pain. Vital signs are 98.5°F, 104, 22, 165/93, 100% RA. His physical
examination is normal except for mild regular tachycardia and dry mucous membranes. His labs
are shown below.
WBC: 6.5 ,Hb: 12.2 ,Plt: 125K ,Na: 132 ,K: 4.4 ,Cl: 90 ,HCO3: 12 ,BUN: 36 ,Cr: 1.4 ,Glu: 106
A. Insulin 10 units IV
C. Metoprolol 5 mg IV
D. D5 0.9 NS 1 L
E. Hydralazine 20 mg IV
Answer D. The patient has a significant anion-gap acidosis (132 − 90 − 12 = 30) with a history of
alcohol abuse and no history of diabetes. Alcoholic ketoacidosis (AKA) is the most likely cause,
resulting from the lack of nutrition other than alcohol, which causes the formation of ketone
bodies. A common mistake in treatment of AKA is to give only saline fluids (without glucose).
Treatment requires glucose (dextrose), either with food or IV fluids, to halt the formation of
ketone bodies from ethanol metabolism. It is important to note that urine dipstick tests can be
negative for ketones despite the severity of AKA. This is because dipstick tests only assess for
the presence of acetoacetate, not beta-hydroxybutyrate. AKA has an even higher ratio of
beta-hydroxybutyrate to acetoacetate compared to diabetic ketoacidosis (DKA). Insulin is
contraindicated in patients with AKA, as it can precipitously drop glucose levels in an already
glycogen-depleted state. Bicarbonate is not generally recommended in the management of
anion-gap acidosis; treatment instead should focus on the underlying cause. For the same
reason, metoprolol is not indicated in what is likely sinus tachycardia. Giving IV hydralazine to
treat a moderately elevated blood pressure without signs of end-organ dysfunction is not
recommended.
6. Which of the following therapies has the fastest onset of action in reducing serum potassium
levels in cases of hyperkalemia?
A. Intravenous calcium
pg. 271
B. Nebulized albuterol
A. Salicylates
B. Methanol
C. Isopropanol
D. Ethylene glycol
E. Isoniazid
Answer C. Isopropanol classically does not cause elevated anion gap when ingested. The
osmolar gap, however, is elevated and should be calculated and measured when there is
suspicion of toxic alcohol overdose. Elevation of the anion gap due to lactic acidosis can occur in
cases of severe isopropanol poisoning if there is associated coma, gastrointestinal hemorrhage,
or hypotension. Choices A, B, D, and E all cause an elevation in the anion gap at some point
during their metabolism.
pg. 272
8. A 57-year-old female with hypertension who has not seen a doctor in 30 years presents with
weakness and fatigue. The EKG is shown below . After appropriate further evaluation, which of
the following is most likely the appropriate next step in management?
A. Defibrillation at 200 J
B. Cardioversion at 50 J
C. Lidocaine 150 mg IV
Answer D. The EKG demonstrates diffuse tall T waves consistent with hyperkalemia. The history
of absence of medical care, untreated hypertension, weakness, and fatigue suggests the
possibility of renal failure as the underlying cause of hyperkalemia. Potassium levels should
obviously be confirmed on laboratory tests, but cardioprotective agents such as calcium chloride
or calcium gluconate should be ordered early. Defibrillation should never be performed in
awake patients. Cardioversion should only be performed with a clear history of hemodynamic
instability. Lidocaine is used for wide-complex tachycardias but has generally been supplanted
by amiodarone for acute use.
pg. 273
9. A 63-year-old female with a history of “kidney problems” presents with a chief complaint of
dizziness and lightheadedness. An EKG reveals profound bradycardia with a rate of 30 that fails
to improve after atropine or transcutaneous pacing. Which of the following may be useful?
A. Calcium
B. Epinephrine
C. Defibrillation
D. Dopamine
E. Norepinephrine
Answer A. In the setting of renal disease, hyperkalemia is the likely culprit of the patient’s
bradycardia. Calcium and bicarbonate should be used to help reverse the patient’s bradycardia.
Many patients with acute renal failure or an acute exacerbation of chronic kidney disease are
acidotic and it is this acidosis that prevents the heart from responding to typical treatments for
bradycardia such as atropine and transcutaneous pacing. While bicarbonate targets the root
issue, calcium opposes potassium’s effects on the myocardium, restoring the cardiac resting
membrane potential, allowing cardiac myocytes to conduct electricity normally. None of the
other treatments listed are helpful in patients with hyperkalemia.
10. A 33-year-old male with a history of type I diabetes presents with nausea, vomiting, and
altered mental status. His wife states that he did not take his insulin for the past few days. His
chemistry panel is below:
Na: 125 ,K: 3.8 ,Cl: 90 ,HCO3: 10, BUN: 30 ,Cr: 1.4 ,Glu: 727
In addition to insulin and IV fluids, which of the following is the most important step in
management?
A. Sodium bicarbonate
B. Potassium
C. Fomepizole
D. Glucagon
Answer B. The patient has an anion-gap acidosis (125 − 90 − 10 = 25) in the presence of elevated
blood sugar, which is DKA until proven otherwise. The most common cause of DKA is medication
noncompliance, followed by infection, myocardial ischemia, and other body stressors.
Treatment with insulin and IV fluids should be supplemented with aggressive potassium
pg. 274
management. The presence of significant acidosis causes the body to shunt hydrogen ion into
cells and exchange them for potassium ions, which are moved into the bloodstream. This results
in a false elevation of potassium levels. When insulin is given, this level can drop precipitously as
the acidosis is corrected, allowing potassium to move back out of the blood stream and into
cells. In DKA, potassium levels above normal should be left alone (unless there are significant
EKG signs of hyperkalemia). Potassium levels in the normal range should be gently
supplemented, and potassium levels below normal should be aggressively supplemented.
Bicarbonate is not routinely indicated in most cases of anion-gap acidosis and likely has no
positive effect on outcomes. Fomepizole is an alcohol dehydrogenase inhibitor used to prevent
formation of metabolites in toxic alcohol ingestion. Glucagon has opposite effects to insulin and
would likely exacerbate the problem of severe insulin deficiency seen in this DKA example.
Although infection is a common trigger of DKA, and aggressive efforts to seek a source should be
pursued, empiric antibiotics are not indicated, especially in the setting of known decreased
insulin compliance.
11. A 40-year-old male presents with delirium, polyuria, and a serum calcium level of 15 mg/dL.
Which of the following is true?
B. Thiazide diuretics are more helpful than loop diuretics in promoting urine calcium excretion.
Answer E. This patient has hypercalcemic crisis (generally defined as any patient with a serum
calcium >14 mg/dL), which is typically characterized by altered mental status, polyuria, and
dehydration. The initial goals are to restore intravascular volume with intravenous saline and to
rapidly lower the serum calcium level by enhancing urinary excretion and reducing bone
resorption (primarily through osteoclast inhibitors such as pamidronate or other
bisphosphonates). Calcium channel blockers (CCBs) such as nifedipine, play no role in the
treatment of hypercalcemia. Loop diuretics are the most effective drugs in enhancing urinary
calcium elimination. In contrast, thiazides increase calcium reabsorption in the distal tubule
resulting in a further increase in serum calcium levels. Calcitonin is the fastest acting, but
weakest agent in reducing serum calcium and cannot be used for monotherapy. Bicarbonate is
not useful in hypercalcemia. However, in the setting of a true alkalosis, more free calcium will be
bound to albumin, thereby decreasing the amount of free ionized calcium. Glucocorticoids are
useful in patients with hypercalcemia caused by an underlying hematologic malignancy or
granulomatous disease due to their effects on vitamin D metabolism and cytokine release.
Glucocorticoids do not exert their effect for 1 to 2 days after initiation of treatment.
pg. 275
12. Which of the following is true regarding adrenal insufficiency?
13. Which of the following is true regarding diabetic ketoacidosis (DKA) and hyperosmolar
hyperglycemic syndrome or state (HHS)?
Answer C. Patients with HHS have a larger fluid deficit and more significant potassium deficiency
than patients with DKA. Although seizures may occur in HHS, the most common immediate life
threat is hypovolemic shock. Hypokalemia is the next most serious immediate risk to patients
with HHS. Thromboembolic events may occur in either DKA or HHS but more commonly
complicate HHS. Thromboembolic events occur as a result of severe dehydration and resulting
hyper viscosity.
pg. 276
14. A 32-year-old female presents with recurrent episodes of headaches, palpitations, and
profuse diaphoresis. Her primary care doctor diagnosed her with an anxiety disorder after her
thyroid function “came back normal” but various selective serotonin reuptake inhibitors have
been ineffective. In the ED, her vital signs include a temperature of 99.0°F, a pulse of 90, and a
blood pressure of 175/100. Which of the following is the best agent to treat her hypertension?
A. Metoprolol
B. Hydrochlorothiazide
C. Phenoxybenzamine
D. Nifedipine
E. Enalapril
15. Upon starting your shift, you receive sign-out on a 46-year-old male diabetic patient who is
being treated with an intravenous insulin drip for diabetic ketoacidosis (DKA). Two hours later, a
repeat chemistry panel reveals the following: Na+ 141 mEq/L, Cl- 112 mEq/L, HCO3- 17 mEq/L,
blood urea nitrogen (BUN) 16 mg/dL, creatinine 0.9 mg/dL, glucose 278 mg/dL. Which of the
following is true?
A. The patient has a mixed high anion gap (AG) ketoacidosis and nonanion gap hyperchloremic
metabolic acidosis (HCMA).
B. He should be given subcutaneous insulin and his insulin infusion can be discontinued after 30
to 60 minutes.
C. His DKA is not yet resolved and he requires an ongoing intravenous insulin infusion.
E. A repeat arterial blood gas (ABG) should be performed to guide further therapy.
pg. 277
Answer B. This patient’s AG is 12, which is normal. Assuming that the patient has a normal
albumin concentration (the major contributor to the AG in healthy patients), the normal AG
reflects a resolution of the ketoacidosis. If substantial ketoacids were still present, the AG would
be persistently elevated and the patient would require an ongoing insulin infusion. The
persistently low bicarbonate signifies that a metabolic acidosis is still present, but it is an HCMA
(nonanion gap). The development of an HCMA is a common complication of DKA therapy. This is
partly due to the infusion of a large volume of saline, which contains chloride in concentrations
far greater than plasma (154 mEq/L). Another major contributor is the loss of ketoanions in the
urine that would normally serve as precursors for bicarbonate regeneration. The development
of HCMA is benign and requires no therapy. In the setting of normal renal function, it will
resolve spontaneously over the next 24 hours due to increased renal acid secretion.
Subcutaneous insulin should always be overlapped with an insulin infusion when discontinuing
continuous insulin therapy.
A. Patients with a recent myocardial infarction (MI) are at increased risk for ventricular
arrhythmias if the potassium concentration is less than 4.0 mEq/L.
Answer A. Hypokalemia is more common and generally better tolerated than hyperkalemia.
Diuretic therapy is the most common cause. Hypokalemia primarily affects the cardiac
(arrhythmias), musculoskeletal (weakness, rhabdomyolysis), GI (ileus), and renal (nephrogenic
diabetes insipidus, metabolic alkalosis) systems. Neurologic manifestations are not common.
Cardiac dysrhythmias are the most serious complication, although patients without pre-existing
heart disease rarely have any complications. In contrast, patients with acute or recent MI may
develop ventricular fibrillation in the setting of even mild hypokalemia (five times increased risk
if the potassium concentration is less than 3.9 mEq/L. Therefore, recent recommendations are
to maintain serum potassium levels greater than 4.5 mEq/L in such patients. The number or
degree of EKG changes does not correlate with the severity of hypokalemia. Neither vomiting
nor nasogastric suctioning causes significant potassium loss. Potassium is the most prominent
intracellular cation, while sodium is the most prominent extracellular cation.
17. Which of the following correctly matches the clinical entity and its effects on phosphate
metabolism?
pg. 278
A. Rhabdomyolysis causes hypophosphatemia.
Answer E. The most common causes of hypophosphatemia in the ED are probably respiratory
alkalosis, treatment of DKA, and alcoholism. The most common mechanism is an intracellular
shift of phosphate (respiratory alkalosis, treatment of DKA). Renal insufficiency leads to
phosphate retention, whereas hyperparathyroidism causes increased renal excretion.
18. A 53-year-old male with a history of alcohol abuse and chronic back pain presents to the
emergency department from an alcohol treatment center for evaluation of diaphoresis and
vomiting. The patient sought treatment for alcohol use after binge drinking for 2 months. His
last drink was about 24 hours prior to your evaluation. While waiting for evaluation at the
treatment center, he began to feel sick with multiple episodes of nonbloody emesis and
abdominal pain. Staff at the treatment center sent him to the ED for further evaluation. His
vitals are: T 99.4°F P 113, RR 18, BP 119/76, SaO2 97% on RA. His abdominal examination was
unremarkable despite his complaints of pain. His blood tests and urinalysis are significant for:
Cl−, 90 mEq/L
HCO3−, 13 mEq/L
BUN, 24 mg/dL
Glucose, 86 mg/dL
Urine ketones 4+
pg. 279
C. IV insulin infusion, IV infusion of 5% dextrose with 40 mEq of potassium chloride
Answer A. This patient is presenting with alcoholic ketoacidosis (AKA). Though patients with
diabetic ketoacidosis (DKA) can present with a normal blood glucose level (euglycemic DKA),
most patients with DKA present with elevated glucose levels and often have a history of
diabetes. Differentiating between the two disorders is usually straightforward but in patients
presenting with an elevated glucose level, a hemoglobin A1C may be helpful. Patients with AKA
are typically chronically malnourished with a history of chronic alcohol abuse. Thiamine is given
before glucosecontaining fluids since thiamine is a cofactor in glucose metabolism and many
patients with AKA are thiamine deficient so glucose administration without thiamine could
theoretically precipitate Wernicke–Korsakoff syndrome.
D. β-hydroxybutyrate is the primary ketone responsible for the acidosis and is produced due to
inadequate glucose stores.
Answer D. AKA is an elevated anion gap metabolic acidosis that usually occurs in chronic
alcoholics after a recent binge of alcohol results in vomiting, starvation, dehydration, and
acidosis. Due to complex pathophysiology, the acidosis is dominated by β-hydroxybutyrate,
which is not detected by standard urinary ketone tests. Furthermore, as the acidosis resolves,
β-hydroxybutyrate is converted to acetoacetate and acetone resulting in a paradoxical (or false)
“worsening” of the acidosis evidenced by increased detection of acetoacetate in the urine.
Traditionally, the osmolal gap is normal, although there are case reports of patients with AKA
and an elevated osmolal gap. It may be difficult to differentiate patients with AKA from chronic
alcohol abusers with toxic alcohol ingestion. Toxic alcohol ingestion should always be in the
differential of AKA and should be the top consideration whenever the osmolal gap is elevated.
The alcohol level is typically zero as AKA is primarily a starvation ketosis. Glucose levels are not
usually elevated and insulin is unnecessary. Treatment is with saline and glucose solutions as
well as electrolyte replacement.
pg. 280
20. A 24-year-old diabetic male presents with rapid, deep breathing in the setting of severe
hyperglycemia. Lungs are clear and SaO2 is 100% on RA. His urine dipstick is only mildly positive
for ketones. Which of the following is the likely explanation?
B. His rapid breathing is likely due to pulmonary pathology not a metabolic acidosis
A. Atrial fibrillation
B. Long QT syndrome
E. Sinus bradycardia
Answer E. The most common dysrhythmia in hypothyroid cardiac disease is sinus bradycardia.
Cardiomegaly and depressed cardiac contractility are other manifestations.
A. Sodium
B. Potassium
C. Magnesium
D. Calcium
pg. 281
E. Chloride
23. A 45-year-old female presents with depressed mental status, hypothermia, hypotension, and
bradycardia. She has not been taking her thyroid replacement therapy. Which of the following
findings is most likely to be present?
A. Hyperglycemia
B. Hypernatremia
C. Hypercapnia
D. Hyper-reflexia
E. Polycythemia
Answer C. Patients with hypothyroidism who present with significantly altered mental status
should be evaluated for myxedema coma. Though lacking precise definition, myxedema coma
generally represents the extreme state of hypothyroidism, where the body’s metabolic
functions are essentially shutting down. Typical findings include bradycardia, hypotension,
hypothermia, hypoventilation, hypoglycemia, anemia, etc. Because of decreased ventilatory
drive, PaCO2 rises. Sodium levels are generally low, usually due to a syndrome of inappropriate
diuretic hormone state. Reflexes are described as pseudomyotonic, which refers to the delayed
relaxation phase and are one of the most common signs seen in hypothyroidism (though they
are seen in other diseases as well). A normocytic normochromic anemia (anemic of chronic
disease) is usually observed in hypothyroidism.
pg. 282
C. Most patients have underlying Hashimoto thyroiditis.
25. Hypernatremia …
C. Is most commonly due to increased body stores of sodium (or sodium gain).
Answer A. Hypernatremia is almost always caused by the loss of free water and rarely by sodium
gain (which is usually iatrogenic). Regardless of the underlying cause, it almost never occurs in
alert patients with an intact thirst mechanism. It is a known complication of multiple doses of
activated charcoal, occurring in 6% of patients receiving such therapy. Treatment with saline
should only occur in patients who have hemodynamic compromise. Almost all patients,
however, require treatment with half-normal saline or more dilute solutions with lower tonicity.
The main complication of therapy is cerebral edema, so the rate of correction should be 0.5 to
1.0 mEq/L/hour on average (though it may be more rapid for the initial few hours if the patient
pg. 283
is suffering life-threatening complications of hypernatremia). Central pontine myelinolysis is a
complication of therapy for hyponatremia.
26. A 17-year-old female with a history of anorexia nervosa is brought to the ED by her parents
for evaluation of significant peripheral edema, shortness of breath, and orthopnea. A chest x-ray
is consistent with pulmonary edema. The patient had recently begun a refeeding program as
part of intensive outpatient treatment. Subsequent blood testing reveals numerous significant
abnormalities. Which of the following is most likely present?
A. Hypophosphatemia
B. Hyperkalemia
C. Hyperalbuminemia
D. Hyponatremia
E. Hypermagnesemia
Answer A. This patient is suffering from heart failure which is a common complication of the
refeeding syndrome. The refeeding syndrome refers to the effects that result from restoration
of normal nutrition in severely malnourished patients such as those with anorexia nervosa or
cancer- associated cachexia. Hypophosphatemia is the hallmark of the disorder and is
accompanied by hypokalemia, hypomagnesemia, and volume overload. During periods of
malnourishment, the phosphate supply is depleted to produce needed ATP and 2,3-DPG
(involved in regulating hemoglobin–oxygen binding). Many patients begin refeeding without
first receiving electrolyte replacement. While refeeding begins to replenish needed nutrients, it
also results in a profound increase in ATP and 2,3-DPG production, which ultimately depletes
phosphate levels and curbs further ATP and 2,3-DPG production. This impacts every organ
system, though the heart is most profoundly affected. Myocardial contractility is reduced,
resulting in heart failure. In addition, patients may experience altered mental status or seizures,
rhabdomyolysis, or hemolysis. The treatment is cessation of refeeding and immediate correction
of electrolyte abnormalities.
A. Neither calcium chloride nor calcium gluconate should ever be used in the setting of
concomitant digoxin use.
pg. 284
E. All of the above
Answer D. Hyperkalemia presumed to be due to digoxin toxicity generally does not require
treatment with therapy specifically directed at decreasing the K+ level. Instead hyperkalemia is a
marker of the severity of digoxin toxicity rather than a cause of toxicity. Treatment with Fab
fragments will result in rapid resolution of hyperkalemia without any adjunctive treatment. If
hyperkalemia is presumed to be due to an alternative cause, it may be treated as detailed here.
Although calcium may potentiate the effects of digoxin on the cell membrane, it is the first-line
agent for the treatment of hyperkalemia resulting in a disturbance of cardiac conduction. In
patients taking digoxin, 10 mL calcium gluconate should be diluted in 100 mL of 5% dextrose in
water and infused over 20 to 30 minutes. The effects of calcium last for 30 to 60 minutes.
Bicarbonate therapy is one of the least effective means of treating hyperkalemia and is less
effective than either albuterol or insulin. Furthermore, some authors recommend completely
discontinuing its use for the treatment of hyperkalemia. Sodium polystyrene sulfate may
exacerbate volume overload due to systemic absorption of sodium in exchange for removed
potassium. However, the most serious complication of sodium polystyrene sulfate use is
ischemic colitis and colonic necrosis, which are more common with the enema form of therapy.
28. A 26-year-old male with a history of bipolar disorder is brought in by his mother with a chief
complaint of agitation. She states that her son has been very anxious and restless, has not been
sleeping, and abruptly stopped taking his lithium medication 2 days ago. His mother has a
history of Graves disease but is otherwise healthy. In the ED, the patient’s vital signs include a
temperature of 104.1°F, pulse of 120, with sinus tachycardia visible on the monitor, and a blood
pressure (BP) of 140/70. On examination he is mildly agitated, warm and diaphoretic,
tremulous, and has a slight lid lag. Which of the following is the next best step?
Answer B. This patient is suffering from thyroid storm. There are no specific criteria for
establishing the diagnosis of thyroid storm, although scoring systems have been developed to
aid in its diagnosis. However, the diagnosis of thyroid storm remains a clinical one, as laboratory
abnormalities in thyroid storm are no different than in patients with hyperthyroidism. The
clinical manifestations of thyroid storm include fever, tachycardia, and systolic hypertension
with a widened pulse pressure, tremor (especially in the hands) as well as dysfunction of the
central nervous system (CNS) and GI disturbances. CNS disturbances range from agitation,
pg. 285
restlessness, and psychosis to confusion and coma. GI manifestations include vomiting and
diarrhea (e.g., hyperdefecation). Most cases of thyroid storm are associated with Graves disease
and occur after a precipitating event such as lithium withdrawal. Lithium inhibits thyroid
hormone release from the thyroid gland, so abrupt withdrawal may lead to a rapid rise in free
thyroid hormone levels. Although lithium can be used for the treatment of thyroid storm,
thioamides such as propylthiouracil (PTU) and methimazole are first-line agents as they prevent
the production, secretion, and peripheral conversion (in the case of PTU) of thyroid hormone.
Lugol solution or other iodine preparations should not be used until at least 1 hour after
thioamides are administered. When iodide preparations are given before PTU or methimazole,
the intrathyroidal increase in iodine results in increased thyroid hormone synthesis and release.
Aspirin should never be given to patients in thyroid storm, because it prevents thyroid hormone
from binding to carrier proteins, resulting in an increase in free thyroid hormone levels.
Dantrolene is a muscle relaxant that may be useful in neuroleptic malignant syndrome,
serotonin syndrome, or malignant hyperthermia. Propranolol is the first-line agent in thyroid
storm. It effectively combats the peripheral adrenergic effects in thyroid storm and rapidly
improves the clinical scenario.
29. A 56-year-old male with long-standing hypertension and mild renal insufficiency presents to
the ED with vague complaints of fatigue and generalized malaise. His EKG is shown in Figure
below. Which of the following is the next best step in management of this patient?
A. Nebulized albuterol
B. Insulin
C. Calcium chloride
E. Furosemide
pg. 286
Answer C. This EKG reveals changes consistent with hyperkalemia. Cardiac arrhythmias are the
most serious consequence of hyperkalemia, and the presence of EKG changes mandates
emergent therapy. This patient’s EKG demonstrates peaked T waves, which are among the early
EKG changes in the setting of hyperkalemia, typically occurring at levels above 6.5 mEq/L. In
general, hyperkalemia decreases cardiac excitability resulting in flattened P waves, a prolonged
PR interval, and a widened QRS interval. Although all the agents listed are beneficial in patients
with hyperkalemia, calcium is the agent of choice, as it has a rapid onset of action (1 to 3
minutes) and stabilizes myocardial membranes. Calcium gluconate or calcium chloride may be
given, but calcium chloride provides three times the amount of elemental calcium per unit dose.
However, calcium chloride may cause tissue necrosis upon extravasation from intravenous lines
and is irritating to local veins. Therefore, most authors recommend that calcium chloride is
delivered through a large-bore central venous catheter.
D. Hypothermia
31. A 65-year-old male with a history of myasthenia gravis (MG) presents with acute generalized
malaise and hypotension. He reports that he ran out of his medicines a week ago but doesn’t
recall their names. His vital signs are 98.5°F, 85, 18, 90/62, 99% RA. His EKG is normal. Which of
the following is the most likely cause of hypotension in this patient?
A. Addison disease
B. Glucocorticoid withdrawal
C. Adrenal hemorrhage
pg. 287
D. Infection
E. Myasthenic crisis
Answer B. Most patients with myasthenia gravis (MG) take glucocorticoids. This patient’s
hypotension is likely caused by adrenal insufficiency precipitated by glucocorticoid withdrawal.
The degree of insufficiency depends partly on the dosage, duration, and frequency. Addison
disease refers to primary adrenal insufficiency and occurs far less commonly than
corticosteroid-induced disease. Adrenal hemorrhage is very rare and most cases do not cause
insufficiency. Viral, fungal, and mycobacterial infections can occasionally cause adrenal
insufficiency; bacteria are almost never implicated. Myasthenic crises are characterized by
generalized or bulbar weakness in most patients and with primary respiratory symptoms in a
few patients. Hypotension is not characteristic.
32. A 55-year-old female on long-term steroids for sarcoidosis is brought in by paramedics with
generalized weakness and altered mental status. You quickly suspect acute adrenal
insufficiency. Which of the following are primarily responsible for the majority of acute
morbidity and mortality in acute adrenal insufficiency?
Answer C. Patients with acute adrenal insufficiency should be aggressively treated for
hypotension and hypoglycemia, which are the most immediately life-threatening complications.
Hypotension from adrenal insufficiency responds to fluids and glucocorticoid replacement
therapy far more readily than vasoactive agents. Most patients have moderate–severe
hypoglycemia and should be given dextrosecontaining fluids. Hypernatremia and hypokalemia
are never seen in adrenal insufficiency. Incontrast, hyponatremia and hypokalemia are common.
However the severity of these electrolyte abnormalities is generally not significant enough to
require aggressive correction. Similarly, hypothermia and hypercalcemia do occur but do not
cause immediately serious complications. Anorexia is a problem in chronic adrenal insufficiency
and can result in significant weight loss. Mucocutaneous hyperpigmentation (occurring in
Addison disease but not in secondary adrenal insufficiency) is not a cause for immediate alarm.
Emergent dermatologic consultation is not indicated.
pg. 288
A. QTc shortening
B. Polyuria
C. Perioral paresthesias
D. Nephrolithiasis
Answer C. The most common symptoms of hypocalcemia are neurologic and generalized
irritability such as twitching and paresthesias which may progress to frank tetany, perioral
paresthesias, as well as Chvostek and Trousseau signs. Chvostek sign is a twitch of the upper lip
when the area around the facial nerve is tapped. Trousseau sign is carpopedal spasm when a
blood pressure cuff is inflated over the arm at greater than systolic blood pressure for longer
than 3 minutes. The other signs and symptoms listed are all manifestations of hypercalcemia.
34. Which of the following medications or medication classes may exacerbate symptoms in
patients with hypercalcemia?
A. Thiazide diuretics
B. Bisphosphonates
C. Calcitonin
D. Loop diuretics
E. Glucocorticoids
Answer A. Thiazide diuretics increase calcium reabsorption, which will exacerbate symptoms of
hypercalcemia. Bisphosphonates, calcitonin, and loop diuretics (such as furosemide) all work to
decrease calcium levels. Calcitonin is faster acting than bisphosphonates, and is usually
combined with intravenous saline to decrease calcium levels in symptomatic patients.
Bisphosphonates are then given for longer term control. Loop diuretics can be given in addition
to intravenous fluids, particularly in patients with congestive heart failure, or symptoms of fluid
overload.
35. An 87-year-old female is brought to the ED by her caretaker with dehydration and excessive
somnolence. Her initial blood work reveals a sodium level of 119 mEq/L and a glucose level of
900. Which of the following represents her actual sodium level?
A. ∼109 mEq/L
B. ∼119 mEq/L
pg. 289
C. ∼125 mEq/L
D. ∼130 mEq/L
E. ∼138 mEq/L
Answer E. Since glucose exerts an osmotic pressure on cell membranes, water is extruded from
cells into the intravascular space, thereby diluting plasma sodium. The sodium level must
therefore be corrected for hyperglycemia by using the following formula:
In their 1999 paper, Hiller et al. found that the classically taught 1.6 correction factor was less
accurate and determined that the actual correction factor was 2.4.
36. The normal anion gap (AG) is primarily due to which of the following?
A. Phosphate
B. Albumin
C. Sulfate
D. Citrate
E. Acetone
Answer B. The AG is used to signify the difference between the concentration of sodium ([Na+]),
and the sum of the concentrations of chloride ([Cl−]) and bicarbonate ([HCO3−]) such that: AG =
[Na+] − [Cl−] − [HCO3−]. However, because of the law of electroneutrality, all aqueous solutions
must have an equal number of positive and negative charges such that the entire solution is
neutral. Therefore, the AG does not reflect a true “positive” or “negative” charge in the plasma.
Instead, it reflects the presence of an anion which the formula is not measuring. In normal
patients, albumin accounts for the bulk of these “unmeasured anions.” Each 1 g/dL decrease in
the concentration of albumin will decrease the expected AG by approximately 2.5 to 3 (the
normal albumin concentration is roughly 4 g/dL × ∼3 ≈ expected AG of 12). Therefore, patients
with hypoalbuminemia (e.g., cirrhosis, malnutrition) will have a smaller, normal AG. Sulfate,
phosphate, and citrate make up the bulk of the remaining unmeasured anions.
37. Which of the following is true regarding osmotic demyelination syndrome (or ODS, also
known as central pontine myelinolysis)?
pg. 290
B. It is more common after correction of acute rather than chronic hyponatremia.
D. Most common initial symptom is burning paresthesias in the hands and feet.
E. Patients do not typically present until a few days after treatment of hyponatremia.
A. Malignancy
B. Paget disease
C. Hyperparathyroidism
D. Adrenal insufficiency
E. Thiazide therapy
Answer C. More than 90% of cases of hypercalcemia are caused by primary hyperparathyroidism
or malignancy. Most cases of primary hyperparathyroidism are caused by benign parathyroid
adenomas. Malignant causes of hypercalcemia occur either through osteolytic metastatic
lesions as in breast cancer or melanoma or through secretion of parathyroid hormone-related
peptide as in squamous cell carcinoma or renal cell carcinoma.
39. A 25-year-old female presents with diffuse myalgias and dark urine after running a
marathon. Which of the following treatments is likely to be most effective?
A. Normal saline
B. Potassium
pg. 291
C. Bicarbonate
D. Calcium
E. Furosemide
Answer A. The patient likely has rhabdomyolysis, which is due to significant skeletal muscle
breakdown from a variety of causes. The most serious complication is acute renal failure from
myoglobin accumulation in renal tubules. Treatment involves normal saline hydration in very
large quantities (up to 10 to 20 L). No pharmacologic intervention has been shown to improve
outcomes in randomized controlled trials. This includes mannitol, bicarbonate, and chelation
therapy. These agents may be employed on a case-by-case basis. Potassium should never be
supplemented, as myocyte breakdown in rhabdomyolysis releases large amounts of intracellular
potassium into the bloodstream. Although hypocalcemia can occur in many cases of
rhabdomyolysis, supplementing calcium can actually increase intracellular calcium and worsen
myocyte injury. Furosemide and other loop diuretics cause urinary acidosis and should be
avoided.
40. A 26-year-old female presents with intermittent nausea and vomiting, orthostatic
lightheadedness, and mild diffuse abdominal pain. She also notes that her skin has darkened
over the last month although she has been indoors. Which of the following laboratory
abnormalities are you most likely to find?
A. Hyponatremia
B. Hyperglycemia
D. Hypokalemia
Answer A. This patient has primary adrenal insufficiency. Due to mineralocorticoid deficiency,
the most common electrolyte abnormalities are hyponatremia and hyperkalemia. Hypoglycemia
due to decreased gluconeogenesis and increased peripheral glucose utilization is also common.
The patient’s hyperpigmentation is due to increased adrenocorticotropic hormone secretion by
the pituitary gland and subsequent stimulation of melanocytes. More than 50% of patients with
adrenal insufficiency have intermittent nausea and vomiting and often present with volume
depletion.
41. A 44-year-old alcoholic man presents with nausea and vomiting. He reports drinking alcohol
for 2 days straight. Physical examination is normal except for a mildly intoxicated patient with
pg. 292
extremely dry mucous membranes. Urine dip is positive for ketones. The chemistry panel is as
given:
Cl−, 92 mEq/L
HCO3−, 18 mEq/L
BUN, 32 mg/dL
A. Bicarbonate
B. Insulin
C. Glucose
D. Potassium
E. Magnesium
Answer C. The patient has nausea, vomiting, elevated anion gap, ketosis, and normal glucose in
the setting of excessive alcohol use with starvation. Alcoholic ketoacidosis (AKA) is the most
likely cause. Treatment of AKA is with fluid resuscitation, glucose, and thiamine. Bicarbonate is
not indicated in most patients with high anion gap metabolic acidosis except in severe,
life-threatening cases. Insulin is indicated in patients with DKA, who rarely present with a
normal glucose level. Since alcoholic patients have adequate pancreatic endocrine function,
glucose administration induces endogenous insulin release which quickly results in glucose
utilization and closure of the anion gap (as the stimulus for ketogenesis is removed). Alcoholics
are frequently thiamine-deficient because of poor nutrition, and thiamine is used during glucose
metabolism, so supplementation should be given concurrently or before glucose administration.
Potassium repletion may be indicated if hypokalemia is present or expected during the course of
therapy. Magnesium supplementation is often indicated in chronic alcoholic patients, but
glucose therapy is of more importance as an energy substrate in patients with alcohol
ketoacidosis.
42. Which of the following is true regarding hyperosmolar hyperglycemic syndrome or state
(HHS), also known as hyperglycemic hyperosmolar nonketotic coma (HHNC)?
pg. 293
B. The mortality rate is lower than in patients with diabetic ketoacidosis (DKA).
D. The degree of mental status change correlates with the plasma pH.
Answer C. Most authors have abandoned the old terminology of HHNC in favor of HHS. HHS
reflects the fact that <20% of these patients present in a coma. HHS is the result of an
unremitting osmotic diuresis due to hyperglycemia and is characterized by extreme
hyperglycemia (mean glucose is >1,000), hyperosmolarity (at least >320 mOsm/L), and
dehydration (average fluid deficit is roughly 10 L). Although a minor ketosis may be present, it is
never significant. Infections such as pneumonia or urosepsis are the most common precipitants,
accounting for 60% of cases. The mortality rate, which hovers at approximately 15%, is
significantly higher than in patients with DKA. The degree of mental status change correlates
with the serum osmolarity rather than the degree of hyperglycemia. Insulin is a second-line
agent in HHS. It should never be given without prior intravenous fluid administration because
insulin delivery will encourage glucose utilization and entry into cells resulting in an acute drop
in intravascular fluid volume and possible circulatory collapse.
43. A 64-year-old female with small cell lung cancer presents with a chief complaint of fatigue,
dizziness, and imbalance. Her blood work is significant for a sodium level of 112 mmol/L. You
suspect she has a syndrome of inappropriate secretion of antidiuretic hormone (SIADH) due to
her lung cancer.
E. Make no difference in her sodium level, and cause water retention and edema.
Answer C. In the setting of SIADH, hypertonic saline (3%) must be given in order to correct
hyponatremia. When isotonic normal saline is given (0.9%), the body “desalinates” it, delivering
the salt to the kidneys to make concentrated urine, retaining the free water, and worsening the
hyponatremia. Therefore, in symptomatic patients with critical hyponatremia (sodium <120
mmol/L), hypertonic saline must be given. However, fluid restriction is the mainstay of
treatment for more mild reductions in plasma sodium. Neoplasms are the most common cause
pg. 294
of SIADH and small cell lung carcinoma is the most common neoplasm associated with the
disorder.
44. A 20-year-old male presents with diffuse lower extremity myalgias and dark urine after
participating in an adventure race. Which of the following is true?
C. Acute kidney injury is likely if the creatine kinase (CK) is >2,000 Units/L
Answer E. This patient presents with rhabdomyolysis. Hypocalcemia is the most common
electrolyte abnormality as calcium floods the intracellular space when myocyte membranes fail.
Because hyperphosphatemia may also occur, treatment of hypocalcemia is not recommended
unless symptoms are severe or unless severe hyperkalemia develops. Otherwise, calcium
phosphate may precipitate and form deposits in tissues. Myoglobin has a plasma half-life of only
1 to 3 hours, making detection sometimes difficult. Although myoglobinuria is pathognomonic
for rhabdomyolysis, measurement of creatine kinase (CK) is the most sensitive method to detect
muscle cell injury. CK has a half-life of 1.5 days. The classic finding of a urine dipstick that is
positive for blood while microscopic urinalysis reveals no red blood cells is present only 50% of
the time. In addition to trauma, HMG-CoA reductase inhibitors are a well-known cause of
rhabdomyolysis. IV fluids are the mainstay of treatment, and should be started when the CK
>5,000 Units/L. Alkalinization may be helpful while loop diuretics offer no benefit.
A. Sarcoidosis
B. Hemorrhage
C. Pituitary insufficiency
D. Idiopathic
E. Iron deposition
Answer D. Increasingly, acute adrenal insufficiency is thought to be a rare condition, and most
cases of acute adrenal insufficiency probably represent an exacerbation of chronic disease.
When acute adrenal insufficiency occurs, it is most commonly due to exogenous glucocorticoid
pg. 295
administration. Chronic adrenal insufficiency is idiopathic (thought to be autoimmune-mediated
destruction of the adrenal gland) in 66% to 75% of cases.
46. Which of the following may worsen acute renal failure due to rhabdomyolysis?
A. Mannitol
B. Saline
C. Bicarbonate
D. Furosemide
E. Deferoxamine
47. Which of the following EKG changes may be seen in the setting of hypercalcemia?
A. ST-segment depression
B. QTc-segment shortening
C. Widened T wave
D. Bradycardia
Answer E. The most commonly cited abnormality is QTc shortening, but any of the listed
changes may occur.
pg. 296
Urology and nephrology
emergency
➢ Chapter includes:
Urological emergency
Obstructed and infected urolithiasis
Testicular torsion
UTI and pyelonephritis
Nephrology and ESRD
✓ Exceptions
Gento-urinary trauma in traumatology chapter
Sexually transmitted disease in infectious chapter
pg. 297
1. Which of the following has the highest sensitivity for ruling out testicular torsion?
B. Presence of Prehn sign (relief of scrotal pain upon elevating the scrotum)
C. Normal urinalysis
D. Absence of fever
E. Absence of vomiting
Answer A. Patients with testicular torsion will almost never exhibit a normal cremasteric reflex
on the affected side. The sensitivity of this sign is extremely high (>95%). Prehn sign refers to
relief of scrotal pain on elevation of the scrotum. Prehn sign was previously thought to
distinguish epididymitis from testicular torsion but has been found to be inaccurate in this
regard. Although urinalysis is usually normal, up to one-third of patients with torsion may have
detectable urinary leukocytes. Fever is present in 20% and vomiting in 30% of cases, so absence
of either should not be used to rule out the diagnosis.
2. A 22-year-old male presents with acute onset of right scrotal pain for 2 hours. He has severe,
colicky pain with nausea and vomiting but no fevers, chills, or dysuria. Vital signs are normal, but
the patient is in extreme discomfort. Abdominal examination is normal. Testicular examination
reveals a tender right testis with an absent ipsilateral cremasteric reflex. Which of the following
is the most appropriate next step in evaluation?
D. Retrograde urethrogram
Answer C. The patient has evidence of acute testicular torsion, with scrotal pain, nausea,
vomiting, and testicular tenderness. Pathophysiology involves twisting of the testis on the
spermatic cord due to an anatomic abnormality or trauma. The most sensitive physical
examination finding is the cremasteric reflex—presence of this reflex virtually rules out the
diagnosis. Diagnosis can be made clinically in some cases, but confirmation is made with color
Doppler ultrasonography, which has excellent sensitivity and specificity when performed by
experienced operators. Prompt diagnosis is essential, as testicular survival is directly dependent
on duration of symptoms—if surgical management is instituted within 6 hours of pain,
approximately 100% of cases are salvageable. Advanced imaging techniques such as CT and MRI
pg. 298
scans do not add to the diagnostic accuracy of testicular torsion and waste valuable time.
Retrograde urethrogram is used to diagnose traumatic injuries to the urethra and is not
indicated here. The bulbocavernosus reflex is used to evaluate spinal cord injuries in trauma
patients and has no role in the evaluation of the acute scrotum.
A. Bacterial infection
B. Viral infection
C. Fungal infection
D. Parasitic infection
E. Allergic urethritis
Answer A. Dysuria in all patients is most commonly due to bacterial UTI. Gram-negative enteric
rods are the number one causative group, with E. coli as the single most likely etiologic agent.
Urethritis due to Chlamydia and gonococcus is also extremely common, as are candidal vaginitis
and bacterial vaginosis. Viral and parasitic infections are uncommon causes of dysuria. Allergic
urethritis may be responsible in patients who have long-term foreign bodies (such as Foley
catheters) in place.
4. A 19-year-old male presents with acute onset of right testicular pain and nausea for 5 hours.
Physical examination reveals a markedly swollen and tender testis in a horizontal lie with an
absent right cremasteric reflex. Which of the following is the most appropriate definitive
management?
A. Antibiotics
B. Analgesia
C. Manual detorsion
D. Operative orchidopexy
Answer D. Testicular torsion is the most likely diagnosis because of the acute testicular pain,
nausea, and absence of ipsilateral cremasteric reflex. Pathophysiology involves twisting of the
testis on the spermatic cord due to an anatomic abnormality or trauma. In unclear cases, color
Doppler ultrasonography is the diagnostic test of choice, but in textbook cases such as this,
emergent urologic consultation is indicated. Prompt diagnosis is essential, as testicular survival
pg. 299
is directly dependent on duration of symptoms—if surgical management is instituted within 6
hours of pain, approximately 100% of cases are salvageable. Definitive surgical management
involves bilateral orchidopexy. Antibiotics are used to treat epididymitis, a common condition in
the differential diagnosis of the acute scrotum. Analgesics and antiemetics should be considered
conjunctive therapy in patients with testicular torsion but do not affect the disease process
itself. Manual detorsion may be used as a temporizing measure, but should never be considered
definitive therapy. Lithotripsy is used to treat kidney stones, and has no role in the management
of testicular torsion.
Answer C. Urinalysis is a crucial diagnostic tool for the evaluation of all urinary system
conditions. The presence of casts in the urine indicates a renal source—RBC casts are associated
with glomerulonephritis and white blood cell casts with parenchymal inflammation, such as
pyelonephritis. Urine dipstick is a rapid screening tool to detect the presence of glucose,
leukocytes, protein, and blood. Unfortunately, sensitivity for most of these parameters is only
on the order of 75% to 85% and negative dipstick should not be used to rule out their presence.
Transitional cells are from a bladder source but are usually a normal finding and do not
necessarily indicate a malignant process. Normal urinary pH is from 5 to 8 and usually mirrors
serum pH except in certain disease states, such as renal tubular acidosis or urinary tract
infection.
A. Female gender
B. Hypoparathyroidism
C. Crohn disease
D. Hyperthyroidism
E. Diabetes mellitus
pg. 300
Answer C. Kidney stones most commonly occur in middle-aged patients, usually men. Risk
factors include age, male gender, family history, and conditions which increase serum and
urinary calcium levels. Kidney stones are divided into four main categories—calcium,
magnesium ammonium phosphate, uric acid, and cystine. Calcium stones represent
approximately two-third of all stones and occur more often in patients with common
precipitants of hypercalcemia, including hyperparathyroidism, milk–alkali syndrome, laxative
abuse, and sarcoidosis. Inflammatory bowel disease also leads to the formation of calcium
oxalate stones, due to hyperoxaliuria. Magnesium ammonium phosphate (struvite) stones
account for one-fifth of all calculi and occur in patients with UTIs due to Proteus, Klebsiella, and
Pseudomonas. Uric acid stones occur in patients with hyperuricemia, often due to gout. They
are usually radiolucent and missed on plain radiographs. Cystine stones are the least common
and are due to hypercystinuria, an inborn error of metabolism usually diagnosed at birth.
Answer E. CT has become the test of choice for diagnosis of kidney stones, replacing the
intravenous pyelogram in this regard. It has excellent sensitivity and specificity and is helpful in
evaluating other conditions in the differential diagnosis of flank pain. Twenty percent of kidney
stones have normal urinalyses, without microscopic hematuria, so a normal urinalysis in highly
suspicious cases by no means rules out the diagnosis of kidney stone. Radiographs have <75%
specificity for the diagnosis, and false positives from phleboliths, calcified lymph nodes, and
bone shadowing are common. Most kidney stones are radiopaque—uric acid stones,
representing approximately 10% of all stones, are radiolucent. Ultrasonography, although
possessing good sensitivity and excellent specificity for hydronephrosis, has relatively poor
sensitivity for ruling out the diagnosis of kidney stone.
8. A 23-year-old female presents with fever, chills, and right flank pain. She just completed
treatment for pyelonephritis with a 2-week course of ciprofloxacin. The patient states that the
symptoms are very similar to when she had pyelonephritis and she cannot understand why she
did not get better with the antibiotics. She admits to having waited “longer than usual” before
seeking care for the pyelonephritis during the first visit, but swears that she took all the
antibiotics as directed. The initial urine culture revealed E. coli that was sensitive to
ciprofloxacin. Which of the following is the most appropriate next step in management?
pg. 301
A. Switch to cefpodoxime for 10 days
Answer E. The patient has failed outpatient therapy for pyelonephritis. The possible reasons for
this include antibiotic noncompliance, antibiotic resistance, a concomitant kidney stone, or a
renal abscess. The patient has had 2 weeks of appropriate antibiotic therapy for pyelonephritis,
which should be adequate for clinical cure unless a kidney stone or renal abscess is present. CT
is recommended if such a complication is suspected. Initiating treatment with a more
narrow-spectrum antibiotic such as TMP-SMX or metronidazole is not indicated, especially if no
urine culture is performed beforehand. In this patient’s case, the urine culture suggests that
ciprofloxacin should normally be adequate therapy. However, given the resistance rates in many
communities, the best empiric choice for outpatient treatment of pyelonephritis is a
third-generation oral cephalosporin such as cefixime. Extending the course of ciprofloxacin
beyond 2 weeks is only indicated in cases of male prostatitis.
9. A 22-year-old male presents with a swollen area on his scrotum shown in Figure below. The
area is firm and nontender to palpation and does not transilluminate. Which of the following is
the most appropriate next step in management?
A. Emergent surgery
B. Azithromycin PO
D. Scrotal elevation
E. Corticosteroids
pg. 302
Answer C. The patient likely has a testicular tumor, usually of germ-cell type in this age-group.
Evaluation may include screening for metastases with a chest x-ray and a CT scan of the
abdomen and pelvis, although this may occur on an urgent outpatient (rather than emergent
inpatient) basis. Emergent surgery is not indicated in patients with testicular tumors except in
certain patients who have evidence of testicular torsion. Azithromycin may be used to treat
chlamydial epididymitis but has no role in the management of testicular tumors. Scrotal
elevation is often used as a diagnostic maneuver to distinguish between epididymitis and
testicular torsion (relief is called Prehn sign), but it is an unreliable finding for this purpose.
Corticosteroids may be used in patients with testicular tumors as part of certain
chemotherapeutic regimens but has no role in the acute management.
10. A 52-year-old male presents with acute, right-sided flank pain. He has a history of kidney
stones and states that he feels this is the same kind of pain. You confirm the diagnosis of a
3-mm, distal ureteral stone with CT imaging. His vital signs, physical examination, urinalysis, and
creatinine are all normal. His pain is under control after parenteral analgesics. Which of the
following is the most important next step in management?
A. Discharge home with instructions to drink 10 L of water per day to “flush out” the stone
B. Discharge home with instructions to take ibuprofen 600 mg every 6 hours as needed for pain
C. Discharge home with instructions to take clonidine 0.1 mg three times a day
Answer B. Patients with kidney stones less than 5 mm who are minimally symptomatic usually
pass the stones spontaneously and do not need further intervention other than nonsteroidal
anti-inflammatory drug (NSAID) therapy, moderate fluid intake, and urologic follow-up. There is
no proven benefit in outcomes to drinking large amounts of free water—this practice may
actually cause harm (due to electrolyte shifts). Clonidine has no role in the management of
kidney stones. Tamsulosin is often prescribed and may be more effective in larger stones (>5
mm). Emergent urologic consultation or admission is not generally indicated, unless patients
have a solitary or transplanted kidney, sepsis, a significantly elevated creatinine, intractable pain
or vomiting, or stones larger than 10 mm.
11. A 45-year-old male presents with progressive scrotal pain for 3 days. He denies any swelling
in the area, but reports mild dysuria. Physical examination demonstrates tenderness in the
pg. 303
epididymis, normal descended testes, and normal bilateral cremasteric reflexes. Which of the
following is the most likely cause of the symptoms?
A. Chlamydia
B. Neisseria gonorrhoeae
C. Escherichia coli
D. Chemical epididymitis
E. Testicular torsion
Answer C. The patient has evidence of acute epididymitis, an infection of the epididymis causing
local tenderness and lacking findings suggestive of testicular torsion (unilateral testicular
tenderness or edema or absent cremasteric reflex). The etiologies in sexually active men
younger than 35 years of age are Chlamydia or gonococcus. In men older than 35 years, E. coli is
the most common cause. Antibiotic therapy is directed to the causative organism.
12. A 34-year-old male presents with acute onset of penile pain and swelling, which occurred
during sexual intercourse. His penis is shown in Figure below. Which of the following is the most
appropriate definitive management?
A. Observation
B. Foley catheterization
C. Surgical repair
D. Penile splinting
pg. 304
Answer C. The patient has ecchymosis and deformity of his penile shaft indicative of penile
fracture. Although important ancillary studies such as ultrasonography, retrograde
urethrography, and cavernosography may be necessary for further evaluation, definitive
management almost always requires urgent urologic repair. Conservative management with
penile splinting and pressure dressings has an unacceptably high risk of complications such as
deformity and impotence. Foley catheterization may be performed in patients without urethral
injuries to help guide the surgical repair.
13. A 50-year-old male develops acute-onset severe right flank pain. A CT scan demonstrates a
calculus in the bladder. The patient has never had a kidney stone before. He asks you what his
risk of getting another stone is. You tell him that the lifetime risk of recurrence is approximately:
A. <1%
B. 10%
C. 25%
D. 50%
E. >99%
Answer D. Kidney stones most commonly occur in middle-aged patients, usually men.
Recurrence occurs in approximately half the number of patients. Risk factors include age, male
gender, family history, and conditions which increase serum and urinary calcium levels. Kidney
stones are divided into four main categories—calcium, magnesium-ammonium-phosphate, uric
acid, and cystine. Calcium stones represent approximately two-third of all stones and occur
more often in patients with common precipitants of hypercalcemia, including
hyperparathyroidism, milk–alkali syndrome, laxative abuse, and sarcoidosis. Inflammatory
bowel disease (IBD) also causes the formation of calcium oxalate stones, due to hyperoxaluria.
Magnesium-ammonium-phosphate (struvite) stones account for one-fifth of all calculi and occur
in patients with urinary tract infections due to Proteus, Klebsiella, and Pseudomonas. Uric acid
stones occur in patients with hyperuricemia, often due to gout. They are usually radiolucent and
missed on plain radiographs. Cystine stones are the least common and are due to
hypercystinuria, an inborn error of metabolism usually diagnosed at birth.
14. A 12-year-old male presents with progressive testicular swelling (Fig. below). Which of the
following is the most common complication of this condition?
A. Testicular torsion
B. Epididymitis
C. Infertility
pg. 305
D. Malignancy
Answer C. The patient has a large left-sided varicocele. Varicoceles are caused by abnormal
dilation of the testicular vein and pampiniform plexus of the scrotum due to venous pooling
from impaired drainage of the left internal spermatic vein into the left renal vein. Large
varicoceles substantially increase the risk of infertility due to impaired blood flow and
temperature of the ipsilateral testis. Testicular torsion may occur in patients with varicoceles
but is not as common as infertility. Epididymitis and malignancy do not occur at appreciably
higher rates in patients with varicoceles. Deep venous thrombosis due to inferior vena cava
thromboses may cause varicoceles, but this is rare.
15. A 60-year-old male presents with painless hematuria. Which of the following is the most
likely cause?
A. Renal carcinoma
B. Bladder carcinoma
D. Glomerulonephritis
E. Nephrotic syndrome
Answer B. The causes of painless hematuria vary by age and gender. The most common cause in
children is glomerulonephritis, in young adults and older women, UTI, and in older men, bladder
pg. 306
cancer. Renal carcinoma is a less common cause of painless hematuria in all age groups.
Nephrotic syndrome causes proteinuria without frank hematuria. The combination of urinalysis
and appropriate imaging studies (such as helical CT scan) yields the diagnosis in most cases.
16. A 60-year-old male presents with testicular pain for 2 days and fever of 101°F. He also
complains of dysuria, but denies scrotal edema, flank pain, nausea, or vomiting. Physical
examination demonstrates moderate tenderness with mild edema and erythema in the scrotal
area. Cremasteric reflexes are present bilaterally. A testicular ultrasound is performed and is
negative for torsion. Which of the following is the most likely etiology of the patient’s
symptoms?
A. Viral
B. Chlamydia
C. N. gonorrhoeae
D. E. coli
E. Pseudomonas aeruginosa
Answer D. The patient has evidence of acute epididymitis. In men older than 35 years, the most
common cause is E. coli. In men younger than 35 years, Chlamydia is the number one cause,
followed by gonococcus. It is crucial for the emergency physician (EP) to distinguish between
testicular torsion and epididymitis. Epididymitis is characterized by the gradual progression of
symptoms, dysuria, and the presence of cremasteric reflexes. Focal epididymal swelling is
followed by generalized edema and erythema of the scrotum. Low-grade fever is present in
most patients. Only half the number of patients with epididymitis have leukocytes in the urine.
Treatment involves antibiotics to cover suspected organisms based on age of the patient:
Ceftriaxone plus doxycycline for patients younger than 35 years, and
trimethoprim–sulfamethoxazole or ciprofloxacin for patients older than 35 years.
17. A 65-year-old male presents with fever, chills, and dysuria for 2 days. He denies vomiting or
back pain. Physical examination reveals a patient in mild discomfort, with normal cardiac,
pulmonary, and abdominal examinations. He lacks costovertebral angle tenderness, but rectal
examination reveals a boggy, tender prostate. Which of the following is the most appropriate
therapy?
B. Azithromycin 1 g PO
pg. 307
D. Ciprofloxacin 500 mg PO b.i.d. for 7 days
Answer E. The patient has acute bacterial prostatitis. Fever, low back pain, and UTI symptoms
are common. A warm, tender prostate is the characteristic physical examination finding. The
prostate should never be massaged because of the possibility of bacteremic spread. Etiology is
almost always due to gram-negative enteric bacilli, most commonly E. coli. Diagnosis is made by
physical examination combined with urinalysis, as there is often a concomitant cystitis.
Treatment is with either trimethoprim–sulfamethoxazole or a fluoroquinolone. Treatment must
be continued for 1 month to assure clinical cure. Ceftriaxone and doxycycline are the drugs of
choice for young men with urethritis. Azithromycin is an acceptable alternative for treating
chlamydial infections. Three-day and 7-day regimens with a fluoroquinolone are used for
treating uncomplicated UTI and complicated UTI, respectively, in women.
C. Gram-negative intracellular diplococci on urine Gram stain indicate Escherichia coli infection.
19. A 44-year-old male with a history of kidney stones presents with progressively worsening left
flank pain for several days, dysuria, nausea, vomiting, and fever to 101°F. Urinalysis
demonstrates 50 WBC per hpf, positive leukocyte esterase, and positive bacteria. A CT scan of
the abdomen and pelvis demonstrates a 7-mm stone at the left ureteropelvic junction. Which of
the following is the most appropriate next step in management?
pg. 308
B. IV antibiotics in the emergency department then discharge home with oral hydrocodone,
promethazine, and antibiotics
Answer E. The patient has an infected kidney stone, which, at 7 mm, is very unlikely to pass
spontaneously. This represents a true emergency and will likely require specific urologic
management. The patient should be admitted to the hospital and given intravenous fluids,
analgesics, antiemetics, and antibiotics after urinary culture has been sent. Other indications for
emergent urologic consultation in patients with kidney stones are the presence of acute renal
failure, high-grade obstruction due to the calculus in a patient with only one kidney, and a stone
>5 mm with intractable symptoms of pain and/or nausea. Discharging the patient with an
infected kidney stone without urologic approval is contraindicated. MRI will not add significantly
to this patient’s diagnosis or management and is an unnecessary waste of time.
A. The best test to determine if the rash and fever are due to the drug is to stop the drug
C. The erythrocyte sedimentation rate (ESR) is usually low or normal in patients with drug fever
E. Most cases of drug fever present within the first 48 hours after starting the drug
Answer A. Antibiotics are the most common cause of drug fever. In particular, penicillins,
sulfonamides, and nitrofurantoin are common causes of drug fever, although this may be due, in
part, to their more widespread use since they have been in use for many years. Unfortunately,
there is no diagnostic laboratory test. While the erythrocyte sedimentation rate (ESR) is often
elevated, an elevated ESR is neither sensitive nor specific for drug fever. The presence of a rash
may make drug fever more likely, although most patients don’t have a rash at the time of
diagnosis. Finally, the median time of onset is 8 days after starting the drug but is highly variable
and not diagnostic.
pg. 309
21. Which of the following is the most common cause of nongonococcal urethritis?
A. Ureaplasma
B. Trichomonas
C. Hemophilus ducreyi
D. C. trachomatis
Answer D. Chlamydia is the most common cause of nongonococcal urethritis, accounting for
over half of all cases. Symptoms are very similar to a urinary tract infection—however, urethral
discharge worse in the morning is more characteristic of urethritis. Screening for other sexually
transmitted diseases should be pursued. Therapy is with doxycycline or azithromycin and sexual
partners should also be treated. Choices A, B, and E all cause urethritis, but are less common
than Chlamydia. Choice C causes chancroid, a syndrome of a painful, ulcerated lesion on the
genitalia in association with inguinal lymphadenopathy.
22. A 46-year-old male presents with a chief complaint of a painful penile erection that has
persisted for 6 hours after using sildenafil. Which of the following is true?
C. Due to high blood flow, future erectile dysfunction doesn’t occur until an erection persists for
18 hours
E. At least 5 to 10 mL of blood should first be aspirated from the corpora cavernosa prior to
injection of a sympathomimetic agent
Answer E. Technically, priapism is defined as a persistent erection not related to sexual desire.
Practically speaking, most studies define it as an erection lasting at least 4 hours. Structural
damage can occur as early as 4 to 6 hours and future sexual functional steadily declines the
longer priapism persists. Priapism is a compartment syndrome of the two corpora cavernosa
within the dorsum of the penis. Sildenafil and other medications cause ischemic priapism (also
called vaso-occlusive priapism), the most common subtype, which results because of inadequate
venous outflow once the corpora are engorged with blood. High-flow priapism is usually due to
arterial–venous fistulas caused by trauma and are uncommon. Aspiration and injection of a
sympathomimetic agent is the cornerstone of ischemic priapism treatment. Phenylephrine is the
preferred agent because of its favorable side effect profile. After a penile block, at least 5 to 10
pg. 310
mL of blood is aspirated from the corpora cavernosa to decompress the corpora. Irrigation with
saline can be used if aspiration is difficult due to blood clotting. Phenylephrine is then injected
into the corpora cavernosa and the penis is reassessed frequently. Repeat injections can be
given every 3 to 5 minutes for up to an hour to achieve detumescence.
23. Which of the following findings on urinalysis is found in pyelonephritis but not cystitis?
B. WBC casts
C. Nitrites
D. Leukocyte esterase
E. Bacteria
Answer B. The presence of WBC casts indicates infection from a renal source. Other indices on
urinalysis cannot distinguish between upper urinary tract infection (UTI; pyelonephritis) and
lower UTI (cystitis or urethritis). Clinically, pyelonephritis usually involves back pain and systemic
symptoms of fever, nausea, vomiting, and signs of sepsis. Symptoms of uncomplicated cystitis
are generally limited to dysuria, increased urinary frequency, and urgency.
24. Which of the following is a common cause of death in patients with acute renal failure?
A. Hypercalcemia
B. Hyperkalemia
C. Hypermagnesemia
D. Hypernatremia
E. Hyperphosphatemia
Answer B. The most common causes of death in patients with acute renal failure are volume
overload and hyperkalemia. Hyperkalemia can result in fatal dysrhythmias. Treatment involves
correction of the renal insufficiency, potassium-binding resin, intravenous calcium for
cardio-protection, bicarbonate, and insulin and glucose. Hypocalcemia, not hypercalcemia,
occurs with acute renal failure due to decreased levels of activated vitamin D.
Hypermagnesemia and hypernatremia may occur in renal failure but are usually clinically
inconsequential. Hyperphosphatemia also occurs but is usually adequately managed with
correction of the renal failure and calcium antacids to bind excess gastrointestinal phosphate.
pg. 311
25. A 30-year-old female presents with painless hematuria and increased urinary frequency.
Which of the following is the most likely cause?
A. Renal carcinoma
B. Bladder carcinoma
D. Glomerulonephritis
E. Nephrotic syndrome
Answer C. The causes of painless hematuria vary by age and gender. The most common cause in
children is glomerulonephritis, in young adults and older women, UTI, and in older men, bladder
cancer. Renal carcinoma is a less common cause of painless hematuria in all age-groups.
Nephrotic syndrome causes proteinuria without frank hematuria. The combination of urinalysis
and appropriate imaging studies (such as helical CT scan) yields the diagnosis in most cases.
26. A 60-year-old male presents with acute urinary retention. He has been unable to urinate for
the last 10 hours and has extreme discomfort in his lower abdomen. His physical examination is
normal except for a distended suprapubic region and nontender prostatic hypertrophy. He
appears very uncomfortable. Which of the following is the most appropriate next step in
management?
C. Renal ultrasonography
Answer E. For any patient with acute urinary retention, the goal is to find the cause of urinary
obstruction and correct it as rapidly as possible. In this case, as in the majority of older men, the
most likely cause is benign prostatic hypertrophy (BPH). Pain and distension in the suprapubic
area indicates bladder distention from a more distal obstruction. The correct management is to
urgently place a Foley catheter to temporarily stent open the prostatic urethra and decompress
the bladder. Any imaging and blood studies may follow this initial management, but should not
delay relief of the obstruction. Further management usually involves a basic chemistry panel to
assess kidney function, urinalysis to look for concomitant infection, and outpatient urology
follow-up.
pg. 312
27. Which of the following is the most frequent complication during hemodialysis?
A. Muscle cramps
B. Headache
D. Hypotension
E. Chest pain
Answer D. Hypotension complicates roughly 15% to 50% of hemodialysis sessions and is more
common among the elderly and female dialysis patients. It is primarily due to the volume and
rate of plasma fluid removal. Patients referred to the ED with asymptomatic hypotension after a
dialysis session should receive a small bolus of normal saline and frequent reassessment.
Though septicemia is a very common cause of death in dialysis patients (second most common,
after cardiovascular disease), most septic patients typically have some additional evidence of
infection, such as fevers, chills, tachycardia or focal symptoms of illness, in addition to
hypotension. Muscle cramps are the second most common complication of dialysis, and are also
thought to be due primarily to the rate of volume removal. Thus, they are treated with a small
fluid bolus, and diazepam, if needed. The FDA has issued a warning against the use of quinine,
which has long been prescribed for muscle cramps, due to its long list of adverse side effects.
28. A 24-year-old female presents with dysuria and increased frequency of urination for 2 days.
She denies fevers, vomiting, or back pain. She is allergic to sulfa drugs and fluoroquinolones.
Urinalysis demonstrates 25 WBCs per high-powered field, leukocyte esterase, and nitrites.
Which of the following is the most appropriate antibiotic regimen?
A. Ciprofloxacin
B. Trimethoprim–sulfamethoxazole (TMP-SMX)
C. Doxycycline
D. Azithromycin
E. Nitrofurantoin
Answer E. The patient has evidence of uncomplicated urinary tract infection (UTI). The most
common cause is E. coli, followed by other gram-negative bacilli, then streptococci. Treatment
in cases of uncomplicated UTI is for 3 days with either TMP-SMX or a fluoroquinolone, or 5 days
with nitrofurantoin, depending on local resistance patterns. The overuse of levofloxacin for
respiratory infections has fostered fluoroquinolone resistance among a significant number of
pg. 313
urinary E. coli isolates, making it useless for empiric treatment of UTIs in many communities.
Regardless, the patient has allergies to both sulfa drugs and fluoroquinolones. Doxycycline and
azithromycin have better coverage against gram-positive organisms and atypicals and often lack
effectiveness against gram-negative bacilli. In some communities, E. coli also remains sensitive
to cephalexin, amoxicillin, or amoxicillin–clavulanic acid.
29. Which of the following is the most important factor in determining the chance of
spontaneous passage of a kidney stone?
C. Degree of pain
D. Degree of nausea
Answer B. The large majority of kidney stones <5 mm will pass spontaneously without the need
for lithotripsy or surgical extraction. The large majority of stones >5 mm will not pass
spontaneously. Stones may become stuck in several areas along the urinary tract—renal calyx,
ureteropelvic junction, midureter at the iliac vessels, ureterovesicular junction, and vesical
opening. Urologic management of kidney stones is through extracorporeal shock wave
lithotripsy, percutaneous nephrolithotomy, or surgical extraction. Pain and nausea are potential
indications for admission of patients with kidney stones but are not independently associated
with low passage rate. Age of the patient and composition of the stone are not directly
associated with passage of the stone.
30. A 45-year-old male presents with acute onset of left flank pain. He is extremely
uncomfortable and writhing in pain. After appropriate pain control, he is sent for a CT scan of
the abdomen and pelvis, which demonstrates a 2-mm kidney stone in his mid-right ureter.
Which of the following is true regarding this patient?
C. The stone has already traversed the narrowest portion of the ureter.
pg. 314
Answer A. The large majority of kidney stones <5 mm will pass spontaneously without the need
for lithotripsy or surgical extraction. The large majority of stones >5 mm will not pass
spontaneously. Urinalysis in patients with kidney stones most often shows microscopic
hematuria, although 10% to 20% of cases will have completely normal urinalyses. The narrowest
portion of the ureter, and one of the most common sites of obstruction, is the ureterovesicular
junction at the distal-most point of the ureter. Medical management of kidney stones involves
aggressive pain and nausea treatment, copious fluid intake, and stone analysis to assess for risk
factors to prevent recurrence. The most common stone types are in decreasing order of
frequency: calcium, struvite, uric acid, and cystine.
31. A 55-year-old male presents with acute, left-sided flank pain. Which of the following
increases the likelihood that the patient’s pain is due to acute ureterolithiasis as opposed to
another etiology?
A. Black race
B. Hypotension
C. Fever
D. Hematuria
Answer D. Patients with acute ureterolithiasis usually (sensitivity of 71% to 95%) have hematuria
on urinalysis. Importantly, the absence of hematuria does not rule out stone, nor does the
presence rule it in. Black race has traditionally been associated with lower likelihood of kidney
stones although more recent research suggests that there is no correlation (either higher or
lower likelihood) with black race. Hypotension argues against the possibility of a kidney stone
and may suggest a vascular process, hemorrhage, or sepsis. Fever can certainly occur in the
setting of a kidney stone, but it does not significantly increase the likelihood of diagnosis,
especially relative to pyelonephritis or sepsis. An abnormal Rovsing sign (eliciting pain in the
right lower quadrant when the left lower quadrant is palpated) is sometimes seen in patients
with acute appendicitis. Abdominal and even flank tenderness are usually absent in patients
with acute ureterolithiasis.
32. A 72-year-old female presents to the ED with asymptomatic acute kidney injury. Her
creatinine is 1.9 mg/dL as measured at her primary care physician’s office. She has a past
medical history of hypertension, osteoarthritis, and neuropathy. Which of her medications is
most likely to blame?
A. Amlodipine
B. Metoprolol
pg. 315
C. Ibuprofen
D. Hydrocodone
E. Gabapentin
pg. 316
Rheumatology emergency
➢ Chapter includes:
Rheumatoid arthritis
Temporal arthritis
Osteoarthritis
Gout and pseudogout
Rheumatology-infectious related emergency
Septic arthritis
Reactive arthritis
Fibromyalgia
pg. 315
1. Which of the following is true regarding gonococcal septic arthritis?
2. A 23-year-old female presents with 2 days of fever and severe right ankle pain and swelling.
She denies a history of trauma. Past medical history is unremarkable. Physical examination
reveals significant edema, effusion, tenderness, and pain on range of motion in the right ankle.
Which of the following is the most likely etiology?
A. S. aureus
B. S. pneumoniae
C. S. pyogenes
D. Salmonella
E. N. gonorrhoeae
Answer E. The patient has evidence of acute monoarticular arthritis. In a young, sexually active
patient without prior history of arthritis, the most likely bacterial cause is gonococcus. The
overall most common cause of septic arthritis is S. aureus. Choices B, C, and D are less common
causes. Septic arthritis is a joint-threatening infection diagnosed by synovial fluid analysis. It
must be aggressively treated with intravenous antibiotics and possible surgical irrigation, even
though gonococcal arthritis rarely requires surgical management.
pg. 316
3. A 64-year-old female presents with a 3-week history of a constant, moderately intense, dull
right sided frontotemporal headache. She also complains of occasional jaw pain as well as
muscle aches and weakness in her shoulders. She denies any visual complaints, fever, or
vomiting. She has a normal eye and neurologic examination. A non-contrast CT brain is normal.
Which of the following is the most appropriate next step?
D. Prescribe carbamazepine
Answer C. There is strong clinical evidence of temporal arteritis, with the unilateral
frontotemporal headache and jaw claudication in a late middle-aged patient. The shoulder
muscle pain and weakness are symptoms of polymyalgia rheumatica, which occurs
concomitantly in as many as half of all cases of temporal arteritis. Diagnosis is suspected with
elevated ESR or CRP. CRP is more sensitive than erythrocyte sedimentation rate (ESR) (97% vs.
86%) and does not increase with normal aging like ESR does. Confirmation can be made with
temporal artery biopsy. The major dangerous complication of temporal arteritis is blindness
from ophthalmic artery inflammation. Treatment for known or suspected cases of temporal
arteritis is with parenteral steroids. MRI for evaluation of subacute headache with a normal
head CT does not generally yield an emergent diagnosis. Lumbar puncture is a reasonable choice
here but does not account for the more emergent diagnosis, which is temporal arteritis.
Carbamazepine is first-line treatment for trigeminal neuralgia, which is on the differential
diagnosis of unilateral headache and jaw pain. However, the pain from trigeminal neuralgia
tends to be episodic, lancinating pain rather than constant, dull pain. To this author’s knowledge
(and moderate consternation), there is no such thing as the migraine vasodilation stimulation
test.
4. A 42-year-old male presents with a swollen, erythematous, and tender left knee. He is in a
long-term monogamous relationship. A concomitant history of recurrent renal stones in this
man suggests a diagnosis of:
A. Reactive arthritis
C. Rheumatoid arthritis
D. Gout
pg. 317
E. Nongonococcal septic arthritis
Answer D. Ten percent to 25% of patients with gout have renal stones, and the rate correlates
with the degree of hyperuricemia. For example, >50% of patients with a serum uric acid level
>13 mg/dL have stones. Reactive arthritis is the name, which is now given to arthritis, urethritis,
and conjunctivitis that occurs after an infection (thus, “reactive”) and which was formerly called
Reiter syndrome.
5. A 25-year-old previously healthy black woman presents to the ED complaining of a facial rash
over her nose, mild fever, and achy wrists. She just returned from an annual weekend trip to the
Florida beaches with a bunch of girlfriends. Which of the following is the most sensitive test to
aid in her diagnosis?
Answer E. This patient has features of systemic lupus erythematosus (SLE). Black women of
childbearing age are most at risk for developing SLE. This patient is presenting with the classic
malar rash after sun exposure in concert with fever and arthralgias. As in rheumatoid arthritis,
arthralgias in SLE are typically symmetric and most commonly involve in the fingers, hands,
wrists, and knees. Fevers are very common in patients with SLE, and nearly all patients will
develop a fever at some point in their course, although greater than one-third will present with
a fever. Patients with SLE have numerous autoantibodies. However, testing for ANA is most
sensitive as 99% of patients with SLE have a positive ANA. The positive predictive value of ANA
testing suffers because 5% to 7% of healthy individuals will also test positive for ANA. Anti-Sm is
the most specific antibody, with a specificity of 99% and a positive predictive value of 97%.
However, its sensitivity is only 25% so it is not a good screening test.
A. Hip
B. Knee
C. Ankle
D. Wrist
E. Shoulder
pg. 318
Answer B. The most commonly affected joints in septic arthritis are knee (40% to 50%), hip (13%
to 20%), shoulder (10% to 15%), ankle (6% to 8%), wrist (5% to 8%), and elbow (3% to 7%).
7. A 64-year-old male presents to the ED with several weeks of heel pain. He used to have pain
only with heavy exercise, but gradually it has started limiting walking as well. It even occurs in
the morning when he wakes up and walks for the first time. He decided not to inform his
primary care doctor about this pain and came to the ED for evaluation. He has mild tenderness
to palpation on the proximal plantar heel but an otherwise unremarkable physical examination.
Radiographs are normal. Which of the following is the most appropriate next step in
management?
Answer B. The patient likely has plantar fasciitis, which is inflammation of the plantar surface of
the foot, just distal to the calcaneus. Patients complain of moderate–severe heel pain on
walking. Pain can be worse in the morning, when feet are allowed to remain in plantar flexion
while sleeping. Treatment involves physical therapy exercises to stretch the Achilles tendon with
dorsiflexion splints. Nonsteroidal anti-inflammatory drugs (NSAIDs) can also be employed for
pain control. Corticosteroid injections are generally not recommended. Stirrup ankle braces are
more useful for patients with ankle sprains to prevent inversion and eversion and do not help
treat a pathologic process that involves plantarflexion and dorsiflexion. Admission for MRI or
surgical consultation in this otherwise healthy patient with no signs of acute emergency
requiring operative evaluation is not advisable. Optimal management in this case also involves
encouragement for outpatient follow-up.
8. Physical examination of a patient with reactive arthritis (formerly known as Reiter syndrome)
may be expected to reveal:
D. Conjunctival injection
pg. 319
E. All of the above
Answer E. Reactive arthritis is the name which is now given to arthritis, urethritis, and
conjunctivitis that occurs after an infection (thus, “reactive”) and which was formerly called
Reiter syndrome. Patients with reactive arthritis frequently have conjunctivitis early in the
disease course. Uveitis (or iritis) may also occur but is less common and unrelated. Roughly 10%
of patients will develop keratoderma blennorrhagica, waxy plaques most commonly present on
the palms and soles. Sausage shaped inflammation of the digits (dactylitis) is another common
occurrence in reactive arthritis. Finally, painless ulcers may develop in the mouth or on the penis
(where they are called balanitis circinata), where they more frequently occur in uncircumcised
men.
9. The synovial white blood cell (WBC) count in cases of septic arthritis usually exceeds at least:
Answer D. In septic arthritis, the synovial WBC usually exceeds 50,000 WBC per mm3 and often
exceeds 100,000 per mm3, with >75% polymorphonuclear cells. However, patients with gout,
pseudogout, and rheumatoid arthritis may also have WBC counts in this range with a similar
differential. Therefore, when the WBC is elevated approximately 50,000 per mm3, septic
arthritis must be presumed until ruled out.
C. A cardiac rub in the setting of arthralgias suggests systemic lupus erythematosus (SLE).
Answer C. Viscosity decreases with any inflammatory process of the joint because of decreased
hyaluronic acid, which is the main contributor to synovial fluid viscosity. RA classically affects the
pg. 320
metaphalangeal and PIP joints of the hand, whereas OA affects the first carpometacarpal joint
as well as the PIP and distal interphalangeal (DIP) joints. SLE may cause inflammation of serosal
surfaces such as the pleura or pericardium. Pericarditis in a patient with SLE may result in an
audible cardiac friction rub. Reiter syndrome may cause sausage-shaped swelling of the digits.
An abducted, externally rotated hip in a neonate suggests infection, even in patients who are
afebrile.
A. Rheumatoid arthritis
B. Osteoarthritis
D. Reactive arthritis
E. Psoriatic arthritis
Answer A. This is the swan neck deformity. It is caused by hyperextension at the proximal
interphalangeal (PIP) joint and flexion at the distal interphalangeal (DIP) joint. The DIP joint
flexion occurs due to elongation or rupture of the extensor tendon attachment to the distal
phalanx (i.e., similar to a mallet injury). Left untreated, PIP hyperextension occurs as a
consequence of the distal mallet deformity. However, the deformity can begin in the PIP joint as
well due to synovitis of the volar capsule resulting in PIP hyperextension. In the latter case, DIP
flexion occurs as a secondary effect. Both the swan neck and boutonniere deformities are
common in rheumatoid arthritis.
A. Staphylococcus aureus
pg. 321
B. Neisseria gonorrhoeae
C. Streptococcus pyogenes
D. Hemophilus influenza
E. Polymicrobial
Answer A. S. aureus remains the most common cause of septic arthritis, accounting for 37% to
56% of infections. It causes 80% of the infections in patients with underlying diabetes or
rheumatoid arthritis. Group A β-hemolytic streptococci are the second most common cause of
septic arthritis. The remainder is caused by other streptococci, gram-negative organisms, and
gonococcus. N. gonorrhoeae accounts for only 20% of monoarticular septic arthritis, although it
accounts for a larger proportion of polyarticular septic arthritis, which is its usual presentation.
Gram-negative organisms are the most common cause of septic arthritis in newborns and in
children younger than 5 years old, although vaccination programs have nearly eliminated H.
influenzae as a cause.
13. A 41-year-old female with fibromyalgia syndrome presents with an acute flare of her pain.
Her only daily medication is gabapentin, and she takes acetaminophen as needed for pain. Her
vital signs are unremarkable. Which of the following is the most appropriate therapy to add to
acetaminophen for pain?
A. Oxycodone
B. Tramadol
C. Aspirin
D. Ibuprofen
E. Prednisone
pg. 322
14. Which of the following patients most likely has giant cell arteritis?
A. An 18-year-old male with chronic daily headaches and a family history of lupus
B. A 55-year-old female with diabetes without vision complaints referred by her optometrist for
evaluation of a dilated right pupil
E. A 37-year-old male with a parietotemporal headache and a family history of berry aneurysms
Answer D. Giant cell arteritis (GCA, formerly called temporal arteritis) is a vasculitis of large- and
medium-sized vessels that occurs almost exclusively in adults older than 50. That fact alone
rules out three of the patients in this question. Though the disease is systemic, it most
commonly produces symptoms when it involves the cranial branches of arteries originating from
the aortic arch. Temporal arteritis, which may lead to vision loss, is one of the most classic and
dangerous manifestations. Symptoms that increase the likelihood of GCA include new
headaches, jaw claudication (jaw pain after chewing), new visual changes, an elevated
erythrocyte sedimentation rate (the ESR increases with age, but the increase is actually quite
modest), unexplained fever, and symptoms of polymyalgia rheumatica. Though the 20-year-old
female has temporal headache and an elevated ESR, an elevated ESR is nonspecific with a broad
differential and the patient’s age makes GCA very unlikely. The 74- year-old female has classic
jaw claudication and fits the epidemiologic profile. The 55-year-old female likely has
microvascular ischemia affecting cranial nerve III, resulting in relative weakness of the pupillary
constrictors. This is common in diabetics and most often resolves without treatment over
several months.
A. Due to hyperparathyroidism
B. Due to hemochromatosis
C. Due to hypomagnesemia
D. Due to hypothyroidism
E. Idiopathic
pg. 323
16. A 38-year-old female presents to the ED with a complaint of extreme hand and finger pain,
which she says is exacerbated any time her fingers are exposed to the cold. She first noticed the
problem when reaching into the freezer to grab a frozen dinner. She notes that her fingers
become “ghost white” or blue at the tips and have a painful ache. Which of the following is the
most common underlying disorder that produces these symptoms?
C. Scleroderma
Answer C. Raynaud phenomenon is nearly universal in patients with scleroderma (also known as
systemic sclerosis) and is the earliest sign of the disease. Raynaud phenomenon is also common
in patients with lupus and RA. The exact mechanism of Raynaud phenomenon in scleroderma
(or in other autoimmune diseases) is not known. Raynaud phenomenon may also be a primary
problem (in which case it is sometimes referred to as Raynaud disease), rather than secondary
to an underlying disease. To be considered a primary process, patients need to suffer no
ischemic damage in the affected digits (e.g., gangrene, necrosis), have negative serology (e.g.,
particularly for antinuclear antibodies), have a normal erythrocyte sedimentation rate, have
symmetric attacks, and lack physical examination findings which suggest a secondary cause.
17. A 49-year-old female presents with chronic intermittent myalgias for several months. She
discussed the diagnosis of fibromyalgia syndrome with her primary care physician and was going
to be sent for evaluation with a rheumatologist when her pain suddenly became worse. Her vital
signs are normal. Which of the following is most consistent with a diagnosis of fibromyalgia
syndrome?
D. Jaw claudication
E. Fatigue
pg. 324
lower thresholds of somatic stimuli. Over 90% of patients with fibromyalgia exhibit fatigue and
over 50% carry a concomitant diagnosis of mood disorder. Sleep is frequently affected as well.
Elevations in CPK are found in diagnoses such as polymyositis, steroid myopathy, and
rhabdomyolysis but are absent in fibromyalgia. ESR and CRP elevations are seen in polymyalgia
rheumatica and other inflammatory diseases, not fibromyalgia syndrome. Jaw claudication is
frequently seen in temporal arteritis, which can occur concomitantly with polymyalgia
rheumatica, not fibromyalgia syndrome.
18. Which of the following is the most effective treatment for fibromyalgia syndrome?
A. Prednisone
B. IV immunoglobulin (IVIG)
C. Aerobic exercise
D. Acupuncture
E. Physical therapy
Answer C. Aerobic exercise is the most effective therapy for treatment of fibromyalgia syndrome
at this time. Resistance exercise may also be effective and is currently being studied.
Fibromyalgia syndrome is a noninflammatory condition of chronic hypersensitivity of muscles in
response to painful stimuli. Patients with fibromyalgia syndrome experience pain at lower
thresholds of somatic stimuli. Over 90% of patients with fibromyalgia exhibit fatigue and over
50% carry a concomitant diagnosis of mood disorder. Sleep is frequently affected as well.
Prednisone and IVIG do not improve fibromyalgia syndrome, as there is no inflammatory
component to target. Acupuncture and physical therapy have been shown to be ineffective in
the majority of patients.
Answer C. Gout occurs due to uric acid crystal accumulation in the joints. Mechanisms include
uric acid overproduction and undersecretion as well as exogenous precursor ingestion of dietary
pg. 325
purines converted to uric acid by xanthine oxidase. High purine foods include mussels, organ
meats, salty fish (sardines, herrings, etc.), alcohol, and poultry. Pseudogout occurs due to
synovial deposition of calcium pyrophosphate crystals. There are no dietary modifications that
will aid in preventing pseudogout flares. Although the most common joint affected in gout is the
first metatarsophalangeal, the knee and the ankle can also be affected. The knee is the number
one joint to be affected in pseudogout, followed by the wrist and the ankle. ESR is elevated in
most acute attacks of both diseases. NSAID therapy is first-line for acute attacks of both.
20. A patient presents for evaluation 2 weeks after recovering from a sore throat. Which of the
following findings is most suggestive of rheumatic fever?
A. Fever
B. Sore throat
D. Sinus tachycardia
E. Choreiform movements
Answer E. Rheumatic fever occurs several weeks after untreated streptococcal pharyngitis. The
diagnosis is made by the Jones criteria: either two major (polyarthritis, erythema marginatum,
chorea, carditis, subcutaneous nodules) or one major and two minor (arthralgias, fever,
increased erythrocyte sedimentation rate or C-reactive protein, prolonged PR interval).
Migratory arthritis of major joints is the most common symptom, followed by carditis. Chorea
and erythema marginatum are uncommon but fairly specific given a history of antecedent
pharyngitis. Despite its name, fever is not common in patients with rheumatic fever.
21. A 55-year-old female with a history of rheumatoid arthritis presents with progressive
swelling and pain in her knee for 6 days. She denies trauma to the area or fever. She is on
prednisone for her rheumatoid arthritis, and states that her standard flares involve her ankles
and fingers. Vital signs are 99.0°F, 100, 20, 132/65, 98% RA. Physical examination reveals a
moderate-sized knee effusion with warmth and tenderness and extreme pain on range of
motion of the joint. Which of the following is the most appropriate next step in management?
A. Joint aspiration
C. Colchicine PO
D. Stress-dose steroids
E. Indomethacin PO
pg. 326
Answer A. Although the patient has a history of rheumatoid arthritis, which may be responsible
for the symptoms, it is crucial to exclude septic arthritis by aspiration of the joint and synovial
fluid analysis. Rheumatoid arthritis, like any process that causes joint destruction, predisposes
patients to developing septic arthritis. MRI is not useful in the acute setting and will
unnecessarily delay appropriate management. Colchicine and indomethacin are used for acute
gouty flares and without a clear diagnosis of crystals in the synovial fluid, neither is indicated.
Stress-dose steroids may be reasonable in this patient, but should not precede evaluation for a
septic joint.
22. A 20-year-old male who has recently recovered from gastroenteritis due to Shigella flexneri
is most at risk for developing which of the following?
A. Ankylosing spondylitis
C. Rheumatoid arthritis
D. Psoriatic arthritis
Answer E. Reactive arthritis is the name which is now given to arthritis, urethritis, and
conjunctivitis that occurs after an infection (thus, “reactive”) and which was formerly called
Reiter syndrome. It is a reactive arthritis that occurs following C. trachomatis infection of the
genitourinary tract or Shigella, Salmonella, Campylobacter, or Yersinia infection of the
gastrointestinal (GI) tract. It is part of a group of arthritides known as the seronegative
spondyloarthropathies. This group includes ankylosing spondylitis, psoriatic arthritis, reactive
arthritis, and the arthropathy of inflammatory bowel disease. They are grouped because of their
common involvement of the sacroiliac joint, lack of rheumatoid factor, and presence of the
HLA-B27 genetic marker. Reiter syndrome is most common in young men aged 15 to 35 and
occurs 2 to 6 weeks after an episode of urethritis or dysentery. The classic triad is arthritis,
urethritis, and conjunctivitis. The arthropathy in reactive arthritis is an enthesopathy, which
refers to pathology at the site of ligament or tendon insertion to bone. It most commonly
involves the lower extremities, particularly the Achilles tendon (“lover’s heel”).
23. Which of the following is true regarding patients with temporal (or giant cell) arteritis?
pg. 327
D. Vertigo is the most sensitive clinical finding.
Answer A. Temporal arteritis (or giant cell arteritis) is a large vessel vasculitis that primarily
affects branches of the carotid artery. The disease is rare before the age of 50, and incidence
peaks in the seventh decade. The most sensitive finding is a new headache, whereas the most
specific finding is jaw claudication. However, the disease often presents vaguely with systemic
symptoms such as a fever of unknown origin, fatigue, malaise, and anorexia. The diagnosis
should be considered in any older person with fever of unknown origin. Scalp pain is a more
specific finding than headache, particularly when localized over the temporal artery. Vertigo is
not a manifestation of the disease. Permanent visual loss occurs in only 15% of patients and is
usually preceded by the development of blurring, diplopia, or amaurosis fugax. Corticosteroids
are the treatment of choice and should never be withheld if the diagnosis is seriously
considered. Multiple studies have demonstrated that treatment with steroids does not affect
the accuracy of biopsy if performed within a few weeks.
Answer C. The DIP joints are never affected in RA, which provides a useful means of
differentiating the disease from osteoarthritis. The arthritis of RA is typically polyarticular and
symmetric, particularly affecting the hands (metacarpophalangeal and proximal interphalangeal
joints), wrists, and elbows. The disease is twice as common in women and peaks in the fourth to
sixth decade. Two thirds of patients with RA develop cervical spine disease, although thoracic
and lumbar disease is uncommon. The disease most commonly involves the occipitoatlantoaxial
junction, and anterior atlantoaxial subluxation may occur. RFs are autoantibodies directed at the
crystallizable fragment (Fc) of human immunoglobulin molecules. The exact incidence of RF
depends on the assay used and the threshold titer used to separate positive from negative
results. In general, roughly 15% of patients with RA will be seronegative (RF within the normal
range), and those patients tend to have milder disease.
25. Which of the following is the most common symptom of rheumatic fever?
pg. 328
A. Chorea
B. Carditis
C. Erythema marginatum
D. Migratory polyarthritis
E. Fever
Answer D. Rheumatic fever occurs several weeks after untreated streptococcal pharyngitis. The
diagnosis is made by the Jones criteria—either two major (polyarthritis, erythema marginatum,
chorea, carditis, subcutaneous nodules) or one major and two minor (arthralgias, fever,
increased erythrocyte sedimentation rate (ESR) or C-reactive protein (CRP), prolonged PR
interval). Migratory arthritis of major joints is the most common symptom, followed by carditis.
Chorea and erythema marginatum are uncommon but fairly specific given a history of
antecedent pharyngitis. Fever is neither sensitive nor specific.
26. A 70-year-old female presents with blurry vision in her right eye and a right-sided headache.
The patient also complains of malaise, and is tender in her right temporal region. Which of the
following is true regarding this patient’s illness?
Answer B. The patient has temporal arteritis. Steroids are the mainstay of management and
should be initiated even before temporal artery biopsy is done. Once significant visual loss has
occurred, only one out of three patients who receive IV steroids will improve in the affected eye.
However, steroids will nearly eliminate contralateral eye involvement. Patients have a
significantly higher risk of thoracic aorta aneurysms, and a slightly higher risk of abdominal aorta
aneurysms. Women are five times more likely to be affected than men. Patients younger than
50 years are rarely affected.
27. A 59-year-old male with a history of diabetes and gout presents for evaluation of a 2-day
history of worsening left knee pain and swelling without fever. He is unable to put any weight on
it or bend it. Vital signs are 99.4°F, 100, 18, 161/80, 99% RA. Examination reveals a warm, tender
pg. 329
knee with a moderate effusion and extreme pain on flexion and extension. Knee radiographs
reveal an effusion but no fracture. Which of the following is the next best step in management?
C. Knee arthrocentesis
D. MRI knee
Answer C. The patient has an inflammatory, monoarticular arthritis. The differential diagnosis
includes septic arthritis, gout, and rheumatoid arthritis. Clinical differentiation between these
three entities can be very difficult. Fever is present in most but not all patients with septic
arthritis. Only 50% of patients with septic arthritis will have an abnormal blood WBC count, and
only 50% will have positive blood cultures. The diagnosis rests on knee arthrocentesis and
synovial fluid analysis. Fluid indices suggestive of septic arthritis include elevated fluid WBC
count (>5,000 cells per mm3), neutrophilic predominance (>75%), decreased glucose, positive
Gram stain, and negative crystal assessment. It is important to recognize that only about
two-thirds of patients with septic arthritis will have a positive Gram stain. Patients with acute
monoarticular arthritis without a pre-existing condition of rheumatoid arthritis or gout should
never be discharged without evaluation for septic arthritis. MRI of the knee should rarely, if
ever, be performed in the ED for any indication. Serum uric acid levels are unreliable markers for
acute gouty arthritis and should never be used to rule out septic arthritis.
28. Which of the following correctly matches the vasculitic syndrome to its primary clinical
manifestations?
Answer A. The vasculitic syndromes have multiple areas of overlap in their clinical
manifestations and it is sometimes difficult for rheumatologists to apply a specific diagnosis.
However, classically, PAN causes mononeuritis multiplex and mesenteric ischemia. Cutaneous
findings are also common. Takayasu arteritis is very common in Japan and results in coronary
ischemia. Wegener granulomatosis initially presents with symptoms of upper airway problems
pg. 330
such as sinusitis, otitis, and nasal congestion while developing glomerulonephritis at a later
stage. Behçet disease is characterized by recurrent oral and genital ulcerations and recurrent
hypopyon (it is rarely seen, but pathognomonic finding). Churg–Strauss syndrome involves the
lungs and most patients have symptoms of asthma in the 2 years preceding a diagnosis.
pg. 331
Neurology and neurosurgery
emergency
➢ Chapter includes:
CVA and weakness
Brain hemorrhage
Peripheral and central neurology and vertigo
SLE
MS
Migraine headache
Isolated nerve emergency
Exception (traumatic neurosurgery in trauma chapter )
pg. 332
1. A 33-year-old female presents with numbness and weakness in the right side of her face for
several days as shown below. The remainder of her examination is normal and she has no other
symptoms. Which of the following is the next best step in management?
A. Valacyclovir
B. Prednisone
D. Sour candy
E. Amoxicillin–clavulanic acid
Answer B. With unilateral upper and lower facial weakness and the absence of other concerning
neurologic features, the diagnosis is very likely idiopathic facial nerve palsy, also known as Bell
palsy. Steroid therapy (prednisone 60 mg daily for 1 week) has been shown to improve
outcomes in Bell palsy, but antiviral therapy in the absence of steroids appears to have no
effect. Tissue plasminogen activator would be indicated for an acute ischemic stroke within 3
hours of presentation, but would not be appropriate in this case. Sour candy can be used to
treat sialolithiasis, but there is no observable swelling to suggest this. Antibiotics are not
indicated for the management of Bell palsy.
pg. 333
Answer D. The fact that cooling improves the symptoms of MG is the basis for the “ice test.”
When ptosis is present, an ice pack is placed over the affected eye for 2 minutes. It is thought
that the local cooling results in a slowing of the kinetics of the acetylcholine receptor, allowing
for a prolonged effect of acetylcholine and an improvement in symptoms. In clinical studies,
80% of patients with ptosis due to MG experienced some improvement with a locally applied ice
pack. MG has a bimodal peak of incidence with the first peak in the 20s and the second peak in
the 50s. Interestingly, women are more commonly affected during the first peak, but men are
more commonly affected in the second peak. Sensory loss is not a feature of MG. The most
frequent initial symptoms of MG are ptosis and diplopia due to ocular muscular weakness or
weakness of the levator palpebrae superioris.
3. A 78-year-old male presents with sudden onset of right-sided arm and leg weakness. He was
brought in by private car by his son, who states that the patient was totally normal 20 minutes
prior to arrival. You immediately recognize the potential for acute stroke and initiate your stroke
protocol, which involves immediate noncontrast CT brain. His blood pressure is 169/95. As the
CT scan is being cleared, performance of which of the following diagnostic maneuvers is of
paramount importance?
A. EKG
B. Temperature
C. Blood glucose
D. Prolactin
E. PO challenge
Answer C. Patients with acute symptoms of stroke should be immediately evaluated for
hypoglycemia. Hypoglycemia can mimic acute stroke and the management of this condition is
radically different from acute ischemic or hemorrhagic stroke. Typically, blood glucose checking
is performed by EMS, but patients who are brought in by other means do not have access to
this. Obtaining an EKG is important, but it will not change the hyperacute management of
potential stroke, even if it shows atrial fibrillation, which could be the etiology of the stroke.
Temperature is certainly important as well, but should not delay checking of blood glucose in
this particular patient as targeted treatment of an elevated temperature with focal neurologic
findings cannot occur immediately. Prolactin levels were used for evaluation of seizure activity,
but inadequate sensitivity and specificity have caused it to fall out of favor. Oral challenge
should never be pursued in patients with possibility for acute stroke as they have a high risk of
pharyngeal compromise and can aspirate.
4. A 65-year-old male presents with acute onset of back pain and bilateral leg weakness after a
recent diagnosis of prostate cancer. Physical examination demonstrates 3/5 strength in both of
pg. 334
his lower extremities and tenderness to palpation of his lower back. An emergent MRI
demonstrates epidural lumbar spinal cord compression secondary to metastasis. Which of the
following is the most appropriate initial consultation?
A. Radiation oncology
B. General surgery
C. Urology
D. Neurology
E. Neurosurgery
Answer A. Acute spinal cord compression due to vertebral column metastasis occurs with many
cancers, including lung, breast, and prostate. Patients present with typical findings of epidural
compression, including pain, weakness, or bowel/bladder dysfunction. Any patient suspected of
having metastatic epidural compression should have an emergent MRI of the spine to evaluate
the symptoms. Rapid diagnosis and management is essential to prevent irreversible neurologic
sequelae. Corticosteroids may be started in the ED to reduce edema of the spinal cord.
Radiation and spinal surgery are the primary treatments of malignancy-related epidural cord
compression. Other specialties need not be emergently consulted in these cases.
5. A 78-year-old male presents with marked left foot weakness and hypoesthesia. In addition,
his family states that he is not acting himself and seems to be having difficulty making decisions.
Which of the following arteries is most likely affected?
A. Vertebrobasilar artery
Answer D. Due to the anterior communicating artery, lesions of the anterior cerebral artery
proximal to the communicating artery are generally well tolerated. Lesions distal to this
anastomosis result predominantly in leg weakness and sensory loss as well as a variety of
personality and behavioral changes. These changes include abulia, which is the inability of
patients to make decisions. The upper extremities may be involved but are typically only mildly
affected. Furthermore, the deficits are usually most marked distally. The tongue and the face
are generally spared.
pg. 335
6. A 29-year-old female with a history of multiple sclerosis (MS) presents to the ED with a chief
complaint of a 2-day history of right arm weakness and clumsiness. She reports that these
symptoms are similar to a past “MS flare.” Her examination reveals proximal and distal right arm
weakness but no other findings. Which of the following is the best next step?
A. CT to exclude a stroke
C. Neurology consult
E. 1,000 mg IV methylprednisolone
Answer E. Multiple sclerosis (MS) is the most common autoimmune inflammatory demyelinating
disease of the central nervous system. In patients with symptoms and findings of an acute
episode of demyelination, steroids remain the mainstay of treatment. While a 5-day course of
1,000 mg IV methylprednisolone is the most common therapy given, oral prednisone regimens
are equally effective, except in patients with optic neuritis. One trial demonstrated higher
relapse rates in patients with optic neuritis who were given an oral prednisone regimen versus
the group given IV methylprednisolone. Plasma exchange is reserved for patients who don’t
respond to steroids. Finally, since this patient has known MS, and since her symptoms and
findings are suggestive of MS, there is no indication for neuroimaging.
7. A 25-year-old female presents with acute onset of a severe occipital headache. CT reveals a
subarachnoid hemorrhage (SAH). Which of the following clinical findings is likely present?
C. Seizure
D. Nausea
E. Intraocular hemorrhage
Answer D. Nausea and vomiting are common in patients with SAH, occurring in about
three-quarters of all cases. Unfortunately, many other headache syndromes will also cause
nausea and vomiting, so they are not specific for SAH. About half of all patients with SAH have
alteration of mental status, which is the next most common associated sign/symptom. Focal
neurologic deficit, preceding exertional activity, seizure, and intraocular hemorrhage each are
present in less than a quarter of all cases. Preceding exertional activity does confer a roughly
threefold increased likelihood that the headache is due to SAH, but sensitivity of this finding is
low.
pg. 336
8. Which of the following is true regarding the management and prognosis of trigeminal
neuralgia?
B. Antiviral medications directed at herpes and corticosteroids have been shown to reduce the
duration of pain and prevent recurrence.
C. In addition to medical treatment, all patients should be referred to a neurologist for further
evaluation by MRI.
D. Fifty percent of patients will eventually require neurosurgical ablation of the trigeminal nerve.
E. All patients should be loaded with phenytoin and prescribed an outpatient regimen.
Answer C. Spontaneous remission is the rule in trigeminal neuralgia as >50% of patients will
experience a remission for 6 months. Antiviral medications directed at herpes and
corticosteroids should be used for patients with postherpetic neuralgia. This is a separate entity
from trigeminal neuralgia and patients should not be placed on antivirals unless they have a
history of herpes zoster (shingles) involving the face. All patients with trigeminal neuralgia
should be referred to a neurologist for further evaluation. Up to 2% to 4% of patients with
trigeminal neuralgia also have multiple sclerosis and up to 10% of patients have intracranial
lesion. Therefore, all such patients should receive an MRI on an outpatient basis. Unfortunately,
roughly 30% of patients will fail medical therapy and require surgical ablation. Phenytoin is not
indicated for trigeminal neuralgia. Carbamazepine is the standard front-line agent and is started
at 100 to 200 mg b.i.d. Although carbamazepine is sometimes poorly tolerated, it produces a
significant positive response in most patients with classic trigeminal neuralgia.
C. Oculomotor nerve palsy is the most common associated cranial nerve palsy.
pg. 337
disease, there can be no evidence of a mass or a structural or vascular lesion on neuroimaging.
Furthermore, the composition of the cerebrospinal fluid must be normal, and any symptoms
resulting from the disease must be completely attributable to papilledema or generalized
elevated intracranial pressure (ICP). The most common presenting symptom is headache, which
tends to be worse in the recumbent position and in the morning (after prolonged recumbency
overnight). The most common serious complication is permanent vision loss. CT scan never
reveals hydrocephalus. Abducens nerve palsy is the only cranial nerve palsy that commonly
occurs and typically presents as intermittent or constant lateral binocular diplopia. Women are
more commonly affected, with a 19 times increased incidence in obese women of childbearing
age.
10. Which of the following is true regarding transient ischemic attack (TIA)?
A. Neurologic findings in patients with TIAs are more commonly “positive” (tingling or
involuntary movements) than “negative” (aphasia, weakness, numbness).
C. TIA was recently redefined as transient neurologic dysfunction that resolves within 1 hour.
Answer C. In 2002, the Transient Ischemic Attack Working Group redefined TIA as “a brief
episode of neurologic dysfunction caused by focal brain or retinal ischemia, with clinical
symptoms typically lasting less than 1 hour, and without evidence of acute infarction.” In 2009,
the American Heart Association and American Stroke Association endorsed similar guidelines
that emphasized that the diagnosis can only be considered in patients without evidence of
central nervous system infarction. Positive findings such as tingling or involuntary movements
are the exception in patients with a TIA. In addition, symptoms affecting multiple different body
parts usually occur simultaneously. “Marching” symptoms are more common in patients with
migraines or seizures. Nonfocal symptoms such as generalized weakness, dizziness,
lightheadedness, and confusion are not commonly due to a TIA. Finally, the most common
mimic of symptoms attributable to a TIA is hypoglycemia.
11. A 55-year-old male presents with a seizure. He has never had a seizure before and
developed tonic– clonic movements for 1 minute followed by a postictal confusional state
lasting about 10 minutes. Physical examination is normal, and the patient is amnestic to the
event. Blood glucose is normal. Which of the following is the most appropriate next step in
management?
A. MRI brain
pg. 338
B. CT brain
D. Lumbar puncture
E. Prolactin level
Answer B. Patients presenting to the ED with a first-time generalized seizure should receive a CT
brain to evaluate for intracranial hemorrhage or mass lesion. MRI should be reserved for those
patients who do exhibit a mass lesion on CT that requires further characterization. Starting
antiepileptic therapy in patients with first-time seizures is established on a case-by-case basis
with neurology consultation. Regardless, phenobarbital is almost never the initial therapy.
Lumbar puncture may be pursued if an infectious source is suspected, but should always be
preceded by a CT to exclude mass lesion in the setting of seizure. Prolactin levels were initially
touted as a way to definitively diagnose a generalized seizure, but later research has shown an
inability to distinguish accurately between seizure and syncope.
12. A 52-year-old male with a longstanding history of diabetes and hypertension presents for
evaluation of vertigo. A head thrust maneuver (head impulse test) is abnormal when his head is
turned to the right. There is no direction-changing nystagmus. There is no skew deviation.
Which of the following is true?
Answer A. The combination of the head impulse test (head thrust maneuver), evaluation of
direction changing nystagmus, and evaluation of eye skew deviation comprise the HINTS (head
impulse– nystagmus–test of skew) test to determine whether vertigo is due to a central or
peripheral lesion. Of specific concern is differentiating peripheral vertigo from a cerebellar
stroke. In the head impulse test, the patient fixates his gaze on a distant object while the
physician rotates the patient’s head from side to side before randomly “thrusting it” rapidly
back to the midline. In an abnormal test, the patient is unable to maintain fixation during the
thrust and the eyes slowly march back to the midline in a saccadic fashion as they re-fixate on
the object. A positive test indicates a problem in the vestibuloocular reflex (VOR) and a
peripheral lesion. In patients with peripheral lesions, the fast phase of nystagmus will always
occur in the same direction regardless of whether the patient is looking left or right. It is easier
to detect if the patient does not fixate on an object. In central lesions the direction of nystagmus
pg. 339
may (or may not) change depending on the direction of gaze. In testing for skew deviation, the
physician asks the patient to fixate on an object while covering and then uncovering and eye
while examining the eye for vertical movement, which is abnormal, after it is uncovered. A
normal head impulse test, or the presence of direction-changing nystagmus or skew deviation
all make a brainstem or cerebellar lesion more likely and MRI imaging should be pursued.
13. A 47-year-old female presents with a severe occipital headache and a general feeling of
malaise. Her neurologic examination is normal. A stat head CT is shown in the image (see Fig.
below). Which of the following is true about this patient?
B. The patient should be given stat IV corticosteroids and loaded with an anticonvulsant.
pg. 340
awake patients with relatively small infarcts (<3 cm), patients may be candidates for observation
in an intensive care setting. All patients with cerebellar hemorrhage, however, may deteriorate
rapidly due to hydrocephalus or progressive brainstem compression. Due to the local mass
effect, the fourth ventricle may become compressed, resulting in an obstructive hydrocephalus.
This requires emergent ventricular drainage. Due to the possibility of rapid deterioration, all
patients with cerebellar infarction should be admitted to an intensive care setting.
Corticosteroids and anticonvulsants have no role in the management of cerebellar hemorrhage.
Corticosteroids help to reduce the vasogenic edema associated with tumors, but not the
cytotoxic edema associated with infarction. Prophylactic anticonvulsants have not proved to be
useful, and because seizures are initiated in the cortex, cerebellar lesions should not trigger
epileptic events. Herniation may occur in these patients, but results in upward transtentorial
herniation, not uncal herniation. Finally, the Cushing response may occur, but results in very
high BPs, with systolic pressures in the neighborhood of 200 mm Hg. This degree of
hypertension usually portends a bad outcome.
14. Which of the following is the most common sign or symptom in patients with normal
pressure hydrocephalus (NPH)?
A. Ataxia
B. Dementia
C. Papilledema
D. Urinary incontinence
E. Headache
Answer A. NPH was initially described in 1965 as a triad of difficulties of gait (ataxia), altered
mentation (dementia), and urinary sphincter dysfunction (incontinence) in concert with
enlarged ventricles (hydrocephalus) but apparently normal cerebrospinal fluid (CSF) pressure
upon LP. Recent research has shown that while the CSF pressure may be normal during a single
LP, patients with NPH experience transient increases in CSF pressure which can now be
appreciated in centers capable of performing CSF monitoring. Therefore, many authors have
advocated changing the name of this illness to “chronic hydrocephalus.” Though there is no
“classic” gait, walking difficulties are the most consistent and prominent feature of NPH.
Furthermore, ataxia is most likely to improve after CSF shunting. Alterations in cognition and
mentation may be so mild that they are not noticed by either the patient or the patient’s family
and the term dementia is overly broad generalization. Urinary incontinence is present only in
the later stages although a sensation of urinary urgency is almost always present. Papilledema is
a sign of increased intracranial pressure. Headache is not a typical feature of patients with NPH.
pg. 341
15. A 34-year-old female with a history of seizure disorder presents with a generalized
tonic–clonic seizure. She is on phenytoin for her seizures. Her postictal state has elapsed and she
is alert and oriented now. Vital signs are 99°F, 85, 155/80, 99% RA. Physical examination is
normal. Which of the following is the most likely cause for her seizure?
A. Alcohol use
B. Febrile seizure
C. Medication noncompliance
E. Mass lesion
Answer C. The number one cause of seizures in patients with a history of seizure disorder is
medication noncompliance. Other important causes include alcohol use, alcohol withdrawal,
and sleep deprivation. Febrile seizures are extremely rare in adults. Central nervous system
infection would be unlikely in a patient without fever, abnormal physical examination, or
suggestive historical findings. Mass lesion is often diagnosed during first-time seizures, but
patients with a history of a seizure disorder often already have had adequate neuroimaging to
evaluate this.
16. Which of the following is seen in almost all patients with Guillain–Barre syndrome within the
first week of illness?
A. Albuminocytologic dissociation
B. Muscle weakness
C. Sensory deficits
D. Areflexia
E. Diarrhea
Answer B. Patients with Guillain–Barre syndrome (GBS) almost always have muscle weakness
that begins early in the course of illness (within the first week). Usually, this is extremity muscle
weakness but the Miller Fisher variant of GBS has oculomotor weakness. Hyporeflexia is also
commonly seen early, but true areflexia is not as common. Sensory deficits are also often seen,
but several variants of GBS lack sensory findings completely (i.e., acute motor axonal
neuropathy, Miller Fisher). Diarrhea as part of antecedent Campylobacter infection may be
reported before neurologic symptoms occur, but this is seen in less than half of cases.
pg. 342
17. A 54-year-old female presents with acute onset of dizziness, vertigo, and nausea. You note
nystagmus on her examination. Which of the following characteristic of nystagmus is most likely
to be associated with a central cause?
A. Horizontal
B. Fatigable
C. Direction-changing
D. Intensity-changing
Answer C. Patients with nystagmus that changes direction based on the gaze direction (i.e.,
looking right causes fast-phase of nystagmus to the right, looking left causes fast-phase of
nystagmus to the left) are at high risk for central cause of their nystagmus. Vertical nystagmus
also predicts a central cause. Horizontal, fatiguable, and intensity-changing (beating faster, but
not changing the direction of the fast component, in response to head movements) nystagmus
are all commonly associated with peripheral causes.
18. A 28-year-old female complaining of bilateral left-sided visual field problems most likely has:
C. A pituitary tumor
D. Multiple sclerosis
Answer B. Patients with visual field complaints affecting both eyes must have lesions affecting
either the optic chiasm or retrochiasmal pathways (posterior to or “before” the optic chiasm)
including the optic tracts, lateral geniculate body, the optic radiations, and the occipital cortex.
This patient complains of a homonymous hemianopia meaning that she is experiencing trouble
with her vision in the same visual field of both eyes. The most common reason for this problem
is a stroke, but in a young person, a tumor is more likely. While pituitary tumors are common,
such tumors affect the optic chiasm and classically produce a bitemporal anopia, which causes
difficulty with vision in the outer half of each eye’s visual field. This is because the central
portion of the optic chiasm contains the fibers from the nasal portion of both retinas which in
turn detect light from the temporal visual fields.
pg. 343
19. A 34-year-old female presents with several weeks of bilateral hand numbness and tingling.
She notes that it is most prominent in the mornings and improves over the course of the day.
You strongly suspect carpal tunnel syndrome. Which of the following is the most sensitive
diagnostic test for this diagnosis?
A. Tinel test
B. Phalen test
C. Allen test
D. Hoffman sign
E. Eichoff test
Answer B. Phalen test (holding hands dorsum-to-dorsum with the wrists flexed at 90 degrees) is
probably the most sensitive physical examination maneuver for evaluation of carpal tunnel
syndrome, though sensitivity is reported to be as low as 51%. Tinel test (tapping the volar wrist
for elicitation of numbness or tingling) has been shown to be useless in the evaluation of carpal
tunnel syndrome and should be eliminated from routine physical examination. Allen test
(compression of radial and ulnar arteries to assess blood flow before a radial arterial
cannulation) has no relationship to carpal tunnel syndrome and is another test of dubious utility.
Hoffman sign (flicking the distal phalanx of the middle finger and looking for movement of the
thumb and index fingers) aims to evaluate cervical spinal cord compression. Recent rigorous
testing of this physical examination finding has also found poor sensitivity and specificity. The
Eichoff test (tucking the thumb into the palm and ulnarly deviating the wrist causes pain in the
radial wrist) is used to evaluate deQuervain tendonitis—the Eichoff test is commonly mislabeled
as the Finkelstein test. Importantly, the Eichoff test is not particularly accurate at evaluating
deQuervain tendonitis.
20. A 41-year-old male with diabetes and hypertension presents with acute left facial weakness,
the inability to close his left eye, and a loudness sensation in his left ear. He has no weakness or
numbness in his left arm or leg. His symptoms started yesterday and he is concerned that he has
had a stroke. Which of the following is true?
A. A CT scan of his brain should be performed to rule out the possibility of stroke.
D. The patient’s diabetes and hypertension do not affect the outcome of his illness.
E. After recovery, he has a 50% chance of having another episode in his lifetime.
pg. 344
Answer C. This patient has Bell palsy. Bell palsy is defined as paresis or paralysis of the facial
nerve (seventh cranial nerve), which is usually unilateral. The first step in the diagnosis is to
determine that the paresis or paralysis is due to a peripheral instead of a CNS lesion. In patients
with CNS lesions, furrowing of the eyebrows and closure of the eye is unaffected. This is due to
the fact that the neurons of the facial nucleus that innervate the upper face receive input from
both cerebral hemispheres (whereas the neurons of the facial nucleus that innervate the lower
face receive input primarily from the contralateral cerebral hemisphere). A CT scan would only
be necessary in patients with evidence of a CNS lesion upon physical examination. The presence
of a vesicular rash over the ear indicates Ramsay–Hunt syndrome. Ramsay–Hunt syndrome, also
known as herpes zoster oticus, results from reactivation of varicella zoster virus in the geniculate
ganglion of the facial nerve. It is much less common and carries a more severe prognosis for
recovery than Bell palsy of unknown cause or Bell palsy due to reactivation of HSV. More than
75% of patients will recover without treatment. Therefore, medical treatment is aimed at the
remaining population. Because it is not possible to identify which patients will recover fully
without treatment, all patients are generally treated upon making the diagnosis. The treatment
for Bell palsy includes corticosteroids, antiviral agents directed against herpes viruses (acyclovir,
valacyclovir, and famciclovir), and artificial tears and eye ointment to prevent corneal drying.
There is disagreement over the best regimen (e.g., some experts recommend antivirals only in
cases of a severe palsy) and the timing of presentation. However, it is generally felt that earlier
treatment yields better results and that corticosteroids are of limited utility if a patient presents
after 7 days of symptoms. Patients with diabetes and hypertension have a more severe course
and less complete recovery. Recurrence of Bell palsy rarely occurs. In cases of recurrence,
consideration should be given to alternative diagnoses.
21. Which of the following is the treatment of choice for acute cluster headache?
A. 100% oxygen
B. Sumatriptan
C. Morphine
D. Lorazepam
E. Dexamethasone
Answer A. Oxygen is the standard of care for cluster headache. Seventy-five percent of patients
with cluster headache given 100% oxygen through face mask will experience complete or
near-complete relief within 15 minutes. Because attacks of cluster headaches are self-limited
and typically last no longer than 90 minutes, patients may not have pain by the time they reach
the ED. Therefore, oxygen is an inconvenient therapy. Sumatriptan is the most effective
self-administered medication. It is administered as a nasal spray and is effective in more than
50% of patients within 15 minutes of use. However, its use is not recommended in patients who
are having more than two attacks per day because this would result in an overdose of the
pg. 345
medication. Dihydroergotamine and zolmitriptan are also effective treatments. Narcotics,
benzodiazepines, and corticosteroids have no role in acute cluster headache management.
22. A 78-year-old female presents with acute onset of right-sided arm and leg weakness 1 hour
prior to arrival. You suspect stroke and immediately order a noncontrast CT brain. The CT of
brain is unremarkable. Which of the following is the most appropriate conclusion to draw from
this CT result?
A. There is no stroke.
C. There is no meningitis.
Answer D. The role of noncontrast CT brain in the evaluation of an acute-onset, focal neurologic
deficit is to establish the lack of hemorrhage or large space-occupying lesion in the brain.
Noncontract CT brain has poor sensitivity for ischemic stroke; because ischemic strokes
represent over threequarters of all strokes, the negative predictive value of noncontrast CT
brain for stroke in general is also poor. Noncontrast CT brain is used to rule out intracranial
hemorrhage to permit consideration for IV tissue plasminogen activator (TPA), which is the only
effective treatment for acute ischemic stroke but is absolutely contraindicated in hemorrhagic
stroke. Meningitis can only be definitively diagnosed using CSF analysis with lumbar puncture.
Cervical artery dissection is best evaluated with either CT angiogram or MR angiogram of the
head and neck.
A. The patient should have been treated with prophylactic phenytoin as soon as the diagnosis of
ischemic stroke was made.
B. The patient should be given a loading dose of phenytoin after his seizure.
C. Status epilepticus occurs more commonly in the setting of ischemic strokes than in other
stroke syndromes.
D. Phenytoin is contraindicated in patients with ischemic stroke due to its potential for causing
ataxia.
pg. 346
E. Although isolated seizures are common in patients with ischemic strokes, treatment with
antiepileptic drugs is unnecessary because recurrence is uncommon.
Answer B. The incidence of seizures after ischemic stroke is uncertain but is cited to be as high
as 13%. Typically, patients with seizures after stroke are divided into patients who have seizures
within 7 days (early seizures) and those who develop seizures after 7 days (late seizures).
Prophylactic use of anticonvulsants in patients with ischemic stroke has not been shown to
reduce either early or late seizures although there are limited data on this topic. Therefore, the
prophylactic use of anticonvulsants in such patients is not recommended. When seizures do
occur, their management is the same as “conventional” seizure management, and status
epilepticus is rare. Current guidelines recommend that patients with an ischemic stroke in the
ED who develop seizures warrant standard treatment (if necessary) to terminate the seizure
followed by treatment with an anticonvulsant to prevent recurrence. There are no special
contraindications to antiepileptic use in stroke patients.
24. A 24-year-old female presents to the ED with left facial pain. She had been diagnosed with
right-sided trigeminal neuralgia 1 year ago and was started on carbamazepine. Although her
symptoms resolved over several weeks, she is now complaining of similar symptoms on the left
side of her face. In addition, she notes a history of mild right arm weakness and numbness
several months ago that seems to have resolved without any intervention. Which of the
following should be considered in this patient?
A. Lyme disease
C. Guillain–Barre syndrome
Answer B. Two percent to 4% of patients with trigeminal neuralgia have MS. This patient’s
history of recurrent attacks of neurologic dysfunction in different regions suggests a diagnosis of
MS. In addition, primary trigeminal neuralgia peaks in the sixth to seventh decade with an
average onset of age 50 and >90% of patients who present are older than 40. Therefore,
physicians need to have a higher suspicion for secondary causes in young patients with
trigeminal neuralgia. The peak age at onset of MS is 25 to 30 years although women have a
slightly younger age at onset than men. In addition, the incidence in women exceeds that in
men by almost 2:1. Roughly 5% of cases are diagnosed before the age of 20 (early onset) and
10% are diagnosed after the age of 50 (late onset). Other than MS, posterior fossa tumors, and
vascular or aneurysmal compression of the trigeminal nerve should be considered.
pg. 347
25. Which of the following is true regarding transverse myelitis?
Answer D. Patients with transverse myelitis usually have bilateral clinical findings referable to
one or two discrete spinal levels. Children and younger adults represent the majority of cases.
CSF is totally normal in about half of cases, and CSF glucose is almost always normal.
Albuminocytologic dissociation, referring to elevated CSF protein in the absence of CSF
pleocytosis, is seen more commonly in variants of Guillain–Barre syndrome than tranverse
myelitis, which usually exhibits a mild–moderate, CSF lymphocytosis. A compressive lesion of
the spinal cord by definition excludes the possibility of transverse myelitis.
26. Which of the following is the average duration of a typical generalized tonic–clonic seizure?
A. 15 seconds
B. 30 seconds
C. 1 minute
D. 2 minutes
E. 3 minutes
Answer C. Electroencephalogram (EEG) changes last for an average of 59.9 seconds (standard
deviation of 12 seconds), whereas behavioral changes last for 52.9 to 62.2 seconds (with a
standard deviation of 14 seconds). Therefore, a seizure that has lasted for 5 minutes is more
than 17 standard deviations longer than the “typical” seizure. This is partly why status
epilepticus is now “operationally” defined as any seizure lasting 5 minutes. The traditional
definition has been any seizure lasting 30 minutes or recurrent seizures without an interictal
return to baseline mental status.
27. Which of the following is true regarding control of hypertension and stroke?
A. Patients with both ischemic and hemorrhagic strokes should have their BP reduced to an SBP
of 140 to 160 mm Hg or prestroke levels.
pg. 348
B. BP should not be controlled in any stroke patient unless the patient is a candidate for tissue
plasminogen activator (tPA).
C. Only patients with ischemic infarcts should have their BP reduced to a target SBP of 140 to
160 mm Hg or prestroke levels.
D. Only patients with intracerebral hemorrhage should have their BP reduced to a target SBP of
140 to 160 mm Hg or prestroke levels.
E. All stroke patients who experience clinical deterioration in the ED should have emergent BP
control to a target SBP of 140 to 160 mm Hg or prestroke levels.
Answer D. Patients with ischemic stroke should not have their BP reduced unless they are either
candidates for tPA, have concomitant aortic dissection, MI or renal failure, or their pressure is
above 220/120 mm Hg. In fact, hypertension is probably neuroprotective in patients with
ischemic stroke by ensuring that cerebral perfusion pressure is maintained. In contrast, patients
with hemorrhagic stroke should have their BP reduced to a target SBP of 140 to 160 mm Hg (or
the patient’s prestrike level if it is higher). Lowering the BP in such patients may reduce the
stimulus for bleeding and prevent hematoma expansion. Because prognosis in patients with
intracerebral hemorrhage is tightly linked to hematoma volume, strict BP control may have a
dramatic positive effect in such patients.
28. Which of the following is more characteristic of a subdural hematoma than an epidural
hematoma?
A. Lucid interval
B. Coma
E. Delayed presentation
Answer E. Subdural hematomas occur due to disruption of the cranial bridging veins from
trauma, causing blood to collect deep to the dura. The elderly are at much higher risk than the
average population due to brain atrophy, which causes these bridging veins to stretch and be
susceptible to even minor trauma. Due to this low-pressure venous bleeding, patients with
subdural hematoma may have a subacute or chronic presentation with mild initial clinical
manifestations of injury. Patients with epidural hematoma, on the other hand, usually have an
arterial source and exhibit signs and symptoms almost immediately after the trauma. Some
patients with subdural hematoma, especially the elderly and alcoholics, may not even recall
their antecedent head trauma. Many patients with subdural hematoma have a lucid interval
(where the mental status is completely normal before becoming altered again), which is more
pg. 349
classically thought to be associated with epidural hematoma. Coma, focal neurologic deficits,
and increased intracranial pressure are associated with both subdural and epidural hematomas.
29. You suspect that a lumbar puncture you performed to evaluate for subarachnoid
hemorrhage (SAH) is a “traumatic tap.” Which of the following CSF findings is most reliable for
differentiating SAH from a traumatic lumbar puncture?
C. A twofold or greater decrease in the number of RBCs from tube 1 to tube 4 is always due to a
traumatic tap.
30. Which of the following is characteristic of facial pain due to trigeminal neuralgia?
pg. 350
E. Most patients have concomitant dental disease.
31. A 42-year-old female presents with progressive lower extremity numbness and weakness
over several days. Her physical examination is remarkable for areflexia and weakness in her
lower extremities. Which of the following findings is most likely to be present in this patient?
D. Fever >102°F
Answer A. The patient likely has the most common form of Guillain–Barre syndrome (GBS),
called acute inflammatory demyelinating polyneuropathy (AIDP). AIDP represents about 90% of
cases of GBS and usually causes symmetric motor weakness, hyporeflexia, elevated CSF protein
in the absence of elevated CSF leukocyte count (albuminocytologic dissociation). Fever,
asymmetric weakness, and severe bowel or bladder abnormalities are not common in AIDP.
Other varieties of GBS include a motor axonal variety without sensory symptoms, a motor and
sensory axonal variety, and Miller Fisher syndrome, characterized by ophthalmoplegia, ataxia,
and areflexia.
32. A 55-year-old male is brought by his wife for confusion, memory loss, and impaired balance
over the past several weeks. The patient’s wife was concerned about a stroke, so she consulted
the primary care physician, who ordered an outpatient noncontrast CT brain, which was normal.
On examination, he has mild memory loss to recent events, nystagmus, and a wide-based gait.
He has no focal weakness or altered sensation, and his cranial nerves, finger-to-nose, and
heel-to-shin tests are normal. Which of the following is most likely to reveal the cause of his
symptoms?
pg. 351
A. CT brain with IV contrast
B. EEG
C. Social history
Answer C. The triad of confusion, ataxia, and oculomotor dysfunction is indicative of Wernicke
encephalopathy, which is usually due to alcohol use. Pathology is likely due to thiamine
deficiency and affected areas of the brain include the thalamic nuclei, mammillary bodies, and
cerebellar vermis. In Wernicke encephalopathy, balance is abnormal more often than
coordination testing such as fingerto- nose because the cerebellar vermis is involved more often
than the cerebellar hemispheres. Oculomotor dysfunction can be as simple as nystagmus; frank
ophthalmoplegia is not required for the diagnosis. CT brain with IV contrast reveals major blood
vessel pathology such as dissection or dural sinus thrombosis, but these would reveal focal
neurologic deficits or headache with possible fever, respectively. Vertebral artery dissection can
certainly cause nystagmus and gait disturbances but almost always causes another focal
neurologic symptom (such as weakness or paresthesias) or neck pain or headache. EEG can
evaluate for seizure activity, but the subacute, constant nature of symptoms without discrete
episodes would be unlikely. CSF analysis could reveal a subacute encephalitis or even
malignancy, but lumbar puncture should not be performed without a thorough history to ensure
no other cause is likely. Orthostatic vital signs are rarely helpful in evaluation of most emergency
conditions, partly due to unproven guidelines of what constitutes a positive change in heart rate
or blood pressure and what duration of time the examiner should wait between testing
positional changes.
33. A 25-year-old female with a history of acne on doxycycline presents with several weeks of
intermittent, diffuse, gradual onset headaches with occasional blurry vision episodes. She has
had no fever. The week prior, she saw her primary care physician, who diagnosed her with
migraines after obtaining a normal CT brain, with and without IV contrast. The patient complains
that her migraine medicines have had only minimal effect on her headaches. Vital signs are
98.6°F, 82, 16, 110/70, 99% RA. Physical examination reveals an obese woman in no acute
distress with a completely normal neurologic examination. Which of the following is the next
best step in management?
B. MRI brain
C. Lumbar puncture
D. EEG
pg. 352
E. Psychiatry consultation
Answer C. The patient has evidence of idiopathic intracranial hypertension (IIH, also known as
pseudotumor cerebri), a syndrome likely caused by increased CSF production combined with
decreased CSF resorption. IIH should be considered in any case involving obese women of
childbearing age with diffuse headaches and visual symptoms. Drugs such as oral
contraceptives, tetracyclines, and vitamin A can increase predisposition to IIH. Diagnosis rests on
exclusion of mass lesion (and dural sinus thrombosis in suspected cases) with CT brain in
conjunction with a CSF opening pressure of greater than 20 cm water. Treatment consists of
withholding suspected causative medications, acetazolamide with loop diuretics to decrease CSF
production and possibly a ventriculoperitoneal shunt in recalcitrant cases. Initial evaluation with
lumbar puncture can also serve as a temporary treatment modality by draining off CSF to
normal opening pressure levels. Repeating the CT brain with IV contrast is unlikely to yield an
answer after one has already been performed and increases risks of contrast administration.
MRI brain may be pursued in the outpatient setting if lumbar puncture is unrevealing. EEG in the
outpatient setting would be useful if seizures were suspected, but intermittent headache is an
uncommon sign of seizure. Psychiatric consultation should not be performed in the ED before
organic causes are ruled out.
34. A 47-year-old noncompliant male with hypertension and hyperlipidemia presents with a
history of 10 minutes of weakness on the right side of his body that has since resolved. His
workup in the emergency department (ED) is normal and he is admitted for further evaluation.
Which of the following should be included in his treatment in the ED?
A. Clopidogrel
B. Ticlopidine
C. Heparin
D. Aspirin
E. Warfarin
Answer D. Although no trial has definitively evaluated the effect of aspirin when given
immediately after a transient ischemic attack (TIA), aspirin has been shown to reduce the
long-term risk of stroke and cardiovascular events by 22%. No other agent has been studied as
thoroughly as aspirin. The optimal dose of aspirin has yet to be determined, as doses ranging
from 75 to 1,300 mg demonstrate similar reductions in vascular events (although the risk of
intracerebral bleeding increases at high doses). Regular strength aspirin (325 mg) is probably
adequate for most patients. The data regarding the immediate initiation of heparin for patients
with TIA and atrial fibrillation are mixed. Finally, patients who are already taking aspirin and who
pg. 353
experience a TIA may be candidates for additional drug therapy such as clopidogrel.
Consultation with a neurologist is suggested before starting such therapy.
35. Which of the following signs and symptoms may accompany a cluster headache?
A. Ptosis
B. Lacrimation
C. Miosis
D. Nasal congestion
Answer E. Cluster headaches may be associated with ipsilateral autonomic instability reflecting
both sympathetic dysfunction such as ptosis, miosis, and forehead as well as facial sweating and
parasympathetic activation, such as rhinorrhea, lacrimation, and nasal congestion. Due to the
combination of these findings in concert with the distribution of pain, it is thought that the area
responsible for cluster headaches is the cavernous sinus. In the cavernous sinus, the trigeminal
nerve, sympathetic and parasympathetic fibers converge.
36. A positive head impulse test, nystagmus that changes direction with different positions of
gaze, or positive skew deviation on eye testing indicate which of the following?
A. A hemiplegic migraine
Answer D. The head impulse test, evaluation for bidirectional nystagmus, and tests for skew
deviation make up the HINTS (head impulse, nystagmus, test of skew) battery of tests used to
differentiate between central and peripheral causes of vertigo. When all three tests are normal,
a peripheral nerve lesion is likely. However, when any of the three tests is positive, a central
lesion is more likely and an MRI should be performed. The HINTS battery is most useful in
differentiating central versus peripheral causes in patients with unremitting rather than episodic
vertigo. The head impulse test is a bedside test to assess the patient’s vestibule–ocular reflex
(VOR). The patient is asked to fix their gaze on a distant object while the provider turns their
head to the left and right. Patients with a normal VOR are able to maintain their fixed gaze when
pg. 354
their head is turned to the left and right. Testing patients with a vestibular lesion demonstrates
that their eyes are “dragged” off target before marching back to the target in a saccadic fashion.
Counterintuitively, a “normal” test in which the patient demonstrates normal fixation is more
concerning for a central lesion and those patients should receive an urgent MRI. Abnormal tests
indicate a peripheral lesion affecting vestibular nerve (cranial nerve VIII) which is more
reassuring. While both central and peripheral lesions produce nystagmus, peripheral lesions
typically produce nystagmus that is fixed in one direction regardless of gaze. Nystagmus that
changes direction, so-called bidirectional nystagmus, is concerning for a central lesion. Finally,
skew deviation is a test in which one eye is covered while the other is open, fixating on a distant
object, and the provider evaluates for vertical shift in the covered eye when it is then abruptly
uncovered. A positive test in which the covered eye shifts its position to fixate on the object
suggests a central lesion.
37. Which of the following is the most common symptom in patients with spinal cord
compression?
A. Urinary retention
B. Saddle anesthesia
C. Motor weakness
D. Bowel incontinence
E. Ascending paresthesias
Answer C. Sixty percent to 85% of patients have motor weakness at the time of presentation.
The most common pattern of muscle weakness is symmetric lower extremity weakness
although any pattern can be seen. Bladder and bowel findings occur late in the course of the
disease. Although it also occurs late in the course of compression, any patient with saddle
anesthesia should be considered to have cauda equina syndrome until proven otherwise. Back
pain is the most common symptom of epidural spinal cord compression, occurring in 83% to
95% of patients, and characteristically precedes neurologic symptoms by approximately 2
months.
38. A 25-year-old male presents with paralysis of the right side of his face on waking this
morning. On examination, you note that the upper half of his face on the right side is
functioning normally, but the lower half is completely paralyzed. He does not have a rash and
has not traveled recently. Which of the following is the most appropriate next step in
management?
A. Prednisone PO
B. Valacyclovir PO
pg. 355
C. Prednisone PO + valacyclovir PO
D. Doxycycline PO
E. CT brain
Answer E. The patient has evidence of an upper motor neuron facial palsy. Sparing of the upper
half of the face implies a lesion in the cerebrum or facial nucleus (rather than a facial nerve).
Neuroimaging, either with CT or MR, should be pursued in these cases. The other answer
choices all assume the diagnosis of a facial nerve (rather than nucleus) lesion. Prednisone plus
an antiherpetic agent is the generally accepted treatment for Bell palsy, the most common
cause of facial nerve palsy. Doxycycline is used to treat Lyme disease, which is a common cause
of facial nerve palsy in endemic areas (Northeast, Mid-Atlantic, Wisconsin, Minnesota).
40. A 38-year-old female presents with progressive, lower extremity weakness and numbness
for 1 week. Her physical examination reveals decreased strength, sensation, and deep tendon
reflexes in both lower extremities. CSF would most likely reveal which of the following?
pg. 356
E. Mild elevation in protein, significant decrease in glucose
Answer B. Patients with Guillain–Barré syndrome (GBS) exhibit significant elevations of protein
with only mild (if any) elevation in cells in their CSF. This is known as albumino-cytologic
dissociation. The presence of more cells (also known as pleocytosis) in the CSF argues more for
an infectious or inflammatory cause other than GBS. The principal symptoms of GBS are
progressive, symmetric muscle weakness accompanied by absent or depressed deep tendon
reflexes. The weakness most often begins in the legs and “ascends.” More than 80% of patients
complain of associated extremity paresthesia, though sensory findings on examination are
minimal as GBS is primarily a motor disease.
C. Patients with central and peripheral causes of vertigo generally have equal difficulty with gait
(ataxia).
Answer B. It may be difficult to clinically differentiate between patients with central and
peripheral vertigo. However, peripheral vertigo is classically associated with marked nausea and
vomiting. Furthermore, peripheral vertigo more commonly occurs along with auditory
symptoms such as hearing loss, tinnitus, or a feeling of pressure or fullness in the ear. Although
all patients with vertigo may have some difficulty walking, many patients with central lesions
cannot even stand or take a single step without falling. In contrast, patients with peripheral
vertigo can usually walk, even in the acute phase of their illness. This is why it is important to
encourage patients with vertigo to walk during bedside examination. Finally, nystagmus is an
important physical examination finding in patients with vertigo. Spontaneous nystagmus of
peripheral origin does not change direction with gaze to either side. Nystagmus in such patients,
however, does increase in amplitude when patients look in the direction of the fast phase
(known as the Alexander law). In patients with central lesions, nystagmus generally changes
direction when the patient looks in the direction of the fast phase. Nystagmus that is purely
vertical (upbeat or downbeat nystagmus) is almost always caused by a central lesion, typically of
brainstem vestibular pathways. Finally, nystagmus due to peripheral lesions typically fatigues or
ceases when the patient fixates his or her vision on a target object. Fixation does not generally
affect nystagmus due to central lesions.
pg. 357
42. Neglect or “hemi-inattention” usually indicates a stroke in which of the following
distributions?
A. Frontal lobe
B. Occipital lobe
C. Left hemisphere
D. Right hemisphere
E. Brainstem
Answer D. Neglect or hemi-inattention results from an infarction of the parietal lobe in the
nondominant hemisphere. In most people, the left hemisphere is the dominant hemisphere
whether they are right handed or left handed. In left-sided lesions, a milder neglect typically
results on the patient’s right side.
43. The classic sequence of color changes in the fingers of patients experiencing Raynaud
phenomenon is:
Answer D. Raynaud phenomenon occurs in three phases. Initially, digital pallor (white) results
from total closure of the palmar and digital arteries causing a cessation in digital blood flow.
When mild relaxation occurs, a trickle of blood is able to perfuse the ischemic digit but the
hemoglobin is rapidly desaturated resulting in cyanosis (blue). Finally, arterial spasm resolves
and restores blood flow to baseline, resulting in a reactive hyperemia (red).
44. Which of the following is more characteristic of transverse myelitis than acute inflammatory
demyelinating polyneuropathy, the most common variant of Guillain–Barre syndrome?
A. Albuminocytologic dissociation
pg. 358
D. Hemodynamic instability
Answer B. Patients with transverse myelitis usually have one or two discrete spinal levels where
inflammation occurs. If the spinal level is in the cervical region, patients will exhibit equal upper
and lower extremity weakness. If the spinal level is in the thoracic region, patients will exhibit no
upper extremity weakness at all. By contrast, the inflammation in acute inflammatory
demyelinating polyneuropathy (AIDP) causes weakness that is more marked in the lower
extremities than the upper extremities in virtually all cases. Albuminocytologic dissociation,
referring to elevated CSF protein in the absence of CSF pleocytosis, is seen more commonly in
AIDP than transverse myelitis, which usually exhibits a mild–moderate, CSF lymphocytosis.
Spinal MRI is usually abnormal in patients with transverse myelitis as a discrete inflamed spinal
level is the cardinal pathophysiology. Spinal MRI is normal in patients with AIDP. Hemodynamic
instability and antecedent Campylobacter infection are seen more often in AIDP; bowel and
bladder malfunction is seen more often in transverse myelitis.
45. A 75-year-old male with a history of hypertension, diabetes, and ischemic stroke causing
mild residual left-sided hemiparesis presents with sudden onset of complete left-sided arm and
leg paralysis with mild confusion. The symptoms occurred 1 hour prior to arrival. His vital signs
are 98.5°F, 75, 16, 168/92, 99% RA. Physical examination reveals a regular heart rhythm and 0/5
strength on the left side. Which of the following is the most appropriate next step in
management?
A. EKG
C. PO aspirin
Answer B. The patient presents with a focal neurologic deficit in the setting of pre-existing
residual stroke symptoms. The differential diagnosis includes not only stroke but also any
process that can disrupt the metabolic supply–demand balance for nutrients or oxygen,
including hypoglycemia or infection. More than three-quarters of all strokes are ischemic and
can be treated within 3 hours (4.5 hours in selected cases) with IV TPA. However, differentiating
ischemic stroke from hemorrhagic stroke is difficult without neuroimaging. The most important
initial test, therefore, is noncontrast CT of the brain. However, among the answer choices, the
only choice that can be easily performed and would change immediate management in this
clinical setting is checking the blood glucose level. While obtaining an EKG is important, it is
unlikely to reveal findings that will change immediate management. Oral aspirin and IV TPA
pg. 359
should never be given before hemorrhagic stroke is excluded. Arterial blood gas is rarely
required in the evaluation of focal neurologic deficit.
46. A 42-year-old female presents for evaluation of headaches and horizontal diplopia. Her
examination reveals normal vital signs, mild papilledema, and she is unable to move her right
eye outward. Her head CT is normal. Which of the following is the most likely finding on lumbar
puncture?
A. Elevated protein
C. Low glucose
E. Oligoclonal bands
Answer B. This patient is presenting with symptoms of idiopathic intracranial hypertension (IIH,
also called pseudotumor cerebri). The patient’s headaches and sixth nerve palsy (and associated
diplopia) are all caused by elevated intracranial pressure. While the exact cause is not known,
there is a clear association with obesity and the disease is more common in females. CSF
composition is normal in patients with IIH but opening pressure is elevated (≤200 mm H2O is
considered normal pressure while ≥250 mm H2O is considered clearly abnormal while pressures
in between may be considered equivocal). Among cranial nerves, the sixth cranial nerve has the
longest intracranial course so it is most impacted by elevations in ICP. Oligoclonal bands are
classically found in patients with MS.
47. A 63-year-old female presents with a chief complaint of progressive tingling and burning in
her bilateral lower legs and feet. Recently, she has also noted some numbness. She says a
neurologist diagnosed her with distal symmetric polyneuropathy. Which of the following is the
most likely cause of these symptoms?
B. Guillain–Barré syndrome
C. Alcoholism
D. Diabetes mellitus
E. Paraneoplastic syndrome
pg. 360
Answer D. Symmetric polyneuropathies produce the classic “stocking and glove” distribution of
sensory loss. Many systemic disorders can produce these symptoms, including diabetes,
alcoholism, and uremia, from chronic renal insufficiency. However, diabetes is, by far, the most
common etiology. Initial symptoms are typically “positive,” such as tingling or burning, rather
than “negative,” such as numbness. The symptoms begin distally and progress more proximally
and affect the lower extremities more than the upper extremities. Over time, weakness
develops usually involving dorsiflexion of the big toe followed by weakness of foot dorsiflexion,
foot drop, loss of ankle jerks, and finally a “step page gait.”
48. A 47-year-old female with a history of factor V Leiden mutation presents for evaluation of a
3-day history of an unremitting, moderate, right-sided headache. Which of the following is true?
Answer C. This vignette describes a patient at high risk for a cerebral venous sinus thrombosis
(CVT). The factor V Leiden mutation produces a factor V protein that is resistant to inactivation
by activated protein C, which usually tempers the activity of factor V (and other coagulation
proteins). Unchecked factor V leads to an increased tendency to develop venous thrombosis.
Most patients with CVT present with a headache, though the characteristics of the headache are
variable and will depend, in part, on the location and extent of the venous thrombosis within
the brain. Magnetic resonance venography (MRI with contrast) is the preferred test to
demonstrate the absence of flow within the affected sinus. Papilledema is a possible finding due
to increased intracranial pressure but its absence doesn’t rule out CVT. Negative d-dimer levels
also decrease the likelihood of CVT but can’t be relied on to exclude it. Seizures are more
common in patients with CVT than in patients with other stroke subtypes.
49. A 34-year-old female presents with fever, headache, and blurry vision. Physical examination
reveals complete oculomotor paralysis bilaterally and mild proptosis. Imaging reveals cerebral
venous thrombosis (CVT). Which venous sinus is most likely affected?
A. Cavernous sinus
B. Sagittal sinus
C. Sphenoid sinus
D. Occipital sinus
pg. 361
E. Straight sinus
Answer A. The patient has clinical evidence of cavernous sinus thrombosis, with fever,
headache, and bilateral ocular paralysis. Among young adults and adolescents, CVT is more
common in women than men by 3:1, probably because of oral contraceptives and the increased
thromboembolic risk associated with pregnancy. Other causes of CVT include malignancy,
infection, and head injury. Magnetic resonance venography (MRV) is the study of choice when
CVT is suspected. Symptoms and signs of CVT depend on the location of the thrombosis, but
most patients present with a headache. Otherwise, hemiparesis or focal monoparesis is the
most predominant finding. Seizures also more commonly occur in patients with a stroke due to
CVT than in patients with more common causes of stroke. In cavernous sinus thrombosis, ocular
symptoms are paramount with proptosis, eye pain, swelling of the conjunctiva (chemosis), and
ocular muscle weakness. Given her fever, this patient’s thrombosis is likely caused by an
underlying infection, most commonly caused by staphylococci and streptococci. Treatment
involves broad-spectrum antibiotics and neurosurgical consultation for possible surgical
drainage. Heparin may also be indicated in patients who have extensive thrombosis. Without
treatment, mortality is close to 100%, and even with treatment it is close to 30%.
50. A 58-year-old female with a history of diabetes, hypertension, and hyperlipidemia presents
to the ED with a chief complaint of diplopia. Examination reveals the patient’s left eye is turned
outward and appears to be looking down. Pupillary response is normal. Which of the following is
the next step?
E. Brain angiogram
Answer A. This patient has a partial, pupil-sparing left cranial nerve III palsy. The primary
anatomic pathology to exclude is an aneurysm of the posterior communicating artery (called a
“PCOM aneurysm”). The best and most consistent test is an MRA though a CT angiogram could
also be used to exclude an aneurysm. MRI helps to evaluate non-aneurysmal causes of a CNIII
palsy (e.g., brainstem stroke) and is superior to CT. The classic position of the eye in the setting
of an oculomotor nerve palsy (cranial nerve III) is “down and out.” Superior and inferior rectus
muscle weakness cancel each other out while the eye moves outward because of medial rectus
weakness and downward because of inferior oblique muscle weakness. In patients with a
pupil-sparing but otherwise complete CNIII palsy, the etiology is almost always microvascular
pg. 362
ischemia rather than an aneurysm. Such patients typically recover without intervention in 8 to
12 weeks, though most patients are started on aspirin or other antiplatelet medications.
A. Meningioma
B. Astrocytoma
C. Medulloblastoma
D. Metastases
E. Pituitary adenoma
Answer D. The American Cancer Society estimated that approximately 17,000 people were
diagnosed with primary brain tumors in 1999 versus >100,000 people who died with metastatic
brain tumors. The most common cause is lung cancer followed by breast carcinoma and colon
carcinoma. Malignant melanoma and renal carcinomas metastasize to the brain less commonly.
The malignant gliomas, anaplastic astrocytoma, and glioblastoma multiforme are the most
common glial tumors, and are typically located in the cerebral hemispheres.
Answer B. Of the primary headache syndromes, cluster headaches have the most consistent
presentation. The average age of onset is 28 to 30 years. They are strictly unilateral headaches
that occur in the ophthalmic division (V1) of the trigeminal nerve. Therefore, the pain is most
commonly maximal in the retro-orbital and temporal region, although patients may experience
radiating pain to the forehead, temple, cheek, and jaw. The pain is severe and is often described
as “boring” or “tearing” in nature. Patients are often very restless or agitated during cluster
headaches and characteristically rock or pace back and forth. The headaches last on average
from 45 to 90 minutes and occur one to three times daily. The typical cluster period (during
which headaches occur) lasts 6 to 12 weeks with typical remissions lasting 12 months.
Traditionally, they have been much more common in men, although recent epidemiologic
pg. 363
studies have revealed a declining predominance of men. In addition, patients affected by cluster
headaches more frequently indulge in cigarette smoking and alcohol abuse. Interestingly, once a
cluster period begins, alcohol usually triggers an attack within minutes. There is also a seasonal
pattern to the clusters, with exacerbations occurring more often in the spring and fall.
53. Which of the following is more characteristic of migraine headaches than cluster headaches?
Answer C. The modified diagnostic criteria for migraine defines migraine headache as a
headache lasting 4 to 72 hours, that includes at least two of the following four symptoms:
Unilateral pain
Throbbing
Nausea or vomiting
Photophobia or phonophobia
Unfortunately, many of these symptoms are similar to those experienced by patients with
cluster headaches. However, the pain in cluster headaches is typically described as “boring” or
“tearing,” despite being of severe intensity. Furthermore, pain associated with cluster
headaches is almost always retro-orbital and in the temporal region (due to involvement of the
V1 branch of the trigeminal nerve). Cluster headaches are not typically associated with nausea,
vomiting, photophobia or phonophobia. The key difference between these two headache
syndromes is the cyclic nature of cluster headache exacerbations and the stereotypical
presentation of patients with cluster headaches. Sumatriptan is useful for the acute treatment
of patients with either headache syndrome.
54. Which of the following is true regarding cerebral venous thrombosis (CVT)?
pg. 364
B. The most common presentation is lethargy.
Answer E. CVT is a rare cause of stroke and is extremely variable in its presentation. The most
common presenting symptom is headache. In contrast to arterial stroke, patients with CVT
present acutely only 30% of the time, subacutely (more than 2 days after symptom onset) in
50% of cases, and chronically (more than 1 month after symptom onset) in 20% of cases. The
area of the sinus involved determines the symptoms. The most common sinuses to be affected
are the superior sagittal sinus, the cavernous sinus, and the transverse (or lateral) sinus.
Thrombosis of other sinuses is less common but may be underdiagnosed because of the
difficulty in recognizing the manifestations of thrombosis. MRI and magnetic resonance
venography is the gold standard for diagnosis. Although changes consistent with CVT may be
visible on CT, its sensitivity is inadequate to exclude the disorder, as it may be normal in up to
30% of cases. Heparin is the mainstay of therapy, even in patients with evidence of hemorrhage,
although some patients may be candidates for catheter directed thrombolysis. In general, CVT
has a better outcome than arterial stroke, although patients with deep cerebral and cerebellar
venous thrombosis have poor outcomes. Women with CVT outnumber men by a ratio of 3:1, in
part due to the use of oral contraceptives and because of the increased risk of venous
thrombosis surrounding pregnancy and the immediate postpartum period. Although either focal
or generalized seizures may occur, most seizures are focal due to the focal irritation of the
cortex affected by the thrombosis.
55. A 55-year-old male presents with right arm and leg weakness and left-sided facial droop.
Which of the following arteries is most likely involved?
E. Basilar artery
Answer E. “Crossed signs,” in which a patient has unilateral cranial nerve deficits but
contralateral hemiparesis and hemisensory loss are diagnostic of brainstem infarction. The
vertebral arteries, which have their origin from the subclavian arteries, merge to form the
basilar artery at the pontomedullary junction. At the junction of the pons and the midbrain, the
basilar artery again separates into the two posterior cerebral arteries. The brainstem, from the
pg. 365
medulla to the midbrain, is therefore supplied by branches of the vertebrobasilar arterial
system. The facial nucleus, which originates within the pons, may be infarcted when branches of
the basilar artery are occluded. As the infarction involves the facial nucleus, the entire face,
including the forehead, is affected. As the descending corticospinal tract has not yet reached the
medullary decussation, occlusion on one side of the pons affects the descending motor fibers
reaching the contralateral body and results in contralateral hemiparesis. When this syndrome
includes an ipsilateral rectus palsy, it is known as Millard–Gubler syndrome due to infarction of
both the facial and abducens nuclei, which are in very close proximity in the pons. Infarction of
the basilar artery proper is typically a catastrophic event, resulting in quadriplegia and often
respiratory failure and death.
56. Which of the following is true regarding ischemic stroke and cerebral edema?
A. Herniation in large strokes resulting from cerebral edema is more common in young patients
than in elderly patients.
B. The effects of hyperventilation on reducing Intracranial pressure (ICP) last for up to 1 week.
C. Methylprednisolone should be given to all stroke patients with evidence of edema on CT.
Answer A. Owing to cortical atrophy, elderly patients have more intracranial space to
accommodate the edema associated with large strokes. Therefore, herniation syndromes are
more common in younger patients with less baseline atrophy. Overall, medical treatment of
cerebral edema is poor. Patients frequently receive corticosteroids, osmotherapy in the form of
mannitol or furosemide, or endotracheal intubation followed by hyperventilation as a means of
decreasing elevated ICP due to cerebral edema. None of these methods has been conclusively
proved to improve outcome. Corticosteroids, in particular, have only been shown to increase
the rate of infections, gastrointestinal bleeding, and hyperglycemia. The effects of
hyperventilation last 1 to 24 hours, whereas osmotherapy lasts for 48 to 72 hours. While
hyperventilation has been part of the standard approach for the management of elevated ICP in
the setting of trauma, the resulting hypocapnia causes cerebral vasoconstriction which limits
cerebral blood flow, which in turn, can worsen cerebral ischemia. Thus, hyperventilation has
largely been abandoned as a treatment modality for traumatic elevations of ICP. Cerebral
edema following ischemic stroke occurs within 24 to 36 hours but typically does not peak for
several days.
57. Which of the following is the most common presenting symptom of multiple sclerosis (MS)?
A. Urinary retention
pg. 366
B. Hemiparesis of the upper extremities
C. Ataxia
D. Aphasia
Answer E. Optic neuritis is the most common cranial manifestation of MS, and describes a
syndrome of monocular eye pain, decreased color perception, and variable visual loss primarily
affecting central vision. However, sensory disturbances, diplopia (internuclear
ophthalmoplegia), Lhermitte sign (trunk and limb paresthesias evoked by neck flexion), limb
weakness, clumsiness, gait ataxia, and neurogenic bladder and bowel symptoms may also be
presenting signs and symptoms. Approximately 20% of patients with MS will present with optic
neuritis and 50% of patients with MS will experience optic neuritis at some point in the disease
course.
58. An 81-year-old male with no prior medical history presents with acute alteration of mental
status with onset 8 hours prior to arrival. His vital signs are 99.0, 100, 22, 185/95, 96% on room
air. He is talking, but very confused. He is sent immediately to CT and on return from CT he
appears more altered and is unable to answer questions and slips in and out of consciousness.
His CT scan is seen on Figure below. After intubation to protect his airway, which of the
following is the next best step in management?
A. Hyperventilation to PCO2 20 mm Hg
C. Administer a 4-factor Prothrombin Complex Concentrate (PCC) such as factor eight inhibitor
bypassing activity (FEIBA)
D. Trendelenburg positioning
E. Administration of IV tPA
pg. 367
Answer B. The patient’s CT demonstrates subarachnoid hemorrhage. Moderate blood pressure
control (upper limit of 140 mm Hg systolic) is reasonable for the care of this patient.
Hyperventilation with accompanying PCO2 reduction can be employed as a strategy to reduce
cerebral blood flow and, consequently, intracranial pressure. However, the lower limit of PCO2
for hyperventilation should be 30 mm Hg—hyperventilation to a PCO2 level lower than this can
cause worsened outcomes. FEIBA stands for factor eight inhibitory bypass activity, and is used to
manage bleeding in hemophilia A and B patients who have developed inhibitors to standard
factor therapies. FEIBA contains activated factor VII as well as inactivated factor VII, IX, and X. It
does not currently have a role in management of nonhemophiliacs. Trendelenburg positioning
increases intracranial pressure and would be contraindicated in patients with subarachnoid
hemorrhage. IV tPA is contraindicated in any patient with active bleeding.
59. A 47-year-old female smoker with a history of hypertension presents to the ED with a
headache that started 12 hours ago. She looks uncomfortable, prefers to sit in a dark room, and
states this headache is more severe than any headache she has had before. Her CT scan is
shown in Figure below. Which of the following is true?
pg. 368
C. Nifedipine 60 mg PO should be given as soon as the CT scan result is obtained.
Answer A. The image demonstrates a subarachnoid hemorrhage (SAH). Seizures may occur in up
to one-third of patients and may result in rebleeding, a common source of morbidity and
mortality in these patients. Although the efficacy of prophylactic anticonvulsant therapy has not
been rigorously tested in these patients, most authors recommend prophylactic anticonvulsant
therapy in all patients with SAH. As the CT scan demonstrates blood in the subarachnoid space,
there is no need for lumbar puncture. Nimodipine 60 mg should be given orally as soon as the
diagnosis of SAH is made and every 4 hours thereafter. In obtunded patients, it should be
crushed and administered through an orogastric tube. Nimodipine is used to prevent
vasospasm, which may result in secondary (or “delayed”) cerebral ischemia. No other calcium
antagonist has been proven to be as effective, and even the effects of nimodipine are not
irrefutably positive. However, because of its safety and ease of use, it is currently recommended
in all patients with aneurysmal SAH. Hypertension should be controlled in the ED with
pg. 369
intravenous labetalol or nicardipine. Sodium nitroprusside and nitroglycerin should be avoided
due to their potential to cause an increase in intracranial pressure.
B. Patients with a rapid onset are more likely to have a benign recovery.
C. Patients with preceding gastroenteritis caused by Campylobacter jejuni have a more benign
course.
D. Autonomic involvement, such as urine retention, ileus, sinus tachycardia, and postural
hypotension is uncommon in GBS.
Answer A. The clinical course of GBS is more severe in the elderly. In children with the disease,
death is the exception, and rapid recovery is the rule. Overall, the mortality rate is between 4%
and 15%, although 20% of survivors have some residual disability. However, rapid-onset GBS
predicts a more severe course. Approximately 25% of patients have a preceding C. jejuni
infection, and these patients typically experience a more severe course and delayed recovery.
Autonomic involvement, resulting in urinary retention, ileus, postural hypotension, sinus
tachycardia, and cardiac dysrhythmia, is common and respiratory failure is more common in this
group of patients. Ultimately, 25% of patients with GBS experience respiratory failure that
requires mechanical ventilation.
61. A 42-year-old female presents with a chief complaint of right-hand numbness. Decreased
sensation in the volar aspect of the little finger indicates a problem in which of the following?
A. Radial nerve
B. C5 nerve root
C. Ulnar nerve
D. Median nerve
E. C6 nerve root
Answer C. One of the guiding principles of peripheral nerve testing is to evaluate nerves in their
“autonomous zone” of innervation (i.e., where there is no overlap of adjacent nerves or nerve
roots). In the hand, the standard autonomous zone for testing the median nerve is the volar
aspect of the index finger, distal to the distal interphalangeal (DIP) joint (the tip of the index
pg. 370
finger); the zone for testing the ulnar nerve is the volar aspect of the little finger, distal to the
DIP joint (the tip of the little finger). The radial nerve, however, has a far less well-defined
autonomous zone as there is much overlap with cutaneous branches of other nerves. The best
area for testing is the dorsal aspect of the webspace between the thumb and the index finger.
This area overlies the first dorsal interosseus muscle, which is an ulnar innervated structure.
However, cutaneous innervation to this area is primarily served by the radial nerve. The C5
nerve root does not have contributions to the hand and the C6 nerve root contributes to the
median and radial nerves but not the ulnar nerve which receives contributions primarily from C8
and T1 and occasionally C7.
A. Children younger than 16 years have the highest mortality due to status epilepticus.
C. The most common side effect of benzodiazepines given for status epilepticus is
hypoventilation.
D. Among adults, the most common etiology of status epilepticus is subtherapeutic antiepileptic
drug levels.
Answer D. Children have the lowest mortality from status epilepticus, whereas elderly adults
have the highest mortality. In the most widely cited study of status epilepticus to date (the
“Richmond” study), overall mortality was 3% in children younger than 16 years and 38% in
elderly adults older than 60. Although lorazepam is favored by many physicians, there is no clear
evidenced-based drug of choice. In the Veterans Affairs Cooperative Study, there was no
statistically significant difference when lorazepam was compared with phenobarbital, diazepam
plus phenytoin, or phenytoin alone as front-line agents. There was only a statistically
insignificant trend in favor of lorazepam. The most common side effect of benzodiazepines given
for status epilepticus is hypotension. Hypoventilation is the second most common side effect.
Subtherapeutic drug levels are the most common trigger of status epilepticus and status
epilepticus due to subtherapeutic drug levels has a low mortality. The next most common cause
of status epilepticus among adults is cerebrovascular accidents.
63. Which of the following is true regarding myasthenia gravis (MG) and Lambert–Eaton
myasthenic syndrome (LEMS)?
pg. 371
D. Ocular muscle weakness is the most common initial presentation.
64. A 42-year-old female with a history of migraines presents for evaluation of a throbbing,
left-sided parietal headache associated with nausea and refractory to serial doses of
sumatriptan at home. Which of the following treatments may be most helpful in treating the
patient’s acute headache?
B. Intravenous dexamethasone
D. Intravenous hydromorphone
E. Intravenous prochlorperazine
Answer E. Abortive therapy for patients presenting with acute migraine headaches includes
nonsteroidal anti-inflammatory drugs (NSAIDs) as intravenous ketorolac, triptans such as
sumatriptan, and dopamine receptor antagonists such as prochlorperazine, metoclopramide, or
chlorpromazine. Since this patient has already tried failed triptans prior to her presentation,
prochlorperazine (Compazine) is a good choice. Intravenous fluid boluses and oxygen therapy
have not been shown to help patients with migraine headaches. Intravenous dexamethasone
has been shown to decrease the rate of headache recurrence and is recommended as part of ED
therapy for migraines but is not helpful in aborting the acute headache.
pg. 372
D. Decreased pain and temperature sensation
Answer C. Tick paralysis is likely due to an unidentified toxin, which is transmitted to the human
host within a week of tick attachment. The classic clinical presentation is that of ascending
flaccid paralysis with loss of deep tendon reflexes, similar to Guillain–Barré syndrome (GBS). Tick
paralysis is not associated with the autonomic instability that often accompanies GBS.
Furthermore, many patients present more vaguely with paresthesias despite a normal sensory
examination and an abnormal gait before progressing to frank paralysis. Respiratory failure can
occur due to diaphragmatic weakness. Cranial nerve and sensory findings are rare. Treatment is
careful removal of the tick, which results in complete resolution of symptoms within 2 days.
66. A 55-year-old male with a history of myasthenia gravis (MG) presents with extreme fatigue
and somnolence. He is barely responsive to painful stimuli. His blood glucose level is 95 mg/dL.
His pulse oximetry is 96% on room air. Which of the following is the most important next step in
management?
A. Chest x-ray
C. Endotracheal intubation
E. CT brain
Answer C. Patients with MG are at risk for severe respiratory muscle weakness and resulting
hype-rcarbic respiratory failure. Patients with MG who present with somnolence should be
assumed to have severe hypercarbia until proven otherwise. Emergent endotracheal intubation
should be performed in any case where the airway appears tenuous. The absence of significant
hypoxia merely underscores the different type of respiratory failure at play (hypercarbic vs.
hypoxemic). Chest x-ray should be performed after the airway is secured—often MG patients
have aspiration that is concomitant with or even causing their myasthenic crisis. Central venous
access is not necessary as long as good peripheral lines are in place. Noninvasive positive
pressure ventilation is contraindicated in patients who have significant alteration of mental
status. CT brain can be performed as part of the altered mental status evaluation but should
never delay airway and respiratory evaluation.
pg. 373
67. A 33-year-old female presents with acute weakness in her lower extremities that is
progressively moving up her body. Along with Guillain–Barré syndrome (GBS), which of the
following usually causes a clinical picture of acute ascending paralysis?
A. Tick paralysis
B. Botulism
C. MG
D. Diphtheria
Answer A. Tick paralysis causes an ascending paralysis clinically similar to most variants of GBS
(the rare Miller–Fischer variant of GBS causes a descending paralysis that starts with cranial
nerve involvement). Signs of tick paralysis start as early as 1 day after the tick has attached.
Removal of the tick is curative. Botulism causes a descending paralysis that usually starts with
cranial nerve involvement. MG causes gradual, chronic, diffuse muscle weakness with repetitive
movements, and ptosis is a common sign. Diphtheria causes an upper respiratory
pseudomembrane associated with pharyngeal muscle weakness that can lead to airway
compromise. Paralytic shellfish poisoning occurs due to toxins in algae ingested by edible
mollusks (mussels, clams, oysters). It causes systemic symptoms such as nausea, vomiting, and
lightheadedness associated with sensory disturbances and occasionally airway and respiratory
muscle weakness.
68. The most specific finding for carpal tunnel syndrome (CTS) is:
A. Normal sensation on the medial side but abnormal sensation on the lateral side of the ring
finger
E. Lumbrical weakness
Answer A. This is known as splitting the fourth digit, and it represents the dividing line between
median and ulnar innervation to the ring finger. The median nerve also serves the “LOAF”
muscles, which include the Lumbricals, as well as the muscles, which allow thumb Opposition,
Abduction, and Flexion. However, the hallmark of CTS is sensory involvement, with motor
abnormalities developing later. The most sensitive finding for CTS is abnormal sensation of the
distal palmar tip of the index finger, as this represents the autonomous zone of the median
pg. 374
nerve (the area where there is no overlap with other cutaneous nerves). Tinel sign is the
presence of distal paresthesias in the setting of median nerve percussion at the wrist. In the
absence of sensory or motor symptoms, Tinel sign has inadequate sensitivity and specificity to
guide referral for further specialized testing.
69. A 67-year-old female with a history of hypertension and diabetes presents to the ED with a
complaint of double vision. On examination you find that she has mild left-sided ptosis and the
inability to move her left eye superiorly and medially. The remainder of her eye movements and
her pupils are normal. A non-contrast CT scan of the brain is normal. Which of the following is
the next step?
A. Cerebral angiogram
Answer C. This patient has a mononeuropathy of cranial nerve III. In addition, the
parasympathetic fibers of the oculomotor nerve seem to be spared. The most likely diagnosis is
a diabetic mononeuropathy, which results from microvascular ischemia of a nutrient artery
feeding the core of the oculomotor nerve. The peripheral aspect of the nerve, which contains
the parasympathetic fibers to the pupil, is less affected because of collateral blood supply.
However, a more ominous possibility is an aneurysm of the posterior communicating artery
([PCOM] aneurysm). The oculomotor nerve exits the brainstem between the PCOM and the
superior cerebellar artery, so a PCOM aneurysm is well positioned to impinge on the nerve as it
exits the brain. However, compression of the nerve usually affects the pupils as well, resulting in
anisocoria. In addition to a PCOM aneurysm, it is important to evaluate the brainstem for signs
of ischemia (vertebrobasilar insufficiency) as well as ischemic or hemorrhagic infarction. These
findings may not be picked up on a routine CT scan of the brain. Although a cerebral angiogram
would be useful for diagnosing an aneurysm, it is more invasive and provides less information
about the brainstem than an MRI.
70. A 66-year-old female with a history of atrial fibrillation presents with acute onset of low back
pain for 1 day. The pain is severe, diffuse, constant, and radiates down both legs. She reports
having difficulty urinating over the last 12 hours. Her medications are metoprolol and coumadin.
Vital signs are 98.6°F, 110, 18, 122/68, 98% RA. Her examination is unrevealing except for
tenderness to palpation in the midline of the lumbar spine. Which of the following is the next
best step in management?
pg. 375
B. CT myelogram
C. CT lumbar spine
E. CT brain
Answer D. The patient has evidence of spinal epidural hematoma. Anticoagulation is a common
causative mechanism. The absence of true neurologic weakness or sensory deficits does not
exclude the diagnosis, as these signs can lag the pain by days. MRI is the gold standard for
diagnosis of spinal epidural processes. Discharging the patient home with oral opioids is not
appropriate in patients with back pain in the setting of anticoagulation and early urinary
retention. CT myelogram can be pursued if MRI is contraindicated (such as in cases with metallic
foreign bodies) but has lower specificity for the diagnosis. CT lumbar spine does not adequately
image the spinal canal and is used more for bony disease. CT brain in this case is less useful
because of the bilateral lower extremity symptoms and absence of headache.
71. A 65-year-old male presents with a chief complaint of double vision. On examination,
covering either eye causes the diplopia to resolve. Which of the following is the most likely
cause?
A. Multiple sclerosis
B. Thyroid disease
D. Myasthenia gravis
E. Orbital abscess
Answer C. The patient has binocular diplopia, which resolves when either eye is covered. Causes
include all the answer choices, of which choice C (cranial nerve palsy) is the most common.
Monocular diplopia occurs in a specific eye and is usually caused by localized eye pathology.
72. Which patients with a transient ischemic attack (TIA) and the following associated signs and
symptoms are at lowest risk for recurrent TIA or future stroke?
pg. 376
D. Left arm weakness in a patient who is already taking aspirin therapy
E. Left arm weakness and facial droop in a diabetic patient that persisted for 45 minutes
Answer A. Interestingly, patients with amaurosis fugax are considered low risk because
treatment with antiplatelet agents (e.g., aspirin) is twice as effective in preventing strokes
versus patients with hemispheric ischemia. High-risk patients include any patient with
new-onset atrial fibrillation or flutter (potential cardioembolic source), patients with crescendo
TIA (more than three discrete ischemic episodes within a 72-hour period), patients who develop
a TIA while already on aspirin therapy (considered aspirin failure), and any patient meeting
several of the “Johnston” criteria. Johnston et al. studied an ED patient population who had
ischemic symptoms for an average of 3.5 hours, which is longer than the typical TIA and longer
than the newly revised definition of TIA. However, he found five risk factors (age older than 60
years, diabetes mellitus, duration >10 minutes, weakness with episode, speech impairment with
episode) that correlated to the risk of stroke within 90 days of ED discharge. The risk ranged
from 0% without any of the risk factors to 34% with all five risk factors. These criteria have yet to
be prospectively validated.
73. The use of “triptans” for the treatment of migraine-related headache should be limited to 2
days per week for which of the following reasons?
Answer B. The triptans are serotonin 5-HT1B/1D receptor agonists. Triptans are well tolerated
though they have a number of irritating side effects, which include tingling, paresthesias, and
sensations of warmth in the head, neck, chest, and limbs. Flushing, dizziness, and neck pain or
stiffness occur less frequently. However, their use is limited because more frequent use results
in rebound headache. In fact, this may occur with other abortive drugs, particularly butalbital.
Triptans can cause coronary artery constriction and may cause chest symptoms, which mimics
angina pectoris. Although these symptoms may be frightening, they are rarely life threatening.
However, there have been a few case reports of significant myocardial ischemia or infarction in
patients using triptans. Therefore, the use of triptans should be considered contraindicated in
the setting of ischemic heart disease, poorly controlled hypertension, and cerebrovascular
disease.
pg. 377
74. A 59-year-old female presents with a chief complaint of an acute headache, dizziness,
balance problems, and vomiting. A head CT is performed as part of her evaluation and reveals
the image shown (Fig. below). Which of the following is true?
Answer C. The CT image reveals a medial cerebellar hemorrhage. Patients with cerebellar
hemorrhage frequently deteriorate because the small space in the posterior fossa can’t
accommodate the edema associated with the hemorrhage leading to brainstem compression
which can be fatal. In addition, cerebellar hemorrhage may compress the fourth ventricle
leading to obstructive hydrocephalus, increased intracranial pressure, and decreased cerebral
perfusion. Thus, such patients require critical monitoring in an ICU and prompt neurosurgical
evaluation. Aphasia and hemiparesis are features of cortical strokes in the middle cerebral
artery distribution. They are not associated with cerebellar strokes. However, patients with
pg. 378
cerebellar bleeds frequently have gaze palsies because of associated cranial nerve injuries in the
adjacent brainstem.
75. Which of the following is the most common cranial nerve affected in patients with multiple
sclerosis (MS)?
A. Optic nerve
B. Oculomotor nerve
C. Trochlear nerve
D. Abducens nerve
E. Facial nerve
Answer A. Optic (CN II) neuritis is a very common manifestation in patients with MS, and its
development often heralds the onset of MS. Oculomotor (CN III) neuropathy causes ptosis,
inability to move the eye superiorly or medially, and often pupillary dilation. Trochlear (CN IV)
neuropathy causes inability to move the eye inferolaterally. Abducens (CN VI) neuropathy
causes inability to move the eye laterally. Facial (CN VII) neuropathy causes facial paralysis and
taste and hearing changes.
Answer E. Patients with spinal stenosis are often elderly and constitute a small minority of
patients with low back pain. Most patients present with subacute or chronic pain that frequently
mimics symptoms of vascular claudication (often termed neurogenic intermittent claudication).
This is problematic because vascular claudication strikes the same population. Symptoms of
neurogenic intermittent claudication include buttock pain, which frequently radiates to the
thighs and lower legs as well as cramping, paresthesias, back pain, and difficulty walking. In
contrast to patients with vascular claudication, however, patients with spinal stenosis typically
maintain a forward-leaning posture to reduce symptoms. In addition, patients are better able to
walk uphill than downhill. Sitting also improves symptoms, which helps to differentiate patients
pg. 379
with spinal stenosis from patients with disk herniation. Although patients may have involvement
at several lumbar levels, L5 is most commonly involved (75%) followed by L4, L3, and L2.
77. The family of a 49-year-old female with a history of hypertension brings her to the
emergency department (ED) with a chief complaint of mumbled, incomprehensible speech that
started approximately 5 hours ago along with weakness of her right arm, leg, and face.
Computed tomography (CT) of her head reveals an area of infarction in her left hemisphere but
no evidence of bleeding. Upon returning from CT, the nurse tells you her blood pressure is
200/105. Which of the following summarizes the best approach to managing this patient’s blood
pressure?
A. Start a sodium nitroprusside drip and titrate to a systolic blood pressure (SBP) of 160 mm Hg.
B. Give the patient her oral antihypertensive medications at her usual doses.
D. Give the patient 60 mg of nimodipine PO because of its dual effects in lowering blood
pressure and in preventing vasospasm.
Answer E. Because cerebral perfusion pressure (CPP) = mean arterial pressure (MAP) –
intracranial pressure (ICP), acute reductions in the MAP may have drastic and potentially grave
effects on a patient’s CPP. One of the most common errors in the treatment of patients with
ischemic strokes is the overly aggressive treatment of hypertension. Because this patient is not a
candidate for tPA (because she presented >3 hours after the onset of her symptoms), the
general consensus is that her hypertension does not warrant treatment unless her blood
pressure is >220/120 mm Hg or her MAP is >130 mm Hg. If this is the case, then either
nitroprusside, labetalol, or IV enalapril may be used to rapidly gain control of the patient’s blood
pressure. Additionally, in patients who are tPA candidates, these medicines can be used to bring
a patient’s blood pressure below the 185/105 mm Hg threshold required for the administration
of tPA. Exceptions to this approach may also apply in patients with ischemic stroke and
concomitant myocardial infarction (MI), aortic dissection, or acute renal failure due to malignant
hypertension. Such patients may require emergent interventions to decrease blood pressure
due to these concomitant conditions. In all other patients with acute ischemic stroke, moderate
hypertension as defined in the preceding text is thought to be neuroprotective by maintaining
adequate CPP. Nimodipine is a calcium antagonist that is used in patients with subarachnoid
hemorrhage (SAH), and it has been shown to improve outcome in such patients. Whether it
exerts its protective effects by reducing vasospasm is still an object of study.
pg. 380
78. Spontaneous intracerebral hemorrhage involving the pons most likely results in which of the
following:
D. Hemiplegia, dysarthria
Answer C. Pontine hemorrhage may result in an initial coma, probably because of disruption of
the reticular activating system. Patients who later awake may be “locked in,” a state in which
they experience total paralysis, pinpoint pupils and speechlessness secondary to severe
dysarthria. In patients who survive the insult, locked in syndrome is the most severe outcome.
More commonly, patients suffer from total paralysis, miotic pupils, and loss of horizontal gaze.
79. A 42-year-old anxious woman presents to the ED with low back pain radiating down her left
leg. She states that she has a history of a herniated disk but has never had advanced imaging or
surgery. While the patient is lying supine, you lift her right leg to approximately 45 degrees,
causing the patient to complain of pain radiating down her left leg below the knee. How do you
interpret this result?
D. The patient has a lumbar radiculopathy probably caused by a left-sided herniated disk.
E. The patient has crossed sensory nerve fibers resulting in paradoxical left-sided pain.
Answer D. The “crossed” straight leg raise (SLR) test is performed by raising the unaffected leg
of a patient complaining of radicular low back pain while keeping the knee straight. The
occurrence of pain radiating below the knee in the affected leg is nearly pathognomonic for a
herniated disk with nerve root compression. Although the normal SLR test (performed on the
affected leg) is more sensitive than the crossed SLR test, it has a low specificity. Therefore, many
patients without true disk disease will have a positive SLR test. Pain which worsens when the
ankle is dorsiflexed (Lasègue sign) may be a helpful adjunct to the initial examination. Of note,
the SLR and crossed SLR tests are considered positive only if the patient complains of radicular
pain radiating down the leg past the knee. The mere presence of back pain is considered a
negative test.
pg. 381
80. Which of the following corresponds to ischemia of the anterior circulation?
E. A, B, and C
Answer E. The anterior circulation refers to the region of brain tissue served by the internal
carotid arteries. Approximately 80% of cerebral blood flow is derived from the carotid arteries.
This includes the anterior cerebral artery, the middle cerebral artery, and the ophthalmic artery
(branch of the internal carotid artery just before joining the circle of Willis). Hemiparesis and
hemisensory loss of the leg is a result of anterior cerebral artery ischemia. Transient monocular
blindness (amaurosis fugax) is a result of ischemia in the ophthalmic artery. Aphasia and
hemiparesis are due to ischemia in the middle cerebral artery. Ataxia and vertigo along with
nausea and vomiting are caused by posterior circulation ischemia (to the cerebellum). The
posterior circulation originates in the vertebral arteries (which originate from the subclavian
arteries), which join to form the basilar artery and then bifurcate to form the two posterior
cerebral arteries. This system supplies the remaining 20% of cerebral blood flow, delivering
blood to the cerebellum and brainstem as well as to the visual occipital lobe and medial aspect
of the temporal lobe.
A. Hypertension
B. Diabetes
E. Osteogenesis imperfecta
Answer B. Carpal tunnel syndrome refers to median neuropathy due to compression at the
wrist. Common conditions associated with carpal tunnel syndrome include diabetes,
hypothyroidism, pregnancy, and rheumatoid arthritis. Findings include pain and paresthesias in
the thumb, index, and long fingers. Phalen test (pain from holding the wrists flexed for 60
seconds) is more sensitive than Tinel test (pain from tapping the volar wrist), but neither is
pg. 382
reliable enough to exclude or prove the diagnosis and nerve conduction studies are the gold
standard. Treatment involves wrist splinting, with surgery reserved for refractory cases.
82. A healthy 23-year-old female presents with acute bilateral facial weakness. She complains
that she “can’t taste anything” and that sounds are abnormally loud. She denies any unusual
travel and notes only her usual summer camping trip. Which of the following is the most likely
cause of her symptoms?
A. Infectious mononucleosis
C. Neurosyphilis
D. Lymphoma
E. Lyme disease
Answer E. This patient has evidence of bilateral facial nerve paresis. Bilateral facial nerve paresis
or paralysis is rare in the setting of Bell palsy and its presence should prompt a search for
alternative diagnoses. In general, patients with bilateral facial nerve paresis should be
considered to have Lyme disease until proved otherwise. This patient’s recent camping trip
suggests the possibility of a remote tick bite, which led to infection with the spirochete Borrelia
burgdorferi. Ten percent of patients with erythema migrans, who are not treated, will develop
neurologic manifestations, the most common of which is facial nerve palsy. Interestingly,
patients with facial nerve paralysis due to Lyme disease typically have other constitutional
symptoms or neurologic findings that point toward the diagnosis. Other causes of bilateral facial
nerve palsy include myasthenia gravis, lymphoma, sarcoidosis, brainstem tumors, and GBS.
83. A 56-year-old diabetic male presents with a chief complaint of room-spinning dizziness,
nausea, and trouble walking. While asking the patient to fixate on a distant object straight
ahead while forcibly turning his head to the right (the head impulse test), his eyes are dragged
to the right off target before slowly marching back to the midline. This indicates:
D. He has a lesion affecting his right vestibular nerve (cranial nerve VIII)
pg. 383
Answer D. The head impulse test (or sometimes the head thrust test), is a bedside test used to
distinguish central versus peripheral pathology in patients who present with a vestibular
syndrome. It is most effective in testing patients with unremitting rather than transient
symptoms. The test assesses the patient’s vestibule–ocular reflex (VOR). Patients with a normal
VOR are able to maintain their fixed gaze when their head is turned to the left and right. Testing
patients with a vestibular lesion demonstrates that their eyes are “dragged” off target before
marching back to the target in a saccadic fashion. Counterintuitively, a “normal” test in which
the patient demonstrates normal fixation is more concerning for a central lesion and those
patients should receive an urgent MRI. Abnormal tests indicate a peripheral lesion affecting
vestibular nerve (cranial nerve VIII) which is more reassuring. Answers A–C are all essentially
similar with respect to being central lesions. Oculumotor nerve palsies create diplopia and
would reveal an eye that is displaced “down and out” compared to normal.
84. A 44-year-old female with a history of migraines presents with a 2-day history of her typical
migraine symptoms including a frontal, throbbing headache with photophobia and
phonophobia. She ran out of her sumatriptan, and acetaminophen has not helped her
headache. Which of the following treatments is likely to be most effective?
A. PO ibuprofen
B. PO hydrocodone–acetaminophen
C. IV hydromorphone
D. IV lorazepam
E. IV metoclopramide
85. A 42-year-old female presents with sudden onset of acute severe headache. You strongly
suspect subarachnoid hemorrhage. Which of the following is the most likely etiology?
A. Arteriovenous malformation
B. Neoplasia
pg. 384
C. Mycotic aneurysm
D. Moyamoya disease
E. Amyloid angiopathy
86. Which of the following neurologic functions is commonly impaired in patients with anterior
cord syndromes?
A. Vibration sensation
C. Temperature sensation
D. Position sensation
Answer C. Anterior cord syndromes cause bilateral deficits in motor function and
pain/temperature sensation below the level of injury. The lateral spinothalamic and the
corticospinal tracts carry pain/temperature and motor fibers, respectively. The dorsal columns,
carrying vibration, position, and fine touch sensation, are commonly spared. Cranial nerves arise
directly out of the brainstem and are not commonly injured in cord syndromes except in rare
circumstances. Causes of anterior cord syndrome include anterior spinal artery injury or
infarction, spinal cord trauma, and intervertebral disk herniation. Prognosis is not as good as
other causes of incomplete cord injuries.
C. Multiple sclerosis
pg. 385
Answer E. Due to the large distribution of vestibular pathways in the brain, there are many
possible centrally located lesions that result in vertigo. The most important of these for
emergency room physicians is vertebrobasilar insufficiency (transient ischemic attacks (TIA) of
the vertebrobasilar arterial circulation) or frank cerebellar stroke. Vertebrobasilar arterial
ischemia is especially common in elderly patients with typical risk factors for vascular disease
(e.g., diabetes, hypertension, hyperlipidemia, and smoking). It is important to recognize these
patients because they require admission and possible monitoring in an ICU setting (depending
on their syndrome). Cerebellar infarctions are known for an unpredictable clinical course and
sudden deterioration of previously alert, stable patients. Neurosurgical intervention is typically
required when the cerebellar infarct results in impingement on the fourth ventricle with
subsequent hydrocephalus. The optimal time of intervention, however, has yet to be
determined.
88. Which of the following is a known risk factor for subarachnoid hemorrhage?
A. Hypotension
B. Smoking
D. Hypercholesterolemia
E. Hypertriglyceridemia
pg. 386
Hematology and oncology
Emergency
pg. 387
1. A 35-year-old male with a history of sickle cell disease presents with acute onset of fever,
malaise, fatigue, and lightheadedness. Physical examination demonstrates a tachycardic patient
with pale conjunctivae. You suspect aplastic crisis and draw a complete blood count with
reticulocyte count. You review his old records and note that the patient’s baseline hemoglobin
level is 8 g/dL. Which of the following laboratory abnormalities is most consistent with an
aplastic crisis?
Answer D. Sickle cell disease is a hemoglobinopathy causing sickling of red blood cells with any
systemic stress, which results in diffuse microinfarctions. Sickle cell trait is present in
approximately 10% of all African Americans, and sickle cell disease is primarily a disease of this
population. Symptoms involve multiple organ systems and result in specific acute
crises—vaso-occlusive, acute chest syndrome, splenic sequestration, and aplastic. Aplastic crises
are characterized by the acute onset of worsening anemia combined with bone marrow failure.
Laboratory abnormalities demonstrate a drop of hemoglobin of 2 g/dL from stable levels and an
inadequate reticulocyte response (<2%) from the bone marrow to this sudden anemia. Aplastic
crises are usually postinfectious and are responsible for 5% of all deaths in sickle cell patients.
2. A 23-year-old female with sickle cell disease presents with pain in her right shin and fevers for
2 weeks. She never has leg pain with her sickle cell pain crises. An x-ray demonstrates evidence
of osteomyelitis. Which of the following is the most likely etiologic agent?
A. S. aureus
B. Salmonella
C. Aspergillus
D. Neisseria gonorrhoeae
E. Pseudomonas
Answer B. The patient has a lytic lesion with fever in a subacute course consistent with
osteomyelitis. The most common cause in all patients is Salmonella. Other gram-negative
organisms such as E. coli, may also cause osteomyelitis in sickle cell patients. S. aureus is the
primary cause of osteomyelitis in healthy hosts. Interestingly, though Salmonella is the most
pg. 388
common cause of osteomyelitis in sickle cell patients, S. aureus remains the predominant cause
of septic arthritis. Aspergillus is a rare cause of septic arthritis. Gonococcus is the most common
cause of septic arthritis in the young, sexually active adult. Pseudomonas causes osteomyelitis in
patients with puncture wounds to the feet and IV drug users.
Answer E. Prothrombin complex concentrates (PCCs) are pooled plasma products that contain
vitamin-K–dependent coagulation factors as well as therapeutic levels of protein C and S, which
inhibit clot formation. Commercially available PCCs contain factors II, IX, and X but are
categorized further as three-factor (3F-PCC) or four-factor (4F-PCC) PCCs depending on whether
they also contain clinically useful amounts of factor VII. While PCCs are plasma products, they
exist in a dried, concentrated powder form, are stored at room temperature so they do not
need to be thawed, are purified so they do not need ABO typing, can be prepared within
minutes and are delivered in much smaller volumes with much shorter infusion times than
plasma. Recent studies have also demonstrated that PCCs are far more effective than fresh
frozen plasma at correcting the coagulopathy and elevated INR associated with vitamin K
antagonists like warfarin.
A. Thrombocytopenia
B. Eosinophilia
D. Aplastic anemia
pg. 389
Answer C. The absolute lymphocyte count is not only the earliest indicator of the acute radiation
syndrome but it also provides prognostic information. Patients with an absolute lymphocyte
count <500 have a very poor prognosis with very high lethality when the absolute lymphocyte
count is <100. In contrast, patients with an absolute lymphocyte count >1,000 have significant,
but usually nonlethal injuries.
5. A 56-year-old female with longstanding hypertension, renal failure on dialysis, and a seizure
disorder presents to the emergency department after a generalized seizure. Though the patient
returned to her neurologic baseline, she has a persistently bleeding tongue wound over the
margin of the tongue. Lab tests are significant for moderate anemia with a hemoglobin of 9.1
g/dL, a platelet count of 162,000 per uL, and severe uremia with a BUN of 108 mg/dL. Which of
the following is the best approach to this patient’s bleeding?
A. Platelet transfusion
B. Emergent hemodialysis
C. IV desmopressin (dDAVP)
E. IV estrogen
Answer C. While hemodialysis and intravenous estrogen can positively impact platelet
dysfunction in uremic patients and reduce bleeding, desmopressin is the least complicated,
fastest approach to address this patient’s ongoing bleeding. Desmopressin is thought to work by
promoting the release of large factor VIII:von Willebrand factor multimers from endothelial
cells. Therefore, it tends to reach maximal effectiveness after two doses as its use depletes
existing multimer stores. Hemodialysis is the most definitive means of correcting uremic
bleeding, but it’s a slow process, and won’t help to gain control of bleeding within the
emergency department. Estrogens can also be used to help control uremic bleeding but they are
slow in onset and aren’t useful for urgent applications. This patient has an adequate number of
platelets so a transfusion won’t be beneficial and suture repair is traumatic and may induce
additional bleeding.
A. Calcium
B. Uric acid
pg. 390
C. Phosphate
D. Iron
E. Lactate dehydrogenase
Answer A. TLS describes the constellation of problems that arises from massive cell death that
occurs in the setting of chemotherapy of rapidly growing, highly chemo-sensitive malignancies.
Hyperuricemia, hyperkalemia, hyperphosphatemia, and hypocalcemia are the most common
problems. The laboratory diagnosis of TLS is made when two of these four abnormalities are
present simultaneously or whenever symptomatic hypocalcemia is present. Clinical TLS occurs
when the laboratory findings are accompanied by renal failure, seizures, cardiac arrhythmias, or
death. Though elevated levels of lactate dehydrogenase are not required to make a diagnosis of
TLS, they will typically be significantly elevated in the setting of massive cellular death.
Hyperkalemia is the most immediate life threat, while hypocalcemia can be symptomatic
(tetany) or life threatening. Hyperuricemia results in significant calcium urate precipitation
(which causes hypocalcemia) and subsequent calcium urate deposition in the kidneys which
leads to renal failure. Aggressive IV hydration is the cornerstone of management of TLS. In
addition, it is critical to reduce the level of uric acid to decrease the likelihood of renal injury.
Traditionally, this has been achieved with allopurinol, which blocks the metabolism of
hypoxanthine and xanthine to uric acid by inhibiting xanthine oxidase. Since allopurinol does not
affect existing uric acid concentrations, rasburicase (a recombinant uric acid oxidase given
parenterally) is used for those with pre-existing hyperuricemia and will likely supplant
allopurinol as the standard of care.
C. Non-Hodgkin lymphoma
pg. 391
hyperviscosity is responsible for part of the phenomenon. Interestingly, however, while patients
with ALL, CML, and CLL frequently have hyperleukocytosis in excess of 100 × 109 per L, they only
rarely suffer from leukostasis, except in patients with CML who are in blast crisis (evolution to
AML). Clinically, patients with leukostasis most commonly present with symptoms related to
lung and brain involvement, such as shortness of breath with pulmonary infiltrates, and
headache, vision and hearing changes, confusion, dizziness, ataxia, and variable degrees of
mental status changes.
8. A 63-year-old male with a history of multiple myeloma presents with acute onset of
generalized weakness, blurry vision, diffuse mucosal bleeding, and headache. A complete blood
count and comprehensive chemistry panel is sent, but the laboratory is unable to perform the
analysis, citing “inappropriate blood sample.” Which of the following is the most appropriate
definitive management?
A. Hemodialysis
B. Plasmapheresis
C. Platelet transfusion
E. Erythropoietin
Answer B. Patients with multiple myeloma are at risk for hyperviscosity syndrome, which is
characterized by extremely high levels of pathologic proteins in the blood, causing increased
viscosity and vascular sludging. Microinfarctions are common, especially in cerebral and ocular
vessels. The triad of vision problems, neurologic symptoms, and mucosal bleeding in a patient
with multiple myeloma or Waldenstrom macroglobulinemia strongly suggests the presence of
hyperviscosity syndrome. Laboratory data, although helpful, often return with errors as most
standard equipment may be unable to analyze the blood because of the elevated protein levels.
Definitive treatment involves plasmapheresis. In patients with severely altered mental status,
simple phlebotomy and saline replacement may rapidly improve the clinical condition.
Hemodialysis is not used in dysproteinemias. Colloids such as platelets and packed RBC
transfusions will likely exacerbate the viscosity. Erythropoietin has no role in the management of
most acute conditions.
9. A 64-year-old female presents with paresthesias in her hands and feet for the last few weeks.
She is concerned that she has diabetes. She denies any past medical history. Physical
examination reveals decreased vibratory sensation and proprioception in her hands and feet.
Laboratory data demonstrate a normal glucose and macrocytic anemia. Which of the following
is most likely to be present on further patient history?
A. Smoking history
pg. 392
B. Chronic aspirin use
D. Chronic melena
Answer C. Neurologic symptoms of paresthesias combined with dorsal column findings in the
face of macrocytic anemia are indicative of vitamin B12 deficiency. Ataxia, depression, and
paranoia may also accompany these symptoms. Pain and temperature are usually spared, as
these sensations are not carried by the dorsal spinal columns. Causes of vitamin B12 deficiency
include chronic malabsorption, strict vegetarian diet, chronic alcohol use, ileal disease, and
pernicious anemia. Chronic smoking can cause COPD, which may lead to polycythemia rather
than anemia. Chronic aspirin use and melena can predispose patients to iron-deficiency anemia,
which is either microcytic or normocytic. Family history of thalassemia may be a risk factor for
younger patients to develop a well-compensated microcytic anemia but has little relevance in an
older adult.
10. Which of the following disease states are all patients with continuous flow left ventricular
assist devices assumed to have?
A. Vitamin D deficiency
B. Hypoproteinemia
C. Cirrhosis
D. Renal failure
Answer E. Patients with continuous flow left ventricular assist devices (LVADs) have continuous
shearing of the blood, cleaving von Willebrand factor multimers into smaller, less active pieces.
Von Willebrand factor is responsible for platelet adhesion and reduction in its active form due to
LVAD cleavage usually causes skin and mucosal bleeding and easy bruising. More serious
bleeding can occur, including epistaxis and gastrointestinal (GI) bleeding. Desmopressin or
platelet transfusion may be necessary to manage bleeding. None of the other disease states are
almost always seen in patients with LVADs, although renal failure may be more common than A,
B, or C.
11. Which of the following characteristics of warfarin is an advantage over the novel oral
anticoagulants (inhibitors of Factors IIa and Xa) in the treatment of venous thromboembolism?
pg. 393
A. No need for monitoring of drug levels
Answer C. In several meta-analyses, the novel oral anticoagulants (NOACs) have exhibited a
higher risk of causing myocardial infarction than warfarin. It is unclear why this occurs (or
whether this risk will continue), but it is currently one of the only advantages that warfarin has
over the NOACs. Choices A, B, and D actually represent advantages that the NOACs have over
warfarin. Venous thromboembolism recurrence rates are about the same with both classes of
drugs.
12. What is the most common cause of death in patients with hemophilia A?
A. Septic shock
C. Gastrointestinal bleeding
D. Intracranial hemorrhage
Answer D. Hemophilia A occurs with a genetic deficiency in factor VIII, predisposing the patient
to hemorrhagic complications. The most common cause of death is from head trauma causing
massive intracranial hemorrhage. Treatment of bleeding episodes in patients with hemophilia A
is with factor VIII. Every patient with hemophilia A should receive factor VIII after head trauma
(whether or not there is evidence of intracranial bleeding), as the risk of delayed bleeding is
high. Sepsis and MI occur with similar incidence to the general population. Gastrointestinal
bleeding does occur in hemophiliacs but is not as common a cause of morbidity or mortality as
intracranial bleeding. Congestive heart failure may be secondary to high-output failure caused
by anemia but is usually a more chronic process and is not primarily responsible for death.
13. A 56-year-old male with chronic hepatitis B presents with mild abdominal pain, weight loss,
and weakness. A CT scan of his abdomen revealed a hypodense lesion in the right lobe of his
liver suspicious for hepatocellular carcinoma. The consulting oncologist is likely to ask you to
add which of the following tests?
pg. 394
A. Beta human chorionic gonadotropin (β-hCG) level
Answer C. Levels of AFP >500 ng/mL are present in 80% to 90% of patients with hepatocellular
carcinoma (in high-incidence populations). This cutoff is used because elevated levels below 500
ng/mL may be present in patients with acute and chronic hepatitis or cirrhosis. Note that
although it has been estimated that hepatitis B is responsible for 75% to 90% of hepatocellular
carcinoma cases worldwide, metastatic disease is the most common cause of hepatic cancer in
the United States.
14. A 25-year-old male with a history of acute myelogenous leukemia presents with acute onset
of generalized weakness. He received his first dose of chemotherapy 3 days before presenting
for evaluation. Which of the following is the most likely abnormality on laboratory analysis?
A. Hyperkalemia
B. Hypercalcemia
C. Hypophosphatemia
D. Hyponatremia
E. Hypomagnesemia
Answer A. The most likely diagnosis is tumor lysis syndrome, a constellation of events that
results from rapid cellular death due to chemotherapy. Rapidly growing and hematologic
malignancies highly responsive to chemotherapy are at highest risk for development of tumor
lysis syndrome. Hyperkalemia, hyperuricemia, and hyperphosphatemia are the most common
laboratory abnormalities. Hypocalcemia is more common than hypercalcemia. Severe
fluctuations in magnesium levels are rare. Renal insufficiency and dysrhythmias are the most
serious complications. Management involves normalization of electrolyte abnormalities
(especially hyperkalemia), intravenous fluids, and treatment of renal insufficiency. Alkalinization
therapy to counteract hyperuricemia is not universally recommended, as it may exacerbate
electrolyte abnormalities. Renal failure requiring dialysis is a poor prognostic indicator.
15. Which of the following is the most common cause of death in patients with sickle cell
disease?
pg. 395
A. Myocardial infarction (MI)
B. Stroke
C. Sepsis
D. Aplastic crisis
E. Splenic sequestration
Answer C. Sickle cell disease is a hemoglobinopathy causing sickling of RBCs with any systemic
stress, which results in diffuse microinfarctions. Sickle cell trait is present in approximately 10%
of all African Americans, and sickle cell disease is primarily a disease of this population.
Symptoms involve multiple organ systems and result in specific acute crises—vaso-occlusive,
acute chest syndrome, splenic sequestration, and aplastic. The most common cause of death in
patients with sickle cell disease is from infection, usually pneumonia. Due to autoinfarction of
the spleen, patients are at risk for overwhelming sepsis from encapsulated organisms, such as
Streptococcus pneumoniae, E. coli, and Haemophilus influenzae. Stroke is another common
cause. Aplasia and splenic sequestration occur less often. MI is rare in sickle cell patients, as
coronary artery disease, although probably accelerated in these patients, does not usually
progress far enough to significantly increase the risk of infarction.
16. A 65-year-old male with multiple myeloma presents with generalized weakness and fatigue.
His physical examination is unremarkable except for severe lethargy. Serum calcium level is 14
mg/dL. Which of the following is the most appropriate next step in management?
A. Magnesium sulfate
B. Potassium phosphate
C. Normal saline
D. Vitamin D
E. Hydrochlorothiazide
Answer C. Hypercalcemia is extremely common in cancer patients and is probably the most
common type of severe metabolic abnormality in this population. Mechanisms include direct
bone destruction with release of calcium into the serum as well as PTH-like hormones secreted
by some tumors. Symptoms of hypercalcemia include fatigue, nausea, vomiting, altered mental
status, and abdominal pain. Treatment involves urgently lowering the serum calcium level with
either oral hydration in mild cases or intravenous saline hydration with optional furosemide
therapy. Bisphosphonates may also be used when the calcium level is extremely elevated.
Magnesium sulfate may be given in patients with concomitant hypomagnesemia but is not
indicated for solitary hypercalcemia. Although hypokalemia commonly accompanies
pg. 396
hypercalcemia (and is often exacerbated by therapies which lower calcium levels), phosphate
salts should be avoided for hypercalcemia, as they may cause precipitation of calcium
phosphate. If potassium levels are borderline or low, potassium chloride should be used for
replacement while calcium levels are being lowered. Vitamin D and hydrochlorothiazide both
increase serum calcium levels and are contraindicated in patients with hypercalcemia.
17. An 85-year-old female presents with a painless mass in the right side of her neck. She first
noticed the mass while brushing her teeth 3 days ago, but waited to see if it would go away
before seeking medical attention. She has had pain in her right ear for the last week. She denies
fevers, weight loss, foreign travel, night sweats, or a history of smoking. She also denies
dysphagia, odynophagia, stridor, or globus. On examination, the patient has a 4 × 2 cm firm,
immobile, nontender mass just lateral to her right sternocleidomastoid muscle at the level of
her thyroid cartilage. Her right tympanic membrane is retracted with a serous effusion. What is
the most likely diagnosis?
A. Viral lymphadenitis
B. Bacterial lymphadenitis
D. Benign neoplasia
E. Malignancy
Answer E. This case demonstrates the “80% rule” of neck masses. Eighty percent of neck masses
in children are benign, 80% of nonthyroid neck masses in adults are neoplastic, and 80% of
those are malignant. Therefore, most nonthyroid neck masses in adults are malignant. Referred
ear pain and signs of otitis media with effusion increase the likelihood of cancer. Any degree of
stridor, dysphagia, or severe hoarseness mandates immediate ENT consultation, as airway
obstruction may be imminent.
18. A 34-year-old male with a history of leukemia undergoing induction chemotherapy presents
with fever of 101.5°F. He denies any symptoms except for fever and chills. Physical examination
is normal except for fever. Basic laboratory work is normal except for a total WBC count of 1,400
cells per mm3 with 5% neutrophils. Which of the following is the most appropriate next step in
management?
B. Discharge home with oral ciprofloxacin and amoxicillin–clavulanate and oncology follow-up
pg. 397
D. Lumbar puncture, dexamethasone, ceftriaxone, vancomycin, ampicillin, admission to the
hospital
Answer C. The patient has neutropenic fever. His absolute neutrophil count (ANC) 0.05 × 1,400 =
70. Patients with neutropenic fever are at considerable risk for serious bacterial infection and
require broad-spectrum antibiotics. These patients are most often admitted to the hospital for
observation and bacterial culture monitoring, though carefully selected, low-risk patients may
be candidates for outpatient management. However, broad-spectrum antibiotics are always
administered. Patients with solid tumors are generally low risk. Patients with hematologic
cancers undergoing transplant or induction chemotherapy are high risk, in part because
chemotherapy regimens result in longer lasting and more severe neutropenia. While there are
many possible drug regimens, inpatients are generally treated with antipseudomonal
monotherapy including cefepime, ceftazidime, a carbapenems, or piperacillin–tazobactam.
While a fever workup should be performed, lumbar puncture is unnecessary unless the patient
complains of a significant headache.
19. A 72-year-old male presents to the ED with a bump on his nose (Fig. below). He denies any
pain associated with the lesion but has noticed that it has been growing over the last several
months. Which of the following is the most likely diagnosis?
B. Melanoma
C. Kaposi sarcoma
D. Mycosis fungoides
pg. 398
Answer A. The lesion represents BCC, which classically appears as a pearly white papule with
raised borders and telangiectasias over the surface of the lesion. BCC is the most common form
of skin cancer and most commonly occurs on the face, with approximately one-third of lesions
appearing on the nose alone. However, it also frequently occurs in relatively sun-protected
areas such as behind the ears. In contrast, SCC, which also most commonly appears on the head
and neck, occurs in areas of maximal sun exposure.
20. A 44-year-old cancer patient with known chemotherapy-induced neutropenia is sent by her
oncologist to the ED with a fever of 101.5°F. Which of the following is the most appropriate
initial empiric antibiotic regimen?
A. Doxycycline
B. Cefepime
C. Clindamycin
D. Metronidazole + gentamicin
E. Metronidazole + aztreonam
Answer B. Neutropenic fever patients are at considerable risk for serious bacterial infection and
require broad-spectrum antibiotics. Either gram-positive (Streptococcus or Staphylococcus
species) or gram-negative (Pseudomonas, E. coli, and others) organisms can be responsible for
infection. Cefepime monotherapy is an appropriate initial antibiotic choice, covering all
organisms except for some strains of methicillin-resistant Staphylococcus aureus (MRSA).
Vancomycin may be added to any antibiotic regimen to adequately cover MRSA in hospitals with
a high incidence of this pathogen. Doxycycline covers some gram-positive organisms and
atypicals but does not adequately cover gram negatives. Clindamycin does not cover
gram-negative organisms. The combination of metronidazole and either gentamicin or
aztreonam leaves out adequate gram-positive coverage.
21. A 45-year-old female presents with headache, fluctuating mild confusion, and fatigue for 1
week. She reports intermittent diffuse headaches, which occur in different locations in her head
and do not follow any predictable pattern. She has also noticed that she feels tired easily on her
daily activities. Her husband states she is confused at times. Physical examination is normal
except for temperature of 100.5°F and pale conjunctivae. She exhibits no meningismus.
Laboratory work is as follows:
pg. 399
Na: 135 mEq/L
K: 4.4 mEq/L
HCO3: 24 mEq/L
A. Platelet transfusion
B. Hemodialysis
C. Splenectomy
D. Plasmapheresis
E. Acyclovir
Answer D. The patient meets clinical criteria for thrombotic thrombocytopenic purpura (TTP).
The classic clinical pentad includes fever, microangiopathic anemia, thrombocytopenia, renal
dysfunction, and neurologic symptoms. However, in the modern era, patients are diagnosed
earlier in the course of the illness and rarely develop all features of the classic pentad. Hemolytic
anemia is the hallmark of the disorder, and when combined with thrombocytopenia and
neurologic symptoms is diagnostic for TTP. Pathophysiology involves formation of microthrombi
in the systemic vasculature, consuming platelets and causing microinfarctions, usually
manifested in the kidneys and brain. Confirmation is made by the presence of schistocytes on
peripheral blood smear. Notably, hematologic laboratory studies such as prothrombin time (PT),
the activated partial thromboplastin time (aPTT or PTT), and disseminated intravascular
coagulation (DIC) panel are almost always normal. Treatment is urgent plasmapheresis with or
without corticosteroids. Platelet transfusions are contraindicated, as they exacerbate
microthrombi formation. Hemodialysis is not usually required as renal dysfunction is rarely
severe. Splenectomy is used as a second-line therapy if plasmapheresis is not successful.
Acyclovir may be used for patients with herpes simplex virus encephalitis—although the patient
has fever and headache, she lacks altered mental status or meningeal findings as would be seen
in encephalomeningitis, and the laboratory findings are more suggestive of TTP.
22. Patients suffering from a hyperviscosity syndrome (leukostasis) due to severely elevated
numbers of WBCs (hyperleukocytosis) most commonly present with symptoms reflecting
involvement of which of the following organ systems?
pg. 400
A. Pulmonary and genitourinary
23. Direct synthesis of which of the following clotting factors is inhibited by coumadin?
A. Factor XII
B. Factor X
C. Factor VIII
D. Factor V
E. Factor III
Answer B. Coumadin directly inhibits vitamin-K–mediated synthesis of the following factors: II,
VII, IX, and X. Indirect inhibition of factor I (fibrin) also occurs due to the downstream effects of
reduced levels of factor II (prothrombin). The prothrombin time is prolonged in patients who are
taking coumadin and is used to track therapeutic anticoagulation.
24. Which of the following is the right remedy for the corresponding bleeding problem?
A. Desmopressin (dDAVP) for patients with severe uremia and persistent oozing epistaxis
pg. 401
B. Prothrombin complex concentrate (PCC) for bleeding due to thrombocytopenia
D. Recombinant von Willebrand factor (VWF) for a bleeding patient with hemophilia A
Answer A. Desmopressin (dDAVP) is thought to work by promoting the release of large factor
VIII:von Willebrand factor multimers from endothelial cells to promote platelet function. It is
fast acting, easy to administer, and effective. While hemodialysis provides a better long-term
solution in uremic patients, dDAVP is a much more effective short-term solution. Given its
mechanism, dDAVP can also be used for patients with hemophilia A or von Willebrand disease
with mild bleeding. Prothrombin complex concentrates (either 4-factor or 3-factor) are
alternatives to fresh frozen plasma in patients with vitamin-K antagonist (e.g., warfarin)
associated bleeding. Thrombocytopeniaassociated bleeding should be treated with a platelet
transfusion. Recombinant factor VIII is used to treat patients with hemophilia A (who have
factor VIII deficiency) as opposed to hemophilia B (who have factor IX deficiency). Recombinant
von Willebrand factor (VWF) is not appropriate for hemophilia A patients in the absence of
recombinant factor VIII.
25. Which of the following laboratory tests is most likely to be normal in patients with acute
disseminated intravascular coagulation (DIC)?
A. Prothrombin time
C. Platelet count
D. Hemoglobin
E. Fibrinogen
Answer D. DIC is an acquired consumptive coagulopathy which may occur in patients who are
critically ill. Disorder of the clotting cascade causes platelets and clotting factors to be
consumed, leading to both hemorrhagic and thrombotic complications. Almost all have
decreased levels of clotting factors, leading to prolongation in the prothrombin and partial
thromboplastin times. Fibrinolysis occurs commonly, leading to decreased fibrinogen levels and
an increase in fibrin split products. Acute anemia may not always occur, as some patients have
relatively low hemorrhagic burden and hemoglobin levels may take time to equilibrate and not
show subtle, acute decreases.
pg. 402
26. A 34-year-old female has a hemoglobin level of 10.0 g/dL with a low mean corpuscular
volume. Which of the following is the most likely cause?
A. α-Thalassemia
B. β-Thalassemia
C. Iron-deficiency anemia
D. Sideroblastic anemia
E. Folate deficiency
Answer C. Iron-deficiency anemia is the most common cause of all anemias in women of
childbearing age, likely due to menstrual blood loss. Iron-deficiency anemia is the number one
cause of microcytic anemia and is characterized by a rapid response to oral iron therapy. Both α-
and β-thalassemia cause microcytic anemia, but are far less common than iron-deficiency
anemia. Microcytosis is more severe in patients with thalassemia than with iron-deficiency
anemia. Definitive diagnosis is made by hemoglobin electrophoresis. Sideroblastic anemia is
usually found in elderly patients, alcoholics, and in those with lead poisoning. Folate deficiency,
although common, causes macrocytic anemia.
Answer C. ITP causes immune-mediated destruction of platelets. Acute ITP is more often seen in
children and chronic ITP is seen in adults. Bleeding is the most common clinical finding and
intracranial bleeding is the most common cause of death. Treatment in children is primarily
supportive with a high rate of spontaneous resolution. Mortality in young adults and children is
below 5%; in older adults, it jumps to almost 50% because of complications surrounding
intracranial hemorrhage. Aspirin is contraindicated because of its irreversible platelet-killing
effects. There is a female predominance in adult patients. Platelets are not indicated until levels
reach <10,000 to 20,000 cells per mm3. Treatment in adults is with high-dose corticosteroids
and/or IVIG.
pg. 403
28. A 34-year-old female with systemic lupus erythematosus (SLE) is diagnosed with lupus
anticoagulant. Which of the following is likely to be seen in this patient?
A. Prolonged PT
B. Venous thrombosis
C. Predisposition to bleeding
Answer B. The term “lupus anticoagulant” is a frustratingly confusing misnomer. Most patients
with lupus anticoagulant do not have SLE, and most patients with SLE do not have lupus
anticoagulant. Additionally, lupus anticoagulant is actually a prothrombotic process, causing an
increased risk of clotting rather than bleeding. Lupus anticoagulant binds to the prothrombin
activator complex and prolongs the partial thromboplastin time (PTT), which is not corrected in
a mixing study with normal blood. Treatment for thrombotic complications usually involves
anticoagulation with heparins, Coumadin, or direct factor Xa inhibitors. The term “lupus
anticoagulant” is used because (1) SLE patients with lupus anticoagulant are more likely to have
clinically significant disease compared to the general population and (2) in vitro, the lupus
anticoagulant does act as a clotting inhibitor (as opposed to the in vivo thrombotic effect).
29. Patients with which of the following tumors is at highest likelihood for developing tumor
lysis syndrome (TLS) in response to chemotherapy?
A. Colon cancer
B. Endometrial cancer
C. Burkitt lymphoma
E. Prostate cancer
Answer C. Tumor lysis syndrome (TLS) describes the constellation of problems that arises from
massive cell death that occurs in the setting of chemotherapy of rapidly growing, highly
chemo-sensitive malignancies. The tumors most commonly associated with TLS are high-grade
lymphomas (such as Burkitt lymphoma and other high-grade non-Hodgkin lymphomas) and
acute myeloid leukemia (AML). However, TLS can occur in the setting of chemotherapy for any
cancer and is in part dependent on the total cancer burden, as well as pre-existing renal disease,
patient hydration status, and the acidity of a patient’s urine.
pg. 404
Otolaryngology, maxillofacial
and dental emergency
➢ Chapter includes:
Ear nose and throat related conditions
Dental emergency and trauma
Maxillofacial emergency
1. A 27-year-old female without past medical history presents with 2 days of pain in her right
ear. The patient notes that the symptoms started with an itchy ear which progressed to pain,
discharge, and hearing loss. Examination reveals an afebrile patient who is nontoxic, with
moderate tenderness on manipulation of the auricle, erythema and edema of the tympanic
canal, and no external rash. Cranial nerve examination is normal. Which of the following will be
most helpful in treating this condition?
A. Antihistamines
B. Tympanostomy tubes
C. Systemic antivirals
D. Adenoidectomy
pg. 405
Answer E. The patient has otitis externa, inflammation of the external ear, and tympanic canal
almost always because of infection. Trauma and excessive moisture are commonly implicated in
the development of the condition. Patients generally present with otalgia and otorrhea.
Debridement of the external ear canal is the most important aspect of treatment. In nontoxic
patients, this is achieved with topical acetic acid washes with or without topical antibiotics and
steroids. A cotton or methylcellulose wick for drainage of the tympanic canal may be placed 1
cm deep in the ear and stays for 2 days. Systemically ill patients or diabetic patients require
systemic antibiotics and sometimes admission. Antihistamines can be used for symptomatic
relief but do not affect the duration of illness. Tympanostomy tubes and adenoidectomy may be
indicated for prevention of chronic/recurrent otitis media but have no effect on acute
management of otitis externa. Herpes zoster can occur in the ear and is referred to as Ramsay
Hunt syndrome. This condition requires antivirals and is associated with a vesicular rash and
sometimes cranial nerve palsies.
2. Which of the following is true regarding the treatment of a peritonsillar abscess (PTA)?
B. Recurrent PTAs typically occur >1 year after the initial episode.
Answer D. Group A streptococci (GAS), S. aureus, and respiratory anaerobes are common causes
of PTAs. Thus, empiric antibiotics directed at these organisms are needed. In many communities,
MRSA is prevalent and should be covered empirically. Thus, clindamycin is a common, effective
agent for monotherapy. If MRSA resistance to clindamycin is significant,
trimethoprim–sulfamethoxazole can be used to address MRSA, while clindamycin can be used to
treat GAS and anaerobes. If they recur, 90% of recurrent PTAs develop within 1 year of the initial
infection with the majority occurring very shortly after the initial infection. Therefore, many
authors consider such recurrences an inadequately treated initial infection or simply a
continuation of the same infection. Although tonsillectomy, whether it is performed emergently
or after the initial infection has resolved, drastically reduces the rate of recurrence, PTAs have
been known to occur after the tonsils have been removed. The treatment of PTAs is surgical
with adjunctive antibiotic therapy. Needle aspiration is often the procedure of choice in select
patients and is equally effective when performed correctly. Antibiotic therapy in the absence of
surgical drainage is not effective. Finally, while CT scanning is not indicated in what otherwise
appears to be an uncomplicated PTA, it is not possible to differentiate a PTA from peritonsillar
cellulitis based on physical examination alone which sometimes leads to unnecessary surgical
treatment.
pg. 406
3. Which of the following is the preferred imaging modality to diagnose a parapharyngeal
abscess in the ED?
Answer C. As with other deep space infections of the neck, CT is the most valuable diagnostic
tool. Although lateral neck x-rays are the preferred initial imaging modality for suspected
retropharyngeal space infections, they do not typically provide useful information in patients
with suspected parapharyngeal infections. Furthermore, CT is better able to localize the specific
site of infection in patients with retropharyngeal abscesses. Anterior–posterior plain films of the
neck are occasionally used to support a diagnosis of croup (steeple sign), but have no role in the
diagnosis of parapharyngeal space infections. Both MRI and ultrasonography are capable of
providing useful information, but MRI is limited by time and access constraints and
ultrasonography provides much less detail and is less able to accurately localize the site of the
problem.
4. The most common associated finding in pharyngitis caused by adenovirus is which of the
following?
A. Pneumonia
B. Encephalitis
C. Peritonsillar abscess
D. Scarlatiniform rash
E. Conjunctivitis
Answer E. Approximately 30% to 50% of patients with adenovirus pharyngitis have an associated
conjunctivitis, which is typically a unilateral, follicular conjunctivitis. Patients presenting with
conjunctivitis, pharyngitis, and fever are said to have pharyngoconjunctival fever, which is
diagnostic for adenovirus infection. No further testing is required. Outbreaks of
community-acquired pharyngoconjunctival fever due to adenovirus have been attributed to
exposure to water from contaminated swimming pools as well as fomites from shared pool
towels. The EP should keep this in mind when treating young children with pharyngitis and
conjunctivitis in the summer. In the absence of conjunctivitis, it is not possible to differentiate
pg. 407
adenovirus-induced pharyngitis from group A β-hemolytic streptococcal pharyngitis as both
organisms may cause an intense exudative pharyngitis.
5. A 42-year-old male presents with left ear and posterior periauricular pain for 3 days. The ear
is shown below . Which of the following is the most important next step in diagnosis?
A. X-rays
C. MRI
D. Tympanocentesis
E. Lumbar puncture
Answer B. The patient has acute mastoiditis. There is erythema behind the auricle in the setting
of otic pain. Mastoiditis is usually seen as a complication of otitis media and occurs when the
mastoid air cells are blocked by infectious debris. Streptococci are the most common cause. The
infection can spread to cause a generalized skull osteomyelitis with associated cranial
neuropathies and meningitis. Diagnosis is primarily made clinically, but CT scan of the mastoid
area can provide important information for the consulting ENT doctor. Plain radiographs are too
insensitive for the diagnosis. MRI is not necessary in the acute setting. Tympanocentesis is
unlikely to change management. Lumbar puncture is indicated only in the setting of signs and
symptoms of associated meningitis, but a CT scan should be performed first to assess for the
possibility of mass effect from localized infection.
6. Appropriate initial therapy in an adult patient with epiglottitis includes which of the
following?
pg. 408
A. Nebulized racemic epinephrine, IV levofloxacin
Answer B. Despite their widespread use, neither intravenous corticosteroids nor racemic
epinephrine has been shown to be beneficial in the management of adult epiglottitis. The
foundation of effective management is the early administration of appropriate antibiotics and
airway management, which may include early intubation or observation in a monitored setting.
While H. influenzae type B remains an important cause of this disease in adults (the H.
influenzae type b [Hib] vaccine has drastically reduced this entity in children), it is found in as
few as 17% of cases. Other important bacterial causes include Streptococcus, and numerous
gram-negative organisms. Although ampicillin was a drug of choice in the past, H. influenzae and
other pathogens are increasingly resistant to this therapy due to the presence of β-lactamase.
Therefore, second- and third-generation cephalosporins are the drugs of choice (e.g.,
ceftriaxone, cefotaxime, and ceftizoxime). Humidified oxygen is another component of
epiglottitis therapy that has no proven benefit. However, because there is little potential harm
in humidifying delivered oxygen; it remains a part of recommended treatment.
7. A 9-year-old male presents with fever, sore throat, and refusal to eat or drink because of
severe odynophagia. His oropharyngeal examination is shown in Figure below. Which of the
following is the most likely etiology?
A. Aphthous stomatitis
B. Streptococcus pyogenes
C. Corynebacterium diphtheriae
D. Coxsackievirus
pg. 409
Answer D. The vesicular lesions on the soft palate are most characteristic of herpangina, which
is usually caused by coxsackie viruses A and B. The clinical syndrome usually starts with fever,
myalgias, dysphagia, and sometimes headache and stiff neck. Oral lesions of herpangina usually
spare the gingiva and hard palate, unlike herpes simplex virus gingivostomatitis. Management of
herpangina is completely supportive and ulcers will recede in 10 days. Attention to appropriate
hydration is integral, as many patients will be unable to even drink liquids because of pain for
the first few days of illness. Aphthous ulcers are classically present on the tongue, buccal
mucosa, and soft palate, but they are rarely associated with systemic symptoms of infection.
Group A streptococci may cause pharyngeal and tonsillar ulcers but not usually more proximal
ulcers. Diphtheria classically causes a grayish tonsillar pseudomembrane without ulcerations.
8. Which of the following is the most common complication of Epstein–Barr virus (EBV)
pharyngitis?
B. Airway obstruction
C. Splenic rupture
D. Hemolytic anemia
E. Meningoencephalitis
Answer A. In practice, probably the most common complication is the ampicillin- or amoxicillin
associated rash that occurs when patients with EBV pharyngitis are mistakenly treated for a
bacterial pharyngitis. Such a rash may occur in up to 95% of patients with EBV pharyngitis who
are treated with one of these two medications. The rash occurs less commonly when patients
are treated with other β-lactam antibiotics. Elevations of the hepatic enzymes AST and ALT
commonly occur (up to 80%) to approximately four times the upper limit of normal and peak
during the second to fourth week of illness. However, levels well over 1,000 may occur. Jaundice
may also be seen (5%), but coagulopathy is not typical. Spontaneous recovery is the rule, and
pg. 410
supportive care is the only required treatment. Airway obstruction is probably the most
important complication for the emergency medicine physician. It is due to tonsillar hypertrophy
and occurs more commonly in young children because of their prominent lymphoid tissue.
Overall, however, this complication occurs in <5% of patients and can usually be managed
conservatively by elevating the head of the bed, giving intravenous fluids and corticosteroids
and using humidified air. Splenic rupture receives a lot of attention but occurs in <0.5% of adults
with EBV infection. The rate in children is thought to be much lower. It most commonly occurs in
the second and third week of illness and is rarely fatal when it does occur. Patients present with
left upper quadrant abdominal pain, with or without radiation to the left shoulder (Kehr sign),
and splenic rupture should be considered especially in the setting of shock. Numerous
hematologic complications are associated with EBV infection, including autoimmune hemolytic
anemia (3%), mild thrombocytopenia (25% to 50%), and mild transient neutropenia (50% to
80%). Headache is the most common neurologic manifestation of EBV infection (50%), although
meningoencephalitis is the most common severe neurologic complication (1% to 5%).
C. Prevertebral soft tissue swelling in excess of 22 mm at the level of C2 is diagnostic for RPA in
children and adults.
Answer B. Patients with RPAs generally prefer to lie supine to prevent the abscess and posterior
wall edema from infringing upon their airway. Such patients should never be forced to sit
upright. Although aggressive treatment of pediatric pharyngitis with early antibiotics has
reduced the incidence of subsequent RPAs, children remain the most commonly affected group.
This is due to the presence of large retropharyngeal lymph nodes in children younger than 4
years, which may become infected and subsequently develop into RPAs. Adults frequently
present with a history of antecedent trauma, such as ingestion of a fish bone or caustic agents
as well as vertebral fractures. Since the retropharyngeal lymph nodes rapidly involute after the
ages of 4 to 6, adults usually require some insult to the intact retropharyngeal mucosa in order
to develop a subsequent infection. An RPA should be suspected if the prevertebral soft tissue
from the anteroinferior aspect of C2 to the border of the tracheal air column is >7mm in
children and adults or the same space at the level of C6 is >14 mm in children and 22 mm in
adults. Although M. tuberculosis may cause an RPA, the most common cause is Staphylococcus.
Finally, the most common fatal complication is airway obstruction. Atlantoaxial separation may
occur due to damage of the transverse ligament of the atlas by the abscess. Such patients
present with neurologic symptoms and an enlarged predental space. All patients diagnosed with
pg. 411
an RPA require immediate ENT consultation, admission to the ICU for airway monitoring, and
broad-spectrum antibiotic coverage.
10. Erosion into the carotid artery is most commonly a complication of which of the following?
A. Ludwig angina
B. Parapharyngeal abscesses
C. Peritonsillar abscesses
D. Retropharyngeal abscesses
E. Epiglottitis
Answer B. Posterior parapharyngeal space infections are more dangerous than anterior
infections as they may encroach on the cervical sympathetic chain as well as the carotid artery
and jugular vein. Patients with such infections may develop an ipsilateral Horner syndrome or
cranial neuropathies of cranial nerves IX, X, XI, and XII. Jugular vein thrombosis may also occur
along with erosion of the carotid artery, resulting in life-threatening hemorrhage or aneurysm
formation. Involvement of the jugular vein may result in septic thrombophlebitis and a
subsequent Lemierre syndrome, which is also known as postanginal septicemia. Patients with
this problem present with symptoms of severe sepsis after their symptoms of pharyngitis have
resolved. All deep space neck infections have the capability of causing such severe complications
by virtue of their ability to extend to adjacent spaces resulting in a posterior parapharyngeal
space infection. However, as the posterior pharyngeal space lies alongside the carotid sheath,
infections in this area most commonly erode into the adjacent vasculature.
11. A 22-year-old female presents with a 3-day history of a severe sore throat and painful
swallowing. She has a muffled voice, but no respiratory distress. Examination reveals
“two-finger” trismus, an erythematous pharynx, swelling of the left anterior pillar, and uvular
deviation to her right. Which of the following is true?
C. Trismus is an uncommon finding that indicates extension of the infection to the deeper neck
tissues.
pg. 412
Answer D. This patient presents with a peritonsillar abscess (PTA). Polymicrobial infections are
the rule in PTAs. Although Fusobacterium sp. is isolated, so are numerous other bacteria,
principally group A β-hemolytic Streptococcus, S. aureus, and numerous anaerobes. While many
patients with PTAs received immediate tonsillectomy in the past, currently either needle
aspiration or incision and drainage, in concert with antibiotics, is considered effective therapy.
Most of the debate in the management of PTAs focuses on the best drainage strategy. To date,
the evidence does not clearly favor one method, and both methods appear to be effective.
Drainage has historically been performed with the patient sitting upright to reduce the chance
of aspiration but recent literature also supports a supine approach in Trendelenberg with the
physician at the edge of the stretcher beyond the patient’s head (as is often done in dentistry).
Suction equipment should also be assembled at the bedside to remove purulent debris. It is not
possible to accurately differentiate between the presence of a PTA and peritonsillar cellulitis
upon physical examination. Furthermore, because collections of pus may be highly viscous, a
needle placed into the center of an abscess may not drain any material, resulting in a falsely
negative aspirate. Patients with suspected PTAs but negative aspirates can undergo CT to
determine if an abscess is present. Finally, trismus is a common finding among patients with a
PTA, and its presence does not indicate extension of the infection to the deeper neck tissues.
12. In patients with a peritonsillar abscess (PTA), the area of greatest fluctuance is most
commonly:
Answer A. Drainage of a PTA is identical to drainage of any other abscess in that aspiration or
incision should initially take place at the point of maximal fluctuance. While this is determined
via digital palpation in other abscesses, physicians frequently use alternative devices to make
this assessment within the oropharynx, such as a cotton swab or tongue blade, which diminishes
the sensitivity of the examination. The use of ultrasound can be enormously helpful in locating
the fluid collection. In some cases no fluid collection is found (peritonsillar cellulitis). When an
abscess does occur, it most commonly develops at the upper pole of the tonsil, causing
fluctuance superior to the tonsillar fossa. This is fortunate, since the carotid artery lies
approximately 1.5 to 2.5 cm posterolateral to the tonsillar fossa. The actual distance from the
mucosal surface overlying the abscess may be larger due to the anterior enlargement of the
anterior pillar (palatoglossal arch) from the underlying purulent contents. However, aspiration
pg. 413
should be done with caution, inserting the needle no more than 1 cm into the presumed abscess
cavity.
13. Which of the following is the most appropriate suture to be used for gaping intraoral
lacerations involving the mucosal surface?
A. 6-0 nylon
C. 4-0 nylon
E. 2-0 silk
Answer D. Intraoral lacerations are best repaired with absorbable sutures such as polyglactin
910 (Vicryl). Vicryl causes less tissue reactivity than silk and is preferred over nylon because it
avoids the problem of a repeat visit for removal. Chromic gut is catgut that has been treated
with chromium salts to improve longevity and is also appropriate for mucosal laceration repair.
Large intraoral lacerations should be repaired primarily to prevent food particles from becoming
entrapped and causing abscess formation and cellulitis. Small intraoral lacerations should be left
to heal by secondary intention. Antibiotics (penicillin or clindamycin) may be given to patients
who have through-and-through lacerations (through external skin and intraoral mucosa).
14. A 53-year-old female presents 3 days after extraction of a premolar tooth with exquisite pain
and tenderness in the area unrelieved by ibuprofen. Halitosis is present. Which of the following
is true regarding this patient?
Answer C. The patient has dry socket, or acute alveolar osteitis. It occurs in 5% to 20% of all
tooth extractions, and the posterior mandibular teeth are at highest risk. Age, smoking, and
poor dental hygiene are strong risk factors. Dry socket is distinguished from normal post
extraction pain by the severity, delayed onset, and lack of relief with NSAIDs. Halitosis is almost
always present in patients with dry socket. Trismus is not present. Direct anesthesia, irrigation,
pg. 414
and gentle packing are indicated. Treatment may also include NSAIDs and narcotics for pain and
penicillin. Recreating a clot is associated with a higher incidence of osteomyelitis.
15. A 44-year-old male presents with a 1-week history of painful, swollen, bleeding gums as well
as malaise and intermittent fever. He smokes, takes no medicines, and denies weight loss, night
sweats, or family history of hematologic malignancy. Examination reveals poor dental hygiene,
inflamed gingival tissue, and mild regional lymphadenopathy. Which of the following is true
regarding this patient?
Answer B. The patient has acute necrotizing ulcerative gingivitis, commonly known as trench
mouth. The gingiva is painful and friable, unlike ordinary gingivitis. Treatment involves oral
antibiotics and good oral hygiene. Spirochetes and fusobacteria predominate in what is likely a
bacterial overgrowth process. Vincent angina refers to extension of acute necrotizing ulcerative
gingivitis (ANUG) to the tonsils, and cancrum oris refers to extension to the lips and buccal
mucosa. Direct contact with secretions does not confer increased risk.
16. A 25-year-old male is punched in the face at a bar and presents to you with dental pain. On
examination, his right lower first premolar has a fracture exposing a yellowish surface. No blood
is seen on the tooth. Which of the following is the correct type of fracture and what is the
proper management?
Answer D. Tooth fractures are classified by the Ellis system—type I is through the enamel and
the tooth appears white; type II is through the dentin and the tooth appears yellow; and type III
is through the pulp and the tooth has a spot of blood which reappears when wiped away. Tooth
pg. 415
fractures should all be followed up by a dentist—the time of follow-up varies by type. Type I
requires only routine follow-up within 1 week, and types II and III require either immediate
dental consultation or next day follow-up. Calcium hydroxide paste may be placed on type II and
III fractures to cover the exposed dentin and pulp.
17. Which of the following is the most common complication of acute sinusitis?
A. Maxillary cellulitis
C. Meningitis
D. Preseptal cellulitis
E. Orbital cellulitis
Answer D. Preseptal cellulitis is the most common complication of acute sinusitis. Orbital
cellulitis may also occur and is often difficult to distinguish from preseptal sinusitis based on
clinical examination alone. CT scans are generally able to differentiate between preseptal and
orbital cellulitis. They also provide additional information about neighboring structures,
including the sinuses. Cavernous sinus thrombosis is a rare, but life-threatening complication
that results from extension of the infection through valve-free veins to the cavernous sinus.
Patients typically present with severe headache as well as CN III and VI palsies along with retinal
engorgement, chemosis, proptosis, and a high fever. Like cavernous sinus thrombosis,
meningitis is another uncommon intracranial complication. Other complications include brain
abscesses, subdural empyema, orbital abscesses, maxillary cellulitis, and localized osteomyelitis.
18. A 63-year-old female presents with low-grade fever, nasal congestion, malaise, cough, and
sore throat for 3 days. She is fairly certain she has sinusitis and states that her primary care
doctor always gives her antibiotics for these symptoms. Her vitals are: 99.7, 85, 16, 123/72, 99%
RA. Her physical examination is normal, including no facial tenderness, periorbital edema, or
cranial nerve abnormalities. Which of the following is the next best step in management?
A. Nasal culture
B. Sinus aspiration
E. Supportive care
pg. 416
Answer E. The patient likely has rhinitis or other nonbacterial upper respiratory infection.
Sinusitis occurs with facial pain or tenderness and is only likely to be bacterial in the presence of
fever >102°F, duration of 10 days or greater, or clinical worsening after initial improvement.
Supportive care is indicated in the vast majority of cases of upper respiratory infection, as viral
etiology is most likely. Nasal culture is not a useful test given low yield and absence of
management change based on results. Sinus aspiration is overly invasive for this patient who is
not clinically toxic. Empiric oral steroids are not indicated in most viral syndromes. Antibiotics
should be reserved in cases with strong suspicion of bacterial etiology as noted above.
19. Which of the following is true regarding imaging of patients with suspected sinusitis?
A. Plain films are more accurate in diagnosing frontal and ethmoid than maxillary sinusitis.
B. Waters view plain film is the most sensitive test for maxillary sinusitis.
C. Computed tomography (CT) scans are able to differentiate between acute bacterial and viral
sinusitis.
D. CT scans are both highly sensitive and highly specific in the diagnosis of sinusitis.
E. Most patients with viral upper respiratory infections (URIs) have abnormal CT scan findings.
Answer E. Sinus imaging has very little role in the ED except in the evaluation of patients with
suspected complications of sinusitis (e.g., orbital cellulitis, cavernous sinus thrombosis).
However, in the modern era it is clear that plain films have such a low sensitivity for sinus
disease that they are virtually never requested. When plain films are used, they are best used
for the diagnosis of maxillary sinusitis as their sensitivity drops when used to image other
sinuses. CT scans are currently the most useful imaging modality in the evaluation of sinusitis.
Although they are highly sensitive tests, they have a poor specificity (high false positive rate).
Several studies have demonstrated that asymptomatic patients with uncomplicated viral URIs
have abnormalities of one or both maxillary sinuses on CT scan. The same is true for
asymptomatic patients with seasonal allergies. Therefore, CT scans should never stand alone in
the diagnosis of sinusitis. A diagnosis of sinusitis should be supported with clinical findings of
facial or dental pain, headache, and purulent nasal discharge, often preceded by a viral URI.
20. Which of the following is true regarding foreign body removal from the external auditory
canal?
B. Smaller instruments are more likely to cause damage to the ear canal.
C. The central portion of the external auditory canal is the smallest segment.
pg. 417
E. Button batteries are best removed by irrigation with mineral oil.
Answer B. The majority of aural foreign bodies can be easily removed in the ED without
otolaryngology referral. There are several effective means of foreign body removal, including
irrigation, or mechanical removal with a curette, right-angled loop, forceps, suction catheters, or
cyanoacrylate (Dermabond) affixed to the blunt end of a cotton swab. One guiding principle is
that smaller instruments have a greater potential for damage, given their often sharp tips. Thus,
the largest, most blunt instrument that will still fit into the ear canal should be used.
Furthermore, sedation or referral should be considered in uncooperative patients to avoid
mechanical trauma to the ear. Irrigation should never be used in cases of vegetable matter or
button battery impaction since the former will swell and increase obstruction, and the latter
may leak. Insects should first be killed, preferably with mineral oil, though alcohol or 1%
lidocaine can also be used. Living insects tend to skip and flutter in the ear canal which is a
distressing sensation for patients that may lead to less cooperation. The external auditory
meatus is the smallest portion of the ear canal.
21. A 24-year-old female previously unvaccinated presents with progressively worsening sore
throat and fever for 3 days. Her oropharyngeal examination reveals an extensive grayish
membrane extending between both tonsils. Which of the following is the most appropriate
management?
A. Piperacillin–tazobactam
Answer C. The patient has diphtheria pharyngitis, due to the gram-positive facultative anaerobe,
Corynebacterium diphtheriae. It occurs most commonly in unvaccinated individuals. The
bacteria release diphtheria toxin, which helps the formation of pseudomembranes in the tonsils,
potentially obstructing the airway. Treatment involves close airway monitoring and antitoxin
administration. Penicillin and erythromycin are first-line agents, but antibiotics are useful mostly
for reducing transmission rates and have little effect on the course of this toxin-mediated
disease. Piperacillin– tazobactam may help reduce transmission rates but will not affect the
clinical course. Acyclovir, rifampin, amikacin, and vancomycin have little effect. Corticosteroids
have not been shown to improve outcomes.
pg. 418
22. A 42-year-old previously healthy woman presents with a “bad” sore throat and painful
swallowing. She is febrile, but nontoxic and in no respiratory distress (Fig. 7-2). Which of the
following is the cause of this patient’s illness?
B. Epiglottitis
D. Bacterial tracheitis
E. Ludwig angina
Answer B. This patient’s x-ray demonstrates the classic “thumb” or “thumbprint” sign signifying
a swollen epiglottis. Although nasopharyngoscopy (NP) is an alternative to diagnose epiglottitis
through direct visualization, patients with respiratory distress are not candidates for NP until a
secure airway is established. Therefore, the lateral soft-tissue neck x-ray will continue to have a
role in supporting a diagnosis of epiglottitis. H. influenzae is the most common organism causing
adult epiglottitis. However, only a minority of throat cultures are positive, suggesting a possibly
significant role for viruses as an etiology as well. Patients with epiglottitis may subsequently
pg. 419
develop an epiglottis abscess, in which case Streptococcus and Staphylococcus are the most
common isolated species. Although there is no age or seasonal prevalence, men and smokers
are most commonly affected. The clinical course is variable, as epiglottitis is frequently preceded
by a prodrome resembling a mild upper respiratory infection. In patients who have a brief
prodrome with rapid progression of their symptoms, airway complications are more common.
Symptoms of epiglottitis include fever, dysphagia, odynophagia, sore throat, and a muffled voice
although fever may be absent in up to 50% of patients. Patients frequently have throat pain out
of proportion to their physical findings on examination. This can be an important clue in
diagnosis, as epiglottitis should be suspected in any patient in whom you expect to find severe
tonsillopharyngitis on examination and yet find a relatively benign-appearing oropharynx. Up to
one-third of patients with adult epiglottitis were seen within 48 hours before admission with
symptoms that were mistaken for another entity, usually pharyngitis. On x-ray, RPA is diagnosed
by prominent swelling of the prevertebral tissues. To avoid artifactual effects as a result of
redundant tissue, lateral neck films should be taken at full extension with deep inspiration. The
retropharyngeal space will appear erroneously enlarged if x-rays are taken in expiration and
flexion. RPA should be suspected if the prevertebral soft tissue from the anteroinferior aspect of
C2 to the border of the tracheal air column is >7 mm in children and adults or the same space at
the level of C6 is >14 mm in children and 22 mm in adults. PTA is a diagnosis that is typically
made on physical examination. A CT may be necessary in cases that are unclear, but a lateral
neck x-ray generally contributes little to the diagnosis. Bacterial tracheitis is uncommon in
adults. Lateral neck films may reveal narrowing of the tracheal air column below the level of the
glottis or a ragged posterior tracheal margin. Finally, patients with Ludwig angina are usually
diagnosed clinically, although lateral neck films will support the diagnosis and show prominent
submandibular soft-tissue swelling.
B. Outbreaks are common in institutional settings such as camps and military bases.
pg. 420
Mycoplasma infections, and may include aseptic meningitis, encephalitis, and Guillain–Barré
paralysis, among other complications. Cold agglutinins are autoantibodies directed toward RBCs,
which have been antigenically altered by Mycoplasma. Their titers do not rise to detectable
levels until approximately 2 weeks after infection and they may not be present in all patients.
Mycoplasma is not a common cause of pneumonia in the elderly.
Answer C. In children, Ludwig angina may occur without an antecedent cause, although the
disease is less common in children than in adults. Asphyxiation is the most common cause of
death, and it results due to upper airway obstruction due to the extensive swelling and edema
of the floor of the mouth and neck. In patients with impending airway compromise, fiberoptic
nasotracheal intubation is the preferred method of airway control. Due to the significant edema,
trismus, secretions, and anatomic distortion of the airway, endotracheal intubation may be
extremely difficult. Furthermore, although a surgical tray should always be present while
fiberoptic intubation is undertaken, cricothyroidotomy is also more difficult to perform in the
setting of Ludwig angina. With early antibiotic therapy, the mortality rate of Ludwig angina is
<10%. Patients who have an underlying oral malignancy are not treated any differently than
other patients with Ludwig angina. All such patients are treated with immediate intravenous
antibiotics, with rigorous observation of the airway.
C. Kiesselbach plexus
pg. 421
Answer D. Posterior nosebleeds are potentially life-threatening causes of hemorrhage due to
the difficulty in management. The most common bleeding vessel is the nasopalatine artery, a
branch of the sphenopalatine artery. Posterior nosebleeds are assumed to be present if a
nosebleed is unable to be controlled with an adequate anterior nasal pack. Posterior packing
devices such as the Epistat may be placed to provide both posterior and anterior tamponade.
Patients with posterior packs should be admitted to units that provide telemetry and pulse
oximetry monitoring (usually the ICU), given antibiotics to cover gram-positive organisms, and
ENT consultation should be sought.
26. A 55-year-old female presents with sore throat, runny nose, intermittent low-grade fevers,
and generalized malaise. She feels like something is stuck in her throat, causing odynophagia.
She is nontoxic appearing, has no respiratory distress, and has normal vital signs. A lateral soft
tissue of the neck is shown in Figure below. Which of the following is the most likely etiology?
A. Foreign body
B. Group A Streptococcus
C. Anaerobes
D. H. influenza
E. Thyromegaly
Answer D. The patient has epiglottitis. The x-ray shows an enlarged epiglottic shadow—the
classic thumbprint sign. The most common cause of epiglottitis in adults is H. influenzae. The
pg. 422
adoption of the HiB vaccine has drastically reduced pediatric cases of epiglottitis, but adults who
have not received HiB are still at risk. Presentation in adults is less dramatic than in children,
with less systemic toxicity and airway compromise. Therefore, many cases are missed on initial
evaluation and diagnosed as simple viral pharyngitis. Lateral soft-tissue neck radiography has a
sensitivity of about 90%. Management involves confirmation of x-rays with direct
nasolaryngoscopy, intravenous antibiotics (usually a third-generation cephalosporin or
ampicillin–sulbactam), and admission for airway monitoring. Consider endotracheal intubation
(with preparation for backup cricothyrotomy) in patients who have any signs of airway
compromise, such as stridor, drooling, or hoarseness. There is no evidence for foreign body on
the x-ray. Group A streptococcal infection can cause epiglottitis, less commonly in adults
compared to children. Anaerobes are commonly a cause of retropharyngeal abscess or Ludwig
angina compared to epiglottitis. Thyromegaly is difficult to appreciate on x-ray but should be
palpable on physical examination if large enough to cause odynophagia.
27. Which of the following is the most common complication of otitis media?
B. Hearing loss
C. Labyrinthitis
D. Meningitis
E. Brain abscess
Answer B. All of the choices are complications of otitis media, but hearing loss is the most
common. The primary reason for antibiotic treatment of otitis media is the prevention of these
complications. Current treatment guidelines by the American Academy of Pediatrics involve
both immediate antibiotic therapy and deferment of therapy with watchful waiting for 48 hours
to observe for development of complications.
28. Which of the following is the most common cause of Ludwig angina?
B. Diabetes mellitus
C. Mandible fractures
D. Tongue piercing
E. Oral malignancy
Answer A. Dental infections are by far the most common cause of Ludwig angina (98% to 99%
are odontogenic), and Ludwig angina may follow dental extraction. Other predisposing
pg. 423
conditions include diabetes mellitus, malnutrition, alcoholism, immunocompromised states such
as AIDS or organ transplantation, mandible fractures, tongue piercing, peritonsillar or
parapharyngeal abscesses, submandibular sialoadenitis, and trauma. Ludwig angina describes a
rapidly progressive gangrenous cellulitis of the soft tissues of the neck and floor of the mouth
that originates in the submandibular space. Patients present with dysphagia, neck swelling, neck
pain, and elevation of the tongue. Airway compromise may occur rapidly and without warning,
so attention to the airway is the primary task of emergency physicians.
29. Which of the following is true regarding temporomandibular joint (TMJ) syndrome?
Answer B. TMJ syndrome refers to a vague set of disorders involving the TMJ, such as pain, joint
locking, and dislocation. It is considered the most common cause of facial pain after dentalgia.
Young women comprise the highest risk category, and many patients have concomitant
psychiatric conditions. The pain is normally unilateral. Evaluation involves imaging to assess for
structural abnormalities and laboratory tests to check for associated systemic diseases such as
RA, degenerative joint disease, and ankylosis. Treatment is with nonsteroidal anti-inflammatory
drugs (NSAIDs), muscle relaxants, and a soft-food diet during acute episodes to prevent further
exercise of the muscles of mastication.
30. Which of the following is true regarding avulsed and subluxed teeth?
A. Avulsed teeth can almost always be successfully reimplanted if returned to their sockets
within 3 hours.
D. Teeth can be temporarily secured for up to 1 week with a periodontal pack made from resin
and catalyst paste.
E. Avulsed teeth should be scrubbed with a povidone–iodine sponge to kill microbes before
reimplantation.
Answer B. Avulsed primary teeth should never be reimplanted, as they may fuse with underlying
secondary teeth and cause considerable cosmetic deformity. Avulsed secondary teeth should be
pg. 424
reimplanted as soon as possible. If teeth are reimplanted within 30 minutes, approximately
100% will be viable, but if 2 hours have elapsed since avulsion, the chance of successful
reimplantation is essentially zero. The best known medium for transporting an avulsed tooth is
its own socket, followed by Hank solution; cold milk is the best alternative if either of these is
not available. Teeth may be secured for up to 48 hours by reimplanting the avulsed tooth and
securing it to two neighboring teeth on either side with a periodontal pack. Before
reimplantation, avulsed teeth should be gently rinsed with saline. Teeth should never be
scrubbed or treated with any cleaning solution as this will destroy the periodontal ligament
fibers which are essential for successful reimplantation.
D. Nebulized racemic epinephrine has been shown to decrease the need for intubation.
Answer B. Other than the epiglottis, epiglottitis may involve several supraglottic structures,
including the vallecula, aryepiglottic folds, arytenoids, lingual tonsils, and base of the tongue.
Inflammation does not extend to the infraglottic tissues because of the robust attachments
between the infraglottic mucosa and submucosa. Owing to the variable involvement of several
supraglottic structures, epiglottitis is sometimes referred to as supraglottitis. Drooling and
stridor are unusual presenting signs in patients with epiglottitis. Historically, however, it has
been thought that patients presenting with these symptoms, especially if they have developed
over a short time period, are at higher risk for subsequent airway obstruction. No large,
prospective trials have been conducted to sort this out. Most often, patients with epiglottitis
present with a severe sore throat and painful dysphagia. Adult epiglottitis does not demonstrate
any seasonal variation, but appears more common in males and smokers. Neither epinephrine
nor corticosteroids have been shown to be beneficial, despite their widespread use. Caution is
advised regarding the use of epinephrine as a temporizing measure in patients with epiglottis
due to possible rebound upper airway constriction after the treatment is completed. Ninety
percent of patients with epiglottitis will have abnormal lateral neck films. The classic finding is
the “thumb” or “thumbprint” sign, indicating the presence of a swollen, inflamed epiglottis.
However, a normal film cannot exclude the disease. Direct nasopharyngoscopy has been the
gold standard of diagnosis, as it allows direct visualization of the tissue in question. Recently,
however, the “vallecula” sign has been suggested as another method of screening for the
presence of epiglottitis on lateral neck films. This method relies on the physician’s ability to
locate the base of the tongue and trace it inferiorly toward the hyoid bone to locate the
vallecula. If the vallecula is not deep and roughly parallel to the pharyngotracheal air column,
pg. 425
then epiglottitis is present. In a small trial, this sign was shown to be 98% sensitive and 100%
specific for epiglottitis.
C. Kiesselbach plexus
Answer C. All of the vessels mentioned provide blood flow to the nose. Kiesselbach plexus is the
most anterior and the most easily traumatized. Anterior nosebleeds are usually easily stopped
by cautery with silver nitrate or packing with Merocel gauze. Nosebleeds that persist despite
adequate packing are posterior in origin until proven otherwise.
33. Which important anatomical structure is at risk of injury during aspiration or incision and
drainage of a peritonsillar abscess (PTA)?
A. Vagus nerve
B. Lingual artery
C. Carotid artery
E. Hypoglossal nerve
Answer C. In adults, the carotid artery lies approximately 1.5 to 2.5 cm posterolateral to the
tonsillar fossa. In pediatric patients, the distance can be as low as 6 mm in small, young children,
though it steadily increases with age and weight, up to 2.5 cm. Therefore, aspiration of a PTA in
very young children is best performed by an otolaryngologist. In adults, needles used for
aspiration should be inserted no more than 1 cm into the abscess cavity in adults. One method
to achieve this is to create a “needle guard” by trimming the needle cap such that only 1 cm of
the needle itself is exposed beyond the trimmed cap edge. Another is to bend the needle
approximately 90 degrees 1 cm from the tip. There are numerous other possibilities. The carotid
sheath contains the internal carotid artery, vagus nerve, and internal jugular vein. While all of
these structures may be injured during incision and drainage of a PTA, the carotid artery is
usually the most medial structure and is therefore most at risk. The facial artery, which gives rise
pg. 426
to the tonsillar artery, also lies lateral to the tonsil and can be injured. However, the potential
consequences of facial artery injury are far less grave than carotid artery injury.
C. High-dose amoxicillin (80 mg/kg/day) should be reserved for those patients who are older
than 2 years of age.
D. Otitis media represents the number one reason for outpatient antimicrobial prescriptions in
the United States.
Answer D. Otitis media is the number one reason for antibiotic prescriptions in the United
States, despite the fact that more than 75% of cases will resolve spontaneously without
treatment. The 2004 American Academy of Pediatrics (AAP) and American Academy of Family
Physicians (AAFP) joint guidelines recommend treatment with antibiotics for patients younger
than 2 years old, and consider an observation period for patients older than 2 years old an
appropriate management strategy if the patient does not have a high fever and is not
systemically ill. A 3-day course of IM ceftriaxone is as effective as a 10-day course of amoxicillin.
High-dose amoxicillin is recommended for patients younger than 2 years of age, who are in day
care, or those with recent exposure to antibiotics. Auralgan, a mixture of benzocaine and
antipyrine, is a local anesthetic that may provide some direct analgesia, but should not be used
in patients with tympanic membrane perforation.
35. A 24-year-old female presents with a chief complaint of bleeding from her mouth after
having her tooth extracted earlier in the day. Which of the following may help stop the
bleeding?
B. Gelfoam
D. Thrombin
pg. 427
Answer E. Tranexamic acid is an antifibrinolytic lysine analog that acts as a procoagulant by
inhibiting plasminogen activation through competitive inhibition at lysine-binding sites. It can be
used as a 5% mouthwash or it can be used to saturate gauze which is then stuffed into the
affected dental socket. Cocaine and thrombin are also used to soak gauze or cotton tips which
are then used to apply pressure directly to the socket. Gelfoam is a hydrocolloid made from
hydrolysis of collagen that has procoagulant effects and is also used in gauze form to pack the
affected socket. While thrombin or cocaine or other topical agents could be used as a spray,
actively bleeding wounds will wash away the topical agent as soon as it’s applied which limits its
effectiveness.
B. Silver nitrate sticks should not be used on bilateral surfaces of the septum.
D. Antibiotics are indicated in patients with posterior packs, but not anterior packs.
E. Admission is indicated in patients with anterior packs, but not posterior packs.
Answer B. Silver nitrate sticks should not be applied to both sides of the septum to avoid
possible septal perforation. Silver nitrate sticks will not work if active bleeding is present. They
are best reserved for raw areas of nasal mucosa to which topical vasoconstrictor agents (such as
cocaine or phenylephrine) have already been applied. Blowing the nose is essential before
placement of any packing material to remove any clot that would prevent effective packing of a
site of continuous bleeding. Cephalexin or amoxicillin–clavulanate is generally used
prophylactically in patients with any nasal packing; TMP-SMX or azithromycin may be used in
the penicillin-allergic patient. Admission is always indicated in patients with a posterior pack.
37. A 22-year-old female presents with a severe sore throat and difficulty swallowing. Her
physical examination is consistent with pharyngitis. Which of the following criteria make group A
streptococcus (GAS) more likely as a cause of her illness?
C. Nonexudative tonsillitis
pg. 428
Answer A. The Centor criteria, which have been validated in several trials, are most useful to
rule out GAS as the cause of pharyngitis. In several trials, patients with none of the four Centor
criteria were found to have had only a 2.5% chance of having a positive throat culture for GAS.
In contrast, patients with all four criteria were found to have a 56% chance of having a positive
throat culture. Furthermore, as the complications of GAS infection are less common in adults
than in children, it seems reasonable that the goal of treating low-risk patients should be
symptomatic relief rather than the prevention of adverse sequelae. The four Centor criteria are
the presence of fever (before the use of antipyretics), the absence of cough, tender anterior
cervical lymphadenopathy, and the presence of exudative tonsillitis. McIsaac et al. modified the
score such that patients younger than 15 years old receive an extra point, although a point is
subtracted from patients who are older than 45. When these data are applied, patients with a
score of 0 to 1 have a 1% chance of having a positive throat culture. In this case (a 22-year-old
female), no modification of her Centor score occurs. Properly implemented throat cultures have
a sensitivity >90%. Posterior cervical lymphadenopathy, nonexudative tonsillitis, and atypical
lymphocytes are all features of Epstein–Barr virus (EBV) infection, also known as infectious
mononucleosis. The presence of a cough, along with other “viral” symptoms such as rhinorrhea
and conjunctivitis, also make GAS less likely.
38. A 6-year-old male is brought in by his parents for evaluation of dental trauma after a fall. The
right maxillary central incisor and left maxillary lateral incisor are loose and somewhat
malpositioned. The left maxillary central incisor was completely avulsed (Fig. below). The fall
occurred only a few minutes before presentation. Which of the following is true?
D. The permanent (secondary) tooth has a better outcome after complete intrusion rather than
complete avulsion of the primary tooth.
pg. 429
Answer C. This patient suffered a complete avulsion of his left maxillary central incisor as well as
luxation injuries to his right maxillary central incisor and left maxillary lateral incisor. Luxation
injuries, in which the teeth are loose and out of position, are more common than complete
avulsion injuries, in which the teeth are removed from their socket. Most subluxed teeth will
passively return to their anatomic position but patients should be urgently referred to a dentist
for teeth that are substantially angulated or displaced. Completely avulsed primary teeth should
not be re-implanted, even if present, because doing so risks injuring the developing secondary
teeth. In the absence of a severe intrusion injury, in which the teeth are subluxed toward the
developing secondary teeth, a CAT scan is unnecessary. Even in the setting of significant
intrusion injuries, often a panoramic x-ray (Panorex) is more than adequate to evaluate the
underlying teeth and alveolar bone. When alveolar bone fractures are present in the setting of
most dental trauma, they are rarely significant and rarely require operative fixation. However,
intrusion injuries are more severe than avulsion injuries because of the risk they pose to the
development of the secondary teeth. Dental avulsion injuries do not permanently interfere with
speech production.
39. A 50-year-old male presents with right lower facial swelling for 3 days. He has had right
lower molar pain for several weeks but has not seen a dentist. The patient has a history of
alcohol abuse. On examination, the patient has a low-grade fever, restricted neck movement,
trismus, and firm swelling in the bilateral submandibular and submental regions. Which of the
following is true?
Answer D. The patient has evidence of Ludwig angina, a cellulitis of the connective tissues of the
mouth and neck. The most commonly affected teeth are the molars—usually lower and
posterior. Although odontogenic infection is the most common cause, trauma or oral
malignancy may also predispose to the condition. Ludwig angina is usually polymicrobial, and
the most common species are Streptococci, Staphylococci, and Bacteroides. Airway obstruction
is the number one cause of mortality, which may be as high as 10% even in the presence of
adequate therapy. Standard oral endotracheal intubation and cricothyroidotomy may be
difficult given the edema, secretions, and friability of tissues. Treatment involves high-dose
penicillin, clindamycin, or broader-spectrum agents such as piperacillin–tazobactam or
pg. 430
ampicillin–sulbactam. The role of steroids is not clearly defined at this time. Intensive care
admission should be initiated and ENT should be consulted to address possible tracheostomy
placement.
40. A 34-year-old male presents with hypertension. He randomly checked his blood pressure at
the drugstore and it was 172/100. His only symptoms are nasal congestion and malaise. He has
been told he had borderline hypertension in the past but has not seen a physician in several
years. You establish primary care follow-up within 24 hours for him. Which of the following is
the next best step in management?
Answer D. Patients with mild essential hypertension will often exhibit rises in their blood
pressure if they use systemic decongestants as symptomatic relief for upper respiratory
infections. This patient should be counseled specifically to avoid decongestants for this reason.
Starting oral antihypertensive therapy in this patient is not warranted given the lack of severe
hypertension, the presence of next day follow-up, and the possibility of side effects or
supratherapeutic effect. Cardiology referral for essential hypertension in an otherwise healthy
34-year-old male is not indicated. Sugar intake reduction, while laudable, will not have a
significant immediate effect on the patient’s blood pressure (reduction in salt intake would
certainly be recommended).
41. A 60-year-old female presents with painful facial swelling that started acutely over the
course of 1 hour (Fig. below). Which of the following is the best imaging modality for evaluation
of this condition?
A. X-ray
C. CT
D. MRI
E. PET scan
pg. 431
Answer C. Given the hyperacute onset of swelling and the location, the patient likely has a
parotid gland stone. Sudden blockage of the Stensen duct can cause almost immediate swelling.
CT imaging is the best modality for evaluation of the stone and exclusion of other etiologies,
though imaging is not always needed in clear-cut cases. X-ray imaging, with or without contrast
misses many parotid gland stones and are not useful in the clinical setting. MRI does not
visualize salivary gland stones well. PET scans are generally used to detect cancer and the
sudden onset of symptoms in the case doesn’t warrant an evaluation for this. Theoretically, an
existing parotid mass could undergo sudden hemorrhage and cause hyperacute swelling, but
parotid cancers are quite rare and occur less commonly than other salivary gland cancers.
42. A 22-year-old male presents with painful oral lesions for 2 days (Fig. below). He has been
unable to drink fluids due to significant odynophagia. Which of the following is the most
important next step in management?
A. Nystatin
C. Valacylovir
D. Methotrexate
E. Prednisone
pg. 432
Answer C. The patient has primary herpetic gingivostomatis due to HSV-1. The lesions are
concentrated on the hard palate and gums, rather than the soft palate more posteriorly (which
would be a sign of herpangina due to coxsackievirus). In most cases, supportive care is all that is
required. However, in cases where oral intake is limited due to pain, treatment with valacyclovir
is indicated. Nystatin is used for candidal therapy and would not be appropriate here. No
abscess exists to perform incision and drainage. Immune-suppressants such as methotrexate or
prednisone are not indicated for this primary viral infection.
43. Which of the following is the most common cause of acute-onset, nontraumatic hearing
loss?
D. Cerumen impaction
Answer D. Conductive problems such as cerumen impaction and foreign body impaction are the
most common causes of acute-onset hearing loss in the absence of trauma. Cerumen impaction
is more common than foreign body impaction, since many foreign bodies are oddly shaped and
do not completely obstruct the external auditory canal, which allows for continued, if
suboptimal, hearing.
pg. 433
44. A 77-year-old male with type 2 diabetes presents with a 4-day history of progressively
worsening left ear pain, hearing loss, and discharge. On examination, he has a temperature of
101°F, he appears fatigued, and his tympanic canal is markedly edematous with foul drainage.
His glucose level is 400. Which of the following is true regarding this condition?
C. Cranial nerve involvement almost always begins with the abducens nerve.
Answer A. The patient has necrotizing (malignant) otitis externa. This condition is seen almost
exclusively in the elderly, the immunocompromised, and diabetic patients. It is far more serious
than simple otitis externa due to the risk of spread of infection to the mastoid bone, the dural
sinuses, and the meninges. Early, mild cases may be treated with outpatient fluoroquinolones
active against Pseudomonas, but most cases will require admission and possible surgical
debridement. Pseudomonas and Staphylococcus aureus are the most common bacterial
pathogens implicated. The facial nerve is the most commonly affected cranial nerve in otitis
externa. CT scan, rather than x-rays, is indicated for evaluation of spread of disease. Antivirals
have no role in management.
45.A 47-year-old female s/p a remote tonsillectomy presents with a chief complaint of a 5-day
history of progressive neck pain radiating to her occiput. She does not recall the moment of
onset but rather describes gradually worsening neck pain over the past several days. In the day
prior to her visit to the ED, she developed a mild sore throat, and more limited range of motion.
Specifically, she has difficulty with neck flexion, and turning to the right. She denies any
shortness of breath and she has no stridor or wheezing on examination. A CT scan was obtained
and is shown in Figure 12-15. The CT image reveals a likely infection in the:
A. Peritonsillar space
B. Retropharyngeal space
C. Submandibular space
D. Epiglottis
pg. 434
Answer B. The degree of retropharyngeal (also known as prevertebral) soft-tissue swelling
indicates that the patient has a retropharyngeal abscess or developing abscess (phlegmon) (Fig.
below). A good rule of thumb is to recall that the soft-tissue width at the level of C2 should be
no greater than half the width of the C2 vertebral body. In this patient’s case, the width of the
soft tissue is approximately the same as the adjacent vertebral body. The CT also clearly
demonstrates that the area of swelling extends several levels, though the normal width of the
prevertebral tissues increases in the lower cervical spine, such that at C5, the normal width is up
to the width of the adjacent vertebral body.
pg. 435
pg. 436
Ophthalmology emergency
➢ Chapter includes:
Ophthalmological related condition
Infectious ophthalmology
Eye trauma
pg. 437
1. A 37-year-old male presents with left eye pain and redness after rubbing his eye the day
before. Slit lamp evaluation with fluorescein stain is shown below. Which of the following is the
most appropriate next step in management?
A. Topical antivirals
B. Topical steroids
C. Topical antibiotics
D. Intravenous acetazolamide
Answer C. The patient has a large central corneal abrasion as demonstrated by the irregular
patch of fluorescein uptake from 3 o’clock to 6 o’clock. Treatment of corneal abrasions involves
pain control with topical or oral analgesics, short-acting cycloplegics, and topical antibiotics to
prevent secondary infection. Topical antivirals would be indicated in patients with herpes
simplex or zoster keratitis, which are signaled by the presence of dendrites or pseudodendrites,
respectively. Topical steroids should never be given by the emergency physician (EP) without
ophthalmologic consultation beforehand, as consequences may be devastating in patients with
herpetic keratitis. Acetazolamide is used to increase aqueous humor excretion as part of
noninvasive temporizing therapies for acute glaucoma attacks. Emergent ophthalmologic
consultation is not indicated in patients with corneal abrasions, even in patients with abrasions
associated with contact lenses. If there is suspicion of corneal ulcer (which would appear as a
yellowish spot on the cornea), then consultation should be obtained emergently.
2. A 37-year-old male presents with eye pain and redness as shown below. He has experienced
no trauma. Which of the following is the most likely diagnosis?
pg. 438
A. Corneal abrasion
B. Anterior uveitis
D. Pinguecula
E. Pterygium
Answer B. The patient has conjunctival injection around the limbus (the junction of the clear
cornea and white sclera), also known as perilimbic injection or ciliary flush. This is usually due to
iritis or anterior uveitis. Corneal abrasion can occur concomitantly with an iritis, but without a
history of trauma, this would be atypical. Acute angle closure glaucoma would not be likely in a
37-year-old male without any prior history. Furthermore, the pupil would likely be fixed,
mid-dilated, or cloudy with significant loss of vision. Pinguecula and pterygium are both chronic,
benign growths in the conjunctiva of little clinical significance except cosmesis. Pterygia can
invade the line of vision and needs to be corrected in those cases.
3. A 65-year-old female presents with right eye pain, irritation, foreign body sensation, and
tearing. Skin lesions are seen on the right side of the forehead and the conjunctivae are injected.
Slit lamp examination reveals pseudodendrites. Which of the following is true?
A. Patients with associated nasal vesicles should not receive topical ophthalmic steroids.
B. A prodrome is uncommon.
pg. 439
E. Systemic antivirals are more effective than topical antivirals.
Answer E. The patient has herpes zoster ophthalmicus. Though the illness is focal, most patients
have a systemic prodrome of headache, fever, and generalized malaise. Hutchinson sign
describes a vesicular eruption over the nose which occurs due to varicella zoster virus (VZV)
involvement of the nasociliary branch of the trigeminal nerve. Since this same branch innervates
the globe, such patients are at increased risk for ocular involvement, and should receive
treatment as for patients who have clear evidence of ocular involvement. Treatment involves
systemic oral antivirals and topical steroids. Topical antivirals can be used as adjunctive therapy
and topical antibiotics can also be used as optional adjunctive therapy if the diagnosis is in
doubt. Urgent ophthalmologic consultation is generally pursued. Corneal hypoesthesia is
common and over three-fourths of patients recover completely. The ophthalmic division of
cranial nerve V is involved. Anterior uveitis occurs often in herpes zoster ophthalmicus and the
frequency is independent of severity of corneal involvement.
4. A 9-year-old male presents with progressively worsening right eyelid swelling, pain, and
redness, for 3 days. He denies blurry vision. Which of the following is more characteristic of
orbital cellulitis than periorbital cellulitis?
A. Fever
B. Periorbital edema
C. Eye tenderness
Answer E. Patients with periorbital (preseptal) cellulitis may have fever, periorbital edema, and
eye tenderness, but ophthalmoplegia is characteristic of orbital cellulitis. Gram-positive cocci
are common causes of both conditions, but S. aureus is more commonly implicated in
periorbital cellulitis. Orbital cellulitis is usually caused by S. pneumoniae, H. influenzae, M.
catarrhalis, S. pyogenes, and polymicrobial infections. Diagnosis can be definitively made by CT
scan of the brain and orbits. Orbital cellulitis mandates hospital admission with IV antibiotics,
whereas uncomplicated cases of periorbital cellulitis may be managed on an outpatient basis.
5. A 58-year-old female presents to the ED with a chief complaint of acute-onset painless flashes
and floaters in her left eye. She denies any other changes to her vision and states she can read
normally. However, on confrontation visual field testing, there is a visual field deficit in the
upper temporal quadrant of her right eye. Which of the following is likely present?
A. Vitreous hemorrhage
pg. 440
B. Posterior vitreous detachment (PVD)
D. Retinal detachment
E. Posterior uveitis
Answer D. Patients presenting with the triad of flashes, floaters, and field deficits should be
emergently referred to an ophthalmologist for presumed retinal detachment. Patients may not
recognize small peripheral detachments that do not affect visual acuity. Many patients,
however, complain of progressive partial vision loss in the affected field. PVD causes both
flashes and floaters but does not cause a field deficit. Vitreous hemorrhage most commonly
results from diabetic proliferative retinopathy, PVD, and trauma. If the degree of hemorrhage is
significant, overall visual acuity will be decreased. Furthermore, the presence of vitreous
hemorrhage significantly increases the likelihood of retinal detachment. In addition to vitreous
hemorrhage, the presence of retinal pigment within the vitreous (called vitreous pigment or
“tobacco dust”) strongly increases the likelihood of retinal detachment (LR+ = 44). Posterior
uveitis is an umbrella term referring to inflammation of the posterior portions of the middle
layer of the eye (e.g., choroiditis). It is characterized by painless floaters that often cause blurred
vision.
6. A 26-year-old female presents for evaluation of an eye injury. There is significant periorbital
soft tissue swelling and the globe can’t be visualized at the bedside. Bedside ultrasound can be
used to help diagnose which of the following?
A. Retinal detachment
B. Vitreous hemorrhage
C. Lens dislocation
D. Retrobulbar hemorrhage
Answer E. Bedside ultrasound of the orbit is one of the easiest and also the most effective uses
of ultrasound. Since the eyelids represent the only tissue between the ultrasound probe and the
eye, resolution is excellent. Using the high frequency, flat probe (7.5 to 10 MHz linear
transducer), the eye can be scanned in both the long and short axes in 1 minute. In addition to
retinal detachment (thick band lying anterior to the retina in the posterior globe), vitreous
hemorrhage (echogenic material within the usually clear black vitreous), lens dislocation (lens
located in a different place than is seen on the normal contralateral eye), and retrobulbar
hemorrhage (hypoechoic fluid collection posterior to the eye not seen on the contralateral eye),
it can be used to diagnose vitreous detachment (thin membrane just anterior to retina), globe
pg. 441
rupture (decreased globe volume with distorted shape), and intraocular foreign body. As noted
above, ultrasound can be used in patients with globe rupture, but a lot of gel should be used
and excessive pressure should not be applied to the eye.
7. A 23-year-old female is struck in the eye with a soccer ball. Penlight eye examination is shown
below. Which of the following is the most appropriate next step in management?
A. Trendelenburg positioning
D. Eye shielding
E. IV antibiotics
Answer D. The patient has a total (or “eight-ball”) hyphema. The pupil is almost completely
obscured by blood in the anterior chamber due to the ocular trauma. ED management is with
ophthalmologic consultation, topical steroids (under the direction of the ophthalmologist only),
head elevation, eye shielding, and tetanus booster. The eye is shielded to prevent secondary
injury and further bleeding. Trendelenburg positioning and NSAIDs will likely worsen the bleed
and are contraindicated. Although factor VII has important procoagulant activities, it has
currently not been approved for use in patients with hyphemas. IV antibiotics are not indicated
except in patients with evidence of globe rupture.
pg. 442
states that he was watching television when he experienced a brief episode in which many
“glittery lights” flashed in his right eye. The episode resolved, but he has subsequently noted a
couple of gray spots floating in his vision. He denies decreased visual acuity. Direct
ophthalmoscopy is unrevealing. Which of the following is the most likely cause of his symptoms?
A. Retinal detachment
B. Ocular migraine
D. Vitreous detachment
E. Optic neuritis
Answer D. PVD is an extremely common problem, and in otherwise healthy adults, it is a normal
occurrence with aging. Over time, pockets of fluid called premacular bursa develop within the
normally gelatinous vitreous. Their presence destabilizes the vitreous which contracts,
eventually pulling away from its posterior attachment to the retina. The detachment stimulates
retinal photoreceptors, resulting in “flashes” of light. The posterior edge of the vitreous
subsequently hangs in front of the remaining retina and is seen as floaters in the visual field.
PVD is occasionally associated with vitreous hemorrhage, which typically resolves without
treatment. Patients with diabetes may experience vitreous hemorrhage independent of PVD,
which reflects underlying proliferative retinopathy requiring ophthalmologic referral. Rarely,
PVD is associated with retinal tears and subsequent retinal detachment. Retinal detachment is
typically visible on direct ophthalmoscopy, particularly when the pupil is first dilated. Retinal
detachment is associated with decreased vision in the area of the defect. Thus, patients who
present with complaints of flashes, floaters, and visual field loss (“the three Fs”) require
emergent referral for repair to prevent extension of the defect to the fovea. Patients with
normal acuity should be referred just as emergently, since their prognosis with repair is better
as normal acuity suggests that the fovea is not involved in the detachment.
9. A 27-year-old male is hammering nails at a construction site without eye protection and feels
something strike his right eye. After washing out the eye, he still complains of pain and presents
to the ED. Which of the following is the safest and most accurate modality for locating the
potential foreign body?
A. X-ray
B. Ultrasonography
C. MRI
D. CT scan
pg. 443
Answer D. MRI is the most sensitive test to detect ocular foreign body, but should never be used
when a metallic ocular foreign body is suspected. Plain radiographs are useful but are not as
sensitive as CT scan. Ultrasonography is highly operator dependent. Nuclear medicine scans
have no role in ocular foreign body detection.
10. A 47-year-old female without any past medical history presents with several days of
progressively worsening left eye pain, blurry vision, and redness. Visual acuity in the left eye is
slightly reduced. The patient notes that exposure of the right eye to light causes increased pain
in her left eye. Her left pupil is constricted and minimally reactive to light with perilimbic
conjunctival injection. There is no discharge. Which of the following is the most appropriate
treatment?
A. Topical antibiotics
C. IV mannitol
D. Ocular massage
Answer E. The patient has iritis, which is treated primarily with topical steroids and mydriatics.
Ophthalmologic consultation is generally pursued before the initiation of steroids. The history of
consensual photophobia and physical examination demonstrating perilimbic conjunctival
injection (ciliary flush) is characteristic. Topical antibiotics are used to prevent bacterial
superinfection in corneal abrasions or viral conjunctivitis. Hypertonic eye drops are used for
corneal hydrops (extreme corneal edema). Mannitol therapy for lowering intraocular pressure is
indicated for patients with glaucoma. Ocular massage is indicated for patients with central
retinal artery occlusion to try to dislodge embolus or thrombus and cause it to migrate to a
more distal site in the circulation.
11. Which of the following is true for patients with suspected globe rupture?
pg. 444
Answer D. Globe rupture is a true ophthalmologic emergency, usually requiring operative care.
Antibiotics and tetanus boosters should be given to all patients with suspected globe rupture to
prevent infectious complications. Succinylcholine without pretreatment with nondepolarizing
paralytics can increase intraocular pressure. Tonometry is contraindicated as this will also
increase intraocular pressure. Eye shielding is mandatory to prevent further damage to the
injured eye and to restrict eye movement.
12. A 24-year-old female presents with bilateral eye redness and pain on waking. She states that
her eyes are extremely watery and irritated but denies purulent discharge. She states her
roommate had the same symptoms a few days ago. She does not wear contact lenses. Her visual
acuities are normal. Physical examination demonstrates bilateral conjunctival infection without
discharge. Slit lamp examination is unremarkable. Which of the following is the most likely
etiology?
A. HSV-1
C. Adenovirus
D. S. pneumoniae
E. Pseudomonas aeruginosa
Answer C. The patient likely has uncomplicated viral conjunctivitis, which is most commonly
caused by adenovirus, given the lack of dendrites (HSV), pseudodendrites or facial rash (VZV), or
purulence on physical examination (bacterial). Viral conjunctivitis is extremely contagious and a
sick contact is usually identified. Bacterial conjunctivitis may be difficult to distinguish early in
the course from viral but is much less common. Contact lens use is the major risk factor for
pseudomonal conjunctivitis.
13. Which of the following is the most characteristic finding on funduscopic examination for
central retinal artery occlusion?
pg. 445
Answer A. Choice B is seen in glaucoma, choice C in central retinal vein occlusion, choice D in
vitreous hemorrhage, and choice E in macular degeneration.
14. Which of the following is rightly matched with the cause of visual loss?
Answer E. Optic neuritis usually causes slowly progressive painful visual loss over a few days,
and is associated with an afferent pupillary defect (APD). This is diagnosed with the swinging
flashlight test in which a light is quickly moved from left to right to examine the direct and
consensual response to light. Since the efferent fibers of the affected eye work properly but the
eye can’t detect light (an afferent problem), the affected pupil will constrict when the light is
shined into the unaffected eye. When the light is swung back to the affected eye, it doesn’t
detect the light so the pupil dilates back to its baseline. Along with retinal detachment, vitreous
hemorrhage, posterior uveitis, and central retinal vein occlusion, central retinal artery occlusion
is a painless event that causes monocular vision loss. Vitreous hemorrhage initially results in
floaters but can progress to more significant vision loss depending on the amount of
hemorrhage. Floaters are also common in retinal detachment along with flashes of light
(photopsias) and classic “curtain-like” decreased vision with a field cut.
A. Corneal abrasion
B. Hypopyon
D. Hyphema
E. Corneal ulcer
Answer C. Retrobulbar hemorrhage is a result of ocular trauma that causes pressure on the
posterior portion of the eye. The globe is pushed outward, and proptosis may be seen on
physical examination. Increased pressure in the orbit can compress the central retinal artery or
vein and cause loss of vision. Choices A, B, D, and E are all pathologic processes involving the
pg. 446
anterior portion of the eye and are not usually caused by a retrobulbar hematoma. Secondary
glaucoma may occur as a result of increased overall pressure in the globe, including in the
anterior chamber. Treatment of a retrobulbar hemorrhage involves emergent lateral
canthotomy and drainage of the hematoma out of the temporal border of the globe. Failure to
perform lateral canthotomy for acute retrobulbar hemorrhage may result in irreversible vision
loss in as little as 90 minutes.
18. A 22-year-old female presents with acute onset of right eye discharge 3 hours before
presentation. The discharge reaccumulates almost immediately after wiping it away. She does
not wear contact lenses. Physical examination demonstrates normal visual acuity and copious
greenish-yellow discharge in the right eye. Conjunctival injection and chemosis are prominent.
Which of the following is the most likely cause?
A. S. pneumoniae
B. H. influenzae
C. M. catarrhalis
D. Klebsiella pneumoniae
E. N. gonorrhoeae
pg. 447
Answer E. Hyperacute bacterial conjunctivitis is usually caused by Neisseria species. It is
differentiated from ordinary bacterial conjunctivitis by the rapidity of onset. Populations at risk
include neonates and sexually active adults. Prompt diagnosis is essential because of the rapid
course and the ability of gonococci to invade intact corneal epithelium. Treatment generally
involves systemic and topical antibiotics covering both Neisseria and Chlamydia, as 30% of
patients are coinfected. Choices A, B, and C are common causes of bacterial conjunctivitis in
children. Klebsiella is not a common cause of conjunctivitis.
19. Which of the following physical examination findings is consistent with a relative left-sided
afferent papillary defect when a penlight is swung between the two eyes in a darkened room?
A. The left pupil dilates when light is shined into the left eye.
B. The left pupil constricts when light is shined into the left eye.
C. The left pupil does not respond to light when light is shined into the left eye.
D. The left pupil dilates when light is shined into the right eye.
E. The left pupil constricts when light is shined into the right eye.
Answer A. The “swinging penlight” test is used to diagnose afferent papillary defects. In the test,
a light is swung between the two eyes in a darkened room, spending approximately 1 second
per eye to determine pupillary response. An afferent pupillary defect is present when the
affected eye appears to dilate in response to light. The presence of an afferent pupillary defect
reflects a problem with light perception from the eye. This could reflect a problem anterior to
the retina (e.g., blood in the anterior or posterior chamber), within the retina, or in the optic
nerve. It most commonly reflects optic nerve lesion on the affected size. Whatever the cause,
the pupil of the affected eye will not respond correctly when exposed to light. Since the efferent
pathways (from brain to eye) are not affected, light shined into the unaffected, contralateral eye
results in a normal, constrictive response. Furthermore, since the efferent pathways are normal,
the two pupils are always the same size as each other. However, since the affected eye does not
sense as much light as the unaffected eye, exposing the affected eye to light produces a weaker
constrictive response than light exposure in the normal eye. When a penlight is swung between
the two eyes, this gives the appearance of dilation when the light is swung to the affected eye.
20. A 46-year-old female presents with a chief complaint of painless flashes and floaters. She has
not noted any trouble reading or other changes to her vision. Which of the following
examination findings increases the likelihood of an associated retinal detachment?
pg. 448
D. Decreased visual acuity
Answer E. In a meta-analysis of 17 studies, subjective vision reduction (LR+ = 5) was the most
important symptom associated with retinal detachment among patients with flashes and
floaters. On examination, bedside ultrasound is a simple, useful tool that reveals a thick
membrane that appears to have lifted away from the retina as it “floats” in the vitreous anterior
to the retina. Vitreous hemorrhage or vitreous pigment (unlikely to be diagnosed by an ED
physician) upon slit-lamp examination also greatly increases the likelihood of associated
detachment, even if one is not seen. Vitreous pigment is unfortunately named, since its
presence reflects retinal pigment within the vitreous (thus, vitreous pigment). A progressive
visual field deficit in the setting of flashes and floaters also indicates retinal detachment until
proven otherwise.
21. Which of the following is the major complication of ischemic central retinal vein occlusion
(CRVO)?
A. Conjunctivitis
B. Iritis
C. Glaucoma
D. Lens dislocation
E. Corneal ulcer
Answer C. CRVO causes backup of blood flow into the eye and carries the potential for increased
intraocular pressure, eventually leading to glaucoma. The classic history is acute or subacute
painless loss of vision, although pain may occur in some cases. Risk factors include hypertension,
diabetes, thrombophilia, funduscopic examination reveals disc edema with tortuous veins and
retinal hemorrhages. ED management is supportive in conjunction with ophthalmologist
consultation. Choices A, B, D, and E may be caused by trauma or anterior eye disorders.
22. Which of the following is the leading cause of legal blindness in the United States?
A. Cataracts
B. Glaucoma
C. Diabetic retinopathy
D. Macular degeneration
pg. 449
E. Retinal detachment
Answer D. Age-related macular degeneration is the most common cause of blindness in the
industrialized world. It occurs primarily because of retinal damage from unknown causes.
Almost one fourth of all Americans older than 90 are affected by macular degeneration.
23. A 19-year-old female presents with worsening unilateral purulent discharge from her left eye
that started the day before presentation. She has blurry vision that resolves when wiping the
drainage out of her eye. She also notes nausea and intermittent fevers for the past few days,
lower abdominal pain, and dysuria. Slit lamp examination demonstrates purulent discharge, but
no corneal or anterior chamber abnormalities. Which of the following is the most appropriate
management strategy?
Answer E. The patient likely has gonococcal conjunctivitis. The patient has signs and symptoms
of pelvic inflammatory disease and urethritis and systemic antibiotic therapy is warranted.
Treatment should include ceftriaxone, doxycycline or azithromycin, topical antibiotics, and
saline irrigation. Admission is not absolute—however, the presence of nausea, vomiting, or
fevers generally warrants in-hospital management.
A. It is generally painless.
Answer C. Inflammation of the iris caused by trauma causes constant pain and photophobia,
especially consensual photophobia (light exposure to the unaffected eye causes pain in the
pg. 450
affected eye due to consensual constriction). Long-acting cycloplegics and steroids are the
mainstay of treatment. The pupil is reactive and constricted, and ciliary flush (conjunctival
injection in a circular rim around the limbus) is prominent. Resolution should occur within 1
week.
A. It is usually painful.
Answer C. Floaters with or without painless visual loss is the usual clinical presentation of retinal
detachment. Direct funduscopy and visual acuities may be completely normal if the detachment
is peripheral or small. The elevated retina will appear out of focus and as a hazy gray membrane.
Ophthalmologic consultation should be emergent since reattachment is often successful if
performed early in the disease course.
26. Which of the following is the most appropriate outpatient management for mechanical
corneal abrasions?
A. Eye patching
B. Topical anesthetic
C. Topical steroids
D. Topical antibiotics
Answer D. Corneal abrasions should be treated with prophylactic topical antibiotics. Short-term
cycloplegics may also be used to reduce the ciliary spasm associated with many abrasions. Eye
patching has been shown to increase infection rates. Topical anesthetics have been thought to
impede corneal healing if used beyond the acute setting. However, recent studies have
contradicted that conventional wisdom. Still, at the time of this writing, topical anesthetics are
not available for home use. Topical steroids are indicated only in cases of iritis and are
absolutely contraindicated when the possibility of herpetic infection exists. Topical saline
pg. 451
solution may be used in corneal abrasions as a remoisturizing medium, but does not reduce
superinfection rates or improve healing.
27. A 26-year-old female presents with eye pain after she was hit in the face with a softball (Fig.
below). Which of the following is the best therapeutic regimen?
A. Topical antibiotics
B. Topical steroids
C. Eye patching
D. A and C
E. A, B, and C
Answer A. The patient has a corneal abrasion, indicated by the fluorescein uptake in the cornea.
Management of corneal abrasions is with topical antibiotics to prevent infection and
occasionally cycloplegics to help reduce painful iris contraction. Steroids should never be given
in patients with corneal abrasions—use of topical steroids should be contingent on
ophthalmologic recommendation. Eye patching was once thought to be effective for corneal
abrasions and is now thought to increasethe possibility of infection.
28. Which of the following indicates a likely globe injury in a patient with an eyelid laceration?
pg. 452
B. Horizontal laceration extending the length of the eyelid
E. Stellate lacerations
Answer C. Eyelids do not contain subcutaneous fat—the presence of fat indicates likely globe
injury. Patients with globe injuries should be seen emergently by the ophthalmologist. ED
management involves broad-spectrum antibiotics, eye shielding, avoidance of recumbent
position or Valsalva maneuvers, tetanus immunization, and treatment of nausea and vomiting.
Choices A, B, D, and E are potential indications for ophthalmologic or plastic surgical repair to
prevent significant cosmetic defects.
29. A 50-year-old male with a history of diabetes and hypertension presents with new-onset
vertical diplopia. His examination reveals normal pupils, mild right sided ptosis, and slight
inferior deviation of his right eye at rest. At times the right eye appears to “get stuck” after
being abducted in downward gaze. Which of the following is most likely affected?
A. Cranial nerve II
C. Cranial nerve IV
D. Cranial nerve VI
E. Occipital lobe
Answer B. The vignette describes a partial oculomotor nerve palsy (cranial nerve III). The
oculomotor nerve controls the levator palpebrae muscle (which lifts the eyelid), as well as the
superior and inferior rectus muscles (which elevate and depress the eye), the medial rectus
(which adducts the eye) and inferior oblique muscles (which elevate the eye in adduction and
laterally rotate the eye in abduction), and the pupillary constrictor. Most patients presenting to
the emergency room with CN III palsies have partial, pupil-sparing palsies brought about by
small vessel ischemia due to poorly controlled or long-standing diabetes, hypertension, and
hyperlipidemia. The pupillary constrictor muscles are spared because the nerve fibers to the
pupillary constrictor lie in the outer portion of the nerve sheath so they are less affected by
inadequate blood flow. Patients with a CN III palsy due to microvascular ischemia tend to have a
good prognosis, as the lesions often heal in 8 to 12 weeks without therapy. Antiplatelet
medications such as aspirin are typically started after an initial event.
pg. 453
30. Which of the following is true regarding topical ophthalmic antibiotics?
Answer C. Ocular antibiotics may be given by ointment or drops. Either form can cause systemic
absorption and side effects. Drops usually require more frequent dosing due to a shorter
duration of action. Ointments are easier to apply in pediatric patients for this reason. Antibiotics
are indicated in most cases of conjunctivitis, as early presentations of bacterial cases may be
clinically indistinguishable from viral ones.
31. An 18-year-old male presents with fever and left periorbital pain and swelling as shown in
Figure below. Which of the following additional tests is indicated at this time?
B. Orbital CT scan
pg. 454
Answer B. Fever and periorbital edema should prompt differentiation between periorbital and
orbital cellulitis. Patients with proptosis, oculomotor dysfunction, and pain on oculomotor
exercises are more likely to have orbital cellulitis. Staphylococci and streptococci are the most
common causes. Diagnosis is confirmed by orbital CT scan. Treatment of orbital cellulitis is with
intravenous antibiotics and possibly surgical drainage. Complications include blindness, death,
and intracranial extension. Brain MRI may detect complications of orbital cellulitis but provides
little advantage in the emergent evaluation. ESR is an extremely nonspecific laboratory test,
which is not useful in the diagnosis of most emergent conditions, except temporal arteritis,
septic arthritis, and osteomyelitis. Lumbar puncture and CSF analysis are not indicated in this
patient in the absence of headache or stiff neck. Slit lamp examination of the affected side is
likely to be impossible given the degree of periorbital edema and not useful as a corneal or
anterior chamber process is not the primary source of pathology.
32. A 10-year-old presents with findings concerning for orbital cellulitis. CT is most likely to
demonstrate which concurrent finding?
A. Dacryocystitis
B. Mastoiditis
D. Deviated septum
E. Ethmoid sinusitis
Answer E. Though most cases of rhinitis and sinusitis do not result in orbital cellulitis, nearly all
cases of orbital cellulitis are preceded by rhinosinusitis. Specifically, ethmoid sinusitis is the most
common precursor because the lamina papyracea, which separates the ethmoid sinuses from
the orbit, is a paper-thin bone with many natural fenestrations allowing fluid in the sinus to pass
easily into the orbit. Dacryocystitis, orbital fractures, orbital foreign bodies, and even dental
infections can all lead to orbital cellulitis but ethmoid sinusitis is far and away the leading cause.
pg. 455
Answer A. Pain with extraocular movements, decrease in visual acuity, proptosis, and
ophthalmoplegia more commonly occur in orbital cellulitis than in periorbital cellulitis. The
majority of cases of orbital cellulitis result from direct spread of adjacent infections in the
paranasal sinuses (such as the ethmoid sinusitis). In contrast, sinusitis precedes a relative
minority of periorbital cellulitis cases. Most cases are caused by local inoculation (insect bites,
trauma), or by local extension from other sources (dacryocystitis). Aspergillus species can cause
a chronic orbital cellulitis lasting from weeks to months. Orbital cellulitis may cause blindness,
and the infection can extend inside the cranium to involve the dural sinuses and meninges. In
the absence of trauma, progression of periorbital cellulitis to orbital cellulitis is rare, even in
untreated cases, and it is very rare for either infection to involve the globe.
34. A 65-year-old male presents with sudden, painful loss of vision in his right eye. His visual
acuity is markedly decreased in the affected eye. Which of the following is the most likely cause
of his symptoms?
D. Retinal detachment
E. Vitreous hemorrhage
Answer A. Of all the choices, acute angle closure glaucoma is most likely to cause painful loss of
vision. All the other choices are much more likely to cause painless (rather than painful) loss of
vision.
35. A 62-year-old male presents with right eyelid swelling and crusting. He reports no pain or
redness in the eye itself. Physical examination of the eyelid is shown in Figure 9-5. Which of the
following is the most appropriate therapy?
A. Topical erythromycin
B. Topical prednisolone
C. Topical proparacaine
D. Intravenous ceftriaxone
E. Intravenous acetazolamide
pg. 456
Answer A. The patient has blepharitis, as seen by the crusting and edema of the upper eyelid.
Staphylococcal infection has been implicated but the complete pathophysiology is not known.
The condition is chronic and there are multiple components to treatment. Eyelid hygiene is very
important, as patients should clean the lids with a gentle soap, apply warm compresses,
gentlymassage the lids, and apply topical erythromycin ointment. Topical steroids should never
be prescribed by the emergency physician (EP) without ophthalmologic consultation. Topical
anesthetics (such as proparacaine) should never be prescribed to patients, as they will retard
corneal healing. Intravenous ceftriaxone is used in patients with hyperacute bacterial
conjunctivitis due to gonococcus. This patient has no evidence of conjunctival involvement on
physical examination. Acetazolamide is used in patients with acute angle closure glaucoma, but
the absence of eye pain, headache, and corneal or conjunctival abnormality effectively rules this
out.
Answer E. Rust rings are the result of iron-containing foreign bodies leaving a residue on the
cornea. Patients should see the ophthalmologist within 48 hours, as the ring will migrate to
more superficial corneal layers over time, allowing for easier removal. Topical steroids have no
pg. 457
role in the management of rust rings or ocular foreign bodies. Metallic foreign body evaluation
of the eye with MRI is absolutely contraindicated.
37. A 45-year-old female presents with a red eye on waking. She is otherwise completely
asymptomatic. She denies any past medical history and takes no medications. The eye is shown
in the Figure below. Physical examination is otherwise normal. Which of the following is the
most appropriate next step in management?
C. Topical antihistamines
D. Topical antibiotics
E. No specific therapy
pg. 458
C. May require analgesic treatment with aspirin
Answer D. Hyphema refers to the presence of blood in the anterior chamber of the eye. The eye
may rebleed after the initial traumatic hyphema, especially in patients with severe myopia or
with large hyphemas. General treatment of traumatic hyphema involves analgesia and
antiemetics, head elevation, restriction of eye movement, and avoidance of therapies that may
cause bleeding. Ophthalmologic consultation should be sought emergently, especially for large
hyphemas, as specific management with topical steroids or operative drainage may be
instituted. There is usually no afferent pupillary defect.
39. Which of the following leads to the most severe acute ocular injury?
A. Acid
B. Alkali
C. Ultraviolet light
D. Hand soap
E. Cigarette ashes
Answer B. Alkali causes liquefaction necrosis, which allows the alkali to spread to deeper tissues
and cause further necrosis. Acid causes coagulation necrosis, which hardens the tissue and limits
spread of the acid to deeper tissues. Chronic exposure to ultraviolet light causes long-term
ocular injury and eventual blindness but acute symptoms are usually mild. Most commercially
available soaps are generally nontoxic and cause only local conjunctival irritation. Cigarette
ashes may cause superficial corneal burns, but the duration of exposure is rarely long enough to
produce serious injury.
40. A 32-year-old female who wears contact lenses presents with eye pain and redness (Fig.
below). Which of the following is the correct diagnosis?
C. Corneal ulcer
D. Pterygium
pg. 459
E. Central retinal artery occlusion
Answer C. The patient has a small whitish patch in the cornea consistent with a corneal ulcer.
Management includes topical antibiotics and emergent ophthalmologic consultation. Acute
angle closure glaucoma would not be likely in a 32-year-old female without any prior history or
in the absence of characteristic ocular findings such as cloudy cornea, fixed and mid-dilated
pupil, or loss of vision. Central retinal vessel occlusion is signified more by acute painless loss of
vision rather than eye pain and redness. Pterygium is a chronic, benign growth in the
conjunctiva of little clinical significance except cosmesis. Pterygia can invade the line of vision
and needs to be corrected in those cases.
41. Which of the following is indicated for treatment of acute angle closure glaucoma?
A. Topical cycloplegics
B. Topical antivirals
C. Aspirin
D. Acetazolamide
E. Lateral canthotomy
Answer D. Acute angle closure glaucoma is caused by increased pressure in the anterior
chamber due to decreased outflow of aqueous humor. Acetazolamide can increase the
excretion of aqueous humor and is indicated as one of the medical therapies, along with
pilocarpine, timolol, and antiemetics. Cycloplegics inhibit ciliary muscle contraction (which limits
miosis), and are contraindicated in glaucoma, as they further inhibit aqueous humor outflow
pg. 460
from the anterior chamber. Aspirin and antivirals play no role in the management of glaucoma.
Lateral canthotomy is used to treat retrobulbar hematoma. Definitive therapy for acute angle
closure glaucoma is surgical and emergent ophthalmologic consultation is necessary.
42. Which of the following conditions is the most common cause of lens dislocation?
A. Tertiary syphilis
B. Homocystinuria
C. Marfan syndrome
D. Trauma
E. Ehlers–Danlos syndrome
Answer D. Although A, B, C, and E all confer an increased risk of lenticular dislocation, trauma as
a whole is the most common cause. Marfan syndrome is the most common cause of inherited
lenticular dislocation. Glaucoma and retinal detachment may complicate lens dislocation and
must be managed in concert with an ophthalmologist. Iridectomy or lensectomy may be
indicated in some cases.
43. A 55-year-old male with diabetes presents with painful vision loss in his left eye, which
occurred when he sat down to watch a movie in the theater. His acuity is markedly reduced in
the left eye and his left pupil is poorly reactive to light and measures 4 mm. Which of the
following is true regarding this patient’s condition?
Answer C. The patient has acute angle closure glaucoma. The history of ambient light suddenly
decreasing and forcing rapid pupillary dilatation is classic. Patients with glaucoma may often
complain of headache, nausea, vomiting, and abdominal pain without any specific ocular
symptoms. Bilateral shallow anterior chambers are the specific anatomic abnormality
predisposing to closure of aqueous outflow—pilocarpine is therefore administered to both eyes
(either eye has an equal chance of being affected by the acute angle closure). Boxcar retinal
venules are characteristically seen in central retinal artery occlusion. Although emergent
pg. 461
ophthalmologic consultation is required, all modalities for treatment of a severe glaucoma
attack should be initiated early—pilocarpine, timolol, apraclonidine, prednisolone,
acetazolamide, and mannitol. Intravenous medications may be withheld if intraocular pressures
are not severely elevated. Sedatives and antiemetics are used as needed. Ocular massage is
absolutely contraindicated, as this will increase intraocular pressure.
44. A 32-year-old male presents with eye pain and redness. Slit lamp examination is shown in
Figure below. Which of the following is the most appropriate next step in management?
A. Topical prednisolone
B. Valacyclovir PO
C. Acetazolamide IV
D. Erythromycin PO
E. Ceftriaxone IV
Answer B. The slit lamp examination demonstrates dendritic lesions with fluorescein uptake
characteristic of herpes simplex keratitis. The dendritic lesions may scar, and ocular HSV is a
common cause of corneal blindness in the United States. Management involves oral and/or
topical antivirals and ophthalmologic consultation to assess for surgical management. Topical
steroids are absolutely contraindicated as they may cause worsening of the corneal epithelial
defect. Acetazolamide is used to increase aqueous humor excretion as part of noninvasive
temporizing therapies for acute glaucoma attacks. Erythromycin is used to treat corneal ulcers
or prevent infections from occurring in patients with corneal abrasions. Ceftriaxone is used in
patients with hyperacute bacterial conjunctivitis, which is usually due to gonococcal infection.
pg. 462
45. A 65-year-old female with hypertension, atrial fibrillation, and type II diabetes presents with
acute painless vision loss in her right eye. On examination, visual acuity is markedly decreased
and the patient has a striking afferent pupillary defect. She also has a right-sided carotid bruit.
Which of the following is the most appropriate next step in management?
A. Lateral canthotomy
B. Globe massage
C. Aspirin
D. Heparin
Answer B. The patient has central retinal artery occlusion (CRAO), probably caused by a
thromboembolus from atrial fibrillation or carotid atherosclerosis. The conditions listed in her
past medical history are all risk factors for CRAO. Intermittent globe massage is indicated to
increase ocular carbon dioxide content, which leads to vasodilation of the retinal arteries.
Lateral canthotomy is used to treat retrobulbar hematoma and has no role in management
here. Aspirin, heparin, and TPA have not been shown to improve outcomes in CRAO.
46. A 55-year-old male presents with a “bump” on his upper eyelid (Fig. below). Which of the
following is the most appropriate next step in management?
A. Intravenous antibiotics
B. Eyelid culture
C. Warm compresses
E. Topical antivirals
pg. 463
Answer C. The superior lid contains a hordeolum (or stye) with surrounding blepharitis. A
hordeolum refers to localized infection of a meibomian gland in the lid, usually because of S.
aureus. Treatment involves warm compresses, with topical erythromycin ointment applied to
the lid if there is a surrounding blepharitis. Systemic antibiotics are rarely indicated in patients
with hordeolum. Eyelid culture is unlikely to be helpful or easy to obtain. Aspiration of the area
is strictly contraindicated given the proximity to the globe. Topical antivirals are indicated only in
patients with herpes keratitis. The lower lid has a chalazion, which is a subacute, granulomatous
lesion also treated with warm compresses.
47. A 16-year-old male is brought to the emergency room by his parents after being struck in the
face by a baseball while playing for his high school team. He complained of immediate eye pain
and vision loss. Which of the following findings is most concerning for an underlying globe
rupture?
A. Hyphema
B. Subconjunctival hemorrhage
E. Bloody chemosis
Answer C. Clinicians should maintain a high degree of suspicion for an open globe rupture when
patients present with marked decreased visual acuity after orbital trauma, which is a common
manifestation of globe rupture. Hyphema and subconjunctival hemorrhage most commonly
occur without concomitant globe rupture but are often present in the setting of globe rupture
as well. While subconjunctival hemorrhage does not typically affect visual acuity, patients can
experience profound subconjunctival hemorrhage, termed bullous subconjunctival hemorrhage,
in which the degree of subconjunctival bleeding and subsequent conjunctival swelling is so
severe that it prevents direct visual inspection of the underlying ocular structures and
completely impedes vision. Patients with hyphema may have normal vision or may only have
light perception, also depending on the severity of bleeding. However, when such patients are
placed in a seated position, the blood in the anterior chamber slowly forms a layer at the
bottom of the chamber, and vision often improves. Blowout fractures are common in patients
with globe rupture but most commonly occur without associated globe rupture or changes in
acuity. Bloody chemosis refers to hemorrhagic swelling to the conjunctiva, which is what results
in severe subconjunctival hemorrhage, as detailed above.
48. A 22-year-old male contact-lens wearer presents to the ED with a chief complaint of right
eye pain, redness, light sensitivity, and a foreign body sensation. Examination reveals no foreign
pg. 464
body but a small, peripheral corneal ulcer and a small amount of yellowish drainage. In addition
to removing the contact lenses, which of the following is the best next step?
D. Trimethoprim–polymyxin B ointment
Answer C. Corneal ulcers in the setting of contact lens use are frequently caused by
Pseudomonas infections and require aggressive treatment with antipseudomonal antibiotics
such as ciprofloxacin. Erythromycin and trimethoprim–polymyxin B are inadequate to treat
Pseudomonas infections. While irrigation may help transiently reduce surface bacterial counts,
this patient needs antibiotic treatment. Patching corneal ulcers should be avoided, particularly
when an infection is present as bacteria may thrive in the warm, dark environment created by
patching.
pg. 465
Obstetrics and gynecology
Emergency
➢ Chapter includes:
Obstetrics and gynecology MCQ’s
(Exception trauma in pregnancy in trauma chapter)
pg. 466
1. A 34-year-old female presents to the ED with increasing low abdominal pain. She was seen by
a colleague a week ago and diagnosed with an early ectopic pregnancy in her left fallopian tube.
Her OB was consulted at that time and the patient was started on methotrexate therapy. What
is the most likely cause of her abdominal pain?
B. Tubal rupture
C. Appendicitis
2. A concerned mother brings her 15-year-old daughter to the ED with a chief complaint of
irregular vaginal bleeding. The patient experienced menarche at age 13 and has never had
regular periods. Recently, the bleeding has been heavier and more irregular than normal. She
reports no history of easy bruising and has no petechiae on examination. Her hemoglobin is 11
g/dL. Which of the following is the most likely cause of her symptoms?
A. Hyperthyroidism
B. Anovulation
C. Endometriosis
Answer B. This patient has dysfunctional uterine bleeding (DUB), which is defined as excessive,
prolonged, or erratic uterine bleeding that is not related to an underlying anatomic uterine
pg. 467
defect or systemic disease. Therefore, DUB is a diagnosis of exclusion. However, anovulation is
by far the most common cause of irregular vaginal bleeding in an adolescent. Although
anovulatory menstrual cycles are most common in the first 2 to 3 years after menarche, it may
take up to 6 years before most cycles become ovulatory. Oral contraceptive pills are very
effective for the management of DUB. Nearly any regimen can be used, and the most common
involves combination oral contraceptives (containing both estrogen and progesterone) with at
least 30 to 35 mg of ethinyl estradiol. The pills are initially used four times daily in women with
more extensive bleeding (and concomitantly low hemoglobin levels) and are gradually tapered
by one pill every 3 days until only one pill is being used on a daily basis. Antiemetics may be
needed due to the nausea that is a frequent side effect of high dose estrogen therapy. In
addition, iron supplementation should be used to boost red blood cell production.
3. A 38-year-old female who has a history of “infertility” presents to the ED with low abdominal
pain. She recently became pregnant after using in vitro fertilization. Her physical examination
reveals mild low abdominal tenderness, no adnexal mass, and a closed cervical os with a normal
bimanual examination. An ultrasonography demonstrates a live intrauterine gestation
appropriately sized for dates but also reveals a slightly enlarged left ovary. Which of the
following is the approximate risk of a heterotopic pregnancy in this patient?
A. 0.25%
B. 1%
C. 10%
D. 25%
E. 50%
pg. 468
A. Abruptio placentae
C. Bacterial vaginitis
D. Pyelonephritis
E. Chorioamnionitis
Answer E. This patient experienced preterm PPROM 2 days before presentation, which
dramatically increased her risk for developing chorioamnionitis. Chorioamnionitis is an
intra-amniotic infection that is most commonly due to vaginal flora that has gained entry to the
amniotic cavity. Risk factors include young age, low socioeconomic status, nulliparity, extended
duration of labor and ruptured membranes, multiple vaginal examinations, and pre-existing
infections of the lower genital tract. The typical causative organisms are group B streptococci
and E. coli and the most widely used antibiotic regimen is a combination of ampicillin and
gentamicin. Ampicillin may be replaced with vancomycin, erythromycin, or clindamycin in
penicillin-allergic patients.
5. A 26-year-old G2P2 presents to the ED 3 days after spontaneous vaginal delivery of a healthy
male infant with a chief complaint of crampy low abdominal pain and a foul-smelling vaginal
discharge. On examination, she has a fever of 102°F and a tender uterus on bimanual pelvic
examination. Which of the following is true?
A. This condition is more common after vaginal delivery than cesarean section.
Answer D. This patient has endometritis, which is the most common puerperal infection. The
primary risk factor for endometritis is cesarean section, although young age, low socioeconomic
status, prolonged stage 2 of labor, prolonged ruptured membranes, and multiple vaginal
examinations are also risk factors. Patients typically present 2 to 3 days after delivery with fever,
abdominal pain, and foul smelling lochia. Infections are polymicrobial and are most commonly
caused by gram-negative enteric pathogens as well as Bacteroides and Prevotella species.
Chlamydia is rarely responsible and may cause late-onset puerperal infection.
pg. 469
A. The fetus should be delivered within 5 minutes of maternal cardiac arrest.
C. It should only be performed if the fetal age is determined to be >20 weeks of gestation.
D. A low horizontal abdominal incision affords the best opportunity for fetal recovery.
Answer A. Perimortem cesarean section should be performed in all pregnant patients suffering
traumatic cardiac arrest with a fetus >24 weeks of gestational age. Maternal resuscitation
always takes precedence over the fetus, but once cardiac arrest has occurred, an immediate
decision to undergo perimortem cesarean section must be made. Seventy percent of fetuses will
survive if delivered within 5 minutes of maternal arrest. None will survive after 25 minutes.
Consent is unnecessary and should not be sought before the procedure. If the fetal gestational
age is unknown, cesarean delivery should be performed if the fundal height exceeds the
umbilicus. To perform the procedure, a large midline vertical incision (“classic” midline incision)
is made from the subxiphoid process to the symphysis pubis. If an anterior placenta is
encountered upon entering the uterus, it should be incised to reach the fetus, as bleeding can
be addressed after the procedure. Ideally, verification of fetal viability through fetal heart tones
would be documented before the procedure, but absence of this information is not a
contraindication and no time should be wasted attempting to document fetal viability.
7. Which of the following is the most appropriate initial antihypertensive medication for
hypertensive crises in pregnant patients?
A. Nitroprusside
B. Nitroglycerin
C. Clonidine
D. Labetalol
E. Hydrochlorothiazide
Answer D. Intravenous medications are most appropriate for rapid control of BP. Labetalol is
safe and effective during pregnancy and produces predictable control of BP during hypertensive
crises such as eclampsia. Hydralazine is an antihypertensive medicine that is also commonly
used. Nitroprusside also provides excellent BP control but has not been proven safe during
pregnancy. Nitroglycerin causes preload reduction but has little effect on afterload reduction,
which is the more important intended mechanism of action for antihypertensive agents during
crises. Clonidine and hydrochlorothiazide are oral medications that have a limited role in acute
management of BP during hypertensive crises due to unreliable absorption and efficacy.
pg. 470
8. Of the following, which is the strongest risk factor for an ectopic pregnancy?
D. Prior C-section
E. Oral contraceptives
Answer A. History of prior ectopic pregnancy is the strongest risk factor for current ectopic
pregnancy. The most common risk factor is PID. The sequelae of PID account for approximately
half of all cases of ectopic pregnancy. Although IUD use increases the relative risk of ectopic
versus intrauterine pregnancy, it decreases the overall risk of all pregnancy significantly.
C-section increases the risk of placenta previa, but not of ectopic pregnancy. Oral contraceptive
use may reduce the symptoms of PID (thus predisposing to more untreated PID), but overall
pregnancy rate (including ectopic) is reduced.
9. Which of the following most commonly complicates normal labor and delivery?
A. Face presentation
B. Breech presentation
C. Shoulder dystocia
D. Brow presentation
pg. 471
10. A 27-year-old female, G2P1, at 5 weeks by dates, presents with abdominal pain. She has had
minor “spotting” but no frank vaginal bleeding, her internal cervical os is closed, and her serum
β-hCG is 750 mIU/mL. Which of the following is true?
A. She should be diagnosed with a threatened abortion and asked to return in 2 days for a
repeat quantitative β-hCG level.
C. She should receive an ultrasound to assess uterine contents and the adnexa.
D. She should be diagnosed with an early pregnancy versus an ectopic pregnancy and told to
return in 2 days for a repeat quantitative β-hCG level and an ultrasound.
Answer C. Although this patient may have an early normal pregnancy and a threatened
abortion, she is at risk for an ectopic pregnancy. Because the patient’s serum β-hCG level is
below the discriminatory threshold (the level at which a normal pregnancy can be detected by
ultrasonography), the main reason for obtaining an ultrasonograph is to search for an ectopic
pregnancy. Approximately 50% of women with an ectopic pregnancy have a β-hCG level <3,000
mIU/mL and symptomatic patients with a β-hCG level <1,000 mIU/mL are four times more likely
to have an ectopic pregnancy than those patients with higher β-hCG levels. If the
ultrasonograph is nondiagnostic and the patient is otherwise stable, she can be discharged with
instructions to follow-up with her OB in 2 days for a serial β-hCG level. In that case, her diagnosis
remains unclear, so she could be given a diagnosis of “possible ectopic pregnancy versus
threatened abortion.” Once her β-hCG level has risen beyond the discriminatory zone, her
ultrasonogram should be repeated to evaluate for the presence of a normal pregnancy.
11. A 38-year-old mildly obese primigravida at 34 weeks’ gestation presents with a chief
complaint of “swollen legs” and abdominal pain. Her blood pressure is 170/100 and she has 3+
protein on urine dipstick. After giving her magnesium for prophylaxis of her seizures and
hydralazine for blood pressure control, the nurse tells you that her urine output seems a bit low,
and asks you what you want to do about her significant edema. The next best step in
management is:
A. Furosemide 40 mg IV push
C. Hydrochlorothiazide 25 mg orally
pg. 472
Answer B. Pre-eclampsia is associated with vasospasm, reduced renal blood flow and glomerular
filtration rate, and increased total body water resulting in edema. However, hypovolemia results
in decreased uteroplacental blood flow and possible fetal injury. Diuretics and hyperosmotic
agents should, therefore, never be used in the setting of pre-eclampsia. Although volume
expanders such as albumin may sound like a good idea, they do not reverse vasospasm or
improve uteroplacental blood flow. Instead, maintenance fluids should be given although
recommendations between authors vary. Due to the risk of pulmonary edema and the inability
of intravenous fluids to reverse vasospasm, however, aggressive large fluid boluses should also
be avoided. One author recommends 5% dextrose in lactated Ringers solution with close
monitoring of urine output, which is maintained at approximately 30 mL/hour. Excessive urine
output may indicate fluid overload, placing patients at risk for pulmonary edema.
A. Midline episiotomy
B. McRoberts maneuver
C. Increased traction
D. Rubin maneuver
Answer B. Shoulder dystocia occurs when further progression of fetal delivery is halted by
impaction of the fetal shoulders within the maternal pelvis. Although it is more common in
diabetic mothers with infants weighing >4,000 g, more than half the cases involve infants <4,000
g and without other risk factors. Rapid recognition and treatment is critical because of an
increased risk of fetal hypoxia and irreversible neurologic damage. Other injuries that
complicate shoulder dystocia include brachial plexus injuries and humerus and clavicular
fractures. With the exception of increased traction, all of the maneuvers listed may be used to
help relieve shoulder dystocia. Most experts advocate using the least invasive maneuvers first.
The best first step is the McRoberts maneuver in concert with suprapubic pressure. The
McRoberts maneuver involves hyperflexion of the maternal hips (placing the maternal knees up
to the chest), which moves the symphysis pubis over the fetal anterior shoulder. This is done in
conjunction with moderate suprapubic pressure to push the fetal anterior shoulder below the
symphysis. If this fails, more invasive maneuvers can be used such as the Rubin or Wood
corkscrew maneuvers. The Rubin involves pushing the posterior fetal shoulder toward the fetal
chest by placing a hand inside the vagina. The Wood corkscrew maneuver involves rotating the
fetus 180 degrees (preferably clockwise) in an attempt to free the shoulders. In general, a liberal
median episiotomy creates more space to allow the posterior shoulder to pass but does not, by
pg. 473
itself, relieve dystocia, and it increases maternal morbidity. If all of these efforts fail, the
posterior arm can be grasped, placed on the fetal chest, and swept over the face and out of the
vagina. This maneuver (Barnum maneuver) may result in fractures or brachial plexus injury.
13. A 22-year-old female presents with a chief complaint of a painful vaginal lump and vulvar
pain while walking (Fig. below). She denies any vaginal discharge, fevers, or abdominal pain. Her
urine human chronic gonadotropin (β-hCG) is negative. Which of the following is true?
C. When not infected, this gland is palpable at the 5 o’clock and 7 o’clock positions around the
vaginal introitus.
D. A Word catheter should be placed after incision and drainage and left in place for 6 to 8
weeks.
E. If left untreated, ascending cystitis, pyelonephritis, and sepsis occur in 35% of patients.
Answer D. This patient has a Bartholin gland abscess, which is an infection of fluid that has
accumulated in the gland. Healthy Bartholin glands are located at 5 o’clock and 7 o’clock
positions around the vaginal introitus and are not palpable. Nonspecific inflammation or trauma
may obstruct the glandular duct, however, resulting in accumulation of glandular fluid inside the
gland and a Bartholin gland cyst. Asymptomatic cysts in young women do not require treatment.
Older women should have the cyst excised by a specialist in order to search for glandular
adenocarcinoma. All patients with an abscess should have incision and drainage followed by the
placement of a Word catheter. The catheter is left in place for 6 to 8 weeks to allow formation
pg. 474
of a fistulous tract, which enables ongoing drainage and prevents recurrence. Bartholin gland
abscesses are almost always caused by polymicrobial vaginal flora. Antibiotics are unnecessary
unless there is an associated cellulitis or unless sexually transmitted organisms are suspected.
Cultures of the abscess can be obtained if there is concern about the etiology.
A. Patients diagnosed with threatened abortion should not receive anti-D immunoglobulin
(RhoGAM) because the antibody may provoke an immune response to the live fetus.
C. Patients diagnosed with threatened abortion should be placed on bedrest restrictions until
the bleeding resolves.
D. All patients who present to the ED with fetal or placental tissue and resolution of vaginal
bleeding can be diagnosed with a complete abortion and discharged.
Answer E. All Rh-negative pregnant patients with first-trimester vaginal bleeding should be given
RhoGAM. The dose in the first trimester is 50 μg, whereas the dose after the first trimester is
300 μg. RhoGAM has not been shown to cause fetal harm. The classic teaching is that 50% of
patients diagnosed with a threatened abortion progress to spontaneous miscarriage. However,
once a fetal heartbeat is identified on ultrasonograph, only 15% of such women will progress to
spontaneous miscarriage whereas the remainder will carry the pregnancy normally to term.
Patients with threatened abortion should be advised to carry out their normal activities
although patients are often advised to avoid tampons, intercourse, and douching to prevent
infection. Patients frequently confuse blood clots with tissue and even in the presence of
laboratory confirmation of products of conception, a diagnosis of complete abortion is
ill-advised. Only if a complete gestational sac or fetus is present should a diagnosis of complete
abortion be considered. Otherwise, an ultrasound should be performed to determine if retained
products of conception are present. Finally, approximately 80% of patients with first-trimester
spontaneous abortion will complete the abortion without intervention. The classic teaching is
that all patients diagnosed with an anembryonic pregnancy (or blighted ovum), intrauterine
fetal demise, missed abortion, or incomplete abortion require surgical evacuation. However,
current research demonstrates that the volume of intrauterine contents is the best predictor of
the need for surgical evacuation and most women do not require intervention.
15. A pregnant patient is receiving a magnesium infusion for pre-eclampsia. Which of the
following findings is the earliest indicator of significant toxicity?
pg. 475
A. Diaphoresis
D. Sinus tachycardia
E. Visual floaters
C. Uterine atony
D. Placenta accrete
E. Uterine rupture
Answer C. Uterine atony is the most common cause of immediate postpartum hemorrhage
(defined as blood loss that occurs within the first 24 hours of delivery), as it is responsible for
approximately 50% of cases. The risk factors for uterine atony are multiparity, prolonged labor,
excessive uterine manipulation, and general anesthesia with halogenated anesthetic agents.
Management involves abdominal or bimanual uterine massage as well as the use of oxytocic
agents such as oxytocin, methylergonovine maleate or ergonovine maleate, or carboprost
tromethamine. Tears of the maternal birth canal may also result in significant hemorrhage and
are the second most common cause (as a group). Retained placental tissue accounts for roughly
10% of immediate postpartum hemorrhage. If uterine massage and oxytocic agents fail to
control bleeding thought to be due to uterine atony, a meticulous search should be conducted
for maternal birth trauma or retained placental tissue.
pg. 476
17. A 38-year-old pregnant woman presents to the ED with vaginal bleeding and abdominal pain
radiating to the back. She is 25 weeks by dates and has had an uncomplicated pregnancy with
routine prenatal care. She admits to smoking one pack of cigarettes per day and otherwise has
no significant history. Her uterus is firm and tender on examination and there is bright red blood
oozing from the cervical os. The most likely diagnosis is:
A. Appendicitis
B. Placenta previa
C. Vasa previa
D. Fibroid degeneration
E. Abruptio placentae
18. The daughter of an 82-year-old female brings her mother in with a chief complaint of a
“foreign body” in her vagina (see Fig. below). The patient had reported the uncomfortable
sensation of sitting on a ball and of something “falling out” of her vagina. Which of the following
is the next best step in management?
B. Discharge the patient with a prescription for metronidazole and an appointment with OB-GYN
in 2 days.
pg. 477
Answer C. This image reveals uterine prolapse. There are no contraindications to the manual
reduction of uterine prolapse, although attempts at reduction may not be successful. The
primary indication for reduction is symptomatic improvement. After reduction, the patient
should be fitted with a pessary to prevent recurrence. The pessary is a short-term solution that
may serve as a bridge to surgical repair. Pessaries should not be used in the setting of
concomitant genital tract infection.
19. What is the primary risk factor for uterine rupture among pregnant women?
B. Diabetes
Answer A. Uterine rupture may occur as a result of substantial maternal trauma. However,
among typical pregnant women, the primary risk factor for uterine rupture is a prior cesarean
delivery (csection), particularly with a “classic” vertical incision. Patients may present with
significant abdominal pain and vaginal bleeding or they may be relatively asymptomatic if only
pg. 478
minor dehiscence has occurred. A more recent meta-analysis suggests that only the risk of
dehiscence is increased among women with a prior cesarean-section (excluding women with a
classic vertical incision). Oxytocin did not increase the risk of rupture. Emergent
cesarean-section is indicated in all cases of uterine rupture.
20. A 25-year-old female presents to the ED with worsening left-sided pelvic pain and vaginal
discharge. She states that she was seen in the office by her primary care doctor 3 days ago for
similar problems, received an injection of an antibiotic, and was given a prescription that she
has not had a chance to fill. In the ED, she has a fever, purulent vaginal discharge, cervical
motion tenderness, and a palpable, tender left-sided adnexal mass. Which of the following is
true?
Answer C. This patient most likely has a tubo-ovarian abscess (TOA) complicating PID. Her
history suggests that she presented to her primary care doctor with symptoms of PID, but
because she only received an antibiotic effective against N. gonorrhoeae and has not filled her
prescription, she may still have untreated C. trachomatis infection. Interestingly, although N.
gonorrhoeae and C. trachomatis are known to be instrumental in the development of a TOA,
they are very rarely obtained at culture. Instead, the abscesses tend to be polymicrobial and
include gram-negative enteric organisms as well as anaerobes, such as Escherichia coli and
Bacteroides fragilis. It is thought that N. gonorrhoeae and C. trachomatis initiate the infection,
whereas other organisms invade and take over once the initial damage allows them to gain
entry and proliferate. All patients with a TOA should be admitted for broad-spectrum antibiotic
therapy. Ampicillin, clindamycin, and gentamicin have been the antibiotic combination of
choice. Abscesses between 4 and 6 cm in diameter respond to antibiotics alone 85% of the time.
However, abscesses >10 cm often require surgical intervention. If untreated, abscesses continue
to expand and may spontaneously rupture, resulting in secondary generalized peritonitis.
Roughly 5% to 10% of women with acute PID will develop perihepatitis known as
Fitz–Hugh–Curtis syndrome. This is primarily because of hematogenous or transperitoneal
spread of C. trachomatis and is characterized by right upper quadrant pain and tenderness,
pleuritic chest pain, and occasionally elevated liver enzymes.
pg. 479
and a rash. The rash, which started on her wrists and ankles, is petechial, and is somewhat
diffuse, including her palms. She returned from visiting relatives who live in North Carolina
approximately 4 days ago. Which of the following is the next best step in management?
A. Doxycycline 100 mg IV
B. Ciprofloxacin 500 mg IV
D. Penicillin V 500 mg IV
Answer C. This patient has RMSF. Classically, patients with RMSF develop a maculopapular rash
on the wrists and ankles 4 days after being bitten by an American dog tick (D. variabilis) or Rocky
Mountain wood tick (D. andersoni) infected with R. rickettsii. Despite its name, North Carolina
and Oklahoma account for more than one-third of RMSF cases. The rash rapidly spreads
centrally to the trunk and extremities and becomes petechial and purpuric. However, up to 15%
of patients do not develop a rash (“spotless fever”). Other common findings include a high fever
(typically >102°F), myalgias (particularly of the gastrocnemius), headache, vomiting, and
malaise. Doxycycline is the treatment of choice in all patients except for pregnant women, who
should receive CAM. CAM is the drug of choice during pregnancy due to the effects of
tetracyclines on fetal bones and teeth. However, CAM should be used with caution and should
be avoided in near-term pregnancies or during the third trimester in order to avoid fetal bone
marrow suppression, which results in the “gray baby syndrome.” Doxycycline should be used in
these cases. Previously, CAM was the recommended agent in children as well, but because
doxycycline is more effective and the risk of teeth staining is low given the short duration of
treatment, doxycycline is now the drug of choice for all patients except pregnant women.
22. A 22-year-old female at 36 weeks of gestation is being treated with magnesium sulfate for
preeclampsia while awaiting transfer to a nearby hospital for definitive management. Upon
reevaluating the patient, you find her to be quite somnolent with markedly decreased deep
tendon reflexes and a decreased respiratory rate. After managing the airway, the next best step
in management is intravenous:
A. Dexamethasone
B. Lidocaine
C. Labetalol
D. Calcium gluconate
E. Atropine
pg. 480
Answer D. Calcium gluconate antagonizes the effect of magnesium and should be given
immediately in all patients with any sign of respiratory depression. The magnesium infusion is
discontinued, and 1 g of calcium gluconate is infused over 2 to 3 minutes.
23. A 21-year-old female presents to the ED for a “recheck” of her β-hCG level. She was
evaluated in the ED 2 days ago after presenting with crampy low abdominal pain and vaginal
spotting. Her β-hCG level at that time was 1,350 mIU/mL but her ultrasound was not diagnostic.
She states that she continues to have intermittent crampy pain but she is no longer spotting.
Her β-hCG level is currently 1,900 mIU/mL. What is the best next step in management?
E. Discharge her with instructions to follow-up with OB in 2 days for serial β-hCG levels.
Answer B. The minimal rise in the β-hCG level of a viable intrauterine pregnancy is >50% over a
2- day period, although 85% of women will have an increase of >66%. However, 21% of patients
with ectopic pregnancy will exhibit a rise in their β-hCG level that mimics a normal intrauterine
pregnancy. This patient’s β-hCG level rose 41% in the 2 days since her previous visit which is
inappropriately low and she continues to have pelvic pain. Therefore, transvaginal ultrasound
should be performed to ensure there is no ectopic pregnancy.
24. A 23-year-old female presents with shortness of breath. She is 3 days postpartum after a
term, normal spontaneous vaginal delivery. She describes bilateral leg swelling, orthopnea, and
cough with frothy sputum. She denies chest pain. Physical examination demonstrates presence
of S3, bilateral pulmonary crackles, and pitting edema of her lower extremities. Which of the
following is true regarding this condition?
pg. 481
Answer D. The patient has peripartum cardiomyopathy, which occurs in approximately 0.03% of
all pregnancies and almost one-third of patients die. Risk factors include greater maternal age,
multiparity, and twin pregnancies. Clinical characteristics and acute management are the same
as that of congestive heart failure due to dilated cardiomyopathy. ECG, not cardiac
catheterization, is usually the first step in assessing structural cardiac abnormalities. Patients are
at high risk of developing peripartum cardiomyopathy in subsequent pregnancies. Aspirin plays
no role in management, although patients may benefit from heparin prophylaxis due to the
thromboembolic risk.
Answer A. The diaphragm elevates during pregnancy, resulting in decreased total lung capacity
and functional residual capacity. However, diaphragmatic excursion actually increases resulting
in an increased tidal volume and mild alveolar hyperventilation despite the fact that the
respiratory rate remains unchanged. The RBC mass and hemoglobin increase throughout
pregnancy. RBC mass rises steadily to term, as it increases 18% without iron supplementation. In
contrast, the plasma volume, which also increases during pregnancy, plateaus at 30 to 34 weeks
of gestation. The peripheral WBC count steadily rises during pregnancy and may be as high as
20,000 to 30,000 mm3 during labor after which it returns to normal in approximately 1 week.
Finally, the glomerular filtration rate increases throughout pregnancy.
D. Hyperemesis gravidarum
pg. 482
Answer E. Hydatidiform moles are placental abnormalities characterized by enlarged and
edematous placental villi and trophoblastic tissue (into grape-like structures) as well as loss of
fetal blood vessels. The two types of hydatidiform moles are complete and incomplete.
Complete moles almost always have a 46, XX karyotype that is paternally derived (due to
duplication of a paternally derived haploid genome), whereas incomplete moles have a
complete trisomy with a karyotype of 69, XXX or 69, XXY. Incomplete moles have two sets of
paternal chromosomes (again due to duplication of a paternally derived haploid genome) and
one set of maternally derived chromosomes. Complete moles are so named because of
complete absence of fetal parts (no fetus, umbilical cord, or amniotic membrane) and swelling
of all placental villi. In contrast, incomplete moles have partial placental villi swelling, and may
have a few fetal parts present and occasionally a complete fetus. The classic features associated
with hydatidiform moles are more common in the setting of complete moles. These include
pregnancy-induced hypertension occurring in the first trimester, uterine enlargement greater
than expected for dates, hyperemesis and very high β-hCG levels, as well as theca lutein ovarian
cysts. Vaginal bleeding is the most common presenting symptom and ultrasonography will
reveal the diagnosis in both cases (molar pregnancy in the case of a complete mole, and possibly
missed abortion or intrauterine fetal demise in the case of an incomplete mole). The classic
ultrasonographic appearance of a complete molar pregnancy is described as a “snowstorm.”
C. Maternal stress
E. Cigarette smoking
Answer A. By far, chromosomal abnormalities are the most common cause of first-trimester
spontaneous miscarriage. At least 50% of first-trimester pregnancy losses are related to fetal
chromosomal abnormalities. Autosomal trisomy is the most common abnormality, with trisomy
16 being the most common specific chromosomal defect. Polyploidy is the next most common
defect, with tetraploidy being most common. Uterine structural abnormalities, cigarette
smoking, and trauma may all contribute to fetal loss. However, minor trauma such as a fall or
strike to the abdomen, is very unlikely to cause fetal loss. Other factors include corpus luteum
failure, antiphospholipid antibody syndrome, and maternal endocrine diseases such as diabetes
mellitus and hypothyroidism.
pg. 483
28. A 26-year-old pregnant woman at 8 weeks presents to the ED with a chief complaint of
nausea and vomiting. Her abdomen and pelvic examinations are normal and her ultrasonogram
reveals a normal live intrauterine pregnancy at 8 weeks. Her urine reveals no ketones. Which of
the following is the best recommendation for reducing her symptoms?
A. Diazepam
B. Promethazine
C. Vitamin B6
D. Ondansetron
E. Dexamethasone
Answer C. Nausea and vomiting are common in pregnancy, with symptoms usually developing
between 4- and 7-weeks’ gestation and resolving by 16 weeks’ gestation. All of the agents listed
have been used in pregnant women suffering from nausea or vomiting in early pregnancy.
However, only vitamin B6 has been demonstrated to be beneficial among those agents listed.
Although vitamin B6 has been proven to be of use in reducing symptoms of nausea, its ability to
reduce vomiting related to pregnancy is less clear. More recently, ginger supplementation has
been shown to reduce both nausea and vomiting related to pregnancy and may be superior to
vitamin B6. The mutagenic effects of ginger are not known, but it is presumed to be safe.
29. A 22-year-old G1 at 9 weeks gestation with proven intrauterine pregnancy presents to the
ED with severe nausea and vomiting for 3 days. She has been unable to eat and can drink only
minimal fluids. Which of the following suggests a diagnosis of hyperemesis gravidarum?
A. Bilious vomiting
B. Hypokalemia
C. Metabolic alkalosis
D. Ketonuria
E. White race
pg. 484
glucose administration (to reduce ketosis created by fatty acid oxidation), nausea control, and
electrolyte and vitamin supplementation. HG is more serious than simple nausea and vomiting
of pregnancy, because weight loss and dehydration can have fetal effects. Ketosis has also been
shown in animal data to cause birth defects—however, this is unproven in humans. Ketones in
the urine have served as a marker for efficacy of therapy. Bilious vomiting is not characteristic of
either HG or nausea and vomiting of pregnancy and another cause should be aggressively
sought. Hypokalemia and metabolic alkalosis can occur with any vomiting illness (due to loss of
gastric acid and intracellular exchange of potassium) and does not reliably distinguish between
the two diagnoses in question.
30. EMS arrives in the ED with a 26-year-old female G2P1 in active labor. In preparing for
delivery, you discover that the baby is at the introitus, with the umbilical cord wrapped tightly
around its neck. If unable to slip the cord over the baby’s head, you should:
A. Cut the cord without clamping and deliver the baby as rapidly as possible
D. Double-clamp and cut the cord and deliver the baby as rapidly as possible
E. Suction the baby’s mouth and give ventilated breaths while in the birth canal
Answer D. As many as a third of deliveries involve a loop of umbilical cord wrapped around the
baby’s neck, called a nuchal cord. Most evidence suggests that such cords do not worsen fetal
outcome at delivery. However, tightly wrapped cords can impede delivery, or sometimes
entangle themselves into a knot which could restrict blood flow to the fetus. If the umbilical
cord can’t be slipped over the baby’s head while in the birth canal, the cord should be quickly
double-clamped and cut before the delivery is completed as rapidly as possible. In cases in
which there is enough slack in the cord, delivery could proceed without manipulating the cord
further. Crowning is a sign of imminent delivery. Do not attempt to push the baby back into the
vaginal canal. While the baby’s airway should be suctioned before delivery is completed,
delivery takes priority over all else since it is impossible to properly resuscitate a neonate while
it remains in the birth canal.
31. A 32-year-old female G3P2 at 39 weeks’ gestation presented to your community ED in active
labor with a fully dilated cervix. Because your hospital has no obstetrics services, you prepare
for delivery. One minute after a successful and apparently uneventful delivery, the patient
becomes abruptly hypoxic, severely hypotensive with a BP of 76/palp, and obtunded. The most
likely diagnosis is:
A. Sepsis
pg. 485
B. Pulmonary embolism
C. Peripartum cardiomyopathy
E. Eclampsia
Answer D. AFE is a rare complication of pregnancy also known as the anaphylactoid syndrome of
pregnancy. Through a still unclear mechanism, amniotic fluid gains entry into the maternal
circulation and triggers as immense inflammatory cascade resulting in pulmonary
vasoconstriction, pulmonary capillary leak, and myocardial depression. Clinically, patients
develop acute hypoxia, hypotension, and altered mental status. Disseminated intravascular
coagulation and seizures may also occur. In patients with eclamptic seizures, however,
hypertension will be present instead of the profound shock of AFE. AFE most commonly occurs
during labor and delivery or within 30 minutes of delivery. Though there are a variety of
experimental treatments, treatment is supportive and the mortality rate is high with most
survivors suffering permanent neurologic injury.
32. Which of the following is true regarding radiographic studies and pregnancy?
C. The cumulative background radiation exposure to a fetus throughout the 9-month gestation
is greater than the exposure due to a single maternal chest x-ray.
D. Lead shields minimally reduce fetal radiation exposure in the third trimester due to scatter
caused by the enlarged uterus.
E. The highest risk period of radiation exposure is in the first 2 weeks of pregnancy.
Answer C. Although there is no consensus, a cumulative dose of 5 rads has been proposed as an
acceptable threshold for safe fetal exposure. Intrauterine exposure of 10 rads is associated with
a small increase in the number of childhood cancers, but does not result in an increase in fetal
malformations, spontaneous abortion, or growth retardation. Maternal plain films of the head,
cervical spine, thoracic spine, chest, and extremities each expose the fetus to <5 mrad (i.e.,
1,000 times less than the safe threshold). Plain films of the lumbar spine, hip, and pelvis expose
the fetus to higher doses, but none of these studies comes close to approaching 5 rads. CT
scanning of the abdomen, however, results in approximately 2.6 rads of fetal exposure. In
comparison to these films, the cumulative radiation exposure to fetus over a 9-month gestation
is between 50 and 100 mrad (far more than a single chest x-ray). Lead shielding of the maternal
abdomen can reduce fetal exposure by 50% to 75%. The risk to the fetus during the first 2 weeks
pg. 486
of pregnancy is so low that normal radiographic procedure can be used. The period of highest
risk is between 2 and 7 weeks (organogenesis). Finally, FAST examination is poorly sensitive in
the second and third trimesters. Therefore, CT scanning should be performed in all patients in
whom abdominal injury is suspected.
33. A 48-year-old female with a history of menorrhagia is referred to the ED for a blood
transfusion after presenting to her primary doctor with mild exertional dyspnea. A CBC reveals a
hemoglobin level of 6.8 g/dL. The remainder of her evaluation is unrevealing. She is guaiac
negative and reports no active menses. Which of the following is true?
A. She should receive two units of packed red blood cells (PRBCs)
C. A repeat hemoglobin should not be checked until 4 hours after the transfusion is complete
Answer E. Any patient with life-threatening anemia should receive a blood transfusion.
However, several anemic patients fall into a gray middle zone, in which it is clear the anemia
needs treatment, yet no immediate life threat is present. This patient presents such a scenario.
Patients with chronic anemia are almost always iron deficient due to chronic blood loss (e.g.,
menorrhagia). While oral iron therapy is a potential solution, many patients don’t tolerate oral
iron therapy due to GI side effects and oral iron therapy may not be able to keep pace with the
degree of blood (and iron) loss. In such patients intravenous iron is a potential alternative to a
blood transfusion. Common options include low molecular weight iron dextran, which can be
given as a single, large infusion over an hour, as well as iron sucrose and ferric gluconate. Each
hospital will likely have different options on formulary. However, the infusions are often shorter
than intravenous levofloxacin, to which no one bats an eye. Anaphylaxis is extremely
uncommon and premedications are not recommended. If a transfusion is ordered instead, only
a single unit should be used and oral iron therapy should be prescribed along with possible
hormonal therapy to attenuate further bleeding. The habit of ordering two units for every
anemic patient only increases the risk to the patient without benefit. Hemoglobin values drawn
15 to 60 minutes after transfusion accurately reflect the new baseline in stable patients (who
are not bleeding).
34. Which of the following most places women at risk for abruptio placentae?
A. Preeclampsia
pg. 487
B. Cigarette smoking
D. Cocaine use
E. Trauma
Answer A. Increasing maternal age, cigarette smoking, cocaine use, twin or multiple gestation,
preexisting or pregnancy-induced hypertension (preeclampsia), trauma, chorioamnionitis,
oligohydramnios as well as thrombophilias all place women at risk for abruptio placentae. Of
these, preeclampsia is the most significant risk factor.
35. Which of the following is true regarding urinary tract infections in pregnancy?
C. All pregnant women with asymptomatic bacteriuria should receive antimicrobial treatment.
36. A 26-year-old female G2P1 at 25 weeks gestation and a history of placenta previa presents
with a chief complaint of lower abdominal pain with vaginal spotting. Which of the following is
true?
pg. 488
A. Most women are diagnosed with placenta previa after an episode of second- or
third-trimester vaginal bleeding.
B. Digital vaginal examination is recommended before ultrasonography for more rapid diagnosis
and obstetrician consultation.
C. Low-lying placentas in early pregnancy tend to migrate toward the cervical os over time.
Answer E. Placenta previa refers to a placenta that overlies or lies in close proximity to the
internal cervical os. The classic presentation of women with placenta previa is painless second-
or third trimester vaginal bleeding. However, most women are asymptomatic and are diagnosed
on routine ultrasonography. Digital vaginal examination should never be performed before
ultrasonography in the second or third trimester because it can provoke disastrous bleeding in
patients with asymptomatic placenta previa. With progression of pregnancy, 90% of low-lying
placentas will migrate away from the cervical os. Disseminated intravascular coagulation is a
common complication of abruptio placentae but not placenta previa.
37. A 27-year-old female presents to the ED with headache and nausea for several days. She is 2
weeks postpartum status post a normal spontaneous special delivery of a healthy infant at term.
Her primary care physician instructed her to take Tylenol or ibuprofen for her headache, but she
has not had any relief with this. Her vital signs are: 99.6, 85, 16, 165/92, 100% RA. Physical
examination is normal, including a normal cranial nerve examination. Which of the following is
the next best step in management?
C. Urinalysis
E. Lumbar puncture
pg. 489
(LDH). Neuroimaging can be warranted in the right clinical setting, however in this patient, it
should follow laboratory testing. Pain control certainly should occur, aggressive search for the
diagnosis should proceed prescriptions for outpatient management. Lumbar puncture may be a
step further down on the diagnostic pathway but should not be performed before the urinalysis
and evaluation of postpartum preeclampsia.
A. Ventilation-perfusion (V/Q) scans expose the fetus to less radiation than helical CT chest
scans in ruling out pulmonary embolism (PE).
Answer C. Thromboembolic disease is the leading cause of death in pregnancy. The risk of deep
venous thrombosis is highest in the postpartum period (puerpium), although the risk is elevated
throughout pregnancy. Warfarin is contraindicated throughout pregnancy due to its strong
association with fetal malformations even when given after the period of organogenesis (days
21 through 56 of fetal life). V/Q scans actually expose the fetus to more radiation than helical CT
scans when looking for PE.
39. A 27-year-old nonpregnant woman presents with pelvic pain. Pelvic ultrasonography reveals
a 4-cm right adnexal cystic mass. Which of the following is the most likely etiology?
B. Dermoid cyst
D. Follicular cyst
E. Ovarian fibroma
Answer D. Follicular cysts are the most frequent adnexal cystic structures in women with normal
ovaries. Follicular cysts represent remnants of previously normal follicles that grew in response
to follicle-stimulating hormone and then failed to involute. They are typically clinically silent but
they may cause pelvic pain or heaviness, as well as urinary frequency and constipation if they
pg. 490
are large enough. They are self-limited and involute over a period of weeks to months. Corpus
luteum cysts are less common but more clinically relevant. They also represent remnants of
formerly normal physiologic structures; in this case, the corpus luteum. Unlike follicular cysts,
they have a propensity to be complicated by intracavitary hemorrhage. If the hemorrhage is
brisk, the intracystic pressure may rise very quickly resulting in rupture. Such an event may
result in acute onset, severe pelvic pain, and may be associated with significant hemorrhage
depending on the size of the cyst. Theca lutein cysts are uncommon, typically bilateral, and
associated with prolonged or excessive ovarian stimulation. Dermoid cysts are benign ovarian
teratomas that contain tissue from all three germ cell layers. They do not pose an immediate
danger, but patients should be referred for further management because they may undergo
malignant transformation, particularly in women older than 40. Finally, ovarian fibromas are the
most common, benign, solid neoplasms of the ovary. They are extremely slow growing but may
grow to very large sizes.
40. A 22-year-old primigravida presents to the ED with crampy low abdominal pain. She is 11
weeks by dates and denies vaginal bleeding. Her physical examination reveals a normal vaginal
vault and a closed internal cervical os. Ultrasound examination reveals a single intrauterine
gestation at 9 weeks, with no fetal heart tones. Persistent failure to expel the fetal and maternal
uterine contents would result in a:
A. Threatened abortion
B. Incomplete abortion
C. Complete abortion
D. Missed abortion
E. Inevitable abortion
Answer D. The absence of fetal heart tones in an intrauterine gestation with a crown-rump
length >5 mm (correlates roughly to 6.2 weeks of gestation) is convincing evidence of fetal
demise. Missed abortion refers to the continued presence of a nonviable fetus aged <20 weeks
of gestation for at least 8 weeks without the passage of maternal or fetal tissue. The persistent
presence of a dead fetus may result in coagulation abnormalities and disseminated intravascular
coagulation. However, this is primarily a historical diagnosis due to the prevalence of US.
Therefore, most women present to a physician with vague complaints of crampy abdominal pain
or vaginal bleeding or they are noted to have a uterus that is too small for their dates.
Subsequent US identifies intrauterine fetal demise before coagulation complications occur, and
the patient is referred to an obstetrician for definitive management.
pg. 491
41. The “discriminatory zone” is the quantitative serum β-hCG level at which a normal
intrauterine gestation should be seen on ultrasonography. The discriminatory zone for
transvaginal ultrasonography is:
Answer C. The discriminatory zone is the quantitative serum β-hCG level at which a normal
pregnancy can be detected by either transvaginal or transabdominal ultrasonography. As
ultrasonographic technology improves, the discriminatory zone continues to drop. Furthermore,
these levels may vary somewhat between hospitals due to technologic differences. However,
the accepted range is 1,000 to 2,000 mIU/mL for transvaginal ultrasonography and 2,400 to
3,600 mIU/mL for transabdominal ultrasonography. Other sources cite a range of 1,500 to 2,500
mIU/mL as the accepted range in transabdominal ultrasonography. Of course, the higher the
discriminatory threshold, the higher the specificity for an abnormal pregnancy, including ectopic
gestation.
42. A 36-year-old primigravida presents to the ED at 32 weeks’ gestation with epigastric pain.
Her BP is 150/100, but other vital signs are normal. While the nurse is performing his
assessment in the room, the patient begins to seize. The next best step in management is:
A. Hydralazine 10 mg IV push
C. Phenytoin 20 mg/kg IV
Answer D. Magnesium remains the drug of choice for the treatment of seizures in eclampsia as
well as for the prophylaxis of seizures in patients with preeclampsia. The recommended dose is
6 g given intravenously over 15 to 20 minutes followed by a continuous infusion at 2 g/hour.
Hydralazine, labetalol, and nimodipine are all agents that have been used for BP control in
patients with eclampsia. Hydralazine is most commonly used and is typically given in 5 to 10 mg
doses every 15 to 20 minutes. Lorazepam and phenytoin are second-line agents for seizure
pg. 492
control in patients with eclampsia. An obstetrician should be involved in the care of all patients
with eclampsia and can help direct therapy if the initial magnesium bolus is ineffective. Finally,
although recommendations vary, treatment of seizures is the first priority. Once seizures are
terminated, BP may be controlled only if the diastolic BP remains elevated above 105 to 110 mm
Hg. In the absence of seizures, the same BP guidelines apply, and magnesium is given for
prophylaxis against seizures.
A. Atrial fibrillation
C. Somnolence
D. Hyperventilation
E. Diarrhea
Answer C. Magnesium depresses the CNS and slows nerve conduction. It is used in preeclampsia
to prevent progression to eclampsia, which is characterized by the presence of seizures.
Magnesium slows neuromuscular conduction and decreases CNS irritability. Remembering this
provides an easy means of remembering the actions of magnesium. It will decrease the
respiratory rate, decrease deep tendon reflexes, and decrease the degree of consciousness. The
loss of deep tendon reflexes is generally the first sign of magnesium toxicity.
44. A woman presents to the ED with bilateral pelvic pain. You consider the diagnosis of pelvic
inflammatory disease (PID). Which of the following is a risk factor for PID?
B. Age >30
C. Diabetes insipidus
D. Pregnancy
pg. 493
Answer A. Most cases of PID occur within 1 week of menses. One hypothesis for this is that
menstrual blood flow provides an optimal culture medium for bacteria. Other risk factors
include young age (<20), history of any sexually transmitted infection (especially gonorrhea or
chlamydia), and multiple sexual partners. PID is rare during pregnancy, as thickened cervical
mucus is thought to prevent bacterial ascension. Diabetes insipidus and ovarian cysts are not
risk factors for PID.
45. A 17-year-old female presents with bilateral lower quadrant abdominal pain and vaginal
discharge for 3 days. She reports having been sexually active with multiple partners in the last
month. Her last menstrual period just finished 3 days ago. She denies fever, vomiting, dysuria,
and diarrhea. Her urine pregnancy test is negative. Which of the following is most likely to yield
the correct diagnosis?
A. Physical examination
B. CBC
C. Urinalysis
Answer A. PID is an infection of the fallopian tubes. Risk factors include young age (15 to 25 is
the highest risk group), multiple sexual partners, smoking, and bacterial vaginosis. It is caused by
Chlamydia, gonococcus, and organisms, which cause bacterial vaginosis. The peak time of onset
is within 1 week of menses, as menstrual flow is thought to provide an optimal culture medium
for bacterial ascension. Symptoms include diffuse pelvic pain, fever, nausea, vomiting, vaginal
discharge, and dyspareunia. Patients usually exhibit bilateral adnexal tenderness with significant
cervical motion tenderness and cervical discharge. PID is a clinical diagnosis with laboratory and
imaging studies useful only to rule out other causes of symptoms. Treatment involves antibiotics
to cover Chlamydia and gonococcus—the most common regimen is IM ceftriaxone plus
doxycycline/azithromycin. Sequelae of untreated PID are extremely serious, including tubal
scarring causing infertility and ectopic pregnancy, chronic pelvic pain, and tubo-ovarian abscess.
46. What is the most common ultrasonographic finding in women with ovarian torsion?
A. Intraovarian hemorrhage
B. Ovarian enlargement
D. Pelvic-free fluid
pg. 494
E. All of the above are common findings
47. A 27-year-old female at 33 weeks of gestation presenting with liver tenderness and evidence
of coagulopathy is most suggestive of:
B. Preeclampsia
C. Hepatitis
D. Cholecystitis
pg. 495
symptoms. Symptomatic patients present with jaundice or scleral icterus, nausea and vomiting,
and right upper quadrant tenderness with aminotransferase levels in the thousands. The course
in pregnant and nonpregnant women is typically benign and indistinguishable from one another.
Cholecystitis is the second most common surgical emergency during pregnancy, and patients
typically present with fever, right upper quadrant pain, nausea and vomiting, and leukocytosis
(although this can be confused with the leukocytosis of pregnancy). ICP is, like AFLP, a rare
disorder that typically complicates the third trimester. Patients present with moderate to severe
pruritus that typically begins on the palms and soles and progresses in an ascending manner.
Approximately 20% of patients will also be jaundiced on presentation. ICP is associated with
increased preterm delivery, increased perinatal mortality, and meconium staining. Fetal
mortality approaches 20% in untreated patients. The optimal treatment for preeclampsia, AFLP,
and ICP is delivery.
48.A pregnant woman at 34 weeks’ gestation is undergoing tocographic and fetal monitoring in
your small community ED after a motor vehicle accident (Fig. 10-9). While discussing the case
with the on-call obstetrician, she asks for your interpretation of the fetal strip. Which of the
following is represented in the strips?
A. Head compression
B. Uterine rupture
D. Placenta previa
E. Uteroplacental insufficiency
pg. 496
Answer E. The strips demonstrate “late decelerations” of the fetal heart rate relative to uterine
contractile activity. Late decelerations typically begin roughly 30 seconds after the onset of a
uterine contraction and their nadir occurs after the peak of the contraction. This most often
represents uteroplacental insufficiency, which is an interruption in uteroplacental blood flow.
Early decelerations have the same gradual slope and shape as late decelerations, but they occur
with different timing relative to uterine contractions. The nadir of early decelerations and the
peak of uterine contractions occur simultaneously and the fetal heart rate returns to baseline by
the end of the contraction. Early decelerations are thought to result from compression of the
fetal head, which causes a vagal reflex. Variable decelerations are the most common type of
pattern seen during fetal cardiac monitoring. These decelerations have an inconsistent
appearance with respect to shape, width, depth, and timing relative to uterine contractions.
They represent umbilical cord compression and are typically benign. However, frequent or
particularly deep decelerations (i.e., representing fetal bradycardia) may be an indicator of fetal
distress.
49. A 22-year-old female presents with sudden onset of palpitations. She is 10 weeks pregnant
with a confirmed intrauterine fetus. Her vital signs are 98.6, 190, 20, 124/67, 98% RA. The EKG
shows a regular, narrow-complex tachycardia with absent P waves. Which of the following is the
most appropriate next step in management?
A. Digoxin 0.125 mg IV
B. Adenosine 6 mg IV
C. Lidocaine 100 mg IV
D. Amiodarone 150 mg IV
E. Procainamide 1 g IV
50. Which of the following laboratory abnormalities is most commonly associated with
hyperemesis gravidarum?
A. Hyperkalemia
pg. 497
C. Thrombocytopenia
D. Hyperglycemia
Answer B. There are no clear diagnostic criteria for hyperemesis gravidarum. However, it is
generally considered to be nausea and vomiting related to pregnancy that is severe enough to
result in weight loss, starvation ketoacidosis, electrolyte imbalance, alkalosis from vomiting, and
dehydration. Liver enzymes may be elevated in up to 25% of hospitalized women, although they
are not normally increased beyond four times the upper normal limit. This more frequently
occurs when hyperthyroidism accompanies hyperemesis. Hyperthyroidism complicating
hyperemesis is transient and usually resolves by 14 to 16 weeks of gestation, along with the
symptoms of hyperemesis. The elevated levels of thyroid hormone are thought to result from
high levels of β-hCG, which is known to stimulate the thyroid-stimulating hormone receptor.
51. An 18-year-old female presents to the emergency department (ED) with acute-onset severe
right lower quadrant pain. Initially, she was doubled over with pain and had immediate nausea
and vomiting just prior to her ED evaluation. The pain resolved almost completely after 15
minutes, and then she had a second episode while in the ED waiting room. The patient is
currently pain-free. Her last menstrual period was 2 weeks ago. She has a family history of
kidney stones. Her vital signs are normal. Physical examination is unremarkable except for mild
fullness and tenderness in the right adnexa. Her complete blood count (CBC), chemistry, and
urinalysis are normal and her urine pregnancy is negative. Which of the following is the most
appropriate next step in management?
D. Pelvic ultrasound
Answer D. In a young healthy female with acute-onset debilitating abdominal pain, the
time-sensitive diagnosis is ovarian torsion. Delays of even several hours can result in irreversible
ovarian necrosis and potential infertility. Evaluation with pelvic ultrasound and gynecologic
consultation is indicated. CT studies produce unacceptable delays in the evaluation of ovarian
torsion. The other major diagnosis in the differential in this case is ureteral colic from kidney
stone. Although kidney stone also can be an emergent issue, immediate operative management
is rarely required when concomitant urinary tract infection is not present. Torsion occurs more
commonly about 2 weeks into the menstrual cycle when ovulation is occurring. An enlarged
pg. 498
ovary (due to tumor, cyst, or abscess) is the strongest risk factor. Ovarian torsion risk is also
increased during pregnancy.
52. A 32-year-old pregnant woman in her third trimester presents with dysuria. She has a
penicillin allergy. Urinalysis demonstrates bacteria, but no white blood cells, leukocyte esterase,
or nitrites. Physical examination is normal. Which of the following is the most appropriate next
step in management?
B. Trimethoprim–sulfamethoxazole
C. Ciprofloxacin
D. Nitrofurantoin
E. Amoxicillin
Answer D. Both bacteriuria and urinary tract infection (UTI) should be treated aggressively in
pregnant women, as they can cause serious complications with delivery. The prevalence of
bacteriuria may be higher in pregnant women than in nonpregnant women, but UTI rates are
comparable. Symptoms are identical between the two groups. Gram-negative enteric bacteria
cause the vast majority of UTIs in pregnant women, but a crucial organism to consider is Group
B Streptococcus, which can cause serious neonatal infection. Nitrofurantoin, penicillins, and
cephalosporins are the drugs of choice. Sulfonamides are not safe during the third trimester
because of the possibility of neonatal hemolysis. Fluoroquinolones cause various congenital
defects. Although amoxicillin would be a reasonable choice in most patients, the allergy to
penicillin is an obvious contraindication.
B. Place a Foley catheter and instill the bladder with 750 mL of saline
D. Elevate the presenting fetal part to relieve compression on the umbilical cord
pg. 499
Answer E. Umbilical cord prolapse occurs in association with fetal malpresentation
approximately 50% of the time (e.g., footling breech). However, the remaining 50% of cases
occur in normal cephalic presentations. The overall incidence is roughly 1 in 160 to 1 in 600
births. Steps to arrange immediate cesarean delivery should be taken without delay. In the
interim, the goal of the ED physician is to preserve umbilical circulation by relieving pressure
upon the umbilical cord. All of the procedures listed may be helpful in the setting of umbilical
cord prolapse. If cesarean delivery is not available, the umbilical cord should be placed back into
the uterus (funic reduction) and the fetus should be delivered vaginally as soon as possible.
Umbilical cord entanglement is a common complication of funic reduction.
pg. 500
Pediatric emergency
➢ Chapter includes
pediatric related medical condition
infectious and diarrheal disease in pediatric
neonatal medical condition
✓ exception (pediatric trauma in trauma chapter)
✓ exception (non-accidental trauma in chapter
psychiatry, abuse and suicide)
1. A first-time mother presents with her 13-day-old infant with a chief complaint of seizures. The
infant had an uncomplicated term delivery, is afebrile and had been well until the day of
presentation. Which of the following is the most likely cause of this infant’s seizures?
A. Hypokalemia
B. Hyponatremia
C. Hypocalcemia
D. Hypomagnesemia
Answer B. In a well newborn beyond the immediate neonatal period, without fever,
hyponatremia is the most common cause of seizures. Water intoxication is the most common
cause of hyponatremia during infancy. Infants are unable to adequately concentrate urine so
parents who dilute formula or give their infants tap water put their infants at risk. Hypocalcemia
is also a very common cause of seizures in the neonate, so serum calcium levels should be
checked. In fact, all infants have a slight decline in serum calcium levels with a nadir at 24 to 48
hours. Symptomatic hypocalcemia is more common in infants of diabetic mothers, preterm
pg. 501
infants, or infants with a history of anoxic encephalopathy. Hypomagnesemia is not as common
as hypocalcemia, but symptoms of hypomagnesemia mimic those of hypocalcemia and it is
difficult to correct hypocalcemia if the serum magnesium is also low. Therefore, infants with
seizures should have a comprehensive evaluation of their electrolytes. Hypokalemia is
uncommon in infancy and does not typically cause seizures. Maple syrup urine disease is a rare
disease resulting from the inability to catabolize branched chain amino acids. Infants typically
present between 4 to 7 days of life with poor feeding, vomiting, or lethargy. Neurologic
manifestations rapidly develop, such as alternating hypotonia and hypertonia, dystonia,
seizures, and encephalopathy.
2. A 5-year-old male presents with confirmed rotavirus diarrhea. He is tachycardic and lethargic
with sunken eyes, poor skin turgor, and dry mucous membranes. Which of the following is the
most appropriate next step in management?
Answer E. The initial intravenous resuscitation fluid of choice in pediatric patients is 0.9 normal
saline. It is given in a 20 mL/kg bolus and may need to be repeated twice in patients with severe
dehydration (up to 60 mL/kg). Dextrose-containing half-normal saline is adequate for
maintenance, but the initial concern is volume repletion, so normal saline should be
administered. When sufficient bolus hydration has been given, maintenance fluids according to
the “4/2/1 rule” are instituted: 4 mL/kg/hour for the first 10 kg of body weight, 2 mL/kg/hour
for the next 10 kg, and 1 mL/kg/hour for every 10 kg after that. The maintenance fluid
composition varies by age and may be looked up in a reference.
3. Which of the following is true regarding gastroesophageal reflux disease (GERD) in infants?
A. Ranitidine and metoclopramide are often required and are the mainstays of medical therapy.
C. Most infants respond to conservative measures such as smaller, thickened feedings and
frequent burpings.
pg. 502
Answer C. GERD is a very common cause of vomiting during infancy. Emesis is typically
nonbilious and begins in early infancy, remaining fairly constant over time. It does not
demonstrate the progressive pattern to projectile emesis that characterizes pyloric stenosis.
Most infants respond to conservative measures. In the past, ranitidine (an H2 histamine
receptor antagonist) or metoclopramide (which increases lower esophageal tone and gastric
emptying) was used to help control symptoms. However, metoclopramide is not considered
useful in infants <1 year of age with uncomplicated GERD and prokinetic agents such as
metoclopramide, have considerable side effects. Proton pump inhibitors, such as omeprazole,
have replaced histamine blockers, such as ranitidine, when acid reduction is required. Most
infants continue to gain weight normally and only rarely do infants demonstrate failure to
thrive. For the most part, infant GERD peaks at 4 months and resolves by 12 months with nearly
all cases resolved by 24 months of age (as the lower esophageal sphincter becomes more
competent with age).
4. Which of the following patients with a febrile seizure most likely requires further evaluation
with a lumbar puncture (LP) to look for an infectious source?
E. An unvaccinated 5-year-old
Answer C. The optimal criteria used to determine which patients require an LP after a first
febrile seizure are constantly evolving and remain unclear. However, the American Academy of
Pediatrics (AAP) released new, less aggressive guidelines in 2011. According to its most recent
guidelines, the AAP states that an LP should be considered for all infants aged 6 to 12 months
who are inadequately vaccinated against H. influenzae or Streptococcus pneumoniae or in
whom the vaccination status is not known. In addition, an LP should be considered for any infant
receiving current antibiotic therapy since the use of antibiotics may mask meningeal signs and
symptoms. Other higher risk groups include patients with focal or prolonged seizures, abnormal
physical examinations, or toxic appearance. Regardless of vaccination status, older patients are
able to participate in a complete physical examination to aid the physician in determining
whether meningitis is a consideration. Most ordinary febrile seizures are treated
symptomatically with antipyretics. Approximately one-third of patients with febrile seizures will
have at least one more febrile seizure episode. Risk factors for this include young age at first
seizure, lower temperature with the seizure episode, a first-degree relative with febrile seizure,
and short duration between fever onset and seizure event. Patients with febrile seizures are
twice as likely to develop epilepsy compared to the general population.
pg. 503
5. Which of the following sleeping positions is the best method to reduce the risk of sudden
infant death syndrome (SIDS)?
A. Prone
B. Supine
C. Side
D. Standing
E. Head down
Answer B. SIDS is defined as the sudden, inexplicable death of any infant whose cause cannot
even be diagnosed by autopsy. The peak age for SIDS is 2 to 4 months. Risk factors include
maternal smoking, young maternal age, preterm age, among others. Apnea and hypoventilation
are the most likely explanations, but dysrhythmias, airway obstruction, and trauma are all
proposed as possible contributors. Infants should be placed on their backs to sleep to help
reduce the incidence of SIDS.
Answer D. In neonates, the heart rate is the most sensitive indicator of adequate
cardiopulmonary status. In addition to inadequate respiratory effort, infants with bradycardia,
specifically when the heart rate falls below 100, need positive pressure ventilation (PPV). PPV is
delivered with bag–mask ventilation (BMV) at a rate of 40 to 60 times per minute for 30
seconds, after which the heart rate is assessed to determine the effectiveness of the
resuscitation. Tracheal intubation is ultimately required if the infant’s heart rate doesn’t
improve despite these efforts.
pg. 504
7. A 3-year-old previously healthy female is brought to the emergency room after ingesting
three of her mother’s 2.5 mg glyburide tablets, believing they were candy. Her mother estimates
that the ingestion occurred almost 2 hours before presenting, but she only noticed the open box
shortly before arrival. She states her daughter seems “ok” but has been a bit more tired and
irritable than normal. The patient’s initial blood glucose is 58. Which of the following is the best
next step?
B. Administration of 5 mL/kg of intravenous D25W and observation in the emergency room for
4hours.
D. Administration of 10 mL/kg of intravenous D10W and observation in the emergency room for
4 hours.
Answer C. Sulfonylureas fall into the “one pill can kill” category of toxic ingestions among
pediatric patients. Glyburide is a commonly used second-generation sulfonylurea with a long
half-life (10 hours) as well as active metabolites. Insulin release is increased within 1 hour after
ingestion, and hypoglycemia rapidly follows. As with several other sulfonylureas, the peak effect
does not occur for 2 to 6 hours, and because of the drug’s prolonged half-life, persistent or
delayed effects both occur. Many pediatric patients are asymptomatic with euglycemia at
presentation. However, most experts recommend admission to the hospital for prolonged
observation, even among such patients. Patients with symptoms at presentation, such as the
patient in this question, should be admitted. In contrast to adults, children should receive more
dilute preparations of dextrose to manage hypoglycemia. D25W can be given to young children
while D10W is preferred for neonates and infants, though it can also be given to older children.
While the exact dextrose dose can deviate from this rule, the easiest way to remember the
amount of dextrose to deliver in acutely hypoglycemic, symptomatic neonates is to follow the
“rule of 50.” The basic formula is % dextrose × mL/kg volume = 50. So:
D10 × 5 mL/kg = 10 × 5 = 50. For a 5 kg child, bolus 5 mL/kg or 25 mL of D10 (as in this patient)
While some experts recommend octreotide therapy to all symptomatic patients receiving
dextrose, others recommend octreotide only in settings of refractory hypoglycemia. Octreotide
works by decreasing calcium influx in pancreatic beta islet cells, which results in decreased
calcium-mediated insulin release.
pg. 505
8. A 1-week-old term male infant who was delivered at home is brought to the ED by his parents
with abdominal distension, difficulty passing stools, failure to thrive, and bilious vomiting. Digital
rectal examination reveals an empty rectal vault but stimulates significant stool and gas
production once the finger is removed. Which of the following is the most likely cause of these
symptoms?
A. Intussusception
B. Pyloric stenosis
D. Hirschsprung disease
E. Incarcerated hernia
Answer D. Hirschsprung disease accounts for roughly 20% of cases of partial intestinal
obstruction in early infancy. Although intussusception is the most common cause of intestinal
obstruction in children younger than 2 years, the typical presentation is one of acute-onset,
severe abdominal pain which may be associated with vomiting and bloody stools. Patients with
pyloric stenosis present with progressive nonbilious projectile emesis. Patients with GERD do
not develop signs of obstruction and most commonly have nonbilious emesis. Hirschsprung
disease is usually diagnosed in the newborn nursery due to failure of newborns to pass
meconium. Ninety-nine percent of full-term infants pass meconium within 48 hours of birth.
However, because there is a spectrum of disease, some infants may present in a delayed
manner and may have a subtle presentation. Since this infant was delivered at home, he was not
observed in the newborn nursery, where stool production is monitored. The absence of stool in
the rectal vault followed by copious stool and gas production following digital examination
indicates a likely distal colonic obstruction (created by the narrowed, aganglionic segment of
bowel) which is transiently relieved by digital examination. These findings in concert with the
patient’s symptoms of obstruction should bring about the consideration of Hirschsprung
disease. Diagnosis is first confirmed through an abnormal “string sign” on barium enema. This is
followed by rectal biopsy revealing the aganglionic segment of bowel responsible for the
disease.
B. Hemolytic anemia
C. Obesity
D. Diabetes
E. Cerebral palsy
pg. 506
Answer B. Patients with hemolytic disorders, such as sickle cell anemia or hereditary
spherocytosis, represent the largest group of pediatric patients with symptomatic cholelithiasis.
Hemolysis puts these patients at risk for the formation of pigmented gallstones. CF and obesity
also put children at risk for cholelithiasis. In CF, patients experience inspissated biliary
secretions, leading to stone formation. Diabetes and cerebral palsy are unrelated diseases.
Neonates often have multiple factors predisposing to gallstones, but these patients are usually
diagnosed in the neonatal intensive care unit (ICU), before discharge and presentation in the ED.
Examples of neonatal risk factors include prematurity, parenteral nutrition, surgery, blood
transfusion, sepsis, and diuretic administration.
10. A first-time mother brings in her 4-day-old male infant to the ED with a chief complaint of
abdominal pain and sudden yellowish-green vomiting. She notes that he had been “fussy” all
day but became ill only a few hours earlier. On examination, he appears ill and has a mildly
distended abdomen. An abdominal film is shown below . Which of the following is the next best
step in management?
A. Barium enema
pg. 507
Answer B. The abdominal film demonstrates gastric and duodenal dilation. Malrotation with
midgut volvulus must be a leading consideration in any toxic-appearing infant with a history of
sudden-onset bilious emesis. Malrotation occurs in approximately 1 in 500 live births, or about
half as often as pyloric stenosis. Of all infants with malrotation, 75% will develop volvulus and
75% of these infants will present in the first month (most in the first week). Most infants present
with acute-onset bilious emesis and obstruction. Plain films are usually nonspecific and may be
normal, but they may also reveal the “double bubble” sign which reflects the dilated stomach
and proximal duodenum (the intervening pylorus separates the two “bubbles,” not shown in Fig.
This may also be seen with duodenal atresia but that is an illness that presents in the newborn
nursery. Malrotation with midgut volvulus is a life-threatening illness that requires emergent
surgical intervention to reduce the volvulus and relieve the ischemia caused by constriction of
the bowel’s mesenteric blood supply. Necrosis of the bowel may occur in as little as 3 hours.
11. EMS arrives with a 32-year-old female in active labor who spontaneously delivers her infant
while she is still on the EMS stretcher. Upon initial evaluation, the infant is lying on its side on
the stretcher, between the mother’s legs, appearing cyanotic, breathing spontaneously but not
crying. What is the next step?
Answer B. Cyanosis is normal after birth and acrocyanosis can persist for as long as 10 minutes
in many infants. Thus, cyanosis alone is not a good indicator of neonatal respiratory distress. If
the neonate appears to have consistent, spontaneous respirations with a good effort, then the
neonate should first be suctioned in the mouth and nose. This clears oropharyngeal and
nasopharyngeal secretions that may be impeding airflow. It also stimulates the baby helping to
provide an indicator of neonatal wellness and responsiveness. In routine deliveries, in which the
neonate’s respiratory effort is clearly strong, suctioning with a bulb syringe is unnecessary and
oral and nasal secretions can be wiped away with a clean towel. However, in cases in which
there is a question about neonatal respiratory effort, suctioning is the first step before moving
forward to positive pressure ventilation.
12. A 3-day-old male infant is brought into the ED by his parents with a rash consistent with
erythema toxicum neonatorum. Which of the following is true?
pg. 508
B. He probably has lesions on his palms and soles
D. The rash will look like macules and pustules on an erythematous base
Answer D. Erythema toxicum neonatorum is a very common, benign rash which is most
common in full-term infants (incidence declines with decreasing gestational age) (see Fig.
below). It is thought to be due to a problem with sebaceous glands within hair follicles called
pilosebaceous follicles. It is most common on the trunk and proximal extremities and spares the
palms and soles. As it resolves spontaneously it requires no treatment.
13. A 3-day-old female neonate is brought to the ED with a chief complaint of rectal bleeding. A
guaiac test of the stool is positive. Which of the following tests can help to determine whether
the blood in the stool is of fetal origin?
B. Meckel scan
C. Rosette test
D. Apt test
pg. 509
Both the Rosette test and the KBT are used to detect the presence of fetal maternal
hemorrhage.
14. A 12-year-old male is brought in by his parents for evaluation of three episodes of bloody
diarrhea. There has been a recent outbreak of E. coli 0157:H7 traced to a petting zoo. The
patient appears fatigued but nontoxic with a benign abdominal exam and vitals T 100.1°F, P 98,
R 18, BP 110/77, and SaO2 97% on RA. His blood tests reveal a white blood cell count of
16,100/mm3, hemoglobin 12.6 g/dL, platelets 142,000/mm3, HCO3 of 17 mEq/L, BUN 28 mg/dL
and creatinine 1.1 mg/dL. Which of the following is true?
Answer C. Enterohemorrhagic E. coli (EHEC) is a common cause of pediatric bloody diarrhea. The
bacterium produces a Shiga-like toxin which is responsible for hemolytic uremic syndrome
(HUS). Though EHEC is the most common cause of HUS, HUS occurs in fewer than 10% of
patients with EHEC infections. HUS develops 5 to 10 days after the onset of diarrhea and
classically comprises hemolytic anemia, thrombocytopenia, and acute kidney injury with as
many as 50% of patients requiring dialysis. The patient in this vignette doesn’t have any of these
symptoms. In well patients like him with symptoms only of hemorrhagic colitis, treatment is
supportive, consisting of rehydration, and electrolyte management. Antibiotics may increase the
rate of HUS because their use can induce expression and release of Shiga toxin. Antidiarrheals
can also worsen systemic symptoms and should be avoided.
15. A 4-year-old previously healthy girl presents to the ED with a 2-day history of cough
productive of yellow sputum, fevers up to 102.4°F, pleuritic chest pain, and dyspnea. On
examination, she is tachypneic and mildly toxic appearing. Her chest x-ray reveals a right middle
lobe infiltrate. Which of the following organisms is the most likely cause of this problem?
A. Klebsiella pneumoniae
B. Mycoplasma pneumoniae
C. Chlamydia trachomatis
E. Streptococcus pneumoniae
pg. 510
Answer E. S. Pneumoniae (also known as Pneumococcus) is the most common bacterial cause of
pneumonia in preschool-aged children (6 months to 5 years). Overall, viruses are the most
common pathogens causing pneumonia in this age group, with RSV being the most common,
followed by parainfluenza and influenza viruses, as well as adenovirus and rhinovirus. However,
this patient did not present with a viral prodrome and is mildly toxic upon examination. It is
critical, therefore, to treat this patient with antibiotics that target S. Pneumoniae. Because of
increasing resistance among S. Pneumoniae isolates, high-dose amoxicillin is the drug of choice,
although patients who are hospitalized may require ampicillin, cefuroxime, or cefotaxime
delivered intravenously. K. pneumoniae is an uncommon cause of pneumonia in children though
it can cause severe infections in immunocompromised hosts. Community-acquired Klebsiella is
primarily a disease of debilitated older men with a history of alcoholism. M. pneumoniae is the
most common pathogen causing pneumonia in children aged 5 to 15 years. C. trachomatis may
cause pneumonia in infants aged 3 weeks to 3 months, typically causing an afebrile, subacute
interstitial pneumonia. RSV bronchiolitis and pneumonia are the primary causes for
hospitalization during the first year of life.
16. Which of the following is the most specific finding in diagnosing necrotizing enterocolitis
(NEC)?
D. The presence of abdominal distension, bilious emesis, and guaiac positive stools
Answer C. NEC is the most common GI emergency affecting neonates. However, because
premature infants are predominantly affected, it is a disease that is most commonly diagnosed
in the neonatal intensive care unit (NICU) and only rarely seen in the ED. Interestingly, however,
the age at onset of NEC is inversely related to the gestational age and birth weight. Therefore,
low–birth-weight infants who initially look well in the NICU may be discharged home before the
development of NEC. Initial findings in NEC are nonspecific and include ileus (multiple dilated
loops of small bowel) or an asymmetric bowel gas pattern. With the progression of the disease,
however, air spreads through ulcerated GI mucosal epithelium resulting in pneumatosis
intestinalis or air within the biliary tract (portal venous). Pneumatosis intestinalis occurs in 75%
of infants with NEC and is the most specific finding.
17. A previously healthy 5-year-old male presents with painless rectal bleeding. The bleeding
seems to have resolved but his mother states that he had four or five large, brick-colored stools
pg. 511
earlier in the day. His stool guaiac test is positive. Which of the following is the most likely cause
of his symptoms?
A. Duodenal ulcer
B. Meckel diverticulum
C. Esophagitis
D. Anal fissure
Answer B. Meckel diverticulum is the most common cause of substantial GI bleeding in children.
The diverticulum is a remnant of the omphalomesenteric (or vitelline duct), which is frequently
lined with gastric mucosa or other heterotopic tissues. It follows the “rule of 2s.” It is present in
2% of the population, and only 2% of patients will ever develop symptoms or complications. It
contains two types of tissue (it includes gastric tissue which produces acid occasionally causing
bleeding ulcers). It is located within 2 feet proximal to the ileocecal valve, is 2 cm long and 2 cm
wide. Half of all patients develop symptoms by the age of 2. Bleeding is usually painless and
often resolves spontaneously due to splanchnic vasoconstriction. Meckel scan which is
performed with technetium Tc 99 m pertechnetate is the diagnostic test of choice.
18. A 3-week-old term neonate is brought by his parents with a fever of 102.0°F. He appears
active and nontoxic, but his right tympanic membrane (TM) is red, and moderately swollen. The
rest of the physical examination is completely unremarkable. Which of the following is the most
appropriate next step in management?
C. Administer a single dose of IM ceftriaxone and discharge home with next-day follow-up.
E. Admit for observation with prophylactic antibiotics and blood, urine, and cerebrospinal fluid
(CSF) cultures.
Answer E. The approach to the neonate with fever is constantly evolving. However, all neonates
less than 28 days old still require a full “septic workup” with blood, urine, and CSF cultures, as
well as prophylactic antibiotics and admission for observation. In this age-group, serious
pg. 512
bacterial infection is common and often completely undetectable by physical examination or
routine blood tests. Current guidelines enable the emergency physician (EP) to use considerable
discretion in the evaluation of older (>28 days) febrile neonates. It is extremely important to
obtain a thorough vaccination status of young infants, since the standard vaccine schedule
aggressively immunizes infants in the first year of life. Although there is no perfect algorithm for
evaluating older neonates, clinicians can use a CBC to help stratify well-appearing infants >28
days old into high- and low-risk categories for further evaluation. In addition, urinary tract
infections are a common cause of occult fever among female infants as well as uncircumcised
male infants. Catheterized urinalysis and culture should be sought in all febrile female infants
<24 months as well as in all uncircumcised, febrile male infants <12 months in whom there is no
alternative source of infection. The potential presence of otitis media should never result in
outpatient management of the febrile neonate. Antibiotic administration without appropriate
culture and laboratory analysis results in the inability to diagnose serious bacterial infection.
19. A 19-month-old female toddler is brought to the ED by her parents with a limp. She has not
been fussy and her parents can’t recall when her limp started. Her mother states that she
appeared to “walk a little strangely” in the morning for the last week but it “seemed to go
away.” The limp has since become more persistent, and her parents have noted some swelling
in her left knee. She has not had a recent infectious illness, and the patient has no hip pain. Her
examination reveals a mildly swollen knee with minimal warmth, no erythema, and good range
of motion. X-rays of the knee are normal. Which of the following is true?
B. Fever, rash, and irritability are common in association with the disorder.
Answer A. This patient’s presentation is most consistent with pauciarticular (also called
oligoarticular) juvenile arthritis, which is the most common type of juvenile arthritis. When
systemic symptoms are present in the setting of arthritis, patients are diagnosed with systemic
onset juvenile arthritis (previously known as Still disease). Pauciarticular juvenile arthritis affects
females more than males, typically begins in the second year of life, and is rare in children older
than 10. The hip joint is almost never affected, but the other large joints, such as the knees,
ankles, elbows, and wrists, are commonly affected. This patient’s presentation of a painless
morning limp that improves throughout the day is common. The course of the illness is
frequently benign, as 50% of patients will experience complete resolution within 6 months. The
disease is most often nonsteroidal anti-inflammatory drug responsive and rarely requires more
powerful therapies. The development of uveitis is the only extraarticular complication common
to the disease. Permanent injury typically precedes symptom onset with respect to uveitis, so
pg. 513
referral to an ophthalmologist is critical. Toxic synovitis is an aseptic inflammatory disorder of
the hip, which is typically postinfectious. Patients with septic arthritis have erythema, pain, and
limited range of motion on examination. Legg–Calvé–Perthes disease is idiopathic avascular
necrosis of the hip.
20. A 14-year-old male is brought to the ED by his mother concerned about Lyme disease. The
patient has a 2-cm circular, erythematous rash on his leg. Which of the following is true?
B. The patient should not be treated unless they recall a tick bite
Answer E. While erythema migrans is classically associated with central clearing, it is more
commonly uniformly erythematous and sometimes has increased central erythema.
Furthermore, a minority of patients recall a tick bite (this varies, but probably about 25%).
Serologic testing should be avoided in the emergency department as it’s often falsely negative in
early Lyme, and positive tests do not indicate active Lyme, particularly in patients who live in an
endemic area. Alternative tests are being developed. Doxycycline remains the treatment of
choice for Lyme even in children. Concerns about dental staining in children <8 years old given a
course of doxycycline are unfounded and not supported by recent published evidence. In
children with allergies, amoxicillin and cefuroxime are alternatives.
21. A 4-year-old male presents with a 1-day history of generalized abdominal pain and multiple
episodes of bloody diarrhea. His mother says he had a hamburger at a local fast-food restaurant
5 days ago but that “they eat there all the time.” His symptoms began with watery diarrhea and
mild abdominal pain but have progressed to more severe pain with bloody diarrhea. He has no
fever. Which of the following statements is true?
E. The hamburger is likely unrelated to this patient’s diarrheal illness as the incubation period is
more commonly 12 to 24 hours
pg. 514
Answer B. E. coli O157:H7 (also known as enterohemorrhagic E. coli or enterohemorrhagic E.
coli [EHEC]) is the most important strain of E. coli that commonly causes diarrhea in the United
States. It is most frequently associated with eating undercooked ground beef. However,
outbreaks from contamination of apple cider, raw milk, and most recently, spinach, have also
been reported. Antibiotics are contraindicated in all cases because they may induce the
expression and release of toxins (Shiga toxins), which may worsen the disease and increase the
risk of developing HUS. HUS is a syndrome characterized by microangiopathic hemolytic anemia,
thrombocytopenia, and renal failure and occurs in as many as 25% of cases (most of which occur
in children). HUS is the most common cause of renal failure in children. TTP is a less frequent
complication of EHEC infection and more commonly occurs in the elderly or
immunocompromised. Infection with EHEC results in a hemorrhagic colitis after an incubation
period, which ranges from 3 to 8 days. Fever is atypical and a different pathogen should be
considered if fever is present.
22. A 5-year-old male presents with right hip pain. There is no history of trauma. Which of the
following is the most likely cause?
B. Osteogenesis imperfecta
C. Transient synovitis
E. Legg–Calve–Perthes disease
Answer C. Transient synovitis is the most common cause of atraumatic hip pain in children. It
usually occurs in children <6 years old, and the cause is unknown. Referred pain to the knee may
be the only complaint, so careful inspection of the hip is mandatory in all patients with isolated
knee pain. Diagnosis is made by excluding other more serious causes of hip pain, namely septic
arthritis, fracture, SCFE, and Legg–Calve–Perthes disease. Patients with transient synovitis are
less likely than those with septic arthritis to have fever, elevated erythrocyte sedimentation rate
(ESR), and tenderness of the hip. Management of transient synovitis is purely supportive with
rest and NSAIDs. Hip fracture in the pediatric patient is suggested by high-force trauma and
severe tenderness is common. Osteogenesis imperfecta is a rare disease that causes problems in
bone synthesis due to collagen defects. Frequent fractures are common and physical
examination may demonstrate blue sclerae, deafness, and ligamentous laxity. Subclinical cases
may be more common than previously recognized. SCFE occurs when the femoral epiphysis slips
off the metaphysis, usually in adolescents. Legg–Calve–Perthes disease is avascular necrosis of
the femoral head due to unknown reasons. It occurs from childhood to adolescence but is less
common than transient synovitis.
pg. 515
23. A 1-week-old infant is brought to the emergency department (ED) with fast breathing,
irritability, and poor feeding. Room air pulse oximetry is 88% and the patient appears to have
decreased activity. His temperature is 99.8ºF. Chest x-ray reveals a slightly enlarged heart with
increased pulmonary consolidation. Lung examination reveals mild wheezes with good air
exchange. Albuterol and oxygen are initiated. Which of the following medications is indicated?
A. Prostaglandin
B. Indomethacin
C. Ribavirin
D. Cefotaxime
E. Propanolol
Answer D. Neonates with significant vital sign abnormalities, including fever or hypoxia, should
be suspected of having a serious bacterial infection until proved otherwise. Even patients who
exhibit strong signs of congenital heart disease should receive antibiotics and an evaluation for
septic cause of the clinical findings. Cefotaxime and ampicillin are indicated for treatment of the
most common pathogens in the neonatal period, Group B streptococci, gram-negative bacilli,
pneumococcus, and Listeria. In the absence of a patient in extremis, therapy that affects ductus
arteriosus patency such as indomethacin or prostaglandin should not be given until the exact
congenital heart defect can be determined. Albuterol is indicated for reactive airways disease,
and ribavirin is used in select patients with respiratory syncytial virus (RSV) bronchiolitis.
Propanolol may be used in certain patients with tetralogy of Fallot during Tet spells to reduce
right ventricular outflow obstruction.
24.A 7-year-old is diagnosed with appendicitis after returning to the ED a day after first
presenting with abdominal pain. Missed cases of appendicitis are most often initially diagnosed
as:
A. Mesenteric adenitis
B. Intussusception
C. Gastroenteritis
E. Pancreatitis
Answer C. In the classic surgical text, “Cope’s Early Diagnosis of the Acute Abdomen,” Cope
states that “the diagnosis of gastritis or gastroenteritis is usually made in the emergency ward
by a young physician who is “not impressed” by a patient’s abdominal pain or physical findings.”
He goes on to write that, “the diagnosis of gastroenteritis in the emergency ward is so often
incorrect as to raise a serious question whenever the emergency physician comes to this
pg. 516
conclusion.” Although this quote employs a bit of hyperbole, gastroenteritis is the most
common misdiagnosis applied to cases of missed appendicitis. In contrast, many patients
initially diagnosed with acute appendicitis have a normal appendix upon appendectomy. In fact,
the rate of negative appendectomy has essentially remained unchanged. Among these patients,
the most common diagnoses are unexplained abdominal pain (35.1%), mesenteric adenitis
(22.8%), lymphoid hyperplasia (10.6%) or other diseases of the appendix (9.9%), gastroenteritis
(4.4%), and ovarian cyst (3.3%).
25. A 4-year-old male presents with progressive periorbital edema, weight gain, anorexia, and
nausea for several weeks. Hypertension is noted on physical examination. Urinalysis
demonstrates 4+ protein. Which of the following is most likely to be present in this patient?
A. Hyperalbuminemia
B. Thrombophilia
C. Hypotriglyceridemia
D. Urinary bacteria
E. Gross hematuria
Answer B. The patient has evidence of nephrotic syndrome with proteinuria accompanied by
generalized fluid overload and nonspecific constitutional symptoms. Periorbital edema is often
the first location where fluid overload is noted. Frank renal failure does not usually occur in
patients with nephrotic syndrome. In children, minimal change disease is the most common
cause. Hypoalbuminemia, thrombophilia, and hyperlipidemia are characteristic. Urinary tract
infections (UTIs) are unrelated. Although microscopic hematuria may be present, gross
hematuria is rare and suggests glomerulonephritis, infection, stone, tumor, or extrarenal cause.
Acute management generally involves careful fluid resuscitation, systemic corticosteroid
therapy, and admission for observation and possible renal biopsy.
26. A 5-year-old male presents with bloody diarrhea for several days, associated with fatigue,
pallor, and malaise. Several kids at school have similar complaints. Blood tests at his
pediatrician’s office demonstrate severe anemia, thrombocytopenia, and renal insufficiency.
Which of the following is the most likely cause?
A. Shigella
B. Salmonella
C. Escherichia coli
D. Rotavirus
pg. 517
E. Child abuse
Answer C. The triad of anemia, thrombocytopenia, and renal insufficiency should prompt
evaluation for either hemolytic uremic syndrome (HUS) or thrombotic thrombocytopenic
purpura (TTP). Fever and neurologic signs and symptoms are more common in the latter, but
the two are thought to be on the same spectrum of disease. The toxin-forming bacterium E. coli
O157:H7 is responsible for most epidemic cases of HUS. Treatment is primarily supportive,
aimed at preventing complications of severe anemia and thrombocytopenia. Plasmapheresis is
used for cases of idiopathic HUS or TTP. Shigella is a less common precipitant of HUS than E. coli.
Salmonella species and rotavirus are not implicated. Child abuse would be unlikely to cause
these symptoms in an epidemic manner.
27. Which of the following best supports a diagnosis of pertussis in a child with cough?
B. Eosinophilia
C. Prolonged course
E. Fever >102°F
pg. 518
severe dehydration. The paroxysmal phase typically lasts from 1 to 6 weeks. During the
convalescent phase, the paroxysms typically become less frequent and less distressing, although
the cough may actually become louder. Although classical pertussis is a clinical diagnosis,
lymphocytosis supports the diagnosis. Absolute lymphocyte counts >20,000 per mm3 may be
seen with a total WBC count >100,000 per mm3. The chest x-ray is most often normal, although
it may demonstrate a “shaggy” right-heart border. However, the presence of an infiltrate may
indicate a secondary infection as bacterial pneumonia may complicate pertussis in up to 20% of
patients. Although the incidence of B. pertussis is rising in the population as a whole, this rise is
almost solely due to an increase in the number of adolescents and adults with the disease.
However, children and infants still represent most cases of B. pertussis. Thirty-eight percent of
cases occur in infants younger than 6 months and 71% of cases occur in children younger than 5
years. Additionally, adolescents and adults more often present with atypical signs and
symptoms such as an isolated spasmodic cough without an associated inspiratory whoop.
Previously vaccinated adults rarely demonstrate the lymphocytosis that is characteristic of
pediatric infections.
28. After physiologic jaundice of the newborn (icterus neonatorum), which of the following is
the most common cause of neonatal jaundice?
B. Cephalohematoma
D. Gilbert syndrome
E. Biliary atresia
Answer A. Breast milk jaundice develops in 2% of breast-fed infants after the seventh day of life.
Levels peak during the second to third week of life and may be as high as 10 to 30 mg/dL.
Treatment is straightforward and involves stopping breast-feeding for 1 to 2 days and
substitution of bottle feeding with formula. This results in a rapid decline in serum bilirubin after
which nursing can be resumed without a recurrence of the prior hyperbilirubinemia. Breast milk
jaundice is an unconjugated hyperbilirubinemia. Hyperbilirubinemia is described as
“conjugated” when direct bilirubin exceeds 2 mg/dL or represents >20% of the total bilirubin
level. Although breast milk jaundice is generally a benign process, phototherapy may be
required if total bilirubin levels exceed 18 to 20 mg/dL. Finally, breast milk jaundice should be
differentiated from breast-feeding jaundice, which is an early-onset unconjugated
hyperbilirubinemia that occurs in the first week in breast-fed infants. The mechanism, as with
breast milk jaundice, is unclear, but is thought to be related to decreased milk intake with
dehydration or reduced caloric intake.
pg. 519
29. An otherwise healthy 12-year-old male is brought to the ED with a chief complaint of chest
pain. The patient has been complaining of the pain over the past week. Which of the following is
true?
A. The friction rub of pericarditis is best heard when patients are lying supine.
B. Cardiac causes of chest pain more commonly occur in the setting of exertion.
D. On physical examination, only diastolic murmurs are cause for concern of an underlying
cardiac problem.
E. Valve disorders are the most common cardiac causes of pediatric chest pain.
Answer B. Chest pain is a frequent complaint among pediatric patients presenting to EDs. Fewer
than 10% of patients are eventually diagnosed with a cardiac cause of chest pain. Most cases are
caused by musculoskeletal etiologies (e.g., costochondritis) or are considered idiopathic. Among
cardiac causes, pericarditis and arrhythmias are the most common. Valve disorders are
uncommon and, when present, are more often diagnosed in early childhood. Recurrent chest
pain that occurs with exertion, chest pain occurring prior to syncope, or a family history of
sudden cardiac death each increases the likelihood of an underlying cardiac problem.
Pericarditis is a common cardiac cause of pediatric chest pain. Patients with pericarditis typically
feel better lying forward and worse when lying supine. However, the friction rub is best heard
with the patient leaning forward or even sitting on all fours. These positions bring the heart
closer to the chest wall allowing for better auscultation. Myocardial infarction is rare in pediatric
patients, but when it occurs, it is most common in males, and it is associated with smoking and
substance abuse (primarily cocaine).
30. A 6-year-old female presents with abdominal pain. She has had moderate, constant
periumbilical pain for several hours with associated nausea. Her parents noted a rash on her legs
and buttocks for several days, which they attributed to poison ivy. Physical examination
demonstrates an afebrile, uncomfortable patient, diffuse abdominal tenderness without true
rebound or guarding, a maculopapular rash on the legs and buttocks, and diffuse joint
tenderness. Which of the following is the most important next step in evaluation?
A. Urinalysis
D. Blood cultures
E. Meckel scan
pg. 520
Answer A. The patient has evidence of Henoch–Schonlein purpura (HSP), an immune-mediated
vasculitis of idiopathic origin. Young children are the highest risk group. Dermatologic,
gastrointestinal, renal, and musculoskeletal findings are seen. The rash of HSP is characteristic: A
maculopapular eruption on the legs and buttocks and almost never involving the upper
extremities or trunk. Abdominal pain is due to intestinal vasculitis or intussusception, which
occurs with higher frequency in patients with HSP than the normal population. Renal
involvement may be due to glomerulonephritis, which is detected as hematuria on urinalysis. In
addition, patients may have mild to nephritic range proteinuria. While the long-term prognosis
of children with renal involvement is good, persistent renal dysfunction accounts for a
significant portion of the morbidity associated with HSP. Current management of HSP involves
potential administration of corticosteroids or intravenous immunoglobulin (IVIG) to prevent and
treat glomerulonephritis. Abdominopelvic CT scan is not indicated, as HSP is a clinical diagnosis.
Despite the possibility of HSP as an immune-mediated response to streptococcal infection, ASO
titers are not routinely indicated, as they do not change management. Blood cultures are not
necessary in these patients except when systemic infection is suspected by fever and focal
abdominal tenderness. A Meckel scan is used to detect the presence of Meckel diverticulum,
which usually presents with painless rectal bleeding rather than the constellation of signs seen
in HSP.
31. A 10-year-old male presents with fever, diarrhea, pallor, and weakness. Renal function is
abnormal and the patient is anemic. A peripheral blood smear reveals schistocytes. Which of the
following is the most likely diagnosis?
A. Henoch–Schönlein purpura
E. Nephrotic syndrome
Answer B. In the setting of renal dysfunction, anemia, and diarrhea, schistocytes suggest a
diagnosis of hemolytic uremic syndrome (HUS). The triad of anemia, thrombocytopenia, and
renal insufficiency should prompt evaluation for either HUS or thrombotic thrombocytopenic
purpura (TTP). Fever and neurologic signs and symptoms are more common in the latter, but
the two are thought to be on the same spectrum of disease. The toxin-forming bacterium E. coli
O157:H7 is responsible for most epidemic cases of HUS. Treatment is primarily supportive,
aimed at preventing complications of severe anemia and thrombocytopenia. Plasmapheresis is
used for cases of idiopathic HUS or TTP. Henoch–Schönlein purpura is a vasculitis heralded by
renal dysfunction in the setting of lower extremity palpable purpura, abdominal pain, and
arthralgias. DIC is due to distortion of the clotting cascade from severe associated illness.
pg. 521
Idiopathic thrombocytopenic purpura causes thrombocytopenia without schistocyte formation.
Nephrotic syndrome causes renal dysfunction without hematologic abnormalities.
32. A 9-month-old male is brought by his parents for evaluation of an abdominal mass that they
noticed while changing his diaper. Physical examination demonstrates a nontoxic, active infant
with a palpable, nontender mass measuring 4 × 6 cm. Which of the following studies is most
likely to reveal the diagnosis?
B. Scrotal ultrasonography
C. Renal ultrasonography
D. Urinalysis
E. Meckel scan
Answer C. Abdominal masses in infants are usually renal in origin, most commonly benign
tumors or cysts. Both neuroblastoma, most often arising from the adrenal glands, and Wilms
tumor, the most common renal malignancy, are frequent causes of abdominal masses. Renal
ultrasonography or CT of the abdomen and pelvis should be performed to better evaluate the
mass. Gall bladder tumors and stones are rare in infants. Scrotal ultrasonography will help to
evaluate groin and testicular pathology but is not useful for abdominal evaluation. Urinalysis is
commonly normal in patients with renal cysts or tumors. A Meckel scan is useful to evaluate a
Meckel diverticulum, which usually presents with painless rectal bleeding rather than mass.
33. A 4-day-old term neonate with an uncomplicated birth history is brought to the ED by his
parents who complain he seems very yellow. He is feeding normally and seems active. Which of
the following is true?
Answer A. Neonatal jaundice is a common and normal part of neonatal life as more than half of
term neonates will develop jaundice in the first week of life. However, severe
hyperbilirubinemia places infants at risk for irreversible neurologic disease as bilirubin is a
pg. 522
neurotoxin. There are several online calculators based on the nomogram developed by Bhutani
et al. that can be used to determine whether or not an infant is at risk for developing dangerous
levels of bilirubin. Phototherapy is the first-line treatment when treatment is indicated except in
the rare cases in which an infant presents with hyperbilirubinemia and clinical evidence of
neurologic compromise (e.g., lethargy). In addition, any infant with a bilirubin level of 25 mg/dL
is in danger of developing neurologic injury and should be admitted for emergent treatment.
The bronze baby syndrome is an uncommon side effect of phototherapy treatment
characterized by a grayish-brown skin color. Bilirubin levels can’t be reliably determined by
physical examination and should always be checked with blood testing, when needed.
34. A 1-year-old presents with sudden onset of high fever for several days, followed by
defervesence and the rash seen on the image (Fig. below). Which of the following is true
regarding this condition?
Answer A. The rash and clinical history describe roseola, caused by human herpesvirus-6 (known
as sixth disease, for this reason). It occurs most commonly in the first 2 years of life and
supportive care is all that is required for management. Lymphadenopathy is extremely common,
pg. 523
seen in almost all cases. Seizures are seen only in a small minority of cases. The “slapped cheek”
rash is actually due to erythema infectiosum, caused by parvovirus B-19.
A. Mitral stenosis
Answer C. The cyanotic heart diseases in children can be remembered by the fact that each
begins with the letter “T”—truncus arteriosus, transposition of the great vessels, tricuspid
atresia, tetralogy of Fallot, and total anomalous pulmonary venous return. Another common
mnemonic is to think of the five cyanotic heart lesions as easy as 1, 2, 3, 4, 5.
Cyanosis generally indicates the presence of right-to-left shunting, causing bypass of the
pulmonary circuit and infusion of deoxygenated blood into the systemic circulation. Cyanosis
due to cardiac disease is generally central (truncal and facial), unlike dehydration or
hypothermia, which usually causes peripheral cyanosis (in the extremities).
A. Anal fissure
B. Hemorrhoids
C. Fistula
D. Ischiorectal abscess
E. Pilonidal cyst
pg. 524
Answer A. Anal fissures are actually the most commonly encountered anorectal problem in all of
pediatrics. However, they are especially common in infants.
37. Parents of a 3-year-old male bring him to the ED with a yellowish, crusting facial rash, as
shown in Figure below. Which of the following is true?
Answer D. This patient has impetigo, which is initially a superficial vesicular eruption that later
develops into multiple honey-crusted lesions. In the United States, S. aureus is the most
common cause, whereas group A Streptococcus is responsible for the bulk of the remainder. In
cases caused by Streptococcus, antibiotic therapy does not reduce the incidence of
poststreptococcal glomerulonephritis. Furthermore, topical therapy with mupirocin is as
effective as systemic therapy, although systemic therapy is recommended when a large area is
involved or when the involvement is near the mouth (allowing topical antibiotics to be licked
away). The lesions are not painful even though they may be pruritic. However, regional
lymphadenopathy is a common associated finding. The lesions are highly contagious and easily
transmitted to other children.
pg. 525
38. An 18-day-old term neonate, weighing 5 kg is brought in by his parents with vomiting and
poor feeding. The patient has a seizure in the emergency room and a point of care blood glucose
reading is 30 mg/dL. The next best step is to infuse:
C. 25 mcg/kg of glucagon
Answer A. Though there is scant evidence to support it, in neonates with hypoglycemia, D10 and
D5 (or D25) are preferred to treat hypoglycemia because D50 is thought to provoke rebound
hypoglycemia in hyperinsulinemic infants. Furthermore, D50 has much higher osmolarity than
the lower concentration fluids which could cause tissue damage if extravasation occurs. While
the exact dextrose dose can deviate from this rule, the easiest way to remember the amount of
dextrose to deliver in acutely hypoglycemic, symptomatic neonates is to follow the “rule of 50.”
The basic formula is % dextrose × mL/kg volume = 50. So:
D10 × 5 mL/kg = 10 × 5 = 50. For a 5-kg child, bolus 5 mL/kg or 25 mL of D10 (this scenario)
39. A 3-year-old previously healthy female is brought to the emergency department (ED) by her
parents with a complaint of fever and cough. Her physical examination findings and chest x-ray
are consistent with pneumonia. She is tolerating PO without difficulty and appears nontoxic.
Which of the following is the most appropriate antibiotic regimen?
A. Doxycycline
B. Erythromycin
C. Levofloxacin
D. Trimethoprim–sulfamethoxazole
E. High-dose amoxicillin
pg. 526
Answer E. Viruses are the predominant etiology of pneumonia in children between the ages of 4
months and 4 years. However, S. pneumoniae is the most common bacterial cause of
pneumonia in this age group. Although chest x-rays are not reliable in distinguishing between
viral and bacterial pneumonias, this patient has an infiltrate, so it seems prudent to treat the
patient with an antibiotic with antipneumococcal activity. Doxycycline and levofloxacin are
contraindicated in children. Erythromycin, a macrolide antibiotic, would be a better choice for
children older than 5 years or younger than 4 months since M. pneumoniae and C. trachomatis
are the most common bacterial causes of pneumonia in these respective groups.
40. One week after returning from India, the parents of a previously healthy 9-year-old female
bring her to the emergency room for evaluation of a 3-day history of progressive fever,
headache, malaise, nausea without vomiting, and generalized abdominal pain with minimal
loose stools but no frank diarrhea. In addition to completing the CDC’s routine vaccination
series, she received additional vaccinations for hepatitis A and yellow fever prior to her trip and
she took mefloquine for malaria prophylaxis throughout her visit. In the emergency room, she is
noted to appear dehydrated but has a pulse in the low 50s. There is also a faint, blanching,
erythematous rash over her chest and upper abdomen. Blood testing reveals anemia with a
minimally elevated white blood cell (WBC) count, and moderately elevated liver function tests.
Which of the following is true?
A. She requires admission for likely dengue fever, and IV clindamycin should be started.
B. Her relative bradycardia in the setting of a high fever may indicate typhoid fever.
E. Her presentation is most consistent with yellow fever, as the vaccine produces incomplete
immunity.
Answer B. This patient’s presentation is most consistent with typhoid fever, a bacterial infection
caused by Salmonella typhi. It is contracted through oral ingestion of contaminated food or
water. While there are two vaccines for typhoid fever (an injectable, unconjugated
polysaccharide vaccine, and an oral, live attenuated vaccine), they cumulatively provide only
about 55% immunity, and this patient did not receive a vaccination prior to travel. In contrast,
the vaccine for yellow fever confers near complete immunity. Ingestion of the organism is
followed by a 1- to 2-week asymptomatic period. Patients then typically develop a fever,
generalized fatigue and malaise, nausea, anorexia, generalized abdominal pain, and a headache.
Children may have diarrhea, but adults frequently have constipation. Nearly a third of patients
also have rose spots, which comprise a blanching, faint erythematous maculopapular rash over
the chest and abdomen. A relative bradycardia in the setting of fever and dehydration is
considered a classic finding, but no studies have determined exactly how often it is present.
Complete blood count (CBC) typically reveals anemia and either leukopenia (more common in
pg. 527
adults) or leukocytosis (more common in children). Liver enzymes are elevated in more than
80% of patients and may create a clinical picture consistent with acute hepatitis.
Fluoroquinolones remain the treatment of choice in adults (bactericidal, concentrated in the
bile), while third-generation cephalosporins are preferred in children. Because of the vague
presentation, other causes of bacterial gastroenteritis or infections which may include
gastroenteritis, such as malaria, dengue fever, amebiasis, or leishmaniasis, must be considered.
Although this patient’s presentation could be consistent with malaria, her compliance with
appropriate prophylactic therapy makes it unlikely. Mefloquine is very effective, providing 91%
efficacy in preventing malaria. Dengue fever is a viral illness that is not treated with antibiotics.
It is the most common mosquito borne viral infection. Patients classically present with fever,
malaise, and severe myalgias and arthralgias (“break-bone fever”). Among returned travelers
with a fever, eosinophilia is most likely a manifestation of helminthic infection such as ascaris or
hookworm.
41. A 10-year-old male presents with chest pain. Which of the following is the most likely cause?
A. Cardiac
B. Gastrointestinal (GI)
C. Psychogenic
D. Musculoskeletal
E. Endocrine
Answer D. Musculoskeletal and pulmonary etiologies combine for roughly half of all cases of
pediatric chest pain. GI, cardiac, and psychogenic conditions each account for approximately
10%. Idiopathic cases comprise a large minority. Endocrine causes are rare. As in adults, most
pediatric patients with acute chest pain should have a screening chest x-ray and EKG to evaluate
for pulmonary and cardiac causes, respectively. Although cardiac causes of pediatric chest pain
tend to not be immediately life threatening, they often radically alter management and are not
as rare as once believed.
42. Which of the following is the most common lead point in cases of pediatric intussusception?
A. Meckel diverticulum
B. Lymphoma
C. Intestinal polyp
E. Peyer patches
pg. 528
Answer E. Recognizable lead points other than Peyer patches are found in only 2% to 8% of
patients. All of the listed choices may serve as lead points in intussusception. However, most
cases are thought to occur when an enlarged Peyer patch telescopes into adjoining bowel. The
most common location is ileocolic.
E. They tend to be more subtler than seizures in older children and adults
Answer E. Neonatal seizures are often subtle, with less motor activity than seizures that occur in
older children or adults. Seizures may involve sucking or chewing, lip smacking, random and
unusual eye movements, rowing, swimming or leg pedaling movements, and unusual sounds.
They may be associated with apnea. This constellation of signs is formally referred to as “subtle”
seizures as opposed to more traditional generalized tonic–clonic events. Absence seizures don’t
typically occur until age 4. Seizures can be due to hyponatremia in which case congenital adrenal
hyperplasia (CAH) should be considered. Barbiturates have long been the first-line therapy for
neonatal seizures although there is no evidence that one particular antiepileptic agent is better
than another.
44. What is the most common cause of lower gastrointestinal (GI) bleeding (LGIB) in children?
A. Anal fissure
B. Hemorrhoids
C. Henoch–Schönlein purpura
D. Food allergy
E. Meckel diverticulum
Answer A. Anal fissures are the most commonly encountered anorectal problem in children and
they are the most common cause of LGIB in this population, particularly in the first 2 years of
life. Meckel diverticulum is the most common cause of substantial GI bleeding in this
population.
pg. 529
45. A 15-year-old male is brought in by his parents with a chief complaint of bloody stools. They
describe the stools as coated with bloody material and report blood on the tissue paper. The
patient reports a history of chronic constipation and straining but has no complaints. He has a
benign abdominal examination, normal vital signs, and a hemoglobin value of 14.5 g/dL. Which
of the following is true?
E. The patient should be referred for outpatient GI evaluation for possible juvenile polyps
Answer B. Hemorrhoids are a common cause of lower GI bleeding in adolescents and adults.
Among other risk factors, hemorrhoids are associated with both diarrhea and constipation
though the exact pathophysiology is not clear. Anal fissures are also a frequent cause of lower GI
bleeding but are most often accompanied by pain. The pain associated with anal fissures may be
less prominent in patients with chronic anal fissures. A 99m technetium pertechnetate infusion,
also called a “Meckel scan” is a test to determine if a patient has a Meckel diverticulum. Meckel
diverticula are uncommon, present in roughly 2% of the population. Of those who have a
diverticulum, only 2% develop complications. GI bleeding can occur because of the acid
produced by ectopic gastric tissue within the diverticulum. Given that this patient has formed
stools coated with blood, a Meckel diverticulum is unlikely. Diverticulosis and polyps are also
uncommon in pediatric patients.
46. A 29-year-old G1P0 is brought in by EMS in active labor. The patient attempted a home birth
but the midwife called EMS after a prolonged, unsuccessful labor. Upon arrival, the patient
spontaneously delivers a male infant who then begins to seize. What is the most likely etiology?
A. Hypoxia
B. Hydrocephalus
C. Intracranial hemorrhage
D. Down syndrome
E. Hypoglycemia
Answer A. Intrapartum or antepartum asphyxia resulting in either global or focal brain ischemia
is the most common cause of seizures in the term neonate. Intracranial hemorrhage accounts
pg. 530
for approximately 15% of cases (most commonly intraventricular or intraparenchymal). Sepsis,
inborn errors of metabolism, metabolic abnormalities (primarily hypoglycemia, hypocalcemia,
and hypomagnesemia), and toxins account for an additional 10%. Neonatal seizures are rarely
idiopathic, so an extensive diagnostic workup must be performed.
47. A 5-year-old female without past medical history presents with fever and pruritic rash on
several areas of her trunk. The parents report a sick contact with similar symptoms at school and
think the rash may be chicken pox. There is no reported cough, shortness of breath, headache,
or stiff neck. Physical examination demonstrates a nontoxic, playful child with a crop of vesicles
in one area and dried crusted lesions in other areas. Which of the following is the most
appropriate next step in management?
A. Aspirin
B. Acetaminophen
C. IV acyclovir
Answer B. Chicken pox is an acute illness caused by varicella zoster virus (VZV) causing fever,
myalgias, and a maculopapular rash progressing to vesicles which then rupture and form dry
crusted lesions. Children are the most common group affected, and serious disease can occur in
adults. Immunocompetent children are treated symptomatically with acetaminophen for fever.
Aspirin should be avoided in children with viral illnesses, as this may predispose to Reye
syndrome. Acyclovir is used in adults, immune-compromised patients, and when there are signs
of encephalitis or pneumonitis. VZV vaccine is indicated for prevention and has no role in acute
management of evident disease. VZV immune globulin is only indicated in
immune-compromised patients in conjunction with acyclovir.
48. Which of the following is the narrowest portion of the pediatric airway?
A. Nasopharynx
B. Oropharynx
C. True cords
D. False cords
E. Cricoid cartilage
pg. 531
Answer E. Unlike the adult, the narrowest portion of the pediatric airway is the cricoid cartilage,
necessitating the use of uncuffed tubes in children younger than 8 years. Other important
considerations for the pediatric airway are the proportionally larger tongue, floppier epiglottis,
more anterior airway position, and shorter tracheal length. These anatomic differences require
slightly different techniques from those used with adult airway management, including an
adjunctive oral airway, frequent use of a straight blade, and more anteriorly directed
laryngoscopic technique. Most children older than 12 years have airway characteristics similar
to adults.
49. A 7-year-old female is brought in by her father after choking on a plastic toy. She was
coughing violently and gasping in the car, so the father tried the Heimlich maneuver and a blind
finger sweep but she seemed to get worse. His daughter is now unconscious and cyanotic. After
performing a jaw thrust maneuver, you fail to locate the foreign body. Attempts to place an
endotracheal tube fail, as the tube seems to be striking an object. What is the best next step?
B. Surgical cricothyroidotomy
E. Needle cricothyroidotomy
Answer E. This patient has complete airway obstruction due to foreign body aspiration. The
classic triad of foreign body aspiration is coughing, wheezing, and decreased or absent breath
sounds. However, approximately 40% of patients may have no significant physical examination
findings. Although this patient initially had partial foreign body obstruction, it progressed to
complete obstruction and she now requires a definitive airway. Back blows and chest thrusts
would be reasonable initial approaches in infants with foreign body aspiration. Abdominal
thrusts can be used in children older than 12 months, although the Heimlich maneuver is the
initial procedure of choice for older children and adults. Blind finger sweeps, which were
advocated in the past, are discouraged as they have the potential of converting a partial airway
obstruction to complete airway obstruction. In the setting of complete airway obstruction, a
definitive airway must be established. The fastest way to accomplish this in this case is by
performing a cricothyroidotomy. A needle cricothyroidotomy instead of a surgical
cricothyroidotomy should be performed in children younger than 8 years. Surgical
cricothyroidotomy is difficult to perform in a small child because of the small size of the
cricothyroid membrane, and it places children at risk for subsequent subglottic stenosis. A
needle cricothyroidotomy is performed using a 12- to 16-gauge angiocatheter and inserting it
through the inferior portion of the cricothyroid membrane into the trachea. The catheter can
then be attached to an adapter from a size No. 3.0 endotracheal tube to allow for bag
pg. 532
ventilation or to high-flow oxygen tubing for percutaneous transtracheal jet ventilation. In either
case, these measures are only temporary until a more definitive airway can be established.
50. A mother brings her 3-year-old daughter to the ED for evaluation of a persistent,
foul-smelling, bloody vaginal discharge. The patient has been complaining of “itching down
there” and her mother noted that she has been frequently placing her hands in her perineal
region. Which of the following is the most likely cause of her symptoms?
A. Trichomonas vaginitis
D. Candidal vaginitis
51. A 4-year-old female presents with signs and symptoms of cystitis. Which of the following is
the most appropriate initial treatment?
A. Amoxicillin–clavulanic acid
B. Ciprofloxacin
C. Doxycycline
D. Cefixime
E. Trimethoprim–sulfamethoxazole (TMP-SMX)
pg. 533
Answer D. Due to rising resistance rates to trimethoprim–sulfamethoxazole, first-generation
cephalosporins, and amoxicillin–clavulanic acid, oral second- or third-generation cephalosporins
should be the first choice for treating uncomplicated pediatric cystitis. Cefixime is a highly
efficacious third-generation cephalosporin. Other choices include cefdinir, cefpodoxime,
ceftibuten. TMP-SMX can be the initial drug of choice if local resistance rates are low.
Ciprofloxacin is a first-line agent for treatment of adult UTIs, but concerns about
musculoskeletal effects in children have prevented its use in the pediatric population.
Doxycycline is not typically used to treat UTIs and has traditionally been contraindicated in
children because it is thought to stain tooth enamel although more recent data suggest that this
doesn’t occur at doses and durations typically used in treating most conventional infections.
52. A mother brings in her 7-year-old daughter with a chief complaint of a rash, colicky
abdominal pain, and achy ankles. The rash is palpable upon physical examination (see Fig.
below). Which of the following is true?
Answer A. This patient has Henoch–Schönlein purpura (HSP), a small-vessel vasculitis that
primarily affects children who present with palpable purpura, arthralgias, abdominal pain, and
glomerulonephritis. Purpura is present in 100% of patients, although 75% of patients have
arthralgias, typically of the ankles, 65% have abdominal pain, and 40% have renal involvement.
Prognosis depends on the presence and severity of renal involvement. Colicky abdominal pain is
pg. 534
the most common gastrointestinal manifestation, though vomiting, bleeding, and more rarely,
intussusception, may occur. In contrast to the typical ileocolic intussusception that occurs in the
general population, patients with HSP experience ileoileal intussusception 70% of the time. In
the absence of renal disease, HSP is self-limited and only supportive care is required. In the
setting of hematuria or proteinuria, corticosteroids may be beneficial but renal consultation
should be sought. When present in adults, HSP is a much more severe disease due to the
increased frequency and severity of nephritis.
53. A 6-month-old male infant is brought in by his mother after 4 days of continuous diarrhea.
He appears mildly dehydrated on examination. Which of the following acid–base disturbances is
likely to be present?
C. Metabolic alkalosis
D. Respiratory acidosis
E. Respiratory alkalosis
Answer B. Diarrhea is the most common cause of a normal anion gap metabolic acidosis. Fluid
from the intestine distal to the stomach is bicarbonate-rich, so diarrhea results in bicarbonate
loss and subsequent metabolic acidosis. As bicarbonate is lost, chloride is avidly reabsorbed by
the kidneys resulting in a hyperchloremic metabolic acidosis.
54. A 4-week-old male infant is brought in by his parents with progressive, projectile, nonbilious
emesis. Labs are drawn and an IV is placed and the patient is sent for an ultrasound.
Ultrasonography reveals a hypertrophic pylorus and the surgeon is consulted for pyloric
stenosis. What are the labs likely to reveal?
Answer C. Infants with pyloric stenosis typically present between 2 and 6 weeks of age with
progressive, projectile, nonbilious emesis. Persistent emesis results in a loss of hydrogen and
chloride ions from the gastric juices (hydrochloric acid) resulting in a hypochloremic alkalosis.
pg. 535
With time, cellular exchange mechanisms pump hydrogen ions into the blood in exchange for
potassium ions resulting in hypokalemia.
55. A 4-year-old female is brought in by her mother, who is worried that “she swallowed
something and it’s stuck.” The patient was in her usual health until this morning’s breakfast,
which she could not swallow and rapidly vomited. An anteroposterior (AP) chest x-ray
demonstrates a coin positioned en face in the upper chest. The patient looks well and is
otherwise asymptomatic. What is the next appropriate step?
A. Glucagon 1 mg IV
B. Endoscopy
C. 12 ounces of soda PO
D. Oral papain
E. Heimlich maneuver
Answer B. Flexible endoscopy is the procedure of choice for removal of esophageal foreign
bodies. A recent trial demonstrated that between 25% and 33% of esophageal coins will
spontaneously pass without complications within 8 to 16 hours of ingestion. Spontaneous
passage is more likely in older, male children with coins lodged in the distal third of their
esophagus. However, this patient has already presented after a tincture of time has passed, and
endoscopy remains standard of care. Papain, a proteolytic enzyme that is a common active
ingredient in commercially available meat tenderizer, has been used in the past to aid in the
passage of impacted meat boluses. However, its use is associated with an unacceptably high
rate of complications including esophageal perforation, aspiration pneumonitis, and
hemorrhagic pulmonary edema. Therefore, its use should be avoided in the ED. Glucagon and
effervescent agents such as carbonated beverages may both prove useful in alleviating an
impacted food bolus. Glucagon is thought to work by relaxing the smooth muscle of the distal
esophagus and most markedly, the lower esophageal sphincter. It does not have any
appreciable effect on upper esophageal motility. Effervescent agents should be avoided in cases
of complete obstruction or obstruction persisting for longer than 24 hours due to the
theoretically increased risk of perforation due to ischemia. The Heimlich maneuver is indicated
as a life-saving technique for laypersons to help dislodge a foreign body which is obstructing a
patient’s airway.
56. A 2-year-old male is brought to the ED by his parents for loud breathing for several hours. He
had upper respiratory symptoms for several days, followed by a persistent, high-pitched cough.
The parents have noticed loud inspiratory noises for the last two evenings. Vital signs are 99.5°F,
112, 24, 93% RA. The patient appears to be in mild respiratory distress, with mild inspiratory
stridor and frequent bouts of coughing. Lungs are clear to auscultation. Which of the following is
the most appropriate management?
pg. 536
A. Amoxicillin
B. Ipratropium
C. Albuterol
D. Dexamethasone
E. Ribavirin
Answer D. The patient likely has laryngotracheobronchitis or croup. Croup refers to viral
inflammation of the upper airway with possible pulmonary involvement. Parainfluenza type 1 is
the most common cause, but other causes include influenza, respiratory syncytial virus,
rhinovirus, and adenovirus. Patients are between 6 months and 6 years of age, and croup is the
most common cause of stridor in this age-group. Fever, respiratory distress, and a barky cough
worse at night are common. Treatment involves humidified oxygen, L-isomer or racemic
epinephrine nebulizer, and steroid therapy with oral or intramuscular dexamethasone.
Admission is required in some cases due to either respiratory distress or hypoxia. Amoxicillin is
not indicated in this viral process. Ipratropium and albuterol are both used for cases of lower
respiratory obstructive disease such as asthma or bronchitis but are not necessary in the
absence of wheezes on pulmonary examination. Ribavirin can be used in some viral infections
but has no proven benefit in croup.
57. A 6-month-old infant born at term is brought by her parents for evaluation of cough. The
patient has been coughing for 3 days, with rhinorrhea and congestion and fevers up to 101°F.
Her past medical history and birth history are unremarkable. Vital signs in the ED are 100.1°F,
133, 42, 90/palp, and 98% RA. The patient is smiling, active, and tachypneic with mild nasal
flaring and lungs exhibit moderate expiratory wheezes. Which of the following is true regarding
this patient’s condition?
B. Ribavirin is indicated.
Answer C. The patient likely has bronchiolitis, most commonly due to RSV. It usually occurs
during the winter months and is more severe in preterm infants. Low-grade fevers, cough, upper
respiratory symptoms, and wheezing are seen commonly. Most patients do not appear toxic,
but up to 10% of patients with bronchiolitis require hospitalization because of hypoxemia and
severe respiratory distress. The only treatment that improves clinical status is oxygen.
pg. 537
Bronchodilator therapy with β- agonists is controversial and not clearly proven to be effective.
Corticosteroids and antibiotics are not indicated. Ribavirin is used in preterm infants or those
with a history of congenital heart/lung disease. Repeated episodes of bronchiolitis as an infant
may increase the risk of developing asthma later in life, but the association between the two
conditions is still unclear and most patients with bronchiolitis do not go on to develop asthma.
58. A 6-year-old male presents with diffuse arthralgias, fatigue, and fever for several days. He
had a “virus” with fever and sore throat several weeks before, which resolved spontaneously.
Physical examination demonstrates a febrile child with significant tenderness and limited range
of motion in his left knee with milder findings in his right wrist. Which of the following is the
most appropriate next step in management?
B. Prednisone 1 mg/kg
C. EKG
D. Urinalysis
E. Lumbar puncture
Answer C. The patient presents with diffuse arthralgias and fever in the setting of a recent
pharyngeal infection. Rheumatic fever is the most important diagnosis to rule out in this setting.
A migratory polyarthritis is common and often involves large joints. Major Jones criteria include
carditis, polyarthritis, chorea, erythema marginatum, and subcutaneous nodules. Minor criteria
include fever, arthralgias, and various study abnormalities. EKG is indicated to assess for the
presence of conduction abnormalities, and further evaluation with echocardiogram may be
necessary to evaluate for valvular abnormalities. Treatment is with anti-inflammatory agents,
antibiotics, and supportive care. Discharging the patient home puts the patient at risk for
valvular and conductive complications of rheumatic fever and is contraindicated. Corticosteroids
are controversial. Urinalysis may be conducted in the course of evaluation to assess for the
presence of poststreptococcal glomerulonephritis but does not aid the diagnosis of rheumatic
fever. Lumbar puncture is not indicated in this case due to the absence of signs and symptoms
of meningitis.
59. A father brings in his 8-year-old daughter with a chief complaint of a low-grade fever and a
facial rash (see Fig. below). The rash seems to spare the nasolabial fold and perioral area. Which
of the following is the cause of this patient’s illness?
A. Rubella
B. Human herpesvirus 6
C. Parvovirus B19
pg. 538
D. Measles virus
E. Group A streptococci
Answer C. This patient has erythema infectiosum (or fifth disease) caused by parvovirus B19.
The illness is common and frequently asymptomatic. In symptomatic children, papules develop
on the cheeks which promptly coalesce to form a bright, erythematous plaque which spares the
perioral area (circumoral pallor) and nasolabial fold. The rash typically lasts for approximately 4
days. Parvovirus B19 is most important in causing aplastic crises in patients with underlying
hemolytic anemias. In addition, infection of nonimmune pregnant women rarely results in fetal
hydrops and death. No treatment of immunocompetent individuals is required, though
practitioners should determine the immune status of exposed pregnant women.
C. T-wave inversion in the anterior precordial leads (V1–V3) is a normal finding in school-aged
children.
Answer C. T-wave examination has limited utility in pediatric EKG analysis. Upright T waves in
the anterior precordial leads (V1–V3) are normal in the neonate, but they invert after the first
week of life and remain inverted until early adolescence, at which time they take on the typical
pg. 539
adult, upright appearance. In some patients, the “juvenile pattern” of T-wave inversion can
persist into a patient’s 20s. In contrast, upright T waves in the anterior precordial leads of an
otherwise healthy child can be a sign of right ventricular hypertrophy (RVH), which may reflect
underlying congenital heart disease. Right axis deviation is a normal finding in healthy neonates
due to the large right ventricular mass, but this resolves over time. Left axis deviation is not
normally seen. Supraventricular tachycardias are the most common dysrhythmias in pediatric
patients. Atrial fibrillation and atrial flutter are rare and are typically only seen in postsurgical
patients after congenital heart disease repair. In general, rhythm disturbances are uncommon
presenting manifestations of congenital heart disease. Ventricular hypertrophy and axis
deviation are common indications of underlying structural heart disease. ST elevation in
otherwise healthy children and adolescents is usually due to benign early repolarization (BER).
However, as in adults, ST elevation can indicate myocardial infarction. Since myocardial
infarction is exceedingly rare in the pediatric population, other findings supporting a diagnosis of
infarction should be sought, such as cardiac enzymes.
61. A 12-month-old female presents with apparent fussiness with urination. Which of the
following is the most appropriate urine collection method for this patient?
A. Diaper collection
B. Bag collection
C. Urethral catheterization
E. Suprapubic catheterization
Answer C. Patients with suspected urinary tract infections require either urethral catheterization
or a midstream clean catch specimen for adequate sampling and culture. In a 12-month-old
child, midstream clean catch would be extremely difficult. Diaper and bag collection methods
are notoriously nonspecific and should never be used. Suprapubic catheterization would be
necessary only in cases of urethral anatomic abnormalities where direct urethral catheterization
would be contraindicated.
62. A mother brings in her 7-year-old daughter to the ED with a chief complaint of painful
vaginal lesions and “bumps,” burning dysuria, and generalized malaise (Fig. 8-11). On
examination, you note tender inguinal lymphadenopathy and the lesions shown. Which of the
following is the most likely cause of her symptoms?
pg. 540
D. Lymphogranuloma venereum (LGV, Chlamydia trachomatis)
Answer A. HSV-2 is the most common cause of ulcerative vulvar and vaginal lesions, as
approximately one in five sexually active adults is infected with the virus. Because it is sexually
transmitted, the presence of HSV-2 in a pediatric patient should trigger a meticulous search for
other signs of abuse. Patients typically develop multiple scattered lesions in varying types and
stages, including vesicles, pustules, and ulcers. The lesions tend to be shallow and painful and
frequently coalesce into larger lesions, particularly in women. Primary infections tend to be
more severe than recurrent infections and are frequently associated with systemic symptoms,
including fever, generalized malaise, headache, and fatigue. Syphilis is initially characterized by a
painless chancre, which disappears without treatment. Chancroid is an uncommon infection in
the United States and is characterized by multiple genital ulcerations associated with a tender
inguinal lymphadenitis called a bubo. The lymphadenopathy is typically unilateral and occurs in
50% of patients. LGV is also a rare disease in the United States and is characterized by a painless
and often overlooked primary genital lesion. Patients typically present during the second stage
of illness, with a tender unilateral lymphadenopathy that may involve the inguinal lymph nodes
both above and below the inguinal ligament resulting in a noticeable groove in between
(“groove sign”). Granuloma inguinale is another rare disease in the United States characterized
by chronic, painless genital ulcerations.
63. A 4-year-old male is brought by his parents because he swallowed some of his
grandmother’s antihypertensive medicine. He is sleepy, bradycardic, and hypotensive. Which of
the following is the most appropriate treatment?
A. Atropine
B. Glucagon
pg. 541
C. Atropine and glucagon
64. A 14-year-old male presents with intermittent bilateral hip pain for several weeks. Physical
examination demonstrates decreased internal rotation of both hips. Radiographs show slippage
of the femoral epiphysis. Which of the following is the most common associated condition?
A. Hyperthyroidism
B. Diabetes mellitus
C. Obesity
D. Renal failure
Answer C. The patient has evidence of slipped capital femoral epiphysis (SCFE). The most
common presenting age is early adolescence, it is more common in boys, and African Americans
are the highest risk population. Obesity is the most common associated condition, but
hypothyroidism may also be involved. Patients typically present with unilateral or bilateral hip or
knee pain, which is intermittent and worse on activity. Physical examination may demonstrate
progressive loss of internal rotation, muscle atrophy, and leg length discrepancy. Lateral, AP,
and frog-leg radiographs show the femoral epiphysis slipping inferiorly and posteriorly off the
femoral neck. Emergency management includes non–weight-bearing and orthopedic
consultation.
pg. 542
65. A 6-year-old female is brought to the emergency room 4 hours after developing a brief
choking episode while playing with her toys. Her chest x-ray is shown in Figure 8-13. Where is
the foreign body located?
A. Esophagus
B. Hypopharynx
C. Trachea
D. Anterior mediastinum
66. A 6-year-old male presents with left hip pain and a limp. There is no history of trauma. The
pain is relieved by rest. Plain radiographs are shown below. Which of the following is true
regarding this condition?
pg. 543
A. It is much more common in boys than in girls.
B. It is usually bilateral.
Answer A. The patient has avascular necrosis of the femoral head, or Legg–Calve–Perthes (LCP)
disease. It is much more common in boys than girls and is usually unilateral (though 10% to 20%
of patients have bilateral involvement). Pain may be referred to the groin or the knee. Young
children are affected more commonly than young adolescents. Etiology is unknown. Obesity is a
risk factor for slipped capital femoral epiphysis (SCFE), but not LCP disease. Management may
be surgical, but this is considered on a case-by-case basis. Joint aspiration is useful to rule out
septic arthritis as a cause for the symptoms, but radiography and MRI are the cornerstones of
diagnosis of LCP disease. Emergency management consists of prompt orthopedic referral or
consultation for consideration of surgical management. Leg-length discrepancy, deformity, and
limitation of movement are important long-term sequelae.
67. A 26-year-old female G2P1 presents to the ED in labor and delivers her infant onto the
stretcher. Upon evaluation, you discover a spontaneously breathing neonate lying relatively still
pg. 544
with its arms and legs flexed, with a heart rate of 120, bluish hands and toes with a pink face,
who gives a slight whimper when suctioned. Its APGAR score is:
A. 5
B. 6
C. 7
D. 8
E. 9
Answer C. The APGAR score is a five-category scoring system in which each category is assigned
0, 1, or 2 points to give a general assessment of neonatal well-being at birth. It is typically scored
at 1 and 5 minutes postdelivery. The test is named for Virginia Apgar, MD, but her last name is
also used as a mnemonic for its categories: Activity (2 for active movement, 1 for flexed limps
and arms, and 0 for being limp), Pulse (2 for pulse >100, 1 for pulse <100, and 0 for no pulse),
Grimace (in response to stimulation with a suction bulb, for example, also called reflex
irritability, scoring 2 points for a strong cry, coughing, sneezing, or pulling away, scoring 1 point
for a whimper or grimace without much responsiveness, and 0 for no response), Appearance
(scoring 2 for being all pink, 1 for peripheral cyanosis, and 0 for central cyanosis or more diffuse
pallor or cyanosis), and Respiratory effort (2 for a strong cry or normal breathing, 1 for irregular
or slow breathing, and 0 for apnea). A score of 7 to 10 is considered normal. This baby receives 1
point for activity (flexed legs), 2 points for pulse (>100), 1 point for grimace (whimper), 1 point
for appearance (peripheral cyanosis), and 2 points for respiration (normal breathing).
A. Fecal leukocytes
B. Fecal erythrocytes
D. Duration of 1 month
Answer E. Rotavirus is an RNA virus that causes a secretory diarrhea in young children, most
often in winter months. The peak age range is between 6 months and 2 years. There are two
commercially available rotavirus vaccines, both of which are given orally as early as 6 weeks of
age. One is a monovalent vaccine while the other contains five human–bovine reassortants.
Both vaccines have demonstrated dramatic benefits in reducing the burden of illness,
decreasing hospitalization rates as well as visits to the ED. While adults may get infected with
rotavirus, they are generally asymptomatic. Among infected children, symptoms include nausea,
vomiting, fever, and severe watery diarrhea. The duration of symptoms is generally <2 weeks.
pg. 545
Rotavirus does not cause an inflammatory gastroenteritis, so fecal leukocytes and erythrocytes
are usually absent. Treatment involves intravenous hydration and supportive care.
Antidiarrheals and antibiotics are not indicated. Patients may require admission for
rehydration—stool studies should be sent for rotavirus culture and the patient should be
contact isolated to help reduce spread of infection to other patients.
Answer D. This patient has Henoch–Schonlein purpura (HSP), which is a systemic, small vessel
vasculitis that most commonly affects children. The classic presentation is a patient with a
palpable, purpuric rash in dependent areas such as the buttocks and lower extremities,
abdominal pain, hematuria, and joint pain. Renal involvement is common, but typically
manifests as microscopic hematuria and resolves without sequelae. However, more severe
presentations from nephritis to nephritic syndrome rarely occur. Long-term prognosis in HSP is
determined primarily by the degree of renal involvement. Patients with HSP do not have
thrombocytopenia. High-dose aspirin is a therapy reserved for patients with Kawasaki disease.
HSP is commonly associated with intussusception. However, in contrast to most patients with
intussusception in which the obstruction occurs in the ileocolic region, patients with HSP
pg. 546
experience ileoileal intussusception. In the great majority of cases, HSP is a benign, self-limited
disease that requires only supportive therapy. Steroids remain controversial and are only used
for patients with severe symptoms.
70. A 6-year-old female is brought to the ED by her parents with multiple loose, nonbloody
stools for 1 day. She has no vomiting or fever. Many of her classmates in school have similar
symptoms. She appears mildly dehydrated, with slightly sunken eyes and dry mucous
membranes. Vital signs are 99°F, 115, 22, 95/55, 100% RA. Basic chemistry and complete blood
count panels are normal. Which of the following is the most appropriate therapy?
D. Metronidazole
E. Nitazoxanide
Answer B. The patient likely has viral gastroenteritis with evidence of mild to moderate
dehydration. She lacks toxic features and does not have signs of severe dehydration. In the vast
majority of cases of pediatric patients with viral gastroenteritis and mild–moderate dehydration,
oral fluid supplementation with a sugar and salt solution is preferred over intravenous hydration
because of lack of complications, reduced cost, and similar clinical outcomes. Normal diet
should be resumed as soon as tolerated by the patient. The BRAT diet has never been shown to
improve outcomes in acute gastroenteritis and is not recommended over a normal diet.
Metronidazole is used for suspected or confirmed cases of C. difficile diarrhea, but the absence
of antibiotic use makes this unlikely. Nitazoxanide is an antiparasitic agent used to treat Giardia
infection, which is usually a subacute watery diarrhea occurring over the course of 1 to 2 weeks.
71. An 11-month-old male infant is brought in by his parents to the ED with a rash (Fig. below).
They state that he appeared to have some discharge from his eyes and then developed a
diffuse, tender “redness” to his skin, which had a “rough” feel to it. Their pediatrician diagnosed
him with a viral syndrome and prescribed oral antipyretics as needed. Since then, his skin
appears to have wrinkled, formed blisters, and is now peeling in large sheets. Which of the
following is the treatment of choice?
A. Amoxicillin
B. Vancomycin
C. Valacyclovir
D. Corticosteroids
pg. 547
E. Continue with supportive care only
Answer B. The patient has staphylococcal scalded skin syndrome, also known as Ritter disease.
The disease is caused by an epidermolytic toxin expressed by S. aureus, phage group II, and
typically occurs in otherwise healthy children. Infection typically begins as an innocuous
infection of the pharynx or conjunctiva until a diffuse erythroderma develops that has a
sandpaper-like feel, resembling scarlet fever. The skin ultimately wrinkles, develops transient
blisters, and then peels in large sheets revealing glossy, moist red skin underneath. Treatment is
directed at S. aureus, and given the prevalence of methicillin-resistant S. aureus, vancomycin is
the best choice.
72. A mother brings her 4-year-old daughter to the ED, who is complaining of persistent perianal
pruritus. The symptoms are worse at night and the mother has had to cut her daughter’s nails
short because she was scratching and irritating her skin. Which of the following is true?
A. The most sensitive test is a stool sample for ova and parasites.
B. The organism responsible for these symptoms can occasionally cause urinary tract infections
and even vulvovaginitis.
Answer B. This patient is infected with the common pinworm or Enterobius vermicularis. It is
probably the most common parasitic infection in the United States. The most common clinical
pg. 548
manifestation is pruritus ani. However, most infections are asymptomatic. Adult worms are
white colored, are approximately 1 cm in length, and live in the cecum. At night, pregnant
female worms containing an average of 10,000 ova migrate to the perianal skin, deposit their
eggs, and die. The resulting pruritic sensation induces the patient to scratch and enables further
autoinoculation or spread to other persons unless the patient engages in proper hand washing
before touching others. The most sensitive test is the “scotch tape” test in which tape attached
to a tongue blade is pressed against the perianal skin in an attempt to affix some of the
Enterobius ova to the tape. The contents of the tape are then spread on a slide and viewed
under a microscope in toluene. Stool samples for ova and parasites are not effective because
the organism is not shed in the stool. Metronidazole is not effective for treatment. Instead,
treatment is with albendazole, mebendazole, or pyrantel pamoate. Eosinophilia generally does
not occur. On occasion, Enterobius may cause urinary tract infections and vulvovaginitis through
retrograde migration into the urethra or vagina. Interestingly, girls with urinary tract infections
are twice as likely to have a concomitant pinworm infection.
Answer A. Though the mechanisms of cerebral edema are unknown, it is clear that children with
DKA are susceptible to cerebral edema as a consequence of treatment. Children <5 years old are
at highest risk. Initially the rate of fluid administration was tagged as the culprit, though it is not
clear that the rate of fluid administration is the cause. Still, most guidelines advise slow fluid
resuscitation with an aim to replete the fluid deficit over 48 hours. Furthermore, there appears
to be no benefit to fast fluid administration. Most patients have a significant total body
potassium deficit but fluid administration should occur before potassium depletion to avoid
dangerous, abrupt increases in potassium. Not only should bolus insulin be avoided, but lower
insulin infusion rates of 0.05 units/kg/hour may cause less hypokalemia without impacting the
time to DKA resolution. Sodium bicarbonate has not been shown to be helpful and is not
recommended unless the patient is severely acidotic (pH <6.9) with evidence of problems with
cardiac contractility due to acidosis or in the case of life-threatening hyperkalemia.
74. A 14-year-old male comes to the ED for a diffuse rash. He was in his primary care doctor’s
office 4 days prior for a sore throat and fatigue and was told he had a viral throat infection. The
pg. 549
patient’s mother insisted that the patient be given an antibiotic, which he has been taking
regularly since then. What antibiotic has this patient most likely been taking?
A. Penicillin V
B. Doxycycline
C. Ciprofloxacin
D. Amoxicillin
E. Clindamycin
Answer D. This patient has Epstein Barr virus (EBV) pharyngitis, also known as mononucleosis. As
it is a viral infection, antibiotics have no role in treatment. Furthermore, the specific use of
amoxicillin may result in the development of a morbilliform (i.e., measles-like) rash. Although
this does not represent a true allergy, nor is it dangerous to the patient, it is an irritating and
unnecessary outcome. The tonsillitis in EBV infections is nonexudative and may be considerably
large, rarely resulting in airway obstruction.
75. A 4-day-old neonate is brought in by her mother for irritability, excessive shaking and
tremulousness, and an abnormal cry. The infant’s glucose level is 20. Which of the following is
the best initial treatment of this infant?
B. 5 mL/kg of D10
Answer B. As with adults, neonatal hypoglycemia is treated with dextrose. Traditionally, D25 and
D50 have been discouraged because of rebound hypoglycemia in hyperinsulinemic infants. In
addition, D25 and D50 are thought to put the infant at risk for a potentially dangerous rise in
plasma osmolarity. In addition, the amount of glucose to deliver to hypoglycemic infants is best
determined by following the “rule of 50.” This simple rule stipulates that the volume of dextrose
given per kg of body weight multiplied by the dextrose concentration should equal 50. For
example:
pg. 550
Then, all that is needed is the weight of the infant. Glucagon is useful as a second-line agent if
the infant is refractory to dextrose. Octreotide and diazoxide are useful agents in infants with
hyperinsulinemia but are not usually necessary in the ED.
76. A 4-year-old male presents with fever, upper respiratory symptoms, and mild conjunctival
injection. His mother states that he is unvaccinated due to personal beliefs of harm due to
vaccines. Physical examination of the oral cavity reveals the image shown (Fig. below). Which of
the following is the likely diagnosis?
A. Mumps
B. Rubella
C. Measles
D. Chickenpox
Answer C. The picture indicates small whitish spots on the buccal mucosa consistent with Koplik
spots. Koplik spots, though uncommonly seen, are pathognomonic for measles. They tend to
occur just before the onset of the classic measles rash (intensely red, diffuse, and
maculopapular), about 3 days after onset of symptoms. Vaccines can prevent the illness. Mumps
causes a nonspecific viral syndrome combined with parotid enlargement. Rubella causes a viral
syndrome with a rash that is less intensely red than measles. Chickenpox occurs from the
varicella zoster virus and causes discrete maculopapular lesions with vesicles. Haemophilus
influenza is a bacterium that causes a variety of illnesses such as otitis media and meningitis.
pg. 551
77. A 2-year-old male is brought in by his parents with a rash on his trunk. On examination, you
discover a rose-colored maculopapular rash on his chest, neck, and arms. The patient is
currently afebrile, but the parents tell you that the patient was seen by one of your colleagues
yesterday for a febrile seizure. Which of the following is the most likely diagnosis?
A. Roseola infantum
B. Rubeola
C. Rubella
D. Erythema infectiosum
E. Scarlet fever
Answer A. Roseola infantum, also known as exanthem subitum (sudden rash), or sixth disease, is
caused by human herpes virus 6. Patients develop a sudden-onset high fever from 103°F to
106°F with hardly any associated symptoms. Owing to the rapid rise in temperature and high
fever, however, febrile seizures may occur. The fever lasts for 3 to 4 days and then abruptly
subsides, at which point the rash begins. The term exanthem subitum or sudden rash refers to
the startling development of a rash just when the patient appears to be recovering. The rash, as
with the illness in general, is benign and self-limited. No treatment is necessary. Rubeola
(measles) is characterized by the finding of Koplik spots on the buccal mucosa. The hallmark of
rubella is generalized lymphadenopathy. Erythema infectiosum is distinguished by its “slapped
cheek” appearance. Scarlet fever is caused by group A streptococci and is characterized by its
sandpaper rash.
78. A 3-year-old male is brought to the ED by his parents with a chief complaint of excessive
drowsiness, fever, and decreased oral intake. Examination reveals an ill-appearing child, with
tachypnea and increased work of breathing. He is pale, and his extremities are cool and mottled
with decreased peripheral pulses. He is not interactive during the examination, and his vitals
reveal T 102.7, P 185, BP not measurable, RR 56, SaO2 93%. An IV is established and the patient
is given broad-spectrum antibiotics and three boluses of normal saline at 20 mL/kg per bolus, for
a total of 60 mL/kg, with little improvement in his perfusion or clinical status. Which of the
following vasopressors is the initial preferred agent?
A. Epinephrine
B. Dopamine
C. Norepinephrine
D. Milrinone
E. Dobutamine
pg. 552
Answer A. This patient is in severe septic shock. Pediatric patients with shock should first receive
aggressive IV fluid resuscitation with three boluses of normal saline at 20 mL/kg per bolus, for a
total of 60 mL/kg of fluid. Patients who do not respond to this fluid infusion are said to be “fluid
refractory” and require vasopressors. However, vasopressors can be considered and used
before 60 mL/kg fluid is infused. The choice of agent depends on the patient’s circulatory status.
Patients in “warm shock” have high cardiac output (CO) and low systemic vascular resistance
(SVR), which results in bounding peripheral pulses, and warm extremities (hypotensive,
vasodilated). Such patients need to increase their SVR, so norepinephrine is used. Patients with
cold shock, such as this patient, have low CO and high SVR, leading to diminished peripheral
pulses and cool, cyanotic, or mottled extremities (hypotensive, vasoconstricted). These patients
are in a more advanced stage of shock relative to patients with warm shock. The priority in such
patients is to increase CO without affecting SVR, so epinephrine is used. It is often impossible to
differentiate the “stage” of shock, and dopamine is often used as the first-line agent, though it is
best used in patients who are normotensive with evidence of poor peripheral perfusion. The
phosphodiesterase inhibitors, such as milrinone, provide increased inotropic support and may
be used as adjuncts but should not be used as monotherapy.
79. A 3-year-old male had a ventriculoperitoneal shunt (VPS) placed at 22 months due to
hydrocephalus. It has not been revised since then and he has had no problems with it. He is now
brought to the EDwith a history of fever of 38.2°C, headache, and “fussiness.” Which of the
following is true about this patient?
Answer E. There is a dearth of data regarding pediatric patients with VPS infections. However,
several studies have suggested that as many as one-third of patients may present with
nonspecific clinical findings. The risk of infection is highest in the first 6 months after insertion or
instrumentation (in the case of revision). In addition, patients younger than 4 years old and
older than 61 years old have been shown to be at increased risk of infection. LP should never be
performed without first obtaining a CT scan and reviewing the findings with a neurosurgeon.
The mortality of VPS infection is roughly 30% to 40%. As many as one-third of patients with VPS
infection may present with signs and symptoms of obstruction or VPS failure with or without a
fever. These include hydrocephalus, papilledema, hypertension with concomitant bradycardia,
and irregular respirations (Cushing response), personality changes, ataxia, and cranial nerve
palsies. Over 80% of patients with a VPS infection have peripheral leukocytosis.
pg. 553
80. The development of lethargy, headache, and vomiting in a pediatric patient being treated
for diabetic ketoacidosis (DKA) suggests the presence of:
A. Meningitis
B. Cerebral edema
C. Worsening acidosis
E. Hyponatremia
Answer B. Cerebral edema is the most feared complication of DKA treatment in children.
Although reports vary, cerebral edema complicates roughly 1% of pediatric DKA cases, with a
mortality rate of 20% to 90%. Survivors have persistent neurologic sequelae 20% to 40% of the
time. The development of cerebral edema is almost exclusively a complication of pediatric DKA.
There is still no consensus regarding the etiology of cerebral edema in pediatric DKA. However,
early clinical signs include headache, vomiting, decreased arousal, relative bradycardia, and
relative hypertension.
81. A 4-year-old male presents with penile pain. His physical examination is shown in Figure
below Which of the following is the correct diagnosis?
A. Phimosis
B. Paraphimosis
C. Balanitis
D. Testicular torsion
E. Scrotal hernia
pg. 554
Answer B. The patient has symmetric swelling of his foreskin behind his glans without active
retraction indicative of a paraphimosis. This is a urologic emergency that can occur in
uncircumcised patients which requires prompt reduction to prevent necrosis of the glans.
Reduction is performed by squeezing the glans for several minutes to clear the capillaries of
blood and pulling the foreskin over the glans. A dorsal slit procedure may need to be performed
if noninvasive reduction is unsuccessful. A phimosis refers to inability to retract the foreskin over
the glans—it does not require emergent reduction. Balanitis refers to bacterial or fungal
infection of the glans without concomitant foreskin infection (which would be called
balanoposthitis). Testicular torsion and scrotal hernia are not often evident on simple inspection
and require palpation and ultrasonography to definitively diagnose.
C. Most frequently occurs in the long bones such as the tibia and femur
Answer E. Though it is not rare, osteomyelitis is a relatively uncommon infection among normal
hosts. However, it occurs with increased frequency in patients with sickle cell disease, HIV, and
in dialysis patients or other chronically ill patients with indwelling catheters. In contrast to
pg. 555
adults, osteomyelitis most commonly occurs due to hematogenous spread in pediatric patients.
Staphylococcus aureus accounts for 50% of infections, though Salmonella is the most common
isolate in patients with sickle cell disease. The long tubular bones are most frequently affected
(tibia > femur > humerus), while the vertebrae are most commonly affected in adults. X-rays are
frequently nonspecific, or normal in the early phase of illness, as are the WBC count, erythrocyte
sedimentation rate, and CRP. The illness typically begins gradually with symptoms of fever and
malaise. However, as the infection intensifies in the affected bone, patients complain of focal
bone pain, and there may be some associated soft-tissue swelling. This should lead to imaging of
the area which yields the diagnosis.
83. A 1-week-old infant is brought to the ED with central cyanosis. Pulse oximetry is 85% on
maximal oxygen therapy, and chest x-ray is shown in Figure below. Which of the following
medications may be indicated in the treatment of this patient?
A. Prostaglandin E1
B. Albuterol
C. Indomethacin
D. Aspirin
E. Ribavirin
Answer A. The chest x-ray shows an enlarged, boot-shaped heart, consistent with tetralogy of
Fallot. Patients with suspected tetralogy of Fallot should be managed in a step-wise
manner—oxygen, fluid resuscitation, morphine, and α-agonist therapy. Prostaglandin E1 may be
indicated in neonatal patients to prevent closure of the ductus arteriosus and ensure enough
pg. 556
shunting of blood to the pulmonary vasculature. The major adverse effects of prostaglandin E1
are apnea and hypotension. Albuterol is useful in cases of reactive airway disease but does not
benefit with congenital heart disease. Indomethacin promotes closure of the ductus and would
be detrimental. Aspirin is never indicated in pediatric patients except in the setting of Kawasaki
disease. Ribavirin is indicated in select patients with RSV bronchiolitis.
84. A term female neonate develops purulent ocular discharge on day 3 of life. Which of the
following etiologies carries the greatest morbidity in this patient?
A. C. trachomatis
B. N. gonorrhoeae
C. S. aureus
E. No organism—chemical conjunctivitis
Answer B. The most dangerous cause of neonatal conjunctivitis is N. gonorrhoeae. It is less often
seen in Western countries due to chemoprophylaxis but is still an important cause of
ophthalmia neonatorum due to its potential for invading the intact corneal epithelium and
causing blindness. Treatment should be prompt with topical and parenteral antibiotics. Choices
A and C occur more often in slightly older infants and cause less severe sequelae. Choices D and
E may be more common given the practice of prophylaxis of all neonates with erythromycin
ointment, but both processes are easily halted by removing the erythromycin and cause no
long-lasting sequelae.
85. A mother brings in her 12-year-old son with a chief complaint of alopecia. The patient denies
any complaints and has been well recently. The lesion is shown in Figure 12-5. Which of the
following is the treatment of choice?
B. Clotrimazole cream FENa = (urine Na) (plasma Cr)/(urine Cr) (plasma Na)
D. Terbinafine lotion
E. Oral griseofulvin
pg. 557
Answer E. This patient has tinea capitis, which is a fungal infection of the scalp usually caused by
Trichophyton tonsurans. Tinea capitis should be suspected in all prepubescent children with hair
loss, particularly if the hair loss is focal and incomplete, or is associated with scale and
lymphadenopathy. There may be scattered hairs that are broken near the base within a
generalized area of alopecia, resulting in the appearance of black dots on the scalp surface
(“black dot” ringworm). Although topical agents such as selenium sulfide and topical
ketoconazole may reduce infectivity, they are insufficient as monotherapy. In all cases, oral
therapy with griseofulvin (either micronized or ultra micronized) is required. Transmission from
person to person is common, and Trichophyton remains infectious in combs and hairbrushes for
long periods.
86. A 6-year-old child presents after a seizure. She has had acute gastroenteritis for the past 2
days. Electrolytes are normal. Which of the following is the most likely cause?
A. Salmonella
B. Shigella
C. Campylobacter
D. Yersinia
E. Vibrio
Answer B. Acute gastroenteritis is most commonly caused by viruses, but bacteria account for
an important subset of cases. In the absence of severe electrolyte abnormalities, seizures
associated with diarrhea are often because of Shigella species, which are the most common
bacterial causes of acute gastroenteritis overall. Salmonella is not typically associated with
pg. 558
seizures. Campylobacter and Yersinia species may cause an illness that mimics appendicitis.
Vibrio species usually cause a typical, nonspecific gastroenteritis.
87. An 18-month-old previously healthy child is brought in by his mother with bilateral oral
commissure burns after biting an extension cord. He cried immediately after the injury, did not
lose consciousness, and he has been acting normally since the injury. On examination, you note
a 5-mm burn to both of the patient’s oral commissures. Which of the following is true?
B. Delayed labial artery bleeding can usually be managed with direct pressure.
C. All patients with bilateral oral commissure burns should have a CT scan of the face to search
for deep tissue injury.
Answer B. Generally, pediatric patients with oral burns can be safely discharged home with
minimal or no ancillary testing. Delayed labial artery bleeding occurs 7 to 14 days after the initial
injury, when the eschar falls off the wound. It typically responds to local, direct pressure,
although patients will occasionally require a figure-8 suture to control bleeding. Otherwise, the
most serious complication of burns to the oral commissure is a cosmetic facial deformity.
Therefore, patients should be urgently referred to a facial or oral surgeon for further evaluation
and possible oral splinting. There is no correlation between an isolated oral burn and cardiac
injury or myoglobinuria. However, the patient should receive a thorough examination to ensure
that there are no other contact wounds indicating a more extensive electrical path.
pg. 559
Answer D. Nearly 90% of newborns with a dislocated hip or reduced hip that can be dislocated
on examination (Barlow test) most often have normal hips by 2 months of age, provided those
with a dislocated hip have the hip reduced upon examination (which allows for normal
development). Testing of the hips is a two-stage process. Barlow test determines whether a
reduced hip can be dislocated or subluxed from the joint. The hips are flexed to 90 degrees, the
thighs are adducted, and posterior pressure is applied in line with the femoral shaft to dislocate
the hips posteriorly. A positive test occurs when dislocation is felt or heard as a click. Ortolani
test confirms that the hips are dislocated or subluxed. Once the hip is posteriorly displaced, the
thigh is abducted, which reduces the femoral head with a palpable clunk. Infants with a positive
test warrant close follow-up with repeat examinations and ultrasound at 4 to 6 weeks. However,
most infants continue to develop normally. The cause is not known, but it is more common in
the left hip and in female infants. While patients with Down syndrome and Ehlers–Danlos
syndrome more frequently have congenital dislocation (now called developmental dysplasia or
developmental dislocation), it is most common in otherwise normal infants. In untreated
patients, a leg-length discrepancy and the Trendelenburg gait pattern (in which weakness of the
ipsilateral hip abductors causes the pelvis to drop to the unaffected side while walking) often
develop.
89. A 4-year-old female presents with vomiting and diarrhea. She is diagnosed with acute
gastroenteritis. Presence of which of the following findings still allows the patient to be a
suitable candidate for oral rehydration therapy (ORT)?
A. Continued diarrhea
B. Lethargy
C. Shock
D. Abdominal rebound
E. Bowel obstruction
Answer A. Standard treatment for pediatric patients with acute gastroenteritis is ORT. Standard
solutions are available and consist of hypotonic fluid and essential electrolytes. Administration
of ORT may occur in the ED, hospital floor, or at home. Patients may continue to have vomiting
and/or diarrhea and still be candidates for ORT. Contraindications to ORT include shock,
lethargy, severe abdominal tenderness or rebound, bowel obstruction, and severe underlying
medical illness. Failure of ORT should necessitate intravenous fluid hydration. Treatment of
acute gastroenteritis with ORT observed in the ED is cost-effective and clinically efficacious.
Hospital admission may be prevented by this ED observation of parent-administered ORT.
pg. 560
90. A 3-year-old female swallows a button battery. Plain radiographs demonstrate that the
battery is lodged in the esophagus. Which of the following is the most appropriate next step in
management?
A. Expectant management
B. Endoscopic removal
D. Activated charcoal
Answer B. Button batteries lodged in the esophagus or trachea can cause obstruction, necrosis,
and perforation. Esophageal button batteries should be removed urgently with upper
endoscopy. Any button battery distal to the esophagus may be managed expectantly and will
likely pass without any specific treatment. Ipecac is rarely indicated for any ingestion anymore.
Activated charcoal is not indicated for foreign body ingestions. Whole bowel irrigation may be
useful to help speed passage of postesophageal button battery that has been slow to progress
with expectant management alone.
91. A 14-year-old female presents with a blanching, erythematous rash all over her chest and
back and arms and legs shortly after starting antibiotics for a sore throat a few days prior to
presenting to the ED. The patient’s mother reports that the strep swab was negative but
antibiotics were started because it “looked like strep.” She has posterior cervical
lymphadenopathy and bilateral tonsillar exudates on examination. Which of the following is
true?
Answer C. The patient presents with findings consistent with infectious mononucleosis (bilateral
tonsillar exudates, posterior cervical lymphadenopathy) as well as a generalized rash after
starting antibiotic. Amoxicillin and ampicillin are the classic agents responsible for the rash,
though it has been associated with other antibiotics. As the patient has Epstein-Barr virus (EBV),
there is no indication to change the antibiotic. The patient does not have scarlet fever, which is
pg. 561
the rash associated with group A strep pharyngitis. Pastia lines are confluent lines of petechiae
found in the skin creases of patients with scarlet fever.
92. A 3-week-old term male neonate is brought in by EMS in respiratory distress, with minimal
wheezes, cyanotic with a room air SaO2 of 60%. His father reports this began after the baby
became fussy while feeding. Applying a 100% nonrebreather results in no improvement. Which
of the following is the best next step?
Answer B. The patient has tetralogy of Fallot (TOF), and is experiencing a “tet spell” which is a
hyper-cyanotic episode due to near complete right ventricular outflow tract (RVOT) obstruction
resulting in severe right to left shunting and cyanosis. Though mild episodes are often
self-limited, they can be life threatening. Typically, oxygen has already been applied to a
cyanotic infant. Thus, the first step is to place the neonate into a knee to chest position to
increase systemic vascular resistance and decrease right to left shunting. If the spell fails to
resolve, morphine is infused at 0.1 mg/kg along with IV fluids (20 mL/kg). If the spell continues,
intravenous beta-blockers (such as esmolol or propranolol) are used followed by phenylephrine
93. Which of the following is true regarding pediatric community-acquired pneumonia (CAP)?
B. Toddlers and young children more frequently develop CAP than middle-aged adults.
Answer B. With the possible exception of elderly patients older than 75 to 80 years, the annual
incidence of pneumonia in children under 5 years is higher than at any other time of life. M.
pneumoniae is the most common cause of CAP in “school-aged children,” 5- to 15-year olds, as
well as in young adults. Increasingly, Chlamydia pneumoniae is thought to be a common cause
of pneumonia in children aged 5 to 15 years, although Mycoplasma remains the chief cause of
pg. 562
pneumonia in this group. The most common causes of pneumonia in the neonate, from birth to
3 weeks, are group B Streptococcus, gram-negative enterobacteria (e.g., E. coli), and Listeria
monocytogenes. Such infections are uncommon but can be severe when present. Between the
ages of 3 weeks and 3 months, Chlamydia trachomatis is most common (i.e., not C.
pneumoniae) followed by S. pneumoniae. Between 4 months and 4 years, viruses (e.g.,
respiratory syncytial virus [RSV], parainfluenza virus, influenza, adenovirus, rhinovirus) are the
most common cause. S. pneumoniae remains an important cause of bacterial pneumonia in this
age-group, but the use of the pneumococcal conjugate vaccine has reduced its role as well as
the morbidity associated with CAP caused by S. pneumoniae. It is no easier to differentiate
between typical and atypical pneumonia in a pediatric population than in adults. Studies have
demonstrated that the presence of symptoms that may suggest a viral etiology to pneumonia
such as rhinorrhea, myalgias, or an illness in a family member do not help to determine the
cause of pneumonia.
94. A 5-year-old female presents with the lesion shown in Figure below. It is soft, boggy, and
very tender. Which of the following is the most appropriate next step in management?
A. Doxycycline PO
C. Griseofulvin PO
D. Topical triamcinolone
E. Topical terbinafine
pg. 563
Answer C. The patient has a kerion, which is an inflammatory lesion caused by an immune
response to tinea capitis (due to Trichophyton or Microsporum species infection). The lesion can
appear similar to an abscess, but kerions are more broad-based and often covered by scales.
Incision, drainage, and oral anti-bacterials are not indicated, as no bacterial infection commonly
exists. Oral antifungal therapy (with griseofulvin or newer antifungals such as itraconazole) is
superior to topical antifungal therapy. Oral therapy must be continued for 6 weeks, as is
common with many fungal infections. Kerions are often diagnosed when patients return to seek
care for lesions which were incised and drained and exhibit no improvement.
95. A 5-year-old male presents with fever and sore throat. Examination reveals exudative
tonsillitis and tender cervical lymphadenopathy. You suspect group A streptococcal pharyngitis.
Which of the following is true regarding treatment for this condition?
C. The risk of side effects from antibiotics is higher than the risk of acute rheumatic fever
Answer C. Group A streptococcal (GAS) pharyngitis carries the risk of acute rheumatic fever and
acute glomerulonephritis. Treatment with antibiotics may decrease the risk of rheumatic fever,
but has no effect on the development of glomerulonephritis. In the Western world, antibiotics
are controversial due to the low rate of rheumatic fever and the greater risk of side effects from
antibiotics. No strains of GAS pharyngitis have been shown to be resistant to penicillins.
Cephalosporins are the preferred alternative if penicillins are contraindicated as resistance to
macrolides has been exhibited. Intramuscular therapy is an excellent alternative to oral therapy
and is the preferred treatment modality when compliance issues are suspected.
96. A 4-year-old male is brought to your office by parents with a fever to 102°F over the last 2
days, malaise, and complaint of right ear pain. Examination reveals an active, febrile child with a
bulging right tympanic membrane (TM) minimally mobile on insufflation. Which of the following
is the most likely pathogen?
A. Mycoplasma pneumoniae
B. Streptococcus pyogenes
C. Moraxella catarrhalis
pg. 564
D. Hemophilus influenza
E. Adenovirus
Answer C. The most common causes of acute otitis media are S. pneumoniae and M. catarrhalis.
M. pneumoniae is an uncommon cause of otitis media, especially in this age-group. The
incidence of infections due to H. influenzae has decreased since the introduction of the HiB
vaccine. Viruses as a group account for up to one out of every six cases of otitis media—RSV is
the single most common viral cause of otitis media. Group A streptococcus is the least common
of the bacterial pathogens listed.
97. A 5-day-old neonate presents for routine follow-up. The child is acting normally.
Funduscopic examination demonstrates bilateral retinal hemorrhages. Which of the following is
the most likely cause?
A. Child abuse
B. Accidental fall
D. Intracerebral hemorrhage
E. Congenital finding
Answer C. Although retinal hemorrhage is a well-known marker for child abuse (shaken baby
syndrome), it is present in almost half of all neonates, due to normal birth trauma. Resolution
tends to occur in 1 month.
pg. 565
pg. 566
Toxicology, drug and food
allergy
➢ Chapter includes:
Drugs abused, over dose, allergy and management
Toxin ingestion and management
Drugs side effects and management
Multiple drugs reaction
Food allergy and management
Anaphylaxis and anaphylactic shock
Steven-Johnson syndrome
Allergy and skin relation
pg. 567
1. A 29-year-old female with a history of gout presents with a diffuse rash consisting of
erythematous target lesions as well as oral sores for several days picture shown below. She
recently started a course of medication for a urinary tract infection the week prior but cannot
remember the name. Which of the following is the most likely cause?
A. Ciprofloxacin
B. Cefdinir
C. Cephalexin
D. Trimethoprim–sulfamethoxazole
E. Macrodantin
Answer D. The patient likely has Stevens–Johnson syndrome (SJS), most commonly caused by
certain drug exposures. Sulfa drugs, allopurinol, carbamazepine, and phenytoin are usually
implicated, though many other drugs have been known to cause SJS. SJS exists on the same
spectrum as toxic epidermal necrolysis and can exhibit multiorgan involvement, though most
cases are restricted to the skin and mucous membranes. Diagnosis is clinical, and treatment
revolves around good supportive care, especially adequate IV hydration. Corticosteroid use is
controversial.
pg. 568
2. Which of the following is true regarding nitroprusside?
Answer B. Nitroprusside is a strong arterial and venous dilator only available as an IV drip. It is
easily titrated and provides effective and predictable BP control. Extravasation of nitroprusside
causes severe skin necrosis. Nitroprusside is metabolized to thiocyanate and excreted renally.
Cyanide toxicity is not common, but thiocyanate toxicity can occur in renal failure, causing
systemic symptoms. Nitroprusside may increase cerebral blood flow and ICP. Thiocyanate may
cause damage to the fetal thyroid, and nitroprusside should be avoided during pregnancy.
Nitroprusside is extremely fast acting, with a rapid onset and offset, making it ideal for rapid,
predictable BP control.
3. A third-year medical student presents to the ED with diffuse arthralgias of the hands, wrists,
and knees. She has been taking isoniazid (INH) because she was exposed to a patient with active
tuberculosis and subsequently had a positive purified protein derivative (PPD) test. She is most
likely suffering from a syndrome mimicking:
A. Systemic sclerosis
C. Gouty arthritis
D. Rheumatoid arthritis
E. Sjögren syndrome
Answer B. Hydralazine, isoniazid, and procainamide may all precipitate a lupus-like syndrome.
SLE typically affects the hands, wrists, and knees and is most common in young women of
childbearing age.
4. A 22-year-old male presents with rash, lightheadedness, and generalized malaise. He denies
fever or pruritus. A few hours before presentation, he was seen in another emergency
department (ED) and received treatment for syphilis. He denies any medication allergies. His
pg. 569
vital signs are 99.2, 94, 16, 134/65, 99% RA. His physical examination demonstrates a normal
uvula, no pulmonary wheezes, and a faint macular rash on his trunk and abdomen, which he
states was there before he received the treatment today. Which of the following is the most
appropriate next step in management?
B. IM epinephrine
D. Prednisone
Answer E. The patient is exhibiting signs of the Jarisch–Herxheimer reaction after treatment for
secondary syphilis with penicillin G. The reaction occurs because massive death of spirochetes
on exposure to the penicillin causes systemic symptoms in a serum sickness-like reaction.
Treatment is symptomatic with acetaminophen or ibuprofen. An allergic reaction to the
antibiotic that he was given is possible, but there were no urticaria demonstrated on physical
examination and the rash that was present was already there before the antibiotic treatment.
The patient should not be intubated or given epinephrine because of the lack of upper airway or
pulmonary symptoms. Prednisone, diphenhydramine, and famotidine would be useful in an
allergic reaction but have little role in the management of the Jarisch–Herxheimer reaction.
5. A 42-year-old female who is a self-described “seafood fanatic” presents with a chief complaint
of an “allergic reaction.” Thirty minutes after eating tuna at a local restaurant she developed a
severe headache, palpitations, nausea, abdominal cramping, and remarkable facial flushing. She
has eaten fish for her entire life without incident. Which of the following is true?
C. Upon discharge, the patient should be advised to avoid all seafood products in the future.
D. The patient should expect symptoms to resolve slowly over the course of 1 week.
Answer E. This patient experienced scombroid fish poisoning. Although not an allergic reaction,
the symptoms are due to excessive histamine levels in the fish and results in symptoms that are
very similar to an allergic response. This patient’s prior history of avid seafood intake also points
against allergy. Histamine levels build up in inadequately refrigerated, or inadequately
preserved darkmuscled fish due to the action of histidine decarboxylase by enteric bacteria in
pg. 570
the fish. Symptoms usually occur within minutes of eating the fish and include severe throbbing
headache, facial flushing and a sense of diffuse warmth, a burning sensation in the mouth and
throat, palpitations, nausea, anorexia, vomiting, abdominal cramps, conjunctival injection, and
pruritus. Symptoms are rapidly relieved after the injection of parenteral antihistamines such as
diphenhydramine or cimetidine and most symptoms will have completely abated within 6 hours.
The patient should not be told that he or she has an allergic reaction and they should not be
prevented from eating fish in the future.
A. Alcohol dehydrogenase has greater affinity for methanol than for ethanol.
B. Alcohol dehydrogenase has greater affinity for ethylene glycol than for ethanol.
C. Alcohol dehydrogenase has greater affinity for ethylene glycol than for methanol.
E. Gastrointestinal (GI) absorption of both methanol and ethylene glycol takes 4 to 5 hours.
7. Which of the following is indicated as supplemental treatment for patients with ethylene
glycol poisoning?
Answer B. During the metabolism of ethylene glycol, glyoxylic acid is produced. Glyoxylic acid
may be metabolized in three ways—two pathways form nontoxic compounds and the third
pg. 571
forms the toxic oxalic acid, which predisposes to calcium oxalate crystals. Pyridoxine and
thiamine are each cofactors in the two pathways that form nontoxic compounds and are
recommended as supplemental therapy in addition to the standard treatment of ethylene glycol
poisoning (bicarbonate, competitive alcohol dehydrogenase inhibitors such as alcohol or
4-methylpyrazole [Fomepizole], dialysis).
A. Cocaine
B. Alcohol
C. Opiates
D. Ecstasy
E. Ephedra
Answer B. Alcohol is the toxin most commonly associated with seizures. Most alcohol-related
seizures are due to alcohol withdrawal and typically occur between 6 and 48 hours after
discontinuation of drinking. However, alcohol withdrawal seizures have been known to occur as
long as 7 days after discontinuation of drinking, particularly in cases of polysubstance abuse with
benzodiazepines and barbiturates. Interestingly, acute alcohol intoxication can also provoke
seizures, and there is some electroencephalographic evidence to suggest a lowered seizure
threshold in this setting.
9. Which of the following constitutes definitive treatment for ethylene glycol poisoning?
A. Ethanol drip
B. Fomepizole
C. Pyridoxine
D. Thiamine
E. Dialysis
Answer E. Only dialysis can definitively remove ethylene glycol from the body. Ethanol and
fomepizole are temporizing measures to inhibit alcohol dehydrogenase from catalyzing the
conversion of toxic alcohols into their toxic metabolites. Pyridoxine and thiamine are cofactors
in the conversion of glyoxylic acid, a toxic metabolite of ethylene glycol, to nontoxic compounds.
They are useful as adjunctive therapies for ethylene glycol poisoning but do not constitute
definitive therapy.
pg. 572
10. Which of the following is the most common finding in propanolol overdoses in adults?
A. Ventricular tachycardia
B. Hypoglycemia
C. Hyperkalemia
D. Seizure
E. Renal failure
Answer D. Propanolol is the most dangerous β-blocker in overdose. It is highly lipophilic and
readily enters the CNS, causing seizures and coma. β-Blocker overdoses commonly cause
bradycardia, AV block, and hypotension, and less often lead to tachydysrhythmias.
Hypoglycemia is much more common in children than in adults, and glucose should be part of
the cocktail of atropine, glucagon, and insulin therapy for patients with β-blocker overdose.
Hyperkalemia and renal failure may also occur, but much less often than seizure.
A. 30 minutes
B. 1 hour
C. 5 hours
D. 10 hours
E. 20 hours
Answer B. Naloxone is a pure opioid antagonist with an extremely rapid onset of action,
duration of 1 to 2 hours, and the ability to be delivered by a variety of routes (IM, IV,
subcutaneous [SC], endotracheal [ET]). The duration is of prime importance, as patients with
opiate overdose who are given a one-time naloxone dose in the ambulance often become
acutely intoxicated again once the naloxone wears off. All opiates have longer durations of
action than naloxone, even heroin, which can last as long as 2 to 3 hours if used by the SC route.
For this reason, all patients with severe opiate overdose should be monitored carefully in the ED
at least as long as the expected peak effect of the particular opiate.
12. A 24-year-old male ingests liquid drain cleaner and immediately presents to the ED. Which of
the following is the most appropriate treatment?
pg. 573
A. Ingestion of a small cup of water
B. Calcium gluconate
C. Dexamethasone
D. Ampicillin
Answer A. Liquid drain cleaner usually contains a strong base, such as sodium or potassium
hydroxide. The treatment of caustic ingestions is generally supportive and involves diagnosis of
severe esophageal burns with endoscopy. Small amounts of water or milk may be taken
immediately after the ingestion to wash away the excess caustic material. Large amounts of
fluids should never be taken, as this may precipitate vomiting, which will dramatically worsen
esophageal injury. Calcium gluconate is indicated in patients with hydrofluoric acid exposure to
replete the calcium, which is bound by the extremely electronegative fluoride ion. Steroids have
not been proved to be beneficial in patients with most caustic injuries, although it may offer
some benefit in patients with moderate esophageal injuries. Antibiotics should not be given
unless that patient has received steroids or unless there are clear signs of perforation.
Neutralization therapy with acid or base should never be pursued, as this will lead to further
injury.
13. Which of the following is the most commonly ingested alcohol after ethanol?
A. Methanol
B. Ethylene glycol
C. Isopropanol
D. Propylene glycol
E. Acetone
Answer C. Isopropanol is the second most commonly ingested alcohol after ethanol. Choices A,
B, and D are less commonly ingested. Choice E, acetone, is not an alcohol because it lacks a
hydroxyl group.
14.Which of the following is classically contraindicated in patients with presumed acute digoxin
toxicity?
A. Transvenous pacing
B. Electrical cardioversion
pg. 574
C. Calcium gluconate infusion
D. Transthoracic pacing
Answer E. There are several “classic” contraindications in the setting of digoxin toxicity. The
most commonly cited is to avoid using calcium to treat hyperkalemia that frequently
accompanies digoxin toxicity (since digoxin inhibits the Na+/K+ ATPase). Interestingly, while
hyperkalemia is a reliable marker of digoxin toxicity, hyperkalemia is not the cause of death in
severely poisoned patients (death is caused by fatal arrhythmias induced by digoxin’s direct
effects on cardiac automaticity and excessive vagal tone) and treatment of hyperkalemia has not
been shown to decrease the risk of death. While there is almost no evidence to support the idea
that “stone heart” (tetany of the myocardium) will result from calcium administration in the
setting of hyperkalemia-associated digoxin toxicity, elevated levels of intracellular calcium are
already present, and hyperkalemia is not the chief problem (but rather reflects the degree of
toxicity). Therefore, intravenous calcium should not be used to treat hyperkalemia in digoxin
toxicity. In addition, patients with digoxin toxicity have an exceptionally excitable myocardium,
so transvenous and transthoracic pacing as well as electrical cardioversion are all classically
contraindicated. Atropine can be used as a temporizing measure in patients with severe
bradycardia, but Fab fragments should be given as soon as possible after the diagnosis of a
digoxin-associated arrhythmia is made. While there remains scant evidence, there is a
theoretical increased risk of developing more malignant arrhythmias in response to pacing and
cardioversion (e.g., ventricular fibrillation and pulseless ventricular tachycardia).
15. A 34-year-old female who takes phenelzine for depression presents with agitation, severe
hypertension, mydriasis, and hyperthermia. Which of the following foods did she most likely eat
before presentation?
A. Oranges
B. Apples
C. Graham crackers
D. Cheese
E. Ice cream
Answer D. The patient has the characteristic “wine-and-cheese” reaction due to ingestion of a
tyramine-containing food with pharmacologic MAOI activity. Tyramine is normally converted to
endogenous stimulatory amines, and monoamine oxidase (MAO) functions to break these
down. Use of monoamine oxidase inhibitors (MAOIs) inhibits this degradation function and
excess dietary tyramine in this setting causes a disorder similar to serotonin syndrome or a
pg. 575
sympathomimetic crisis. Tyramine is present in high quantities in cheese, alcohol, dried meats
and fruits, and soy.
16. After toxic acetaminophen ingestion, what is the maximum amount of time after which
Nacetylcysteine (NAC) administration still results in 100% prevention of hepatic injury?
A. 2 hours
B. 4 hours
C. 8 hours
D. 12 hours
E. 24 hours
17. A 22-year-old female presents to the ED with symptoms of extreme panic. She tells you that
she just used lysergic acid diethylamine (LSD) for the first time. Which of the following is true
regarding this patient?
D. The lethal dose of LSD is very close to the typical dose taken to induce a “trip.”
pg. 576
18. Urine containing crystals suggests ingestion of which of the following substances?
A. Ethylene glycol
B. Methanol
C. Isopropanol
D. Salicylates
E. Acetaminophen
Answer A. The metabolism of ethylene glycol is ethylene glycol → glycoaldehyde → glycolic acid
→ → → oxalic acid. Oxalic acid forms calcium oxalate crystals which can deposit in the renal
tubules and cause renal insufficiency, and the other metabolites of ethylene glycol are directly
nephrotoxic as well. Approximately one-fourth of ethylene glycol is directly excreted in the
kidneys, but hepatic metabolism with alcohol dehydrogenase catalyzes the formation of the
toxic metabolites. The goals of therapy in patients with ethylene glycol toxicity are to block the
availability of alcohol dehydrogenase with either fomepizole or ethanol and to hemodialyze the
unmetabolized ethylene glycol. Methanol toxicity results in the formation of formic acid, which
accumulates in the brain and causes blindness and death. Isopropanol causes generalized CNS
depression similar to ethanol intoxication. Salicylate overdose results in direct nephrotoxicity,
metabolic acidosis, electrolyte abnormalities, and pulmonary and cerebral edema.
Acetaminophen overdose causes fulminant hepatic failure.
19. A 56-year-old male presents with generalized fatigue, weakness, and vomiting. He tells you
that he has taken an overdose of his doxepin medication. His BP is 155/95, and his EKG
demonstrates a regular, wide-complex tachycardia. Which of the following is the most
appropriate next step in management?
A. Cardioversion at 50 J
B. Lidocaine
C. Procainamide
D. Sodium bicarbonate
E. Propafenone
Answer D. The treatment of choice for QRS prolongation and wide-complex tachycardias in
patients with tricyclic antidepressant overdose is sodium bicarbonate. Tricyclics inhibit fast
sodium channel conductance, and the sodium bicarbonate counteracts this effect. Stable
dysrhythmias do not require immediate cardioversion. Lidocaine has no proven efficacy in
pg. 577
patients with wide-complex tachycardia due to tricyclic overdose. Procainamide (IA
antidysrhythmic) and propafenone (IC antidysrhythmic) further exacerbate the inhibition of
sodium channel conductance and are contraindicated.
20. Which of the following routes of administration causes the fastest onset of action of
cocaine?
A. Intranasal
B. Sublingual
C. Oral
D. Inhalation
E. Transdermal
Answer D. Cocaine may be taken in a variety of routes, most commonly intranasal and
inhalational (crack). Inhalational and intravenous use causes the quickest onset of action,
followed by intranasal, and then oral. The duration of action is longest in oral, followed by
intranasal, then intravenous/inhalational. The transdermal route is not used for cocaine abuse.
pg. 578
22. A 22-year-old female presents after a drug overdose in a suicide attempt. She states she
took 60 of her fluoxetine tablets 2 hours before presentation. She is asymptomatic, awake, and
alert, and vital signs and physical examination are normal. Which of the following is true
regarding this patient?
Answer C. Fluoxetine is a type of selective serotonin reuptake inhibitor (SSRI). As a class, SSRIs
are generally benign in overdose, causing mild GI and CNS symptoms and rarely leading to
dysrhythmias. Cyproheptadine is a serotonin antagonist that has unproven clinical efficacy in
most SSRI overdoses. Its use is mainly limited to patients with serotonin syndrome, a
constellation of neurologic, GI, and cardiac findings. Dysrhythmia is rare and should be treated
according to current ACLS guidelines. Unlike with tricyclic antidepressant overdose, there is no
indication for routine use of sodium bicarbonate to treat dysrhythmias. As in virtually all
overdoses, ipecac is not recommended as a method of gastric decontamination. Hemodialysis is
not indicated in patients with SSRI overdose, as the drug is highly bound to plasma proteins.
23. A 75-year-old male with a history of atrial fibrillation presents with fatigue, nausea, and
halos in his vision. He states that he has been depressed lately and took some pills in an effort to
commit suicide. The serum digoxin level is elevated. Which of the following is an indication to
administer digitalis antibody (Fab) fragments?
Answer B. Digitalis toxicity causes lethal dysrhythmias, and treatment involves management of
electrolytes, digitalis antibody (Fab) fragment therapy, and dialysis. The most important
indications for Fab fragment therapy in acute digitalis toxicity are hyperkalemia, ventricular
dysrhythmias, and coingestions of other cardiotoxic drugs. Elevated digoxin level and massive
ingestion are other relative indications, but usually mandate additional rhythmic disturbance to
pg. 579
warrant Fab fragment therapy. Supraventricular dysrhythmias are only an indication for Fab
fragment therapy if the patient is hemodynamically unstable. Magnesium level aberrations may
exacerbate hypokalemia or hyperkalemia, but in the absence of potassium abnormalities, they
do not constitute an indication for Fab fragment therapy.
D. Significant bleeding through the gastrointestinal (GI) tract is present in most patients.
Answer B. Otic symptoms are the earliest symptoms observed in salicylate toxicity. The Done
nomogram is not predictive of serious pathology in salicylate toxicity and is not used clinically
(unlike the Rumack–Matthew nomogram for acetaminophen toxicity). Metabolic acidosis due to
uncoupling of oxidative phosphorylation is much more likely to be observed than metabolic
alkalosis. Despite the clear role of chronic aspirin and NSAID use in GI bleeding, acute toxicity
causes far less serious GI bleeding. Hypokalemia is far more common than hyperkalemia with
salicylate toxicity, due to a variety of renal and extrarenal mechanisms. Therapy with
bicarbonate may further exacerbate this potassium loss.
pg. 580
26. Which of the following is used in addition to acetaminophen concentration to determine
severity in overdose?
A. Lipase
B. Amylase
E. Alkaline phosphatase
27. A 22-year-old female presents in anticholinergic crisis. She is delirious, agitated, and requires
sedation. Which of the following medications would be most appropriate to sedate this patient?
A. Thorazine
B. Fluphenazine
C. Lorazepam
D. Etomidate
E. Ketamine
pg. 581
28. A 27-year-old male being treated for a performance anxiety disorder with propranolol is
brought to the ED with profound hypotension. He admits to taking “the entire bottle” of tablets
because he has been feeling depressed recently and “couldn’t take it anymore.” He started to
regret the decision later, and confided in his girlfriend who called EMS. EMS found the patient
somewhat somnolent and complaining of lightheadedness with a BP of 70/47, P 46, RR 18, and
SaO2 98% on RA. In the ED, the patient’s BP is 65/43, without any significant change in his other
vital signs. Over the next 2 hours, a central venous catheter is placed, and he is given aggressive
IV hydration, intravenous atropine, glucagon, calcium, and an epinephrine drip. Despite these
therapies, he remains hypotensive. Which of the following is the best next step?
B. Emergent hemodialysis
Answer D. The majority of beta-blocker overdoses are successfully managed with supportive
care. The front-line agents for managing the cardiovascular complications evident in this patient
include aggressive IV fluids, and atropine, with glucagon and calcium (either chloride or
gluconate) used if initial measures are ineffective. Epinephrine is the vasopressor of choice if all
these measures fail. In patients refractory to these treatments, high-dose insulin and glucose is
the therapy of choice. Insulin circumvents the beta-receptor, inducing increased levels of cyclic
AMP production through its own receptor, thus mirroring the effect of the G-protein-coupled
beta-receptor. To be effective, insulin must be used at extremely large doses. The commonly
used protocol calls for 1 unit per kg of regular insulin as a bolus, followed by a 0.5 unit per kg
insulin infusion. Dextrose infusions are required to maintain euglycemia. The effects of this
therapy are delayed 30 to 60 minutes after initiation. Lipid emulsion therapy remains
experimental, but there are positive case reports of its use in some betablocker overdoses.
Dialysis is not used in beta-blocker overdose, and propranolol is not removed by hemodialysis.
Phosphodiesterase inhibitors such as milrinone, intraaortic balloon pumps, and temporary
transvenous pacing may be helpful in patients who are refractory to all other therapies.
29. Which of the following is the most effective treatment for lithium poisoning?
A. Activated charcoal
B. Sodium bicarbonate
C. Glucagon
D. Potassium chloride
E. Hemodialysis
pg. 582
Answer E. Lithium toxicity generally causes gastrointestinal (nausea, vomiting, abdominal pain),
renal (diabetes insipidus), and neurologic (tremor, ataxia, coma) dysfunction. Management
involves whole bowel irrigation with polyethylene glycol, intravenous saline rehydration, and
dialysis in severe cases. Activated charcoal does not bind lithium. Bicarbonate is used to treat
tricyclic antidepressant overdoses. Glucagon is used to treat β-blocker overdoses. Potassium
chloride is not useful for management of lithium toxicity except in cases of severe hypokalemia.
30. A 45-year-old female with a history of hypertension presents with lower lip swelling, as
shown in Figure below. She was recently started on a new blood pressure medication. Despite
therapy with steroids, antihistamines, and epinephrine, her condition continues to worsen. In
addition to measures for airway control, which of the following is most likely to be helpful in this
patient?
A. Verapamil
B. Metoprolol
D. Danazol
E. Aldosterone
Answer C. The patient has severe lower lip swelling, likely from angiotensin-converting enzyme
(ACE) inhibitor-induced angioedema given the history of new antihypertensive medication.
Pathophysiology involves buildup of bradykinin due to inhibition of ACE. Acute treatment with
epinephrine, steroids, and antihistamines is generally ineffective. Fresh frozen plasma contains
kininase II, which can cleave bradykinin and reduce the angioedema. Verapamil, metoprolol, and
aldosterone have no role in the management of angioedema. Danazol is used for angioedema
pg. 583
due to hereditary C1 esterase inhibitor deficiency, but its role is generally limited to prophylaxis
rather than acute treatment.
31. A 28-year-old male presents to the ED stating that he drank a whole bottle of antifreeze 4
hours before presentation. He had drunk a fifth of liquor just before drinking the antifreeze.
Except for moderate intoxication, he is asymptomatic and his vital signs and physical
examination are normal. Which of the following is the most appropriate next step in
management?
B. Check the oxalic acid level and discharge him if <50 mg/dL.
Answer E. Toxic alcohol ingestions often present with delayed morbidity and mortality,
especially when ethanol is co-ingested. Ethylene glycol is the main toxic alcohol present in
antifreeze, and its half-life without co-ingestants is up to 9 hours. In the presence of ethanol, the
half-life roughly doubles. Therefore, patients who have ingested both ethanol and ethylene
glycol may be asymptomatic on presentation (other than inebriation). Diagnosis involves cardiac
monitoring, basic chemistry labs, ethanol level, blood gas, EKG, urinalysis, and creatine
phosphokinase (CPK). Fomepizole, a pharmacologic alcohol dehydrogenase inhibitor, is
administered if there is suspicion of ethylene glycol overdose, especially if the ethanol level is
negative, which indicates that alcohol dehydrogenase is free to convert ethylene glycol to its
toxic metabolites. Fomepizole does not detoxify the parent compound—it simply buys time for
the definitive removal of the toxic alcohol by dialysis. Choices A and B are inappropriate because
they fail to consider the delayed toxicity of co-ingested toxic alcohol and ethanol. Regarding
choices C and D, only approximately half the number of patients with ethylene glycol poisoning
develop urine crystals or urinary fluorescence on presentation.
32. An 18-year-old intoxicated appearing male is brought in by EMS after ingesting some
windshield wiper fluid on a dare from his drunk friend. In addition to fomepizole, which of the
following is an important component of treatment?
A. Cobalamin
B. Folate
C. Niacin
D. Vitamin D
pg. 584
E. Vitamin K
Answer B. This patient is presenting with methanol intoxication as windshield wiper fluid often
contains methanol. Methanol toxicity results in the formation of formic acid due to the effects
of alcohol dehydrogenase. Formic acid accumulates in the brain and causes blindness and death.
Ethanol and fomepizole are temporizing measures to inhibit alcohol dehydrogenase from
catalyzing the conversion of toxic alcohols, such as methanol, into their toxic metabolites. Folate
is a cofactor for the conversion of methanol’s toxic metabolite, formic acid, to carbon dioxide
and water. Once formic acid is produced, significant toxicity is probably inevitable, but the
addition of folate to the standard treatment of methanol overdose (bicarbonate, alcohol
dehydrogenase inhibitors, and dialysis) may attenuate further injury.
A. Tachycardia
B. Mydriasis
C. Diaphoresis
D. Seizures
34. Which of the following is true regarding treatment of acute lead toxicity?
pg. 585
D. Succimer should be the first chelator given in patients with severe lead poisoning.
Answer B. Chelation therapy for acute lead toxicity is indicated in patients with worsening
clinical course or severe CNS or GI symptoms. Several chelation therapies exist for lead.
Dimercaprol (or British antilewisite) should be the first chelator given in patients with severe
poisoning. It should be given before calcium disodium EDTA, as the latter, if given first, will
cause chelated lead to cross the blood–brain barrier. Acute lead encephalopathy should be
treated aggressively with chelation and management of attendant cerebral edema
(hyperventilation and mannitol). Activated charcoal does not bind lead or other heavy metals.
Patients deemed stable enough for outpatient chelation therapy should be given oral succimer.
Penicillamine is a less effective alternative to succimer and should be given only if succimer is
not tolerated due to GI side effects.
35. A 23-year-old male reaches into a snake cage at a zoo and is bitten by the snake pictured
(Fig. below). Which of the following is the most likely effect?
C. Compartment syndrome
D. Neurotoxicity
E. Renal failure
Answer D. The snake pictured is the coral snake, part of the elapid family of snakes whose
venom contains potent neurotoxins. The red-on-yellow coloration indicates that it is definitely
pg. 586
venomous— however, snakes not native to the United States without the red touching the
yellow can also be venomous. Bites can cause paralysis and diaphragmatic weakness, leading to
respiratory failure. Abdominal muscle spasms are seen with black widow spider bites. DIC and
compartment syndrome can occur with crotalid snake envenomations. Renal failure is not a
common sequela of envenomation.
36. Which of the following is true regarding treatment for acute aspirin toxicity?
37. Cold allodynia, the sensation of pain or dysesthesia when coming into contact with cool or
cold objects (often called cold reversal) is virtually pathognomonic for which of the following
causes of acute food poisoning?
A. Scombroid
B. Shigellosis
C. Clostridium perfringens
D. Ciguatera
E. Bacillus anthracis
Answer D. Ciguatera fish poisoning is the most common cause of fish-related food poisoning in
the United States. The classic syndrome includes the development of both GI and neurologic
pg. 587
manifestations. The relative amount of neurologic or GI symptoms varies by region. GI
symptoms occur first and include nausea, vomiting, watery diarrhea, and crampy abdominal
pain. Neurologic manifestations are more variable but >90% of patients have distal and perioral
paresthesias, cold allodynia, and numbness. Cold allodynia is often incorrectly referred to as
temperature reversal due to the burning sensation patients experience when coming into
contact with cool or cold objects (although it actually represents a painful, burning dysesthesia).
Symptoms last for an average of 1 to 2 weeks. There is no effective antidote; therefore,
treatment is supportive. Rare cases may be associated with bradycardia and hypotension due to
anticholinesterase activity. Such cases may require atropine and dopamine.
38. A 43-year-old alcoholic woman presents in coma. A relative states that the patient
overdosed on her alprazolam, which she has been taking for many years as an anxiolytic. After
initial airway management and IV hydration, which of the following is the most appropriate next
step in pharmacologic management?
A. Fomepizole
B. Flumazenil
C. Physostigmine
D. Glucagon
E. Thiamine
Answer E. The patient presents after benzodiazepine overdose and may or may not have
co-ingested alcohol. The history of chronic alcohol use and altered mental status dictates the
use of thiamine therapy, along with folate, multivitamin, magnesium, and dextrose. Fomepizole
is indicated only in cases of toxic alcohol poisoning. Flumazenil, a specific benzodiazepine
antagonist, is contraindicated here, as it can precipitate withdrawal seizures in patients who are
chronically using benzodiazepines. Physostigmine is an acetylcholinesterase inhibitor used in
selected patients with anticholinergic toxicity. Glucagon is used in patients with β-blocker and
calcium channel blocker toxicity.
39. A 34-year-old male overdoses on bupropion. Which of the following serious sequelae is most
likely?
A. Coma
B. Seizure
C. Torsades de pointes
D. Hypotension
pg. 588
E. Hypertension
A. Lithium
B. Ethanol
C. Iron
D. Toluene
E. Acetaminophen
Answer E. Activated charcoal prevents absorption of many drugs from all sites in the
gastrointestinal (GI) tract. Notable exceptions to the drugs absorbed by activated charcoal are
ions, heavy metals, ethanol, and hydrocarbons. Choices A to D all fall into one of these
categories, and only acetaminophen is absorbed by activated charcoal.
41. A 24-year-old male is brought to the ED for altered mental status. A friend states that they
were eating the seeds of a jimson weed plant to get high. Vital signs are 99.6°F, 100, 18, 156/94,
98% RA. The patient is agitated, responds only to name, but has intact airway reflexes. Which of
the following is the most appropriate next step in management?
A. Atropine
B. Pyridostigmine
C. Edrophonium
D. Pralidoxime
pg. 589
Answer E. Seeds from Datura stramonium, commonly referred to as jimson weed or thorn
apple, contain belladonna alkaloids, including atropine and scopolamine, which cause
anticholinergic symptoms. Anticholinergic crises are treated supportively, with
decontamination, IV hydration, benzodiazepines for agitation or seizures, hyperthermia control,
and cardiac monitoring. Physostigmine is an acetylcholinesterase inhibitor that may be used in
select anticholinergic poisonings. Physostigmine increases the amount of acetylcholine in the
synaptic cleft, allowing it to compete with the anticholinergic agent for the acetylcholine
receptor. Physostigmine is contraindicated in patients with tricyclic antidepressant overdoses, as
its use may precipitate intractable seizures and asystole. Pyridostigmine is a quaternary amine
acetylcholinesterase inhibitor which does not cross the blood– brain barrier, rendering it useless
as an agent to reverse agitation in anticholinergic toxicity. Edrophonium is a short-acting
acetylcholinesterase inhibitor which is used mainly to diagnose myasthenia gravis (MG) by
improving muscle strength in patients with MG and worsening muscle strength in cholinergic
crisis patients. Pralidoxime is used in organophosphate overdoses, where it breaks up the
organophosphate–acetylcholinesterase complex and frees acetylcholinesterase—this action
would exacerbate the problem in patients with anticholinergic crises.
A. Fentanyl
B. Meperidine
C. Propoxyphene
D. Hydrocodone
E. Morphine
Answer B. Meperidine may cause serotonin syndrome in patients who are chronically taking
selective serotonin reuptake inhibitors (SSRIs) or monoamine oxidase inhibitors (MAOIs).
Dextromethorphan also exhibits this effect.
43. The agent of choice to inactivate coelenterate nematocyst toxin (such as jellyfish) is:
D. Ammonia
E. Alcohol
pg. 590
Answer A. Many agents have been advocated, but vinegar is the agent of choice. However,
some coelenterates have species-specific treatment and most jellyfish stings require no
treatment at all. However, in cases in which tentacles and nematocysts remain adherent to skin,
vinegar should be liberally poured over the nematocysts before their removal. Tentacles and
nematocysts should then be removed by a gloved hand. Fresh water and alcohol may induce
further nematocyst discharge and should not be used. Gently rinsing the wound with seawater
is advocated as part of general wound care but it does not inactivate the toxin like vinegar.
44. A 26-year-old is brought to the ED by emergency medical services (EMS) 3 hours after
ingesting an
E. Ten vials of Digibind (Fab fragments) should be given to prevent cardiac toxicity.
Answer C. Activated charcoal may be helpful for gastric decontamination when given within 1 to
2 hours of ingestion. Most patients present after this time interval, and patients with significant
acute ingestions frequently experience nausea and vomiting which limits oral decontamination.
Enhanced elimination by using multidose activated charcoal takes advantage of digoxin’s small
enterohepatic circulation but has had mixed results. Whole bowel irrigation has not proven to
be beneficial in patients with digoxin toxicity. Digoxin has a large volume of distribution, so drug
levels drawn before 6 hours overestimate the steady-state plasma concentration. Toxic cardiac
effects are correlated to the concentration of digoxin in the heart. However, in the initial hours
after ingestion, the drug is located primarily in the plasma. This explains why more significant
evidence of toxicity is not seen early in the course of patients who have taken overdoses despite
apparently very toxic drug levels (e.g., 16 ng/mL). These patients remain at risk for toxicity, as
the drug distributes into the periphery, but drug levels are not useful until at least 6 hours after
ingestion. PVCs and bradydysrhythmias are the most common rhythm disturbances. Digibind
should be given empirically to patients with evidence of severe cardiac toxicity, such as
high-degree AV block, severe bradycardia, with or without hypotension, or for hyperkalemia
>5.0 mg/dL. It is not given as a prophylactic measure. The empiric dose of Digibind in patients
with acute toxicity is 10 vials, while 5 vials are given in cases of chronic toxicity.
A. Cyanide
pg. 591
B. Zinc
C. Toluene
D. Organophosphate
E. Hydrogen sulfide
Answer D. Patient or toxin odor may provide important clues to the toxic agent. The smell of
garlic can be caused by organophosphates, arsenic, or selenium. Cyanide smells like almonds,
zinc has a fishy odor, and toluene smells like glue. Hydrogen sulfide has the odor of rotten eggs.
46. Which of the following toxins is suggested by a fruity odor to the breath?
A. Cyanide
B. Isopropanol
C. Salicylate
D. Acetaminophen
E. Arsenic
Answer B. Patient or toxin odor may provide important clues to the toxic agent. Isopropanol is
metabolized to acetone, which causes a fruity odor to the breath. Ethanol and certain
hydrocarbons can also produce this finding. Cyanide smells like almonds, methyl salicylate like
wintergreen, and arsenic like garlic. Acetaminophen has no particular odor, but fulminant
hepatic failure from acetaminophen toxicity may cause fetor hepaticus.
47. Which of the following is an indication for Fab fragment administration in the setting of a
digoxin overdose?
Answer B. The use of digoxin-specific Fab fragments to treat digoxin toxicity has dramatically
decreased morbidity and mortality from digoxin toxicity. Fab fragments should be given to any
pg. 592
patient presenting with hemodynamic instability, malignant or symptomatic rhythm
disturbances (any ventricular arrhythmia, high-grade AV block [Mobitz type I second-degree
block or third-degree heart block, as Mobitz type II almost never occurs in the setting of digoxin
toxicity], symptomatic bradycardia), a potassium level >5 mEq/L (an elevated potassium level is
a marker of toxicity rather than a cause of toxicity), or a digoxin level >10 ng/mL in the acute
setting or >4 ng/mL in the chronic setting. The drug level can be used to determine the number
of vials needed:
However, patients who arrive in the ED with a clear indication for Fab administration should be
given Fab empirically:
48. Which of the following is the most effective therapy for acute arsenic poisoning?
A. Activated charcoal
B. Ipecac
C. Dimercaprol
D. Penicillamine
E. Deferoxamine
Answer C. Acute arsenic poisoning affects multiple organs, including the liver, kidneys, lungs,
and heart. It replaces phosphate in high-energy adenosine triphosphate (ATP) bonds and
decreases energy production. Management is supportive plus chelation therapy. The first-line
therapy for chelation is dimercaprol (or British antilewisite [BAL]). Activated charcoal does not
absorb arsenic or heavy metals. Ipecac is almost never indicated for any poisoning. Penicillamine
is a less effective alternative to dimercaprol and is only used when the latter’s GI side effects are
prohibitive. Deferoxamine is used for chelation of iron.
49. A patient with a history of depression on paroxetine presents with acute onset of altered
mental status, muscle rigidity, tremor, hyperreflexia, and hyperthermia. Which of the following
additional medications is the patient most likely using?
A. Dextromethorphan
B. Pseudoephedrine
pg. 593
C. Diphenhydramine
D. Acetaminophen
E. Valproate
Answer A. The patient has evidence of serotonin syndrome, a constellation of neurologic, GI,
and cardiac findings caused by excessive serotonin activity due to medications. The diagnostic
criteria include specific symptoms, the presence of two or more serotonergic drugs, and the
absence of neuroleptic agents or other cause for the symptoms. Hyper-reflexia, hyperthermia,
altered mental status, and diarrhea are characteristic. Medications associated with serotonin
syndrome include combinations of the following: Selective serotonin reuptake inhibitors (SSRIs),
monamine oxidase inhibitors, catecholamine releasers (cocaine, amphetamines, and
dextromethorphan), nonselective serotonin reuptake inhibitors (tricyclic and atypical
antidepressants, carbamazepine, meperidine, methadone), and serotonin agonists (buspirone,
lithium, LSD, sumatriptan). Treatment of serotonin syndrome is supportive, but cyproheptadine,
a serotonin antagonist, may be helpful in some cases. Choices B to E do not contribute to
serotonin syndrome.
Answer E. Nonaspirin NSAIDs, including ibuprofen, produce generally benign and self-limited
conditions in overdose. Symptoms will occur within 4 hours of ingestion, are usually mild, and
resolve within 24 hours. Patients rarely have life-threatening overdoses and almost never
require antidotes, decontamination, augmented renal excretion, or invasive therapies such as
hemodialysis. Serum levels of nonaspirin NSAIDs are not clinically useful. Of overdoses with
nonaspirin NSAIDs, phenylbutazone and mefenamic acid are more serious, potentially causing
multiorgan dysfunction and seizures, respectively.
51. Which of the following rhythm disturbances is considered pathognomonic for digoxin
toxicity?
pg. 594
A. Atrial flutter with premature ventricular contractions (PVCs)
Answer C. Digoxin toxicity may result in nearly any rhythm disturbance. PVCs are the most
common, and typically represent the earliest rhythm disturbance. However, the two rhythms
that are considered pathognomonic for digoxin toxicity are bidirectional tachycardia and
paroxysmal atrial tachycardia with AV block. Clinically, these rhythms are extremely malignant
and frequently precede the onset of nonperfusing rhythms. Thus, many patients are treated
before these arrhythmias develop and they are not commonly observed.
52. In a patient with benzodiazepine poisoning, which of the following is an indication for
flumazenil use?
D. Alcoholic patient
E. Seizure activity
53. Which of the following is the most common toxicity associated with cyclosporine?
B. Hyperlipidemia
C. Nephrotoxicity
pg. 595
D. Hepatotoxicity
E. Hypertension
Answer C. Cyclosporine exhibits all the listed toxicities, and may also cause tremor,
hyperkalemia, hirsutism, and gingival hyperplasia. However, the most common toxicity
associated with its use is dose-dependent nephrotoxicity, which occurs in one-third of patients.
54. Which of the following effects does digitalis exhibit at therapeutic levels?
E. T-wave inversion
Answer E. Digitalis inhibits the membrane Na–K ATPase which normally functions to pump
sodium out of the cell and potassium into it. Digitalis, therefore, increases intracellular sodium
and decreases intracellular potassium. The increased intracellular sodium causes an increase in
intracellular calcium, which produces a positive inotropic effect. In therapeutic doses, digitalis
reduces the heart rate and can cause slight ST depression and T-wave inversions.
55. A 64-year-old female is brought in by emergency services (EMS) with a chief complaint of
“anaphylaxis.” The patient has a known peanut allergy and inadvertently ingested some ground
nuts in a dish prepared by a friend. She has a known history of coronary artery disease and is
taking metoprolol. Her symptoms do not respond to epinephrine, corticosteroids, or
antihistamines. She is intubated but remains hypotensive and unstable. Which of the following
may be of benefit?
A. Calcium chloride
B. Atropine
C. Glucagon
D. Nebulized albuterol
E. Terbutaline
pg. 596
Answer C. Patients on β-blockers may be difficult to treat in the setting of anaphylaxis. On the
one hand, β-blockade may blunt or prevent some of the beneficial effects of epinephrine.
However, epinephrine also has the potential to worsen anaphylaxis in the setting of
beta-blockade due to subsequent unopposed alpha receptor stimulation. This may result in an
increased release of the vasoactive mediators in anaphylaxis. Clinically, this may be manifest in
worsened bronchoconstriction, bradycardia, and coronary vasoconstriction. Like epinephrine,
glucagon exerts its influence through the formation of intracellular cyclic AMP. However,
glucagon bypasses the β- adrenergic receptor by binding to a discrete G protein receptor.
Therefore, glucagon may be effective even in the setting of β-blockade. This is also the basis for
glucagon’s use in β-blocker overdose. Other agents that may be beneficial include vasopressors
(dopamine, norepinephrine), nebulized albuterol (specifically for relief of bronchospasm),
atropine (for bradycardia), and isoproterenol (as a last resort).
D. VX–PM monotherapy
pg. 597
57. A 33-year-old male presents with severe agitation, psychosis, and violent behavior. Physical
examination demonstrates vertical nystagmus. Which of the following is the most likely drug
ingested?
A. Cocaine
B. Phencyclidine (PCP)
D. Heroin
E. Methylenedioxymethamphetamine (MDMA)
Answer B. PCP causes extreme dissociation, agitation, psychosis, and violent behavior.
Superhuman strength often occurs in patients with PCP intoxication, sometimes requiring a
dozen people to adequately restrain them. Vertical or rotary nystagmus is a physical
examination finding characteristic of PCP intoxication. Cocaine intoxication may cause agitation,
psychosis, and mydriasis but not nystagmus. LSD is a typical hallucinogen, and MDMA or ecstasy
is similar to a combination of a hallucinogen and an amphetamine. Heroin causes a typical
opioid toxidrome, with constricted pupils, sedation, and respiratory depression.
58. A 68-year-old male with multiple medical problems including diabetes, coronary artery
disease, and hypertension is brought to the ED with a chief complaint of hypoglycemia. His wife
called emergency medical services (EMS) after she could not arouse the patient from what she
thought was a nap. She subsequently found a suicide note next to his empty bottle of glipizide.
His initial blood sugar was 24. EMS administered an ampule of D50 which brought his sugar to
115, but upon arrival to the ED, his sugar has dropped back to 76. What other adjuncts should
be given in addition to dextrose?
Answer E. Glipizide is a sulfonylurea with a fairly long half-life (7 hours) and prolonged duration
of action (12 to 24 hours). Like other sulfonylureas, the duration of action is increased further
when taken in overdose. As a result, patients frequently experience prolonged and severe
hypoglycemia after sulfonylurea overdose. While dextrose therapy is a critical part of treatment,
its use often results in transient hyperglycemia which further increases insulin secretion causing
pg. 598
rebound hypoglycemia. This is particularly true when it is used as a bolus injection. Octreotide
works by decreasing calcium influx in pancreatic beta islet cells, which results in decreased
calcium-mediated insulin release. Octreotide is continued for 24 hours, after which the patient is
observed for a prolonged period for recurrent episodes of hypoglycemia. While glucagon and
corticosteroids may increase blood glucose, their use does not affect insulin secretion, in
contrast to octreotide. Epinephrine has no role in management.
59. An 18-year-old depressed male is brought to the ED after a suicide attempt by ingestion of
some sort of powdered poison. The patient called EMS almost immediately after ingesting
“about a teaspoon” of the poison, stating he had “second thoughts” and “didn’t want to die.” En
route to the hospital, the patient appears to have a seizure but remains awake throughout the
event, and has an awkward, persistent “smile” per EMS providers. They administered 5 mg of
midazolam IV which seemed to have a small beneficial effect. Which of the following is likely
responsible for the patient’s symptoms?
A. Superwarfarin
B. Thallium
C. Arsenic
D. Strychnine
E. Ergocalciferol
Answer D. The “awake seizure” is the hallmark of strychnine poisoning. Strychnine is a glycine
antagonist and its use results in disinhibition of glycine-mediated inhibitory tone at the level of
the spinal motor neurons. This causes significant unopposed motor neuron activity resulting in
distinctive muscle spasms called opisthotonus (arched back with a rigid trunk) as well as a
unique facial appearance called risus sardonicus (sardonic smile), which is a tetanic contraction
of facial muscles resulting in a persistent grimace or smile. The muscle contractions wax and
wane without intervention and may be followed by flaccid periods. Ultimately, a rigid chest wall
results in respiratory failure, hypoxia, and death if aggressive supportive care is not initiated
(ventilation). There are no antidotes. Benzodiazepines are the class of drugs most often
recommended, while barbiturates may be necessary in refractory cases. Superwarfarins are the
most common agents used in rodenticides and the most common toxic exposure due to rodent
poison overdose. Thallium is a heavy metal that causes hair loss and a painful neuropathy.
Arsenic causes severe gastrointestinal symptoms, and patients classically present with breath
that smells like garlic. In severe poisonings, the initial symptoms lead to shock, acute respiratory
distress syndrome, cardiac irritability, and death. Some rodent poisons contain vitamin D
(ergocalciferol) which causes hypercalcemia, but such poisons are rarely significantly toxic in
humans.
pg. 599
60. An 82-year-old female taking digoxin for congestive heart failure (CHF) is brought to the ED
with confusion and an arrhythmia. Her blood tests subsequently reveal a supratherapeutic
digoxin level. At baseline, the patient is not known to have any rhythm disturbance or block.
Which of the following rhythm disturbances is most likely present?
D. Sinus tachycardia
Answer A. Digoxin toxicity may result in nearly any rhythm disturbance. However, PVCs are the
most common and typically represent the earliest rhythm disturbance. The key point is to recall
that digoxin toxicity exerts its effects in two ways: Increased vagal tone leads to
bradyarrhythmias and AV block, while increased automaticity leads to frequent ectopic rhythms.
The hallmark of digoxin toxicity is when these two conditions are combined, as in the often cited
paroxysmal atrial tachycardia (PAT) (increased automaticity) with AV block (increased vagal
tone). However, sinus bradycardia (increased vagal tone) combined with PVCs (increased
automaticity) is much more common. Rapidly conducted atrial dysrhythmias (e.g., atrial
fibrillation with rapid ventricular response, or SVT) are not observed because of the high degree
of AV block associated with digoxin toxicity. However, since digoxin continues to be used as a
second-line agent in patients with atrial fibrillation, many patients with digoxin toxicity present
with atrial fibrillation (which is their baseline rhythm, as opposed to a result of drug toxicity).
Other common findings include bradydysrhythmias, junctional tachycardia, and bidirectional
ventricular tachycardia which along with PAT with block, are frequently considered
pathognomonic. While digoxin toxicity results in AV block, Mobitz type II second-degree AV
block is among the least common rhythm disturbances caused by digoxin toxicity.
61. Aside from the eyes, methanol overdose most commonly has pathologic effects on which of
the following sites?
A. Basal ganglia
B. Facial nerve
C. Vagus nerve
D. Olfactory nerve
E. Glossopharyngeal nerve
pg. 600
Answer A. Methanol is metabolized to formaldehyde by alcohol dehydrogenase, and
formaldehyde is converted to formic acid by aldehyde dehydrogenase. Formic acid accumulates
preferentially in the ocular tissues and the brain, most commonly the basal ganglia. Long-term
morbidity of methanol overdose includes blindness and a parkinsonian syndrome, with
bradykinesia and rigidity. Treatment of methanol and ethylene glycol overdoses involves
correction of the metabolic acidosis with sodium bicarbonate, inhibition of alcohol
dehydrogenase with fomepizole or ethanol, and dialysis of the toxic alcohol.
62. A 15-year-old female is brought to the ED by her parents because they suspect her of drug
use. Friends dropped their daughter off after going to a party where she became confused and
started grinding her teeth. They noted her heart was beating very fast, and she had trouble
focusing her eyes. Her friends report that she took something to “feel closer” to her friends.
Which of the following is the most likely drug used?
A. LSD
B. PCP
C. 3, 4-MDMA
D. Sertraline
E. Phenelzine
Answer C. Choice C is also known as MDMA or ecstasy. It is in a newly assigned class of drugs
known as entactogens. These drugs have properties of both hallucinogens and amphetamines,
causing mild hallucinations, increased interpersonal emotions, and stimulatory neurotransmitter
release. Ecstasy is commonly ingested at raves and other dance parties. Pathophysiology
includes sympathomimetic effects, hyperthermia, and hyponatremia (both from a central
antidiuretic hormone like effect and increased thirst, causing water consumption). Choice A is
LSD, which is not as commonly used currently as it was in the 1960s to 1980s. Choice B is PCP,
causing an unpleasant psychosis and violence. Choice D is a selective serotonin reuptake
inhibitor, and choice E is a monoamine oxidase inhibitor, neither of which is used recreationally.
63. A 24-year-old healthy female is brought to the ED by EMS after having ingested “a large
number” of her mother’s digoxin tablets. Her EKG reveals asymptomatic sinus bradycardia with
first-degree AV block and occasional PVCs. Her vital signs reveal a pulse of 48 but are otherwise
normal. Subsequent laboratory tests demonstrate a potassium level of 6.8 mEq/L. How should
the elevated potassium level be treated?
pg. 601
D. Continuous albuterol nebulization
Answer A. While hyperkalemia is very common in the setting of digoxin overdose, it is not a
cause of digoxin toxicity but rather a manifestation of toxicity. Furthermore, the specific
treatment of hyperkalemia in this setting has not been shown to reduce mortality. However,
since hyperkalemia is a sign of significant toxicity in the setting of digoxin overdose, its presence
is a well-defined indication for digoxin Fab administration (Digibind). Digoxin-specific Fab
administration should be given to all patients with a potassium level >5.0 mEq/L. Hyperkalemia
will resolve with Fab administration itself. Additional therapy with “conventional” therapies for
hyperkalemia is not necessary, and may precipitate hypokalemia once Fab is given. In addition,
calcium administration is a “classic” contraindication in the setting of digoxin toxicity because of
the risk of “stone heart” or sudden cardiac arrest.
64. A 65-year-old female presents to the emergency room with signs and symptoms of digitalis
toxicity, ventricular tachycardia, and a digoxin level of 8.5 ng/mL. She is treated with digitalis
antibody fragment therapy and the cardiac rhythm reverts to normal sinus rhythm. However, a
repeat digoxin level after the fragments are given is 12 ng/mL. Which of the following is the
most appropriate next step in management?
A. No acute therapy
B. Cardioversion at 50 J
C. Procainamide 1 g IV
D. Calcium chloride 1 g IV
Answer A. The standard serum digoxin assay measures levels of all digoxin in the body, including
drug bound to Fab fragments. It is not useful to measure digoxin levels once Fab has been given.
Precedence should be given to dialysis to remove the drug–Fab complexes. Cardioversion may
be performed in unstable patients but is unlikely to be curative in patients with digitalis toxicity.
Procainamide should be avoided in patients with digitalis toxicity as it may exacerbate
dysrhythmias. Calcium chloride should be avoided in patients with digitalis toxicity to prevent
theoretical risk of “stone heart,” which occurs from massive calcium influx into cardiac myocytes
causing sustained contraction. Potassium chloride should be given with extreme caution in
patients with digitalis toxicity, as hyperkalemia is life threatening in this setting.
65. A 56-year-old male presents with tremor, anxiety, tachycardia, and hypertension. He has a
past medical history of chronic renal insufficiency. He reports heavy alcohol use but has not had
pg. 602
a drink in the last 2 days. Which of the following is the most appropriate next step in
management?
A. Haloperidol
B. Lorazepam
C. Chlordiazepoxide
D. Diphenhydramine
E. Droperidol
Answer B. Alcohol withdrawal occurs as early as 6 hours after cessation of alcohol consumption
and generally peaks after 2 to 3 days of abstinence. Signs and symptoms are similar to a
sympathomimetic toxidrome—hyperthermia, tachycardia, hypertension, tremor, anxiety,
hallucinations, and seizures. Particularly severe cases are often referred to as delirium tremens,
which may be life threatening. Management of alcohol withdrawal involves aggressive
supportive care with fluids, vitamin and electrolyte supplementation, and benzodiazepine
therapy. Diazepam and chlordiazepoxide are preferable because their long half-life is thought to
yield a smoother withdrawal with a lower risk of recurrent withdrawal after benzodiazepines are
discontinued as well as a lower risk of seizures. However, chlordiazepoxide’s long half-life is
further prolonged by renal failure so this patient is better served by lorazepam which has a
shorter half-life and fewer metabolites. Haloperidol and droperidol are two drugs of the
butyrophenone class which are used as adjunctive therapy to manage agitation, but they have
no effect in preventing seizures. Chlordiazepoxide is a benzodiazepine that may be used to treat
alcohol withdrawal but has a very prolonged half-life in patients with renal insufficiency.
Diphenhydramine, an antihistamine and anticholinergic agent, has no role in the management
of alcohol withdrawal.
66. A 42-year-old male with hypertension presents with significant swelling of his lips and
tongue. He has been taking lisinopril for the past 8 months. Which of the following is true?
A. This adverse drug event is most common in the first week after therapy.
Answer E. Patients receiving angiotensin-converting enzyme (ACE) inhibitors are at risk for
developing angioedema, which is a nonpitting, symmetric edema that typically involves the face,
pg. 603
tongue, and supraglottic tissues. Most patients present with lip and tongue swelling and do not
have either urticaria or pruritus. The response is most common within the first week of therapy
but can occur months or years after starting the drug. It is not safe for patients developing
angioedema due to ACEI therapy to take ARBs. ARBs are used when patients can’t tolerate the
side effects, such as cough, due to ACEI therapy. Finally, patients with angioedema due to ACEI
therapy may be resistant to all first-line agents for anaphylaxis including epinephrine,
corticosteroids, and antihistamines. Symptoms typically resolve within 24 to 48 hours of
discontinuation of the drug, but elective intubation should be performed early in the course if
there is any sign of respiratory compromise. Typical “rescue” airways may fail in angioedema
due to edema of the glottic structures. Therefore, ENT and anesthesia should be involved in the
case, and intubations without the use of paralytics should be considered.
67. A 65-year-old female with a history of chronic alcoholism presents with confusion, ataxia,
and nystagmus. Which of the following is the most appropriate therapy?
A. Lorazepam
B. Haloperidol
C. Thiamine
D. Pyridoxine
E. Potassium
Answer C. Altered mental status, oculomotor dysfunction, and ataxia comprise the clinical trial
of Wernicke encephalopathy. Alcoholics develop this emergent condition partly due to thiamine
deficiency and supplementation remains the mainstay of management. Wernicke
encephalopathy may deteriorate into Wernicke–Korsakoff syndrome, which adds the elements
of memory disturbance and confabulation. Magnesium therapy, glucose, and intravenous fluids
are important adjunctive therapies for these disorders. Lorazepam is used to treat alcohol
withdrawal seizures but has no role in the management of Wernicke encephalopathy.
Haloperidol may be used to treat agitation and psychosis in alcohol withdrawal. Pyridoxine
therapy may be used as part of a multivitamin that contains thiamine, but it is not essential to
treat Wernicke encephalopathy. Potassium repletion may be indicated if hypokalemia is present
or expected during the course of therapy.
68. Which of the following effects is directly responsible for the QRS prolongation seen in
tricyclic antidepressant poisoning?
C. α-1 antagonism
pg. 604
D. Anticholinergic activity
Answer A. Tricyclics block fast sodium channels, slowing phase zero myocardial depolarization
and causing QRS prolongation. Negative inotropy occurs due to reduced numbers of opened
calcium channels. Potassium efflux blockade causes QT prolongation from impaired
repolarization, α-1 antagonism causes hypotension, and anticholinergic effects cause
tachycardia, hyperthermia, urinary retention, and agitation. Tricyclics have no known effect on
magnesium channels.
69. A 2-year-old female ingested the leaves of this flowering plant (Fig. below). Which of the
following effects can be expected?
A. Dry mouth
B. Renal failure
C. Cardiac dysrhythmias
E. Liver failure
pg. 605
Answer C. The plant in the picture is the foxglove plant, also known as Digitalis purpurea, from
which digoxin is derived. Digoxin blocks Na–K ATPase, increasing intracellular sodium, and
leading to an increase in intracellular calcium, causing increased cardiac contractility. It is also
used as an antidysrhythmic agent, but can cause cardiac dysrhythmias in overdose. Dry mouth is
an anticholinergic effect not typically associated with digitalis. Renal and liver failure is seen with
other toxins. Altered mental status can occur with digitalis toxicity, but this is less common than
cardiac effects.
70. A 34-year-old female presents with weakness, fatigue, rash, and fever for several days. She
was prescribed an antibiotic by her primary care physician for an upper respiratory infection just
before the symptoms started. The rash is diffuse, maculopapular, and confluent. Laboratory
work demonstrates normal electrolytes, but an elevated creatinine at 3.6 mg/dL. Peripheral
blood and urine contain eosinophils. Which of the following is the most likely etiologic agent?
A. Amoxicillin
B. Doxycycline
C. Clindamycin
D. Azithromycin
E. Erythromycin
Answer A. The onset of renal insufficiency, serum sickness-like signs and symptoms, and
eosinophilia points to allergic interstitial nephritis as the most likely cause. A hypersensitivity
reaction to various drugs is the usual cause—penicillins, NSAIDs, sulfa drugs, phenytoin, and
diuretics are most commonly implicated. Macrolides and tetracyclines are less often involved.
Interstitial nephritis may also be due to infectious and immunologic causes. Urinalysis generally
demonstrates sterile pyuria with possible hematuria and mild proteinuria. Eosinophilia occurs in
approximately half the number of patients. Definitive diagnosis is by kidney biopsy. Renal
insufficiency due to allergic interstitial nephritis is usually reversible, and treatment always
involves removal of the offending drug.
A. Coprine
B. Ibotenic acid
C. Amatoxin
D. Orellanine
E. Psilocybin
pg. 606
Answer C. Mushroom poisonings are divided into the seven major toxins responsible for the
pathologic effects:
A. α-1 Agonist
C. Muscarinic agonist
E. Antihistamine
Answer E. Tricyclic antidepressants cause blockade of the following: α-1, muscarinic, histamine,
GABA, cardiac potassium efflux, fast sodium channel, serotonin reuptake, and norepinephrine
reuptake. Because of these effects and the potential for lethality in overdose, they are no longer
indicated as first-line therapy for the management of major depression.
73. A 56-year-old female presents with acute organophosphate overdose, with severe
bronchorrhea, bradycardia, and coma. She is intubated for airway protection, and atropine
therapy is initiated. After 10 mg of atropine, her heart rate (HR) is 130, blood pressure (BP) is
160/90, and her secretions are still copious. Which of the following is the most appropriate next
step in management?
pg. 607
E. Continue atropine therapy and add pralidoxime
74. A 25-year-old male ingests the mushroom, which he thought was edible (Fig. below). Which
of the following effects can be expected?
A. Hallucinations
B. Liver failure
C. Renal failure
D. Blindness
E. Adrenal infarction
Answer A. The mushroom in the picture is Amanita muscaria, which contains muscimol, a
hallucinogenic. It is intentionally ingested in parts of the world for the hallucinogenic effect.
Despite its name, there is very little muscarine present in Amanita muscaria mushrooms and no
toxic effect from this substance. Liver failure is seen with Amanita phalloides mushrooms, also
pg. 608
known as the death cap. Renal failure and blindness are seen with ethylene glycol and methanol
poisoning, respectively. Adrenal infarction is not a typical sequela of any common toxicologic
agent.
75. Which of the following is the most common serious complication of the edrophonium
(Tensilon) test?
A. Bradycardia
B. Atrial fibrillation
C. Oculogyric crisis
D. Cough
E. Seizure
76. A 25-year-old male ingests an unknown quantity of iron tablets 2 hours before presentation
in an overdose attempt. Which of the following is the most appropriate next step in
management?
A. Activated charcoal
B. Gastric lavage
C. Ipecac
D. Polyethylene glycol
E. Hemodialysis
pg. 609
Answer D. Iron toxicity can be life threatening. Ingestion of large quantities of iron overwhelms
the body’s iron-binding capacity and causes GI, cardiac, CNS, hepatic, and renal damage.
Nausea, vomiting, diarrhea, and GI bleeding are the most common symptoms. Diagnosis
involves serial serum iron levels and plain abdominal radiographs to demonstrate passage of the
radiopaque iron pills. Treatment involves whole bowel irrigation with polyethylene glycol,
deferoxamine chelation in patients with severe overdoses (defined as rising iron levels, absolute
level >500 μg/dL, or worsening clinical course), and dialysis of the chelated iron when renal
failure is present. Activated charcoal does not adequately bind heavy metals. Gastric lavage is
rarely indicated for any overdose, except in certain cases when patients present within 30
minutes of overdosing. Ipecac is almost never indicated for any overdose. Hemodialysis is only
necessary when renal failure limits the body’s ability to clear chelated iron.
77. A 15-year-old male comes to the ED complaining of shortness of breath and nausea. He
admits to “huffing” glue before presentation. Which of the following is the most appropriate
therapy at this time?
B. Antibiotics
C. Corticosteroids
D. Diuretics
E. Activated charcoal
A. Hypocalcemia
B. Hypokalemia
C. Microcytic anemia
D. Thrombocytopenia
E. Hypermagnesemia
pg. 610
Answer A. Ethylene glycol is metabolized to glycoaldehyde by alcohol dehydrogenase, and
glycoaldehyde is converted to glycolic acid by aldehyde dehydrogenase. Glycolic acid is then
converted to glyoxylic acid, which is converted to oxalic acid. Oxalic acid binds calcium and
forms calcium oxalate crystals, which can precipitate in the renal tubules, brain, and lungs,
causing necrosis. Approximately one-third of patients with ethylene glycol poisoning have
hypocalcemia, which can lead to QT prolongation and seizures. Hyperkalemia is more common
than hypokalemia, probably due to the metabolic acidosis caused by ethylene glycol poisoning.
Anemia, thrombocytopenia, and hypermagnesemia do not usually occur in patients with
ethylene glycol poisoning.
79. A 30-year-old, 80-kg male patient presents to the ED with a severe allergic reaction not
responding to intravenous diphenhydramine, steroids, and famotidine. You decide to treat him
with epinephrine. The appropriate dose is:
B. 0.5 mL 1:10,000 SQ
D. 0.5 mL 1:1,000 IV
E. 0.3 mL 1:10,000 IM
Answer C. Epinephrine dosing remains a confusing topic, largely because guidelines vary in their
nomenclature. To make things simple, it is easy to remember that 1:1,000 doses are always
given SQ or IM, whereas 1:10,000 doses (which are less concentrated) are given intravenously.
The concentration of a 1:1,000 solution of epinephrine is 1 mg/mL, whereas the concentration
of a 1:10,000 solution is 0.1 mg/mL (i.e., 10 times less concentrated). Therefore, 0.3 mL of a
1:1,000 solution is 0.3 mg. Delivery of 0.2 to 0.5 mg of epinephrine IM is the preferred dose and
method of delivery in the setting of anaphylaxis. Subcutaneous delivery, which was favored in
the past, results in slower and more erratic absorption. IM delivery is quick, effective, and
extremely easy (and is the basis for the commercially available EpiPen, so IM delivery can be
demonstrated to patients who may later need to self-administer the medication). It is critical to
administer epinephrine early in the course of anaphylaxis. Epinephrine functions better as a
front-line treatment than as “rescue” therapy late in the course. This dose may be repeated
every 5 to 15 minutes.
80. Comatose patients with bilateral miosis thought to be due to narcotic overdose may
sometimes be confused with patients with which of the following?
B. Locked-in syndrome
pg. 611
C. Pontine hemorrhage
D. Pseudobulbar palsy
E. Retinal detachment
Answer C. Although some patients with pontine hemorrhage may have only very small areas of
involvement and only moderate symptomatology, pontine hemorrhagic stroke typically involves
the entire anterior aspect of the pons, resulting in coma, miosis (pinpoint pupils), horizontal
gaze palsy, and erratic breathing requiring intubation and mechanical ventilation. Miosis results
from disruption of ascending sympathetic fibers that innervate the pupillary dilator apparatus.
Horizontal gaze palsy results from involvement of the abducens nucleus (CN VI), which is located
in the central pons. Lateral medullary syndrome is due to occlusion of the posterior inferior
cerebellar artery (PICA), whereas locked-in syndrome is due to occlusion of the basilar artery.
Neither syndrome involves miosis and patients with locked-in syndrome retain normal eye
movement. Pseudobulbar palsy is due to the bilateral lesions of the corticobulbar tracts to lower
cranial nerves and may result in dysphagia, dysarthria, and dysphonia. It spares the cranial
nerves controlling the extraocular muscles (CN III, IV, and VI).
81. The best route and location of epinephrine administration for anaphylaxis is
A. IM in the deltoid.
Answer C. Recent studies have demonstrated that serum epinephrine levels are higher and rise
faster in patients given epinephrine through IM injection in the lateral thigh instead of through
SQ injection. SQ administration results in variable absorption and may be delayed by the
vasoconstrictive effect of epinephrine. Patients with epinephrine autoinjectors are taught to
inject epinephrine into the lateral thigh musculature (vastus lateralis). Although no trials
assessing outcome and comparing SQ and IM administration have been performed, the current
recommendation is that epinephrine be delivered through the IM route in the lateral thigh.
82. A 16-year-old previously healthy male presents for evaluation of difficulty breathing, throat
swelling, nausea, and vomiting after eating cake at a birthday party. He is successfully treated
with epinephrine and his vital signs are stable. Which of the following is true regarding biphasic
anaphylactic reactions?
pg. 612
A. They are more common when initial treatment is suboptimal
83. A patient in your ED refuses to take penicillin because she is “allergic.” The historical feature
most suggestive of a true allergy in this patient is:
A. Paresthesias
B. Vomiting
C. Fatigue
D. Urticaria
E. Palpitations
Answer D. Adverse drug reactions (ADRs) are serious events that may have fatal consequences.
The incidence of ADRs among hospitalized patients is estimated at 15.1%, with half of these
events characterized as “serious,” whereas the incidence of fatal ADRs is estimated at 0.32%,
resulting in >100,000 deaths per year. However, true allergic drug reactions (i.e., through type I
IgE-mediated hypersensitivity) represent only 6% to 10% of ADRs (some reports quote as high as
25%). Although 10% of patients claim to be penicillin-allergic, 90% of those patients are
subsequently found through skin testing not to have an allergy. Among those mentioned earlier,
urticaria is the most likely manifestation of a true allergic response. The remaining symptoms
are most likely expected side effects of drug therapy. However, it is very difficult to rely on the
pg. 613
patient’s history as nearly a third of patients with proven penicillin allergy by skin testing have
vague histories.
Answer B. Foods are the single most common cause of anaphylaxis although up to one-third of
causes are unknown. Exercise accounts for 7% of anaphylaxis cases. Anaphylaxis occurs as an
immediate type I hypersensitivity reaction mediated by immunoglobulin E (IgE) antibodies. It
requires prior sensitization to the allergen in order to develop allergen-specific IgE antibodies.
Subsequent exposure to the allergen allows mast cell and basophil degranulation leading to the
subsequent anaphylactic response. Anaphylactoid reactions are immediate type I
hypersensitivity reactions that do not require prior allergen sensitization and which clinically
mimic anaphylaxis. Anaphylactoid reactions (e.g., response to iodinated radiographic contrast
material) typically require a larger dose of the offending agent, but the clinical management is
identical to anaphylaxis. Patients at the extremes of age are less likely to have anaphylaxis,
probably because of less mature immune responses.
pg. 614
Environmental disease
Emergency
➢ Chapter includes:
Animals bites
Electrical, heat stroke and heat exhaustion
High attitude sickness
Drowning and submersion
Inhalation toxic gases
Burns and frozen bites
hypothermia
multiple environmental MCQ’
pg. 615
1. A 45-year-old male presents in a coma after being exposed to smoke from a building fire. The
patient is immediately intubated. An arterial blood gas (ABG) demonstrates metabolic acidosis
and an extremely elevated lactate level. Which of the following is the most important
medication to administer?
A. Amyl nitrite
B. Sodium nitrite
C. Sodium thiosulfate
D. Methylene blue
E. Dexamethasone
Answer C. Patients with smoke inhalation from building fires are exposed to two major toxins,
carbon monoxide and hydrogen cyanide. A metabolic acidosis with elevated lactate level in
these patients is highly suggestive of dual toxicity. Carbon monoxide binds to hemoglobin with
more than 200 times greater affinity than oxygen, and prevents hemoglobin from carrying
oxygen. Cyanide poisons complex IV of the mitochondrial electron transport chain. Treatment of
carbon monoxide alone is 100% oxygen and hyperbaric oxygen is severe cases. Treatment of
hydrogen cyanide alone involves two major steps: Induction of methemoglobinemia with
nitrites, which pulls the cyanide molecule off the electron transport chain, and detoxification of
cyanide with thiosulfate. When there is poisoning with both cyanide and carbon monoxide,
induction of methemoglobinemia by nitrites should be avoided, as it would further prevent
hemoglobin from carrying oxygen. The treatment of choice in this case would be sodium
thiosulfate. Methylene blue is the treatment for severe methemoglobinemia. Dexamethasone
has no role in the management of carbon monoxide or cyanide poisonings.
A. Hemoconcentration
B. Hypoglycemia
C. Metabolic alkalosis
D. Oliguria
E. Seizures
Answer A. Due to decreased plasma volume, the hematocrit increases approximately 2% for
every 1°C decrease in the core temperature. This is thought to be due to increased vascular
permeability and third spacing of fluids as well as cold-induced diuresis and free water loss. In
acute hypothermia in otherwise healthy individuals, hyperglycemia occurs because of the
pg. 616
circulating catecholamines and cold-related inhibition of insulin secretion. There is a tendency
toward metabolic acidosis in hypothermic patients although limited experimental data suggest
that patients may present either acidotic or alkalotic. Patients experience respiratory depression
with decreasing temperature, in part due to the decrease in metabolism that occurs as the body
cools. This raises the Pco2 and decreases the pH. Other factors that contribute to the acidotic
state include lactate production from shivering and decreased tissue perfusion and impaired
hepatic function. These effects are blunted somewhat because of the fact that as blood cools, it
becomes more alkalotic. A progressive diuresis, not oliguria, occurs as the temperature cools.
The most common neurologic finding in hypothermic patients is a decreased level of
consciousness. EEGs in hypothermic patients demonstrate generalized slowing and decreased
amplitude. In addition, pupillary responses and deep tendon reflexes are decreased and patients
tend to have increased muscle tone.
Answer E. The traditional scheme for categorizing burn injuries has been replaced by a surgical
model which describes burns as superficial (first degree), superficial partial-thickness (superficial
second degree), deep partial-thickness (often requiring surgery), and full-thickness (third
degree). The term “fourth degree” is still used to describe burns that injure subcutaneous
tissues, including muscle, tendons, and bone. One significant problem with initial burn wound
evaluation is that the character of the wound may change over time in a process known as
“wound conversion.” Superficial wounds may become deeper over time depending on the
adequacy of blood flow and healing, as well as the patient’s underlying medical problems. In
addition, the depth of injury is often variable throughout the wound such that both deep and
superficial components may be present. Deep partial-thickness wounds invariably heal with
significant, hypertrophic scarring that impedes joint function. Such burns are not painful unless
palpated, and they do not blanch when pressure is applied. Clinically, they are often difficult to
differentiate from full-thickness burns. Furthermore, deep partial-thickness burns leave very few
viable epithelial cells remaining, so wound healing (via re-epithelialization) is extremely slow and
often requires skin grafting.
4. A 19-year-old female is brought to the ED after an accidental submersion. She was drinking
with friends at a dock when she accidentally fell into the water. She was submerged in the water
pg. 617
for approximately 1 minute before her friends pulled her out. She was gagging and coughing
when they pulled her from the water but she did not require cardiopulmonary resuscitation. Her
blood alcohol level is 120 mg/dL and her pulse oximetry reveals an oxygen saturation of 94% on
room air. An initial chest x-ray is normal. Assuming the remainder of her examination is normal
and she has no signs of trauma, what is the best course of management?
B. Observation for 4 hours, then discharge if she is asymptomatic with SaO2 >94% on room air.
C. Repeat chest x-ray in 8 hours, then discharge if she is asymptomatic with SaO2 >94% on room
air.
D. Observe for 6 hours, administer oral steroids, prescribe a short pulse of steroids to limit
aspiration pneumonitis and discharge home if SaO2 >94% on room air.
E. Administer oral antibiotics and oral steroids, prescribe a course of both medicines, and
discharge home without observation.
Answer B. Neither antibiotics nor corticosteroids have any role in prophylactic therapy of
potential aspiration. Corticosteroids may be of use in patients with a history of reactive airway
disease, who have active symptoms upon presentation to the ED. Otherwise, asymptomatic
patients should be observed for a minimum of 4 hours in the ED. Repeat x-rays are not required.
If the patient remains asymptomatic and maintains an oxygen saturation >94% without
supplemental oxygen, she may be safely discharged.
5. What is the clinical factor that best differentiates heat stroke from heat exhaustion?
Answer D. Neurologic dysfunction is the hallmark of heatstroke. Patients with heat exhaustion
present with intact mental status, although they may present with generalized malaise, fatigue,
headache, impaired judgment, vertigo, as well as nausea and vomiting. Patients with heat
exhaustion also typically present with persistent and profuse sweating, with a core temperature
that may be only mildly elevated and which is always <104°F. Patients with heat stroke usually
have a core temperature >105°F, resulting in multiorgan failure. Patients may be anhidrotic and
have elevated hepatic transaminases, although these findings are not required for the diagnosis.
pg. 618
In addition, patients with heat exhaustion may also develop elevated hepatic transaminases
although the increase is less severe. Most significantly, patients with heat stroke have an altered
sensorium (delirium) and may develop coma or seizures.
6. Bites from which of the following snakes would most likely result in respiratory failure and
death?
A. Rattlesnake
C. Coral snake
D. Copperhead snake
Answer C. Coral snakes are part of the Elapidae family, whereas the remaining snakes listed are
part of the Crotalidae family. Elapidae venom is neurotoxic, as several of the venom
components block acetylcholine transmission. In contrast to victims of pit viper envenomation,
victims of coral snake envenomation usually experience minimal pain and swelling at the bite
site. However, signs of neurotoxicity may develop rapidly or be delayed for up to 12 hours.
Ptosis is frequently the initial sign of neurotoxicity and may be followed by delirium, tremors,
drowsiness, hypersalivation, and multiple cranial nerve abnormalities (dysarthria, diplopia, and
dysphagia). In severe envenomations, respiratory muscle paralysis occurs, leading to respiratory
failure and death.
Answer A. As with other spider or insect bites, a diagnosis of a brown recluse spider bite cannot
be reliably made without the spider. Furthermore, there remains no evidence for any of the
variety of therapies for brown recluse spider bites, including early surgical excision, dapsone,
electric shock, steroids, hyperbaric oxygen, colchicine, antihistamines, anticoagulants, or
prophylactic antibiotics. Despite this, many of these therapies are still used. Dapsone, in
pg. 619
particular, has been advocated as a means of limiting the toxic effects of the venom. It is not
used as an antibiotic. Most bites result in burning pain, mild erythema, pruritus, and minimal
swelling. Occasionally, hemorrhagic vesicles develop along with central necrosis in the days
following the bite. Even in the case of these more severe bites, supportive therapy is all that is
required.
8. A 22-year-old male presents to the ED after being struck by lightning. He feels fine now but
has a rash on his back in a feather or fern pattern. Which of the following is the best step in
management of this complication?
E. No specific therapy
Answer E. The patient has a feathering rash caused by lightning. It is not an actual burn injury,
but rather a local irritation to electron showering that can occur during a lightning strike. The
rash is temporary and requires no specific management. Antibiotic, steroid, and antifungal
therapy is not indicated. Preventive therapy with a triple antibiotic ointment is not useful (as
there is no actual burn present) and may put patients at risk for neomycin-induced contact
dermatitis.
A. Nerve
B. Blood
C. Muscle
D. Fat
E. Mucous membrane
Answer D. Electrical resistance refers to the ability of the substance to resistant passage
(conductance) of electricity. High-resistance tissues convert electrical energy to heat energy,
damaging the tissue itself but putting other structures at less risk for damage. Low-resistance
tissues allow electricity to pass, which can cause potentially serious organ damage at sites away
from the initial entry point. Fat, tendon, and bone have the highest electrical resistance. The
pg. 620
other answer choices represent the least resistant tissues. Dry, undamaged skin is of
intermediate resistance; calloused skin has high resistance, but wet skin has very low resistance.
10. While working in the ED, a paramedic calls from emergency medical services (EMS) to
provide a mass casualty alert from a local stadium. Many fans watching a basketball game in a
small arena became suddenly ill, and several slumped to the ground and either lost
consciousness or seemed to be “convulsing.” The building was rapidly evacuated. Ambulatory
evacuees complained of severe headache, diaphoresis, excessive “spit” and runny nose, as well
as shortness of breath with a frothy cough, nausea, vomiting, and muscle aches. Though several
survivors noted a “mist” coming from some of the vents inside the building, there was no
significant odor noted. In addition to high-flow oxygen, which of the following is the best course
of treatment among survivors?
B. Amyl nitrite
Answer A. The patients described in this vignette are suffering from a cholinergic syndrome. The
nerve agents sarin, VX, tabun, and soman are all organophosphorus compounds that strongly
inhibit acetylcholinesterase resulting in symptoms of acetylcholine excess. The classic mnemonic
used to recall the symptoms of a cholinergic toxidrome is SLUDGE: salivation, lacrimation,
urination, defecation, gastrointestinal upset, and emesis. The “SLUDGE” mnemonic does not
address the pulmonary muscarinic effects (bronchorrhea, bronchospasm), cardiac muscarinic
effects (bradycardia), nor the nicotinic effects in the central nervous system (muscle weakness,
fasciculations, and flaccid paralysis). For this reason, the “SLUDGE” mnemonic is sometimes
appended as “SLUDGE/BBB” for bronchospasm, bronchorrhea, and bradycardia. There are
several alternative mnemonics as well (e.g., DUMBELS). Since these drugs work through
excessive acetylcholine action, atropine is the natural antidote and should be given immediately
to any patient with evidence of moderate or more severe toxicity. The dose is titrated until the
pulmonary symptoms are resolved (no shortness of breath, no wheezing, no excessive
secretions). If initial doses of atropine are ineffective, the dose should be doubled every 3 to 5
minutes and repeated. 2-PAM is used to address the nicotinic effects of these agents, since
atropine does not bind to nicotinic receptors. Like atropine, 2-PAM should be used liberally and
should be given to any patient with evidence of toxicity. Even in the absence of significant acute
toxicity, many nerve agents cause delayed neurologic effects which may be prevented by 2-PAM
administration. In addition, 2-PAM should be given early because the nerve agents become
irreversibly bound to acetylcholinesterase over time (called “aging”). Both atropine and 2-PAM
can be given IV or IM. Since succinylcholine is metabolized by acetylcholinesterase, it should
pg. 621
never be used for airway management in patients with organophosphate toxicity, as its use
results in prolonged and excessive paralysis. Diazepam is used to treat the seizures that may
occur. Sodium thiosulfate is used for cyanide intoxication while amyl and sodium nitrite are used
for either cyanide or hydrogen sulfide exposure.
11. Which of the following tissues has the greatest resistance to electrical flow?
A. Nerves
B. Blood
C. Fat
D. Skin
E. Muscle
Answer C. Fat, tendons, and bones have the greatest resistance to electrical flow (bones have
the highest resistance of all), whereas nerves, blood, mucous membranes, and muscle have the
least resistance. The resistance of dry skin is intermediate, although it varies greatly depending
on the skin surface involved. Tissues with high resistance tend to heat up and coagulate in
response to electrical flow.
12. A 26-year-old male presents concerned he has suffered a brown recluse spider bite. He has a
1-cm red lesion on his forearm without scaling or vesicular fluid. Which of the following is the
most appropriate next step in management?
B. Dapsone 100 mg PO
D. Latrodectus antivenin
E. Bactrim DS
Answer A. The vast majority of people who claim to have a spider bite usually have either a
developing abscess due to methicillin-resistant Staphylococcus aureus (MRSA) or self-limited
minor, local skin irritation. To make the diagnosis of a spider bite, the patient actually has to
witness the spider biting them and the spider has to be one that is known to bite humans.
Treatment for minor skin lesions (such as the one described in the question) involves education
and reassurance. Humanbiting spiders that are known to cause significant disease include the
brown recluse (Loxosceles reclusa) and the black widow (Latrodectus mactans). Brown recluse
pg. 622
bites can sometimes cause large necrotic skin lesions. Dapsone has unclear benefits though
some experts recommend its use in the setting of true skin necrosis. Most brown recluse bites
should be managed with supportive care only. Black widow spider bites rarely cause skin lesions.
Calcium gluconate is a somewhat outdated therapy for black widow spider bites based on the
finding that many patients with black widow bites presented with hypocalcemia. This electrolyte
abnormality was more likely a result of (rather than a cause of) constant muscle contraction,
including the classic “pseudoperitoneum” of the abdomen that occurs with some black widow
spider bites. Black widow bites should be treated with supportive care and benzodiazepines.
Bactrim can be a useful adjunct in patients with abscesses presumed secondary to MRSA.
13. A 34-year-old male is brought to the ED by paramedics after collapsing due to extreme
exhaustion while running a marathon. Which of the following features indicate a diagnosis of
heat stroke rather than simple heat exhaustion?
A. Temperature >103.5°F
B. Myalgia
D. Syncope
E. Paresthesias
Answer C. Heat stroke is defined as the presence of elevated temperature (generally >104°F)
due to some heat stress combined with alteration of mental status. The degree of temperature
elevation by itself does not signify the presence of heat stroke; there must be some measure of
alteration of mental status such as agitation, confusion, delirium, or frank somnolence.
Myalgias, syncope, paresthesias, edema, rash, fatigue, and other nonspecific symptoms are
categorized under heat exhaustion. Treatment of heat exhaustion requires aggressive fluid
therapy along with passive cooling measures. Treatment of heat stroke mandates undressing
the patient and aggressive active and passive cooling measures such as cooled IV fluids, fanned
cool mist, and ice water immersion.
14. A 33-year-old male is bitten on his hand by his friend’s pet rattlesnake. Which of the
following is the most important next step in management?
A. Heat packs
D. Splint immobilization
pg. 623
E. IV methylprednisolone
Answer D. Rattlesnakes are part of the crotalid family of snakes whose venom contains cytotoxic
compounds that cause direct skin necrosis and sometimes are severe enough to result in
disseminated intravascular coagulation and delayed compartment syndrome. Signs of
envenomation generally occur within 6 hours of the bite, unlike elapid envenomation, which can
be delayed up to 12 hours. Management of crotalid envenomation involves immobilization with
a loose-fitting splint to minimize skin and soft-tissue destruction, tetanus prophylaxis, and
antivenin. Home remedies such as heat, ice, electric shock therapy, and tourniquets are rarely
helpful and can easily worsen tissue necrosis. Oral suction is never indicated and usually seed
the open wound with oral flora, which can cause considerable morbidity. Commercial suction
devices have not been shown to improve clinical outcomes or remove sufficient quantities of
venom in vivo. Corticosteroid therapy is not indicated in cases of envenomation and can
increase the risk of secondary infectious complications.
15. A 76-year-old female presents to the ED in the middle of summer complaining of swelling of
her ankles and feet. The daily high temperature has exceeded 100°F for the last 10 days. Which
of the following is true?
pg. 624
16. Which of the following is a unique aspect of tarantulas?
A. Their abdominal hairs can become embedded in the skin resulting in subsequent allergic
reactions.
B. In addition to venom, they serve as a vector for Borrelia spp. and may transmit Lyme disease.
C. Their venom triggers widespread mast cell degranulation resulting in diffuse flushing and
pruritus.
Answer A. Although tarantula bites may inflict a significant amount of pain, there is usually
minimal erythema and swelling at the bite site. Severe envenomation is extremely uncommon
and fatalities have not been described in the United States. However, tarantulas are covered
with “urticating hairs” that it can cast out toward a victim. The hairs become embedded in the
skin and may cause an intense inflammatory response, resulting in pruritus and occasionally
erythematous papules. After rubbing the area, patients may also unintentionally transfer the
hairs to their eyes resulting in a severe keratoconjunctivitis that requires ophthalmologic referral
for treatment and hair removal.
17. Which of the following areas is susceptible to thermal burns that are often deeper than they
initially appear?
A. Ears
B. Medial thighs
D. Perineum
Answer E. All of these areas are characterized by thin skin which is easily damaged when
exposed to heat. Thus, physicians who initially manage these wounds should presume that
burns to these areas are deep and take a very conservative approach to therapy and follow-up.
18. A 23-year-old previously healthy female is brought in for evaluation after a house fire. The
patient had been sleeping when she was pulled out of a burning bedroom by her father. She is
awake but somnolent, and complains of a headache as well as a burning sensation to her arms
pg. 625
and face. EMS reports her CO level was 20% on initial assessment. What is the best means to
determine if she has concomitant cyanide toxicity?
D. Order coximetry
Answer C. Unfortunately, cyanide levels are not available at the point of care. Instead, lactate is
frequently used as a surrogate for cyanide intoxication, as lactate levels >10 mmol/L are strongly
suggestive of cyanide poisoning. In addition, patients will have a severe metabolic acidosis with
a significant anion gap. Thus, a lot of useful information can be learned quickly with a venous
blood gas. Co-oximetry could verify EMS’ report of carboxyhemoglobin as well as the
methemoglobin level, but neither of these things will change presumed cyanide management in
this setting. Amyl nitrite (or other nitrites) is contraindicated in combined CO and CN poisoning
since induced methemoglobinemia will further decrease oxygen carrying capacity and worsen
tissue hypoxia.
19. Which of the following treatments has been shown to be effective in the prevention of acute
mountain sickness (AMS)?
A. Propranolol
B. Acetazolamide
C. Furosemide
D. Caffeine
E. Nifedipine
Answer B. Only acetazolamide and dexamethasone have been shown to be effective for the
prevention of AMS. Acetazolamide is a carbonic anhydrase inhibitor, which prevents the
reabsorption of bicarbonate in the proximal tubule. The resulting bicarbonate diuresis causes a
metabolic acidosis within 1 hour of ingestion, which stimulates ventilation and speeds
ventilatory acclimatization. In addition, acetazolamide reduces periodic breathing during sleep,
thereby eliminating the associated apnea and hypoxia. Dexamethasone has also been shown to
reduce the symptoms of AMS, although its mechanism of action is not known. Furthermore,
patients can experience a rebound phenomenon upon stopping the drug, so it is generally
reserved for the treatment of AMS rather than prophylaxis. Nifedipine is useful for prophylaxis
pg. 626
of high-altitude pulmonary edema (HAPE), but it has no role in AMS. Caffeine stimulates the
hypoxic ventilatory response but has not specifically been shown to reduce the incidence of
AMS.
20. What is the half-life of carboxyhemoglobin with a 100% oxygen nonrebreather mask?
A. 6 hours
B. 3 hours
C. 90 minutes
D. 60 minutes
E. 30 minutes
21. A 29-year-old male presents with an extremely pruritic rash several hours after running
through a forest (Fig. below). Which of the following is the next best step in management?
A. Supportive care
B. Prednisone
C. Methotrexate
D. Cephalexin
E. Trimethoprim–sulfamethoxazole
pg. 627
Answer B. The patient has evidence of poison ivy, or Toxicodendron dermatitis. The
pathogenesis is due to an allergic contact dermatitis to urushiol, a substance found in poison ivy,
sumac, and oak. Symptoms can develop in as little as a few hours, but can also be delayed for
several days after exposure. While supportive care alone is reasonable, an oral prednisone
course of 2 to 3 weeks can improve symptoms significantly. Methotrexate is too strong an
immune-suppressant for management of poison ivy. Antibiotics are not indicated except in
cases of secondary cellulitis.
22. A 22-year-old male is brought to the ED after a near-drowning episode. He was at a party on
his friend’s yacht and fell off the boat while trying to impress his friends. He was submerged for
about a minute and briefly lost consciousness before being transported to the ED. He currently
complains of mild shortness of breath. His vital signs are 99°F, 100, 18, 120/70, 95% RA. His
physical examination is unremarkable. Chest x-ray is normal. Which of the following is the most
appropriate next step in management?
A. Discharge home
C. IV methylprednisolone
D. IV piperacillin–tazobactam
Answer B. Drowning victims can develop serious, delayed pulmonary complications, such as
acute respiratory distress syndrome and pneumonia. Observation of symptomatic or hypoxic
patients is indicated. Arterial blood gas can be helpful in evaluating for hypercarbia, which is
more difficult to assess noninvasively than hypoxia. Corticosteroids and prophylactic antibiotics
have not been shown to improve outcomes in drowning victim patients (or other aspiration
patients). Endotracheal intubation should be reserved for patients with altered mental status,
hypercarbia, or hypoxemia refractory to maximal mask oxygen.
23. A 22-year-old female presents with cyanosis. She complains of shortness of breath,
headache, and slurred speech. Her friends report that she’s been getting high, but they don’t
know what she’s using. SaO2 is 85% despite aggressive oxygen therapy. Blood drawn from the
patient appears extremely dark. Which of the following is the most likely drug ingested?
A. Lorazepam
B. Dextromethorphan
C. Amyl nitrate
D. Diphenhydramine
pg. 628
E. Ketamine
B. Direct exposure to dry heat sources such as heat lamps or open fires
Answer C. There is some disagreement regarding the optimal temperature for rewarming or
thawing frostbitten tissue. However, all authors agree that optimal rewarming occurs through
immersion in a water bath with a closely regulated temperature. Most sources cite 40°C to 42°C
as the optimal range, although temperatures as low as 35°C have been recommended as they
tend to cause less pain during rewarming. Upon arrival, the affected area should be rapidly
rewarmed for 15 to 30 minutes or until thawing is complete. Indicators of successful thawing
include increased flexibility, erythema, and hyperemia. Rewarming can be intensely painful, and
parenteral analgesics may be required, especially in cases of deep frostbite. Direct tissue
massage should never be performed as it may cause increased tissue loss. In addition, field
rewarming and rewarming with direct heat sources should never be performed because of the
high risk of incomplete thawing and refreezing which results in increased tissue loss.
pg. 629
B. Nitrogen narcosis
E. Alternobaric vertigo
Answer A. AGE is the second most common cause of diving-related death and the most severe
form of pulmonary barotraumas. As a diver ascends, alveoli expand due to the decreasing
atmospheric pressure. If the diver does not continuously expire during ascent, the alveoli will
expand and may rupture. Air may then cross the ruptured alveolar–capillary membrane, enter
the pulmonary venous circulation, and subsequently embolize to any organ system.
Embolization to the coronary or cerebral (usually anterior or middle cerebral) arteries is most
catastrophic. The most common presentation of AGE is neurologic, consisting of decreased
consciousness, dizziness, confusion, headache, cranial nerve symptoms, hemiplegia, and
hemisensory loss. Any diver who surfaces unconscious or who loses consciousness within 10
minutes of surfacing should be presumed to have AGE until proven otherwise. The loss of
consciousness (LOC) is a sharp contrast from DCS, in which LOC is rare. Alternobaric vertigo is
vertigo that results from a pressure differential between the two middle ears. It results when
patients with unilateral Eustachian tube dysfunction or blockage (when one Eustachian tube is
more or less patent than the other) are exposed to the ambient pressures associated with diving
or flying.
26. An 18-year old male presents with the rash shown (see Fig. below). Which of the following
substances is the likely trigger?
A. Arsenic
B. Brass
C. Nickel
D. Dermatium
E. Rhus species
pg. 630
Answer C. The patient likely has contact dermatitis from a nickel-containing belt buckle. Nickel
sensitivity is seen in about 10% of the population. Treatment is simple removal of the offending
agent and topical antihistamines as needed to manage pruritus. Arsenic does not typically cause
contact dermatitis but can cause cancer if chronically exposed to the skin. Brass is an alloy of
copper and zinc and does not usually cause dermatitis, unless it contains nickel. Dermatium is a
made-up name. Rhus is the former name of the Toxic dendron species which cause poison ivy,
oak, and sumac. It can certainly cause contact dermatitis, but the location is atypical and more
likely related to a belt buckle containing nickel.
27. A 5-year-old male is brought by his mother for a rash on his lower abdomen (Fig. below). You
suspect contact dermatitis from the metal on his jean button. Which of the following is the most
likely cause?
A. Silver
B. Platinum
C. Copper
D. Nickel
E. Zinc
pg. 631
Answer D. Nickel is the most common overall cause of contact dermatitis in the world and the
second most common cause in the United States (after poison ivy/oak). Nickel is found in many
metals on clothes, and many silver and gold chains use nickel as a filler metal. Notably, “sterling
silver” chains do not contain significant amounts of nickel. Symptoms of allergic contact
dermatitis include a pruritic, maculopapular rash. Treatment involves avoiding nickel substances
and mild topical corticosteroids.
28. Two men are hiking in the mountains of Colorado when one complains of feeling sick. Which
of the following is the most sensitive indicator of high-altitude cerebral edema (HACE)?
A. Cerebellar ataxia
B. Vomiting
D. Seizures
E. Slurred speech
Answer A. HACE is the most severe form of high-altitude illness and typically follows acute
mountain sickness (AMS). Therefore, symptoms of AMS (headache, fatigue, nausea, dizziness,
anorexia, and difficulty sleeping) precede the development of HACE. However, AMS may
progress to HACE in as a little as 12 hours although 1 to 3 days is more typical. Most cases of
severe HACE occur above 12,000 ft. In addition to symptoms of AMS, the cough and dyspnea of
high altitude pulmonary edema (HAPE) are also typically present. Specific signs for HACE include
generalized seizures, slurred speech, rare neurologic deficits, delirium, and ataxia.
29. Patients with plantar puncture wounds that occurred in the absence of any footwear:
Answer B. Infection of plantar wounds suffered in the absence of footwear occurs in roughly
10% of patients, but depends on several factors, including the degree of contamination and the
pg. 632
patient’s underlying health. When infections do occur, Streptococcus and Staphylococcus
(including MRSA) are the most common causative organisms. Pseudomonas aeruginosa does
not typically cause infection of plantar wounds unless the patient was wearing a sneaker at the
time of the wound puncture. Patients wearing tennis shoes saturated with sweat are at
increased risk for Pseudomonas infection. Unfortunately, ciprofloxacin resistance is common
and rising, and there are no good oral alternatives as newer generation fluoroquinolones have
decreased activity against Pseudomonas. While gentle cleaning of puncture wounds should
always be performed, high-pressure irrigation should be avoided since there is a theoretical risk
of introducing bacteria into the deeper tissues.
30. Which of the following is the most useful treatment modality in patients with heat stroke?
B. Intravenous dantrolene
Answer A. There are two accepted and commonly used rapid cooling modalities in heat stroke.
One involves pouring or spraying tepid water over a patient and using a fan to enhance
evaporative cooling through convection. The second is immersion in an ice water bath. Some
authors favor ice water immersion because it is thought to result in more rapid cooling.
However, ice water immersion presents challenges as patient monitoring and resuscitation is
more difficult and immersion containers may not be readily available. The application of ice
packs to the axilla and groin should be considered an adjunct only as its use is inferior to tepid
water and fanning alone. Antipyretics and dantrolene play no role in the management of heat
stroke although dantrolene may be considered in refractory cases, thought to be due to
malignant hyperthermia. More invasive cooling means, such as gastric, pleural, or bladder
irrigation, as well as intravascular cooling are reserved for only the most severe, refractory cases
and have not been studied to support their routine use.
31. A group of children are playing outside in the rain when one of them suffers a witnessed
lightning strike. The strike did not directly hit the child but it hit the ground very close to where
the child was standing. Which of the following is true?
C. Lower extremity paralysis accompanied by mottled, blue, cool, and pulseless extremities
typically resolves without treatment.
pg. 633
D. “Flashover” commonly causes diffuse superficial burns to >50% of the total body surface
area.
A. QT interval prolongation
B. Sinus bradycardia
C. Atrial fibrillation
D. J waves
Answer E. Though a transient tachycardia may occur, sinus bradycardia develops and the heart
progressively slows as the body cools. In fact, the finding of a relative tachycardia in a
hypothermic patient should trigger a search for a cause. The PR, QRS, and QT intervals may all
become prolonged although this finding is most common in the QT segment and also correlates
with the degree of hypothermia. J waves, or Osborn waves, are the most discussed EKG feature
in hypothermia. They are additional waves at the junction of the QRS complex and the ST
segment and tend to be upright in aVL, aVF, and the lateral precordial leads. Any atrial or
ventricular dysrhythmia may occur, but atrial fibrillation is the most common and it usually
spontaneously converts to sinus rhythm upon rewarming.
33. A 23-year-old male is brought to the ED by paramedics after a suicide attempt. He had
locked himself in the garage and turned on his car for several minutes until his roommates
caught him. He is somnolent and barely responsive. You suspect carbon monoxide poisoning. His
pg. 634
pulse oximetry is 99%. Which of the following is the most accurate statement regarding this
patient?
A. Because pulse oximetry is 99%, he is unlikely to have had a significant carbon monoxide
exposure.
C. PaO2 level is the most accurate test in determining the extent of carbon monoxide poisoning.
Answer E. Carbon monoxide displaces oxygen from hemoglobin due to its far greater affinity for
the heme subunit. Pulse oximetry senses only two specific wavelengths of light—oxygenated
hemoglobin and deoxygenated hemoglobin—and calculates a percentage based on the ratio of
the two. Unfortunately, carbon monoxide is sensed as oxygenated hemoglobin and pulse
oximetry is falsely normal. Primary pathophysiology in carbon monoxide poisoning is inadequate
oxygen delivery to tissues despite a normal to even slightly elevated dissolved oxygen level
(PaO2). Carboxyhemoglobin to oxyhemoglobin ratio is the test of choice in evaluating carbon
monoxide poisoning. A ratio of above 25% is considered serious toxicity, though this is subject to
various other clinical features. Pyridostigmine is a cholinesterase inhibitor used to treat MG and
has no role in the management of carbon monoxide poisoning. Patients with carbon monoxide
poisoning should get maximal possible oxygen therapy to compete with carbon monoxide for
hemoglobin binding. This can be performed noninvasively with a 100% nonrebreather mask or
using endotracheal intubation and mechanical ventilation. Any patient with altered mental
status or somnolence should have emergent blood glucose level checked. Additionally, patients
with suicidal attempts often ingest large quantities of alcohol, which can cause hypoglycemia.
34. Which of the following mechanisms of heat loss can be increased therapeutically to treat
heat stroke?
A. Evaporation
B. Conduction
C. Radiation
D. A and B
E. A, B, and C
Answer D. Evaporation is the most significant mechanism of heat loss at most climates (except
for very cold areas). It refers to the transfer of heat that occurs to convert a liquid to a gas. For
pg. 635
example, in a patient who is sweating from running, liquid sweat evaporates to gas by using up
excess body heat. Conduction refers to a colder object taking heat from a warmer object. Fan
mist therapy uses both evaporation and conduction in decreasing heat. Radiation refers to
constant background emission of heat from all bodies and cannot be increased or decreased
therapeutically.
35. What is the most common electrocardiographic (EKG) abnormality in patients with
heatstroke?
A. Sinus bradycardia
B. Atrial fibrillation
C. QT interval prolongation
D. Ventricular fibrillation
Answer C. QT interval prolongation is the most common EKG abnormality in patients with heat-
related illness. Interestingly, QT interval prolongation is also very common in patients with
hypothermia. Other common EKG findings include sinus tachycardia, atrial fibrillation, SVT, right
bundle branch block, and occasional ST segment changes. Sinus bradycardia and ventricular
fibrillation do not commonly occur.
36. The joints most commonly affected by decompression sickness (DCS) are:
B. Knees
Answer D. DCS is due to the presence of nitrogen bubbles in the blood and tissues. It is divided
into type I DCS and type II DCS. Type I DCS affects the musculoskeletal system, skin, and
lymphatic vessels. Type II DCS involves all other organ systems. The shoulder and elbow are the
most common joints involved in type I DCS. The arthralgias experienced by patients with type I
DCS are known as the bends. Joint pain may be reduced by inflating a blood pressure cuff over
the affected joint to 150 to 200 mm Hg. This may also be used to aid in diagnosis although it has
a poor sensitivity.
pg. 636
37. Risk factors for decompression sickness (DCS) include which of the following?
B. Obesity
D. Dehydration
Answer E. The presence of a patent foramen ovale is probably the most significant risk factor for
DCS. In general, the risk of DCS increases with the length and depth of a dive (e.g., violating the
nodecompression limits). Other risk factors include increasing age, obesity (nitrogen is lipid
soluble), dehydration, fatigue, diving at high altitude, flying soon after diving, performing heavy
work while diving, exercise after diving, cold water temperature, and rough seas.
38. A 28-year-old male presents to the ED with an extensive vesicular, weeping, and crusted
eruption arranged in a linear pattern on his lower legs. He tells you that he cleaned out the
brush from the woods behind his house a couple of days ago. You suspect he has a contact
dermatitis due to poison ivy. In addition to cool compresses and antihistamines to help control
pruritus, the ideal course of corticosteroids should be:
Answer D. Severe poison ivy requires systemic corticosteroids. Commercially available “dose
packs” should be avoided because they provide an inadequate amount and duration of
medication. In addition, a prolonged course of prednisone treatment is generally required due
to the high chance of rebound dermatitis if prednisone is discontinued abruptly. Therefore, a 14-
to 21-day taper generally works best. Not surprisingly, most ED clinicians do not prescribe such a
lengthy course of prednisone, perhaps because of their lack of exposure to rebound dermatitis
in follow-up.
pg. 637
39. A 45-year-old male presents with abrupt onset of fever, cough, chills, and shortness of
breath. One week earlier, he was present in a government office where an explosion thought to
be a terrorist attack occurred. He appears toxic and in moderate respiratory distress. Chest x-ray
is shown in Figure below. Which of the following is the most appropriate therapy at this time?
A. Ciprofloxacin
B. Aztreonam
C. Tobramycin
D. Ceftriaxone
E. Methylprednisolone
Answer A. The patient likely has pulmonary anthrax due to the gram-positive organism Bacillus
anthracis. Anthrax is a CDC Category A agent of bioterrorism, with the highest risk of
weaponization and transmission. Pulmonary anthrax is caused by the inhalation of anthrax
spores, which causes a pneumonia-like picture a week after exposure. Hemorrhagic
mediastinitis is the characteristic radiographic manifestation, though CT scan should be ordered
if initial chest x-ray is negative. Firstline therapy is with ciprofloxacin and second-line is with
doxycycline. Aztreonam and tobramycin both cover gram-negative bacteria with minimal gram-
positive coverage and should not be used. Anthrax has demonstrated intermediate resistance to
ceftriaxone. Corticosteroids such as methylprednisolone are not routinely indicated in the
treatment of anthrax. Anthrax spores should be disinfected with bleach solution, as ordinary
alcohol solutions have no effect.
pg. 638
40. An otherwise healthy 22-year-old male presents to the ED after a “spider bite.” He states
that he felt a pinprick sensation on his hand when he lifted the hood of an old car he has been
storing in his driveway. He continued working, but soon developed crampy muscle aches that
spread up his arm and now seem most severe in his chest and abdomen. In the ED, his abdomen
is rigid and the patient is complaining of dizziness, nausea, and severe “stomach cramps.” Which
of the following is the likely culprit?
A. Tarantula
B. Hobo spider
D. Wolf spider
Answer E. Black widow spider bites are characterized by an initial pinprick sensation followed by
a mild local inflammatory response. However, within 1 hour, crampy myalgias develop at the
bite site and spread up the extremity, eventually involving the entire body. Classically, myalgias
are most intense in the chest and abdomen and patients may present with a rigid abdomen that
is impossible to differentiate clinically from peritonitis. Patients also frequently have associated
hypertension, diaphoresis, nausea, vomiting, headache, dizziness, and weakness. Symptoms
typically begin to abate within a few hours with only supportive care. Due to their small size,
however, children may suffer from complete cardiovascular collapse with the same degree of
envenomation.
41. A 33-year-old male presents with acute onset of bilateral lower extremity weakness several
minutes after returning from a deep scuba dive. His physical examination reveals 3/5 strength in
both lower extremities and he has patchy loss of sensation throughout the lower half of his
body. Which of the following is the most likely cause of his symptoms?
A. Epidural abscess
B. Epidural hematoma
Answer C. The patient likely has type II decompression sickness (DCS). DCS occurs when nitrogen
bubbles present in the tissues at high pressures underwater expand to form larger bubbles at
the lower pressure of sea level when ascending. Type I DCS is also known as the bends and
pg. 639
manifests as severe arthralgias and pruritus. Type II DCS is more serious, causing nitrogen
bubble formation in high-fat–containing tissues like white matter. Significant spinal cord
damage can occur from nitrogen bubble formation. Paresthesias and weakness can be
asymmetric, as nitrogen bubble formation and expansion is irregularly distributed. Spontaneous
epidural abscess and hematoma formation would be unlikely in a 33-year-old male without
known past medical history. GBS is possible but tends to be symmetric and does not exhibit
patchy sensation loss. Middle ear barotrauma refers to the inability to equilibrate the pressure
on both sides of the tympanic membrane, resulting in rupture and consequent dizziness,
vertigo, and delayed infection.
42. Which of the following is true regarding the after-drop phenomenon in hypothermic
patients?
C. After-drop is caused by the return of cold peripheral blood to the core upon rewarming.
Answer E. Core temperature afterdrop refers to the phenomenon of a further decline in core
temperature after the initiation of warming. It has been blamed for the development of cardiac
dysrhythmias and active external rewarming methods are thought to amplify the drop. This has
led to recommendations to heat the core before the extremities to avoid peripheral vasodilation
and a brisk return of cool, acidic blood back to the core where it decreases core temperature
and pH. In most healthy patients afterdrop has not been shown to be of clinical importance but
elderly patients with less cardiopulmonary reserve and patients with volume depletion may be
adversely impacted.
43. A 52-year-old homeless male presents for evaluation of frostbite in his hands (Fig. below). In
patients suffering severe frostbite, which of the following is a positive prognostic sign?
pg. 640
Answer D. Hyperemia and erythema are expected findings upon rewarming frostbitten tissue,
but a residual violaceous color is an ominous sign. Positive prognostic signs include a return to
normal pliability, early return of normal sensation, and early formation of large clear blebs in the
affected area. Persistent firmness of the SQ tissue, lack of edema, or the delayed development
of hemorrhagic blebs all portend a worse outcome.
44. A 33-year-old male is bitten by the spider pictured (Fig. below). Which of the following is the
next best step in management?
B. Antivenom
C. Hemodialysis
pg. 641
Answer A. The spider shown is the black widow spider, also known as Latrodectus mactans.
Bites from this spider can cause severe muscle spasms, including in the abdominal wall, leading
to the term “pseudo-peritoneal signs.” Most bites do not cause such severe muscle spasms and
mild to moderate symptoms can be treated symptomatically with analgesics and
benzodiazepines. Severe envenomations, heralded by severe spasms and vomiting, can be
treated with antivenom. Although very effective at alleviating symptoms, antivenom does carry
with it the risks of anaphylaxis, allergic reaction, and serum sickness. Hemodialysis is not usually
needed for black widow spider bites. Plastic surgery consultation may be pursued in the
outpatient setting for brown recluse spider bites which have caused the characteristic necrotic
lesion. A negative inspiratory force (NIF) assessment is used to determine the degree of
diaphragmatic weakness and respiratory failure in cases of Guillain–Barre syndrome and elapid
snake or scorpion envenomations. A normal NIF is 60 cm water; <20 indicates significant
respiratory muscle weakness portending respiratory failure.
45. A 47-year-old police officer is brought to the ED after being found unresponsive. The officer
was called to respond to an unidentified young female, who was apparently sleeping in a car
parked at a shopping center. Bystanders note that after knocking on the windows, the officer
opened the door and collapsed to the ground shortly afterward. The bystanders at the scene
stated that there was a sharp, foul odor similar to a “rotten egg” that appeared to be coming
from the vehicle, so they kept their distance. Which of the following antidotes is most likely
beneficial in treating this officer?
A. Methylene blue
B. Sodium nitrite
C. Hydroxycobalamin
D. Sodium thiosulfate
E. Succimer
Answer B. The officer’s sudden collapse in the setting of a gaseous “rotten egg” smell strongly
suggests the presence of hydrogen sulfide. The use of hydrogen sulfide as a relatively easy,
painless means to commit suicide has gained popularity in recent years. At low concentrations,
hydrogen sulfide may cause only minor irritation, cough, and a sensation of dyspnea. At higher
levels, hydrogen sulfide inhibits mitochondrial cytochrome oxidase, which uncouples electron
transport and terminates cellular respiration. This has profound, rapid effects on the nervous
system and quickly leads to coma. Patients who remain awake will often improve after being
moved to an area with uncontaminated air and with supplemental oxygen. More severely
affected patients require aggressive supportive care (ventilation) and specific treatment with
sodium nitrite, which induces methemoglobinemia. As in patients with cyanide intoxication,
induction of methemoglobinemia is helpful by providing an alternative binding site for hydrogen
sulfide. The combination of hydroxycobalamin (direct cyanide binding) and sodium thiosulfate
pg. 642
(enhanced cyanide detoxification) is the treatment of choice for cyanide poisoning. They are not
effective in treating hydrogen sulfide poisoning. Methylene blue is used to treat
methemoglobinemia. Succimer (meso-2,3-dimercaptosuccinic acid) is an adjunctive treatment
for lead poisoning.
46. The most common manifestation of barotrauma associated with scuba divers during descent
is:
A. Nitrogen narcosis
B. Baro-sinusitis
D. Facial barotrauma
Answer E. Middle ear barotrauma or “middle ear squeeze” is the most common barotrauma-
related problem during descent. Ear pain during descent is the most common symptom,
although transient hearing loss may occur. Additionally, if the diver continues to deeper water,
further increases in pressure may result in tympanic membrane (TM) rupture. TM rupture may
alleviate some of the pain but it also exposes the middle ear to cold water, which may result in
nystagmus and vertigo. A facial nerve palsy also uncommonly occurs. Nitrogen narcosis occurs
during descent as increased levels of nitrogen are “forced” into the tissues. Symptoms typically
occur at approximately 100 ft and resemble alcohol intoxication. Barosinusitis presents as facial
pain that results from pressure changes in one of the facial sinuses. Facial barotrauma occurs
when divers fail to equilibrate the airspace created by a dive mask over the eyes and nose. The
relative negative pressure causes petechial hemorrhages on the face, subconjunctival
hemorrhage, and conjunctival edema. Temporomandibular joint dysfunction is caused by teeth
clenching and malocclusion resulting from a poorly fitting mask. The pain is felt near the ear and
can be confused with middle ear barotrauma.
47. A 22-year-old male is brought in for evaluation after drowning. Which of the following is the
best predictor of a good neurologic outcome?
D. Absence of cyanosis
pg. 643
Answer B. Short submersion time (<5 minutes) has been consistently shown to predict better
neurologic outcomes. All other predictive factors have not been as robust in terms of their
predictive value. However, predictors of a poorer prognosis include age >14 years old, time to
resuscitation >10 minutes, duration of CPR >25 minutes, GCS <5, and initial pH <7.1.
B. In high-voltage injuries, the extent of cutaneous burns is a good predictor of internal tissue
damage.
C. Asystole is the most common dysrhythmia resulting from low-voltage electrical injury.
D. In contrast to other mass casualty traumatic events, patients without signs of life should be
resuscitated first.
Answer D. For a given voltage, AC is thought to be three times more dangerous than DC. This is
due to the fact that AC current causes repetitive muscle contraction or tetany once the “let-go
current” is exceeded. This results in prolonged exposure and more severe injury. High-voltage
electrical injuries should be treated like crush injuries, because there is often a large amount of
tissue damage underneath normal appearing skin. It is impossible to predict the degree of
underlying damage from the extent of cutaneous burns. Fewer than 10% of patients
experiencing low-voltage electrical injury develop a cardiac dysrhythmia. In those patients who
do suffer cardiac arrest due to an arrhythmia, ventricular fibrillation is most common. Triage
priorities are different in cases of high-voltage electrical injury or lightning strikes. Patients with
obvious signs of life tend to do well and can afford a small delay in definitive care. Furthermore,
due to the possibility of a good outcome with cardiopulmonary resuscitation (CPR), patients
without signs of life should receive immediate care.
pg. 644
Traumatology
➢ Chapter includes:
Adults, pediatric, geriatrics and pregnancy trauma
Head, neck, chest, abdomen trauma
Ventilation technics and procedure in trauma patient
ABCDE in trauma patient
Trauma patient wound and management
Traumatic brain hemorrhage
Hemorrhagic shock in trauma patient
➢ Exceptions
✓ Dental trauma in otolaryngology and dental chapter
✓ Eye trauma in ophthalmology chapter
✓ Non-accidental trauma in chapter psychiatry and
abuse
✓ Isolated extremity trauma in orthopedic chapter
pg. 645
1. Which of the following is the optimal vascular access for adult trauma patients?
E. Intraosseous catheter
Answer B. Trauma patients require rapid access with large-bore peripheral IVs to optimize fluid
administration. Two 14-g or 16-g IVs are ideal. Rate of fluid flow is inversely proportional to the
length of the vessel and directly proportional to the radius of the vessel to the fourth power.
Therefore, short, wide-bore catheters are preferred over long, narrow-bore catheters. A single
IV is not adequate, given the risk of expulsion from patient movement during trauma
resuscitation. Triple lumen catheters, though generally large-bore lines, are limited in their fluid
passage rates by their length, which can be up to 10 times that of a peripheral line. Cordis
catheters are shorter than triple lumens, and they are large-bore (8.5 Fr). Thus, they can achieve
significant infusion rates with a pressure bag or pump and are often included in trauma
resuscitations when massive transfusion or fluids are needed. Intraosseous catheters provide
ready vascular access in pediatric patients but are not preferred in the adult trauma patient due
to limited flow rates, difficulty of placement, and potential complications.
2. A 34-year-old male presents after a high-speed motor vehicle collision with shortness of
breath. A large flail segment is noted on his right lateral chest. Which of the following is the
most appropriate therapy at this time?
Answer D. Flail chest occurs during blunt thoracic trauma when three or more ribs are each
fractured in two places, causing a discrete chest wall segment that is unattached to the rest of
the chest wall. Paradoxical motion of the flail segment is characteristic: The flail segment moves
inward during inspiration and outward during expiration. Severity of the flail chest injury is due
to the underlying pulmonary contusion that results from the blunt trauma. Diagnosis is made by
physical examination and confirmed by either chest x-ray or CT. Management is directed at
pg. 646
treating the underlying pulmonary contusion and should first involve administration of 100%
oxygen to assess for the presence of severe pulmonary shunting. If the patient does not respond
to noninvasive oxygen and is persistently hypoxemic, then endotracheal intubation should be
performed. Hemothorax or pneumothorax may also be present and requires tube
thoracostomy. Decubitus positioning is unlikely to be helpful in treating the flail chest and may
exacerbate atelectasis in the contused lung region. A heavy weight placed on the flail segment is
also likely to exacerbate the pulmonary contusion with little benefit.
3. Which of the following is true regarding the focused assessment of sonography in trauma
(FAST) scan
Answer D. The FAST examination has greater accuracy than any single element of the history or
physical examination in blunt abdominal trauma. It is not as accurate for penetrating trauma as
it is for blunt. Ultrasound cannot reliably distinguish between blood, urine, or other fluids (e.g.,
ascites). FAST has been associated with significant operational improvements, including time to
surgery, CT utilization, and avoidable hospitalizations. Specificity tends to be higher than
sensitivity, and blunt abdominal trauma patients who appear ill despite a negative FAST
examination should have serial FAST examinations or CT imaging to fully evaluate the abdomen.
4. Which of the following is true regarding the potential space between the labeled structures
shown below?
pg. 647
Answer E. The potential space in this right upper quadrant view of the FAST scan is called
Morison pouch. In trauma, free fluid (assumed to be blood) will accumulate preferentially in the
right upper quadrant in a supine patient—this process will intensify with Trendelenburg
positioning. The rectouterine pouch is known as the pouch of Douglas and it is the most
posterior part of the peritoneal cavity. Morison pouch is part of the peritoneal cavity, not
retroperitoneum (which cannot be adequately visualized with the FAST scan). The pericardial
view should be the first view obtained in the FAST scan, as it addresses the most immediate life
threat in trauma after the primary survey (pericardial tamponade).
5. A 22-year-old G1 female at 28 weeks gestational age presents to the ED after a motor vehicle
collision. She has right-sided pneumothorax and a unilateral pubic ramus fracture, for which she
is being treated. Review of records indicates that her blood type is A negative. Which of the
following is the most appropriate next step in management?
E. No specific management
Answer D. Rh-negative patients with significant trauma should have RhIG administered to
prevent isoimmunization. The dosage varies with gestational age—in gestational age <12 weeks,
a 50-mcg dose is sufficient to prevent isoimmunization though there is some evidence that
pg. 648
treatment at all may be unnecessary. Patients at gestational age >12 weeks should have the
300-mcg dose. Patients at risk for greater fetomaternal hemorrhage (possible when gestational
age >16 weeks) should have the Kleihauer–Betke test performed to determine whether
additional doses of RhIG are needed. Transfusions of blood products in pregnant trauma
patients should be performed if clinical situations suggest hemodynamic compromise, ongoing
bleeding, or coagulopathy.
6. A 23-year-old female presents with pain in her right lateral chest after a low-speed motor
vehicle collision. She is most tender in the fifth rib at the posterior axillary line. Her vital signs
are normal. Which of the following is the most appropriate next step in evaluation?
A. Chest x-ray
B. Rib x-rays
C. CT abdomen/pelvis
D. CT brain
Answer A. The patient likely has a rib fracture or contusion given the focal pain and tenderness
in her fifth rib. However, the true danger of a rib fracture is not the bony injury itself, but
potential injury to the underlying structures, such as pleura, lung, liver, spleen, or kidney. The
fifth rib is cephalad enough that an intra-abdominal injury would be less likely than thoracic
injury. A chest xray is indicated to evaluate the lung parenchyma and the pleural lines. Rib x-rays
are not routinely indicated in patients with thoracic trauma, except in cases where multiple
fractures are suspected or elderly patients are involved, as significant intrathoracic or
intra-abdominal injuries occur at higher frequency in these instances. Abdominal CT scan may
be indicated with corroborative physical examination findings, but is never undertaken before
routine x-rays of the chest or pelvis in the trauma patient. Brain CT scan and cervical spine
radiographs are not indicated without loss of consciousness, altered mental status, focal
neurologic deficit, headache, or neck pain/tenderness.
7. A 35-year-old female presents in a coma (Glasgow Coma Scale 3) after a motor vehicle crash
and is intubated for airway protection. Further evaluation reveals no life-threatening chest,
abdomen, or pelvic injuries. Vital signs are normal. A computed tomography (CT) scan of the
head is normal. Which of the following is the most likely diagnosis?
A. Epidural hematoma
B. Subdural hematoma
pg. 649
D. Cerebral contusion
E. Intraparenchymal hematoma
Answer C. DAI is an important traumatic cause of coma that is not due to a mass lesion or frank
intracerebral hemorrhage. Initial CT scan is almost always normal in patients with DAI, but MRI
may show diffuse white matter disruption due to axonal fiber injury. Because of the difficulty in
gauging DAI on neuroimaging, the prognosis is based totally on clinical parameters. The duration
of coma obviously correlates with severity of injury. Patients with DAI who awaken from coma
within 24 hours may have few permanent disabilities. Those in coma for longer than 24 hours
tend to have much more grim outcomes, including persistent vegetative state or extreme
cognitive dysfunction. Most types of acute intracranial hemorrhage severe enough to lead to
coma would be evident on initial CT scan, including epidural or subdural hematomas, cerebral
contusions, and intraparenchymal hematoma. An important exception is subarachnoid
hemorrhage, which may not be visible on CT scan and is a common hemorrhagic cause of
altered mental status after trauma.
8. A 44-year-old male presents with hypotension after a motor vehicle collision. His chest x-ray is
normal. Pelvis x-ray is shown below. Which of the following is the most important next step in
management?
A. Foley catheterization
D. ED thoracotomy
pg. 650
Answer C. The patient has an open-book pelvis fracture in association with hypotension, which
may be rapidly fatal if not treated promptly. Temporizing management revolves around
reducing the effective volume into which hemorrhage can occur by tightly securing the pelvis
with a commercial device or simple bedsheet. Definitive management involves angiography with
embolization to control hemorrhage and surgical fixation to repair the pelvis fracture. Foley
catheterization may be performed in patients with pelvic fractures if there are no hard signs of
urethral trauma (e.g., blood at the urethral meatus), but priority should be given to hemorrhage
control rather than evaluation of urethral trauma. CT should never be performed on the
hemodynamically unstable trauma patient. Thoracotomy is not indicated in patients with blunt
traumatic mechanisms as survival rates are dismally low. Additional radiographs of the pelvis
should be performed after hemodynamic compromise has been addressed.
9. When compared to younger adults, which of the following traumatic injuries is more common
in elderly patients?
A. Subdural hematoma
B. Odontoid fracture
C. Flail chest
Answer E. There are several normal physiologic changes that occur with aging and that place
elderly patients at greater risk of serious injury from trauma. Cerebral atrophy results in
stretching of the dural bridging veins, increasing the risk of subdural hematoma formation after
falls and relatively minor head trauma. Degenerative joint disease and osteoporosis result in an
increased tendency to fracture bones after falls and blunt trauma. Due to these degenerative
changes, the cervical spine is less mobile in elderly patients and is more commonly fractured.
Type 2 odontoid fractures are the most common cervical spine fractures in the elderly. In
addition, hyperextension injuries lead to central cord syndrome in which the ligamentum flavum
is thought to buckle into the spinal cord, resulting in a contusion to the cord’s central elements.
This results in flaccid paralysis of the upper extremities but relatively unaffected lower
extremities (although they may suffer from spastic paralysis in large cord lesions). The chest wall
becomes more rigid and the lungs become less compliant in elderly patients. This places them at
risk for flail chest, even from relatively minor injuries such as a simple fall. Elderly patients with
even single rib fractures should be admitted in the setting of concomitant lung disease such as
COPD.
pg. 651
10. A 34-year-old male is brought into the emergency department (ED) after a motor vehicle
collision. Which of the following findings is an indication to perform a computed tomography
(CT) of the abdomen/pelvis with IV contrast to look for renal injury in this patient?
A. Microscopic hematuria
B. Gross hematuria
C. Flank pain
D. Flank ecchymosis
E. Penile hematoma
Answer B. In adult blunt trauma patients, the indications to perform an evaluation for renal
injury include gross hematuria, microscopic hematuria plus hypotension, or significant
deceleration injury. In the absence of shock, microscopic hematuria is not an indication for CT in
blunt trauma patients. However, microscopic hematuria may also be a marker of urethral or
bladder injury and may warrant further investigation depending on the mechanism of injury and
findings on physical examination. The presence of flank physical examination findings is not
indicative of renal injury significant enough to warrant imaging. Penile involvement can certainly
cause urethral damage, but it does not confer a higher risk of renal injury.
11. A 26-year-old female involved in an MVC has persistent abdominal tenderness after a
negative CT of the abdomen and pelvis. Her vital signs are normal and there is no seatbelt sign.
Which of the following is true?
A. She should be admitted for further observation and serial physical examinations
B. She should undergo ED observation with repeat CT imaging in 6 hours if her tenderness
persists
C. A FAST scan should be performed to detect intraperitoneal bleeding that may have been
missed by CT
E. She should be under repeat CT testing with oral contrast to further investigate possible bowel
injuries
pg. 652
significant improvement in diagnostic yield. Furthermore, isolated abdominal tenderness is not
predictive for worrisome intraabdominal injury. However, disposition decisions should be
individualized depending on clinician concern and degree of suspicion.
12. A 34-year-old male complaining of neck pain and leg pain is brought in by EMS after hitting a
tree while driving his car. A cervical spine CT is negative and x-rays of his foot reveal no fracture.
He has no neurologic complaints and his neurologic examination is normal. However, he has
persistent cervical spine tenderness on examination. Which of the following is true?
Answer E. Modern, multidetector CT scanners, which are ubiquitous in the United States, are
nearly 100% sensitive in detecting clinically significant cervical spine injuries. Flexion and
extension views don’t add additional information. While MRI is more sensitive for detecting
ligamentous injuries, those injuries are almost never clinically relevant in patients without
neurologic complaints or findings. Symptomatic patients most commonly have stable traumatic
cervical disk herniations and symptoms and findings can guide management. Patients with
persistent tenderness without neurologic complaints or findings may be discharged home with
outpatient follow-up. Some guidelines suggest discharging patients in a rigid collar (such as a
Miami-J, Aspen, or Philadelphia collar) with short-term follow-up, ideally in consultation with
the trauma or neurosurgical team.
13. An 11-year-old male hit the curb while riding his bicycle and was thrown forward into his
handlebars. The classic injury associated with this accident is:
A. Myocardial contusion
B. Pancreatic injury
C. Liver contusion
D. Splenic contusion
E. Diaphragmatic rupture
pg. 653
Answer B. Although handlebar injuries are simply a form of blunt abdominal trauma, which
therefore put patients at risk for liver and spleen injuries, there is an increased risk of pancreatic
and small bowel injuries. Classically, pancreatic or duodenal injuries are associated with
pediatric handlebar injuries. Patients with pancreatic injury often develop delayed symptoms
and may have a relatively benign presentation initially. Eventually, they develop abdominal pain,
nausea, and vomiting and have evidence of pancreatic injury by elevated enzymes on laboratory
analysis. Acute closed-loop small bowel obstruction and rupture may also occur with handlebar
injuries. Owing to the lack of significant blood loss, patients may again be relatively
asymptomatic. Since the small bowel contains relatively little air, up to 85% of cases will have a
normal upright abdominal film (i.e., no free air) and 50% will have no signs of peritonitis on
examination. Therefore, physicians must maintain a high suspicion for injury in cases of pediatric
handlebar trauma.
14. A 65-year-old female pedestrian presents after being struck by a car moving at about 20
mph. She has an obvious, open, deformed leg fracture and was unable to walk at the scene. Her
prehospital vital signs are P 105, 85/55, and 100% RA. She is awake and alert and in significant
pain. You confirm that her airway, breathing, and pulses are intact. On visual inspection, she has
an open tibial shaft fracture and has decreased sensation distal to the fracture. Which of the
following is the most important next step in management?
D. Tetanus immunization
Answer E. Although the patient has an obvious leg fracture, her hypotension suggests that there
may be another, more significant injury present. In general, only a few sites of injury in adults
can cause enough hemorrhage to result in hypotension. These include thorax, intraperitoneum,
retroperitoneum, pelvis, and bilateral femurs. Tibial fractures by themselves do not usually
result in hypotension due to hemorrhage. Appropriate adjuncts to the primary survey include
radiographs of the chest and pelvis and focused assessment of sonography in trauma (FAST)
scan. These tests should be done before any management of the open tibial fracture. Specific
management of the open tibial fracture can occur only if there are extra care providers to
perform this concomitantly with the adjuncts to the primary survey. Tetanus immunization can
be carried out at any time within 72 hours of the injury and need not be an emergency
procedure.
pg. 654
15. Which of the following is true regarding falls from buildings?
B. Feet-first falls cause retroperitoneal bleeding more often than intraperitoneal bleeding.
D. Calcaneal fracture is the most common cause of death from all falls.
Answer B. Falls from buildings are an important mechanism of multisystem blunt trauma.
Feet-first falls tend to cause lower extremity, spinous, and pelvic injuries, the last of which
results in retroperitoneal bleeding. Mortality is related to the height of the fall—half the number
of patients who fall from four stories die. Falls onto the back commonly cause spinous and
retroperitoneal injuries, including serious trauma to the kidneys. Calcaneal fracture often occurs
in patients with feetfirst falls but rarely by itself causes mortality. Instead, it is a sign of other
potential injury, including spinous or pelvic fracture. Falls onto prone position can cause serious
abdominal and thoracic injuries and may easily result in death.
16. A 44-year-old male presents after a motor vehicle crash (MVC) with scrotal pain. Blood was
noticed initially at the urethral meatus, but a 16-Fr Foley catheter was mistakenly placed with
return of yellow urine. Which of the following is the most appropriate next step in
management?
Answer E. Urethral injury occurs most commonly in men due to the length of the male urethra.
The injury is suspected on physical examination by a variety of findings—blood at the urethral
meatus, scrotal/penile hematoma, severe pelvic fracture, or high-riding or mobile prostate.
Evaluation of suspected urethral injury generally involves retrograde urethrogram to assess for
extravasation of contrast. However, once a Foley catheter is successfully placed and the bladder
is drained of urine, it should not be removed until urologic consultation is obtained. The key to
managing urethral injuries is to prevent urine from leaking out of the bladder into the
retroperitoneum, pelvis, or abdomen. If the Foley has already successfully decompressed the
bladder, the urethral injury may be evaluated later in the operating room (OR) or with a
pg. 655
modified bedside urethrogram around the Foley, if needed. Removal of the Foley can convert a
partial urethral disruption into a complete one and should be avoided.
E. Digital examination should be performed to determine whether there is cervical dilation and
the patient is in active labor.
18. A 52-year-old male is brought to the ED after a high-speed motor vehicle crash. His primary
survey is as follows:
Airway: Intact
pg. 656
Circulation: Normal pulses throughout
His vital signs are 99.0, 90, 22, 155/85, 92% RA. Chest x-ray is shown below. Which of the
following is the next best step in management?
E. Supplemental oxygen
Answer E. The patient has a large left upper lobe consolidation, which in the clinical setting of
blunt trauma is likely pulmonary contusion. No definite pneumothorax is seen on either side.
Hemothorax is on the differential diagnosis but would either appear as diffuse haziness of the
lung fields on a supine radiograph or inferior layering (like a pleural effusion) on an upright
radiograph. Pulmonary contusions can cause inadequate oxygenation and lead to pneumonia.
Treatment is largely supportive, with optimal hydration and supplemental oxygenation. Needle
thoracostomy should be reserved for cases of hemodynamically unstable traumatic
pneumothorax.
pg. 657
19. A 27-year-old male is brought to the ED after an accident riding an all terrain vehicle (ATV).
He was riding in an open field with a helmet and body armor when he ran into a wire fence that
struck him in the neck, knocked him off the bike and knocked him out. He now complains of a
mild headache and neck and back soreness but is otherwise without complaints. His
examination reveals left-sided ptosis and anisocoria with a smaller left pupil. Which of the
following is most likely to reveal significant injury?
B. CT angiography
C. Laryngoscopy
E. Chest x-ray
Answer B. This patient has Horner syndrome caused by disruption to the sympathetic fibers that
encircle the carotid artery. Division of the sympathetics results in ptosis, miosis, and anhidrosis
on the side of the injury and can occur after either blunt or penetrating trauma. Though the
presence of Horner syndrome is not a life-threatening emergency, it may represent a
life-threatening vascular emergency due to the proximity of the sympathetic chain to the carotid
artery. Therefore, all patients with Horner syndrome should have a definitive evaluation of the
carotid artery to exclude intimal injury (e.g., carotid angiography or helical CT angiography,
which has largely replaced dedicated angiography in most centers). Interestingly, delayed
presentation of neurologic deficits is typical of vascular injuries to the neck due to blunt trauma.
In the absence of Horner syndrome, most patients experience stroke symptoms between 1 and
24 hours after injury due to carotid or vertebral artery dissection or thrombosis. Vascular injury
should be suspected in all patients with neurologic findings that are incongruent with head CT
findings.
20. An 82-year-old male with a history of osteoporosis presents for evaluation of back pain after
a fall. The patient reports he slipped in his kitchen and fell to his buttocks. He presents for
evaluation of lumbar back pain. His x-ray is shown below . Which of the following is the best
next step in management?
D. TLSO bracing
pg. 658
Answer E. The x-ray demonstrates a significant vertebral compression deformity. Fortunately,
vertebral compression fractures are stable fractures with few neurologic complications. While
retropulsed fragments can impact the spinal cord or cause cauda equine syndrome, this patient
has no neurologic symptoms that would mandate an MRI. CT scans are sometimes performed to
determine stability in patients with wedge fractures but this patient’s injury is clearly a
compression deformity which is a stable injury. A thoracolumbosacral orthosis (TLSO) is a very
large brace intended to immobilize the spine typically after spinal surgery. While smaller lumbar
braces can be used (e.g. “chair back” braces), they can’t be used while sitting, limit mobility
which can lead to other problems, and aren’t clearly efficacious. Kyphoplasty, a form of
vertebral augmentation meant to reduce the fracture is reserved for patients with persistent
pain who fail conservative management. Most patients with these injuries remain able to
ambulate and can be discharged from the hospital with pain control and follow-up.
21. A 22-year-old male is dropped off by his friends at the ED who stated he was shot with a gun
in the middle of his abdomen. The most likely structure to be injured is the:
A. Liver
B. Small intestine
pg. 659
C. Spleen
D. Colon
E. Diaphragm
Answer B. In order of decreasing frequency, the small intestine, liver, and colon are the most
commonly injured abdominal organs in the setting of penetrating abdominal trauma.
22. A 44-year-old female presents after a motor vehicle collision with a complaint of neck pain.
Neurologic examination reveals that bilateral upper extremity strength is 1/5 and bilateral lower
extremity strength is 4/5. Which of the following is the most likely pathophysiologic process?
C. Brown–Séquard syndrome
Answer B. The classic neurologic deficit seen in central cord syndrome is upper extremity
weakness greater than lower extremity weakness. This is because of the cervical motor axons
being closer to the midline than the lumbar motor axons. Large central cord injuries can initially
be clinically indistinguishable from complete cord syndromes. Anterior cord syndrome results in
deficits of bilateral motor function and pain/temperature sensation with sparing of
vibration/position sensation. Brown–Séquard syndrome, or cord hemisection, results in deficits
in ipsilateral motor function and vibration/position sensation and contralateral
pain/temperature sensation. Cauda equina syndrome, usually due to disk herniation,
preferentially affects the lower extremities and bowel/bladder function. Complete cord injury
affects all neurologic functions below the level of injury.
23. A 35-year-old male presents with severe head trauma. Funduscopic examination
demonstrates papilledema and increased intracranial pressure (ICP) and impending herniation is
suspected. Mannitol is given and a decision is made to hyperventilate the patient as a last-ditch
effort while waiting for neurosurgical evaluation. Which of the following is an appropriate target
level of PCO2 for therapeutic hyperventilation?
A. 22 mm Hg
B. 27 mm Hg
pg. 660
C. 32 mm Hg
D. 37 mm Hg
E. 42 mm Hg
Answer C. In the past, patients with signs of intracranial hypertension after head trauma were
hyperventilated to PCO2 <25 mm Hg in order to cause reflex cerebral vasoconstriction and
reduced cerebral blood volume. However, more recent research suggests that reducing the
PCO2 levels to below 30 mm Hg may cause cerebral hypoxia in many areas of the brain,
potentially worsening the chances for neurologic recovery. The appropriate PCO2 range for
hyperventilation appears to be between 30 and 35 mm Hg—this will result in modest cerebral
vasoconstriction without hypoxia. An arterial catheter is useful for measuring rapid, serial blood
gases to maintain the PCO2 in this range.
D. Hangman fracture
E. Jefferson fracture
Answer C. Spinal injuries are classified by the mechanism of injury and mechanical stability in
reference to their potential to cause neurologic damage. Unstable injuries are considered likely
to cause spinal cord damage and usually require surgical stabilization. Stable spinal injuries are
more common than unstable ones and are easier to remember because there are only a few
types—wedge fracture, spinous and transverse process fractures, unilateral facet dislocation,
and vertebral burst fracture (with the exception of a Jefferson fracture, which is a burst fracture
of C1). All other spinal injuries are considered potentially or definitely unstable. A flexion
teardrop fracture occurs when the anterior portion of the vertebral body shears off from a
flexion force, causing ligamentous disruption. Bilateral facet dislocation is an extremely unstable
injury resulting from significant flexion, causing the superior facets of the inferior vertebra to
lose their articulation with the inferior facets of the vertebra immediately superior to it. Spinal
cord injury is common due to the significant displacement of the vertebrae. Solitary transverse
process fracture is usually of no clinical significance as it is far removed from the articular
surfaces of the vertebrae and spinal cord. A Hangman fracture occurs from extreme extensor
forces, causing bilateral fractures of the pedicles of C2 and dislocation of C2 from C1. Unlike the
past where hanging gave this fracture its name, the common mechanism now is motor vehicle
collision. The Jefferson fracture results from vertical compression forces causing the anterior
pg. 661
and posterior portions of the ring of C1 to break and putting the spinal cord at extreme risk for
severe injury.
25. A 44-year-old previously healthy male presents for evaluation of a headache after falling
backward 8 feet from a ladder. The patient’s neurologic examination is unrevealing and his
Glasgow Coma Scale (GCS) is 15. CT reveals an occipital epidural hemorrhage. Which of the
following is true?
B. The patient should undergo an emergent suboccipital craniectomy to evacuate the blood
C. Hypertonic saline infusions help to reduce the increased intracranial pressure associated with
these injuries
Answer A. Traumatic epidural hemorrhage in the posterior fossa is less common than other
locations. However, even patients who are initially well appearing with a normal examination
can rapidly deteriorate due to sudden brainstem compression. Thus, such patients are best
observed in a highly monitored setting such as an ICU. Suboccipital craniectomy is the
appropriate procedure to evacuate occipital epidural hematomas but the procedure isn’t usually
necessary in patients without neurologic complaints or findings. Hypertonic saline is not an
accepted adjunct to treatment of these patients. While visual field cuts may occur after an
occipital lobe injury, complete cortical blindness is uncommon because it requires an injury
affecting the bilateral occipital lobes.
26. In a patient with refractory hypotension after blunt abdominal trauma, the pictured finding
on FAST examination, as shown in Figure below:
pg. 662
Answer A. The image demonstrates hemoperitoneum with blood in Morison pouch (right upper
quadrant). In an unstable patient, this is an indication for immediate laparotomy. In
unmistakably stable patients, a CT scan of the abdomen should be performed to better
delineate the injury and the potential need for surgery. The sensitivity of FAST for the detection
of 100 to 500 mL of blood is as high as 95%. Therefore, while at least 500 mL of blood is present,
it is not possible to state unequivocally that 2.5 L is present. Furthermore, 2.5 L of blood loss
would place the average adult male in class IV hemorrhagic shock which typically presents with
a systolic blood pressure <70 mm Hg and a heart rate >140. After confirming the presence of
hemoperitoneum by ultrasonography, CT scanning should only be performed in undoubtedly
stable patients if a clear reason for laparotomy does not already exist.
27. Diagnosis of which of the following findings is the primary utility of the focused assessment
with sonography in trauma (FAST) scan?
A. Pelvic fracture
B. Renal injury
C. Diaphragmatic rupture
D. Hemoperitoneum
E. Aortic injury
Answer D. The FAST scan consists of a series of ultrasonographic images designed to assess for
the presence of hemoperitoneum. Ultrasonography of the right upper quadrant, left upper
quadrant, and suprapubic regions aids in this determination. A fourth view in the subxiphoid
region focuses on the pericardium to assess for effusion or tamponade. The FAST scan is ideally
performed in the patient with blunt trauma just after the primary survey is complete and in
conjunction with plain radiographs of the chest and pelvis. Pelvic fracture is assessed by the
pg. 663
initial radiograph of the pelvis. The kidneys and abdominal aorta are retroperitoneal structures
which cannot be diagnosed with the use of FAST scan. Diaphragmatic injury is notoriously
difficult to diagnose with noninvasive studies (including CT, FAST, and diagnostic peritoneal
lavage [DPL]) and may require thoracoscopy or laparoscopy for definitive diagnosis.
28. A 23-year-old female presents for right ear pain and drainage after being struck on the side
of the head with a basketball. Her distal external auditory canal and tympanic membrane are
shown in Figure below. Which of the following is the most appropriate next step in
management?
E. No specific therapy
Answer E. The figure demonstrates a TM perforation in the setting of head trauma. Treatment
of traumatic TM perforations in a dry environment is purely supportive with close ENT
follow-up. Perforations in a wet environment require prophylaxis with antipseudomonal
antibiotics. Perforations associated with preceding symptoms of otitis media also require
standard antibiotic therapy with an aminopenicillin. All patients should be instructed to keep the
ear canal dry. Healing of TM perforations occurs over several weeks to months.
pg. 664
29. A 47-year-old female on rivaroxaban for a DVT is brought to the emergency department with
a scalp abrasion after falling on an icy sidewalk. Head CT is negative. Which is the best next
step?
Answer C. Anticoagulated patients with negative head CTs after head trauma rarely develop
clinically relevant delayed bleeding. There is no evidence that repeat head CT is helpful or
required. Furthermore, in the absence of bleeding, no reversal agents are needed. Admission for
observation or ED observation without repeat imaging is a reasonable strategy. However, it is
reasonable to discharge patients home with a reliable family member who can monitor them for
signs of clinical decompensation. Better guidelines about how to manage these patients will
slowly be developed as their use proliferates. Data thus far appear to demonstrate lower or
similar risks of bleeding complications compared to warfarin.
30. A 15-year-old male is brought to the ED after being submerged in a lake for “a minute or
two.” He had been water skiing when he lost control and “wiped out” and then was lying face
down in the water without moving. He was not breathing when his friends pulled him out of the
water, but he regained spontaneous respirations after they performed cardiopulmonary
resuscitation. In the ED, he is awake, but somnolent, breathing spontaneously with a pulse
oximetry of 95% on room air and making purposeful movements and following commands. The
next most important step in management is:
A. IV antibiotics
C. IV dexamethasone
E. ED thoracotomy
Answer D. Trauma in the setting of submersion injuries is usually because of motor vehicle
accidents (in which the vehicle crashes into water) or accidents involving diving, boating, or falls
from a height into water. Although antibiotics are of use in patients who were submerged in
grossly contaminated fluid (e.g., sewage), they have no role in routine fresh or saltwater
pg. 665
submersion. Corticosteroids also have no role in drowning patients unless patients have a
history of reactive airway disease and have evidence of bronchospasm on physical examination.
Since the patient is breathing spontaneously and appears to be able to protect his airway,
intubation is not indicated. However, because this patient was involved in a high-speed crash,
trauma-related injury should be the next most important issue after ensuring an adequate
airway, breathing, and circulation.
31. Which of the following deficits below the level of injury is consistent with an anterior cord
syndrome?
E. Loss of pain and temperature sensation and motor function and bladder incontinence.
Answer E. In trauma patients, anterior cord syndrome most commonly occurs in hyperflexion
injuries in which herniated vertebral discs or vertebral body fragments compress the anterior
aspect of the spinal cord or the anterior spinal artery. The anterior spinal artery provides blood
supply to the anterior two-thirds of the spinal cord. The primary structures affected in the spinal
cord are the spinothalamic tract, which is responsible for the transmission of pain and
temperature sensory input, and the corticospinal tract, which carries descending voluntary
motor signals. In addition, anterior cord syndrome affects the descending autonomic tracts for
bladder control, resulting in incontinence or variable degrees of bladder dysfunction. The dorsal
or posterior columns, which are responsible for proprioception and vibration sensation, are
unaffected so those functions are preserved.
32. A 38-year-old male presents with right-sided thoracic pain after tripping and falling against a
counter. His vital signs are: 98.6, 80, 16, 122/72, and 100% on RA. He has tenderness along the
mid axillary line at the level of the nipple. He has no abdominal pain or tenderness. PA and
lateral chest x-rays are negative for pneumothorax or hemothorax. He is very anxious about a
rib injury. What is the most important next step in management?
pg. 666
E. Provide pain control, education, and reassurance
Answer E. The patient may have a clinical rib fracture, which will require pain control and
possibly incentive spirometry. However, most rib fractures do not need to be diagnosed
radiographically, as management will not change irrespective of the rib x-ray result. Educating
the patient that the rib may be fractured and that the important function of the ribs was fulfilled
(i.e., protection of the lungs and pleura) is the best management. CT chest without contrast
would be indicated if the patient were suspected of having an occult pneumothorax or
hemothorax. CT chest with IV contrast is not typically indicated with this trauma mechanism and
should be reserved for patients suspected of aortic injury. Hospital security action should be
reserved for patients who represent physical danger to themselves or others.
33. A 11-year-old male presents after a motor vehicle collision. Primary survey is intact and vital
signs are normal. Genitourinary examination is shown in Figure below. Which of the following is
the most appropriate next step in management?
B. Retrograde urethrogram
C. Retrograde cystogram
D. Urinalysis
Answer B. The patient has frank blood at the urethral meatus, which is a sign of likely urethral
injury. This precludes placement of a Foley catheter by anyone but a urologist. Initially, a
pg. 667
retrograde urethrogram is performed to evaluate for urethral injury. If this is normal, a Foley
catheter may be placed and retrograde cystogram performed to evaluate for bladder rupture.
Urinalysis in a trauma patient with grossly bloody urine is not helpful. Aspiration of the corpora
cavernosa is indicated only in patients with priapism.
34. When evaluating and managing patients with unstable pelvic fractures, pelvic binders:
B. Should be applied over the bilateral greater trochanters extending above the umbilicus
Answer D. Pelvic binders are a fast, easy, effective means to limit pelvic bleeding in patients with
unstable pelvic fractures. Traditionally, a bedsheet wrapped around the pelvis has been used to
compress the pelvis in an effort to limit bleeding. However, there are a number of commercial
devices available as well. Binding the pelvis has been thought to reduce pelvic volume, which
lays the groundwork for tamponade of ongoing venous bleeding. However, their use achieves
relatively modest pelvic volume reduction and may disrupt the retroperitoneum which
diminishes the tamponade effect. Thus, it is now thought that binders decrease hemostasis
primarily by decreasing pelvic motion of unstable bony fragments by a splinting mechanism.
Regardless of the exact etiology, binders are associated with decreased blood transfusions and
shorter intensive care unit (ICU) stays and are at least as effective as external fixators in
controlling venous hemorrhage. Neither modality is effective in controlling arterial hemorrhage.
Binders are placed over the greater trochanters to the iliac crests, such that the superior margin
of the binder should lie inferior to the umbilicus. The use of military antishock trousers prevents
vascular access and is associated with complications without clear benefit, so their use is
discouraged.
35. A 12-year-old female is brought in by EMS after a gunshot wound. The patient was injured
incidentally when her older brother was “showing off” his gun to his friends. She presents with
two soft-tissue wounds to her right leg, with significant swelling about the right knee. An x-ray
demonstrates a small amount of shrapnel, and a small fracture to the distal cortex of her femur.
Clinically, there is expanding, nonpulsatile, swelling in the area of the knee. Which of the
following is the next best step?
C. Take her to the OR for open reduction and internal fixation of her femur
pg. 668
D. Ultrasound of the leg
Answer A. The presence of a large, expanding hematoma is a classic “hard sign” of vascular
injury. When a hard sign of vascular injury is present, ancillary studies such as CT angiography,
angiography, or ultrasound are unnecessary and delay definitive treatment. Other hard signs of
injury include obvious pulsatile bleeding, a pulseless extremity, a bruit or thrill over the vessel,
and shock with a presumed vascular injury and no alternative explanation. In the setting of an
injured leg with “soft signs” of injury, such as a small, nonpulsatile hematoma, an associated
peripheral nerve deficit (due to the proximity of nerves to vascular structures in the
neurovascular bundle), or a diminished distal pulse or decreased ankle–brachial index (ABI),
imaging should be performed. In such cases, CT angiography is rapidly becoming the gold
standard, although ultrasound may be useful in select cases of pregnant women and children to
avoid radiation.
36. Which of the following describes the appropriate means of examining the pelvis in a trauma
patient?
B. Superior and posterior force applied to the bilateral anterior–superior iliac spine (ASIS) to
“rock” the pelvis back and forth
D. Palpate the bilateral ASIS and “open” the pelvis like a book
Answer C. Gentle, lateral compression of the iliac wings is the recommended physical
examination approach to assess for pelvic instability. Rocking the pelvis by applying superior and
posterior force on the bilateral ASIS is no longer recommended since doing so may worsen
hemorrhage from an unstable fracture, and the procedure causes unnecessary pain. While
pelvic fracture patterns on x-ray do not adequately predict hemodynamic stability, physical
examination maneuvers may perform even worse, and such maneuvers may cause unnecessary
pain or provoke further bleeding, as above. Thus, the role of “aggressive” pelvic physical
examination maneuvers, particularly in hemodynamically unstable patients, is increasingly
limited. In stable patients without overt evidence of severe trauma, physical examination is a
more useful adjunct to identify those patients who need further investigation with radiologic
studies.
pg. 669
37. An 8-year-old male restrained back-seat passenger is brought to the ED after a car accident.
He was wearing a lap belt only when the car in which he was riding was struck in a front-end
collision with another vehicle at high speed. The restraint puts him at increased risk for which of
the following injuries?
A. Jejunal perforation
D. Pancreas hematoma
Answer E. Patients wearing isolated lap-belt restraints may present with a “seatbelt syndrome,”
which classically consists of a transverse abdominal wall contusion, a chance fracture, and
visceral trauma (e.g., hollow viscus rupture, of which jejunal rupture is most common, or solid
organ injury, such as liver injury). Injury to the abdominal aorta has also been described but is
rare. A chance fracture is an anterior compression fracture of the vertebral body with an
associated transverse fracture of the posterior portion of the vertebral body as well as
ligamentous disruption and possible vertebral body subluxation. Injury to the spinal cord is
uncommon but can occur. Chance fractures occur when the spine is suddenly and forcefully
flexed around the seatbelt, which serves as a fulcrum. Like the other injuries listed, chance
fractures are most common among children in whom the seatbelt is improperly worn across the
abdomen instead of low across the hips. These types of injuries are most common in 5- to 9-year
old, who are often too large to fit comfortably into a booster seat, yet too small to be
adequately restrained by adult-sized shoulder harnesses.
38. A 26-year-old previously healthy male construction worker is brought in by EMS after he fell
approximately 20 feet while painting. He did not lose consciousness, but he complains of severe
back and pelvic pain. His initial vital signs are HR 136, BP 86/46, RR 22, SaO2 100% on a
nonrebreather mask. Physical examination reveals multiple ecchymoses and abrasions over the
low back, buttocks, and pelvis with an unstable pelvis on gentle lateral compression. Focused
assessment with sonography for trauma (FAST) examination is negative. Pelvic x-ray reveals
displaced fractures of the left anterior and posterior hemipelvis. Which of the following is the
next best step?
C. External fixation
D. Endotracheal intubation
pg. 670
Answer E. This patient has a mechanism of injury, physical examination, and x-ray consistent
with an unstable pelvic fracture. While at least 50% of patients with unstable pelvic fractures
experience major hemorrhage outside the pelvis, this patient has no evidence of intraperitoneal
hemorrhage on FAST examination. Such patients should have a pelvic binder immediately
applied and be taken to the angiography suite for control of possible arterial bleeding via
embolization (binders are intended as adjuncts to control venous, not arterial, hemorrhage). If
angiography is negative and the patient remains hemodynamically unstable, FAST examination
should be repeated or the patient should simply be taken to the OR for a diagnostic laparoscopy
or laparotomy.
39. Which of the following is the most commonly injured abdominal organ in pediatric blunt
trauma?
A. Liver
B. Spleen
C. Kidney
D. Small intestine
E. Large intestine
Answer B. The spleen and the liver (in that order) are the most commonly injured abdominal
organs in children with blunt trauma. Liver lacerations tend to have higher mortality than splenic
lacerations. Unlike the past, splenic lacerations are currently nonoperatively managed as much
as possible, because of the deleterious immunologic effects of splenectomy. Renal injury is also
common, given its proportionally larger size in children relative to adults. Bowel injury in blunt
trauma is rare.
40. A 44-year-old male presents with chest pain after a motor vehicle collision. He receives an
upright chest x-ray, which is shown in Figure below. Which of the following is the most likely
diagnosis?
A. Pneumothorax
B. Hemothorax
C. Pulmonary contusion
E. Cardiac contusion
pg. 671
Answer C. The patient has air-space consolidation in the right mid-lung field in the setting of
trauma, which is consistent with a pulmonary contusion. Management is directed at adequate
oxygenation and ventilation and prevention of secondary complications such as acute
respiratory distress syndrome (ARDS) and pneumonia. There is no obvious pneumothorax
present, although an occult pneumothorax may be picked up if CT chest is performed. On an
upright chest x-ray, hemothorax would appear as a pleural effusion around the lower lung
segments. Chest radiography is not specific for the diagnosis of traumatic aortic rupture, which
requires CT angiography for definitive diagnosis. Cardiac contusion is an older term to describe
blunt cardiac injury, which is diagnosed by a combination of EKG, echocardiography, and
sometimes cardiac markers.
41. Which of the following is an element of the NEXUS criteria for cervical spine imaging?
B. Absence of smoking
Answer C. The NEXUS criteria include absence of the following: focal neurologic deficit, posterior
midline neck tenderness, intoxication, altered mental status, distracting injury. The NEXUS
pg. 672
criteria have a sensitivity between 90% and 99% for ruling out clinically significant fractures. The
Canadian C-spine criteria, which focus on excluding high-risk mechanisms and instituting an
assessment on range of motion, may be more sensitive and specific than the NEXUS criteria.
42. A 42-year-old female is brought to the ED after she rear-ended a tractor trailer while driving
a small sedan which partially submarined under the tractor trailer’s rear bumper. Airbags were
deployed, and she states she was “out for a second.” She complains of headache, neck pain,
back pain, as well as transient numbness over her “entire right side” that has since resolved. Her
physical examination reveals multiple contusions over her face and scalp, mild left-sided ptosis,
as well as abrasions over her extremities and a seatbelt sign over her left neck, chest, and
abdomen. Subsequent CT scans of her head, cervical spine, chest, abdomen, and pelvis are
unrevealing. Which of the following is the best next step?
E. Neurology consultation
Answer B. This patient presents with findings concerning traumatic carotid artery dissection
leading to a partial ipsilateral Horner syndrome with concomitant contralateral hypoesthesia.
Patients experiencing sudden, compressive forces to the neck may develop traumatic carotid
artery dissection. The sympathetic fibers that innervate the face and eye are also damaged by
direct compression since they travel within the wall of the artery, resulting in an ipsilateral
Horner syndrome (miosis, ptosis, anhidrosis, and enophthalmos). Horner syndrome may be the
only indication of underlying carotid artery dissection. This patient also experienced transient
contralateral hypoesthesia indicating ischemia of the ipsilateral brain in the middle cerebral
artery distribution, likely because of internal carotid artery dissection. Thus, the patient should
receive either CT angiography or magnetic resonance angiography (MRA) to further elucidate
the vascular anatomy. MRI of the brain may demonstrate an acute infarct in the middle cerebral
artery distribution, but without an angiographic study, carotid dissection would be missed.
43. A 45-year-old female with a history of arthritis presents with weakness in her arms and
hands after being involved in a motor vehicle crash in which her head hit the windshield. She is
able to walk, but has 3/5 strength in her bilateral upper extremities. Which of the following is
the most likely diagnosis?
pg. 673
C. Central cord syndrome
D. Brown–Sequard syndrome
E. Spinal shock
Answer C. Central cord syndrome is caused by a hyperextension mechanism of the cervical spine
where the ligamentum flavum is pushed into the spinal cord, causing central cord compression.
Because of the distribution of motor fibers, upper extremity weakness is more profound than
lower extremity weakness. Central cord syndrome is the most common of all partial cord
injuries. Complete cord transection results in total loss of motor and sensory function below the
level of injury. Anterior cord syndrome results from hyperflexion injury and causes paralysis and
decreased pain and temperature sensation below the level of illness but with preserved
vibration and position sense. The dorsal columns are spared in anterior cord syndrome.
Brown–Sequard syndrome, or hemisection of the cord, is usually due to penetrating trauma and
causes ipsilateral paralysis and loss of position and vibration sensation and contralateral loss of
pain and temperature sensation. Spinal shock is a temporary loss of all motor and sensory
function below the level of the lesion lasting from several hours to several days. Onset of
recovery is usually marked by return of the bulbocavernosus reflex.
44. A 76-year-old female presents after a fall from standing height onto a countertop. She
landed on the right side of her ribcage and complains of pain in that area and difficulty taking a
deep breath. Vital signs are normal and physical examination is remarkable only for point
tenderness in the right lateral fourth and fifth ribs. A chest x-ray done in the ED is normal. Which
of the following is the most appropriate next step in management?
B. Discharge home
Answer D. Elderly patients with even mild blunt chest trauma are at high risk for pulmonary
complications, including atelectasis, delayed contusion, infection, and acute respiratory distress
syndrome (ARDS). Aggressive pain management and pulmonary toilet is mandatory for these
patients to maintain proper lung expansion in order to prevent the complications mentioned
earlier. Most patients are successfully managed on an outpatient basis—admission is indicated if
the patient has significant chest wall or pulmonary injury revealed on chest x-ray or chest CT, or
has hypoxia, or other severe symptoms. Prophylactic antibiotics are not recommended in the
absence of clinical and radiographic findings of infection.
pg. 674
45. Which of the following is true regarding renal injury in trauma?
B. In blunt trauma, microscopic hematuria alone is rarely associated with renal injury.
Answer B. Significant renal injury in blunt trauma rarely occurs in the absence of other organ
injury. Gross hematuria is the standard indication for further evaluation of the urologic tract,
and CT scan with IV contrast is the best imaging tool to evaluate renal injuries. In stable patients
with microscopic hematuria due to blunt trauma, significant renal injury is uncommon.
Penetrating trauma to the back or abdomen may cause renal injury in the absence of either
gross or microscopic hematuria and should be evaluated with CT imaging. The vast majority of
renal injuries do not require operative management; exceptions include large renal lacerations
and major vascular injuries. The kidneys are among the most commonly injured abdominal
organs in blunt abdominal trauma in children.
46. A 44-year-old male is struck on the head with a baseball bat. A CT scan of the brain is shown
in Figure below. Which of the following is the most likely diagnosis?
A. Epidural hematoma
B. Subdural hematoma
C. Subarachnoid hemorrhage
D. Cerebral contusion
pg. 675
Answer C. The CT scan shows diffuse, bilateral bright signal consistent with acute hemorrhage in
the subarachnoid space. Epidural and subdural hematomas are usually focal, unilateral, and
often cause midline shift. Cerebral contusion appears as blood in the parenchyma rather than
the cisterns. DAI usually does not appear on an emergent brain CT and requires clinical
evaluation and MRI for diagnosis.
47. A 34-year-old male presents after a high-speed motor vehicle crash. Chest x-ray is performed
and shown in Figure below. Which of the following is the most likely diagnosis?
A. Pneumothorax
C. Duodenal hematoma
D. Diaphragmatic rupture
E. Hemothorax
pg. 676
Answer D. The chest x-ray demonstrates presence of the gastric bubble inside the thoracic
cavity, indicating a large diaphragmatic rupture from blunt trauma. The left diaphragm is much
more likely than the right to rupture due to protection on the right from the liver. Management
involves nasogastric decompression and surgical repair. Patients with penetrating
thoracoabdominal trauma may have delayed abdominal herniation into the thorax as far out as
decades after the initial injury. Right-sided diaphragmatic injuries are less likely to have
abdominal herniation into the thorax, also because of the presence of the liver. Diaphragmatic
injuries may be very subtle and diagnosed only on direct visualization with laparoscopy or
thoracoscopy. Diagnostic peritoneal lavage (DPL), focused assessment of sonography in trauma
(FAST), and CT scan all lack sufficient sensitivity for ruling out the diagnosis.
48. A 35-year-old female presents after a high-speed MVC. She was unrestrained and there was
considerable damage to the vehicle. She complains of chest pain and right leg pain. Paramedics
report that the right ankle is visibly deformed. The patient is brought in on a backboard and in a
C-collar. Primary survey in the ED is intact. Secondary survey reveals normal pulses and
sensation in the deformed right ankle. Vital signs are T 99.0°F, P 90, RR 22, BP 144/92, POx 95%
RA. Which of the following is the most appropriate next step in management?
A. Chest x-ray
pg. 677
Answer A. As this patient is complaining of both chest pain and leg pain, radiographs of the
chest and pelvis are indicated after the primary survey to rule out important causes of
immediate death, including pneumothorax, hemothorax, and pelvic fracture. Ultrasonography in
the form of a FAST scan may also be performed to evaluate for significant intraperitoneal
hemorrhage. After these initial studies are performed (or in conjunction with them) the
secondary survey is conducted to identify injuries that may cause significant morbidity without
mortality. Obvious external injuries may distract the trauma leader from identifying the
immediate life threat. In this patient’s case, the broken right ankle, though impressive, is
unlikely to be the cause of death. If the initial chest x-ray is omitted, however, the potential
pneumothorax missed on physical examination may be lethal. This is the main reason for the
stepwise, algorithmic approach to trauma which is targeted to identify immediate life threats
first and other injuries later. Cervical spine radiographs may be left until after the secondary
survey, assuming appropriate spine precautions are used when moving the patient. CT scans
should not be initiated until the primary and secondary surveys are complete, except in special
circumstances of isolated, severe head injury.
49. Which of the following is the best rationale for using the primary survey in trauma?
C. Ease of documentation
Answer B. The primary survey in trauma is designed to identify and address the most immediate
life threats. This includes airway obstruction, pneumothorax or hemothorax, hemorrhage, and
brain and spinal cord abnormalities. Although the primary survey does tend to follow a
cranio-caudal direction, this is not the main reason for the elements of the primary survey.
Documentation should be tailored around the primary survey, not vice versa. The complete
physical examination is important during the secondary survey, as this will identify important,
but not emergent, issues. Medication and allergy history can be done either immediately after
the primary survey or concomitantly with the secondary survey.
50. A 24-year-old male is brought to the emergency room by emergency medical services (EMS)
after being assaulted in a robbery with an unknown blunt object. He does not know if he lost
consciousness, but he complains of a severe left-sided headache and left ear pain. His secondary
survey reveals left-sided hemotympanum. Which of the following is also most likely present?
A. Epidural hematoma
pg. 678
B. Cervical spine injury
C. Nasal fracture
Answer A. Hemotympanum is one of the signs of a basilar skull fracture. Of the five bones that
make up the base of the skull, the temporal bone is by far the most frequently injured, typically
due to a direct blow from an assault or motor vehicle collision (MVC). Due to the relative
weakness of the lateral aspect of the temporal bone, direct trauma may result in disruption of
the middle meningeal artery leading to an epidural hematoma. This patient’s questionable
history of temporary loss of consciousness with subsequent awakening is a classically described
feature of epidural hemorrhage known as the “lucid interval.” Patients subsequently become
increasingly symptomatic and experience a progressive decline in mental status over the next
several hours. This patient’s mechanism of injury and symptoms do not support a diagnosis of a
cervical spine injury or nasal fracture. Carotid artery dissection has been described after blunt
trauma to the temporal bone but is a rare occurrence. Finally, patients with temporal bone
trauma occasionally present with cranial nerve palsies. However, such palsies are not evident
until 2 to 3 days after the initial injury and are due to compression or contusion. They typically
resolve with time and may benefit from treatment with steroids. Acute facial nerve palsies are
rare as they are due to complete nerve transection. Such injuries do not respond to steroids and
typically lead to significant, permanent deficits.
51. A 28-year-old female at 29 weeks gestation is brought to the ED by ambulance after a minor
motor vehicle accident. She was the restrained driver of a car traveling approximately 20 mph
when she lost control on “black ice” and collided with a road sign. There was minimal damage to
the car according to EMS but they placed her in a cervical spine collar and on a backboard for
transport. She has no complaints in the ED except for discomfort related to the board and collar.
After finding that her primary and secondary survey is intact, she is asking to go home. Which of
the following is the next best step in management?
pg. 679
Answer D. All pregnant patients with a viable fetus (defined as 24 weeks of gestation and
beyond) should undergo continuous cardiotocographic monitoring for 4 hours before discharge
after possible abdominal and pelvic trauma. This duration has not been validated, and some
experts recommend shorter observation periods of only 2 hours. Even in the setting of “minor”
trauma, approximately 4% of pregnant patients will develop placental abruption. In the setting
of major trauma or in the presence of vaginal bleeding or any uterine contractions, patients
should be admitted for 24 hours of cardiotocographic monitoring. Cardiotocographic monitoring
is the most sensitive indicator of trauma-related fetal distress. Furthermore, as the fetus is more
sensitive than the mother to decreases in maternal blood pressure and blood flow, fetal distress
can be an early indicator of occult maternal shock. Most fetal losses in trauma occur due to
placental abruption and cardiotocographic monitoring is the most sensitive marker of fetal
distress due to abruption. In contrast, ultrasonography is notoriously insensitive, detecting only
50% of placental abruptions. Finally, in patients with a nonviable fetus, intermittent
documentation of fetal heart tones, as well as ultrasonography to assess fetal viability are
probably adequate.
52. A 23-year-old male presents with flaccid bilateral lower extremity paralysis 1 day after being
assaulted. An anterior lumbar spinal cord injury is identified. Which of the following is the most
appropriate therapy at this time?
B. Naloxone
C. Methylprednisolone
D. Vancomycin
E. Vasopressin
Answer A. Acute spinal cord injuries may benefit from high-dose corticosteroid therapy if given
within 8 hours of injury. However, their use and efficacy is constantly debated, and neither
administering nor withholding glucocorticoids can be considered the standard of care. If they
are administered, a longer delay than 8 hours may actually result in worsened outcomes.
Normal saline or lactated Ringer hydration should be given to all trauma patients. Naloxone may
be given to patients with altered mental status as empiric therapy for opioid intoxication but is
not generally used in patients with spinal cord injury. Vancomycin and other antibiotics are not
routinely indicated in trauma patients, except in clinically evident cases of post-traumatic
infection. Vasopressin may be used in patients with medical causes of shock who do not
respond to crystalloid or colloid administration but is unlikely to be useful in patients with
traumatic shock.
pg. 680
53. A 70-year-old female is brought in by EMS with right-sided rib pain and painful breathing
after a fall against a dresser. Her SaO2 is 96% on room air. A chest x-ray reveals a solitary right
7th rib fracture but no pneumothorax. Which of the following is true?
C. Elastic chest binders are helpful to reduce pain without impacting respiration
D. Elderly patients with rib fractures can be discharged home unless they have ≥6 rib fractures
Answer A. Given their decreased cardiopulmonary reserve, elderly patients more often suffer
complications from rib fractures. However, in the absence of hypoxia or a clinically significant
hemothorax, pneumothorax, or pulmonary contusion, even elderly patients with isolated rib
fractures may be safely discharged home. Pain control and incentive spirometry are important
components of treatment to avoid atelectasis and possible pneumonia. Chest x-rays detect less
than 50% of rib fractures. Dedicated rib series detect about 75% of fractures, while chest CT is
considered the gold standard. Still, in most adult patients only a chest x-ray is needed after
minor trauma since detecting injuries not seen on chest x-ray usually doesn’t change treatment
or outcome. In patients older than 65, however, rib fractures lead to higher rates of pulmonary
complications such as atelectasis and pneumonia so understanding the exact extent of thoracic
injury is more important. Older patients with ≥6 rib fractures have a 30% mortality rate and
should be admitted. In fact, the mortality rate really begins to climb once elderly patients have
suffered ≥3 fractures and most such patients should be admitted for observation to ensure they
don’t develop a pulmonary contusion and respiratory decompensation.
54. Which of the following is true regarding laboratory testing in patients with abdominal
trauma?
A. Liver enzymes are used to help distinguish minor contusions from high-grade lacerations.
B. Elevated serum amylase and lipase are always indicative of pancreatic injury.
C. Microscopic hematuria may indicate a need for abdominal CT scanning in pediatric blunt
trauma patients.
Answer C. Laboratory testing has a limited, but an important role in the evaluation of patients
with abdominal trauma. The hematocrit is primarily important because it establishes a baseline
pg. 681
value before resuscitation and redistribution. However, although the hematocrit is primarily
useful for following serial levels in the setting of solid organ injury, it is also valuable in any
patient who presents with a very low hematocrit in the setting of trauma because it most likely
indicates that significant blood loss has already occurred. Liver enzymes are not helpful and are
not used to distinguish between minor and major liver contusions or lacerations. Although often
elevated in the setting of pancreatic injury, serum amylase and lipase are nonspecific and poorly
sensitive. Therefore, normal levels do not exclude pancreatic injury and high levels may be due
to a host of other causes. Conflicting data exist regarding the utility of microscopic hematuria in
pediatric blunt trauma patients. Low-risk patients have <5 RBCs/HPF, while patients complaining
of abdominal pain or patients who provide other reasons to be suspicious of injury in the setting
of microscopic hematuria >5 RBCs/HPF should receive a CT.
55. Which of the following is the most common cause of multiple rib fractures in children?
B. Child abuse
C. Fall
D. Sports injury
E. Gunshot wound
Answer B. Nonintentional blunt thoracic injury in children rarely causes multiple rib fractures,
due to the compliance of the pediatric chest wall. For this reason, external injury is often absent,
although pulmonary and cardiac injuries may be prominent. Multiple rib fractures are usually
caused by child abuse, especially rib fractures that are observed radiographically to be in various
stages of healing. Falls and sports injuries rarely cause rib fractures in children. Gunshot wounds
are much more likely to cause thoracic organ damage than serious chest wall injuries.
56. When compared with adults, which of the following is true when evaluating infants and
children for cervical spine (c-spine) injuries?
A. There are no clinical decision rules available to assist with clinical c-spine clearance in children
B. C-spine injuries in infants tend to occur at higher c-spine levels than in adults
pg. 682
Answer B. Infants with c-spine injuries tend to be injured at C1-4 while older children, teenagers
and adults tend to have lower c-spine injuries. This occurs because the c-spine fulcrum lies at
C2-3 at birth and moves inferiorly as the child grows, reaching C5-6 by age 8. While the NEXUS
study did not include many young children with c-spine injuries in their study, there is a PECARN
c-spine rule which is similar as well as guidelines from the American Association for the Surgery
of Trauma (AAST) that allow clinical c-spine clearance without imaging. Furthermore, as the
incidence of significant c-spine trauma is very low in children, plain films are very useful to
evaluate the c-spine when patients present after a low-energy mechanism of injury with a low
clinical suspicion for serious injury on examination. CT is best used when the suspicion for injury
is at least moderate and modern CT scanners detect all serious injuries. MRI is reserved for
patients with neurologic findings and should not be used routinely in patients with persistent
neck pain after a negative c-spine CT.
57. A 22-year-old female presents to the ED after a domestic dispute with a boyfriend in which
she was stabbed in the neck just lateral to her thyroid cartilage. Which of the following is an
indication for mandatory operative exploration?
A. Palpable thrill
B. Subcutaneous emphysema
Answer C. This patient has an injury to zone II of the neck defined as the region of the neck
superior to the cricoid cartilage and inferior to the angle of the mandible. Trauma to zone II is
most common but also most amenable to repair due to relatively uncomplicated surgical
exposure and vascular control. Traditionally, all patients with violation of the platysma have
been taken to OR for exploration. In the modern era of rapidly improving radiologic testing,
operative repair may increasingly be deferred in favor of diagnostic testing such as helical CT
angiography (e.g., to evaluate vascular injury indicated by a bruit or thrill), laryngoscopy, or
esophagoscopy (e.g., to evaluate subcutaneous emphysema).
58. When evaluating a trauma patient with a focused assessment with sonography in trauma
(FAST) scan, which of the following views is ideally performed first?
A. Right flank
B. Left flank
C. Pericardial
pg. 683
D. Suprapubic
E. Thoracic
Answer C. Following the principle of trauma management that the most immediate life threats
should be addressed earliest, the pericardial view is the ideal first view during the FAST scan.
This will identify cardiac tamponade. The other views can be done after the pericardial view in
the following order: right flank, left flank, suprapubic, and thoracic.
Answer D. The skull is a small, fixed space which can’t accommodate much more than the brain
and its associated neurovascular structures. The majority of epidural hematomas occur in the
parietal and temporal regions. When blood accumulates in the epidural space, it compresses the
subjacent brain which pushes the brain inferiorly, compressing the oculomotor nerve against
the tentorium cerebelli producing pupillary dilation. Brainstem compression may also occur, but
is a later, even more serious problem. Traumatic mydriasis is common after a direct blow to the
eye, but there is no history suggesting direct eye trauma. A direct injury to the occipital lobe will
not produce anisocoria with a dilated pupil but could cause cortical blindness or visual field cuts.
60. Which of the following studies has the highest sensitivity for traumatic pericardial
tamponade?
C. Electrocardiogram (EKG)
pg. 684
D. Focused assessment with sonography in trauma (FAST) scan
Answer D. Pericardial tamponade usually results from a penetrating thoracic mechanism causing
cardiac or mediastinal injury and accumulation of blood into the pericardium. Pericardial
effusion and tamponade are readily seen on the subxiphoid and parasternal views of the FAST
scan. Chest x-ray may show a large, water-bottle heart if the effusion is large enough, but
tamponade physiology can occur with small, rapidly accumulating effusions which may be
invisible on plain radiography. The EKG finding of electrical alternans due to swinging of the
heart throughout the cardiac cycle is highly specific for pericardial effusion, but occurs in less
than one-third of cases. DPL is highly sensitive for intraperitoneal injury but has no utility in
screening for pericardial effusion.
61. Which of the following is true regarding patients with sternal fractures?
Answer C. Sternal fractures are usually caused by blunt thoracic trauma and passengers wearing
seat belts are at much higher risk than those who are unrestrained. The belt’s position across
the chest is thought to put extreme force on the sternum during rapid deceleration. Coexisting
mediastinal hematoma or myocardial contusion may occur in <10% of cases, but aortic injury
does not occur at appreciably higher rates. Mortality of isolated sternal fractures is <1%. Routine
AP views of the chest often miss sternal fractures, but tenderness of the sternum should prompt
either a lateral radiograph or CT scan, both of which demonstrate sternal fracture. Concomitant
rib fractures are the most common bony injuries associated with sternal fractures, but vertebral
fractures do occur at higher rates as well. Treatment involves evaluation for other thoracic
injuries and aggressive pain control.
62. A 22-year-old intoxicated male is brought in by EMS in hemorrhagic shock after a major
motor vehicle accident. Which of the following is true?
pg. 685
C. Tranexamic acid may substantially increase the patient’s risk of a life-threatening blood clot
63. What is the most reliable early indicator of shock in a pregnant patient after blunt abdominal
trauma?
A. Hypotension
B. Elevated lactate
C. Tachycardia
Answer B. Signs and symptoms of shock may be misleading in the pregnant trauma patient.
Acid– base abnormalities such as decreased bicarbonate or elevated lactate or base deficit are
the earliest indicators of maternal shock. Due to an increased intravascular volume, pregnant
patients will often not develop hypotension or tachycardia until significantly more blood volume
is lost than their nonpregnant peers. Thus, early crystalloid administration is indicated to sustain
the patient’s increased blood volume and to avoid fetal hypoperfusion. If blood loss occurs more
gradually, a pregnant woman can lose as much as 35% of her blood volume before the
development of blatantly abnormal vital signs. Additionally, because of the physiologic decrease
in peripheral vascular tone during pregnancy, pregnant patients in shock may remain warm and
dry even in the setting of shock. Finally, abdominal examination is less sensitive in pregnant
patients. As many as 50% of pregnant patients with hemoperitoneum will not have peritoneal
signs on physical examination. Therefore, ED physicians need to maintain a high suspicion for
injury and a low threshold for further diagnostic testing.
pg. 686
64. A 42-year-old male is brought into the ED after being thrown from a train during a
derailment. He is hypotensive but has no hemothorax, a normal mediastinum, and no pelvic
fracture. Focused assessment of sonography in trauma scanning reveals hemoperitoneum.
Which of the following is most likely injured?
A. Spleen
B. Kidneys
C. Liver
D. Diaphragm
E. Pancreas
Answer A. The spleen is the most common organ injured in blunt abdominal trauma. In order of
decreasing frequency, it is followed by the liver, kidney, small bowel, bladder, colon, diaphragm,
pancreas, and retroperitoneal duodenum.
65. A 35-year-old male presents after a motor vehicle collision with hypotension, tachycardia,
and altered mental status. Which of the following is the most appropriate blood product to
administer immediately?
Answer D. The patient presents in stage IV hemorrhagic shock, with tachycardia, hypotension,
and alteration of mental status. These patients will require both crystalloid and colloid fluid
resuscitation during the primary survey. Men in hemorrhagic shock can receive Rh positive or
negative blood, but preferably should receive un-crossmatched O positive blood so that O
negative stores can be reserve for women. Un-crossmatched blood is easily obtainable in the ED
and may be given immediately, without laboratory analysis. The Rh factor may be positive in the
donor unit when transfusing men and women beyond childbearing years, as there is no danger
of formation of antibodies which might occur in future pregnancies in Rh negative women of
childbearing age. Fresh frozen plasma is indicated to keep up with clotting factor losses when
patients have received four to five units of packed red blood cell (RBC) transfusions during the
course of a resuscitation.
pg. 687
66. An 8-year-old male presents with right ear pain after a fight. He tells you that he was
punched and kicked in the head and ear. On examination, you note the finding visible in the
image (see Fig. below). Which of the following is the best step in management?
Answer A. This patient has an auricular hematoma resulting from bleeding in the potential space
between the auricular cartilage and the perichondrium to which it is normally adherent. It is
important to incise and drain these hematomas because failure to do so may lead to necrosis of
the cartilage and subsequent cosmetic deformation of the ear such as “cauliflower ear.” This is
the same principle that guides treatment of nasal septal hematomas (cartilage necrosis). Though
aspiration is a reasonable strategy, recurrence is common and aspiration is sometimes
incomplete. Incision and drainage is more definitive and simple to perform. After drainage of
the hematoma, the ear is packed in a bulky pressure dressing to prevent reaccumulation of the
hematoma. Moist cotton is placed within the folds of ear and then covered with bulky dry
cotton. Several pieces of trimmed gauze are placed between the ear and the scalp. The ear is
then bandaged circumferentially against the supporting gauze. The patient is discharged with
follow-up arranged within 48 hours to assess for re-accumulation.
pg. 688
67. A 55-year-old female presents with a laceration on her arm after falling from her bicycle. She
does not know the last time she had a tetanus booster, but wants to know why she should have
one. Which of the following is true regarding tetanus?
68. Which of the following patients should be taken to the operating room (OR) for emergent
vascular exploration?
A. A 24-year-old male with a lower leg gunshot wound and an ankle–brachial index of 0.7
B. A 32-year-old female who suffered a knee dislocation in a motor vehicle collision (MVC) with
normal pedal pulses
C. A 22-year-old male stabbed in the arm with persistent pulsatile blood loss and an absent
distal pulse
D. A 36-year-old male with steady, slow oozing from a left upper leg gunshot wound and an
ankle– brachial index of 1.0
E. A 62-year-old male whose lower leg was inadvertently impaled on a pitchfork with a large
amount of bruising and soft-tissue swelling with a diminished dorsalis pedis pulse
pg. 689
Answer C. There are both “hard” signs and “soft” signs of vascular injury. Hard signs indicate
clear vascular injury that will require operative repair while soft signs reflect an increased risk of
vascular injury requiring further study. In reality, many surgeons prefer to evaluate stable
patients, even in the setting hard signs of vascular injury, with a CT angiogram before
proceeding to the OR. However, most examinations stick to the “rule” that patients with hard
signs of vascular injury in the setting of extremity trauma are taken emergently to the OR. Hard
signs include obvious limb ischemia, absent pulses, compartment syndrome, active hemorrhage,
a pulsatile or expanding hematoma, a bruit or thrill in the area of the injury. Soft signs include
venous oozing, a nonexpanding hematoma, diminished distal pulses, or an abnormal
ankle–brachial index (ABI) or brachial–brachial index (BBI). The ABI is determined by first
inflating a blood pressure cuff over the distal lower leg, and, using a Doppler probe,
documenting the pressure at which sound is first heard indicating return of flow in artery. This
should be done in both the dorsalis pedis and posterior tibial pulses and the highest value
should be used. The same test is then performed in both arms while measuring the first
pressure at which flow returns to the radial arteries. Both arms should be measured and the
higher of the two values should be used. The ratio of the lower extremity to the upper extremity
pressure is the ABI. Values <0.9 are abnormal and should prompt CT angiography (CTA) of the
affected extremity. In upper extremity injuries, the BBI is used with the same cutoff value of 0.9.
In this case, the values obtained from the injured arm are compared to the values from the
healthy arm. In the scenarios posed in the question, only the 22-year-old who suffered stab
wound has hard signs of vascular injury. All the other victims have either soft signs of injury that
warrant CTA.
Answer D. The KBT detects the presence and quantifies the volume of fetal RBCs in the maternal
circulation. Unfortunately, it is an insensitive test, requiring a minimum of 5 mL of fetal
hemorrhage for detection. Because as little as 0.01 to 0.03 mL of fetal blood may result in
maternal Rh sensitization, the KBT is not useful in most pregnant patients. Therefore, all
Rh-negative pregnant patients should be given Rh-immune globulin after significant abdominal
trauma. The dose is 50 μg in the first 12 weeks and 300 μg after 12 weeks of gestation.
However, in cases in which extensive MFH is suspected, the KBT may be useful because it can
identify patients in whom >30 mL of MFH has occurred. Such patients should receive a second
300-μg dose of Rh-immune globulin because each 300-μg dose is only enough to prevent
pg. 690
sensitization from 30 mL of fetal blood. The only patients in whom this occurs are patients who
have suffered catastrophic trauma (<1% of pregnant trauma victims require a second dose).
Therefore, the KBT should not be performed in most pregnant ED patients who have suffered
trauma.
A. Tonsillar herniation
B. Uncal herniation
C. Subfalcine herniation
D. Sphenoid herniation
Answer B. This patient’s symptoms of severe hypertension in concert with a decreased pulse
and irregular respirations are consistent with the development of elevated intracranial pressure
(ICP). Due to his history of severe hypertension, he likely suffered an acute intracerebral
hemorrhage resulting in an acute increase in ICP and transtentorial (or uncal) herniation. The
oculomotor nerve (cranial nerve III) is compressed between the uncus and the tentorium
cerebelli resulting in parasympathetic paralysis and subsequent mydriasis (“blown pupil”).
Tonsillar herniation results from an increase in posterior fossa pressure (e.g., due to a posterior
fossa mass). Symptoms may be subtle or may result in acute cardiorespiratory dysfunction (e.g.,
when a lumbar puncture is performed in the setting of an undiagnosed posterior fossa mass).
Subfalcine herniation occurs in association with a cerebral mass lesion, causing the medial
surface of the affected hemisphere to be pushed against the rigid falx cerebri and then herniate
underneath it. The cingulate gyrus is most commonly affected and it is often clinically silent.
A. Continuous chest tube output of 300 mL/hour for 4 hours is an indication for thoracotomy.
pg. 691
Answer A. Traumatic hemothorax may occur after both blunt and penetrating thoracic trauma.
Hemorrhage results most often from parenchymal vessel damage, then intercostal and internal
mammary artery lacerations, and uncommonly from great vessel injury. At least 200 mL of blood
must be present for costophrenic angle blunting to appear on upright chest x-ray. Tube
thoracostomy with a large chest tube (36-French or greater) at the fifth intercostal space and
midaxillary line is the treatment of choice. Smaller tubes will cause blood to clot and prevent
adequate drainage. Pneumothorax occurs concomitantly in almost a third of hemothoraces and
requires suction drainage. Initial chest tube drainage of 1,500 mL, 250 mL/hour of drainage over
4 hours, worsening hemothorax, hemodynamic instability, and cardiac arrest are all indications
for thoracotomy.
72. A 66-year-old male is brought to the ED after a high-speed motor vehicle collision. He
complains of chest pain. Given the extreme deceleration mechanism, you suspect traumatic
aortic injury. Which of the following is present in most ED patients with traumatic aortic injury?
A. BP >140/90
B. BP <90/50
D. Systolic murmur
Answer A. Patients with traumatic aortic rupture who make it to the ED alive usually (over 70%
of cases) have elevated blood pressures. This is because aortic hematomas tend to stretch
sympathetic fibers, causing an increase in systemic vascular resistance. Patients with traumatic
aortic injuries tend to have either moderately–severely elevated blood pressures (range 140/90
to 190/120) or no blood pressure at all—the in-between state of normal–low blood pressure is
not encountered as often because true rupture of the aorta would lead to almost immediate
death if hematoma containment were violated. Positive upper extremity pulses with decreased
lower extremity pulses (“pseudocoarctation”) occur in about one-third of cases. Systolic
murmurs also occur in about onethird of cases. Chest pain radiating to the back is more
commonly seen in atraumatic aortic dissection and is present only in a minority of cases of
traumatic aortic rupture. This statistic is likely confounded by the fact that many patients with
traumatic aortic ruptures have significant distracting injuries or altered mental status preventing
accurate assessment of symptomatology.
73. Which of the following fluids may precipitate a hyperchloremic metabolic acidosis if large
volumes are given during trauma resuscitations?
pg. 692
A. D10 W
B. Lactated Ringers
C. D5 0.45 N NaCl
D. 0.45 N NaCl
E. 0.9 N NaCl
Answer E. The ideal crystalloid solution for trauma resuscitation remains an area of debate.
Recently, many experts have begun recommending lactated Ringers (LR) over 0.9 N NaCl in
trauma and sepsis resuscitations because LR is a more “balanced” fluid and does not induce the
hyperchloremic metabolic acidosis that comes from large volume 0.9 N NaCl infusions. However,
at the time of this writing, there remains debate about whether one fluid has substantial clinical
advantages over another. However, D10 water, D10 normal saline, and D5 1/2 normal saline are
hypertonic solutions containing glucose, which offer no survival advantage and may predispose
to hyperglycemia. Half normal saline alone is hypotonic and would eventually cause electrolyte
abnormalities and excessive peripheral edema if given continuously during trauma resuscitation.
74. Which of the following is useful as a sensitive screening test for clinically significant
complications of blunt cardiac injury?
B. Troponin I
C. Troponin T
D. EKG
Answer D. Blunt cardiac injury results from blunt trauma directed at the sternum, usually from
patients striking the steering wheel in a motor vehicle collision. Patients with blunt cardiac injury
(formerly known as cardiac contusion) may develop myocardial stunning, congestive heart
failure (CHF), dysrhythmia, and in rare instances, when a coronary vessel is damaged, MI. The
diagnosis should be suspected in any case of blunt thoracic trauma, but physical examination is
often not revealing. The best screening tool for the diagnosis is EKG. Patients with any significant
abnormal finding on EKG should be admitted for observation, telemetry monitoring, and
confirmatory echocardiogram. Cardiac markers have been extensively studied to evaluate for
screening or confirming the diagnosis but are not particularly useful in either regard. Stress
testing is not indicated in patients with suspected blunt cardiac injury, as the tachycardic
response may actually exacerbate the traumatic insult.
pg. 693
75. Injury to which of the following is the most common cause of traumatic death in children?
A. Head
B. Chest
C. Abdomen
D. Pelvis
E. Femur
Answer A. Head trauma accounts for the large majority of all pediatric traumatic deaths. Falls
and MVCs are the most common mechanisms. When compared to adults, children’s heads are
proportionally larger and heavier relative to the rest of their body resulting in a higher likelihood
of serious injury. The possibility of intentional injury should be sought in all cases of pediatric
head trauma. Unlike adults, in whom the extent of intracranial bleeding is limited by the fixed
bony skull, infants may develop severe hemorrhagic shock from intracranial bleeding into a
more flexible skull. Furthermore, intracranial catastrophes such as epidural hematomas may
occur due to venous bleeding (rather than arterial bleeding as is more common in the adult)
resulting in delayed presentations.
E. On the field of play, simple orientation questions are adequate to detect concussion.
Answer D. Concussion is defined as a clinical syndrome following mild traumatic brain injury
with or without loss of consciousness. The symptoms are most commonly confusion, amnesia
surrounding the traumatic event, headaches, and nausea. Other neurologic signs such as slurred
speech, attention deficits, incoordination, mild personality changes, and disorientation may
occur in conjunction with amnesia and confusion. Acute neuroimaging is usually normal as
concussion is characterized by functional, rather than structural findings. MRI may detect subtle
signs of cerebral contusion or axonal injury a few days after the injury in a minority of cases.
Generally accepted guidelines indicate that athletes who have had a concussion should not
return to sports until approximately 1 week after symptoms have resolved. Simple orientation
tests are inadequate to detect concussion. The Standardized Assessment of Concussion (SAC) is
a point-based tool meant to be used on the field of play. It assesses short-term memory,
pg. 694
concentration, delayed recall, neurologic testing, and exertional tests such as sprints and sit-ups
in addition to orientation. Anosmia, the absence of the sense of smell, may occur in concussions
and may be permanent, also causing alterations in taste.
C. The teardrop sign refers to fat extending from the globe into the optic nerve.
E. Patients with orbital wall fractures should be instructed to blow their noses every 6 hours to
reduce nasal congestion.
Answer A. The thin orbital floor is the most easily damaged part of the orbit in trauma. Globe
injuries occur in one-fourth of patients with orbital floor fractures. The teardrop sign seen on
plain radiographs or CT scan is soft tissue that extends inferiorly from the orbital floor into the
maxillary sinus, indicating a floor fracture. Antibiotics are often given to patients with orbital
wall fractures but are recommended only if the fracture extends through an infected sinus.
Patients with orbital wall fractures should be instructed not to blow their nose, as it may worsen
the degree of herniation of globe contents into the sinuses. Nasal congestion should be treated
with a 3-day course of nasal decongestants.
78. A 56-year-old male presents after a high-speed motor vehicle collision. He complains of
severe chest pain and chest x-ray demonstrates a widened mediastinum. Chest CT scan shows a
traumatic aortic injury (TAI). The patient’s heart rate is 95 and blood pressure is 175/77.
Operative repair is scheduled in 30 minutes. Which of the following is the most appropriate
therapy at this time?
B. Clonidine
C. Hydralazine
D. Labetalol
E. Enalaprilat
Answer D. TAI occurs most commonly from high-speed motor vehicle collisions causing blunt
thoracic trauma. Most traumatic aortic ruptures are immediately fatal, but patients who survive
pg. 695
to ED evaluation are usually successfully treated. The descending aorta just distal to the
subclavian artery is the most commonly injured site. Chest and back pain are the most common
symptoms. Diagnosis is made by a combination of chest x-ray and CT aortography. Management
of TAI involves operative repair, but blood pressure and heart rate control with β-blockers is
essential to prevent further damage to the aorta from shear forces. Labetalol is an ideal single
agent for reduction of blood pressure and heart rate. Clonidine, hydralazine, and enalaprilat all
reduce blood pressure, but often cause reflex tachycardia and require a β-blocker in addition to
reduce the number of heart beats and shear forces to the aorta. Observation alone will result in
worsening aortic injury and risks immediate death if the aorta ruptures completely.
79. A 63-year-old female presents with an acute, severe headache after an MVC and is
diagnosed with a subarachnoid hemorrhage (SAH). Her vital signs are normal. Which of the
following should be part of her treatment regimen?
A. Labetalol
B. Lorazepam
C. Isoproterenol
D. Verapamil
E. Nimodipine
Answer E. Traumatic SAH results from bleeding in the subarachnoid vessels, causing blood to
accumulate in around the brain parenchyma and in the sulci. It is an extremely common cause
of traumatic intracranial hemorrhage. Symptoms include headache, vomiting, and photophobia.
Focal neurologic deficits are rare due to the diffuse nature of the bleeding and generally low
incidence of accompanying increased intracranial pressure. Cerebral vasospasm may occur as a
result of the SAH and is associated with worse outcomes. The use of peripheral calcium channel
blockers such as nimodipine improves outcomes though it does not actually appear to treat
vasospasm. Furthermore, vasospasm is not evident until about 3 days after hemorrhage, so its
role in ED treatment is limited.
80. A 50-year-old male presents after a motor vehicle collision. His primary survey is intact. The
pericardial view of his FAST scan is shown in Figure below. Which of the following is the next
best step in management?
A. Emergent pericardiocentesis
D. Needle thoracostomy
pg. 696
E. Cardiology consultation
Answer C. The patient has a normal subxiphoid pericardial view on the FAST scan. There is no
hypoechoic stripe indicating pericardial fluid (which is assumed to be blood in trauma) to
warrant any further procedure in this patient aside from completion of the remainder of the
FAST scan. Cardiology consultation should not be pursued just because the patient has a heart.
81. A 35-year-old male presents with a severe head injury after being struck in the head with a
baseball bat. His vital signs are HR 135, BP 82/45, RR 20. Which of the following is the most likely
cause of his hypotension?
A. Epidural hematoma
B. Subdural hematoma
D. Cerebral contusion
E. Extracranial cause
Answer E. Adult trauma patients with head injury are rarely hypotensive because of the
intracranial process itself, except in the end stages of herniation or severe scalp injuries. The
fixed bony skull limits the degree of hemorrhage in adult patients. In infants, the flexibility and
larger proportional size of the skull may allow enough bleeding to cause hypotension. In adult
pg. 697
trauma patients, an extracranial cause of hypotension should aggressively be sought, such as
bleeding in the chest, abdomen, retroperitoneum, pelvis, or femurs. Treatment of hypotension
in the head-injured patient should be undertaken quickly because cerebral blood flow is
dependent on mean arterial pressure (MAP) and limited by intracranial pressure (ICP).
82. A 23-year-old male presents after being assaulted by several men. Per eyewitness report,
the patient was kicked and struck with a baseball bat several times. The patient is brought in on
a backboard wearing a cervical collar. He is extremely agitated and combative, punching and
kicking staff and climbing off the bed. He is yelling, “I want to get out of here!” when any
question is asked of him. He appears to be moving every extremity except his left arm. You
complete the primary survey, which is intact except for the paralyzed left arm. His vital signs are
98.0, 95, 22, 166/94, and 98% RA. Which of the following is the most appropriate next step in
management?
A. CT brain
B. MRI brain
C. CT cervical spine
D. Rapid-sequence intubation
Answer E. Although the patient has clear evidence of brain or spinal injury from blunt trauma,
no radiographic evaluation can take place until the patient is cooperative. In cases where
patients put themselves or caregivers at risk of physical harm, the trauma team leader’s first
responsibility is to control the patient’s behavior to prevent this occurrence. Sedating the
patient with lorazepam or haloperidol is the ideal first-line management, but if this is not an
option, sedation and paralysis with rapid-sequence intubation should be performed. This will
allow the trauma evaluation to take place expediently and reduce the risk of harm to the patient
and caregivers. Once the patient’s combative behavior is controlled, CT of the brain and spinal
cord, along with the rest of the secondary survey may be initiated.
83. Which cardiac chamber is most commonly injured in penetrating thoracic injury?
A. Right atrium
B. Left atrium
C. Right ventricle
D. Left ventricle
pg. 698
Answer C. Due to its anterior position and size, the right ventricle is the most commonly
affected cardiac chamber from penetrating thoracic injury. The left ventricle is affected next,
followed by both atria equally. Multiple chambers are injured in almost one-third of cases.
Death occurs from either exsanguination or pericardial tamponade, the latter of which is treated
by ED thoracotomy and pericardial incision.
84. A 45-year-old male presents after a high-speed motor vehicle crash. He was the unrestrained
driver of a vehicle traveling 70 mph when he rear-ended a car stopped in front of him. He
remembers that he struck his chest on airbag and he complains of severe chest pain. His primary
survey is intact and his vital signs are normal. A chest x-ray is performed and is normal. Pelvic
x-ray and focused assessment with sonography in trauma (FAST) scans are negative. Secondary
survey reveals no additional injuries and the cervical spine is cleared clinically. Which of the
following is the most important next step in management?
A. Angiogram
E. Discharge home
Answer C. The patient is at risk for traumatic aortic injury (TAI), which is a common cause of
immediate death in motor vehicle crashes. TAI may result in sudden hemodynamic instability
and death in patients who initially appear to be stable after blunt trauma. Lateral- and
front-impact motor vehicle crashes at high speed, steering wheel impact to chest, and sudden
deceleration injuries each constitutes a high-risk mechanism for TAI. Although a normal chest
x-ray has good sensitivity (∼90%), patients who have a high-risk mechanism and symptoms
consistent with the diagnosis of TAI should undergo CT angiography (negative predictive value of
close to 100%). Diagnostic peritoneal lavage is usually indicated in patients who are
hemodynamically unstable to evaluate for presence of intraperitoneal injury. Pericardiocentesis
is only indicated in patients with pericardial tamponade in whom a pericardial window may not
be performed at the bedside. Admission to the hospital could eventually be indicated, but a
delay in the diagnosis of TAI could cause immediate death, and observation alone would be
unhelpful in prevention of this occurrence. Discharging the patient home without any further
workup for either TAI or blunt cardiac injury is contraindicated in a patient with such a high-risk
mechanism and suggestive symptoms.
85. A 26-year-old male is brought to the ED after being bludgeoned in the head with a baseball
bat. His Glasgow Coma Scale is 7. After intubation, a noncontrast CT brain demonstrates a
pg. 699
subdural hematoma. Which of the following is a proven benefit of administering seizure
prophylaxis to this patient?
Answer A. Patients with significant head injury requiring mechanical ventilation and possibly
operative care have worse outcomes if exposed to episodes of hypotension, hypercarbia, and
hypoxia. Early seizures in the setting of intracranial hemorrhage can cause significant periods of
hypercarbia and hypoxia, which may further worsen the ischemic penumbra by increasing
intracranial pressure. Seizure prophylaxis in the acute setting does nothing more than to reduce
the possibility of early seizures. Delayed seizures and chronic epilepsy are not prevented by
acute administration of antiepileptics. The goal of antiepileptic therapy is actually to maintain
cerebral perfusion pressure. Reductions in mean arterial pressure will reduce the cerebral
perfusion pressure according to the following formula:
86. A 48-year-old male presents after a motor vehicle collision in which his car was severely
rear-ended. He has an intact primary survey, but exhibits significant midline cervical spine
tenderness at C7-T1. You decide to order cervical spine imaging. Which of the following is most
correct regarding this patient?
Answer D. CT of the cervical spine is the optimal method of ruling out clinically significant
fractures. Plain radiographs have, at best, 85% sensitivity, so high-risk cases should have CT
instead. Clinical decision rules such as the NEXUS criteria or the Canadian C-spine rules can be
very helpful in avoiding unnecessary spinal imaging. Most cervical spine fractures occur at C2,
C6, or C7. Pain medication should never be withheld as localization is not decreased with
adequate pain control.
pg. 700
87. A 34-year-old male presents with chest pain and shortness of breath after being struck in the
right side of the chest with a baseball bat. Chest x-ray demonstrates a moderate to large
pneumothorax on the right. Which of the following is the most appropriate management at this
time?
E. Observation alone
A. Delayed neurologic deficits after blunt neck trauma suggest carotid artery dissection.
B. All patients suspected to have an esophageal injury should receive a barium contrast
esophagram.
D. All neck wounds should be probed to determine the depth of the wound and integrity of vital
structures.
E. Impaled objects should always be removed in patients with penetrating neck trauma.
pg. 701
experience stroke symptoms such as hemiparesis, hemiplegia, or aphasia between 1 and 24
hours after injury due to carotid artery dissection or thrombosis. Furthermore, a significant
percentage of patients will not develop symptoms until after 24 hours has passed. Vertebral
artery injury can also occur, although it is less common. Because the vertebral arteries combine
to form a single basilar artery, injuries to the vertebral artery do not produce lateralizing
symptoms. Patients may present with nausea, vomiting, central vertigo, and visual changes.
Esophageal injuries are relatively uncommon, frequently subtle, and often missed in the setting
of neck trauma. Because barium may provoke an inflammatory mediastinitis, patients with
suspected esophageal injury should first receive an examination with water-soluble contrast
such as gastrografin. If the initial study is negative, a follow-up examination with barium can be
performed because of its superior sensitivity for smaller defects. Zone II injuries are most
amenable to surgical repair due to the relatively uncomplicated surgical exposure and vascular
control. Neck wounds should never be probed outside the operating room. If careful
examination fails to determine platysma integrity, a surgeon should be consulted for presumed
violation of the platysma. Impaled objects should always be left in place because they may
tamponade vascular injuries. They should only be removed in the operating room under direct
visualization.
89. A 26-year-old female is brought to the ED after she was ejected in a motor vehicle accident.
Initial evaluation reveals a confused patient with multiple scalp wounds and vital signs of P 130,
BP 85/55. After intubation and fluid resuscitation, initial plain films reveal clear lungs but an
obvious pelvic fracture. Focused assessment with sonography in trauma (FAST) examination
reveals hemoperitoneum. Which of the following is the next step in management?
B. Pelvic angiography
D. Exploratory laparotomy
E. Use diagnostic peritoneal lavage (DPL) cell counts to determine the need for laparotomy
pg. 702
90. While working in the ED, a mass-casualty alert is called in by EMS after a bomb was
detonated outside an office building downtown. After the most critically ill patients are
successfully managed, a 29-year-old male is brought to the ED by EMS. EMS estimates that he
was approximately 100 feet from the detonation site. The patient states he was thrown a few
feet and knocked down but did not suffer loss of consciousness. He complains of headache,
dizziness, shortness of breath, and decreased hearing with ringing of the ears. Which of the
following is true?
B. Oropharyngeal petechiae are always present in the setting of critical abdominal solid organ
trauma.
D. Blast lung injury is the most common fatal injury among initial survivors.
E. High-energy explosives generate blast waves which maintain their pressure and velocity over
long distances.
Answer D. High-energy explosives, such as TNT, C-4, dynamite, nitroglycerin, and ammonium
nitrate, produce supersonic overpressurization shock waves. However, the pressure and velocity
within these waves rapidly decays with distance and time. As a result, injuries due to pressure
from the primary blast wave are relatively uncommon as those who are close enough to be
injured by the primary blast wave often suffer catastrophic secondary injuries (penetrating
trauma due to shrapnel or blunt trauma due to being struck by debris or due to being thrown
against solid objects). Blast lung injury is the most common fatal injury among initial survivors.
Chest x-ray reveals a classic “batwing” or “butterfly” appearance. Air embolism, due to the
creation of microscopic alveolovenous fistulas, may also occur. Historically, the integrity of the
TMs has been used as a surrogate marker for more serious internal injury. While the pressure
required to induce TM rupture is much less than what is required to affect the lungs, brain, or
gastrointestinal system, it is a very unreliable marker of more serious injury since several factors
affect the likelihood of TM rupture including orientation to the blast wave (perpendicular is
worse than parallel) and presence of occlusive cerumen in the ear canal (protective unless
directly adjacent to the TM). However, the presence of TM rupture and oropharyngeal
petechiae should indicate a higher likelihood of more serious underlying trauma. Due to blast
wave physics, air-containing organs such as the ears, lungs, and intestines are more commonly
affected than solid organs.
91. Which of the following traumatic mechanisms is the best candidate for emergency
department (ED) thoracotomy?
pg. 703
C. Blunt trauma to the pelvis
Answer D. Emergency thoracotomy is most useful and successful in patients with penetrating
chest wounds who develop traumatic cardiac arrest during or just before the ED resuscitation
begins. Patients with pericardial tamponade due to stab wounds have the highest survival rates
after ED thoracotomy. In the past, thoracotomies were performed routinely in the ED for
traumatic arrest patients. However, concerns about risk to hospital personnel, cost, and low
success rates of the procedure have restricted indications for thoracotomy to penetrating chest
wounds. Any patient with blunt trauma (even to the chest) is not a candidate for ED
thoracotomy.
92. A 19-year-old male is brought to the ED after being stabbed in the neck. His primary survey
is intact, but his secondary survey reveals a large, oblique laceration over his right lateral neck,
starting from just lateral to the sternal notch and extending superolaterally to the posterolateral
right neck, below the angle of the mandible. The patient denies odynophagia, dysphagia,
dysphonia, or dyspnea. Examination of his wound reveals a small amount of ongoing bleeding,
but no evidence of an expanding hematoma, clearly audible bruit or thrill. Which of the
following is the next best step?
A. Barium swallow
D. Esophagoduodenoscopy (EGD)
E. Bronchoscopy
Answer B. Historically, the evaluation of penetrating neck injuries is based on the zone in which
the injury occurs. Zone I is a small area from the sternal notch to the cricoid cartilage. Zone II is
the largest and most commonly injured area, extending from the cricoid cartilage to the angle of
the mandible. Zone III is another relatively small area which extends above the angle of the
mandible to the ear. Traditionally, patients with zone II injuries that are found to violate the
platysma upon local exploration are brought to the OR for surgical exploration. However in the
absence of other evidence of injury, this leads to a large number of negative surgical
explorations and unnecessary added morbidity. Patients with zone I injuries receive a complete
workup for tracheal, esophageal, and vascular injury including bronchoscopy, barium swallow,
EGD, and a four-vessel angiogram. Though zone III contains the pharynx, the chief concern is a
vascular injury, so patients have traditionally received four-vessel angiograms plus additional
pg. 704
studies to evaluate the pharynx depending on the extent of the injury. A better approach to
evaluating patients with penetrating neck injuries is to evaluate them based on their physical
findings and symptoms. There are three broad groups of patients with penetrating neck trauma:
Patients with hard signs of vascular or aerodigestive tract injury, patients with soft signs of
injury, and asymptomatic patients. Hard signs of vascular injury include significant bleeding, a
pulsatile hematoma, an audible bruit or palpable thrill, and hemorrhagic shock or altered mental
status secondary to bleeding. Hard signs of aerodigestive tract injury include a gross laceration
to the trachea or bubbling in the wound, or blood in the airway or gastrointestinal tract
manifested by hemoptysis, or hematemesis. Patients with hard signs of vascular or
aerodigestive tract injury are brought to the OR for further investigation. Most patients,
however, have soft signs of injury, that is, a smaller amount of bleeding in the setting of normal
blood pressure, no hematoma, mild trouble swallowing or vocal changes, and minimal
subcutaneous emphysema. These patients benefit most from CT angiography. CT angiography
has replaced the comprehensive workup in most trauma centers because of its speed, the
breadth of information obtained, decreased morbidity relative to traditional angiography, and
relative ease of patient monitoring. Further evaluation and management is dictated by the
findings on CT angiography. Patients who are truly asymptomatic are amenable to primary
wound closure and observation. This patient has mild ongoing bleeding. Thus, while there is no
pulsatile hematoma or palpable thrill ,there is some risk for an important vascular injury and
should undergo CT angiography (fig. below)
pg. 705
93. A 27-year-old male is brought to the ED after being stabbed in the neck with a knife and
robbed. Upon examination, you note a 3-cm wound to zone II of the neck with an intact
platysma. Which of the following is the next best step in management?
D. Carotid angiography
E. CT of the neck
Answer A. The platysma is the most superficial muscular layer of the neck, as it is sandwiched
between the superficial and deep fascia of the neck. There are no vital structures superficial to
the platysma. Therefore, patients with penetrating trauma who have an intact platysma can be
given local wound care and discharged for routine follow-up. However, although neck wounds
may be carefully examined to determine whether the platysma is intact, wounds should never
be probed outside of the OR. If, after shallow examination, there is any uncertainty regarding
platysma integrity, a surgeon should be consulted for further evaluation and treatment.
A. Vagus nerve
B. Phrenic nerve
C. Sympathetic chain
pg. 706
Answer B. Emergency thoracotomy is indicated in patients who have traumatic arrest in the ED
or shortly before arrival from penetrating thoracic trauma. The initial incision of the chest wall
begins in the sternal area and sweeps along the superior border of the rib all the way laterally to
the edge of the bed. The rib spreaders are then placed and the left lung is moved out of the way
to expose the pericardium. An incision to the pericardium should be made anterior to the
prominent phrenic nerve, which is visible in the figure. Damage to the phrenic nerve may cause
diaphragmatic weakness and seriously impair respiratory function.
95. A 45-year-old male arrives hypotensive after a motor vehicle collision. His physical
examination demonstrates clear lungs but an unstable pelvis. The pelvis is secured tightly with a
bedsheet, but the patient continues to be hypotensive despite crystalloid and blood
replacement. Bedside focused assessment of sonography in trauma scan is negative. Which of
the following is the most appropriate next step in management?
A. CT chest
B. CT abdomen/pelvis
C. CT brain
E. Laparotomy
Answer D. Patients with a combination of hypotension and pelvic fracture have extremely high
mortality and should be managed aggressively. Initial stabilization involves securing the pelvis
with a bedsheet or tightening device to reduce the volume into which hemorrhage can occur
and copious crystalloid and blood resuscitation. If the patient stabilizes, further evaluation with
CT scan may be performed to better delineate the injuries. However, if the patient remains
unstable, angiography with embolization should be performed to limit the extent of
hemorrhage. As a general rule, hemodynamically unstable trauma patients should never have
CT scans performed.
96. A 26-year-old female at 31 weeks’ gestation is brought to the ED after a high-speed motor
vehicle accident. She was an unrestrained passenger but was not ejected from the vehicle. She
is brought in lying on a backboard with a c-collar and is complaining of difficulty breathing. Her
room air oxygen saturation reads 82%, although it increases to 93% on a nonbreather. A chest
x-ray was taken and is shown in Figure below. Which of the following is true?
A. Chest tubes should be placed in the fifth intercostal space at the midaxillary line.
pg. 707
C. Chest tubes are contraindicated during pregnancy.
Answer E. Chest tubes should generally be placed in the third or fourth intercostal space during
pregnancy and should never be placed lower than the fourth intercostal space. Since the
diaphragm elevates approximately 4 cm during pregnancy, there is an increased risk of
abdominal placement of chest tubes if they are placed in the typical fifth intercostal space. As in
nonpregnant women, the midaxillary line is the least muscular area of the chest wall, which
makes it an ideal location for chest tube insertion. Fetal evaluation should always be delayed
until maternal evaluation, treatment, and stabilization, even in the setting of obvious fetal
distress.
97. A 35-year-old female presents after a fall from a fourth-story window. She was initially
moaning, but then became completely unresponsive. Her pupils were initially miotic and equal,
but then became dilated and unreactive. She exhibits decorticate posturing in response to pain.
Which of the following is the most likely mechanism?
A. Skull fracture
B. Uncal herniation
D. Subarachnoid hemorrhage
E. Brain death
pg. 708
Answer B. Cerebral herniation syndromes are a result of severely increased intracranial pressure
(ICP), usually because of trauma. Herniation occurs when the increased ICP causes movement of
various parts of the brain across fixed structures, such as the falx cerebri (dividing left from
right), tentorium cerebella (dividing the cortex from the brainstem and cerebellum), or foramen
magnum (dividing intracranial from extracranial structures). Mortality is virtually 100% in
untreated patients. Uncal herniation is the most common type, in which the ipsilateral temporal
lobe is compressed against the tentorium cerebella and subsequently puts pressure on the
underlying brainstem. The initial clinical manifestation is oculomotor nerve injury, causing
ipsilateral ptosis, oculomotor dysfunction, and mydriasis. As herniation progresses, contralateral
hemiparesis occurs and the contralateral uncus is compressed, leading to bilateral decorticate
posturing. Ultimately, uncal herniation often progresses further until the brainstem herniates,
causing respiratory failure and death.
98. A 6-month-old infant falls out of her high chair onto the kitchen floor and is brought by her
parents for evaluation. The parents note that there was no definite loss of consciousness (LOC),
vomiting, or seizure activity. She has not been abnormally sleepy, but her parents note that she
should be hungry at this time and has been refusing feeds. Her physical examination, including
neurologic examination, is unremarkable. Which of the following is the most appropriate next
step in management?
B. Skull x-rays
C. CT brain
D. MRI brain
E. Discharge home
Answer C. Head trauma accounts for the large majority of all pediatric traumatic deaths.
Children’s heads are proportionally larger and heavier relative to the rest of their bodies than
adults and are more likely to be seriously injured. The PECARN investigators have established
the following criteria as low risk in head injured patients <2 years old:
No LOC
No severe mechanism of injury (fall >3 feet, head struck by high impact object, MVC with
ejection, death or rollover, pedestrian vs auto)
pg. 709
Since this patient is behaving abnormally and fell >3 feet, a CT scan should be performed to
evaluate for intracranial injury. Controversy exists as to whether brief LOC should be considered
as a sole indication for CT scan in the older child, but in the infant it is an accepted indication
given the difficulty of neurologic examination. The EP should always consider contacting DCFS in
cases of suspected child abuse, but a clear, consistent history with a normal physical
examination is less likely to have high potential for abuse. Skull x-rays are almost never indicated
in head trauma as they do not often change management and require further testing with CT
scan anyway if abnormal. MRI offers little benefit over CT scan in the acute setting and is much
more technically difficult in the infant. Discharging the patient home without CT scan or serial
neurologic examinations is not recommended.
99. A 23-year-old male presents after a stab wound to his right lumbar spinal area. Which of the
following is the most likely neurologic deficit?
Answer C. Patients with penetrating injuries just lateral to the spine often have incomplete cord
injury, usually causing hemisection of the cord, known as Brown–Sequard syndrome. Neurologic
findings include loss of ipsilateral motor function and vibratory/position sensation and
contralateral pain/temperature sensation distal to the injury. The fibers carrying
pain/temperature sensation decussate (cross) a few levels after they enter the cord from the
periphery, so contralateral findings are observed. The fibers carrying motor axons and
vibratory/position sensation do not decussate until high in the spinal cord, so ipsilateral findings
predominate. The functional outcome for patients with cord hemisection is favorable overall
with few patients losing bowel, bladder, or ambulatory function.
100. A 64-year-old female with a past history of sick sinus syndrome and recent pacemaker
placement presents with neck pain after a low-speed motor vehicle collision. She has midline
cervical spine tenderness at C6–7. A cervical spine plain film series is inadequate for visualizing
these segments, so a CT scan is performed. The CT scan is normal, but the patient still has pain
in her neck. Which of the following is the most appropriate next step in management?
pg. 710
D. Oblique cervical spine x-rays
Answer C. Modern, multidetector CT scanners, which are ubiquitous in the United States, are
nearly 100% sensitive in detecting clinically significant cervical spine injuries. Flexion and
extension and oblique views don’t add additional information. While MRI is more sensitive for
detecting ligamentous injuries, those injuries are almost never clinically relevant in patients
without neurologic complaints or findings. Patients with persistent tenderness without
neurologic complaints or findings may be discharged home with outpatient follow-up. Some
guidelines suggest discharging patients in a rigid collar (such as a Miami-J, Aspen, or Philadelphia
collar) with short-term follow-up, ideally in consultation with the trauma or neurosurgical team.
As pacemakers are increasingly designed to be MRI-compatible, MRI utilization may increase in
this patient population. Additionally, reasons for the motor vehicle collision should be
aggressively sought—with a recent pacemaker placement for sick sinus syndrome, pacemaker
malfunction and dysrhythmia causing syncope may be the causative process. Discharge,
therefore, should not take place before both medical and traumatic issues are completely
evaluated.
101. A 16-year-old male is brought to the ED by his parents after he fell from a bicycle while
practicing tricks. He struck the vertex of his helmet as he hit the ground and the fall was
witnessed by his brother who said the patient was “out for a second.” There was no vomiting,
and the patient denies any headache, but he says he cannot remember what happened, and
does not remember that he was riding a bike. His mother also states he has been “forgetting
new things” since the accident. Which of the following is true?
A. Anterograde amnesia is typically more severe than retrograde amnesia after blunt head
trauma.
B. Inability to recall one’s birthdate and name in an otherwise alert patient after trauma is most
often a sign of malingering.
E. There are well-studied, evidenced-based guidelines regarding the optimal time to return to
athletic activities.
Answer B. This patient has suffered a concussion. However, the use of the word “concussion”
often causes confusion due to competing and vague definitions. The American Academy of
Neurology defines concussion as a trauma-induced alteration of mental status that may or may
not be associated with loss of consciousness. In the literature, “concussion” is frequently used in
reference to those patients with the mildest form of traumatic brain injury. In patients with
pg. 711
head trauma, retrograde amnesia is more common and longer in duration than anterograde
amnesia. Furthermore, patients who appear unable to recall their name or birthdate and are
otherwise well are likely malingering. In contrast to past beliefs, simple (i.e., not depressed) skull
fractures do not strongly predict underlying brain hemorrhage. In fact, the fracture of the skull
may well dissipate the majority of the energy delivered from the impact before it is transmitted
to the brain. Post-concussion syndrome refers to the constellation of symptoms that occurs
after head injury, including headache, dizziness, memory problems, and occasional
neuropsychiatric complaints. It rarely occurs in young children. While there are several
consensus-based guidelines regarding the optimal time to return to athletic activities, there is a
lack of prospective data to guide these decisions.
102. Which of the following is the most common serious complication of pulmonary contusion?
B. Pneumonia
C. Pulmonary embolism
D. Myocardial contusion
E. Pericardial tamponade
Answer B. Pulmonary contusion refers to direct parenchymal injury from blunt thoracic trauma.
It is often associated with other intrathoracic and chest wall injuries, most commonly multiple
rib fractures. Impaired oxygenation and ventilation may cause severe deficits in respiratory
function requiring endotracheal intubation and mechanical ventilation. The diagnosis is made by
initial chest radiography, although CT scan is more sensitive and able to detect other thoracic
injuries as well. Atelectasis and pneumonia are the most common complications of pulmonary
contusion but may take several days after the injury to develop. Prophylactic antibiotics before
the onset of clinical signs and symptoms of postcontusion pneumonia are not recommended.
ARDS may develop in patients with particularly severe pulmonary contusions but is less common
than concomitant pneumonia. Pulmonary embolism may also occur in patients who are
bedridden after severe traumatic injuries but do not occur at significantly higher rates in
patients with pulmonary contusion. Myocardial contusion and pericardial tamponade may occur
from the same blunt traumatic forces that caused the pulmonary contusion but do not generally
occur as a complication of isolated pulmonary contusion.
103. A 13-year-old female is brought to the ED by EMS after a car accident. The patient was the
restrained right-sided back-seat passenger within a car that was struck by another vehicle on the
same side as her seat. She was extricated by EMS, placed in a cervical collar and on a backboard,
and transported to the ED. Upon arrival, her vital signs are HR 164, BP 88/50, T 99.0°F, SaO2
99% on RA. She is awake and alert but complaining of flank, abdominal, and hip pain. There are
pg. 712
abrasions over the right side of her chest and abdomen. A bedside fluid diagnosed by bedside
ultrasound (FAST) scan reveals a small amount of fluid in her right upper quadrant. Portable
chest x-ray is normal while portable pelvis x-ray reveals a right superior ramus fracture. Which
of the following is the next best step?
A. CT scan
104. A 23-year-old male presents to the ED with loss of consciousness after being kicked in the
head. He is now awake and alert and complains of a headache. Brain computed tomography
(CT) is performed and a slice is shown (Fig. below). Right after the CT scan, he becomes
unresponsive. Which of the following is the most appropriate next step in management?
B. Endotracheal intubation
D. Emergent thoracotomy
E. Repeat CT scan
pg. 713
Answer B. The CT scan shows a right-sided epidural hematoma. While the patient is initially
awake and alert, this may be indicative of the lucid interval that is often seen in patients with
epidural hematoma (although it is neither sensitive nor specific for the diagnosis). Acute
worsening of the clinical status mandates returning to the ABCs of trauma evaluation. Although
burr hole placement is the most important definitive management, control of the airway is the
most important initial action. CT and Focused Assessment with Sonography in Trauma (FAST)
scans have largely replaced DPL in the ED. In an unstable patient with a suspected
intraperitoneal source of bleeding despite a negative FAST scan, DPL may be performed if the
patient is too unstable for CT scan (e.g., persistent, severe hypotension). However, in solitary
head trauma, such a scenario is unlikely. Thoracotomy is indicated in patients with penetrating
trauma to the chest who arrest in the ED or shortly beforehand. Nasogastric tube placement is
contraindicated in patients with severe head and facial trauma as damage to the inferior portion
of the skull may allow the tube to pass from the nose into the cranium.
105. Which of the following wounds will most likely develop a wound infection?
pg. 714
C. A 4-cm hand wound suffered 10 hours prior to evaluation
Answer B. Wounds >5 cm in length, in locations other than the head and neck, in patients with
diabetes are most likely to be infected. Interestingly, the 6 and 12 hour time windows that are
often used to determine infection risk do not actually predict higher infection risk. Wounds with
visible contamination also increase wound infection rates, but not as much as the other factors
above, and this risk is mitigated through appropriate wound irrigation with tap water or saline.
106. The parents of a 12-year-old male bring him to the emergency room after he hit the back of
his head while diving for a basketball just prior to arrival. He did not lose consciousness, had no
period of confusion, and has not been vomiting, but he has been complaining of a moderate
generalized headache and dizziness since the incident. His vitals are stable and his neurologic
examination is normal. What guidance should you provide the parents?
A. He hasn’t had a concussion and can engage in all activities without restriction
B. He’s likely had a concussion but he does not require activity restriction
D. He’s had a concussion and he should undergo physical and cognitive rest until his symptoms
resolve
E. He hasn’t had a concussion, but he should have neurology follow-up to ensure the headache
and dizziness resolve.
Answer D. Historically, the diagnosis of a concussion has been based on a history of loss of
consciousness or a change in cognition that may also be associated with amnesia. While all of
these symptoms may be present in patients with concussions, very few patients with symptoms
suggestive of a concussion have amnesia or have lost consciousness. Instead, the diagnosis of
concussion is based on a constellation of symptoms in after head trauma. Headache and
dizziness or unsteadiness are the most common symptoms of concussion. Other symptoms
include trouble concentrating, light or sound sensitivity, drowsiness, and irritability or
personality changes. Neuroimaging studies are not helpful unless there is a strong suspicion for
skull fracture or intracranial hemorrhage. Patients with a concussion should undergo physical
and cognitive rest until all their symptoms are completely resolved, after which they may return
to participate in regular activities in a graded fashion.
pg. 715
107. Among trauma patients, which of the following is the most common incomplete spinal cord
syndrome?
A. Brown–Séquard syndrome
Answer B. Central cord syndrome is the most common incomplete spinal cord syndrome. Unlike
anterior cord syndrome, central cord syndrome typically results from forced hyperextension
injuries in arthritic middle-aged and older adults. The posterior ligamentum flavum is thought to
buckle and compress the cord against anterior osteophytes, resulting in a contusion to the
central aspects of the cord. Due to the topography of the spinal cord, extension injuries
occurring at the level of the cervical spine result primarily in flaccid paralysis of the upper limbs;
to a lesser extent, spastic paralysis or paresis of the lower extremities occurs in the setting of
large cord lesions. Furthermore, distal muscles are affected more than proximal muscles. There
is variable sensory dysfunction, but because most sensory neurons are located peripherally,
sensory findings tend to be less prevalent. Finally, there should be preserved perianal sensation,
voluntary rectal motor function, big toe flexor activity, and preservation of the bulbocavernosus
reflex. Together, the presence of these functions is referred to as sacral sparing.
108. A 22-year-old male presents with left-sided facial and periorbital pain after a fight. A CT
scan reveals the image shown in Figure 12-6. Which of the following examination findings may
be present?
C. Enophthalmos
D. Periorbital emphysema
pg. 716
Answer E. The CT reveals the classic “teardrop” sign associated with orbital floor fracture. Direct
trauma to the orbit causes an immediate rise in intraorbital pressure fracturing the weakest
borders of the orbit. This most commonly results in fractures of the orbital floor and medial
orbital wall. Due to swelling and stretching of the infraorbital nerve, there is often transient
hypoesthesia in its distribution lasting for 1 to 2 weeks. In addition, the inferior rectus muscle
may become entrapped in the orbital floor defect; effectively tethering the globe such that
upward gaze is limited or not possible. Patients may complain of diplopia if partial upward gaze
is possible. Enophthalmos results from loss of orbital volume and supporting structures. It may
not be visible acutely because of the swelling associated with the injury. However, delayed
repair is required if it is present 2 weeks after the injury. Periorbital emphysema results from
communication of the orbit with the sinus, an air filled structure. It is important to note that
exophthalmos resulting from a retroorbital hematoma can also occur. This is a true emergency
that may require immediate lateral canthotomy.
109. A 52-year-old previously healthy male presents by EMS for evaluation of a scalp laceration
after a fall. His initial CT reveals a small subdural hematoma. However, he later seems more
confused and you suspect he will need intubation. Which of the following is the best means of
preoxygenation before intubation?
D. “Blow by” oxygen over the patient’s mouth connected to wall oxygen set as high as possible
pg. 717
Answer C. Though most hospital flow meters are only demarcated to 15 L/minute, most of them
can achieve flow rates well beyond this rate. FiO2 levels greater than or equal to 90% are
achievable with flow rates of 30 to 60 L/minute while using a conventional reservoir face mask
(commonly called a nonrebreather even though most such masks are actually rebreather
masks). Spontaneously breathing patients achieve a maximal reservoir of oxygen and
denitrogenation after about 3 minutes of normal breathing or after eight vital capacity breaths.
Noninvasive positive pressure ventilation is an excellent adjunct for patients who can’t achieve
adequate oxygen saturation levels with a standard reservoir mask as above. “Blow by” oxygen
provides only ambient FiO2 levels, while nasal cannulas and venturi masks don’t provide
adequate oxygen levels to maximize preoxygenation.
110. A 10-year-old male is brought to the ED after a head injury. Based on his CT scan which of
the following physical findings is most likely (Fig. below)?
B. Bitemporal hemianopia
E. Left pupillary constriction, left eye deviated inferiorly and laterally, coma
pg. 718
Answer D. The CT reveals an epidural hematoma, as shown by its biconcave (sometimes called
lentiform) shape. Children are more prone to forming epidural hematomas because the dura is
less tightly attached to the skull than it is in adults. This patient’s CT demonstrates a right-sided
parietal epidural hematoma. This applies superior and lateral pressure to the entire right brain,
pushing the brain toward the midline and toward the tentorium cerebelli. This latter pressure
squeezes the oculomotor nerve against the temporal bone, resulting in partial oculomotor nerve
(cranial nerve III) palsy and ipsilateral pupillary dilation. Complete third nerve palsies, in which
the affected eye is deviated laterally and inferiorly (“down and out”) may also occur but are a
late finding. Contralateral palsies can also occur but also indicate more severe trauma and are a
late finding. Bitemporal hemianopsia, in which there are bilateral temporal visual field cuts, are
caused by pressure on the optic chiasm, classically due to a pituitary tumor.
111. A 22-year-old male presents with a gunshot wound to the left chest. Chest x-ray is shown in
Figure below. Which of the following is the most likely diagnosis?
A. Pneumothorax
B. Pericardial tamponade
C. Hemothorax
D. Pneumoperitoneum
E. Diaphragmatic rupture
Answer C. The patient has nearly complete opacification of the left lung. In trauma patients who
are usually supine, this indicates a large hemothorax, as blood will layer through gravity
throughout the entire lung field. Treatment involves tube thoracostomy drainage and possible
pg. 719
thoracotomy if the exsanguination is rapid or severe. Pneumothorax may be concomitant but is
not evident on this chest x-ray. Pericardial tamponade is best diagnosed by FAST scan.
Pneumoperitoneum is not evident—air under the left diaphragm is much more likely to be a
physiologic gastric bubble. Diaphragmatic rupture would be indicated by abdominal organ
herniation into the thorax which is not evident.
112. Which of the following is true regarding traumatic aortic injury (TAI)?
Answer C. TAI occurs most commonly from high-speed motor vehicle collisions (MVCs) causing
blunt thoracic trauma. Most traumatic aortic ruptures are immediately fatal, but patients who
survive to ED evaluation are usually successfully treated. The descending aorta just distal to the
subclavian artery is the most commonly injured site. Chest and back pain are the most common
symptoms. The initial screening test is plain chest x-ray—however, the sensitivity of plain films is
only up to 85%. In cases where suspicion for TAI is high, confirmatory CT aortography should be
performed, as it has close to 100% sensitivity. Transesophageal echocardiography may be used
in select cases where CT scan is not possible, but transthoracic echocardiography is much less
accurate and should be used only to evaluate for pericardial effusion or tamponade, not TAI.
Management of TAI involves operative repair, but blood pressure and heart rate control with
β-blockers is essential to prevent further damage to the aorta from shear forces.
pg. 720
Answer A. Traumatic diaphragmatic injuries are almost equally caused by blunt and penetrating
mechanisms. They much more commonly occur on the left side, because the liver serves as
excellent protection against both injury and post-traumatic herniation. Once the diaphragm is
injured, the defect remains without spontaneous healing due to the constant pressure subjected
to the diaphragm by normal respiratory forces. The main problem with missed diagnosis is the
high rate of delayed herniation of abdominal contents into the thorax, which can occur months
or even years after the initial injury. None of the standard diagnostic tests for trauma (FAST, CT
scan, DPL, radiography) is sensitive or specific enough to accurately evaluate for diaphragmatic
injury. Direct visualization with thoracoscopy or laparoscopy in suspected cases is required.
pg. 721
Orthopedic emergency
➢ Chapter includes:
Isolated extremity trauma
Orthopedic related condition
Pediatric and pregnancy isolated extremity trauma
1. A 9-year-old male presents with right wrist pain after a fall from playground equipment at
school. He has minimal swelling, no ecchymoses, and moderate tenderness over the distal
radius, but x-rays demonstrate no fracture. Which of the following is true?
A. The patient is at high risk for growth restriction affecting the radius
D. The patient should be put in a long-arm posterior mold splint made with plaster or fiberglass
Answer C. This patient’s injury is consistent with a Salter Harris Type I fracture, clinically defined
as tenderness over the growth plate with no radiographic evidence of fracture. Based on the
information provided, there is no clinical evidence a serious injury. There is a lack of
evidenced-based data to guide decision making in these patients. However, in the limited data
available, patients with Type I Salter Harris fractures achieve equal outcomes whether they are
splinted with removable or more permanent fiberglass or plaster splints. Thumb spica splints are
reserved for thumb-specific injuries, and Salter Harris Type II injuries reflect a fracture of the
distal metaphysis as well as the growth plate but this patient’s x-rays were normal.
pg. 722
2. Which of the following is true regarding reduction of an anterior shoulder dislocation?
3. Which of the following is the best modality to diagnose posterior sternoclavicular dislocation?
D. CT chest
E. Thoracic ultrasound
Answer D. Sternoclavicular dislocations may occur in the anterior or posterior direction. Anterior
dislocations are more common and occur because of a medially directed force to the shoulder.
These injuries are unlikely to cause serious injury beyond the dislocation itself. Posterior
dislocations result from a posteriorly directed force to the sternoclavicular joint and may cause
great vessel, mediastinal, or airway injury. CT is more accurate than either plain films or
ultrasound for the diagnosis of both anterior and posterior sternoclavicular dislocations.
4. Which of the following is true regarding the Ottawa ankle rules (OAR)?
pg. 723
A. Patients who do not meet the OAR criteria never have an ankle fracture.
Answer B. The OAR are a set of criteria that were devised to limit unnecessary ankle radiography
in patients presenting with ankle pain. The criteria require ankle radiography in any patient with
ankle pain who has the following:
1. Unable to ambulate four steps both at the time of injury and in the ED, or
2. Bony tenderness to palpation of the tip of either malleolus or tenderness of the distal 6 cm
posterior to either malleolus.
The original paper, published in 1992, described a sensitivity of 100% and a specificity of 40%.
Since then, however, several other studies have published a variety of other values, suggesting a
wide variability for specificity and sensitivity somewhere >96%. However, even in the original
paper (sensitivity of 100%), occasional patients had “insignificant” fractures such as chip
fractures that would be managed in the same way as a sprain. The squeeze test involves
squeezing the tibia and fibula approximately 5 cm proximal to the malleoli in an effort to
provoke pain in the ankle. Increased ankle pain indicates disruption of the distal tibiofibular
syndesmosis. The OAR have not been validated for pediatric use. Like other clinical decision
rules, the OAR should not be used in unreliable patients such as patients with decreased mental
status or intoxicated patients.
Answer D. Posterior shoulder dislocations occur much less commonly than anterior dislocations.
Mechanisms include seizure (due to stronger internal rotator muscles compared with external
rotator muscles), electrocution, and fall on an outstretched hand. Patients with posterior
shoulder dislocation are almost never able to abduct or externally rotate their affected arms.
pg. 724
Neurovascular injury is much less common than with anterior dislocations due to the anterior
position of the neurovascular structures. Posterior shoulder dislocation is often confused with
adhesive capsulitis and may simply present as stiffness and limited range of motion rather than
frank pain. Recurrent injury does occur, but less commonly than in anterior shoulder
dislocations. Management is with early reduction and orthopedic consultation.
A. Lunate
B. Scaphoid
C. Capitate
D. Hamate
E. Pisiform
Answer C. The capitate articulates directly with the lunate and dorsally dislocates when there is
enough carpal instability due to fracture or ligamentous injury. On lateral radiographs, the
lunate is in proper position relative to the radius, but the capitate falls posterior to the lunate
and may even articulate with the distal radius. In lunate dislocation, the lunate dislocates volarly
relative to the radius and the capitate is in line with radius. The median nerve is the most
common peripheral nerve injured in cases of lunate or perilunate dislocation.
7. A 27-year-old male is brought to the ED after a motor vehicle accident complaining of right
arm pain. His arm is swollen and shortened compared with the unaffected side. His x-ray reveals
the image shown below . Which of the following associated findings is most likely to be present
upon physical examination?
pg. 725
Answer A. The radial nerve may be injured in as many as 20% of humeral shaft fractures because
of its close proximity to the humerus as it travels posteriorly in the spiral groove. Most of these
injuries are transient neurapraxias and will improve without intervention. Therefore, such
injuries should be well documented and followed by close outpatient observation. Injuries to
the radial nerve in this area result primarily in wrist drop along with weakness of finger
extension and hypoesthesia and decreased two-point discrimination in the distribution of the
radial nerve. The radial nerve also innervates the supinators of the wrist, resulting in difficulty
with supination. Because the radial nerve sends branches to the triceps before its entrance into
the spiral groove, elbow extension is unaffected (although it may be weak due to pain resulting
from the fracture).
8. A 7-year-old male is brought in for evaluation of elbow pain after a fall on an outstretched
hand. His x-ray is shown. Which of the following is the most likely diagnosis? (Fig. shown below)
pg. 726
A. Radial head fracture
B. Olecranon fracture
pg. 727
Answer E. Supracondylar fractures are the most common pediatric elbow fractures. They
frequently occur after a fall on an outstretched hand with the elbow “locked” in extension or
hyperextension. The supracondylar region of bone is thin and weak making it prone to fracture.
This patient’s x-ray reveals an anterior and posterior fat pad sign as well as a very subtle fracture
of the anterior and posterior cortex (Fig. ). Even without these latter findings, a supracondylar
fracture should be suspected and the patient should be placed into a long arm posterior splint.
Volkmann ischemic contracture is the classic complication of brachial artery injury that can
occur in posteriorly displaced supracondylar fractures.
9. A 26-year-old female presents to the ED with ankle pain. She was playing tag with her kids in
the yard when her foot got caught in an open pipe, snapping it outward as she fell to the
ground. Her x-ray is shown below. Which of the following is true?
Answer E. This woman has an oblique fracture of the distal fibula at the level of the mortise as
well as rupture of the deltoid ligament. It is clear that the deltoid ligament is disrupted because
of the widened medial joint space on the mortise view. Such an injury would be classified as a
pg. 728
Weber class B (supination external rotation) fracture. It is the most common fibular fracture and
results in an oblique injury at the tibio-fibular syndesmosis which is usually disrupted. On the
lateral view, a “spike” seen on the posterior apex of the distal fragment is diagnostic. Since this
patient has widening of the deltoid ligament, her injury is unstable and requires operative
repair. The Maisonneuve fracture is also an eversion injury resulting in a medial malleolar
fracture or rupture of the deltoid ligament in concert with an oblique fracture of the proximal
fibula. Any patient with a medial malleolar fracture or deltoid ligament rupture (medial joint
space widening) as well as lateral displacement of the fibula without a fracture of the distal
fibula should be suspected of having a Maisonneuve fracture and the examination should
include palpation of the proximal fibula.
10. A 51-year-old female presented for evaluation of left wrist pain after a fall on an
outstretched hand. Her x-ray is shown. In the course of her evaluation, it is clear that she has a
neurologic deficit due to an injury to a nerve. Which of the following findings is most likely? (Fig.
below)
pg. 729
Answer A. This patient has an angulated, displaced, comminuted distal radius fracture. These
injuries are associated with median nerve injuries, the most important of which is muscle
weakness in the thumb and index finger. While sensory abnormalities may be present, they are
usually due to a nerve compression and resolve uneventfully. Answers B, C, and D all result from
a radial nerve injury while answer E results from an ulnar nerve injury. Neither of these nerves is
commonly injured in patients with a distal radius fracture.
11. A 28-year-old female presents with a painful left index finger after it was inadvertently
caught in a car door. X-rays are negative for a fracture, but there is a 50% subungual hematoma.
The volar finger pad and paronychial tissues appear intact. Which of the following is true?
A. The nail should be removed to find and repair any nailbed laceration.
C. Trephination alone results in similar cosmetic and functional outcomes as nail removal.
E. Prophylactic antibiotics have been shown to decrease complications and improve outcome.
Answer C. Historically, nail removal with subsequent primary repair of any nailbed laceration
was the standard approach to patients with subungual hematomas. However, patients with
intact nail folds and an intact volar pad do not require nail removal. In such patients,
trephination alone results in identical cosmetic and functional outcomes. Studies to date
suggest that the outcome is the same regardless of the size of the hematoma, which has
sometimes been used to determine which patients require nail removal. Leaving the nail in
place provides a splinting effect that likely maintains apposition of the underlying nailbed while
small lacerations heal. Patients with a clear nail deformity, or a laceration of the nail should have
the nail removed to explore the nailbed. If a laceration is present, 6-0 absorbable sutures or
2-octyl cyanoacrylate glue can be used to achieve closure. Patients with simple subungual
hematomas do not require prophylactic antibiotic treatment. Distal phalanx fractures associated
with subungual hematomas are considered “open” fractures. However, even among such
patients, antibiotic prophylaxis has not improved outcome or decreased the infection rate in
otherwise healthy patients. Admittedly, usage of the language “open fracture” will continue to
encourage antibiotic use in the setting of a distal phalanx fracture, regardless of whether this
practice improves outcomes.
pg. 730
D. Jersey finger: Primary involvement of the ring finger.
Answer D. Mallet finger is the disruption of the extensor tendon at the level of distal
interphalangeal (DIP) joint with or without an associated avulsion fracture of the dorsal base of
the distal phalanx. It is caused by a flexion force on the volar tip with an extended DIP joint.
Conversely, jersey finger results from avulsion of the FDP tendon at the level of the DIP joint. It
most commonly occurs when an extension force is applied to a flexed DIP such as that occurs
during tackling another player in football by grabbing his jersey. The ring finger is involved in
75% of cases. In contrast to the mallet finger, surgical repair is the treatment of choice in nearly
all cases. A Bennett fracture is an intraarticular fracture of the base of the thumb metacarpal (at
the carpometacarpal joint) with lateral displacement and retraction of the distal segment due to
the abductor pollicis longus. Such fractures require thumb spica splinting and frequently need
operative fixation. Trigger finger refers to a stenosing flexor tenosynovitis typically due to
overuse. It results in the formation of a nodule in the flexor tendon sheath, which prevents
extension of the digit at the level of the metacarpophalangeal (MCP) joint. It most commonly
occurs in the ring and long fingers, and local corticosteroid injection usually results in significant
improvement. Patients should then be splinted in extension and referred to a hand specialist for
further evaluation. Gamekeeper thumb is an avulsion injury of the ulnar collateral ligament
(UCL) at the thumb–MCP joint. It most commonly occurs during a skiing accident in which the
patient’s thumb is trapped in the loop of the pole, resulting in forced abduction and extension of
the thumb. An avulsion fracture may also occur at the site of the UCL insertion. Patients should
be placed in a thumb spica splint and referred to a hand surgeon for further evaluation. Volar
plate entrapment may occur with dorsal proximal interphalangeal (PIP) dislocations preventing
ED reduction.
13. Which of the following is the correct diagnosis of the injury shown in Figure below?
A. Trigger finger
B. Mallet finger
C. Bennett fracture
D. Jersey finger
pg. 731
Answer B. Mallet finger is a disruption of the extensor tendon at the level of distal
interphalangeal (DIP) joint with or without an associated avulsion fracture of the dorsal base of
the distal phalanx. It is caused by a flexion force on the volar tip with an extended DIP joint and
may commonly occur during ball sports in which the participant may describe it as a jammed
finger. Patients should be splinted in extension with either a padded aluminum splint applied to
the dorsal aspect of the distal phalanx or an unpadded aluminum splint applied to its volar
aspect. There is some disagreement regarding the optimal management of mallet finger when it
involves a fracture. Some advocate operative repair if subluxation is present whereas other
authors opt for splinting in extension for 6 to 8 weeks.
14. A 42-year-old male presented for evaluation of posterior shoulder pain after a fall backward
from a ladder. X-rays reveal a scapular fracture. Which of the following is true?
A. Associated thoracic injury is usually more serious than the fracture itself.
pg. 732
Answer A. Scapular fractures occur in patients with high-force blunt trauma such as motor
vehicle crashes or falls from heights. Because of its well-protected position, isolated fracture of
the scapula is rare, and associated injuries to the chest and upper extremity are present in the
large majority of cases. Operative repair of the scapular fracture is not usually necessary as most
patients heal with conservative management and range-of-motion exercises. Patients with
scapular fractures hold their ipsilateral arm in full adduction and any movement elicits extreme
pain. Bilateral scapular fracture is uncommon and usually signifies life-threatening injuries.
Answer D. The position described is called the intrinsic plus position or the safe hand position.
When the MCP joint is extended, the collateral ligaments are relatively flaccid, while they are
stretched during flexion. If patients are splinted with the MCP joints in extension, contraction of
the collateral ligaments occurs and patients may not be able to fully flex their digits once the
splint is removed. In contrast, extension of the IP joints ensures that its collateral ligaments will
also be taut.
16. Which of the following muscles is most commonly injured in rotator cuff tears?
A. Supraspinatus
B. Infraspinatus
C. Subscapularis
D. Teres minor
E. Deltoid
Answer A. The rotator cuff is a group of muscles composed of the supraspinatus, infraspinatus,
teres minor, and subscapularis (SITS). Its primary function is to stabilize the shoulder joint. It is
most commonly injured chronically with repetitive motions or acutely with a fall on an
outstretched hand. The supraspinatus is the most common of the rotator cuff muscles to be
pg. 733
injured. Physical examination of patients with supraspinatus tears indicates inability to maintain
active abduction at 90 degrees without limitation in passive range of motion. Diagnosis may be
confirmed with MRI and acute tears may require early surgery. The deltoid muscle is not
technically part of the rotator cuff.
17. An 85-year-old female presents with right shoulder stiffness. She was diagnosed with a
shoulder sprain 3 weeks ago after a fall, and has been wearing a sling since then. Physical
examination demonstrates an afebrile patient with painful, restricted range of motion of the
shoulder in all directions. Which of the following is the most likely diagnosis?
B. Adhesive capsulitis
C. Septic arthritis
E. Rheumatoid arthritis
Answer B. Limited range of motion in the shoulder in patients with preceding trauma is most
likely to be due to adhesive capsulitis (also called frozen shoulder). Adhesive capsulitis is
characterized by stiffness with or without pain in all directions of shoulder movement.
Prevention and treatment of adhesive capsulitis after shoulder injury are accomplished by range
of motion exercises. Rotator cuff tear is usually heralded by significant pain when the arm is
moved in a particular direction. Septic arthritis usually involves a febrile patient with extreme
pain on any movement of the joint. Scapular fracture is rare in patients without high-force
mechanisms such as motor vehicle crashes or falls from height. Rheumatoid arthritis would be
unlikely in an 85-year-old without prior history.
18. A 46-year-old female presents with right ankle pain after a car accident. Her x-ray is shown.
Which of the following is true (Fig. below)?
A. A femoral nerve block provides the best anesthesia for further treatment
D. The patient should receive hip films to ensure there is no occult fracture
pg. 734
Answer B. The patient’s x-ray reveals an ankle fracture dislocation. Ankle dislocations are
sometimes relatively easy to reduce because they tend to occur as a result of high-energy
mechanisms of injury that result in considerable damage to connective tissues creating laxity
within the joint. However, in muscular patients with less disruption, flexing the knee to 90
degrees is helpful as it releases the tension on the gastrocnemius making ankle reduction easier.
This is best achieved by having the patient sit up on the side of the bed or by flexing the hip and
knee with the patient supine, while having an assistant hold counter traction in the popliteal
fossa. The best regional nerve block for ankle procedures is a sciatic nerve block performed at
the level of the popliteal fossa. The femoral nerve block is most commonly used for knee surgery
and procedures of the upper leg. While nearly any injury affecting the ankle can be splinted for
follow-up, an ankle dislocation with skin tenting must be reduced emergently to avoid skin
necrosis which creates an open fracture.
19. A 55-year-old female presents with wrist pain after falling on her outstretched right hand. A
lateral wrist radiograph is shown in Figure below. Which of the following is the most likely
diagnosis?
A. Scaphoid fracture
B. Lunate dislocation
C. Perilunate dislocation
pg. 735
Answer B. The lateral wrist radiograph demonstrates volar displacement and angulation of the
lunate relative to the radius, indicating a lunate dislocation. The “spilled teacup” sign is present
with the lunate appearing as a teacup that is tilted forward. Treatment involves orthopedic
consultation and urgent surgical repair. Scaphoid fractures are best seen on an anteroposterior
(AP) wrist or dedicated scaphoid view. Perilunate dislocation involves the capitate dislocating
relative to the lunate and radius. Distal radius fractures and metacarpal fractures are best seen
on AP views.
20. A 62-year-old male presents after a motor vehicle collision with severe ankle pain after his
left foot got caught under the accelerator. His foot is shown in Figure below. Which of the
following is likely to be true regarding this patient?
pg. 736
E. Pain control is rarely needed due to associated nerve injury.
Answer B. The appearance of the foot and ankle is highly indicative of an ankle dislocation. The
skin between the foot and the ankle appears tented and the foot is rotated relative to the rest
of the leg. Ankle dislocations almost always involve a fracture of one or more of the bones in the
mortise (malleoli, talus). With this degree of deformity, simple ankle sprain is highly unlikely.
Nerves injured in ankle dislocation include the tibial, superficial peroneal, and sural. Emergent
closed reduction with splinting is the most appropriate initial management. If there is no
neurovascular compromise, it is reasonable to wait for plain radiographs to perform closed
reduction. Definitive care usually requires operative management. Pain control is absolutely
indicated as total anesthesia with ankle dislocations is rare.
B. State x-rays are not needed in patients with isolated patella tenderness
C. State x-rays are required in patients complaining of knee pain who walk with a significant limp
pg. 737
E. Were developed to determine which patients need x-rays among those with chronic knee
pain
Answer D. The Ottawa knee rules were derived to help physicians determine which patients
need x-rays in the setting of acute knee trauma. The rules state that a patient needs imaging if
any of the following are present:
• Age ≥55
• Inability to bear weight or walk four steps both at the time of the accident and in the
emergency room (limping is allowed)
The criteria have been prospectively validated and are nearly 100% sensitive in identifying
patients with clinically significant fractures. Successive studies have validated the use of the
rules in children as young as 2 years old, though given the difficulty in examining very young
children, the clinical utility of the rules in children younger than 5 has been questioned.
C. A 58-year-old diabetic factory worker with an amputation of his index finger at the
metacarpophalangeal (MCP) joint.
D. A 23-year-old laboratory worker with a middle finger amputation at the level of his middle
phalanx who stored his finger packed in dry ice for the last 12 hours.
Answer A. In general, indications for replantation include multiple digit amputations, thumb
amputations, wrist and forearm amputations, single digit amputations between the proximal
interphalangeal (PIP) and distal interphalangeal (DIP) joints, and any pediatric amputation.
Amputations distal to the DIP joint are typically debrided and closed along with amputations
proximal to the PIP. Amputations that are between the PIP (distal to the flexor digitorum
superficialis insertion) and DIP joints tend to do well. However, single digit amputations are
often considered unnecessary to replant. In addition, patients with underlying vascular disease,
diabetes, congestive heart failure, or other medical problems may not have a good outcome
even when the characteristics of the amputation are encouraging for replantation. Finally, all
pg. 738
patients with amputation should wrap the amputated digit in saline-soaked gauze, place the
gauze in a ziplocked bag, and place the bag on ice or in an ice-water bath. Freezing the digit
should be avoided as it results in irreversible damage to cellular structures due to ice crystal
formation.
23. A 10-year-old male presents with elbow pain after falling off the bed. Radiographic
visualization of which of the following usually indicates an occult elbow fracture?
Answer B. Initial radiographs of pediatric patients with elbow trauma may not clearly
demonstrate a fracture. The posterior fat pad, which is closely applied to the posterior portion
of the humerus, is normally invisible in the intact elbow. Blood in the elbow joint due to fracture
will cause this fat pad to show up as a dark line just posterior to the supracondylar region of the
humerus. The anterior fat pad may be present in the intact elbow as a dark line, but
enlargement of this into a sail shape indicates likely injury. On anteroposterior view, the
Baumann angle is formed by a line parallel to the capitellar growth plate and a line parallel to
the long axis of the humerus. A normal Baumann angle is 75 degrees, and bilaterally equal
Baumann angles reduce the likelihood of unilateral fracture. On a lateral view of the normal
elbow, the anterior humeral line should bisect the capitellum. With a supracondylar fracture,
the anterior humeral line lies anterior to the midpoint of the capitellum.
24.A 30-year-old female presents for evaluation of left elbow pain after a fall on an outstretched
hand. She reports her arm was “locked” at the time of the fall. She rested the elbow overnight
but awoke in the morning with increased pain, swelling, and decreased range of motion (Fig.
below). Which of the following is the most likely injury?
A. Supracondylar fracture
B. Olecranon fracture
D. Radial–ulnar dislocation
pg. 739
Answer C. Radial head fractures are common fractures in adults who have fallen on an
outstretched hand. The force of the injury is directed axially through the radius, impacting the
radial head against the capitellum of the humerus. In children, these injuries most frequently
result in supracondylar fractures, but supracondylar fractures are rare in adults, in whom the
radial head is the weakest link in the chain. The patient’s x-ray reveals both an anterior and
posterior fat pad sign indicative of a hemarthrosis as well as a subtle lucency at the radial head
suggestive of an undisplaced fracture (Fig.below). Even in the absence of the subtle fracture on
x-ray, a radial head fracture should be suspected based on the patient’s symptoms and the
anterior and posterior fat pads on x-ray. A child with the same symptoms and x-ray most likely
has a supracondylar fracture. Olecranon fractures usually result from a direct blow to the elbow,
while a sprain of the ulnar collateral ligament typically results from a throwing injury.
pg. 740
25. A 26-year-old male is brought to the ED after a motor vehicle accident. He was the
restrained passenger in a jeep traveling 45 mph when the driver lost control and struck a tree.
The patient’s left knee struck the dashboard and he is now complaining of pain in his left knee
and hip (Fig. below). Which of the following is the most likely diagnosis?
Answer D. This patient has a posterior hip dislocation. Posterior hip dislocations account for 90%
of all hip dislocations. The most common mechanism of injury is a “dashboard injury,” in which a
seated patient strikes the dashboard with a flexed knee, driving the femur posteriorly. Due to
the force required to dislocate the well-protected hip joint, posterior hip dislocations are often
associated with multisystem trauma. Patients will present with a shortened leg, with the hip
pg. 741
internally rotated, adducted, and slightly flexed. Posterior hip dislocations must be reduced
emergently due to the high risk of avascular necrosis of the femoral head. Radiographs should
be obtained before reduction, unless a pulse deficit is present.
26. A 45-year-old male presents with locking and clicking of his knee for several days. He twisted
it several weeks ago and did not seek medical care at the time. Physical examination
demonstrates no knee instability or patellar tenderness. His knee clicks with flexion and
occasionally gets locked just before full extension. Which of the following structures is most
likely injured?
C. Medial meniscus
D. Patella tendon
Answer C. Locking or clicking of the knee is often due to a meniscal injury, usually medial.
Patients may not recall a specific traumatic event and chronic, repeated trauma may predispose
to the injury. The medial meniscus is less mobile and therefore more predisposed to injury than
the lateral. Knee locking from medial meniscal tears may be relieved by applying valgus stress
and gentle extension.
27. A 22-year-old male presents to the ED with a left posterior hip dislocation sustained in a car
accident. Which of the following neurologic findings is most likely present?
B. Erectile dysfunction
Answer C. Ninety percent of hip dislocations are posterior hip dislocations, typically caused by
high velocity MVCs in which a patient’s knee strikes the dashboard, resulting in posterior
directed forces through the hip. Approximately 10% of cases are complicated by a sciatic nerve
palsy, which travels posterior to the hip joint. The peroneal division of the nerve is more
commonly affected than the tibial branch, though both are commonly affected. This results in
pg. 742
weakness of ankle dorsiflexion and foot eversion, as well as decreased sensation along the
entire posterior lower leg (below the knee). The skin of the posterior upper leg is innervated by
the posterior femoral cutaneous nerve, which is derived from the lumbosacral plexus. With
timely reduction, patients with sciatic nerve palsies typically recover without any residual
deficits. The femoral nerve innervates most of the remainder of the leg via its many branches,
including cutaneous innervations to the anterior thigh and lower leg, as well as the
posteromedial lower leg via the saphenous nerve.
28. A 78-year-old previously healthy woman presents with right hip pain after a fall. Her hip
x-ray reveals a right femoral neck fracture. Which of the following is the most important factor
in returning the patient to optimal mobility?
Answer D. Minimizing the time to operation for geriatric patients significantly improves their
postoperative outcomes and is the basis for geriatric fracture programs throughout the country.
Screening questions targeted to suicidality, depression, HIV, and others in the ED have not been
shown to improve the outcome of patients with such obvious serious pathology and can delay
access to care. Preoperative physical therapy or medication reconciliation will likely have little
effect on outcomes compared to early surgical management for these patients. Postoperative
primary care follow-up is certainly necessary but will not likely be more effective than
postoperative orthopedic or physical therapy follow-up.
29. A 23-year-old male presents with shoulder pain after falling on his left shoulder. Physical
examination demonstrates tenderness in his lateral clavicle. He is able to touch his opposite
shoulder with his left hand. There are no neurovascular deficits (Fig. below). Which of the
following is the most likely diagnosis?
D. Acromioclavicular separation
E. Sternoclavicular dislocation
pg. 743
Answer D. The patient has a prominent lateral clavicular border with the deltoid and humeral
margin displaced inferiorly. The most likely cause for this is an acromioclavicular separation due
to a torn acromioclavicular ligament. Radiographs will demonstrate this injury more definitively.
Acute management involves sling immobilization and orthopedic follow-up. If there were a
shoulder dislocation, it would be impossible for the patient to touch his opposite shoulder with
the affected hand. A sternoclavicular dislocation is not apparent due to the lack of abnormalities
noted on the medial sternal border.
30. A 35-year-old male complains of progressively worsening right leg pain 2 hours after
undergoing a closed reduction in the ED for a mid-shaft tibia fracture. After the splint is
removed, physical examination reveals a firm, tender compartment of his right anterior leg with
diminished sensation in the area. Pulses in the foot are 2+. Compartment pressure is 30 mm Hg.
The patient’s vital signs are T 98.8°F, P 98, RR 18 BP 110/55 SaO2 98% on RA. Which of the
following is the most appropriate course of action?
pg. 744
Answer E. Compartment syndrome refers to ischemia that occurs in the extremities when
pressure in the soft tissues exceeds that of the microcirculation. Most experts use a definition
based on the difference between the diastolic pressure and the compartment pressure. When
the diastolic pressure minus the compartment pressure (called the acute compartment
syndrome [ACS] delta pressure) is <30, a fasciotomy should be considered. This patient’s ACS
delta pressure is 25 (55 − 30 = 25). Normal compartment pressure is zero, and pressures >30
mm Hg are usually enough to predispose to compartment syndrome. Long bone fractures are
the usual reason for compartment syndrome, causing extravasated blood and soft tissue edema
to accumulate. The tibia is the most commonly affected bone. The most common symptoms are
pain and paresthesias. Diminished pulses generally occur only in extremely advanced cases, as
pressure in compartment syndrome is usually well below arterial pressure. For this reason,
ankle–brachial indices are usually normal in compartment syndrome unless there is
simultaneous arterial insufficiency. Patients with compartment syndrome should not be
discharged. Imaging may help delineate the cause of the compartment syndrome, but diagnosis
is still clinical. Diagnosis is made by directly measuring compartment syndromes with a Stryker
needle device. Treatment is with urgent fasciotomy.
31. A 75-year-old male presents with a fall on an outstretched hand. Radiographs demonstrate a
distal radius fracture with dorsal displacement of the distal segment. Which of the following is
the most likely nerve injury?
A. Median
B. Radial
C. Ulnar
D. Axillary
E. Vagus
Answer A. The patient has a Colles fracture, which puts him at risk for median nerve injury.
Injury to this nerve can result from either the fracture itself or during therapeutic reduction.
Falling on an outstretched hand is the most common mechanism for Colles fracture. A volarly
displaced distal radius fracture is referred to as reverse Colles or Smith fracture, which usually
results from trauma to the dorsum of the hand. Smith fracture also involves the median nerve
more often than other peripheral nerves of the upper extremity. An intra-articular fracture of
the distal radius with displacement of the carpal bones is referred to as a Barton fracture.
A. The highest incidence of avascular necrosis (AVN) occurs in patients with intertrochanteric
fractures.
pg. 745
B. Isolated fractures of the lesser trochanter are most commonly seen in young adults.
D. The development of acute respiratory distress in a patient with an acute femoral shaft
fracture but no other trauma is most likely due to blood loss into the soft tissues of the thigh.
Answer B. Femoral neck fractures and intertrochanteric fractures account for 90% of all hip
fractures. They both most commonly occur in osteoporotic elderly patients after a low-energy
fall. In contrast, young patients develop these fractures in the setting of high-energy trauma
such as in highspeed motor vehicle accidents. Therefore, many of those patients have evidence
of multisystem trauma. The intertrochanteric femur has a better blood supply than the femoral
neck, resulting in a much smaller incidence of AVN. As many as 40% of patients with femoral
neck fractures may develop AVN. The use of a femoral nerve block in the setting of a femur
fracture is an attractive means of delivering adequate pain control to patients. Although it has
not been widely used in the United States, several recent papers demonstrate that it is an
effective, safe means of pain control in both pediatric and adult patients. Isolated fractures of
the lesser trochanter almost always occur in young adults. The fracture represents an
apophyseal avulsion due to a forceful contraction of the iliopsoas muscle. A similar injury can
occur at the greater trochanter. Patients with isolated lesser trochanter fractures are normally
able to ambulate, although they will complain of pain. Physical examination will demonstrate a
patient’s inability to lift the affected leg from the floor while in a seated position (iliopsoas
insufficiency). Patients will also have pain when they are asked to flex their hip against
resistance. PE is most common in the postoperative patient. Acute respiratory distress is most
likely because of fat embolism, which complicates 2% to 23% of patients with isolated femoral
shaft fractures. Acute respiratory distress, associated with altered mental status, tachycardia,
and fever are hallmarks of the illness. Although diffusely scattered petechiae are nearly
pathognomonic for the syndrome, they do not normally occur until late in the course of the
illness and only occur in 50% of patients.
33. Which of the following is the most common area to be affected in compartment syndrome?
A. Upper arm
B. Forearm
C. Hand
D. Thigh
E. Lower leg
pg. 746
Answer E. Compartment syndrome refers to ischemia that occurs in extremities when pressure
in the soft tissues exceeds that of the microcirculation. Such high pressures occur from either
increased content in the compartment or external compression. Normal compartment pressure
is zero, and pressures >30 mm Hg are usually enough to predispose to compartment syndrome.
Long bone fractures are the usual reason for compartment syndrome, causing extravasated
blood and soft tissue edema to accumulate. Tibial fracture is the single most common cause of
compartment syndrome, but all the other regions listed can also be involved. The most common
symptoms are pain and paresthesias. Diminished pulses generally occur only in extremely
advanced cases, as pressure in compartment syndrome is usually well below arterial pressure.
Diagnosis is made by directly measuring compartment syndromes with a Stryker needle device.
Treatment is with urgent fasciotomy.
34. In patients with tibial shaft fractures, which of the following is the most common associated
finding?
A. Fibular fracture
Answer A. Patients with tibial shaft fractures usually have concomitant fibular fractures as well,
due to the close proximity of the two bones. Mechanism of injury usually involves direct trauma;
only rarely do pathologic fractures occur in the tibia. Due to the sparse soft tissue surrounding
the anterior surface of the tibia, fractures are often open and require emergent operative
repair. The most common site for compartment syndrome is in the leg, due to tibial fracture.
The common peroneal nerve is the most common nerve to be injured in tibial fractures, but this
does not occur in most patients. Vascular compromise with tibial fractures is rare, but when
present, mandates aggressive management.
C. Are more commonly associated with fractures of the femoral head than anterior dislocations.
D. Are associated with an inability to see the lesser trochanter on an anterior–posterior view.
E. Cause the patient to have an externally rotated, abducted, and shortened leg.
pg. 747
Answer D. Posterior hip dislocations account for 90% of all hip dislocations. The most common
mechanism of injury is a “dashboard injury,” in which a seated patient strikes the dashboard
with a flexed knee, driving the femur posteriorly. Owing to the force required to dislocate the
well-protected hip joint, posterior hip dislocations are often associated with multisystem
trauma. Patients will present with a shortened leg, with the hip internally rotated, adducted,
and slightly flexed. Approximately 10% of posterior hip dislocations are associated with sciatic
nerve injury, which is usually a neurapraxia. After reduction, this is manifest by hamstring
weakness as well as weakness of all lower leg muscles, loss of ankle reflex, and hypoesthesia of
the posterior thigh and complete lower leg. Posterior hip dislocations must be reduced
emergently because of the high risk of avascular necrosis of the femoral head. Films should be
obtained before reduction to determine the presence of associated femur or pelvis fractures,
however, unless a pulse deficit is present. Femoral head fractures are much more common in
anterior femoral dislocations, occurring in as many as 77% of patients versus only 10% of
patients with posterior dislocations. Because the femur is internally rotated and adducted, the
shadow of the lesser trochanter is not seen on an AP projection. This is one critical tool to help
differentiate posterior dislocations from anterior dislocations. Another is to examine the size of
the femoral head. Because the posteriorly dislocated femoral head is closer to the x-ray
cassette, it often appears smaller than the unaffected femoral head.
36. What is the most commonly injured ligament during ankle sprains?
A. Anterior talofibular
B. Calcaneofibular
C. Posterior talofibular
E. Deltoid
Answer A. The lateral or fibular collateral ligament complex comprises three ligaments that tend
to rupture in an anterior to posterior sequence during ankle sprains. The anterior talofibular
ligament is the weakest, and rupture results in a positive anterior ankle drawer test. The
calcaneofibular is next and the posterior talofibular is the most posterior portion of the fibular
collateral ligament. The deltoid ligament is 20% to 50% stronger than its lateral counterpart and
is infrequently injured in isolation. The anterior inferior tibiofibular ligament is the weakest
ligament of the four syndesmotic ligaments that attach the distal tibia and fibula. The
syndesmosis prevents displacement of the tibia and fibula relative to one another, and
disruption can contribute to significant instability.
37. An 8-year-old male presents with wrist pain after a fall on his outstretched right hand (Fig.
below). Which of the following is the correct type of injury?
pg. 748
A. Salter–Harris I
B. Salter–Harris II
C. Salter–Harris III
D. Salter–Harris IV
E. Salter–Harris V
Answer C. The x-ray demonstrates a Salter–Harris III fracture of the distal radius. The
Salter–Harris classification is used to describe pediatric long bone fractures near the growth
plate. Type I fractures go through the physis only, type II from the metaphysis into the physis,
type III from the epiphysis into the physis, type IV is a combination of types II and III, and type V
is a crush injury to the physis. The most common is type II. Types I and V may be invisible on
initial plain films. Type V carries the poorest prognosis.
38. A 35-year-old male is struck in his leg by the bumper of a car. He is tender just distal to his
knee. His knee X-ray is shown (Fig. below). Which of the following is true regarding this injury?
pg. 749
A. Compartment syndrome is a possible complication
Answer A. The x-ray reveals a tibial plateau fracture. These injuries usually require operative
management. Given the location in the tibia, compartment syndrome is definitely possible and
should be guarded against. Lateral collateral ligament injuries are more common than medial
collateral ligament injuries, but neither is seen in most cases. Neurologic injuries can certainly
occur, but are not seen in the majority of cases. Lateral tibial plateau fractures represent the
majority of cases. Radiographs can be negative with tibial plateau fracture up to 20% of the
time.
39. Which of the following is the most common complication of anterior shoulder dislocation?
C. Recurrence
pg. 750
E. Adhesive capsulitis
Answer C. Patients with anterior shoulder dislocation and subsequent relocation are at
extremely high risk of recurrent dislocation. Young patients are the highest risk group, probably
because of a combination of associated cartilaginous injury and overly aggressive return to
previous activity. Surgical stabilization is recommended in these patients to prevent recurrence.
The other answer choices are all complications of anterior shoulder dislocation but occur less
commonly than recurrent dislocation.
40. A 28-year-old left-hand–dominant man presents with left-hand pain and swelling for 1 day.
His hand is shown in Figure 8-1. He states he hurt his hand after falling down the stairs. An x-ray
is negative for fracture. Which of the following is the next best step in management?
C. Doxycycline PO
D. Ampicillin–sulbactam IV
pg. 751
Answer D. Injuries to the metacarpals of the ring and little digits commonly occur after punching
someone in the face, the so-called fight bite. Metacarpal joint space violation can be serious and
the range-of-motion limiting if not treated properly. Optimal management is with
broad-spectrum antibiotics and careful wound exploration to ensure that no foreign bodies or
tendon injuries are present. Wounds are always left open to heal by secondary intention to
avoid the significant possibility of infection if primary closure is performed. Oral antibiotics are
inferior to intravenous antibiotics with this injury. Splinting the joint is indicated if x-ray
indicates a fracture, but the warm wrapping of the splint could increase the possibility of
infection. In this case, an ulnar gutter splint is more appropriate than a radial gutter.
41. Which of the following radiographic views of the knee best identifies longitudinal patellar
fractures?
A. Anteroposterior (AP)
B. Lateral
C. Medial oblique
D. Lateral oblique
E. Sunrise
Answer E. Patellar fractures usually occur due to direct blows to the patella from falls or motor
vehicle collisions. They may be invisible on standard AP and lateral views of the knee, and
patellar sunrise views may be necessary for diagnosis. Transverse patellar fractures are the most
common type, but longitudinal fractures also occur and are easily seen on the sunrise view.
42. A father and his daughter each present to the ED complaining of elbow pain after falling
down ice skating. They were holding hands when they lost their balance and each fell to their
outstretched hands. Each of them has limited elbow range of motion on examination but x-rays
reveal only a joint effusion. Which of the following is true?
C. Both the father and the daughter should be placed in a long-arm posterior mold with plaster
or fiberglass
pg. 752
Answer B. Adults with elbow pain, limited range of motion, and a joint effusion after a fall on an
outstretched arm typically have an occult, undisplaced radial head fracture. Such injuries are
rarely operative, and treatment with a sling and early mobilization is preferred. However,
children with the same symptoms and mechanism of injury typically suffer a supracondylar
fracture with a higher rate of morbidity and need for operative repair. Therefore, children
should be placed in a long-arm posterior mold with plaster or fiberglass. Compartment
syndrome can complicate supracondylar humerus fractures in children but are more common
when there is an associated forearm bone fracture.
43. A 22-year-old male presents with severe left elbow pain and swelling after being tackled in a
football game (Fig. below). His elbow appears to be locked in partial flexion and he guards
against any motion. X-rays reveal the image shown. Which of the following is the most common
complication of this injury?
D. Compartment syndrome
Answer B. The image reveals a posterior elbow dislocation. Brachial artery disruption or injury is
the most serious complication of posterior elbow dislocations, but ulnar nerve injuries are the
most common complication. Median nerve injuries are the second most common associated
injuries and may occur in concert with ulnar nerve injuries. Decreased function in the
distribution of either the ulnar, median, or radial nerves after reduction is an indication for
surgical exploration and decompression. Postreduction functional loss most commonly occurs
because of entrapment of the median nerve. Functional loss that exists before reduction is most
pg. 753
commonly neurapraxia and spontaneous recovery is the rule. Therefore, such injuries should be
well documented and followed by close outpatient observation. The brachial artery is the most
commonly injured vascular structure in posterior elbow dislocations. Although the presence of a
radial pulse is reassuring, it does not ensure an intact brachial artery, particularly in the setting
of a compartment syndrome. Therefore, physicians should have a low threshold to perform
angiography on individuals at risk for brachial artery injury. The presence of a distal pulse deficit
mandates exploration and repair. A compartment syndrome at the elbow may result in
Volkmann ischemic contracture due to ischemia, injury, and fibrosis of forearm structures. In its
most severe form, Volkmann ischemic contracture results in elbow flexion, forearm pronation,
wrist flexion, thumb adduction, metacarpophalangeal joint extension, and finger flexion.
44. While watching her grandson’s tackle football game, a 72-year-old female presents with
right knee and leg pain after being struck by an opposing player. She was knocked over and has
had pain on ambulation. She is tender on the lateral aspect of her proximal anterior leg, just
distal to her patella. An X-ray of her knee is shown (Fig. below). Which of the following is the
next best step in management?
A. Weight-bearing as tolerated
pg. 754
Answer D. The x-ray is without significant abnormality, but the patient is at significant risk for
occult tibial plateau fracture. Radiographs can be negative with tibial plateau fracture up to 20%
of the time. The optimal management for suspected tibial plateau fractures is
non–weight-bearing and repeat x-ray imaging in 1 week or CT scan without contrast to evaluate
immediately. In a 72-year-old female, non–weight-bearing can be extremely inconvenient and
difficult, so immediate CT is preferred. CT angiogram is not needed in this case, and would be
more useful with suspected knee dislocation. Delayed weight-bearing recommendations would
not be appropriate for this patient with suspected tibial plateau fracture. X-ray of the ankle is
not warranted if there is no specific ankle pathology noted.
45. In an otherwise normal x-ray, the finding of a “posterior fat pad” on a lateral x-ray of the
elbow in adult and pediatric patients is suggestive of which of the following?
Answer C. In the setting of trauma, >90% of patients with a “posterior fat pad” sign have an
intraarticular elbow injury. The most probable injury in children is a supracondylar fracture,
whereas radial head fractures are the most common entity in adults. Although a small anterior
fat pad is a common finding in healthy patients, a posterior fat pad is always an abnormal
finding.
46. Which of the following Salter–Harris fractures carries the poorest prognosis?
A. Type I
B. Type II
C. Type III
D. Type IV
E. Type V
Answer E. The Salter–Harris classification is used to describe pediatric long-bone fractures near
the growth plate. Type I fractures go through the physis only, type II from the metaphysis into
pg. 755
the physis, type III from the epiphysis into the physis, type IV is a combination of types II and III,
and type V is a crush injury to the physis. The most common is type II. Types I and V may be
invisible on initial plain films. Type V carries the poorest prognosis.
47. A 22-year-old male presents with forearm pain after being assaulted. Radiographs
demonstrate a proximal ulnar fracture with dislocation of the radial head. Which of the
following is the most likely nerve injury?
A. Median
B. Radial
C. Ulnar
D. Axillary
E. Brachial
Answer B. The patient has evidence of a Monteggia fracture. Proximal ulnar fracture along with
dislocation of the radial head from the capitellum (which may be subtle) is the specific finding.
The typical mechanism is a blow to the forearm or fall on outstretched hand. Significant
displacement of the radial head can put the patient at risk for radial nerve injury, exhibited by
wrist drop. Treatment is surgical in most cases.
48. Which of the following is one of Kanavel cardinal signs of flexor tenosynovitis?
Answer E. Flexor tenosynovitis is a purulent infection of the flexor tendon sheath that usually
results from a penetrating wound of the digital flexor surface. The infection spreads unimpeded
throughout the sheath and the patient presents with acute pain and signs of inflammation in the
involved digits. Kanavel’s four cardinal signs of flexor tenosynovitis include all of the signs listed,
although tenderness along the flexor tendon sheath is frequently cited as the most significant of
these findings. Although early infections may be treated with intravenous antibiotics directed at
Staphylococcus, incision and drainage is often required. Due to the limited space in the sheath,
purulent infections may cause a rapid increase in compartmental pressure resulting in ischemia
pg. 756
to tendons and nerves. A hand specialist should be consulted as soon as the diagnosis is
seriously considered.
49. A 44-year-old female presents with right leg pain. She has noticed increasing pain in the
middle of her tibia over the last few weeks since she started training for a marathon. Physical
examination of the entire lower extremity is normal except for mild tenderness to palpation in
the area of pain. Radiographs of the tibia are normal. Which of the following is true regarding
this patient’s condition?
C. Magnetic resonance imaging (MRI) is the test of choice to differentiate shin splints from stress
fractures.
Answer C. While this patient could have a stress fracture, her presentation is most consistent
with shin splints (medial tibial stress syndrome). Like stress fractures, shin splints are due to
overuse, and are more common in women than men. While the tibia is the obvious bone
involved in shin splints, it is also the most common involved bone in patients with stress
fractures (metatarsals are second most common). Radiographs are normal in patients with shin
splints, and are most often normal in patients with stress fractures. Subacute radiographs (3 to 4
weeks) of patients with stress fractures are abnormal up to 50% of the time. Bone scan has
much higher sensitivity than radiographs, but a much lower specificity. Therefore, MRIs have
become the test of choice to differentiate shin splints from true stress fractures if the diagnosis
is unclear. However, advanced imaging is not indicated in the vast majority of cases, as
management is rarely affected and the diagnosis is most often clinical. Standard treatment with
rest (though patients can continue to run at reduced mileage), ice, and nonsteroidal
anti-inflammatory drugs (NSAIDs) results in resolution of almost all cases of shin splints
syndrome. Stress fractures require more prolonged rest, participation in nonimpact activities
only, and a monitored, more graduated return to running.
50. Which of the following is the most sensitive physical examination test for an anterior
cruciate ligament (ACL) tear?
A. Anterior drawer
B. Posterior drawer
C. Lachman
pg. 757
D. McMurray
E. Thompson
Answer C. The ACL is the most commonly injured ligament in the knee which requires surgery.
Diagnosis is often made on history, with acute knee swelling and audible “pop” after twisting or
lateral force to the knee. Lachman test is the most sensitive acute physical examination test to
evaluate for an ACL tear in the acute setting. It involves placing the knee in 20 to 30 degrees
flexion and pulling anteriorly on the leg while holding the distal thigh stable and observing for
laxity relative to the contralateral knee. The anterior drawer test is another test for the ACL
which is not nearly as sensitive as the Lachman, especially acutely. It is important for the
emergency physician (EP) to remember that the Lachman is not 100% sensitive in the acute
setting due to limited range of motion from joint effusion. The posterior drawer test is used to
assess the posterior cruciate ligament (PCL), which is rarely injured. The McMurray test assesses
the medial meniscus. The Thompson test checks for integrity of the Achilles tendon.
51. An 80-year-old female presents with a distal radius fracture of her nondominant hand after a
trip and fall. She did not strike her head and this is her first fall ever. She has no other injuries
and takes no other medications. Her only medical history is chronic intermittent constipation.
Which of the following is the most appropriate pain management regimen for her?
Answer D. Scheduled acetaminophen is the safest and most effective pain management regimen
in the older adult. Scheduled therapy is preferable to as needed therapy because older adults
tend to limit use of pain medications unnecessarily when prn prescriptions are written. In
addition to agerelated decreases in creatinine clearance, NSAID therapy carries a high risk of
renal injury as well as atrial fibrillation and cardiovascular ischemia. While opiate therapy is
reasonable in specific cases, low-dose hydrocodone is preferred to oxycodone and always
should be prescribed with concomitant stool softeners (especially in this chronically constipated
patient).
52. A 23-year-old male presents with difficulty breathing, altered mental status, and a petechial
rash. He suffered a proximal tibia fracture the previous day after being kicked by a horse. Which
of the following is the most likely diagnosis?
pg. 758
A. Meningococcemia
B. Fat embolism
C. Pulmonary thromboembolism
D. Pneumothorax
E. Pneumonia
Answer B. Fat embolism can occur after fracture of any long bone, usually in the lower
extremities. Fat droplets from the bone marrow reach the systemic circulation and can embolize
to the lungs, brain, eyes, and extremity microvasculature. Respiratory distress, altered mental
status, fever, and petechiae commonly occur. Diagnosis is aimed at ruling out other causes of
symptoms, and treatment is primarily supportive. Meningococcemia can occur acutely and
cause altered mental status and petechiae, but the history of the leg fracture and the
respiratory distress point away from this. PE due to thrombus can occur in the post-trauma
patient but usually occurs later in the course of the recovery. Pneumothorax and pneumonia in
a previously healthy patient are unlikely to cause altered mental status or petechiae.
53. A 34-year-old male presents with severe left knee pain after a motor vehicle collision. He has
a large joint effusion and his knee is unstable in all directions. Plain radiographs demonstrate no
fracture. Pulses in the left lower extremity are normal. Which of the following is the most
appropriate next step in management?
B. MRI knee
C. CT angiography
D. CT knee
E. Arthroscopy
Answer C. Patients with complete knee instability after significant trauma are suspicious for
having a knee dislocation. The knee can spontaneously relocate and be mistaken for a severe
sprain with large effusion. The popliteal artery is at high risk for damage, and angiography
should be performed in every patient with knee dislocation. The presence of pulses in the foot
does not rule out popliteal artery injury, as up to 10% of patients with popliteal injury will have
intact peripheral pulses. The most common nerve injury in knee dislocation is the peroneal
nerve, which may be tested with foot dorsiflexion and dorsal foot sensation.
pg. 759
54. A 44-year-old female presents with right wrist pain. She fell on her hand the previous day
and has pain in her radial wrist. The wrist is tender in the anatomic snuffbox. Plain radiographs
of the wrist are completely normal. Which of the following is the most appropriate next step in
management?
A. Orthopedics consultation
Answer D. Patients with fall on outstretched hand (FOOSH) injuries may have bony or
ligamentous damage in any part of their upper extremity. The bones of the wrist are most
susceptible to injury. The anatomic snuffbox is demarcated by the extensor pollicis brevis and
longus tendons just proximal to the thumb and tenderness in this region indicates possible
fracture to the scaphoid. Scaphoid fracture is particularly dangerous because of the high rate of
AVN and resulting limitation of thumb function. Almost one-fifth of all scaphoid fractures are
invisible on acute radiographs, so appropriate management in suspected cases includes
immobilization in a thumb spica cast and repeat radiographs in 1 to 2 weeks. There is no need
for emergent orthopedic consultation in patients with negative x-rays. Discharging the patient
home without a splint that immobilizes the thumb is contraindicated. Admission for observation
is not an appropriate use of resources.
55. A 32-year-old male presents to the ED after an accident while cleaning a paint sprayer. He
was trying to dislodge a clog in the nozzle when he inadvertently triggered the sprayer with his
index finger over the nozzle. He now comes in with mild pain and a nearly punctate wound at
the tip of his left index finger. Which of the following is the next best step in management?
A. Tetanus prophylaxis, oral antibiotics, splint in the “safe” position, and discharge with
orthopedic follow-up in 2 days
B. Tetanus prophylaxis, irrigate the wound with tap water, oral antibiotics, and discharge with
orthopedic follow-up in 2 days
D. Tetanus prophylaxis, oral antibiotics, incision, and drainage of the volar tip of the finger.
E. Tetanus prophylaxis, oral antibiotics, and digital block with thorough wound exploration to
determine the extent of injury
pg. 760
Answer C. High-pressure injection injuries to the hand are associated with a very high rate of
amputation depending on the injected substance and the rapidity of treatment. Amputation
rates as high as 80% have been reported. The index finger of the nondominant hand is the most
common digit involved. Entrance wounds from high-pressure injection injuries may look
deceptively benign. The jet of liquid is under such high pressure that it easily penetrates the skin
and gains access to the tendon sheaths causing rapid distension and an inflammatory response.
Over time, compartment pressures increase, the inflammatory cascade mushrooms, and the
tissues become ischemic and necrotic. The flexor sheaths of the thumb and index finger (most
commonly involved) extend to the thenar space while the long, ring, and little finger sheaths
extend to the midpalmar space. In addition to tetanus prophylaxis and antibiotics, a hand
surgeon should be consulted immediately for emergent wide incision and debridement to
decompress the hand and eliminate inflammatory debris (e.g., paint or grease). Digital blocks
are contraindicated by ED physicians due to the subsequent increase in compartment pressures.
56. A 43-year-old male presents with a chief complaint of lower leg pain behind his left ankle. He
says he was doing sprints with friends in a morning workout when he felt a sudden “pop” and
excruciating pain in the back of the heel. Which of the following examination findings is most
sensitive for an Achilles tendon rupture?
B. Visualizing ecchymoses and swelling over the Achilles tendon insertion site
C. Absent passive plantar flexion when the knee is flexed to 90 degrees while the patient is lying
prone
D. Diminished or absent ankle plantar flexion when the calf muscles are squeezed
Answer D. The Thompson test, in which ankle plantar flexion is assessed as the calf muscles are
squeezed, is the most sensitive test to predict Achilles tendon rupture. The Matles test, which
assesses the degree of passive (i.e., natural) ankle plantar flexion when the patient is lying prone
with the knee flexed to 90 degrees is also a sensitive and specific test but it is not as predictive
as the Thompson test. Finally, while a definite palpable Achilles tendon defect may be
diagnostic, frequently soft tissue swelling will limit the ability to determine if a defect is present
and the sensitivity and specificity of this test is low.
57. A 24-year-old female presents with an ankle injury while playing soccer. She inverted her
ankle and has pain and tenderness in the lateral malleolar area. She has no tenderness to
palpation in the base of the fifth metatarsal, calcaneus, or proximal fibula. An ankle x-ray reveals
no fracture. She is able to bear weight, albeit with moderate pain. Which of the following is the
most appropriate home care instruction for her?
pg. 761
A. Non-weight-bearing with crutches until seen by orthopedic surgeon
Answer C. The patient likely has a moderate grade lateral ankle sprain. A stirrup-style ankle
brace restricts only inversion and eversion but allows for dorsiflexion and plantarflexion, which
is important in optimal recovery from ankle sprains. Gentle range-of-motion exercises (such as
tracing letters with toes) along with standard rest–ice–compression–elevation management are
also indicated. A lace-up style brace can work as well, but restricts all range of motion in the
ankle joint, potentially slowing recovery. Non-weight-bearing or toe-touch ambulation for ankle
sprains is not typically indicated unless there are accompanying signs of occult fracture.
Recommending direct force trauma to the joint soon after the ankle sprain is suboptimal.
58. A 23-year-old male presents with shoulder pain after falling. A shoulder radiograph is shown
in Figure below. Which of the following is the most likely additional injury?
pg. 762
Answer A. The radiograph demonstrates an anterior shoulder dislocation, with the humeral
head displaced anteriorly relative to the glenoid. Common complications include (in order of
decreasing frequency) axillary nerve injury (causing deltoid anesthesia), humeral head fracture
(Hillman–Sachs deformity), and glenoid rim disruption (Bankart lesion). Vascular compromise in
shoulder dislocations is rare. Acromioclavicular separation and clavicular fractures are rarely
associated with shoulder dislocations because of different mechanisms. The mechanism of an
anterior dislocation is trauma to the abducted and externally rotated upper extremity. Clavicle
fractures and acromioclavicular separations tend to occur with trauma to the adducted arm.
59. A 34-year-old female presents with shoulder pain following a fall. Physical examination is
normal except for tenderness in the middle of the clavicle. Radiographs demonstrate a
nondisplaced clavicle fracture. Which of the following is the most appropriate course of action?
A. Operative repair
B. Figure-of-eight brace
D. Shoulder sling
E. Shoulder arthrocentesis
Answer D. Clavicular fracture is extremely common with shoulder trauma. The vast majority of
clavicular fractures are treated conservatively with shoulder sling and orthopedic follow-up.
Operative repair is almost never indicated in nondisplaced, closed fractures, because
conservative management results in excellent functional outcomes. There is no proven
advantage of the figure-of eight brace over simple sling, and it may actually cause skin necrosis if
applied too tightly. Emergent MRI is almost never indicated in acute injuries, and clavicle
fracture with concomitant rotator cuff injury is rare. Shoulder arthrocentesis is indicated only in
cases of suspected septic arthritis and has little role in acute trauma.
pg. 763
pg. 764
Psychiatry, abuse and suicide
emergency
➢ Chapter includes:
Psychiatry emergency condition
Non- accidental trauma
Suicide
Medical ethics
pg. 764
1. Which of the following is the most common cause of death among African-American
adolescents?
A. Infection
B. Cancer
D. Gunshot wound
E. Drug overdose
Answer D. The United States has the highest rate of homicide due to firearms in the
industrialized world. The ready availability of firearms in the United States combined with
complex socioeconomic inequalities put urban African-American youths at the highest risk for
firearm-related death. The leading cause of death in this subset of the population is due to
homicide from handguns.
2. Which of the following patients has the highest statistical chance of completed suicide?
Answer B. Elderly white men have the highest rate of completed suicide, representing more
than three-fourths of all suicide deaths, and women have the highest rate of suicide attempts.
White patients are more likely to commit suicide than blacks or Hispanics, and nonpregnant
women of childbearing age are more likely than pregnant women to do so. Divorced patients
have higher rates than unmarried patients, who, in turn, have higher rates than married
patients. Most successful suicide attempts involve firearms and most unsuccessful attempts
involve drug ingestions. The presence of a firearm in the house is an independent risk factor for
completed suicide, and the patient should be directly asked about this on history. Substance
abuse, especially alcohol and cocaine, is extremely common in patients who complete suicide.
Patients who present to the ED with attempted suicide must be evaluated for medical illness
that may masquerade as mood disorder or thought disorder leading to the suicide attempt.
Roughly 20% of patients with major depression and 10% of patients with schizophrenia commit
suicide.
pg. 765
3. A 26-year-old male with no significant past medical history is brought to the emergency room
by his father and wife. The patient’s boss called his wife, asking why the patient had not shown
up for work over the past 2 days. The patient’s wife was unaware the patient had not been
going to work, since he had been leaving the house on a daily basis, though he had been “saying
strange things” for several months. His vital signs are normal. A urine drug screen is negative. He
is not known to abuse alcohol and has an undetectable blood alcohol level. Upon evaluating the
patient, it is clear that he is experiencing auditory hallucinations as he is having a conversation
with someone who is not physically present. When you introduce yourself, he says that “the
CEO of your company is getting together with the CEO of my company to cut down all the
trees.” Which of the following is most likely true of his underlying condition?
Answer C. The patient most likely has schizophrenia, a chronic, debilitating psychiatric condition,
characterized by a combination of psychosis (delusions, hallucinations, and disorganized
thoughts), negative symptoms (e.g., flat affect, social withdrawal, and decreased verbal
expression), cognitive impairment (affecting attention, language, memory, and executive
function), and a mood disorder. The presentation can be highly variable, and none of these
findings is pathognomonic for the diagnosis. However, this patient’s chronic course (“saying
strange things” for months) leading to an acute “psychotic break” involving auditory
hallucinations (auditory hallucinations, usually in the form of hearing voices, are more common
than other hallucinations) is typical of schizophrenia. Psychosis related to drug abuse and
withdrawal is common but develops acutely and in the setting of a history of abuse or findings
that support abuse (e.g., a positive drug screen). Two-thirds of patients with schizophrenia have
delusions, which are the most common “positive symptom” (positive symptoms are
synonymous with “psychosis”). Suicide is reported in as many as 10% of patients with the
diagnosis and is most common early in the course of the illness at the beginning of treatment or
shortly after the resolution of an acute psychotic episode. It also occurs more commonly among
patients with mild impairment, which is thought to reflect the higher executive functioning and
better insight among such patients. Improved self-recognition may lead to more severe
depression, while intact executive functioning allows such patients to act on suicidal thoughts.
4. A 34-year-old male with schizophrenia is brought to the ED by his family because he “keeps
ignoring” them. They report that for the last few hours, when they argue with him about taking
his risperidone, he stares off into the space and does not acknowledge them in any way. This
pg. 766
lasts for a few minutes and gradually improves until the next conversation. They want to speak
with the psychiatrist about putting him on a new antipsychotic medication. His vital signs are
normal and physical examination is unremarkable except that when you question him about his
medication, he becomes visibly angry and his eyes look up to the ceiling. After he calms down in
a few minutes, he resumes normal eye contact and conversation. Which of the following is the
most appropriate action at this time?
C. Haloperidol 5 mg IM
D. Lorazepam 2 mg IM
E. Benztropine 1 mg IM
Answer E. The patient appears to have evidence of a specific dystonic reaction known as the
oculogyric crisis, where both the patient’s eyes stare upward and do not come back to the
neutral position. Symptoms can fluctuate based on emotional state. Dystonic reactions are
usually due to an excess of cholinergic activity due to overblockade of dopaminergic receptors
by antipsychotic medications. The normally inhibitory effect of dopamine on the cholinergic
neurons is reduced with the use of antipsychotic agents. Treatment for acute dystonic reactions
is with an anticholinergic agent, either benztropine or diphenhydramine. Discharging the patient
home neither treats the patient nor adequately manages the social situation. Psychiatric
consultation for a new psychotropic medication prescription is not appropriate until the primary
cause of the complaint has been evaluated and addressed. Haloperidol would further worsen
the oculogyric crisis. Lorazepam might sedate the patient but would probably not improve the
dystonia.
B. Most states mandate reporting of domestic violence against competent adult women.
Answer A. Domestic violence against women may occur in as many as 20% of pregnancies. If
abuse is already present in a relationship, it often worsens during pregnancy. Females
(regardless of pregnancy) presenting for ED care are at increased risk for abuse relative to
pg. 767
females presenting for primary care or nonurgent care. Specifically, women with chronic
headaches, chronic abdominal pain, and trauma victims are all at increased risk for abuse.
Noncompliance is also common among abused patients. With respect to abuse in general, while
the majority of victims are women, up to 40% of abused patients may be men. Like abused
females, these patients confront significant barriers against reporting abuse. There are myriad
organizations who issue various and sometimes conflicting advice about the appropriate
manner to screen for abused patients. The Massachusetts Medical Society Committee on
Violence suggests asking the following single question: “At any time has a partner hit, kicked or
otherwise hurt or threatened you?” Interestingly, the incidence of domestic violence is the same
in gay couples as it is among straight couples. However, studies show that gay couples are
infrequently screened for domestic violence.
6. Which of the following is true regarding interpersonal violence and intimate partner abuse
(IPA)?
Answer E. IPA encompasses the following old terms—domestic violence, domestic abuse, and
spousal abuse. It is likely to be vastly under-reported by both men and women. ED patients are
at high risk for being victims of IPA, but large majority of patients will not report IPA unless
specifically asked by a care provider in the absence of their intimate partners. Screening of all
patients is the only way to accurately evaluate IPA among ED patients. Most violence against
women is perpetrated by intimate partners, but most violence against men is perpetrated by
strangers. Almost one-fourth of women and 10% of men of all sexual orientations report an
incident of IPA. Victim substance abuse is a marker for IPA but is not thought to contribute to
IPA—perpetrator substance is known to contribute to IPA.
7. A patient presents with new-onset psychosis. Which of the following suggests a medical,
rather than a psychiatric, cause for the symptoms?
A. Auditory hallucinations
B. Age <35
C. Gradual onset
D. Aphasia
pg. 768
E. Flat affect
Answer D. Patients with new-onset psychosis must be evaluated in the ED for treatable medical
illness before being assigned a psychiatric diagnosis. Signs of a medical cause of the psychosis
include acute onset, older patient, visual hallucinations, disorientation, and impaired
consciousness. Abnormal physical examination findings, such as abnormal vital signs, aphasia,
ataxia, and cranial nerve abnormalities usually indicate a medical cause. Many pharmacologic
agents can also cause psychosis, including corticosteroids, antihistamines, antidepressants, and
sedative hypnotic. True psychiatric disease is suggested by a young adult patient, auditory
hallucinations, gradual progression, flat affect, and intact orientation and consciousness.
8. Which of the following is the most common relationship of the caregiver to the victim in
Munchausen syndrome by proxy?
A. Biologic father
B. Stepfather
C. Biologic mother
D. Stepmother
E. Nonparent guardian
9. A 6-year-old female is brought to the emergency room by her mother, who is concerned that
she is being touched inappropriately by her ex-husband. The patient told her mother “daddy
showed me how babies are made.” Which of the following examination findings is most
concerning for sexual assault?
A. Annular hymen
B. Anal fissure
D. Suprapubic tenderness
pg. 769
E. Hymenal bump
Answer C. Sexual assault victims infrequently have significant genitourinary findings upon
physical examination. Evidence of extragenital trauma, such as ecchymoses, erythema, or
abrasions, is far more common (∼65%) than evidence of vaginal, perineal, or anal trauma
(∼25%). Traumatic anogenital findings are more common among virgins, as well as victims of
anal insertion. Among patients with an abnormal genitourinary examination, anal fissures are
nonspecific and an annular hymen describes one of the many normal hymen variants, in which
the hymen encircles the vaginal opening, forming a complete 360-degree ring. Hymenal bumps
are normal elevations of hymenal tissue caused by an attachment of the hymen to the
underlying intravaginal rugae. However, a deep hymenal cleft, or complete hymenal transection
along the inferior hemisphere of the hymenal ring, for example at 6 o’clock, is abnormal and is
specific for abuse.
10. The drug of choice for sedation in the setting of acute delirium is
A. Haloperidol
B. Diazepam
C. Diphenhydramine
D. Morphine
E. Promethazine
Answer A. Haloperidol is a potent dopamine antagonist, which does not have anticholinergic or
hypotensive effects. Phenothiazines, such as prochlorperazine and chlorpromazine, cause
orthostatic hypotension, lower the seizure threshold, and have strong anticholinergic properties
that can exacerbate delirium. Diphenhydramine, while sedating, also shares these
anticholinergic properties. Opioids may induce dysphoria and can exacerbate brain dysfunction.
Diazepam has a long half-life due to its metabolites and may result in hypotension and
respiratory depression. Promethazine is primarily an antihistamine that also has strong
anticholinergic properties.
11. A 78-year-old male admitted for urinary tract infection and dementia becomes increasingly
agitated while boarding in your ED for 5 hours. Which of the following is the most appropriate
next step in management?
B. Lorazepam 1 mg IV
pg. 770
C. Diphenhydramine 12.5 mg IV
D. Diphenhydramine 25 mg IV
E. Haloperidol 2 mg IV
Answer A. The first-line management for delirium is to reassess the patient to ensure that there
is no untreated medical cause and redirect with verbal reassurance and environmental
improvements. Expediting patient transport up to an inpatient bed by involving hospital
administration is a key component in a comprehensive strategy to manage this patient.
Benzodiazepines and anticholinergics are strongly discouraged as part of the Beers list of
potentially inappropriate medications for geriatric patients. Benzodiazepine clearance is
significantly slowed in older adults and anticholinergic drugs cause significant side effects and
can worsen delirium. Haloperidol used to be suggested as the first-line treatment for acute
delirium, but increasing reports of side effects have rendered this a second-line (or even
third-line) option.
E. Arachnophobia
Answer C. Family history of suicide is an independent risk factor for completed suicide. Other
risk factors include depression, schizophrenia, substance abuse, prior attempts, presence of a
firearm in the home, and feelings of hopelessness and long-term loneliness. Young women have
the lowest rate of completed suicide but have the highest rate of suicide attempts. Patients with
antisocial personality disorder do not have higher rates of suicide completion unless there is
comorbid substance abuse. Generalized anxiety disorders and simple phobias do not confer a
higher risk of completed suicide in the absence of depression or substance abuse.
13. A 19-year-old female, accompanied by her college roommate, presents to the emergency
room for evaluation of “an eating problem.” The patient’s roommate states that the already thin
patient began losing excessive weight after she decided to become a vegetarian because she
thought she “was too fat” 2 months ago. The patient is awake and alert, but is a reluctant
historian, though she willingly came to the ED for evaluation. She is a self-described
“perfectionist.” Her vitals are T 97.6°F, P 42, RR 16, BP 90/58. She is 5 ft 6 in tall, and weighs 43
pg. 771
kg (95 lb), which puts her in the third percentile for weight based on her age and gender. An EKG
reveals sinus bradycardia. Which of the following is the next best step in her evaluation and
management?
Answer E. This patient is presenting with profound weight loss due to anorexia nervosa. There
are both psychiatric and medical indications for admission of patients with anorexia nervosa.
The psychiatric indication for admission is primarily limited to those patients who have a high
degree of suicidality, or active suicidality, or comorbid depression that may interfere with their
ability to comply with treatment. Medical indications for admission are described by both the
Society for Adolescent Medicine and the APA, and include body weight <75% of average body
weight for age, height, and gender, bradycardia <50 or any significant arrhythmia, hypotension
(<80/50), hypothermia (<96°F), severe orthostatic changes, severe electrolyte disturbances (e.g.,
severe hypophosphatemia, hypokalemia, or hypomagnesemia), or other medical complications
of anorexia, including pancreatitis, heart failure, syncope, or seizures. Committing a patient to
inpatient therapy may be needed if they refuse admission, but this patient arrived willingly and
has given no indication that she intends to refuse therapy.
14. Among the elderly, which of the following is the most common cause of delirium?
A. Stroke
B. Electrolyte abnormalities
C. Infection
E. Trauma
Answer D. Twenty-two percent to 39% of cases are caused by medications. Delirium that begins
while patients are taking a drug usually ceases once the drug is discontinued. All of the other
findings listed can cause delirium in elderly patients.
pg. 772
15. Which of the following is a known effect of haloperidol?
E. Agranulocytosis
Answer D. There are no fractures that are pathognomonic for abuse. However, rib fractures,
particularly posterior rib fractures, are more than twice as likely to be due to abuse then other
fractures. Femur and skull fractures are also common in the setting of abuse. Concerning skull
fracture patterns include multiple fractures, bilateral fractures, and fractures that cross suture
lines. Many physically abused patients will have suffered more than one fracture. It is important
to remember that while rib fractures are commonly associated with abuse, chest x-rays are a
poor screening modality for such fractures. Dedicated rib series should be obtained for patients
in whom there is suspicion for a rib fracture due to abuse.
A. Delusions
pg. 773
B. Hallucinations
C. Flat affect
D. Disorganized speech
E. Disorganized behavior
18. A 55-year-old female presents with uncontrollable twisting movements of her face and
tongue. She has been on an antipsychotic for 25 years for treatment of schizophrenia. Which of
the following is true regarding this condition?
19. A 25-year-old male with schizophrenia presents with acute agitation. According to a family
member, he was seen by a psychiatrist during the previous week, diagnosed with schizophrenia,
and started on a new medication. He is extremely agitated, tachycardic, diaphoretic, febrile, and
exhibits muscle rigidity. Which of the following is the most appropriate next step in
management?
pg. 774
A. Acetaminophen
B. Lorazepam
C. Amantadine
D. Bromocriptine
E. Haloperidol
Answer B. The patient’s recent diagnosis of schizophrenia and initiation of likely antipsychotic
medication strongly suggest neuroleptic malignant syndrome in the setting of his presentation.
Hyperthermia, muscle rigidity, altered mental status, and elevated creatine phosphokinase
levels are characteristic. Treatment involves aggressive sedation with benzodiazepines, cooling,
and paralysis with neuromuscular blockade in severe cases. Acetaminophen is unlikely to be of
benefit in patients with hyperthermia because hyperthermia does not exhibit an elevation of the
hypothalamic set point as is seen in fever. Amantadine and bromocriptine are dopamine
agonists which have not been proven to be beneficial in patients with neuroleptic malignant
syndrome. Haloperidol is an antipsychotic which might further exacerbate the pathophysiologic
process in this case. Dantrolene, which blocks calcium release in muscle cells may afford some
benefit but is unlikely to be more effective than benzodiazepines and paralytics.
20. Which of the following personality disorders is seen in most patients with somatization
disorder?
A. Histrionic
B. Antisocial
C. Schizoid
D. Schizotypal
E. Narcissistic
pg. 775
the EP to review old records of patients suspected of somatization disorder, as these patients
often undergo repeated unnecessary testing in the ED. Discussions with the patient’s primary
care physician is mandatory as this will help to tailor emergent workup. Pharmacotherapy is of
little benefit in the acute setting.
21. A 5-year-old male is brought by his mom for a “rash” on his back. His back is shown with
close-up of the left flank lesion. The remainder of his physical examination is normal (Fig. 7-4).
Which of the following is the most likely cause?
A. Henoch–Schönlein purpura
C. Lyme disease
D. Child abuse
E. Roseola
Answer D. The patient has scattered human bite marks, which are circular in nature and have a
perforated, erythematous border. Human bites are a common sign of child abuse. Management
involves tetanus prophylaxis, treatment of associated cellulitis, and notification of appropriate
social services. The rash of Henoch–Schönlein purpura presents as palpable purpura on the
pg. 776
buttocks and lower extremities. ITP presents as nonpalpable petechiae and purpura with
laboratory evidence of low platelets. The rash of Lyme disease, erythema migrans, is an
erythematous rash with central clearing, classically on the trunk in a patient with travel to an
endemic area. Roseola presents as sudden onset of fever in young children, followed by a
generalized macular rash after the patient has completely defervesced.
22. A 21-year-old male is brought to the ED by his ex-girlfriend because she is concerned about
his suicidal ideation. While the patient has a history of depression, he states that he “spiraled
downhill” after his recent breakup with her. In the course of his evaluation, however, he
answers questions appropriately and directly, and he verbally “contracts for safety.” Such
contracts:
Answer B. Despite their widespread use, verbal and written “contracts for safety” or “no-harm
contracts” have not been shown to reduce suicide rates, are discouraged by the APA, and do not
increase liability protection. Instead, their use can actually increase liability risk, and their use
risks alienating patients, particularly if they are used as an administrative procedure in place of a
thorough evaluation to assess actual risk. Such contracts were initially described in a paper by
Drye et al. in 1973, in which the contract was used as a risk assessment tool in outpatients with
whom the physician had already established rapport. Patients who responded to the contract’s
firm statements in a negative manner were thought to be at higher risk of future suicide. While
the initial researchers did not advocate widespread use of such contracts in crisis settings, their
use rapidly expanded nonetheless. Physicians can effectively use such contracts as a tool to
express concern about a suicidal patient, but such contracts should never be used as a means to
justify discharge or mitigate liability risk.
23. Which of the following symptoms or findings best distinguishes patients with serotonin
syndrome from patients with other drug- or toxin-related effects?
A. Tachycardia
B. Clonus
C. Shivering
pg. 777
D. Hyperthermia
E. Muscular rigidity
24. The parents of a 4-year-old female are referred to the ED by their pediatrician for evaluation
of abdominal pain. After arrival, the patient’s father refuses to sign the hospital’s general
consent form which permits the hospital and its providers to treat the patient and bill for their
services because he “doesn’t want to get gouged.” Which of the following is true?
A. The patient’s father should be asked to sign an AMA (against medical advice) form indicating
that he has refused care for his daughter
E. Evaluation and treatment of the child should proceed regardless of the father’s refusal to sign
the form.
Answer B. Though the parents may not be aware of it, by bringing the child to the ED and
requesting an examination, they have triggered EMTALA, the Emergency Medical Treatment and
Active Labor Act, that requires hospitals to provide an MSE to any person who comes to the ED
and requests examination or treatment; and if an emergency medical condition is determined to
exist, provide necessary stabilizing treatment or an appropriate transfer. It is not appropriate to
delay the MSE despite the parents’ consent. It is appropriate to explain to the patient’s parents
that by bringing her to the ED, they have triggered a federal law that requires you to screen the
child for an emergency medical condition. It may be helpful to add that if no emergency medical
condition is found to exist, no treatment or services will be provided to the child without their
consent. In cases in which the child has a clear emergency condition (e.g., bleeding profusely, or
in respiratory distress), it is appropriate to provide any necessary treatment to prevent the
child’s death or deterioration of her condition regardless of the parents’ wishes.
pg. 778
25. Which of the following is true regarding elder abuse?
Answer A. Elder abuse is an under-reported problem that is slowly increasing in incidence with
better layperson and health provider education and recognition. Although much data are still
lacking, it is clear that elders who undergo abuse are at higher risk for morbidity and mortality
than non-abused elders. Women, the extreme elderly, and patients with severe physical and
cognitive dysfunction are all at higher risk. The most common type of elder abuse is neglect;
sexual abuse occurs but is uncommon. Physicians comprise only a small minority of reporters of
elder abuse for a variety of reasons, including lack of awareness or education about the
problem. Most perpetrators are family members or direct caregivers. Every ED should have
specific protocols for screening for and reporting elder abuse, along with an action plan for
confirmed abuse.
26. Which of the following is the most common class of psychiatric disorder seen by emergency
and primary care physicians?
A. Thought disorder
C. Anxiety disorder
D. Somatoform disorder
E. Factitious disorder
Answer C. The single most common group of psychiatric disorders seen by emergency and
primary care physicians is anxiety disorders. This includes simple phobia, generalized anxiety
disorder, panic disorder, and obsessive–compulsive disorder. Treatment usually entails
long-term SSRIs combined with benzodiazepines, as needed for acute anxiety attacks. Mood
disorders are also extremely commonly seen and treated by primary care physicians. Thought
disorders such as schizophrenia are usually managed primarily by psychiatrists, as are
somatoform disorders and factitious disorders. Most patients who use recreational drugs do not
inform their primary care physicians about their drug use and those who do tend to be referred
pg. 779
to addiction psychiatry specialists and outpatient rehabilitation centers. Appropriate referral
from the ED for anxiety disorders and mood disorders can involve primary care follow-up
exclusively. However, patients with thought, somatoform, or factitious disorders should receive
dedicated psychiatric follow-up if they do not already have a preexisting therapeutic relation
with a psychiatrist.
27. Which of the following is the most common error leading to malpractice claims?
A. Failure to diagnose
C. Delay in treatment
D. Delay in diagnosis
Answer A. Failure to make the correct diagnosis accounts for two-third of malpractice claims.
Delay in diagnosis and improper medical management account for most of the remainder.
Procedural errors and delay in treatment are less commonly cited reasons for malpractice
claims.
28. A 9-month-old infant is brought to the emergency department (ED) with a bruise on his thigh
suffered from falling out of his high chair. Radiographs reveal a midshaft femur fracture. Which
of the following is the most likely contributing factor?
A. Child abuse
B. Osteogenesis imperfecta
C. Bone tumor
D. Bone cyst
E. Hypocalcemia
Answer A. Fractures are found in most abused children. Although no fracture is 100% specific for
child abuse, several types are extremely high risk—any fractures in infants (especially of the
femur) and spiral, multiple, rib, metaphyseal, humerus, and scapula fractures. Complete skeletal
surveys are indicated for patients younger than 5 years who are suspected of being abused.
Osteogenesis imperfecta is a rare disease, which causes problems in bone synthesis due to
collagen defects. Frequent fractures are common and physical examination may demonstrate
blue sclerae, deafness, and ligamentous laxity. Subclinical cases may be more common than
pg. 780
previously recognized. Bone tumors and cysts and hypocalcemia may predispose to fracture but
are not as likely to cause femoral fractures in the infant as child abuse.
29. A 22-year-old gang member is brought to an urban ED with a gunshot wound to his arm.
Primary survey of the patient is intact, but a handgun is found in the patient’s clothes. Which of
the following is the most appropriate action by the physician at this time?
B. Fire the gun into the floor to discharge all the ammunition.
C. Place the gun away from care providers and notify local law enforcement.
Answer C. During primary and secondary survey of all victims of violent crime, care must be
taken to identify objects that are potentially lethal to caregivers. Handguns are particularly
dangerous as they may discharge with only minimal movement during exposure of the patient.
Any firearm should preferably not be handled at all—if it is in a potentially dangerous position, it
should be placed carefully in a safe location and secured by law enforcement. Care providers
should never check to see if the firearm is loaded, fire it, or carry it on their person in case
spontaneous discharge occurs. Using the firearm for any reason is obviously contraindicated.
30. Which of the following is the most common etiology of death from child abuse?
A. Retroperitoneal hemorrhage
B. Hemothorax
C. Intracranial hemorrhage
D. Burns
E. Drowning
Answer C. More than 2,000 children per year die of child abuse. The most common mechanism
is head injury, followed by intra-abdominal bleeding. Evaluation and treatment of injuries from
child abuse are often delayed due to the abuser’s status as primary or secondary caretaker.
Intracranial injuries commonly include subdural hematoma, SAH, and cerebral contusions. A
vigorous shaking mechanism alone in an infant is enough to cause a fatal brain hemorrhage. The
other answer choices listed are less common causes of death from child abuse. The most
pg. 781
common overall manifestations of child abuse are soft-tissue injuries, followed by long-bone
fractures.
31. Which of the following is the most common mode of completed suicide?
A. Drug ingestion
B. Firearms
C. Carbon monoxide
D. Hanging
E. Wrist cutting
Answer B. The majority of completed suicides among both men and women involve firearms.
The presence of a firearm in the house is an independent risk factor for completed suicide and
the patient should be directly asked about this on history. Drug ingestion, usually with
antidepressants, is the most common method of suicide attempts, and the second most
common method of completed suicide by women. Carbon monoxide poisoning is employed less
often. Hanging is the second most common method of completed suicide by men. Wrist cutting
almost never results in completed suicide.
Answer C. Major depression is a type of mood disorder, which requires either depressed mood
or anhedonia (loss of interest or pleasure in pleasurable activities) for the diagnosis. According
to the Diagnostic and Statistical Manual of Mental Disorders, Fourth Edition (DSM-IV) criteria,
symptoms need to be present for 2 or more weeks for formal diagnosis. Symptoms include
disturbances in appetite, sleep, concentration, activity, and energy and thoughts of guilt or
suicide. The symptoms should not be caused by a thought disorder, medical condition, or
substance abuse. Delusions and visual hallucinations are more characteristic of thought
disorders than major depression. Major depression is almost 10 times as common as bipolar
pg. 782
disorder. It is extremely difficult to distinguish dementia from depression in the elderly on
clinical grounds alone. Sometimes a trial of antidepressant therapy may be necessary to make
the diagnosis—major depression is extremely responsive to pharmacotherapy whereas
dementia is not.
33. A 34-year-old male with a history of schizophrenia is brought to the ED by police with acute
agitation. He was reported to be threatening passersby on the street. He now begins to threaten
staff, stating that he will kill anyone who comes near him, and starts to swing punches at people
standing near him. The patient is physically restrained by security staff and secured to a cart in
four-point restraints. He is still yelling at the top of his lungs and struggling against the
restraints. Which of the following is the most appropriate next step in management?
A. Administer medications for agitation and check a CK level to determine if he has neuroleptic
malignant syndrome (NMS).
Answer D. Any patient with acute agitation and the potential for violence either to self or others
must be physically and chemically restrained. Rapid tranquilization is ideally accomplished with
a combination of haloperidol and benzodiazepine. Benztropine could also be given to reduce the
incidence of acute dystonic reactions that may accompany administration of the antipsychotic
medication. Observing the patient in an agitated state struggling against restraints is
contraindicated, as the patient may cause harm to himself while fighting. The diagnosis of NMS
is a clinical one based on the presence of fever, altered mental status, muscular rigidity, and
frequent autonomic instability. While affected patients often have significant creatine kinase
elevations, the test is no diagnostic for NMS. Trying to perform a CT scan with the patient in an
acutely agitated, violent state will be impossible and just put the patient and staff at risk for
injury. Rapid sequence intubation is not indicated unless rapid tranquilization is unsuccessful
and serious morbidity is suspected in the patient, necessitating emergent workup for acute
medical or traumatic cause for the psychosis.
pg. 783